Activity 1 - Managerial Accouting

profilevoyage
BUS535-FinancialandManagerialAccounting8thEdition-JohnWild.pdf

Information for Decisions

Financial & Managerial Accounting

8th edition

JOHN J. WILD KEN W. SHAW

Financial & Managerial Accounting

8thedition

John J. Wild University of Wisconsin at Madison

Ken W. Shaw University of Missouri at Columbia

INFORMATION FOR DECISIONS

To my students and family, especially Kimberly, Jonathan, Stephanie, and Trevor. To my wife Linda and children Erin, Emily, and Jacob.

FINANCIAL AND MANAGERIAL ACCOUNTING: INFORMATION FOR DECISIONS, EIGHTH EDITION

Published by McGraw-Hill Education, 2 Penn Plaza, New York, NY 10121. Copyright ©2019 by McGraw-Hill Education. All rights reserved. Printed in the United States of America. Previous editions ©2018, 2016, and 2013. No part of this publication may be reproduced or distributed in any form or by any means, or stored in a database or retrieval system, without the prior written consent of McGraw-Hill Education, including, but not limited to, in any network or other electronic storage or transmission, or broadcast for distance learning.

Some ancillaries, including electronic and print components, may not be available to customers outside the United States.

This book is printed on acid-free paper.

1 2 3 4 5 6 7 8 9 LWI 21 20 19 18

ISBN 978-1-260-24785-5 (bound edition) MHID 1-260-24785-6 (bound edition) ISBN 978-1-260-41719-7 (loose-leaf edition) MHID 1-260-41719-0 (loose-leaf edition)

Executive Portfolio Manager: Steve Schuetz Product Developers: Michael McCormick, Christina Sanders Marketing Manager: Michelle Williams Content Project Managers: Lori Koetters, Brian Nacik Buyer: Sandy Ludovissy Design: Debra Kubiak Content Licensing Specialist: Melissa Homer Cover Image: Runner: ©Maridav/Shutterstock; Statistics icons: ©A-spring/Shutterstock; Background image: ©Vector work/Shutterstock Compositor: Aptara®, Inc.

All credits appearing on page or at the end of the book are considered to be an extension of the copyright page.

Library of Congress Cataloging-in-Publication Data Names: Wild, John J., author. | Shaw, Ken W., author. Title: Financial and managerial accounting : information for decisions / John J. Wild, University of Wisconsin at Madison, Ken W. Shaw, University of Missouri at Columbia. Description: 8th Edition. | Dubuque, IA : McGraw-Hill Education, [2018] | Revised edition of Financial and managerial accounting, [2018] | Includes bibliographical references and index. Identifiers: LCCN 2018035310| ISBN 9781260247855 (alk. paper) | ISBN 1260247856 (alk. paper) Subjects: LCSH: Accounting. | Managerial accounting. Classification: LCC HF5636 .W674b 2018 | DDC 658.15/11—dc23 LC record available at https://lccn.loc.gov/2018035310

The Internet addresses listed in the text were accurate at the time of publication. The inclusion of a website does not indicate an endorsement by the authors or McGraw-Hill Education, and McGraw-Hill Education does not guarantee the accuracy of the information presented at these sites.

mheducation.com/highered

iii

About the Authors JOHN J. WILD is a distinguished pro- fessor of accounting at the University of Wisconsin at Madison. He previously held appointments at Michigan State University and the University of Manchester in England. He received his BBA, MS, and PhD from the University of Wisconsin.

John teaches accounting courses at both the undergraduate and graduate levels. He has received numerous teaching honors, in- cluding the Mabel W. Chipman Excellence-in-

Teaching Award and the departmental Excellence-in-Teaching Award, and he is a two-time recipient of the Teaching Excellence Award from business graduates at the University of Wisconsin. He also received the Beta Alpha Psi and Roland F. Salmonson Excellence-in-Teaching Award from Michigan State University. John has received several research honors, is a past KPMG Peat Marwick National Fellow, and is a recipient of fellowships from the American Accounting Association and the Ernst and Young Foundation.

John is an active member of the American Accounting Association and its sections. He has served on several committees of these organizations, including the Outstanding Accounting Educator Award, Wildman Award, National Program Advisory, Publications, and Research Committees. John is author of Financial Accounting, Managerial Accounting, Fundamental Accounting Principles, and College Accounting, all published by McGraw-Hill Education.

John’s research articles on accounting and analysis appear in The Accounting Review; Journal of Accounting Research; Journal of Accounting and Economics; Contemporary Accounting Research; Journal of Accounting, Auditing and Finance; Journal of Accounting and Public Policy; Accounting Horizons; and other journals. He is past associate editor of Contemporary Accounting Research and has served on several editorial boards including The Accounting Review and the Journal of Accounting and Public Policy.

In his leisure time, John enjoys hiking, sports, boating, travel, people, and spending time with family and friends.

Courtesy of John J. Wild

KEN W. SHAW is an associate profes- sor of accounting and the KPMG/Joseph A. Silvoso Distinguished Professor of Accounting at the University of Missouri. He previously was on the faculty at the University of Maryland at College Park. He has also taught in international programs at the University of Bergamo (Italy) and the University of Alicante (Spain). He received an accounting degree from Bradley University and an MBA and PhD from the

University of Wisconsin. He is a Certified Public Accountant with work experience in public accounting.

Ken teaches accounting at the undergraduate and graduate levels. He has received numerous School of Accountancy, College of Business, and university-level teaching awards. He was voted the “Most Influential Professor” by four School of Accountancy graduating classes and is a two-time recipient of the O’Brien

Excellence in Teaching Award. He is the advisor to his school’s chapter of the Association of Certified Fraud Examiners.

Ken is an active member of the American Accounting Association and its sections. He has served on many committees of these organizations and presented his research papers at na- tional and regional meetings. Ken’s research appears in the Journal of Accounting Research; The Accounting Review; Contemporary Accounting Research; Journal of Financial and Quantitative Analysis; Journal of the American Taxation Association; Strategic Management Journal; Journal of Accounting, Auditing, and Finance; Journal of Financial Research; and other journals. He has served on the editorial boards of Issues in Accounting Education; Journal of Business Research; and Research in Accounting Regulation. Ken is co-author of Fundamental Accounting Principles, Managerial Accounting, and College Accounting, all published by McGraw-Hill Education.

In his leisure time, Ken enjoys tennis, cycling, music, and coaching his children’s sports teams.

Courtesy of Ken W. Shaw

Author Letter Using Learning Science and Data Analytics We use data to make decisions and maximize performance. Like the runner on the cover who uses data to track her progress, we used stu- dent performance data to identify content areas that can be made more direct, concise, and systematic.

Learning science reveals that students do not read large chunks of text, so we streamlined this edition to present it in a more focused, succinct, blocked format to improve student learning and retention. Our new edition delivers the same content in 112 fewer pages. Visual aids and numer- ous videos offer additional learning aids. New summary Cheat Sheets conclude each chapter to visually reinforce key concepts and procedures.

Our new edition has over 1,500 videos to engage students and improve outcomes: •   Concept Overview Videos—cover each chapter’s learning objectives with multimedia presentations that include Knowledge Checks to

engage students and assess comprehension. •  Need-to-Know Demos—walk-through demonstrations of key procedures and analysis to ensure success with assignments and tests. •  Guided Examples (Hints)—step-by-step walk-through of assignments that mimic Quick Studies, Exercises, and General Ledger.

iv

Difference Makers in Teaching . . . Learning Science Learning analytics show that students learn better when material is broken into “blocks” of content. Each chapter opens with a visual preview. Learning objective numbers highlight the location of re- lated content. Each “block” of content concludes with a Need-to-Know (NTK) to aid and reinforce student learning. Visual aids and concise, bullet-point dis- cussions further help students learn.

Learning Objectives

CONCEPTUAL C1 Explain the steps in processing

transactions and the role of source documents.

C2 Describe an account and its use in recording transactions.

C3 Describe a ledger and a chart of accounts.

PROCEDURAL P1 Record transactions in a journal and post

entries to a ledger.

P2 Prepare and explain the use of a trial balance.

P3 Prepare financial statements from business transactions.

C4 Define debits and credits and explain double-entry accounting.

ANALYTICAL A1 Analyze the impact of transactions on

accounts and financial statements.

A2 Compute the debt ratio and describe its use in analyzing financial condition.

Chapter Preview

2 Recording Transactions

NTK 2-4

TRIAL BALANCE

P2 Trial balance preparation and use

Error identification

NTK 2-5

FINANCIAL STATEMENTS

P3 Financial statement preparation

A2 Debt ratio

NTK 2-3

RECORDING TRANSACTIONS

P1 Journalizing and posting

A1 Processing transactions— Examples

NTK 2-1

SYSTEM OF ACCOUNTS

Using financial statements

C1 Source documents

C2 Types of accounts C3 General ledger

NTK 2-2

DEBITS AND CREDITS

T-account

C4 Debits and credits

Normal balance

wiL16960_ch02_044-083.indd 44 5/3/18 2:14 PM

New Revenue Recognition •   Wild uses  the popular gross method 

for merchandising transactions (net method is covered in an appendix). The gross method is widely used in practice and best for student success.

•   Adjusting entries for new revenue rec- ognition rules are included in an ap- pendix.  Assignments  are  clearly  marked and separated. Wild is GAAP  compliant.

154 Chapter 4 Accounting for Merchandising Operations

Z-Mart’s Merchandise Inventory account at the end of the year has a balance of $21,250, but a physical count shows only $21,000 of inventory exists. The adjusting entry to record this $250 shrinkage is

Dec . 31 Cost of Goods Sold . . . . . . . . . . . . . . . . . . . . . . . . . . . . . . . . . . 250

Merchandise Inventory . . . . . . . . . . . . . . . . . . . . . . . . . . . 250

Adjust for $250 shrinkage.

Assets = Liabilities + Equity −250 −250

Sales Discounts, Returns, and Allowances—Adjusting Entries Revenue recognition rules require sales to be reported at the amount expected to be received. This means that period-end adjusting entries are commonly made for Expected sales discounts. Expected returns and allowances (revenue side). Expected returns and allowances (cost side).

These three adjustments produce three new accounts: Allowance for Sales Discounts, Sales Refund Payable, and Inventory Returns Estimated. Appendix 4B covers these accounts and the adjusting entries.

Preparing Financial Statements The financial statements of a merchandiser are similar to those for a service company described in prior chapters. The income statement mainly differs by the addition of cost of goods sold and gross profit. Net sales is affected by discounts, returns and allowances, and some additional expenses such as delivery expense and loss from defective merchandise. The balance sheet dif- fers by the addition of merchandise inventory as part of current assets. (Appendix 4B explains inventory returns estimated as part of current assets and sales refund payable as part of current liabilities.) The statement of retained earnings is unchanged.

Closing Entries for Merchandisers Closing entries are similar for service companies and merchandising companies. The difference is that we close some new temporary accounts that come from merchandising activities. Z-Mart has temporary accounts unique to merchandisers: Sales (of goods), Sales Discounts, Sales Returns and Allowances, and Cost of Goods Sold. The third and fourth closing entries are identical for a mer- chandiser and a service company. The differences are in red in the closing entries of Exhibit 4.11.

EXHIBIT 4.11 Closing Entries for a Merchandiser

Step 1: Close Credit Balances in Temporary Accounts to Income Summary.

Step 2: Close Debit Balances in Temporary Accounts to Income Summary.

Dec . 31 Income Summary . . . . . . . . . . . . . . . . . . . . . . . . . . . . . . . . . . . . 308,100 Sales Discounts . . . . . . . . . . . . . . . . . . . . . . . . . . . . . . . . 4,300 Sales Returns and Allowances . . . . . . . . . . . . . . . . . . . 2,000 Cost of Goods Sold . . . . . . . . . . . . . . . . . . . . . . . . . . . . . 230,400 Depreciation Expense . . . . . . . . . . . . . . . . . . . . . . . . . . . . 3,700 Salaries Expense . . . . . . . . . . . . . . . . . . . . . . . . . . . . . . . 43,800 Insurance Expense . . . . . . . . . . . . . . . . . . . . . . . . . . . . . . 600 Rent Expense . . . . . . . . . . . . . . . . . . . . . . . . . . . . . . . . . . 9,000 Supplies Expense . . . . . . . . . . . . . . . . . . . . . . . . . . . . . . . 3,000 Advertising Expense . . . . . . . . . . . . . . . . . . . . . . . . . . . . 11,300 Close debit balances in temporary accounts.

Step 3: Close Income Summary.

Dec . 31 Income Summary . . . . . . 12,900 Retained Earnings . . 12,900

Step 4: Close Dividends.

Dec . 31 Retained Earnings . . . . . . . . . 4,000 Dividends . . . . . . . . . . . . 4,000

Dec . 31 Sales . . . . . . . . . . . . . . . . . . . . . . . . . . . . . . . . . . . . . . . . . . . . . . 321,000 Income Summary . . . . . . . . . . . . . . . . . . . . . . . . . . . . . . . 321,000 Close credit balances in temporary accounts.

wiL47856_ch04_142-189.indd 154 9/20/18 9:12 AM

Up-to-Date This book reflects changes in accounting for revenue recognition, investments, leases, and extraordinary items. It is important that students learn GAAP accounting.

Less Is More Wild has markedly fewer pages than competing books covering the same material. •  The text is to the point and uses visuals to aid student learning. •  Bullet-point discussions and active writing aid learning. •  The 8th edition has 112 fewer pages than the 7th edition—a 10% reduction!

Visual Learning •   Learning  analytics  tell  us  to-

day’s students do not read large blocks of text. Wild has adapted  to student needs by having in- formative visual aids through- out. Many visuals and exhibits are new to this edition.

$180 3

= $60 each

2. Last-in, first-out (LIFO) Costs flow in the reverse

order incurred.

3. Weighted average Costs flow at an average

of costs available.

1. First-in, first-out (FIFO) Costs flow in the order

incurred.

× 2× 1

Income Statement Net sales.................... $100

Cost of goods sold.. 45

Gross profit................ $ 55

Balance Sheet Inventory.................... $135

Income Statement Net sales.................... $100

Cost of goods sold.. 70

Gross profit................ $ 30

Balance Sheet Inventory.................... $1 10

Income Statement Net sales.................... $100

Cost of goods sold.. 60

Gross profit................ $ 40

Balance Sheet Inventory.................... $120

$65 May 3

$45 May 1 G

o o d s

s o l d

G o o d s

s o l d

G o o d s

s o l d

G o o d s

l e f t

G o o d s

l e f t

G o o d s

l e f t

$70 May 6

$65 May 3

$45 May 1

$70 May 6

$70 May 6

$65 May 3

$45 May 1

v

Videos •   A growing number of students now learn 

accounting online. Wild offers over 1,500 videos designed to increase student en- gagement and improve outcomes.

•   Hundreds  of  hint  videos  or  Guided  Examples provide a narrated, animated, step-by-step walk-through of select exer- cises similar to those assigned. These short presentations, which can be turned on or off by instructors, provide reinforcement when students need it most. (Exercise PowerPoints are available for instructors.)

•   Concept  Overview  Videos  cover  each  chapter’s learning objectives with nar- rated, animated presentations that fre- quently  assess  comprehension.  Wild’s  concept overview presentations cover learning objectives broken down into over 700 videos. 

Chapter 7 Accounting for Receivables 277

Recovering a Bad Debt If an account that was written off is later collected, two en- tries are made. The first is to reverse the write-off and reinstate the customer’s account. The second is to record the collection of the reinstated account. If on March 11 Kent pays in full his account previously written off, the entries are

Exhibit 7.6 portrays the allowance method. It shows the creation of the allowance for future write-offs—adding to a cookie jar. It also shows the decrease of the allowance through write- offs—taking cookies from the jar.

A dj

us tin

g en

tr ie

s

Adjusting entries add to allowance for doubtful accounts.

Allowance for doubtful accounts

Write-o�s

Allowance for doubtful accounts

Bad debt write-o�s subtract from allowance for doubtful accounts.

Increase Allowance Decrease Allowance

Bad Debts Expense… # Allow. for Doubtful Accts… #

Allow. for Doubtful Accts… # Accts Receivable—J.Kent… #

EXHIBIT 7.6 Increases and Decreases to the Allowance for Doubtful Accounts

Assets = Liabilities + Equity +520 −520

Assets = Liabilities + Equity +520 −520

Mar . 11 Accounts Receivable—J . Kent . . . . . . . . . . . . . . . . . . . . . . . . . . 520

Allowance for Doubtful Accounts . . . . . . . . . . . . . . . . . . . 520

Reinstate account previously written off.

Mar . 11 Cash . . . . . . . . . . . . . . . . . . . . . . . . . . . . . . . . . . . . . . . . . . . . . . 520

Accounts Receivable—J . Kent . . . . . . . . . . . . . . . . . . . . . 520

Record full payment of account.

Kent paid the entire amount previously written off, but sometimes a customer pays only a por- tion. If we believe this customer will later pay in full, we return the entire amount owed to accounts receivable (in the first entry only). If we expect no further collection, we return only the amount paid.

A retailer uses the allowance method. Record the following transactions.

Dec. 31 The retailer estimates $3,000 of its accounts receivable are uncollectible at its year-end. Feb. 14 The retailer determines that it cannot collect $400 of its accounts receivable from a customer

named ZZZ Company. Apr. 1 ZZZ Company unexpectedly pays its account in full to the retailer, which then records its

recovery of this bad debt.

Solution

P2

Entries under Allowance Method

NEED-TO-KNOW 7-3

Dec . 31 Bad Debts Expense . . . . . . . . . . . . . . . . . . . . . . . . . . . . . . . . . . 3,000

Allowance for Doubtful Accounts . . . . . . . . . . . . . . . . . . . 3,000

Record estimated bad debts.

Feb . 14 Allowance for Doubtful Accounts . . . . . . . . . . . . . . . . . . . . . . . 400

Accounts Receivable—ZZZ Co . . . . . . . . . . . . . . . . . . . . . 400

Write off an account.

Apr . 1 Accounts Receivable—ZZZ Co . . . . . . . . . . . . . . . . . . . . . . . . . . 400

Allowance for Doubtful Accounts . . . . . . . . . . . . . . . . . . . 400

Reinstate an account previously written off.

Apr . 1 Cash . . . . . . . . . . . . . . . . . . . . . . . . . . . . . . . . . . . . . . . . . . . . . . 400

Accounts Receivable—ZZZ Co . . . . . . . . . . . . . . . . . . . . . 400

Record cash received on account. Do More: QS 7-4, QS 7-5, E 7-5

wiL47856_ch07_270-301.indd 277 9/21/18 10:06 AM

Need-to-Know Demos Need-to-Know demonstrations are located at key junctures in each chapter. These demonstrations pose questions about the material just presented—content that students “need to know” to learn  accounting. Accompanying solutions walk students through key procedures and analysis neces- sary to be successful with homework and test materials. Need-to-Know demonstrations are supplemented with narrated, animated, step-by-step walk- through videos led by an instructor and available via Connect.

Comprehensive Need-to-Know Comprehensive Need-to-Knows are problems that draw  on material from the entire chapter. They include a complete solution, allowing students to review the entire problem-solving process and achieve success.

vi

504 Chapter 13 Analysis of Financial Statements

EXHIBIT 13.9 Common-Size Comparative Income Statements

APPLE

as representing one sales dollar, the remaining items show how each revenue dollar is distrib- uted among costs, expenses, and income.

Exhibit 13.9 shows common-size comparative income statements for each dollar of Apple’s net sales. The past two years’ common-size numbers are similar with two exceptions. One is the increase of 0.4 cents in research and development costs, which can be a positive development if these costs lead to future revenues. Another is the increase in cost of sales of 0.6 cent and increase in selling, general and administrative costs of 0.1 cent. We must monitor the growth in these expenses.

Common-Size Graphics Exhibit 13.10 is a graphic of Apple’s current-year common-size income statement. This pie chart shows the contribution of each cost component of net sales for net income.

Exhibit 13.11 takes data from Apple’s Segments footnote. The exhibit shows the level of net sales for each of Apple’s five operating seg- ments. Its Americas segment gener-

ates $96.6 billion net sales, which is roughly 42% of its total sales. Within each bar is that segment’s operating income margin (Operating income/Segment net sales). The Americas seg- ment has a 32% operating income margin. This type of graphic can raise questions about the profitability of each segment and lead to discussion of further expansions into more profitable segments. For example, the Japan segment has an operating margin of 46%. A natural question for management is what potential is there to expand sales into the Japan segment and maintain

Cost of sales 61.5%

Selling, general, administrative,

and other income 6.7%

Research and development

5.1%

Income taxes 6.9%

Net income, excluding non-

operating income and expenses

19.8%

EXHIBIT 13.10 Common-Size Graphic of Income Statement

N et

S al

es (i

n bi

l.)

$0

$20

$40

$100

$80

$60

35%46%30%32%

$15.2$17.7

$54.9

$96.6

38%

$44.8

Americas Europe China Japan Asia Pacific

Segment percentages based on: Operating income/Net sales

EXHIBIT 13.11 Sales and Operating Income Margin Breakdown by Segment

APPLE INC. Common-Size Comparative Income Statements

Common-Size Percents* $ millions Current Yr Prior Yr Current Yr Prior Yr

Net sales . . . . . . . . . . . . . . . . . . . . . . . . . . . . . . . . . . . . . . . . . . . $229,234 $215,639 100.0% 100.0% Cost of sales . . . . . . . . . . . . . . . . . . . . . . . . . . . . . . . . . . . . . . . . 141,048 131,376 61.5 60.9 Gross margin . . . . . . . . . . . . . . . . . . . . . . . . . . . . . . . . . . . . . . . . 88,186 84,263 38.5 39.1 Research and development . . . . . . . . . . . . . . . . . . . . . . . . . . . . 11,581 10,045 5.1 4.7 Selling, general and administrative . . . . . . . . . . . . . . . . . . . . . . 15,261 14,194 6.7 6.6 Total operating expenses . . . . . . . . . . . . . . . . . . . . . . . . . . . . . . 26,842 24,239 11.7 11.2 Operating income . . . . . . . . . . . . . . . . . . . . . . . . . . . . . . . . . . . . 61,344 60,024 26.8 27.8 Other income, net . . . . . . . . . . . . . . . . . . . . . . . . . . . . . . . . . . . . 2,745 1,348 1.2 0.6 Income before provision for income taxes . . . . . . . . . . . . . . . . 64,089 61,372 28.0 28.5 Provision for income taxes . . . . . . . . . . . . . . . . . . . . . . . . . . . . . 15,738 15,685 6.9 7.3 Net income . . . . . . . . . . . . . . . . . . . . . . . . . . . . . . . . . . . . . . . . . $ 48,351 $ 45,687 21.1% 21.2%

*Percents are rounded to tenths and thus may not exactly sum to totals and subtotals.

wiL47856_ch13_496-533.indd 504 9/22/18 3:23 PM

504 Chapter 13 Analysis of Financial Statements

EXHIBIT 13.9 Common-Size Comparative Income Statements

APPLE

as representing one sales dollar, the remaining items show how each revenue dollar is distrib- uted among costs, expenses, and income.

Exhibit 13.9 shows common-size comparative income statements for each dollar of Apple’s net sales. The past two years’ common-size numbers are similar with two exceptions. One is the increase of 0.4 cents in research and development costs, which can be a positive development if these costs lead to future revenues. Another is the increase in cost of sales of 0.6 cent and increase in selling, general and administrative costs of 0.1 cent. We must monitor the growth in these expenses.

Common-Size Graphics Exhibit 13.10 is a graphic of Apple’s current-year common-size income statement. This pie chart shows the contribution of each cost component of net sales for net income.

Exhibit 13.11 takes data from Apple’s Segments footnote. The exhibit shows the level of net sales for each of Apple’s five operating seg- ments. Its Americas segment gener-

ates $96.6 billion net sales, which is roughly 42% of its total sales. Within each bar is that segment’s operating income margin (Operating income/Segment net sales). The Americas seg- ment has a 32% operating income margin. This type of graphic can raise questions about the profitability of each segment and lead to discussion of further expansions into more profitable segments. For example, the Japan segment has an operating margin of 46%. A natural question for management is what potential is there to expand sales into the Japan segment and maintain

Cost of sales 61.5%

Selling, general, administrative,

and other income 6.7%

Research and development

5.1%

Income taxes 6.9%

Net income, excluding non-

operating income and expenses

19.8%

EXHIBIT 13.10 Common-Size Graphic of Income Statement

N et

S al

es (i

n bi

l.)

$0

$20

$40

$100

$80

$60

35%46%30%32%

$15.2$17.7

$54.9

$96.6

38%

$44.8

Americas Europe China Japan Asia Pacific

Segment percentages based on: Operating income/Net sales

EXHIBIT 13.11 Sales and Operating Income Margin Breakdown by Segment

APPLE INC. Common-Size Comparative Income Statements

Common-Size Percents* $ millions Current Yr Prior Yr Current Yr Prior Yr

Net sales . . . . . . . . . . . . . . . . . . . . . . . . . . . . . . . . . . . . . . . . . . . $229,234 $215,639 100.0% 100.0% Cost of sales . . . . . . . . . . . . . . . . . . . . . . . . . . . . . . . . . . . . . . . . 141,048 131,376 61.5 60.9 Gross margin . . . . . . . . . . . . . . . . . . . . . . . . . . . . . . . . . . . . . . . . 88,186 84,263 38.5 39.1 Research and development . . . . . . . . . . . . . . . . . . . . . . . . . . . . 11,581 10,045 5.1 4.7 Selling, general and administrative . . . . . . . . . . . . . . . . . . . . . . 15,261 14,194 6.7 6.6 Total operating expenses . . . . . . . . . . . . . . . . . . . . . . . . . . . . . . 26,842 24,239 11.7 11.2 Operating income . . . . . . . . . . . . . . . . . . . . . . . . . . . . . . . . . . . . 61,344 60,024 26.8 27.8 Other income, net . . . . . . . . . . . . . . . . . . . . . . . . . . . . . . . . . . . . 2,745 1,348 1.2 0.6 Income before provision for income taxes . . . . . . . . . . . . . . . . 64,089 61,372 28.0 28.5 Provision for income taxes . . . . . . . . . . . . . . . . . . . . . . . . . . . . . 15,738 15,685 6.9 7.3 Net income . . . . . . . . . . . . . . . . . . . . . . . . . . . . . . . . . . . . . . . . . $ 48,351 $ 45,687 21.1% 21.2%

*Percents are rounded to tenths and thus may not exactly sum to totals and subtotals.

wiL47856_ch13_496-533.indd 504 9/22/18 3:23 PM

Difference Makers in Teaching . . . Driving Decisions Whether we prepare, analyze, or apply accounting infor- mation, one skill remains essential: decision making. To help develop good decision-making habits and to show the relevance of accounting, we use a learning framework. •   Decision Insight provides context for business decisions. •   Decision Ethics and Decision Maker are role-playing

scenarios that show the relevance of accounting. •   Decision Analysis provides key tools to assess company

performance.

260 Chapter 7 Accounting Information Systems

The five components of accounting systems are source documents, input devices, information processors, information storage, and output devices. These components apply whether a system is computerized or manual. Exhibit 7.2 shows these components.

SYSTEM COMPONENTS Point: Computerized systems provide more accuracy and speed than manual.

Output Devices

Source Document

Input Devices

Information Processor

Information Storage

Cloud Storage

AppleApple

EXHIBIT 7.2 Accounting System Components

Source Documents Source documents provide the information processed by an ac- counting system. Examples include bank statements and checks, invoices from suppliers, cus- tomer bills, sales receipts, and employee earnings records. Accurate source documents are crucial to accounting information systems. Input of wrong information damages the reliability of the information system.

Input Devices Input devices take information from source documents and transfer it to information processing. These devices convert data on source documents to a form usable by the system. Journal entries are a type of input device. Keyboards and scanners are the most com- mon input devices in business.

Information Processors Information processors summarize information for use in analysis and reporting. An information processor includes journals, ledgers, working papers, and posting procedures. Each assists in transforming raw data to useful information.

Information Storage Information storage keeps data accessible to information pro- cessors. After being input and processed, data are stored for use in future analyses and reports. Auditors rely on this database when they audit both financial statements and a company’s con- trols. Modern systems depend increasingly on cloud storage.

Output Devices Output devices make accounting information available to users. Common output devices are printers, monitors, and smartphones. Output devices provide users a variety of items including customer bills, financial statements, and internal reports.

Point: Control procedures limit the possibility of entering wrong data.

Point: Controls ensure that only authorized individuals input data into the system.

©Amble Design/Shutterstock

Match each of the numbered descriptions with the principle, component, or descriptor that it best reflects. Indicate your answer by entering the letter A through J in the blank provided.

System Principles and Components

NEED-TO-KNOW 7-1

C1

A. Control principle B. Relevance principle C. Compatibility principle D. Flexibility principle

E. Cost-benefit principle F. Source documents G. Input devices H. Information processors

I. Information storage J. Output devices

System’s Fine Print Nintendo’s stock increased greatly after the huge success of Pokémon Go. However, few investors read Nintendo’s disclosures that said it owned less than one-third of the company that developed the app. When investors realized this, the stock dropped 17%, representing over $6 billion in value. ■

Decision Insight

©Eric Audras/Getty Images

the benefits of producing a specific report must outweigh the costs of time and effort to produce that report. Decisions regarding other system principles (control, relevance, compatibility, and flexibility) are also affected by the cost-benefit principle.

wiL16960_ch07_258-289.indd 260 8/1/18 1:04 PM

When a buyer is responsible for paying transportation costs, the payment is made to a carrier or directly to the seller. The cost principle requires that transportation costs of a buyer (often called transportation-in or freight-in) be part of the cost of merchandise inventory. Z-Mart’s entry to record a $75 freight charge from UPS for merchandise purchased FOB shipping point is

Point: If we place an order online and receive free shipping, we have terms FOB destination.

(d) Nov . 24 Merchandise Inventory . . . . . . . . . . . . . . . . . . . . . . . . . . . . . . . 75 Cash . . . . . . . . . . . . . . . . . . . . . . . . . . . . . . . . . . . . . . . . . . 75

Paid freight costs on goods.

Assets = Liabilities + Equity +75 −75

When a seller is responsible for paying shipping costs, it records these costs in a Delivery Expense account. Delivery expense, also called transportation-out or freight-out, is reported as a selling expense in the seller’s income statement.

Itemized Costs of Purchases In summary, purchases are recorded as debits to Merchandise Inventory (or Inventory). Purchases discounts, returns, and allowances are credited to (subtracted from) Merchandise Inventory. Transportation-in is debited (added) to Merchandise Inventory. Z-Mart’s itemized costs of merchandise purchases for the year are in Exhibit 5.8.

The accounting system described here does not provide separate records (accounts) for total purchases, total pur- chases discounts, total purchases returns and allowances, and total transportation-in. Many companies collect this information in supple- mentary records to evaluate these costs. Supplementary records, or supplemental records, refer to information outside the usual ledger accounts.

Point: INcoming freight costs are charged to INventory. When inventory EXits, freight costs are charged to EXpense.

Itemized Costs of Merchandise Purchases

Invoice cost of merchandise purchases . . . . . . . . . $ 235,800

Less: Purchases discounts received . . . . . . . . . . . . (4,200)

Purchases returns and allowances . . . . . . . . . (1,500)

Add: Costs of transportation-in . . . . . . . . . . . . . . . .       2,300

Total net cost of merchandise purchases . . . . . . $232,400

EXHIBIT 5.8 Itemized Costs of Merchandise Purchases

Point: Some companies have separate accounts for purchases discounts, returns and allowances, and transportation-in. These accounts are then transferred to Merchandise Inventory at period- end. This is a hybrid system of perpetual and periodic. That is, Merchandise Inventory is updated on a perpetual basis but only for purchases and cost of goods sold.

Payables Manager As a new accounts payable manager, you are being trained by the outgoing manager. She explains that the system prepares checks for amounts net of favorable cash discounts, and the checks are dated the last day of the discount period. She tells you that checks are not mailed until five days later, adding that “the company gets free use of cash for an extra five days, and our department looks better.” Do you continue this policy? ■ Answer: One point of view is that the late payment policy is unethical. A deliberate plan to make late payments means the company lies when it pretends to make payment within the discount period. Another view is that the late payment policy is acceptable. Some believe attempts to take discounts through late payments are accepted as “price negotiation.”

Decision Ethics

Prepare journal entries to record each of the following purchases transactions of a merchandising com- pany. Assume a perpetual inventory system using the gross method for recording purchases.

Oct. 1 Purchased $1,000 of goods. Terms of the sale are 4∕10, n∕30, and FOB shipping point; the in- voice is dated October 1.

3 Paid $30 cash for freight charges from UPS for the October 1 purchase. 7 Returned $50 of the $1,000 of goods from the October 1 purchase and received full credit. 11 Paid the amount due from the October 1 purchase (less the return on October 7). 31 Assume the October 11 payment was never made. Instead, payment of the amount due, less the

return on October 7, occurred on October 31.

Solution

P1 Merchandise Purchases

NEED-TO-KNOW 5-2

Oct . 1 Merchandise Inventory . . . . . . . . . . . . . . . . . . . . . . . . . . . . . . . 1,000

Accounts Payable . . . . . . . . . . . . . . . . . . . . . . . . . . . . . . . 1,000

Purchased goods, terms 4∕10, n∕30. Oct . 3 Merchandise Inventory . . . . . . . . . . . . . . . . . . . . . . . . . . . . . . . 30

Cash . . . . . . . . . . . . . . . . . . . . . . . . . . . . . . . . . . . . . . . . . . 30

Paid freight on purchases FOB shipping point.

[continued on next page]

wiL16960_ch05_166-213.indd 173 8/2/18 7:20 AM

Chapter 2 Analyzing and Recording Transactions 61

sheet lists its assets: cash, supplies, prepaid insurance, and equipment. The upper right side of the balance sheet shows that it owes $6,200 to creditors and $3,000 in services to customers who paid in advance. The equity section shows an ending capital balance of $33,195. Note the link between the ending balance of the statement of owner’s equity and the capital balance. (This presentation of the balance sheet is called the account form: assets on the left and liabili- ties and equity on the right. Another presentation is the report form: assets on top, followed by liabilities and then equity. Either presentation is acceptable.)

Entrepreneur You open a wholesale business selling entertainment equipment to retail outlets. Most of your cus- tomers want to buy on credit. How can you use the balance sheets of customers to decide which ones to extend credit to? ■ Answer: We use the accounting equation (Assets = Liabilities + Equity) to identify risky customers to whom we would not want to extend credit. A balance sheet provides amounts for each of these key components. The lower a customer’s equity is relative to liabilities, the less likely you would be to extend credit. A low equity means the business already has many creditor claims to it.

Decision Maker

©REDPIXEL.PL/Shutterstock

Presentation Issues Dollar signs are not used in journals and ledgers. They do appear in financial statements and other reports such as trial balances. We usually put dollar signs be- side only the first and last numbers in a column. Apple’s financial statements in Appendix A show this. Companies commonly round amounts in reports to the nearest dollar, or even to a higher level. Apple, like many large companies, rounds its financial statement amounts to the nearest million. This decision is based on the impact of rounding for users’ decisions.

Prepare a trial balance for Apple using the following condensed data from its recent fiscal year ended September 30 ($ in millions).

Preparing Trial Balance

NEED-TO-KNOW 2-4

P2Owner, Capital . . . . . . . . . . . . . . . . . . . . . . . . . . $128,249 Accounts payable . . . . . . . . . . . . . . . . . . . . . . . . 49,049

Other liabilities . . . . . . . . . . . . . . . . . . . . . . . . . . 192,223

Cost of sales (and other expenses) . . . . . . . . . . 141,048

Cash. . . . . . . . . . . . . . . . . . . . . . . . . . . . . . . . . . . 20,289

Revenues. . . . . . . . . . . . . . . . . . . . . . . . . . . . . . . 229,234

Owner, Withdrawals . . . . . . . . . . . . . . . . . . . . . . . . . $ 42,553

Investments and other assets. . . . . . . . . . . . . . . . . . 303,373

Land and equipment . . . . . . . . . . . . . . . . . . . . . . . . . 33,783

Selling and other expense . . . . . . . . . . . . . . . . . . . . 39,835

Accounts receivable . . . . . . . . . . . . . . . . . . . . . . . . . 17,874

Solution ($ in millions)

APPLE Trial Balance

September 30

Do More: E 2-8, E 2-10

APPLE

Debit Credit

Cash. . . . . . . . . . . . . . . . . . . . . . . . . . . . . . . . . . . . . . . . . . . $ 20,289

Accounts receivable . . . . . . . . . . . . . . . . . . . . . . . . . . . . . . 17,874

Land and equipment. . . . . . . . . . . . . . . . . . . . . . . . . . . . . . 33,783

Investments and other assets . . . . . . . . . . . . . . . . . . . . . . 303,373

Accounts payable . . . . . . . . . . . . . . . . . . . . . . . . . . . . . . . . $ 49,049

Other liabilities . . . . . . . . . . . . . . . . . . . . . . . . . . . . . . . . . . 192,223

Owner, Capital . . . . . . . . . . . . . . . . . . . . . . . . . . . . . . . . . . 128,249

Owner, Withdrawals . . . . . . . . . . . . . . . . . . . . . . . . . . . . . . 42,553

Revenues. . . . . . . . . . . . . . . . . . . . . . . . . . . . . . . . . . . . . . . 229,234

Cost of sales (and other expenses) . . . . . . . . . . . . . . . . . . 141,048

Selling and other expense . . . . . . . . . . . . . . . . . . . . . . . . . 39,835

Totals . . . . . . . . . . . . . . . . . . . . . . . . . . . . . . . . . . . . . . . . . $598,755 $598,755

wiL16960_ch02_044-083.indd 61 5/3/18 2:15 PM

It is important to assess a company’s risk of failing to pay its debts. Companies finance their assets with either liabilities or equity. A company that finances a relatively large portion of its assets with liabilities is said to have higher financial leverage. Higher financial leverage means greater risk because liabilities must be repaid and often require regular interest payments (equity financing does not). One measure of the risk associated with liabilities is the debt ratio as defined in Exhibit 2.17.

Costco’s total liabilities, total assets, and debt ratio for the past three years are shown in Exhibit 2.18. Costco’s debt ratio ranges from a low of 0.63 to a high of 0.70. Its ratio exceeds Walmart’s in each of the last three years, suggesting a higher than average risk from financial leverage. So, is financial leverage good or bad for Costco? The answer: If Costco is making more money with this debt than it is paying the lenders, then it is successfully borrowing money to make more money. A company’s use of debt can turn unprofitable quickly if its return from that money drops below the rate it is paying lenders.

This problem extends Need-To-Know 1-6 from Chapter 1: Jasmine Worthy started a haircutting business called Expressions. The following events occurred during its first month. a. Aug. 1 Worthy invested $3,000 cash and $15,000 of equipment in Expressions. b. 2 Expressions paid $600 cash for furniture for the shop. c. 3 Expressions paid $500 cash to rent space in a strip mall for August. d. 4 Expressions purchased $1,200 of equipment on credit for the shop (recorded as accounts

payable). e. 15 Expressions opened for business on August 5. Cash received from haircutting services in the

first week and a half of business (ended August 15) was $825. f. 16 Expressions provided $100 of haircutting services on account. g. 17 Expressions received a $100 check for services previously rendered on account. h. 18 Expressions paid $125 to an assistant for hours worked for the grand opening. i. 31 Cash received from services provided during the second half of August was $930. j. 31 Expressions paid $400 cash toward the account payable entered into on August 4. k. 31 Worthy made a $900 cash withdrawal from the company for personal use.

Required

1. Open the following ledger accounts in balance column format (account numbers are in parentheses): Cash (101); Accounts Receivable (102); Furniture (161); Store Equipment (165); Accounts Payable (201); J. Worthy, Capital (301); J. Worthy, Withdrawals (302); Haircutting Services Revenue (403); Wages Expense (623); and Rent Expense (640). Prepare general journal entries for the transactions.

COMPREHENSIVE

Journalizing and Posting Transactions, Statement Preparation, and Debt Ratio

NEED-TO-KNOW 2-5

EXHIBIT 2.17 Debt Ratio Debt ratio =

Total liabilities Total assets

62 Chapter 2 Analyzing and Recording Transactions

A2 Compute the debt ratio and describe its use in analyzing financial condition.

Debt RatioDecision Analysis

Investor You consider buying stock in Converse. As part of your analysis, you compute the company’s debt ratio for 2017, 2018, and 2019 as 0.35, 0.74, and 0.94, respectively. Based on the debt ratio, is Converse a low-risk investment? Has the risk of buying Converse stock changed over this period? (The industry debt ratio averages 0.40.) ■ Answer: The debt ratio suggests that Converse’s stock is of higher risk than normal and that this risk is rising. The average industry ratio of 0.40 supports this conclusion. The 2019 debt ratio for Converse is twice the industry norm. Also, a debt ratio approaching 1.0 indicates little to no equity.

Decision Maker

EXHIBIT 2.18 Computation and Analysis of Debt Ratio

Company $ millions Current Year 1 Year Ago 2 Years Ago

Costco Total liabilities . . . . . . . . . . . . . . . . . . . . . . . . . $25,268 $20,831 $22,174 Total assets . . . . . . . . . . . . . . . . . . . . . . . . . . . $36,347 $33,163 $33,017 Debt ratio . . . . . . . . . . . . . . . . . . . . . . . . . . . . 0.70 0.63 0.67 Walmart Debt ratio . . . . . . . . . . . . . . . . . . . . . . . . . . . . 0 .59 0 .58 0 .58

wiL16960_ch02_044-083.indd 62 7/12/18 5:20 AM

Accounting Analytics New to this edition, Accounting Analysis  assignments have students evaluate the most current financial statements from Apple, Google, and Samsung. Students  compute key metrics and compare perfor- mance between companies and industry. These assignments are auto-gradable in Connect and are included after Problem  Set B in the text.

Chapter 7 Accounting for Receivables 299

Required

1. Prepare the adjusting entry to record bad debts expense on March 31, 2020, under each separate assumption. There is a zero unadjusted balance in the Allowance for Doubtful Accounts at March 31.

a. Bad debts are estimated to be 1% of total revenues. b. Bad debts are estimated to be 2% of accounts receivable. (Round to the dollar.) 2. Assume that Business Solutions’s Accounts Receivable balance at June 30, 2020, is $20,250 and that

one account of $100 has been written off against the Allowance for Doubtful Accounts since March 31, 2020. If Rey uses the method in part 1b, what adjusting journal entry is made to recognize bad debts expense on June 30, 2020?

3. Should Rey consider adopting the direct write-off method of accounting for bad debts expense rather than one of the allowance methods considered in part 1? Explain. ©Alexander Image/Shutterstock

Check (2) Dr. Bad Debts Expense, $48

GENERAL LEDGER PROBLEM

The General Ledger tool in Connect automates several of the procedural steps in accounting so that the financial professional can focus on the impacts of each transaction on various financial reports and perfor- mance measures.

GL 7-1 General Ledger assignment GL 7-1, based on Problem 7-5A, focuses on transactions related to accounts and notes receivable and highlights the impact each transaction has on interest revenue.

GL

COMPANY ANALYSIS A1

Accounting Analysis

AA 7-1 Use Apple’s financial statements in Appendix A to answer the following. 1. What is the amount of Apple’s accounts receivable as of September 30, 2017? 2. Compute Apple’s accounts receivable turnover as of September 30, 2017. 3. How long does it take, on average, for the company to collect receivables for the fiscal year ended

September 30, 2017? 4. Apple’s most liquid assets include (a) cash and cash equivalents, (b) short-term marketable securities,

(c) accounts receivable, and (d ) inventory. Compute the percentage that these liquid assets (in total) make up of current liabilities as of September 30, 2017, and as of September 24, 2016.

5. Did Apple’s liquid assets as a percentage of current liabilities improve or worsen as of its fiscal 2017 year-end compared to its fiscal 2016 year-end?

APPLE

AA 7-2 Comparative figures for Apple and Google follow.

Apple Google

Current One Year Two Years Current One Year Two Years $ millions Year Prior Prior Year Prior Prior

Accounts receivable, net . . $ 17,874 $ 15,754 $ 16,849 $ 18,336 $14,137 $11,556

Net sales . . . . . . . . . . . . . . . 229,234 215,639 233,715 110,855 90,272 74,989

COMPARATIVE ANALYSIS A1 P2

APPLE GOOGLE

Required

1. Compute the accounts receivable turnover for (a) Apple and (b) Google for each of the two most recent years using the data shown.

2. Compute how many days, on average, it takes to collect receivables for the two most recent years for (a) Apple and (b) Google.

3. Which company more quickly collects its accounts receivable in the current year?

Hint: Average collection period equals 365 divided by the accounts receivable turnover.

wiL47856_ch07_270-301.indd 299 9/21/18 10:06 AM

Keep It Real Research shows that students learn best when using current data from real companies. Wild uses  the most current data from real companies for assignments, examples, and analysis in the text. See  Chapter 13 for use of real data.

APPLE

Samsung GOOGLE

504 Chapter 13 Analysis of Financial Statements

EXHIBIT 13.9 Common-Size Comparative Income Statements

APPLE

as representing one sales dollar, the remaining items show how each revenue dollar is distrib- uted among costs, expenses, and income.

Exhibit 13.9 shows common-size comparative income statements for each dollar of Apple’s net sales. The past two years’ common-size numbers are similar with two exceptions. One is the increase of 0.4 cents in research and development costs, which can be a positive development if these costs lead to future revenues. Another is the increase in cost of sales of 0.6 cent and increase in selling, general and administrative costs of 0.1 cent. We must monitor the growth in these expenses.

Common-Size Graphics Exhibit 13.10 is a graphic of Apple’s current-year common-size income statement. This pie chart shows the contribution of each cost component of net sales for net income.

Exhibit 13.11 takes data from Apple’s Segments footnote. The exhibit shows the level of net sales for each of Apple’s five operating seg- ments. Its Americas segment gener-

ates $96.6 billion net sales, which is roughly 42% of its total sales. Within each bar is that segment’s operating income margin (Operating income/Segment net sales). The Americas seg- ment has a 32% operating income margin. This type of graphic can raise questions about the profitability of each segment and lead to discussion of further expansions into more profitable segments. For example, the Japan segment has an operating margin of 46%. A natural question for management is what potential is there to expand sales into the Japan segment and maintain

Cost of sales 61.5%

Selling, general, administrative,

and other income 6.7%

Research and development

5.1%

Income taxes 6.9%

Net income, excluding non-

operating income and expenses

19.8%

EXHIBIT 13.10 Common-Size Graphic of Income Statement

N et

S al

es (i

n bi

l.)

$0

$20

$40

$100

$80

$60

35%46%30%32%

$15.2$17.7

$54.9

$96.6

38%

$44.8

Americas Europe China Japan Asia Pacific

Segment percentages based on: Operating income/Net sales

EXHIBIT 13.11 Sales and Operating Income Margin Breakdown by Segment

APPLE INC. Common-Size Comparative Income Statements

Common-Size Percents* $ millions Current Yr Prior Yr Current Yr Prior Yr

Net sales . . . . . . . . . . . . . . . . . . . . . . . . . . . . . . . . . . . . . . . . . . . $229,234 $215,639 100.0% 100.0% Cost of sales . . . . . . . . . . . . . . . . . . . . . . . . . . . . . . . . . . . . . . . . 141,048 131,376 61.5 60.9 Gross margin . . . . . . . . . . . . . . . . . . . . . . . . . . . . . . . . . . . . . . . . 88,186 84,263 38.5 39.1 Research and development . . . . . . . . . . . . . . . . . . . . . . . . . . . . 11,581 10,045 5.1 4.7 Selling, general and administrative . . . . . . . . . . . . . . . . . . . . . . 15,261 14,194 6.7 6.6 Total operating expenses . . . . . . . . . . . . . . . . . . . . . . . . . . . . . . 26,842 24,239 11.7 11.2 Operating income . . . . . . . . . . . . . . . . . . . . . . . . . . . . . . . . . . . . 61,344 60,024 26.8 27.8 Other income, net . . . . . . . . . . . . . . . . . . . . . . . . . . . . . . . . . . . . 2,745 1,348 1.2 0.6 Income before provision for income taxes . . . . . . . . . . . . . . . . 64,089 61,372 28.0 28.5 Provision for income taxes . . . . . . . . . . . . . . . . . . . . . . . . . . . . . 15,738 15,685 6.9 7.3 Net income . . . . . . . . . . . . . . . . . . . . . . . . . . . . . . . . . . . . . . . . . $ 48,351 $ 45,687 21.1% 21.2%

*Percents are rounded to tenths and thus may not exactly sum to totals and subtotals.

wiL47856_ch13_496-533.indd 504 9/22/18 3:23 PM

504 Chapter 13 Analysis of Financial Statements

EXHIBIT 13.9 Common-Size Comparative Income Statements

APPLE

as representing one sales dollar, the remaining items show how each revenue dollar is distrib- uted among costs, expenses, and income.

Exhibit 13.9 shows common-size comparative income statements for each dollar of Apple’s net sales. The past two years’ common-size numbers are similar with two exceptions. One is the increase of 0.4 cents in research and development costs, which can be a positive development if these costs lead to future revenues. Another is the increase in cost of sales of 0.6 cent and increase in selling, general and administrative costs of 0.1 cent. We must monitor the growth in these expenses.

Common-Size Graphics Exhibit 13.10 is a graphic of Apple’s current-year common-size income statement. This pie chart shows the contribution of each cost component of net sales for net income.

Exhibit 13.11 takes data from Apple’s Segments footnote. The exhibit shows the level of net sales for each of Apple’s five operating seg- ments. Its Americas segment gener-

ates $96.6 billion net sales, which is roughly 42% of its total sales. Within each bar is that segment’s operating income margin (Operating income/Segment net sales). The Americas seg- ment has a 32% operating income margin. This type of graphic can raise questions about the profitability of each segment and lead to discussion of further expansions into more profitable segments. For example, the Japan segment has an operating margin of 46%. A natural question for management is what potential is there to expand sales into the Japan segment and maintain

Cost of sales 61.5%

Selling, general, administrative,

and other income 6.7%

Research and development

5.1%

Income taxes 6.9%

Net income, excluding non-

operating income and expenses

19.8%

EXHIBIT 13.10 Common-Size Graphic of Income Statement

N et

S al

es (i

n bi

l.)

$0

$20

$40

$100

$80

$60

35%46%30%32%

$15.2$17.7

$54.9

$96.6

38%

$44.8

Americas Europe China Japan Asia Pacific

Segment percentages based on: Operating income/Net sales

EXHIBIT 13.11 Sales and Operating Income Margin Breakdown by Segment

APPLE INC. Common-Size Comparative Income Statements

Common-Size Percents* $ millions Current Yr Prior Yr Current Yr Prior Yr

Net sales . . . . . . . . . . . . . . . . . . . . . . . . . . . . . . . . . . . . . . . . . . . $229,234 $215,639 100.0% 100.0% Cost of sales . . . . . . . . . . . . . . . . . . . . . . . . . . . . . . . . . . . . . . . . 141,048 131,376 61.5 60.9 Gross margin . . . . . . . . . . . . . . . . . . . . . . . . . . . . . . . . . . . . . . . . 88,186 84,263 38.5 39.1 Research and development . . . . . . . . . . . . . . . . . . . . . . . . . . . . 11,581 10,045 5.1 4.7 Selling, general and administrative . . . . . . . . . . . . . . . . . . . . . . 15,261 14,194 6.7 6.6 Total operating expenses . . . . . . . . . . . . . . . . . . . . . . . . . . . . . . 26,842 24,239 11.7 11.2 Operating income . . . . . . . . . . . . . . . . . . . . . . . . . . . . . . . . . . . . 61,344 60,024 26.8 27.8 Other income, net . . . . . . . . . . . . . . . . . . . . . . . . . . . . . . . . . . . . 2,745 1,348 1.2 0.6 Income before provision for income taxes . . . . . . . . . . . . . . . . 64,089 61,372 28.0 28.5 Provision for income taxes . . . . . . . . . . . . . . . . . . . . . . . . . . . . . 15,738 15,685 6.9 7.3 Net income . . . . . . . . . . . . . . . . . . . . . . . . . . . . . . . . . . . . . . . . . $ 48,351 $ 45,687 21.1% 21.2%

*Percents are rounded to tenths and thus may not exactly sum to totals and subtotals.

wiL47856_ch13_496-533.indd 504 9/22/18 3:23 PM

vii

852 Chapter 21 Flexible Budgets and Standard Costs

Refer to the information in QS 21-14. Compute the overhead volume variance for November and classify it as favorable or unfavorable.

QS 21-15 Volume variance P4

Alvarez Company’s output for the current period yields a $20,000 favorable overhead volume variance and a $60,400 unfavorable overhead controllable variance. Standard overhead applied to production for the period is $225,000. What is the actual total overhead cost incurred for the period?

QS 21-16 Overhead cost variances

P4

Refer to the information in QS 21-16. Alvarez records standard costs in its accounts. Prepare the journal entry to charge overhead costs to the Work in Process Inventory account and to record any variances.

QS 21-17A Preparing overhead entries

P6

Refer to the information from QS 21-18. Compute the variable overhead spending variance and the vari- able overhead efficiency variance and classify each as favorable or unfavorable.

QS 21-19A Overhead spending and efficiency variances P5

Farad, Inc., specializes in selling used trucks. During the month, Farad sold 50 trucks at an average price of $9,000 each. The budget for the month was to sell 45 trucks at an average price of $9,500 each. Compute the dealership’s sales price variance and sales volume variance for the month and classify each as favor- able or unfavorable.

QS 21-20 Computing sales price and volume variances A1

AirPro Corp. reports the following for November. Compute the total overhead variance and controllable overhead variance for November and classify each as favorable or unfavorable.

QS 21-14 Controllable overhead variance

P4 Actual total factory overhead incurred . . . . . . . . . . . . . . . . . . . . . . . . . . . . $28,175 Standard factory overhead:

Variable overhead . . . . . . . . . . . . . . . . . . . . . . . . . . . . . . . . . . . . . . . . . . $3 .10 per unit produced

Fixed overhead ($12,000∕12,000 predicted units to be produced) . . . . $1 per unit Predicted units to produce . . . . . . . . . . . . . . . . . . . . . . . . . . . . . . . . . . . . . . 12,000 units

Actual units produced . . . . . . . . . . . . . . . . . . . . . . . . . . . . . . . . . . . . . . . . . . 9,800 units

QS 21-18A Total variable overhead cost variance

P5

Mosaic Company applies overhead using machine hours and reports the following information. Compute the total variable overhead cost variance and classify it as favorable or unfavorable.

Actual machine hours used . . . . . . . . . . . . . . . . . . . . . . . . . . . . . . . . . . . . . . . . . . . . . . 4,700 hours

Standard machine hours (for actual production) . . . . . . . . . . . . . . . . . . . . . . . . . . . . . . 5,000 hours

Actual variable overhead rate per hour . . . . . . . . . . . . . . . . . . . . . . . . . . . . . . . . . . . . . $4 .15

Standard variable overhead rate per hour . . . . . . . . . . . . . . . . . . . . . . . . . . . . . . . . . . $4 .00

In a recent year, BMW sold 182,158 of its 1 Series cars. Assume the company expected to sell 191,158 of these cars during the year. Also assume the budgeted sales price for each car was $30,000 and the actual sales price for each car was $30,200. Compute the sales price variance and the sales volume variance.

QS 21-21 Sales variances A1

MM Co. uses corrugated cardboard to ship its product to customers. Management believes it has found a more efficient way to package its products and use less cardboard. This new approach will reduce shipping costs from $10.00 per shipment to $9.25 per shipment. (1) If the company forecasts 1,200 shipments this year, what amount of total direct materials costs would appear on the shipping depart- ment’s flexible budget? (2) How much is this sustainability improvement predicted to save in direct materials costs for this coming year?

QS 21-22 Sustainability and standard costs

P1

wiL47856_ch21_820-867.indd 852 9/4/18 9:51 AM

Doing What’s Right Companies increasingly issue sustainability reports, and accountants are being asked to prepare, analyze, and audit them.  Wild includes brief sections in the managerial chapters. This material focuses on the importance of sustainability within the  context of accounting, including standards from the Sustainability Accounting Standards Board (SASB). Sustainability assign- ments cover chapter material with a social responsibility twist.

Chapter 18 Managerial Accounting Concepts and Principles 667

Value Chain The value chain refers to the series of activities that add value to a company’s products or services. Exhibit 18.18 illustrates a possible value chain for a retail cookie company. Companies can use lean practices across the value chain to increase efficiency and profits.

Acquire raw materials Baking Sales Service

EXHIBIT 18.18 Typical Value Chain (cookie retailer)

How Lean Principles Impact the Value Chain Adopting lean principles can be challenging because systems and procedures that a company follows must be realigned. Managerial account- ing has an important role in providing accurate cost and performance information. Developing such a system is important to measuring the “value” provided to customers. The price that cus- tomers pay for acquiring goods and services is a key determinant of value. In turn, the costs a company incurs are key determinants of price.

Corporate Social Responsibility In addition to maximizing shareholder value, cor- porations must consider the demands of other stakeholders, including employees, suppliers, and society in general. Corporate social responsibility (CSR) is a concept that goes beyond fol- lowing the law. For example, to reduce its impact on the environment, Three Twins Ice Cream uses only cups and spoons made from organic ingredients. United By Blue, an apparel and jewelry company, removes one pound of trash from waterways for every product sold. Many companies extend the concept of CSR to include sustainability, which considers fu- ture generations when making business decisions.

Triple Bottom Line Triple bottom line focuses on three measures: financial (“profits”), social (“people”), and environmental (“planet”). Adopting a triple bottom line impacts how businesses report. In response to a growing trend of such reporting, the Sustainability Accounting Standards Board (SASB) was established to develop report- ing standards for businesses’ sustainability activities. Some of the business sectors for which the SASB has developed reporting standards include health care, nonrenewable re- sources, and renewable resources and alternative energy.

Point: Companies like Microsoft, Google, and Walt Disney, ranked at the top of large multinational companies in terms of CSR, disclose CSR results on their websites.

Economic

EnvironmentalSo ci

al

Triple Bottom Line

Balanced Scorecard The balanced scorecard aids continuous improvement by augmenting financial measures with information on the “drivers” (indicators) of future financial performance along four dimensions: (1) financial—profitabil- ity and risk, (2) customer—value creation and product and service differentiation, (3) internal business processes— business activities that create customer and owner satisfaction, and (4) learning and growth— organizational change, innovation, and growth. ■

Decision Insight

In creating sustainability accounting standards, the Sustainability Accounting Standards Board (SASB) has created reporting guidelines. The SASB considers sustainability information as material if its disclosure would affect the views of equity investors on a company’s financial condition or operating performance.

Material information can vary across industries; for example, while environmental “planet” issues such as air quality, wastewater management, and biodiversity impacts are important for investments in companies in the nonrenewable resources sectors, such issues are likely not as important for investments in banks. In contrast, “people” issues such as diversity and inclusion, fair labor practices, and employee health are considered material for most sectors, particularly those that use considerable direct labor.

SUSTAINABILITY AND ACCOUNTING

©MoringaConnect

wiL16960_ch18_650-685.indd 667 6/1/18 9:37 AM

Cheat Sheets New to this edition, Cheat Sheets are provided at the end of each chapter. Cheat Sheets are roughly one page in length and  include key procedures, concepts, journal entries, and formulas.

194 Chapter 5 Accounting for Merchandising Operations

If the invoice is paid on (or before) November 12 within the discount period, it records

Gross Method—Periodic Net Method—Periodic

Accounts Payable . . . . . . . . . . . . 500 Accounts Payable . . . . . . . . . . . . . . . . 490

Purchases Discounts . . . . . 10

Cash . . . . . . . . . . . . . . . . . . 490 Cash . . . . . . . . . . . . . . . . . . . . . . 490

Gross Method—Periodic

Accounts Payable . . . . . . . . . . . . 500 Accounts Payable . . . . . . . . . . . . . . . . 490

Discounts Lost . . . . . . . . . . . . . . . . . . . 10

Cash . . . . . . . . . . . . . . . . . . 500 Cash . . . . . . . . . . . . . . . . . . . . . . 500

If the invoice is paid after the discount period, it records

SALES—Periodic For sales transactions, the perpetual and periodic entries are identical except that under the periodic system the cost-side entries are not made at the time of each sale nor for any subsequent returns. Instead, the cost of goods sold is computed at period-end based on a physical count of inventory. This entry is shown in Exhibit 5A.1.

APPENDIX

Work Sheet—Perpetual System5D This appendix along with assignments is available online.

MERCHANDISING ACTIVITIES Merchandise: Goods a company buys to resell. Cost of goods sold: Costs of merchandise sold. Gross profit (gross margin): Net sales minus cost of goods sold. Computing net income (service company vs. merchandiser):

EqualsMinusEqualsMinus Expenses Netincome

Net sales

Merchandiser

Expenses NetincomeRevenues

Service Company Minus Equals

Gross profit

Cost of goods sold

Inventory: Costs of merchandise owned, but not yet sold. It is a current asset on the balance sheet. Merchandise Cost Flows:

Net purchases

Merchandise available for sale

Cost of goods sold

Ending inventory

Beginning inventory

Perpetual inventory system: Updates accounting records for each pur- chase and each sale of inventory. Periodic inventory system: Updates accounting records for purchases and sales of inventory only at the end of a period.

Summary: Cheat Sheet

MERCHANDISING PURCHASES Cash discount: A purchases discount on the price paid by the buyer; or, a sales discount on amount received for the seller. Credit terms example: “2/10, n/60” means full payment is due within 60 days, but the buyer can deduct 2% of the invoice amount if payment is made within 10 days. Gross method: Initially record purchases at gross (full) invoice amounts. Purchasing Merchandise for Resale Entries:

Transportation Costs and Ownership Transfer Rules:

Purchasing merchandise Merchandise Inventory . . . . . . . . 500 on credit Accounts Payable . . . . . . . . 500

Paying within discount period Accounts Payable . . . . . . . . . . . . 500 (Inventory reduced by Merchandise Inventory . . . 10 discount taken) Cash . . . . . . . . . . . . . . . . . . 490

Paying outside discount Accounts Payable . . . . . . . . . . . . 500 period Cash . . . . . . . . . . . . . . . . . . 500

Recording purchases Cash or Accounts Payable . . . . . 30 returns or allowances Merchandise Inventory . . . 30

Ownership Transfers at

Goods in Transit Owned by

FOB shipping point Shipping point

Transportation Costs Paid byShipping Terms

FOB destination Destination

Buyer

Seller

Buyer Merchandise Inventory . . . # Cash . . . . . . . . . . . . . . . #

Seller Delivery Expense . . . . . . . . # Cash . . . . . . . . . . . . . . . . #

wiL16960_ch05_166-213.indd 194 8/2/18 7:20 AM

You’re in the driver’s seat. Want to build your own course? No problem. Prefer to use our turnkey, prebuilt course? Easy. Want to make changes throughout the semester? Sure. And you’ll save time with Connect’s auto-grading too.

They’ll thank you for it. Adaptive study resources like SmartBook® help your students be better prepared in less time. You can transform your class time from dull definitions to dynamic debates. Hear from your peers about the benefits of Connect at www.mheducation.com/highered/connect

Make it simple, make it affordable. Connect makes it easy with seamless integration using any of the major Learning Management Systems—Blackboard®, Canvas, and D2L, among others—to let you organize your course in one convenient location. Give your students access to digital materials at a discount with our inclusive access program. Ask your McGraw-Hill representative for more information.

Solutions for your challenges. A product isn’t a solution. Real solutions are affordable, reliable, and come with training and ongoing support when you need it and how you want it. Our Customer Experience Group can also help you troubleshoot tech problems—although Connect’s 99% uptime means you might not need to call them. See for yourself at status.mheducation.com

Students—study more efficiently, retain more and achieve better outcomes. Instructors—focus on what you love—teaching.

SUCCESSFUL SEMESTERS INCLUDE CONNECT

65% Less Time Grading

©Hill Street Studios/Tobin Rogers/Blend Images LLC

For Instructors

Effective, efficient studying. Connect helps you be more productive with your study time and get better grades using tools like SmartBook, which highlights key concepts and creates a personalized study plan. Connect sets you up for success, so you walk into class with confidence and walk out with better grades.

Study anytime, anywhere. Download the free ReadAnywhere app and access your online eBook when it’s convenient, even if you’re offline. And since the app automatically syncs with your eBook in Connect, all of your notes are available every time you open it. Find out more at www.mheducation.com/readanywhere

No surprises. The Connect Calendar and Reports tools keep you on track with the work you need to get done and your assignment scores. Life gets busy; Connect tools help you keep learning through it all.

Learning for everyone. McGraw-Hill works directly with Accessibility Services Departments and faculty to meet the learning needs of all students. Please contact your Accessibility Services office and ask them to email [email protected], or visit www.mheducation.com/accessibility for more information.

“I really liked this app—it made it easy to study when

you don't have your text- book in front of you.”

- Jordan Cunningham, Eastern Washington University

Chapter 12 Quiz Chapter 11 Quiz

Chapter 7 Quiz

Chapter 13 Evidence of Evolution Chapter 11 DNA Technology

Chapter 7 DNA Structure and Gene...

and 7 more...

13 14

©Shutterstock/wavebreakmedia

For Students

x

Connect helps students learn more efficiently by providing feedback and practice material when they need it, where they need it. Connect grades homework automatically and gives immediate feedback. ▪ Wild has auto-gradable and algorithmic

assignments; most focus on one learning objective and are targeted at introductory students.

▪ 90% of Wild’s Quick Study, Exercise, and Problem Set A assignments are available in Connect with algorithmic options.

▪ Over 210 assignments new to this edition—all available in Connect with algorithmic options. Nearly all are Quick Studies (brief exercises) and Exercises.

General Ledger Problems offer students the ability to record financial transactions and see how these transactions flow into financial statements. Easy minimal-scroll navigation, instant “Check My Work” feedback, and fully integrated hyperlinking across tabs show how inputted data affect each stage of the accounting process. General Ledger Problems expose students to general ledger software similar to that in practice, without the expense and hassle of downloading additional software. Algorithmic versions are available. All are auto- gradable.

General Ledger Problems

NEW! Concept Overview Videos Concept Overview Videos teach each chapter’s learning objectives through an engaging multimedia presentation. These learning tools enhance the text through video, audio, and checkpoint questions that can be graded—ensuring students complete and comprehend the material. Concept Overview Videos harness the power of technology to appeal to all learning styles and are ideal in all class formats. The Concept Overview Videos replace the previous edition’s Interactive Presentations.

SUPERIOR ASSIGNMENTS

xi

Excel Simulations Simulated Excel Questions, assignable within Connect, allow students to practice their Excel skills—such as basic formulas and formatting—within the context of accounting. These questions feature animated, narrated Help and Show Me tutorials (when enabled), as well as automatic feedback and grading for both students and professors. These questions differ from Applying Excel in that students work in a simulated version of Excel. Downloading the Excel application is not required to complete Simulated Excel Questions.

Guided Examples The Guided Examples (Hints) in Connect provide a narrated, animated, step-by-step walk-through of most Quick Studies, Exercises, and General Ledger Problems similar to those assigned. These short presentations can be turned on or off by instructors and provide reinforcement when students need it most.

Exercise Presentations Animated PowerPoints, created from text assignments, enable instructors to be fully prepared for in-class demonstrations. Instructors also can use these with Tegrity (in Connect) to record online lectures.

NEW! Applying Excel Applying Excel enables students to work select chapter problems or examples in Excel. These problems are assignable in Connect and give students instant feedback as they work through the problems in Excel. Accompanying Excel videos teach students how to use Excel and the primary functions needed to complete the assignment. Short assessments can be assigned to test student comprehension of key Excel skills.

xii

Content Revisions Enhance Learning

Chapter 1 Updated opener—Apple and entrepreneurial assignment. Updated salary info for accountants. Revised business entity section along with adding LLC. Updated section on FASB objectives and accounting constraints. New layout for introducing the expanded accounting equation. New layout for introducing financial statements. Updated Apple numbers for NTK 1-5. New Cheat Sheet reinforces chapter content. Updated return on assets analysis using Nike and Under Armour. Added a new Exercise assignment and Quick Study assignment. Added new analysis assignments: Company Analysis, Comparative Analysis, and Global Analysis.

Chapter 2 NEW opener—Fitbit and entrepreneurial assignment. New visual for process to get from transactions to financial statements. New layout on four types of accounts that determine equity. Improved presentation of “Double-Entry System” section. Updated Apple data for NTK 2-4. Updated debt ratio analysis using Costco and Walmart. New Cheat Sheet reinforces chapter content. Added four new Quick Studies. Added three new Exercises. Added new analysis assignments: Company Analysis, Comparative Analysis, and Global Analysis.

Chapter 3 NEW opener—Snapchat and entrepreneurial assignment. Revised learning objectives and chapter preview—each type of adjusting entry is assigned its own learning objective. Updated “Recognizing Revenues and Expenses” section. New streamlined “Framework for Adjustments” section. Enhanced Exhibit 3.12 on summary of adjustments. Enhanced Exhibit 3.19 on steps of accounting cycle with images. Streamlined section on classified balance sheet. Updated profit margin analysis using Visa and Mastercard. Updated current ratio analysis using Costco and Walmart. Improved layouts for Exhibits 3A.1 through 3A.5. New Cheat Sheet reinforces chapter content. Added three new Quick Studies. Added two new Exercises. Added new analysis assignments: Company Analysis, Comparative Analysis, and Global Analysis.

Chapter 4 NEW opener—Build-A-Bear and entrepreneurial assignment. Updated introduction for servicers vs. merchandisers using Liberty Tax and Nordstrom. Revised NTK 4-1 covers basics of merchandising. Reorganized “Purchases” section to aid learning. New Decision Insight on growing number of returns for businesses. Enhanced entries on payment of purchases within discount period vs. after discount period. Improved discussion of entries for sales with discounts vs. sales without discounts. Color-coded Exhibit 4.12 highlights different merchandising transactions. Updated acid-test ratio and gross margin analysis using Nike and Under Armour. Appendix 4B explains adjusting entries for future sales discounts, returns, and allowances. Appendix 4C covers the net method. Appendix 4D moved to online only. New Cheat Sheet reinforces chapter content. Added three new Quick Studies. Added four new Exercises. Added new analysis assignments: Company Analysis, Comparative Analysis, and Global Analysis.

Chapter 5 NEW opener—Shake Shack and entrepreneurial assignment. New Ethical Risk on the alleged fraud of Homex. Simplified introduction to inventory costing. Shortened explanation for specific identification. Enhanced layout to explain effects of inventory errors across years. Updated inventory turnover and days’ sales in inventory analysis using Costco and Walmart. Added colored arrow lines to Exhibits 5A.3 and 5A.4 to show cost flows from purchases to sales. New Cheat Sheet reinforces chapter content. Added one new Quick Study. Added two new Exercises. Added new analysis assignments: Company Analysis, Comparative Analysis, and Global Analysis.

Chapter 6 NEW opener—Care.com and entrepreneurial assignment. New COSO framework to guide internal control, including COSO cube. New discussion of internal control failure at Amazon that cost customers $150 million. Simplified bank statement for learning. Revised “Bank Reconciliation” section to separate bank balance adjustments and book balance adjustments.

New summary image on adjustments for bank balance and for book balance. Removed collection expenses and NSF fees—most are immaterial and covered in advanced courses. Updated days’ sales uncollected analysis using Starbucks and Jack in the Box. New Cheat Sheet reinforces chapter content. Added three new Quick Studies. Added eight new Exercises. Added new analysis assignments: Company Analysis, Comparative Analysis, and Global Analysis.

Chapter 7 NEW opener—Facebook and entrepreneurial assignment. Updated company data in Exhibit 7.1. Streamlined direct write-off method. Enhanced Exhibit 7.6 showing allowances set aside for future bad debts along with journal entries. New calendar graphic added as learning aid with Exhibit 7.12. New Excel demo to compute maturity dates. Updated accounts receivable analysis using Visa and Mastercard. New Cheat Sheet reinforces chapter content. Added five new Quick Studies. Added one new Exercise. Added new analysis assignments: Company Analysis, Comparative Analysis, and Global Analysis.

Chapter 8 NEW opener—New Glarus Brewery and entrepreneurial assignment. Updated company data in Exhibit 8.1. Added entry with Exhibit 8.3 and Exhibit 8.4. Simplified “Partial-Year Depreciation” section. Added margin table to Exhibit 8.14 as a learning aid. New Decision Insight box on extraordinary repairs to SpaceX’s reusable orbital rocket. New simple introduction to finance leases and operating leases for the new standard. Updated asset turnover analysis using Starbucks and Jack in the Box. Simplified Appendix 8A by postponing exchanges without commercial substance to advanced courses. New Cheat Sheet reinforces chapter content. Added two new Quick Studies. Added one new Exercise. Added two new Problems. Added new analysis assignments: Company Analysis, Comparative Analysis, and Global Analysis.

Chapter 9 NEW opener—Pandora and entrepreneurial assignment. Updated data in Exhibit 9.2. Streamlined “Short-Term Notes Payable” section. Simplified explanation of FICA taxes.

Updated payroll tax rates and explanations. Revised NTK 9-4. New W-4 form added to Appendix 9A. New Cheat Sheet reinforces chapter content. Added two new Quick Studies. Added four new Exercises. Added new analysis assignments: Company Analysis, Comparative Analysis, and Global Analysis.

Chapter 10 NEW opener—e.l.f. Cosmetics and entrepreneurial assignment. Updated IBM bond quote data. Simplified numbers in Exhibit 10.7. Simplified Exhibit 10.10 on premium bonds. Simplified numbers in Exhibit 10.11. Bond pricing moved to Appendix 10A. Simplified Exhibit 10.12 for teaching the note amortization schedule. Updated debt-to-equity analysis using Nike and Under Armour. New Excel computations for bond pricing in Appendix 10A. Simplified numbers in Exhibits 10B.1 and 10B.2. Revised Appendix 10C for new standard on finance leases and operating leases. New Cheat Sheet reinforces chapter content. Added five new Quick Studies. Added four new Exercises. Added four new Problems. Added new analysis assignments: Company Analysis, Comparative Analysis, and Global Analysis.

Chapter 11 NEW opener—Yelp and entrepreneurial assignment. New Decision Insight on bots investing in stocks based on erroneous news. New AT&T stock quote explanation. New graphic visually depicting cash dividend dates. New table summarizing differences between small stock dividends, large stock dividends, and stock splits. Updated Apple statement of equity in Exhibit 11.10. Updated PE ratio and dividend yield using Amazon, Altria, Visa, and Mastercard. Simplified book value per share explanation and computations. New Cheat Sheet reinforces chapter content. Added six new Quick Studies. Added four new Exercises. Added new analysis assignments: Company Analysis, Comparative Analysis, and Global Analysis.

Chapter 12 NEW opener—Vera Bradley and entrepreneurial assignment. Slightly revised infographics on cash flows from operating, investing, and financing. Streamlined sections on analyzing the cash account and noncash accounts.

Instructors and students guided this edition’s revisions. Revisions include ∙ New Accounting Analysis assignments—all available in Connect—

using real-world data from Apple, Google, and Samsung. ∙ Many new and revised General Ledger and Excel assignments. ∙ New assignments that focus on financial statement preparation. ∙ Updated videos for each learning objective in new Concept Overview Video format. ∙ Many new Need-to-Know (NTK) demos and accompanying videos to reinforce

learning.

∙ New Cheat Sheets at each chapter-end visually reinforce key chapter concepts. ∙ More concise text covering the same content. New 8th edition has 112 fewer

pages than 7th edition. ∙ Over 210 new assignments—all available in Connect with algorithmic options. ∙ Gross method is used for merchandising transactions, reflecting practice—adjusting

entries for new revenue recognition rules are set in an appendix. ∙ Revised Investments chapter for the new standard.

xiii

New presentation to aid learning of indirect adjustments to income. Simplified T-accounts to reconstruct cash flows. New box on Tesla’s cash outflows and growing market value. Simplified reconstruction entries to help compute cash flows. Updated cash flow on total assets analysis using Nike and Under Armour. New Cheat Sheet reinforces chapter content. Added ten new Quick Studies. Added four new Exercises. Added new analysis assignments: Company Analysis, Comparative Analysis, and Global Analysis.

Chapter 13 Updated opener—Morgan Stanley and entrepreneurial assignment. Updated data for all analyses of Apple using horizontal, vertical, and ratio analysis. Updated comparative analysis using Google and Samsung. Streamlined section on ratio analysis. Streamlined the “Analysis Reporting” section. Shortened Appendix 13A. New Cheat Sheet reinforces chapter content. Added eight new Quick Studies. Added two new Exercises. Added new analysis assignments: Company Analysis, Comparative Analysis, and Global Analysis.

Chapter 14 NEW opener—MoringaConnect and entrepreneurial assignment. Added discussion on role of managerial accounting for nonaccounting and nonbusiness majors. New margin exhibit showing product and period cost flows. Added equation boxes for total manufacturing costs and cost of goods manufactured. Added lists of common selling and administrative expenses. Updated and edited several exhibits for clarity. New Cheat Sheet reinforces chapter content. Added new analysis assignments: Company Analysis, Comparative Analysis, and Global Analysis.

Chapter 15 NEW opener—HoopSwagg and entrepreneurial assignment. Revised discussions of manufacturing costs and link between job cost sheets and general ledger. Added graphic linking job cost sheets and general ledger accounts. Enhanced exhibit of 4-step overhead process. Added formula for computing applied overhead. New short discussion of cost-plus pricing. Added margin T-accounts and calculations for clarity.

New Cheat Sheet reinforces chapter content. Added one new Quick Study. Added new analysis assignments: Company Analysis, Comparative Analysis, and Global Analysis.

Chapter 16 NEW opener—Azucar Ice Cream and entrepreneurial assignment. Revised discussion comparing process and job order costing systems. Added cost flow graphic. New margin graphic illustrating EUP. Revised discussion of weighted-average versus FIFO method of process costing. Revised discussion of using the process cost summary. New graphic on FIFO goods flow. Added margin T-accounts and calculations for clarity. New Cheat Sheet reinforces chapter content. Added one new Exercise. Added new analysis assignments: Company Analysis, Comparative Analysis, and Global Analysis.

Chapter 17 NEW opener—Sycamore Brewing and entrepreneurial assignment. New graphic showing activities for service businesses. Added examples to discussion of ABC for service businesses. New Cheat Sheet reinforces chapter content. Added one new Discussion Question. Added one new Quick Study. Added new analysis assignments: Company Analysis, Comparative Analysis, and Global Analysis.

Chapter 18 NEW opener—Ellis Island Tropical Tea and entrepreneurial assignment. Added margin graphs of fixed, variable, and mixed costs. New Excel steps to create a line chart. Moved details of creating scatter plot to Appendix 18A, with Excel steps. Revised discussion of scatter plots. Moved details of creating a CVP chart to Appendix 18C, with Excel steps. New Cheat Sheet reinforces chapter content. Added one new Exercise. Added new analysis assignments: Company Analysis, Comparative Analysis, and Global Analysis.

Chapter 19 NEW opener—Lantern Inn B&B and entrepreneurial assignment. New Cheat Sheet reinforces chapter content. Added new analysis assignments: Company Analysis, Comparative Analysis, and Global Analysis.

Chapter 20 NEW opener—Misfit Juicery and entrepreneurial assignment.

Added T-accounts and steps to exhibit margins. Added numbered steps to several exhibits. Expanded discussion of cost of goods sold budgeting. New exhibit for calculation of cash paid for interest. Expanded discussion with bulleted list on use of a master budget. New Cheat Sheet reinforces chapter content. Added one new Quick Study. Added one new Exercise. New assignment on CMA exam budgeting coverage. Added new analysis assignments: Company Analysis, Comparative Analysis, and Global Analysis.

Chapter 21 NEW opener—Away and entrepreneurial assignment. Added graph to flexible budget exhibit. Revised discussion of flexible budget. New exhibit and discussion of computing total cost variance. Edited discussion of direct materials cost variance. Edited discussion of evaluating labor variances. Edited discussion of overhead variance reports. New exhibit for summary of variances. New Cheat Sheet reinforces chapter content. Added two new Exercises. Added new analysis assignments: Company Analysis, Comparative Analysis, and Global Analysis.

Chapter 22 NEW opener—Jibu and entrepreneurial assignment. Updated Walt Disney ROI example. New Decision Analysis on cash conversion cycle. New Cheat Sheet reinforces chapter content. Added two new Quick Studies. Added two new Exercises. Added new analysis assignments: Company Analysis, Comparative Analysis, and Global Analysis.

Chapter 23 NEW opener—Solugen and entrepreneurial assignment. Organized decision scenarios into three types: production, capacity, and pricing. Expanded discussion of product pricing. Added other pricing methods: value-based, auction-based, and dynamic. New Decision Insight on blockchain technology. New Decision Analysis on time and materials pricing of services. New Cheat Sheet reinforces chapter content. Added four new Quick Studies. Added one new Exercise.

Added new analysis assignments: Company Analysis, Comparative Analysis, and Global Analysis.

Chapter 24 NEW opener—Fellow Robots and entrepreneurial assignment. Added example of investment in robotics. New discussion of postaudit of investment decisions. New Cheat Sheet reinforces chapter content. Added two new Exercises. Added new analysis assignments: Company Analysis, Comparative Analysis, and Global Analysis.

Appendix A New financial statements for Apple, Google, and Samsung.

Appendix B New Decision Maker on postponed retail pricing. Continued Excel demos for PV and FV of lump sums. Continued Excel demos for PV and FV of annuities.

Appendix C New learning objective P4 for new category of stock investments. Revised and simplified Exhibit C.2 for new standard on investments. Reorganized text to first explain debt securities and then stock securities. Revised trading and available-for-sale securities to cover only debt securities given the new standard. New section on stock investments with insignificant influence. New Exhibit C.6 to describe accounting for equity securities by ownership level. Updated component-returns analysis using Costco and Walmart. New Cheat Sheet reinforces chapter content. Added three new Quick Studies. Added four new Exercises. Added two new Problems. Added new analysis assignments: Company Analysis, Comparative Analysis, and Global Analysis.

Appendix D NEW appendix on lean principles and accounting. Describes lean business principles. Measures production efficiency. Illustrates how to account for product costs using lean accounting. New: 13 Discussion Questions, 14 Quick Studies, 14 Exercises, and 3 Problems.

xiv

Darlene Adkins, University of Tennessee–Martin Peter Aghimien, Indiana University South Bend Janice Akao, Butler Community College Nathan Akins, Chattahoochee Technical College John Alpers, Tennessee Wesleyan University Sekhar Anantharaman, Indiana University of Pennsylvania Karen Andrews, Lewis-Clark State College Chandra D. Arthur, Cuyahoga Community College Steven Ault, Montana State University Victoria Badura, Metropolitan Community College Felicia Baldwin, City College of Chicago Reb Beatty, Anne Arundel Community College Robert Beebe, Morrisville State College George Henry Bernard, Seminole State College of Florida Cynthia Bird, Tidewater Community College, Virginia Beach Pascal Bizarro, Bowling Green State University Amy Bohrer, Tidewater Community College, Virginia Beach John Bosco, North Shore Community College Nicholas Bosco, Suffolk County Community College Jerold K. Braun, Daytona State College Doug Brown, Forsyth Technical Community College Tracy L. Bundy, University of Louisiana at Lafayette Marci Butterfield, University of Utah Ann Capion, Scott Community College Amy Cardillo, Metropolitan State University of Denver Anne Cardozo, Broward College Crystal Carlson-Myer, Indian River State College Julie Chasse, Des Moines Area Community College Patricia Chow, Grossmont College Maria Coclin, Community College of Rhode Island Michael Cohen, Lewis-Clark State College Jerilyn Collins, Herzing University Scott Collins, Penn State University, University Park William Conner, Tidewater Community College Erin Cornelsen, University of South Dakota

Mariah Dar, John Tyler Community College Nichole Dauenhauer, Lakeland Community College Donna DeMilia, Grand Canyon University Tiffany DeRoy, University of South Alabama Susan Dickey, Motlow State Community College Erin Dischler, Milwaukee Area Technical College–West Allis Holly Dixon, State College of Florida Vicky Dominguez, College of Southern Nevada David Doyon, Southern New Hampshire University Chester Drake, Central Texas College Christopher Eller, Appalachian State University Cynthia Elliott, Southwest Tennessee Community College–Macon Kim Everett, East Carolina University Corinne Frad, Eastern Iowa Community College Krystal Gabel, Southeast Community College Harry Gallatin, Indiana State University Rena Galloway, State Fair Community College Rick Gaumer, University of Wisconsin–Green Bay Tammy Gerszewski, University of North Dakota Pradeep Ghimire, Rappahannock Community College Marc Giullian, Montana State University, Bozeman Nelson Gomez, Miami Dade College–Kendall Robert Goodwin, University of Tampa Steve G. Green, U.S. Air Force Academy Darryl Greene, Muskegon Community College Lisa Hadley, Southwest Tennessee Community College–Macon Penny Hahn, KCTCS Henderson Community College Yoon Han, Bemidji State University Becky Hancock, El Paso Community College Amie Haun, University of Tennessee–Chattanooga Michelle Hays, Kalamazoo Valley Community College Rhonda Henderson, Olive Harvey College Lora Hines, John A. Logan College

Acknowledgments John J. Wild, Ken W. Shaw, and McGraw-Hill Education recognize the following instructors for their valuable feedback and involvement in the development of Financial and Managerial Accounting. We are thankful for their suggestions, counsel, and encouragement.

xv

Rob Hochschild, Ivy Tech Community College of Indiana– South Bend John Hoover, Volunteer State Community College Roberta Humphrey, Southeast Missouri State University Carley Hunzeker, Metro Community College, Elkhorn Kay Jackson, Tarrant County College South Elizabeth Jennison, Saddleback College Mary Jepperson, Saint John’s University Vicki Jobst, Benedictine University Odessa Jordan, Calhoun Community College Susan Juckett, Victoria College Amanda Kaari, Central Georgia Technical College Ramadevi Kannan, Owens Community College Jan Klaus, University of North Texas Aaron P. Knape, The University of New Orleans Cedric Knott, Henry Ford Community College Robin Knowles, Texas A&M International University Kimberly Kochanny, Central Piedmont Community College Sergey Komissarov, University of Wisconsin–La Crosse Stephanie Lareau Kroeger, Ocean County College Joseph Krupka, Lander University Tara Laken, Joliet Junior College Suzanne Lay, Colorado Mesa University Brian Lazarus, Baltimore City Community College Kevin Leifer, Long Island University, CW Post Campus Harold Levine, Los Angeles Valley College Yuebing Liu, University of Tampa Philip Lee Little, Coastal Carolina University Delores Loedel, Miracosta College Rebecca Lohmann, Southeast Missouri State University Ming Lu, Santa Monica Community College Annette C. Maddox, Georgia Highlands College Natasha Maddox, KCTCS Maysville Community and Technical College Rich Mandau, Piedmont Technical College Robert Maxwell, College of the Canyons Karen McCarron, Georgia Gwinnett College Michael McDonald, College of Southern Nevada Gwendolyn McFadden-Wade, North Carolina A&T University Allison McLeod, University of North Texas

Kate McNeil, Johnson County Community College Jane Medling, Saddleback College Heidi H. Meier, Cleveland State University Tammy Metzke, Milwaukee Area Technical College Jeanine Metzler, Northampton Community College Michelle Meyer, Joliet Junior College Pam Meyer, University of Louisiana at Lafayette Deanne Michaelson, Pellissippi State Community College Susan Miller, County College of Morris Carmen Morgan, Oregon Tech Karen Satterfield Mozingo, Pitt Community College Haris Mujahid, South Seattle College Andrea Murowski, Brookdale Community College Jaclynn Myers, Sinclair Community College Micki Nickla, Ivy Tech Community College of Indiana–Gary Dan O’Brien, Madison College–Truax Jamie O’Brien, South Dakota State University Grace Odediran, Union County College Ashley Parker, Grand Canyon University Pamela Parker, NOVA Community College Alexandria Margaret Parrish, John Tyler Community College Reed Peoples, Austin Community College Rachel Pernia, Essex County College Brandis Phillips, North Carolina A&T University Debbie Porter, Tidewater Community College–Virginia Beach M. Jeff Quinlan, Madison Area Technical College James E. Racic, Lakeland Community College Ronald de Ramon, Rockland Community College Robert J. Rankin, Texas A&M University–Commerce Robert Rebman, Benedictine University Jenny Resnick, Santa Monica Community College DeAnn Ricketts, York Technical College Renee Rigoni, Monroe Community College Kevin Rosenberg, Southeastern Community College David Rosser, University of Texas at Arlington Michael J. Rusek, Eastern Gateway Community College Alfredo Salas, El Paso Community College Carolyn Satz, Tidewater Community College–Chesapeake Kathy Saxton, Bryant & Stratton College

xvi

Many talented educators and professionals have worked hard to create the materials for this product, and for their efforts, we’re grateful. We extend a special thank you to our contributing and technology supplement authors, who have worked so diligently to support this product.

Contributing Author, Connect Content, General Ledger Problems, and Exercise PowerPoints: Kathleen O’Donnell, Onondaga Community College

Text and Supplements Accuracy Checkers: Dave Krug, Johnson County Community College; Mark McCarthy, East Carolina University; Kate McNeil, Johnson County Community College; Wanda Wong, Chabot College; and Beth Kobylarz

Test Bank Authors and Accuracy Checkers: Melodi Bunting, Madison College; Brian Schmoldt, Madison College; M. Jeff Quinlan, Madison College; and Teri Zuccaro, Clarke University

LearnSmart Author, Concept Overview Videos, PowerPoint Presentations, and Instructor Resource Manual: April Mohr, Jefferson Community and Technical College, SW

Wilson Seda, Lehman College–CUNY Perry Sellers, Lonestar College–North Harris James Shimko, Ferris State University Philip Slater, Forsyth Technical Community College Clayton Smith, Columbia College Chicago Patricia Smith, DePaul University Jane Stam, Onondaga Community College Natalie Strouse, Notre Dame College Erica Teague-Friend, Gwinnett Technical College Louis Terrero, Lehman College Geoff Tickell, Indiana University of Pennsylvania Judith A. Toland, Bucks County Community College Debra Touhey, Ocean County College Jim Ulmer, Angelina College Bob Urell, Irvine Valley College Kevin Veneskey, Ivy Tech Community College

Teresa Walker, North Carolina A&T University Terri Walsh, Seminole State College of Florida Eric Weinstein, Suffolk County Community College, Brentwood Andy Welchel, Greenville Technical College Joe Welker, College of Western Idaho Jean Wells, Howard University Denise White, Austin Community College Jonathan M. Wild, Oklahoma State University Kenneth Wise, Wilkes Community College Shondra Woessner, Holyoke Community College Mindy Wolfe, Arizona State University Jan Workman, East Carolina University Lori Zaher, Bucks County Community College Jessie Zetnick, Texas Woman’s University Laurence Zuckerman, Fulton-Montgomery Community College

Special recognition extends to the entire team at McGraw-Hill Education: Tim Vertovec, Steve Schuetz, Natalie King, Michelle Williams, Julie Wolfe, Michele Janicek, Christina Sanders, Michael McCormick, Lori Koetters, Xin Lin, Kevin Moran, Debra Kubiak, Brian Nacik, and Daryl Horrocks. We could not have published this new edition without your efforts.

John J. Wild Ken W. Shaw

xvii

Brief Contents 1 Accounting in Business 2 2 Accounting for Business

Transactions 44

3 Adjusting Accounts for Financial Statements 84

4 Accounting for Merchandising Operations 142

5 Inventories and Cost of Sales 190 6 Cash, Fraud, and Internal Control 234 7 Accounting for Receivables 270 8 Accounting for Long-Term Assets 302 9 Accounting for Current Liabilities 340 10 Accounting for Long-Term Liabilities 380 11 Corporate Reporting and Analysis 416 12 Reporting Cash Flows 452 13 Analysis of Financial Statements 496 14 Managerial Accounting Concepts and

Principles 534

15 Job Order Costing and Analysis 570 16 Process Costing and Analysis 610

17 Activity-Based Costing and Analysis 656 18 Cost Behavior and Cost-Volume-Profit

Analysis 696

19 Variable Costing and Analysis 738 20 Master Budgets and Performance

Planning 770

21 Flexible Budgets and Standard Costs 820

22 Performance Measurement and Responsibility Accounting 868

23 Relevant Costing for Managerial Decisions 912

24 Capital Budgeting and Investment Analysis 946

A Financial Statement Information A-1 B Time Value of Money B C Investments C D Lean Principles and Accounting D-1 CA Chart of Accounts CA BR Brief Review BR-1

xviii

Preface iv

1 Accounting in Business 2 Importance of Accounting 3

Users of Accounting Information 4 Opportunities in Accounting 4

Fundamentals of Accounting 6 Ethics—A Key Concept 6 Generally Accepted Accounting Principles 7 Conceptual Framework 7

Business Transactions and Accounting 9 Accounting Equation 10 Transaction Analysis 11 Summary of Transactions 14

Communicating with Users 15 Income Statement 15 Statement of Retained Earnings 17 Balance Sheet 17 Statement of Cash Flows 17

Decision Analysis—Return on Assets 18 Appendix 1A Return and Risk 21 Appendix 1B Business Activities 22

2 Accounting for Business Transactions 44 Basis of Financial Statements 45

Source Documents 45 The “Account” Underlying Financial Statements 45 Ledger and Chart of Accounts 48

Double-Entry Accounting 49 Debits and Credits 49 Double-Entry System 49

Analyzing and Processing Transactions 51 Journalizing and Posting Transactions 51 Processing Transactions—An Example 52 Summarizing Transactions in a Ledger 57

Trial Balance 58 Preparing a Trial Balance 58 Financial Statements Prepared from Trial Balance 58

Decision Analysis—Debt Ratio 62

3 Adjusting Accounts for Financial Statements 84

Timing and Reporting 85 The Accounting Period 85 Accrual Basis versus Cash Basis 86 Recognizing Revenues and Expenses 86 Framework for Adjustments 87

Deferral of Expense 87 Prepaid Insurance 87 Supplies 88 Other Prepaid Expenses 89 Depreciation 89

Deferral of Revenue 91 Unearned Consulting Revenue 91

Accrued Expense 93 Accrued Salaries Expense 93 Accrued Interest Expense 94 Future Cash Payment of Accrued

Expenses 94

Accrued Revenue 95 Accrued Services Revenue 96 Accrued Interest Revenue 96 Future Cash Receipt of Accrued Revenues 96 Links to Financial Statements 97

Trial Balance and Financial Statements 98 Adjusted Trial Balance 98 Preparing Financial Statements 99

Closing Process 100 Temporary and Permanent Accounts 101 Recording Closing Entries 101 Post-Closing Trial Balance 104

Accounting Cycle 104 Classified Balance Sheet 105

Classification Structure 105 Classification Categories 106

Decision Analysis—Profit Margin and Current Ratio 108

Appendix 3A Alternative Accounting for Prepayments 111

Appendix 3B Work Sheet as a Tool 113 Appendix 3C Reversing Entries 115

Contents

Contents xix

Valuing Inventory at LCM and the Effects of Inventory Errors 200 Lower of Cost or Market 200 Financial Statement Effects of Inventory

Errors 201

Decision Analysis—Inventory Turnover and Days’ Sales in Inventory 203

Appendix 5A Inventory Costing under a Periodic System 209

Appendix 5B Inventory Estimation Methods 214

6 Cash, Fraud, and Internal Control 234 Fraud and Internal Control 235

Purpose of Internal Control 235 Principles of Internal Control 236 Technology, Fraud, and Internal Control 237 Limitations of Internal Control 237

Control of Cash 238 Cash, Cash Equivalents, and Liquidity 238 Cash Management 238 Control of Cash Receipts 239 Control of Cash Payments 241

Banking Activities as Controls 245 Basic Bank Services 245 Bank Statement 246 Bank Reconciliation 247

Decision Analysis—Days’ Sales Uncollected 250 Appendix 6A Documentation and

Verification 252

7 Accounting for Receivables 270 Valuing Accounts Receivable 271 Direct Write-Off Method 274 Allowance Method 275 Estimating Bad Debts 278

Percent of Sales Method 278 Percent of Receivables Method 278 Aging of Receivables Method 279

Notes Receivable 281 Computing Maturity and Interest 282 Recording Notes Receivable 283 Valuing and Settling Notes 283 Disposal of Receivables 285

Decision Analysis—Accounts Receivable Turnover 285

4 Accounting for Merchandising Operations 142

Merchandising Activities 143 Reporting Income for a Merchandiser 143 Reporting Inventory for a Merchandiser 144 Operating Cycle for a Merchandiser 144 Inventory Systems 144

Accounting for Merchandise Purchases 145 Purchases without Cash Discounts 145 Purchases with Cash Discounts 145 Purchases with Returns and Allowances 147 Purchases and Transportation Costs 148

Accounting for Merchandise Sales 150 Sales without Cash Discounts 150 Sales with Cash Discounts 151 Sales with Returns and Allowances 151

Adjusting and Closing for Merchandisers 153 Adjusting Entries for Merchandisers 153 Preparing Financial Statements 154 Closing Entries for Merchandisers 154 Summary of Merchandising Entries 155

More on Financial Statement Formats 156 Multiple-Step Income Statement 156 Single-Step Income Statement 157 Classified Balance Sheet 158

Decision Analysis—Acid-Test and Gross Margin Ratios 159

Appendix 4A Periodic Inventory System 163 Appendix 4B Adjusting Entries under New Revenue

Recognition Rules 167 Appendix 4C Net Method for Merchandising 168

5 Inventories and Cost of Sales 190 Inventory Basics 191

Determining Inventory Items 191 Determining Inventory Costs 192 Internal Controls and Taking

a Physical Count 192

Inventory Costing under a Perpetual System 193 Inventory Cost Flow Assumptions 193 Inventory Costing Illustration 194 Specific Identification 194 First-In, First-Out 195 Last-In, First-Out 195 Weighted Average 196 Financial Statement Effects of Costing Methods 197 Tax Effects of Costing Methods 198

xx Contents

Warranty Liabilities 350 Multi-Period Estimated Liabilities 351

Contingent Liabilities 352 Accounting for Contingent Liabilities 352 Applying Rules of Contingent Liabilities 353 Uncertainties That Are Not Contingencies 353

Decision Analysis—Times Interest Earned Ratio 353

Appendix 9A Payroll Reports, Records, and Procedures 356

Appendix 9B Corporate Income Taxes 361

10 Accounting for Long-Term Liabilities 380 Basics of Bonds 381

Bond Financing 381 Bond Issuing 382 Bond Trading 382

Par Bonds 382 Discount Bonds 383

Bond Discount or Premium 383 Issuing Bonds at a Discount 384

Premium Bonds 386 Issuing Bonds at a Premium 386 Bond Retirement 388

Long-Term Notes Payable 390 Installment Notes 390 Mortgage Notes and Bonds 391

Decision Analysis—Debt Features and the Debt-to-Equity Ratio 392

Appendix 10A Bond Pricing 395 Appendix 10B Effective Interest

Amortization 397 Appendix 10C Leases and Pensions 398

11 Corporate Reporting and Analysis 416 Corporate Form of Organization 417

Corporate Advantages 417 Corporate Disadvantages 417 Corporate Organization and Management 418 Corporate Stockholders 418 Corporate Stock 419

Common Stock 420 Issuing Par Value Stock 420 Issuing No-Par Value Stock 421 Issuing Stated Value Stock 421 Issuing Stock for Noncash Assets 421

8 Accounting for Long-Term Assets 302 SECTION 1—PLANT ASSETS 303 Cost Determination 304

Machinery and Equipment 304 Buildings 304 Land Improvements 304 Land 304 Lump-Sum Purchase 305

Depreciation 305 Factors in Computing Depreciation 305 Depreciation Methods 305 Partial-Year Depreciation 309 Change in Estimates 310 Reporting Depreciation 310

Additional Expenditures 311 Ordinary Repairs 312 Betterments and Extraordinary Repairs 312

Disposals of Plant Assets 312 Discarding Plant Assets 313 Selling Plant Assets 313

SECTION 2—NATURAL RESOURCES 315 Cost Determination and Depletion 315 Plant Assets Tied into Extracting 316

SECTION 3—INTANGIBLE ASSETS 317 Cost Determination and Amortization 317 Types of Intangibles 317

Decision Analysis—Total Asset Turnover 320 Appendix 8A Exchanging Plant Assets 323

9 Accounting for Current Liabilities 340 Known Liabilities 341

Characteristics of Liabilities 341 Examples of Known Liabilities 342 Accounts Payable 343 Sales Taxes Payable 343 Unearned Revenues 343 Short-Term Notes Payable 343

Payroll Liabilities 346 Employee Payroll and Deductions 346 Employer Payroll Taxes 347 Internal Control of Payroll 348 Multi-Period Known Liabilities 348

Estimated Liabilities 349 Health and Pension Benefits 349 Vacation Benefits 350 Bonus Plans 350

Contents xxi

13 Analysis of Financial Statements 496 Basics of Analysis 497

Purpose of Analysis 497 Building Blocks of Analysis 497 Information for Analysis 498 Standards for Comparisons 498 Tools of Analysis 498

Horizontal Analysis 498 Comparative Statements 498 Trend Analysis 501

Vertical Analysis 502 Common-Size Statements 502 Common-Size Graphics 504

Ratio Analysis 506 Liquidity and Efficiency 506 Solvency 508 Profitability 509 Market Prospects 510 Summary of Ratios 511

Decision Analysis—Analysis Reporting 512 Appendix 13A Sustainable Income 515

14 Managerial Accounting Concepts and Principles 534

Managerial Accounting Basics 535 Purpose of Managerial Accounting 535 Nature of Managerial Accounting 536 Fraud and Ethics in Managerial Accounting 537 Career Paths 538

Managerial Cost Concepts 539 Types of Cost Classifications 539 Identification of Cost Classifications 541 Cost Concepts for Service Companies 541

Managerial Reporting 542 Manufacturing Costs 542 Nonmanufacturing Costs 542 Prime and Conversion Costs 543 Costs and the Balance Sheet 543 Costs and the Income Statement 543

Cost Flows and Cost of Goods Manufactured 546 Flow of Manufacturing Activities 546 Schedule of Cost of Goods Manufactured 547 Trends in Managerial Accounting 550

Decision Analysis—Raw Materials Inventory Turnover and Days’ Sales in Raw Materials Inventory 552

Dividends 422 Cash Dividends 422 Stock Dividends 423 Stock Splits 425 Financial Statement Effects of Dividends and

Splits 425

Preferred Stock 426 Issuance of Preferred Stock 426 Dividend Preference of Preferred Stock 427 Reasons for Issuing Preferred Stock 427

Treasury Stock 429 Purchasing Treasury Stock 429 Reissuing Treasury Stock 429

Reporting of Equity 431 Statement of Retained Earnings 431 Statement of Stockholders’ Equity 432

Decision Analysis—Earnings per Share, Price-Earnings Ratio, Dividend Yield, and Book Value per Share 432

12 Reporting Cash Flows 452 Basics of Cash Flow Reporting 453

Purpose of the Statement of Cash Flows 453 Importance of Cash Flows 453 Measurement of Cash Flows 453 Classification of Cash Flows 454 Noncash Investing and Financing 455 Format of the Statement of Cash Flows 455 Preparing the Statement of Cash Flows 456

Cash Flows from Operating 457 Indirect and Direct Methods of Reporting 457 Applying the Indirect Method 457 Summary of Adjustments for Indirect Method 460

Cash Flows from Investing 461 Three-Step Analysis 461 Analyzing Noncurrent Assets 461

Cash Flows from Financing 463 Three-Step Analysis 463 Analyzing Noncurrent Liabilities 463 Analyzing Equity 464 Proving Cash Balances 464

Summary Using T-Accounts 466 Decision Analysis—Cash Flow Analysis 467 Appendix 12A Spreadsheet Preparation of the

Statement of Cash Flows 470 Appendix 12B Direct Method of Reporting

Operating Cash Flows 472

xxii Contents

Decision Analysis—Hybrid Costing System 627

Appendix 16A FIFO Method of Process Costing 631

17 Activity-Based Costing and Analysis 656 Assigning Overhead Costs 657

Alternative Methods of Overhead Allocation 657

Plantwide Overhead Rate Method 658 Departmental Overhead Rate Method 660 Assessing Plantwide and Departmental

Overhead Rate Methods 662

Activity-Based Costing 663 Steps in Activity-Based Costing 663 Applying Activity-Based Costing 664 Assessing Activity-Based Costing 668

Activity-Based Management 669 Activity Levels and Cost Management 669 Costs of Quality 670 Lean Manufacturing 671 ABC for Service Providers 671

Decision Analysis—Customer Profitability 673

18 Cost Behavior and Cost-Volume-Profit Analysis 696

Identifying Cost Behavior 697 Fixed Costs 698 Variable Costs 698 Graphing Fixed and Variable Costs against

Volume 698 Mixed Costs 698 Step-wise Costs 699 Curvilinear Costs 700

Measuring Cost Behavior 701 Scatter Diagram 701 High-Low Method 702 Regression 702 Comparing Cost Estimation Methods 702

Contribution Margin and Break-Even Analysis 703 Contribution Margin and Its Measures 703 Break-Even Point 704 Cost-Volume-Profit Chart 706 Changes in Estimates 706

Applying Cost-Volume-Profit Analysis 707 Margin of Safety 707 Computing Income from Sales and Costs 708

15 Job Order Costing and Analysis 570 Job Order Costing 571

Cost Accounting System 571 Job Order Production 571 Job Order vs. Process Operations 572 Production Activities in Job Order Costing 572 Cost Flows 573 Job Cost Sheet 573

Materials and Labor Costs 574 Materials Cost Flows and Documents 574 Labor Cost Flows and Documents 577

Overhead Costs 578 Set Predetermined Overhead Rate 579 Apply Estimated Overhead 579 Record Actual Overhead 581 Summary of Cost Flows 582 Using Job Cost Sheets for Managerial

Decisions 583 Schedule of Cost of Goods Manufactured 584

Adjusting Overhead 585 Factory Overhead Account 585 Adjust Underapplied or Overapplied

Overhead 585 Job Order Costing of Services 586

Decision Analysis—Pricing for Services 587

16 Process Costing and Analysis 610 Process Operations 611

Organization of Process Operations 611 Comparing Process and Job Order Costing

Systems 612 Equivalent Units of Production 613

Process Costing Illustration 614 Overview of GenX Company’s Process

Operation 614 Pre-Step: Collect Production and Cost Data 615 Step 1: Determine Physical Flow of Units 616

Step 2: Compute Equivalent Units

of Production 616 Step 3: Compute Cost per Equivalent Unit 617 Step 4: Assign and Reconcile Costs 617 Process Cost Summary 619

Accounting for Process Costing 620 Accounting for Materials Costs 621 Accounting for Labor Costs 622 Accounting for Factory Overhead 623 Accounting for Transfers 624 Trends in Process Operations 626

Contents xxiii

Investing and Financing Budgets 781 Capital Expenditures Budget 781 Cash Budget 781

Budgeted Financial Statements 785 Budgeted Income Statement 785 Budgeted Balance Sheet 786 Using the Master Budget 786 Budgeting for Service Companies 786

Decision Analysis—Activity-Based Budgeting 787

Appendix 20A Merchandise Purchases Budget 795

21 Flexible Budgets and Standard Costs 820 Fixed and Flexible Budgets 821

Fixed Budget Reports 822 Budget Reports for Evaluation 823 Flexible Budget Reports 823

Standard Costing 827 Standard Costs 827 Setting Standard Costs 827 Cost Variance Analysis 828

Materials and Labor Variances 830 Materials Variances 830 Labor Variances 832

Overhead Standards and Variances 833 Flexible Overhead Budgets 833 Standard Overhead Rate 833 Computing Overhead Cost Variances 835 Standard Costing—Management

Considerations 838

Decision Analysis—Sales Variances 839 Appendix 21A Expanded Overhead Variances and

Standard Cost Accounting System 844

22 Performance Measurement and Responsibility Accounting 868

Responsibility Accounting 869 Performance Evaluation 869 Controllable versus Uncontrollable Costs 870 Responsibility Accounting for Cost Centers 870

Profit Centers 872 Direct and Indirect Expenses 872 Expense Allocations 873 Departmental Income Statements 874 Departmental Contribution to Overhead 877

Computing Sales for a Target Income 709 Evaluating Strategies 710 Sales Mix and Break-Even 711 Assumptions in Cost-Volume-Profit Analysis 713

Decision Analysis—Degree of Operating Leverage 714

Appendix 18A Using Excel for Cost Estimation 716 Appendix 18B Variable Costing and Performance

Reporting 717 Appendix 18C Preparing a CVP Chart 720

19 Variable Costing and Analysis 738 Introducing Variable Costing and Absorption

Costing 739 Computing Unit Product Cost 740

Income Reporting Implications 741 Units Produced Equal Units Sold 741 Units Produced Exceed Units Sold 743 Units Produced Are Less Than Units Sold 744 Summarizing Income Reporting 745 Converting Income under Variable Costing to

Absorption Costing 746

Comparing Variable Costing and Absorption Costing 746 Planning Production 746 Setting Prices 748 Controlling Costs 748 CVP Analysis 749 Variable Costing for Service Firms 749

Decision Analysis—Pricing Special Orders 751

20 Master Budgets and Performance Planning 770

Budget Process and Administration 771 Budgeting Process 771 Benefits of Budgeting 772 Budgeting and Human Behavior 772 Budget Reporting and Timing 773 Master Budget Components 773

Operating Budgets 773 Sales Budget 775 Production Budget 775 Direct Materials Budget 776 Direct Labor Budget 777 Factory Overhead Budget 778 Selling Expense Budget 779 General and Administrative Expense Budget 780

xxiv Contents

24 Capital Budgeting and Investment Analysis 946

Capital Budgeting 947 Capital Budgeting Process 947 Capital Investment Cash Flows 948

Methods Not Using Time Value of Money 948 Payback Period 948 Accounting Rate of Return 951

Methods Using Time Value of Money 952 Net Present Value 952 Internal Rate of Return 956 Comparison of Capital Budgeting Methods 958 Postaudit 958

Decision Analysis—Break-Even Time 960 Appendix 24A Using Excel to Compute Net Present

Value and Internal Rate of Return 962

Appendix A Financial Statement Information A-1 Apple A-2 Google A-10 Samsung A-14 Appendix B Time Value of Money B Appendix C Investments C Appendix D Lean Principles and Accounting D-1 Index IND-1 Chart of Accounts CA Brief Review Managerial Analyses and Reports BR-1 Financial Reports and Tables BR-2 Selected Transactions and

Relations BR-3 Fundamentals and Analyses BR-4

Investment Centers 878 Return-on-Investment and Residual Income 878 Investment Center Profit Margin and Investment

Turnover 880

Nonfinancial Performance Evaluation Measures 881 Balanced Scorecard 881 Transfer Pricing 883

Decision Analysis—Cash Conversion Cycle 884 Appendix 22A Cost Allocations 887 Appendix 22B Transfer Pricing 889 Appendix 22C Joint Costs and Their Allocation 890

23 Relevant Costing for Managerial Decisions 912

Decisions and Information 913 Decision Making 913 Relevant Costs and Benefits 914

Production Decisions 914 Make or Buy 915 Sell or Process Further 916 Sales Mix Selection When Resources Are

Constrained 917

Capacity Decisions 919 Segment Elimination 919 Keep or Replace Equipment 920

Pricing Decisions 921 Normal Pricing 921 Special Offers 923

Decision Analysis—Time and Materials Pricing 925

Design elements: Lightbulb: ©Chuhail/Getty Images; Blue globe: ©nidwlw/Getty Images and ©Dizzle52/Getty Images; Chess piece: ©Andrei Simonenko/Getty Images and ©Dizzle52/Getty Images; Mouse: ©Siede Preis/Getty Images; Global View globe: ©McGraw-Hill Education and ©Dizzle52/Getty Images; Sustainability: ©McGraw-Hill Education and ©Dizzle52/Getty Images

Financial & Managerial Accounting

Learning Objectives

CONCEPTUAL C1 Explain the purpose and importance of

accounting.

C2 Identify users and uses of, and opportunities in, accounting.

C3 Explain why ethics are crucial to accounting.

C4 Explain generally accepted accounting principles and define and apply several accounting principles.

PROCEDURAL P1 Analyze business transactions using the

accounting equation.

P2 Identify and prepare basic financial statements and explain how they interrelate.

C5 Appendix 1B—Identify and describe the three major activities of organizations.

ANALYTICAL A1 Define and interpret the accounting

equation and each of its components.

A2 Compute and interpret return on assets.

A3 Appendix 1A—Explain the relation between return and risk.

Chapter Preview

1 Accounting in Business

FINANCIAL STATEMENTS

P2 Income statement Statement of retained earnings

Balance sheet

Statement of cash flows

A2 Financial analysis

NTK 1-5

TRANSACTION ANALYSIS

A1 Accounting equation and its components

Expanded accounting equation

P1 Transaction analysis— Illustrated

NTK 1-3, 1-4

ETHICS AND ACCOUNTING

C3 Ethics C4 Generally accepted

accounting principles

Conceptual framework

NTK 1-1

ACCOUNTING USES

C1 Purpose of accounting

C2 Accounting information users

Opportunities in accounting

Learning Objectives are classified as conceptual, analytical, or procedural

Chapter Preview is organized by “blocks” of key content and learning objectives followed by Need-to-Know (NTK) guided video examples

NTK 1-2

3

“We ran the business . . . with just a few hundred bucks”—Steve Wozniak

Big Apple

CUPERTINO, CA—“When I designed the Apple stuff,” says Steve Wozniak, “I never thought in my life I would have enough money to fly to Hawaii or make a down payment on a house.” But some dreams do come true. Woz, along with Steve Jobs and Ron Wayne, founded Apple (Apple.com) when Woz was 25 and Jobs was 21.

The young entrepreneurs faced challenges, including how to read and interpret accounting data. They also needed to finance the company, which they did by selling Woz’s HP calcu- lator and Jobs’s Volkswagen van. The $1,300 raised helped them purchase the equipment Woz used to build the first Apple computer.

In setting up their company, the owners chose between a part- nership and a corporation. They decided on a partnership that in- cluded Ron as a third partner with 10% ownership. Days later, Ron withdrew when he considered the unlimited liability of a partner- ship. He sold his 10% share to Woz and Jobs for $800. Within nine months, Woz and Jobs converted Apple to a corporation.

As Apple grew, Woz and Jobs had to learn more accounting, along with details of preparing and interpreting financial state- ments. Important questions involving transaction analysis and financial reporting arose, and the owners took care to do things

right. “Everything we did,” asserts Woz, “we were setting the tone for the world.”

Woz and Jobs focused their accounting system to provide information for Apple’s business decisions. Today, Woz believes that Apple is key to the language of technology, just as account- ing is the language of business. In retrospect, Woz says, “Every dream I have ever had in life has come true ten times over.”

Sources: Apple website, January 2019; Woz.org, January 2019; Apple 2016 Sustainability Report, April 2016; Greenbiz, October 2014; iWoz: From Computer Geek to Cult Icon, W.W. Norton & Co., 2006; Founders at Work, Apress, 2007

©Miguel Medina/AFP/Getty Images

Why is accounting so popular on campus? Why are there so many openings for accounting jobs? Why is accounting so important to companies? The answer is that we live in an informa- tion age in which accounting information impacts us all.

Accounting is an information and measurement system that identifies, records, and commu- nicates an organization’s business activities. Exhibit 1.1 shows these accounting functions.

IMPORTANCE OF ACCOUNTING

Our most common contact with accounting is through credit checks, checking accounts, tax forms, and payroll. These experiences focus on recordkeeping, or bookkeeping, which is the recording of transactions and events. This is just one part of accounting. Accounting also includes analysis and interpretation of information.

Decision Feature launches each chapter showing the relevance of accounting for a real entrepreneur; Entrepreneurial Decision assignment returns to this feature with a mini-case

C1 Explain the purpose and importance of accounting.

Select transactions and events Input, measure, and log Prepare, analyze, and interpret

Identifying Recording Communicating

Examples are Apple’s sale of iPhones and TicketMaster’s receipt of ticket money.

Examples are dated logs of transactions measured in dollars.

Examples are reports that we analyze and interpret.

EXHIBIT 1.1 Accounting Functions

4 Chapter 1 Accounting in Business

Technology plays a major role in accounting. Technology reduces the time, effort, and cost of recordkeeping while improving accuracy. As technology makes more information available, the demand for accounting knowledge increases. Consulting, planning, and other financial services are closely linked to accounting.

Users of Accounting Information Accounting is called the language of business because it communicates data that help people make better decisions. People using accounting information are divided into two groups: exter- nal users and internal users. Financial accounting focuses on the needs of external users, and managerial accounting focuses on the needs of internal users.

External Users External users of accounting information do not directly run the organi- zation and have limited access to its accounting information. These users get accounting infor- mation from general-purpose financial statements. Following is a partial list of external users and decisions they make with accounting information. Lenders (creditors) loan money or other resources to an organization. Banks, savings and

loans, and mortgage companies are lenders. Lenders use information to assess if an organiza- tion will repay its loans.

Shareholders (investors) are the owners of a corporation. They use accounting reports to de- cide whether to buy, hold, or sell stock.

Boards of directors oversee organizations. Directors use accounting information to evaluate the performance of executive management.

External (independent) auditors examine financial statements to verify that they are prepared according to generally accepted accounting principles.

Nonmanagerial and nonexecutive employees and labor unions use external information to bargain for better wages.

Regulators have legal authority over certain activities of organizations. For example, the Internal Revenue Service (IRS) requires accounting reports for computing taxes.

Voters and government officials use information to evaluate government performance. Contributors to nonprofits use information to evaluate the use and impact of donations. Suppliers use information to analyze a customer before extending credit. Customers use financial reports to assess the stability of potential suppliers.

Internal Users Internal users of accounting information directly manage the organiza- tion. Internal reports are designed for the unique needs of managerial or executive employees, such as the chief executive officer (CEO). Following is a partial list of internal users and deci- sions they make with accounting information. Purchasing managers need to know what, when, and how much to purchase. Human resource managers need information about employees’ payroll, benefits, and performance. Production managers use information to monitor costs and ensure quality. Distribution managers need reports for timely and accurate delivery of products and services. Marketing managers use reports to target consumers, set prices, and monitor consumer needs. Service managers use reports to provide better service to customers. Research and development managers use information on projected costs and revenues of

innovations.

Opportunities in Accounting Accounting has four areas of opportunities: financial, managerial, taxation, and accounting- related. Exhibit 1.2 lists selected opportunities in each area.

C2 Identify users and uses of, and opportunities in, accounting.

23.90 15.00 15.34 17.89 19.45 13.67 13.60 25.65 15.45 18.85 23.56 18.85 17.23

+3.58% +12.3% +5.34% +5.94% +2.13% +6.43% -11.6% +23.1% +5.56% -3.67% +11.3% +2.54% +12.3%

400.20 253.95 285.32 248.20 989.26 320.34 208.98 432.62 765.23 564.23 256.25 524.65 754.62

530.000 320.000 430.000 900.000 600.000 380.000 220.000 750.000 250.000 120.000 158.000 245.000 658.000

Annual Repor t

–4% 2009 2007

2005 2003 2001

–2%

0%

2%

4%

6%

8%

10%

12%

Circuit City Best Buy

Return on Assets:

• Managers

• Officers and directors

• Internal auditors

• Sales sta ff

• Budget officers

• Controllers

Annual Budge t

–4% 2009 2007

2005 2003 2001

–2%

0%

2%

4%

6%

8%

10%

12%

Circuit City Best Buy

Return on Assets:

• Managers

• Officers and directors

• Internal auditors

• Sales sta ff

• Budget officers

• Controllers

Point: Technology is only as useful as the accounting data available, and users’ decisions are only as good as their understanding of accounting.

Chapter 1 Accounting in Business 5

• Preparation • Analysis • External auditing • Regulatory • Consulting • Planning • Criminal investigation

• Preparation • Planning • Regulatory • Investigations • Consulting • Enforcement • Legal services • Estate plans

• General accounting • Cost accounting • Budgeting • Internal auditing • Consulting • Controller • Treasurer • Strategy

• Lenders • Consultants • Analysts • Traders • Directors • Underwriters • Planners • Appraisers

• FBI investigators • Market researchers • Systems designers • Merger services • Business valuation • Forensic accounting • Litigation support • Entrepreneurs

Opportunities in Accounting

Financial Taxation Accounting-relatedManagerial

EXHIBIT 1.2 Accounting Opportunities

Exhibit 1.3 shows that the majority of opportunities are in private accounting, which are employees working for businesses. Public accounting involves accounting services such as auditing and taxation. Opportunities also exist in government and not-for-profit agen- cies, including business regulation and law enforcement.

Accounting specialists are highly regarded, and their professional standing is often denoted by a certificate. Certified public accountants (CPAs) must meet education and experience requirements, pass an exam, and be ethical. Many accounting specialists hold certificates in addition to or instead of the CPA. Two of the most common are the certificate in management accounting (CMA) and the certified internal auditor (CIA). Employers also look for specialists with designations such as certified bookkeeper (CB), certified payroll professional (CPP), certified fraud examiner (CFE), and certified foren- sic accountant (CrFA).

Accounting specialists are in demand. Exhibit 1.4 reports average annual salaries for several accounting positions. Salaries vary based on location, company size, and other factors.

Public accounting

24%

Government and

not-for-profit 22%

Private accounting

54%

EXHIBIT 1.3 Accounting Jobs by Area

Point: The largest accounting firms are EY, KPMG, PwC, and Deloitte.

Point: Higher education yields higher pay: Master’s degree $73,738 Bachelor’s degree 56,665 Associate’s degree 39,771 High school degree 30,627 No high school degree 20,241

EXHIBIT 1.4 Accounting Salaries

Public Accounting Salary

Partner . . . . . . . . . . . . . . . . . . . . . . $245,000

Manager (6–8 years) . . . . . . . . . . 112,000

Senior (3–5 years) . . . . . . . . . . . . 90,000

Junior (0–2 years) . . . . . . . . . . . . 62,500

Private Accounting Salary

CFO . . . . . . . . . . . . . . . . . . . . . . . . $290,000

Controller/Treasurer . . . . . . . . . . . 180,000

Manager (6–8 years) . . . . . . . . . . 98,500

Senior (3–5 years) . . . . . . . . . . . . 81,500

Junior (0–2 years) . . . . . . . . . . . . . 58,000

Recordkeeping Salary

Full-charge bookkeeper . . . . . . . . $60,500

Accounts manager . . . . . . . . . . . . 58,000

Payroll manager . . . . . . . . . . . . . . 59,500

Accounting clerk (0–2 years) . . . . 39,500

Identify the following users of accounting information as either an (a) external or (b) internal user.

C1 C2 Accounting Users

NEED-TO-KNOW 1-1 1. Regulator 2. CEO 3. Shareholder

4. Marketing manager 5. Executive employee 6. External auditor

7. Production manager 8. Nonexecutive employee 9. Bank lender

Solution

1. a 2. b 3. a 4. b 5. b 6. a 7. b 8. a 9. a. Do More: QS 1-1, QS 1-2, E 1-1,

E 1-2, E 1-3

NEED-TO-KNOWs highlight key procedures and concepts in learning accounting; instructional audio/video recordings accompany each one

6 Chapter 1 Accounting in Business

Ethics—A Key Concept For information to be useful, it must be trusted. This demands ethics in accounting. Ethics are beliefs that separate right from wrong. They are accepted standards of good and bad behavior.

Accountants face ethical choices as they prepare financial reports. These choices can affect the salaries and bonuses paid to workers. They even can affect the success of products and ser- vices. Misleading information can lead to a bad decision that harms workers and the business. There is an old saying: Good ethics are good business. Exhibit 1.5 gives a three-step process for making ethical decisions.

C3 Explain why ethics are crucial to accounting.

FUNDAMENTALS OF ACCOUNTING

Use ethics to recognize an ethical concern.

Consider all consequences.

Choose best option after weighing all consequences.

1. Identify ethical concerns 2. Analyze options 3. Make ethical decisionEXHIBIT 1.5 Ethical Decision Making

Point: A Code of Conduct is available at AICPA.org.

Fraud Triangle: Ethics under Attack The fraud triangle shows that three factors push a person to commit fraud. Opportunity. A person must be able to commit fraud with a low risk of getting caught. Pressure, or incentive. A person must feel pressure or have incentive to commit fraud. Rationalization, or attitude. A person justifies fraud or does not see its criminal nature.

The key to stopping fraud is to focus on prevention. It is less expensive and more effective to prevent fraud from happening than it is to detect it.

To help prevent fraud, companies set up internal controls. Internal controls are procedures to protect assets, ensure reliable accounting, promote efficiency, and uphold company policies. Examples are good records, physical controls (locks), and independent reviews.

Enforcing Ethics In response to major accounting scandals, like those at Enron and WorldCom, Congress passed the Sarbanes-Oxley Act, also called SOX, to help stop financial abuses. SOX requires documentation and verification of internal controls and emphasizes effec- tive internal controls. Management must issue a report stating that internal controls are effective. Auditors verify the effectiveness of internal controls. Ignoring SOX can lead to penalties and criminal prosecution of executives. CEOs and CFOs who knowingly sign off on bogus account- ing reports risk millions of dollars in fines and years in prison.

Dodd-Frank Wall Street Reform and Consumer Protection Act, or Dodd-Frank, has two important provisions. Clawback Mandates recovery (clawback) of excessive pay. Whistleblower SEC pays whistleblowers 10% to 30% of sanctions exceeding $1 million.

O pp

or tu

nit y

Rationalization

Pressure

Point: An audit examines whether financial statements are prepared using GAAP.

Point: SOX requires a business that sells stock to disclose a code of ethics for its executives.

Ethics Pay The $100 million mark in total payments made by the SEC to whistleblowers was recently surpassed. Since the SEC began awarding whistleblowers a percentage of money from sanctions, over 14,000 tips have been reported. Many of the tips come from accountants. ■

Ethical Risk

Ethical Risk boxes highlight ethical issues from practice

Chapter 1 Accounting in Business 7

Generally Accepted Accounting Principles Financial accounting is governed by concepts and rules known as generally accepted account- ing principles (GAAP). GAAP wants information to have relevance and faithful representa- tion. Relevant information affects decisions of users. Faithful representation means information accurately reflects the business results.

The Financial Accounting Standards Board (FASB) is given the task of setting GAAP from the Securities and Exchange Commission (SEC). The SEC is a U.S. government agency that oversees proper use of GAAP by companies that sell stock and debt to the public.

International Standards Our global economy demands comparability in accounting re- ports. The International Accounting Standards Board (IASB) issues International Financial Reporting Standards (IFRS) that identify preferred accounting practices. These standards are similar to, but sometimes different from, U.S. GAAP. The FASB and IASB are working to reduce differences between U.S. GAAP and IFRS.

Conceptual Framework The FASB conceptual framework in Exhibit 1.6 consists of the following. Objectives—to provide information useful to investors, creditors,

and others. Qualitative characteristics—to require information that has rele-

vance and faithful representation. Elements—to define items in financial statements. Recognition and measurement—to set criteria for an item to be

recognized as an element; and how to measure it.

Principles, Assumptions, and Constraint There are two types of accounting principles (and assumptions). General principles are the assumptions, concepts, and guidelines for preparing financial statements; these are shown in purple font in Exhibit 1.7, along with key as- sumptions in red font. Specific principles are de- tailed rules used in reporting business transactions and events; they are described as we encounter them.

Accounting Principles There are four general principles. Measurement principle (cost principle)

Accounting information is based on actual cost. Cost is measured on a cash or equal-to-cash basis. This means if cash is given for a service, its cost is measured by the cash paid. If something besides cash is exchanged (such as a car traded for a truck), cost is measured as the cash value of what is given up or received. Information based on cost is considered objective. Objectivity means that information is supported by independent, unbiased evidence. Later chapters cover adjust- ments to market and introduce fair value.

Revenue recognition principle Revenue is recognized (1) when goods or services are pro- vided to customers and (2) at the amount expected to be received from the customer. Revenue (sales) is the amount received from selling products and services. The amount received is usually in cash, but it also can be a customer’s promise to pay at a future date, called credit sales. (To recognize means to record it.)

C4 Explain generally accepted accounting principles and define and apply several accounting principles.

Point: CPAs who audit financial statements must disclose if they do not comply with GAAP.

Objectives of financial accounting

Recognition and measurement

Qualitative characteristics Elements

EXHIBIT 1.6 Conceptual Framework

GAAPGAAP

Measurement

Full disclosure

P

Revenue recognition

Expense recognition

sepExpExpen tiogrecogrec tionionoongnitr

Business entity

Time period

F lFul sclosuuurescdiscd ssd clos

Monetary unit

Going concern

Cost-benefit

Principles

Assumptions

Constraint

EXHIBIT 1.7 Building Blocks for GAAP

Point: A company pays $500 for equipment. The cost principle requires it be recorded at $500. It makes no difference if the owner thinks this equipment is worth $700.

Example: A lawn service bills a customer $800 on June 1 for two months of mowing (June and July). The customer pays the bill on July 1. When is revenue recorded? Answer: It is recorded over time as it is earned; record $400 revenue for June and $400 for July.

8 Chapter 1 Accounting in Business

Expense recognition principle (matching principle) A company records the expenses it incurred to generate the revenue reported. An example is rent costs of office space.

Full disclosure principle A company reports the details behind financial statements that would impact users’ decisions. Those disclosures are often in footnotes to the statements.

Example: Credit cards are used to pay $200 in gas for a lawn service during June and July. The cards are paid in August. When is expense recorded? Answer: If revenue is earned over time, record $100 expense in June and $100 in July.

Measurement and Recognition Revenues for the Seattle Seahawks, Atlanta Falcons, Green Bay Packers, and other professional football teams include ticket sales, television broadcasts, concessions, and advertising. Revenues from ticket sales are earned when the NFL team plays each game. Advance ticket sales are not revenues; instead, they are a liability until the NFL team plays the game for which the ticket was sold. At that point, the liability is removed and revenues are reported. ■

Decision Insight

©Shane Roper/CSM/REX/Shutterstock

Accounting Assumptions There are four accounting assumptions. Going-concern assumption Accounting information presumes that the business will con-

tinue operating instead of being closed or sold. This means, for example, that property is re- ported at cost instead of liquidation value.

Monetary unit assumption Transactions and events are expressed in monetary, or money, units. Examples of monetary units are the U.S. dollar and the Mexican peso.

Time period assumption The life of a company can be divided into time periods, such as months and years, and useful reports can be prepared for those periods.

Business entity assumption A business is accounted for separately from other business entities and its owner. Exhibit 1.8 describes four common business entities.

EXHIBIT 1.8 Attributes of Businesses

Sole Proprietorship Partnership Corporation Limited Liability Company (LLC)

Number of owners 1 owner; easy to set up . 2 or more, called partners; easy to set up .

1 or more, called stockholders; can get many investors by selling stock or shares of corporate ownership .*

1 or more, called members .

Business taxation No additional business income tax .

No additional business income tax .

Additional corporate income tax . No additional business income tax .

Owner liability Unlimited liability . Owner is per- sonally liable for proprietorship debts .

Unlimited liability . Partners are jointly liable for partnership debts .

Limited liability . Owners, called stock- holders (or shareholders), are not liable for corporate acts and debts .

Limited liability . Owners, called mem- bers, are not personally liable for LLC debts .

Legal entity Not a separate legal entity . Not a separate legal entity . A separate entity with the same rights and responsibilities as a person .

A separate entity with the same rights and responsibilities as a person .

Business life Business ends with owner death or choice .

Business ends with a partner death or choice .

Indefinite . Indefinite .

*When a corporation issues only one class of stock, it is called common stock (or capital stock).

Tax Services

Accounting Constraint The cost-benefit constraint, or cost constraint, says that infor- mation disclosed by an entity must have benefits to the user that are greater than the costs of providing it. Materiality, or the ability of information to influence decisions, is also sometimes mentioned as a constraint. Conservatism and industry practices are sometimes listed as well.

Point: Proprietorships, partner- ships, and LLCs are managed by their owners. In a corporation, the owners (shareholders) elect a board of directors who hire managers to run the business.

Chapter 1 Accounting in Business 9

Entrepreneur You and a friend develop a new design for ice skates that improves speed. You plan to form a busi- ness to manufacture and sell the skates. You and your friend want to minimize taxes, but your big concern is potential lawsuits from customers who might be injured on these skates. What form of organization do you set up? ■ Answer: You should probably form an LLC. An LLC helps protect personal property from lawsuits directed at the business. Also, an LLC is not subject to an additional business income tax. You also must examine the ethical and social aspects of starting a business where injuries are expected.

Decision Ethics Decision Ethics boxes are role-playing exercises that stress ethics in accounting

Solution

a. no b. no c. no d. no e. yes f. yes g. yes h. yes i. no j. yes k. yes l. yes

Part 1: Identify each of the following terms/phrases as either an accounting (a) principle, (b) assumption, or (c) constraint.

C3 C4 Accounting Guidance

NEED-TO-KNOW 1-2 1. Cost-benefit 2. Measurement 3. Business entity

4. Going-concern 5. Full disclosure 6. Time period

7. Expense recognition 8. Revenue recognition

Solution

1. c 2. a 3. b 4. b 5. a 6. b 7. a 8. a

Part 2: Complete the following table with either a yes or a no regarding the attributes of a partnership, corporation, and LLC.

Attribute Present Partnership Corporation LLC

Business taxed . . . . . . . . . . . a . e . i .

Limited liability . . . . . . . . . . . b . f . j .

Legal entity . . . . . . . . . . . . . . c . g . k .

Unlimited life . . . . . . . . . . . . d . h . l .

Do More: QS 1-3, QS 1-4, QS 1-5, QS 1-6, E 1-4, E 1-5,

E 1-6, E 1-7

Accounting shows two basic aspects of a company: what it owns and what it owes. Assets are resources a company owns or controls. The claims on a company’s assets—what it owes—are separated into owner (equity) and nonowner (liability) claims. Together, liabilities and equity are the source of funds to acquire assets.

Assets Assets are resources a company owns or controls. These resources are expected to yield future benefits. Examples are web servers for an online services company, musical instru- ments for a rock band, and land for a vegetable grower. Assets include cash, supplies, equip- ment, land, and accounts receivable. A receivable is an asset that promises a future inflow of resources. A company that provides a service or product on credit has an account receivable from that customer.

Liabilities Liabilities are creditors’ claims on assets. These claims are obligations to pro- vide assets, products, or services to others. A payable is a liability that promises a future out- flow of resources. Examples are wages payable to workers, accounts payable to suppliers, notes (loans) payable to banks, and taxes payable.

Equity Equity is the owner’s claim on assets and is equal to assets minus liabilities. Equity is also called net assets or residual equity.

Point: “On credit” and “on account” mean cash is paid at a future date.

BUSINESS TRANSACTIONS AND ACCOUNTING A1 Define and interpret the accounting equation and each of its components.

Point: Double taxation means that (1) the corporation income is taxed and (2) any dividends to owners are taxed as part of the owners’ personal income.

10 Chapter 1 Accounting in Business

Accounting Equation The relation of assets, liabilities, and equity is shown in the following accounting equation. The accounting equation applies to all transactions and events, to all companies and orga- nizations, and to all points in time.

Assets = Liabilities + Equity

We can break down equity to get the expanded accounting equation. Point: This equation can be rearranged. Example: Assets − Liabilities = Equity

Big Data The SEC keeps an online database called EDGAR (sec.gov/edgar) that has accounting information for thousands of companies, such as Columbia Sportswear, that issue stock to the public. The annual report filing for most publicly traded U.S. companies is known as Form 10-K, and the quarterly filing is Form 10-Q. Information ser- vices such as Finance.Yahoo.com offer online data and analysis. ■

Decision Insight

©Greg Epperson/Shutterstock

Part 1: Use the accounting equation to compute the missing financial statement amounts.

Accounting Equation

NEED-TO-KNOW 1-3

A1

Company Assets Liabilities Equity

Bose $150 $ 30 $ (a)

Vogue $ (b) $100 $300

Solution

a. $120 b. $400

Part 2: Use the expanded accounting equation to compute the missing financial statement amounts.

Company Assets Liabilities Common Stock Dividends Revenues Expenses

Tesla $200 $ 80 $100 $5 $ (a) $40

YouTube $400 $160 $220 $ (b) $ 120 $90

Solution

a. $65 b. $10 Do More: QS 1-7, QS 1-8,

E 1-8, E 1-9

We see that equity increases from owner investments, called stock issuances, and from reve- nues. It decreases from dividends and from expenses. Equity consists of four parts.

Equity

Assets = Liabilities + Contributed Capital + Retained Earnings

= Liabilities + Common Stock − Dividends + Revenues − Expenses

Common Stock

Common stock reflects inflows of cash and other net assets from stockholders in exchange for stock (stock is part of contributed capital and covered in later chapters).

Dividends Dividends are outflows of cash and other assets to stockholders that reduce equity.

Revenues

Revenues increase equity (via net income) from sales of products and services to customers; examples are sales of products, consulting services provided, facilities rented to others, and commissions from services.

Expenses

Expenses decrease equity (via net income) from costs of providing products and services to customers; examples are costs of employee time, use of supplies, advertising, utilities, and insurance fees.

+ − + −

Contributed capital

Retained earnings

Chapter 1 Accounting in Business 11

Transaction Analysis Business activities are described in terms of transactions and events. External transactions are exchanges of value between two entities, which cause changes in the accounting equation. An example is the sale of the AppleCare Protection Plan by Apple. Internal transactions are exchanges within an entity, which may or may not affect the accounting equation. An example is Target’s use of its supplies, which are reported as expenses when used. Events are happen- ings that affect the accounting equation and are reliably measured. They include business events such as changes in the market value of certain assets and liabilities and natural events such as fires that destroy assets and create losses.

This section uses the accounting equation to analyze 11 transactions and events of FastFor- ward, a start-up consulting (service) business, in its first month of operations. Remember that after each transaction and event, assets always equal liabilities plus equity.

Transaction 1: Investment by Owner On December 1, Chas Taylor forms a consult- ing business named FastForward and set up as a corporation. FastForward evaluates the performance of footwear and accessories. Taylor owns and manages the business, which will publish online re- views and consult with clubs, athletes, and others who purchase Nike and Adidas products.

Taylor invests $30,000 cash in the new company and deposits the cash in a bank account opened under the name of FastForward. After this transaction, cash (an asset) and stockholders’ equity each equals $30,000. Equity is increased by the owner’s investment (stock issuance), which is included in the column titled Common Stock. The effect of this transaction on FastForward is shown in the accounting equation as follows (we label the equity entries).

P1 Analyze business transac- tions using the accounting equation.

FASTForward

Transaction 2: Purchase Supplies for Cash FastForward uses $2,500 of its cash to buy supplies of Nike and Adidas footwear for performance testing over the next few months. This transaction is an exchange of cash, an asset, for another kind of asset, supplies. It simply changes the form of assets from cash to supplies. The decrease in cash is exactly equal to the increase in supplies. The supplies of footwear are assets because of the expected future benefits from the test results of their performance.

Assets = Liabilities + Equity

Cash = Common Stock (1) +$30,000 = +$30,000 Owner investment

Transaction 3: Purchase Equipment for Cash FastForward spends $26,000 to acquire equipment for testing footwear. Like Transaction 2, Transaction 3 is an exchange of one asset, cash, for another asset, equipment. The equipment is an asset because of its expected fu- ture benefits from testing footwear. This purchase changes the makeup of assets but does not change the asset total. The accounting equation remains in balance.

Assets = Liabilities + Equity

Cash + Supplies = Common Stock Old Bal . $30,000 = $30,000 (2) − 2,500 + $2,500 ________ ________ ________ New Bal . $27,500 + $ 2,500 = $30,000

$30,000 $30,000

⎧ ⎪ ⎪ ⎪ ⎪ ⎪ ⎨ ⎪ ⎪ ⎪ ⎪ ⎪ ⎩ ⎧ ⎪ ⎪ ⎪ ⎪ ⎪ ⎪ ⎨ ⎪ ⎪ ⎪ ⎪ ⎪ ⎪ ⎩

Assets = Liabilities + Equity

Cash + Supplies + Equipment = Common Stock Old Bal . $27,500 + $2,500 = $30,000 (3) −26,000 + $26,000 _________ _______ ___________ ________ New Bal . $ 1,500 + $2,500 + $ 26,000 = $30,000

$30,000 $30,000

⎧ ⎪ ⎪ ⎪ ⎪ ⎪ ⎪ ⎪ ⎪ ⎨ ⎪ ⎪ ⎪ ⎪ ⎪ ⎪ ⎪ ⎪ ⎩ ⎧ ⎪ ⎪ ⎪ ⎪ ⎨ ⎪ ⎪ ⎪ ⎪ ⎪ ⎩

In vo

ic e

B ill

In vo

ic e

B ill Lones

Bes t Bu

y St ock

BANK

Assets Liabilities + Equity=

Real company names are in bold magenta

12 Chapter 1 Accounting in Business

Transaction 4: Purchase Supplies on Credit Taylor decides more supplies of footwear and accessories are needed. These additional supplies cost $7,100, but FastForward has only $1,500 in cash. Taylor arranges to purchase them on credit from CalTech Supply Company. Thus, FastForward acquires supplies in exchange for a promise to pay for them later. This purchase increases assets by $7,100 in supplies, and liabilities (called accounts payable to CalTech Supply) increase by the same amount.

Example: If FastForward pays $500 cash in Transaction 4, how does this partial payment affect the liability to CalTech? Answer: The liability to CalTech is reduced to $6,600 and the cash balance is reduced to $1,000.

Assets = Liabilities + Equity

Cash + Supplies + Equipment = Accounts + Common + Revenues Payable Stock Old Bal . $1,500 + $9,600 + $26,000 = $7,100 + $30,000 (5) +4,200 + $4,200 Consulting _________ ________ __________ ________ __________ _________ New Bal . $5,700 + $9,600 + $26,000 = $7,100 + $30,000 + $ 4,200

$41,300 $41,300

⎧ ⎪ ⎪ ⎪ ⎪ ⎪ ⎪ ⎪ ⎪ ⎪ ⎪ ⎨ ⎪ ⎪ ⎪ ⎪ ⎪ ⎪ ⎪ ⎪ ⎪ ⎪ ⎩ ⎧ ⎪ ⎪ ⎪ ⎪ ⎪ ⎪ ⎪ ⎪ ⎪ ⎪ ⎪ ⎨ ⎪ ⎪ ⎪ ⎪ ⎪ ⎪ ⎪ ⎪ ⎪ ⎪ ⎪ ⎩

Transaction 5: Provide Services for Cash FastForward plans to earn revenues by selling online ad space and consulting with clients about footwear and accessories. It earns net income only if its revenues are greater than its expenses. In its first job, FastForward pro- vides consulting services and immediately collects $4,200 cash. The accounting equation re- flects this increase in cash of $4,200 and in equity of $4,200. This increase in equity is shown in the far right column under Revenues because the cash received is earned by providing consult- ing services.

Point: Revenue recognition prin- ciple requires that revenue is rec- ognized when work is performed.

Transactions 6 and 7: Payment of Expenses in Cash FastForward pays $1,000 to rent its facilities. Paying this amount allows FastForward to occupy the space for the month of December. The rental payment is shown in the following accounting equation as Transaction 6. FastForward also pays the biweekly $700 salary of the company’s only em- ployee. This is shown in the accounting equation as Transaction 7. Both Transactions 6 and 7 are December expenses for FastForward. The costs of both rent and salary are expenses, not assets, because their benefits are used in December (they have no future benefits after December). The accounting equation shows that both transactions reduce cash and equity. The far right column shows these decreases as Expenses.

Point: Expense recognition prin- ciple requires that expenses are recognized when the revenue they help generate is recorded.

Assets = Liabilities + Equity

Cash + Supplies + Equipment = Accounts + Common Stock Payable Old Bal . $1,500 + $2,500 + $26,000 = $30,000 (4) + 7,100 +$7,100 _______ _______ ________ __________ ________ New Bal . $1,500 + $9,600 + $26,000 = $ 7,100 + $30,000

$37,100 $37,100

⎧ ⎪ ⎪ ⎪ ⎪ ⎨ ⎪ ⎪ ⎪ ⎪ ⎪ ⎩⎧ ⎪ ⎪ ⎪ ⎪ ⎪ ⎪ ⎪ ⎪ ⎨ ⎪ ⎪ ⎪ ⎪ ⎪ ⎪ ⎪ ⎪ ⎩

Assets = Liabilities + Equity

Cash + Supplies + Equipment = Accounts + Common + Revenues − Expenses Payable Stock Old Bal . $5,700 + $9,600 + $26,000 = $7,100 + $30,000 + $4,200 (6) −1,000 − $1,000 Rent _________ ________ __________ _________ __________ _________ _________ Bal . 4,700 + 9,600 + 26,000 = 7,100 + 30,000 + 4,200 − 1,000 (7) − 700 − 700 Salaries _________ ________ __________ _________ __________ _________ _________ New Bal . $4,000 + $9,600 + $26,000 = $7,100 + $30,000 + $4,200 − $ 1,700

$39,600 $39,600

⎧ ⎪ ⎪ ⎪ ⎪ ⎪ ⎪ ⎪ ⎪ ⎨ ⎪ ⎪ ⎪ ⎪ ⎪ ⎪ ⎪ ⎪ ⎪ ⎪ ⎩ ⎧ ⎪ ⎪ ⎪ ⎪ ⎪ ⎪ ⎪ ⎪ ⎪ ⎪ ⎪ ⎪ ⎪ ⎪ ⎨ ⎪ ⎪ ⎪ ⎪ ⎪ ⎪ ⎪ ⎪ ⎪ ⎪ ⎪ ⎪ ⎪ ⎪ ⎩

Increases in expenses yield decreases in equity.

Chapter 1 Accounting in Business 13

Transaction 8: Provide Services and Facilities for Credit FastForward pro- vides consulting services of $1,600 and rents its test facilities for an additional $300 to Adidas on credit. Adidas is billed for the $1,900 total. This transaction creates a new asset, called ac- counts receivable, from Adidas. Accounts receivable is increased instead of cash because the payment has not yet been received. Equity is increased from the two revenue components shown in the Revenues column of the accounting equation.

Point: Transaction 8, like 5, records revenue when work is performed, not necessarily when cash is received.

Transaction 9: Receipt of Cash from Accounts Receivable The client in Transaction 8 (Adidas) pays $1,900 to FastForward 10 days after it is billed for consulting ser- vices. This Transaction 9 does not change the total amount of assets and does not affect liabili- ties or equity. It converts the receivable (an asset) to cash (another asset). It does not create new revenue. Revenue was recognized when FastForward performed the services in Transaction 8, not when the cash is collected.

Point: Transaction 9 involved no added client work, so no added revenue is recorded.

Point: Receipt of cash is not always a revenue.

Transaction 10: Payment of Accounts Payable FastForward pays CalTech Supply $900 cash as partial payment for its earlier $7,100 purchase of supplies (Transaction 4), leaving $6,200 unpaid. This transaction decreases FastForward’s cash by $900 and decreases its liability to CalTech Supply by $900. Equity does not change. This event does not create an ex- pense even though cash flows out of FastForward (instead the expense is recorded when FastForward uses these supplies).

Assets = Liabilities + Equity

Cash + Accounts + Supplies + Equipment = Accounts + Common + Revenues − Expenses Receivable Payable Stock Old Bal . $4,000 + $9,600 + $26,000 = $7,100 + $30,000 + $4,200 − $1,700 (8) + $1,900 + 1,600 Consulting + 300 Rental ________ _________ _______ _________ _______ _________ _______ __________ New Bal . $4,000 + $ 1,900 + $9,600 + $26,000 = $7,100 + $30,000 + $6,100 − $1,700

$41,500 $41,500

⎧ ⎪ ⎪ ⎪ ⎪ ⎪ ⎪ ⎪ ⎪ ⎪ ⎪ ⎪ ⎨ ⎪ ⎪ ⎪ ⎪ ⎪ ⎪ ⎪ ⎪ ⎪ ⎪ ⎪ ⎪ ⎩ ⎧ ⎪ ⎪ ⎪ ⎪ ⎪ ⎪ ⎪ ⎪ ⎪ ⎪ ⎪ ⎨ ⎪ ⎪ ⎪ ⎪ ⎪ ⎪ ⎪ ⎪ ⎪ ⎪ ⎪ ⎪ ⎪ ⎩

Assets = Liabilities + Equity

Cash + Accounts + Supplies + Equipment = Accounts + Common + Revenues − Expenses Receivable Payable Stock Old Bal . $4,000 + $1,900 + $9,600 + $26,000 = $7,100 + $30,000 + $6,100 − $1,700 (9) +1,900 − 1,900 _________ _________ _______ _________ _______ _________ _______ ________ New Bal . $5,900 + $ 0 + $9,600 + $26,000 = $7,100 + $30,000 + $6,100 − $1,700

$41,500 $41,500

⎧ ⎪ ⎪ ⎪ ⎪ ⎪ ⎪ ⎪ ⎪ ⎪ ⎪ ⎪ ⎨ ⎪ ⎪ ⎪ ⎪ ⎪ ⎪ ⎪ ⎪ ⎪ ⎪ ⎪ ⎪ ⎩ ⎧ ⎪ ⎪ ⎪ ⎪ ⎪ ⎪ ⎪ ⎪ ⎪ ⎪ ⎪ ⎨ ⎪ ⎪ ⎪ ⎪ ⎪ ⎪ ⎪ ⎪ ⎪ ⎪ ⎪ ⎪ ⎩

Assets = Liabilities + Equity

Cash + Accounts + Supplies + Equipment = Accounts + Common + Revenues − Expenses Receivable Payable Stock Old Bal . $5,900 + $ 0 + $9,600 + $26,000 = $7,100 + $30,000 + $6,100 − $1,700 (10) −900 −900 _______ __________ _______ _________ _______ _________ ________ ________ New Bal . $5,000 + $ 0 + $9,600 + $26,000 = $6,200 + $30,000 + $6,100 − $1,700

$40,600 $40,600

⎧ ⎪ ⎪ ⎪ ⎪ ⎪ ⎪ ⎪ ⎪ ⎪ ⎪ ⎪ ⎨ ⎪ ⎪ ⎪ ⎪ ⎪ ⎪ ⎪ ⎪ ⎪ ⎪ ⎪ ⎪ ⎩ ⎧ ⎪ ⎪ ⎪ ⎪ ⎪ ⎪ ⎪ ⎪ ⎪ ⎪ ⎪ ⎨ ⎪ ⎪ ⎪ ⎪ ⎪ ⎪ ⎪ ⎪ ⎪ ⎪ ⎪ ⎪ ⎩

14 Chapter 1 Accounting in Business

Summary of Transactions Exhibit 1.9 shows the effects of these 11 transactions of FastForward using the accounting equa- tion. Assets equal liabilities plus equity after each transaction.

Assets = Liabilities + Equity

Cash + Accounts + Supplies + Equipment = Accounts + Common − Dividends + Revenues − Expenses Receivable Payable Stock Old Bal . $5,000 + $ 0 + $9,600 + $26,000 = $6,200 + $30,000 + $6,100 − $1,700 (11) − 200 − $200 Dividends _______ _______ _______ ________ _______ _________ ______ _______ _______ New Bal . $4,800 + $ 0 + $9,600 + $26,000 = $6,200 + $30,000 − $200 + $6,100 − $1,700

$40,400 $40,400

⎧ ⎪ ⎪ ⎪ ⎪ ⎪ ⎪ ⎪ ⎪ ⎪ ⎪ ⎨ ⎪ ⎪ ⎪ ⎪ ⎪ ⎪ ⎪ ⎪ ⎪ ⎩ ⎧ ⎪ ⎪ ⎪ ⎪ ⎪ ⎪ ⎪ ⎪ ⎪ ⎪ ⎪ ⎪ ⎪ ⎪ ⎨ ⎪ ⎪ ⎪ ⎪ ⎪ ⎪ ⎪ ⎪ ⎪ ⎪ ⎪ ⎪ ⎪ ⎪ ⎩

Increases in dividends yield decreases in equity.

Transaction 11: Payment of Cash Dividend FastForward declares and pays a $200 cash dividend to its owner (the sole shareholder). Dividends (decreases in equity) are not reported as expenses because they do not help earn revenue. Because dividends are not expenses, they are not used in computing net income.

Assets = Liabilities + Equity

Cash + Accounts + Supplies + Equipment = Accounts + Common − Dividends + Revenues − Expenses Receivable Payable Stock (1) $30,000 = $30,000 (2) − 2,500 + $2,500 __________ ________ __________ Bal . 27,500 + 2,500 = 30,000 (3) −26,000 + $26,000 __________ ________ ____________ __________ Bal . 1,500 + 2,500 + 26,000 = 30,000 (4) + 7,100 = +$7,100 __________ ________ ____________ _________ __________ Bal . 1,500 + 9,600 + 26,000 = 7,100 + 30,000 (5) + 4,200 + $4,200 __________ ________ ____________ _________ __________ ________ Bal . 5,700 + 9,600 + 26,000 = 7,100 + 30,000 + 4,200 (6) − 1,000 − $1,000 __________ ________ ____________ _________ __________ ________ ________ Bal . 4,700 + 9,600 + 26,000 = 7,100 + 30,000 + 4,200 − 1,000 (7) − 700 − 700 __________ ________ ____________ _________ __________ ________ ________ Bal . 4,000 + 9,600 + 26,000 = 7,100 + 30,000 + 4,200 − 1,700 (8) + $1,900 + 1,600 + 300 __________ ________ ________ ____________ _________ __________ ________ ________ Bal . 4,000 + 1,900 + 9,600 + 26,000 = 7,100 + 30,000 6,100 − 1,700 (9) + 1,900 − 1,900 __________ ________ ____________ _________ __________ ________ ________ Bal . 5,900 + 0 + 9,600 + 26,000 = 7,100 + 30,000 + 6,100 − 1,700 (10) − 900 − 900 __________ ________ ________ ____________ _________ __________ ________ ________ Bal . 5,000 + 0 + 9,600 + 26,000 = 6,200 + 30,000 + 6,100 − 1,700 (11) − 200 − $200 __________ ________ ________ ____________ _________ __________ ______ ________ ________ Bal . $ 4,800 + $ 0 + $ 9,600 + $ 26,000 = $ 6,200 + $ 30,000 − $ 200 + $6,100 − $ 1,700

EXHIBIT 1.9 Summary of Transactions Using the Accounting Equation

Assume Tata Company began operations on January 1 and completed the following transactions during its first month of operations. Arrange the following asset, liability, and equity titles in a table like Exhibit 1.9: Cash; Accounts Receivable; Equipment; Accounts Payable; Common Stock; Dividends; Revenues; and Expenses.

Jan. 1 Jamsetji Tata invested $4,000 cash in Tata Company in exchange for its common stock. 5 The company purchased $2,000 of equipment on credit. 14 The company provided $540 of services for a client on credit. 21 The company paid $250 cash for an employee’s salary.

Transaction Analysis

NEED-TO-KNOW 1-4

P1

Do More: QS 1-10, QS 1-11, E 1-10, E 1-11, E 1-13

Chapter 1 Accounting in Business 15

Solution

Assets = Liabilities + Equity

Cash + Accounts + Equipment = Accounts + Common − Dividends + Revenues − Expenses Receivable Payable Stock Jan. 1 $4,000 = $4,000 Jan. 5 + $2,000 +$2,000 Bal . 4,000 + 2,000 = 2,000 + 4,000 Jan. 14 + $540 + $540 Bal . 4,000 + 540 + 2,000 = 2,000 + 4,000 + 540 Jan. 21 −250 − $250 Bal . 3,750 + 540 + 2,000 = 2,000 + 4,000 + 540 − 250

⎧ ⎪ ⎪ ⎪ ⎪ ⎪ ⎪ ⎪ ⎪ ⎪ ⎨ ⎪ ⎪ ⎪ ⎪ ⎪ ⎪ ⎪ ⎪ ⎩ ⎧ ⎪ ⎪ ⎪ ⎪ ⎪ ⎪ ⎪ ⎪ ⎪ ⎪ ⎪ ⎪ ⎪ ⎪ ⎪ ⎪ ⎪ ⎨ ⎪ ⎪ ⎪ ⎪ ⎪ ⎪ ⎪ ⎪ ⎪ ⎪ ⎪ ⎪ ⎪ ⎪ ⎪ ⎪ ⎪ ⎩

$6,290 $6,290

Financial statements are prepared in the order below using the 11 transactions of FastForward. (These statements are unadjusted—we explain this in Chapters 2 and 3.) The four financial statements and their purposes follow.

COMMUNICATING WITH USERS P2 Identify and prepare basic financial statements and explain how they interrelate.

Income Statement FastForward’s income statement for December is shown at the top of Exhibit 1.10. Information about revenues and expenses is taken from the Equity columns of Exhibit 1.9. Revenues are reported first on the income statement. They include consulting revenues of $5,800 from Trans- actions 5 and 8 and rental revenue of $300 from Transaction 8. Expenses are reported after revenues. Rent and salary expenses are from Transactions 6 and 7. Expenses are the costs to generate the revenues reported. Net income occurs when revenues exceed expenses. A net loss occurs when expenses exceed revenues. Net income (or loss) is shown at the bottom of the state- ment and is the amount reported in December. Stockholders’ investments and dividends are not part of income.

Financial Statement Purpose

Income statement Describes a company’s revenues and expenses and computes net income or loss over a period of time.

Statement of retained earnings Explains changes in retained earnings from net income (or loss) and any dividends over a period of time.

Balance sheet Describes a company’s financial position (types and amounts of assets, liabilities, and equity) at a point in time.

Statement of cash flows Identifies cash inflows (receipts) and cash outflows (payments) over a period of time.

Layout

Revenue – Expenses

Net income

Beg. retained earnings + Net income – Dividends

End. retained earnings

Assets = Liabilities + Equity

+/– Operating C.F. +/– Investing C.F. +/– Financing C.F.

Change in cash

Point: Net income is sometimes called earnings or profit.

Key terms are in bold and defined again in the glossary

16 Chapter 1 Accounting in Business

FASTFORWARD Balance Sheet

December 31, 2019

Assets Liabilities Cash . . . . . . . . . . . . . $ 4,800 Accounts payable . . . . . . . . . . . . . . . . $ 6,200 _____________ Supplies . . . . . . . . . . 9,600 Total liabilities . . . . . . . . . . . . . . . . . . 6,200

Equipment . . . . . . . . . 26,000 Equity Common stock . . . . . . . . . . . . . . . . . . 30,000

Retained earnings . . . . . . . . . . . . . . . 4,200 _____________ Total equity . . . . . . . . . . . . . . . . . . . . . 34,200 _________ _____________ Total assets . . . . . . . . . $ 40,400 Total liabilities and equity . . . . . . . . . $ 40,400 _________ _____________ _________ _____________

Point: The income statement, the statement of retained earnings, and the statement of cash flows are prepared for a period of time. The balance sheet is prepared as of a point in time.

FASTFORWARD Income Statement

For Month Ended December 31, 2019

Revenues

Consulting revenue ($4,200 + $1,600) . . . . . . . . . . . . . . . . . . . . . $ 5,800 Rental revenue . . . . . . . . . . . . . . . . . . . . . . . . . . . . . . . . . . . . . . . . . . . . . . 300 ____________ Total revenues . . . . . . . . . . . . . . . . . . . . . . . . . . . . . . . . . . . . . . . . . $ 6,100

Expenses

Rent expense . . . . . . . . . . . . . . . . . . . . . . . . . . . . . . . . . . . . . . . . . . 1,000

Salaries expense . . . . . . . . . . . . . . . . . . . . . . . . . . . . . . . . . . . . . . . 700 ____________ Total expenses . . . . . . . . . . . . . . . . . . . . . . . . . . . . . . . . . . . . . . . . 1,700 _____________ Net income . . . . . . . . . . . . . . . . . . . . . . . . . . . . . . . . . . . . . . . . . . . . . . $ 4,400 _____________ _____________

FASTFORWARD Statement of Cash Flows

For Month Ended December 31, 2019

Cash flows from operating activities

Cash received from clients ($4,200 + $1,900) . . . . . . . . . . . . . . $ 6,100 Cash paid for expenses ($2,500 + $900 + $1,000 + $700) . . . . (5,100) __________ Net cash provided by operating activities . . . . . . . . . . . . . . . . . . $ 1,000

Cash flows from investing activities

Cash paid for equipment . . . . . . . . . . . . . . . . . . . . . . . . . . . . . . . (26,000) __________ Net cash used by investing activities . . . . . . . . . . . . . . . . . . . . . . (26,000)

Cash flows from financing activities

Cash investments from shareholders . . . . . . . . . . . . . . . . . . . . . . 30,000 Cash dividends to shareholders . . . . . . . . . . . . . . . . . . . . . . . . . . (200) __________ Net cash provided by financing activities . . . . . . . . . . . . . . . . . . 29,800 ___________ Net increase in cash . . . . . . . . . . . . . . . . . . . . . . . . . . . . . . . . . . . . . $ 4,800

Cash balance, December 1, 2019 . . . . . . . . . . . . . . . . . . . . . . . . . . 0 ___________ Cash balance, December 31, 2019 . . . . . . . . . . . . . . . . . . . . . . . . . $ 4,800 ___________ ___________

Point: A single ruled line means an addition or subtraction. Final totals are double underlined. Negative amounts may or may not be in parentheses.

EXHIBIT 1.10 Financial Statements and Their Links

Point: A statement’s heading iden- tifies the company, the statement title, and the date or time period.

FASTFORWARD Statement of Retained Earnings

For Month Ended December 31, 2019

Retained earnings, December 1, 2019 . . . . . . . . . . . . . . . . . . . . . . . $ 0

Plus: Net income . . . . . . . . . . . . . . . . . . . . . . . . . . . . . . . . . . . . . . 4,400 _____________ 4,400

Less: Dividends . . . . . . . . . . . . . . . . . . . . . . . . . . . . . . . . . . . . . . . 200 _____________ Retained earnings, December 31, 2019 . . . . . . . . . . . . . . . . . . . . . $     4,200 _____________ _____________

Point: Arrow lines show how the statements are linked. 1 Net income is used to compute retained earnings. 2 Retained earnings is used to

prepare the balance sheet. 3 Cash from the balance sheet is

used to reconcile the statement of cash flows.

3

2

1

Chapter 1 Accounting in Business 17

Statement of Retained Earnings The statement of retained earnings reports how retained earnings changes over the reporting period. This statement shows beginning retained earnings, events that increase it (net income), and events that decrease it (dividends and net loss). Ending retained earnings is computed in this statement and is carried over and reported on the balance sheet. FastForward’s statement of retained earnings is the second report in Exhibit 1.10. The beginning balance is measured as of the start of business on December 1. It is zero because FastForward did not exist before then. An existing business reports a beginning balance equal to the prior period’s ending balance (such as from November 30). FastForward’s statement shows the $4,400 of net income for the period, which links the income statement to the statement of retained earnings (see line 1 ). The state- ment also reports the $200 cash dividend and FastForward’s end-of-period retained earnings balance.

Balance Sheet FastForward’s balance sheet is the third report in Exhibit 1.10. This statement shows FastForward’s financial position at the end of business day on December 31. The left side of the balance sheet lists FastForward’s assets: cash, supplies, and equipment. The upper right side of the balance sheet shows that FastForward owes $6,200 to creditors. Any other liabilities (such as a bank loan) would be listed here. The equity balance is $34,200. Line 2 shows the link between the ending balance of the statement of retained earnings and the retained earnings balance on the balance sheet. (This presentation of the balance sheet is called the account form: assets on the left and liabilities and equity on the right. Another presentation is the report form: assets on top, followed by liabilities and then equity at the bottom. Both are acceptable.) As always, the accounting equation balances: Assets of $40,400 = Liabilities of $6,200 + Equity of $34,200.

Statement of Cash Flows FastForward’s statement of cash flows is the final report in Exhibit 1.10. The first section reports cash flows from operating activities. It shows the $6,100 cash received from clients and the $5,100 cash paid for supplies, rent, and employee salaries. Outflows are in paren- theses to denote subtraction. Net cash provided by operating activities for December is $1,000. The second section reports investing activities, which involve buying and selling assets such as land and equipment that are held for long-term use (typically more than one year). The only investing activity is the $26,000 purchase of equipment. The third section shows cash flows from financing activities, which include long-term borrowing and repay- ing of cash from lenders and the cash investments from, and dividends to, stockholders. FastForward reports $30,000 from the owner’s initial investment and a $200 cash dividend. The net cash effect of all financing transactions is a $29,800 cash inflow. The final part of the statement shows an increased cash balance of $4,800. The ending balance is also $4,800 as it started with no cash—see line 3 .

Point: Payment for supplies is an operating activity because supplies are expected to be used up in short-term operations (typically less than one year).

Point: Investing activities refer to long-term asset investments by the company, not to owner investments.

Prepare the (a) income statement, (b) statement of retained earnings, and (c) balance sheet for Apple using the following condensed data from its fiscal year ended September 30, 2017 ($ in millions).

P2 Financial Statements

NEED-TO-KNOW 1-5

Accounts payable . . . . . . . . . . . . . . . . . . . . . . . . $ 49,049 Revenues . . . . . . . . . . . . . . . . . . . . . . . . . . . $229,234

Other liabilities . . . . . . . . . . . . . . . . . . . . . . . . . . 192,223 Investments and other assets . . . . . . . . . . . 303,373

Cost of sales . . . . . . . . . . . . . . . . . . . . . . . . . . . . 141,048 Land and equipment (net) . . . . . . . . . . . . . . 33,783

Cash . . . . . . . . . . . . . . . . . . . . . . . . . . . . . . . . . . . 20,289 Selling, general, and other expenses . . . . . 39,835

Common stock . . . . . . . . . . . . . . . . . . . . . . . . . . 35,867 Accounts receivable . . . . . . . . . . . . . . . . . . . 17,874

Retained earnings, Sep . 24, 2016 . . . . . . . . . . . 96,998 Net income . . . . . . . . . . . . . . . . . . . . . . . . . . 48,351

Dividends . . . . . . . . . . . . . . . . . . . . . . . . . . . . . . 47,169 Retained earnings, Sep . 30, 2017 . . . . . . . 98,180

APPLE

©Pavel1964/Shutterstock

18 Chapter 1 Accounting in Business

Solution ($ in millions)

APPLE Income Statement

For Fiscal Year Ended September 30, 2017

Revenues . . . . . . . . . . . . . . . . . . . . . . . . . . . . . . . . . . . . . . . . . . . . . . . . . . . . . . . . . $229,234 Expenses Cost of sales . . . . . . . . . . . . . . . . . . . . . . . . . . . . . . . . . . . . . . . . . . . . . . . . . . . . . $141,048 Selling, general, and other expenses . . . . . . . . . . . . . . . . . . . . . . . . . . . . . . . . . 39,835 ______________ Total expenses . . . . . . . . . . . . . . . . . . . . . . . . . . . . . . . . . . . . . . . . . . . . . . . . . . . 180,883 ______________ Net income . . . . . . . . . . . . . . . . . . . . . . . . . . . . . . . . . . . . . . . . . . . . . . . . . . . . . . . . $ 48,351 ______________ ______________

APPLE Statement of Retained Earnings

For Fiscal Year Ended September 30, 2017

Retained earnings, Sep . 24, 2016 . . . . . . . . . . . . . . . . . . . . . . . . . . . . . . . . . . . . . . $ 96,998 Plus: Net income . . . . . . . . . . . . . . . . . . . . . . . . . . . . . . . . . . . . . . . . . . . . . . . . . . 48,351 ______________ 145,349 Less: Dividends . . . . . . . . . . . . . . . . . . . . . . . . . . . . . . . . . . . . . . . . . . . . . . . . . . . 47,169 ______________ Retained earnings, Sep . 30, 2017 . . . . . . . . . . . . . . . . . . . . . . . . . . . . . . . . . . . . . . $ 98,180 ______________ ______________

APPLE Balance Sheet

September 30, 2017

Assets Liabilities Cash . . . . . . . . . . . . . . . . . . . . . . . . . . . . . . . . . $ 20,289 Accounts payable . . . . . . . . . . . . . . . . . . . $ 49,049 Accounts receivable . . . . . . . . . . . . . . . . . . . . . 17,874 Other liabilities . . . . . . . . . . . . . . . . . . . . . 192,223 ___________ Land and equipment (net) . . . . . . . . . . . . . . . . 33,783 Total liabilities . . . . . . . . . . . . . . . . . . . . . . 241,272 Investments and other assets . . . . . . . . . . . . . 303,373 Equity Common stock . . . . . . . . . . . . . . . . . . . . . 35,867 Retained earnings . . . . . . . . . . . . . . . . . . 98,180 ___________ Total equity . . . . . . . . . . . . . . . . . . . . . . . . 134,047 ___________ ___________ Total assets . . . . . . . . . . . . . . . . . . . . . . . . . . . . $375,319 Total liabilities and equity . . . . . . . . . . . . $375,319 ___________ ___________ ___________ ___________

Do More: QS 1-12, QS 1-13, QS 1-14, E 1-15, E 1-16,

E 1-17

Return on AssetsDecision Analysis

We organize financial statement analysis into four areas: (1) liquidity and efficiency, (2) solvency, (3) profitability, and (4) market prospects—Chapter 13 has a ratio listing with definitions and groupings by area. When analyzing ratios, we use a company’s prior-year ratios and competitor ratios to identify good, bad, or average performance. This chapter presents a profitability measure: return on assets. Return on assets is useful in evaluating management, analyzing and forecasting profits, and planning activities. Return on assets (ROA), also called return on investment (ROI), is defined in Exhibit 1.11.

Decision Analysis (a section at the end of each chapter) covers ratios for decision making using real company data. Instructors can skip this section and cover all ratios in Chapter 13

A2 Compute and interpret return on assets.

EXHIBIT 1.11 Return on Assets Return on assets =

Net income Average total assets

Net income is from the annual income statement, and average total assets is computed by adding the begin- ning and ending amounts for that same period and dividing by 2. Nike reports total net income of $4,240 million for the current year. At the beginning of the current year its total assets are $21,396 million, and at the end of the current year they total $23,259 million. Nike’s return on assets for the current year is:

Return on assets = $4,240 million

($21,396 million + $23,259 million)/2 = 19.0%

Chapter 1 Accounting in Business 19

Is a 19.0% return on assets good or bad for Nike? To help answer this question, we compare (benchmark) Nike’s return with its prior performance and the return of its competitor, Under Armour (see Exhibit 1.12). Nike shows a stable pattern of good returns that reflects effective use of assets. Nike has outperformed Under Armour in each of the last three years. Its management performed well based on Nike’s return on assets.

EXHIBIT 1.12 Nike and Under Armour Returns

Return on Assets Current Year 1 Year Ago 2 Years Ago

Nike . . . . . . . . . . . . . . . . . . . 19 .0% 17 .5% 16 .3% Under Armour . . . . . . . . . . 7 .9 9 .4 11 .4

Business Owner You own a winter ski resort that earns a 21% return on its assets. An opportunity to purchase a winter ski equipment manufacturer is offered to you. This manufacturer earns a 14% return on its assets. The industry return for competitors of this manufacturer is 9%. Do you purchase this manufacturer? ■ Answer: The 14% return on assets for the manufacturer exceeds the 9% industry return. This is positive for a potential purchase. Also, this purchase is an opportunity to spread your risk over two businesses. Still, you should hesitate to purchase a business whose 14% return is lower than your current 21% return. You might better direct efforts to increase investment in your resort if it can earn more than the 14% alternative.

Decision Maker

Decision Analysis ends with a role-playing scenario to show the usefulness of ratios

After several months of planning, Jasmine Worthy started a haircutting business called Expressions. The following events occurred during its first month of business.

a. Aug. 1 Worthy invested $3,000 cash and $15,000 of equipment in Expressions in exchange for its common stock.

b. 2 Expressions paid $600 cash for furniture for the shop. c. 3 Expressions paid $500 cash to rent space in a strip mall for August. d. 4 Purchased $1,200 of equipment on credit for the shop (recorded as accounts payable). e. 15 Expressions opened for business on August 5. Cash received from haircutting services in the

first week and a half of business (ended August 15) was $825. f. 16 Expressions provided $100 of haircutting services on credit. g. 17 Expressions received a $100 check for services previously rendered on credit. h. 18 Expressions paid $125 cash to an assistant for hours worked for the grand opening. i. 31 Cash received from services provided during the second half of August was $930. j. 31 Expressions paid $400 cash toward the accounts payable entered into on August 4. k. 31 Expressions paid a $900 cash dividend to Worthy (sole shareholder).

Required

1. Arrange the following asset, liability, and equity titles in a table similar to the one in Exhibit 1.9: Cash; Accounts Receivable; Furniture; Store Equipment; Accounts Payable; Common Stock; Dividends; Revenues; and Expenses. Show the effects of each transaction using the accounting equation.

2. Prepare an income statement for August. 3. Prepare a statement of retained earnings for August. 4. Prepare a balance sheet as of August 31. 5. Prepare a statement of cash flows for August. 6. Determine the return on assets ratio for August.

PLANNING THE SOLUTION Set up a table like Exhibit 1.9 with the appropriate columns for accounts. Analyze each transaction and show its effects as increases or decreases in the appropriate columns. Be

sure the accounting equation remains in balance after each transaction. Prepare the income statement, and identify revenues and expenses. List those items on the statement,

compute the difference, and label the result as net income or net loss. Use information in the Equity columns to prepare the statement of retained earnings. Use information in the last row of the transactions table to prepare the balance sheet. Prepare the statement of cash flows; include all events listed in the Cash column of the transactions

table. Classify each cash flow as operating, investing, or financing. Calculate return on assets by dividing net income by average assets.

COMPREHENSIVE

Transaction Analysis, Statement Preparation, and Return on Assets

NEED-TO-KNOW 1-6

Comprehensive Need-to-Know is a review of key chapter content; the Planning the Solution section offers strategies in solving it

20 Chapter 1 Accounting in Business

SOLUTION 1.

Assets = Liabilities + Equity

Cash + Accounts + Furniture + Store = Accounts + Common − Dividends + Revenues − Expenses Receivable Equipment Payable Stock a. $3,000 $15,000 $18,000 b. − 600 + $600 _______ ______ _________ _________ Bal . 2,400 + 600 + 15,000 = 18,000 c. − 500 − $500 _______ ______ _________ _________ ______ Bal . 1,900 + 600 + 15,000 = 18,000 − 500 d. + 1,200 +$1,200 _______ ______ _________ _________ _________ ______ Bal . 1,900 + 600 + 16,200 = 1,200 + 18,000 − 500 e. + 825 + $ 825 _______ ______ _________ _________ _________ _______ ______ Bal . 2,725 + 600 + 16,200 = 1,200 + 18,000 + 825 − 500 f. + $100 + 100 _______ ______ ______ _________ _________ _________ _______ ______ Bal . 2,725 + 100 + 600 + 16,200 = 1,200 + 18,000 + 925 − 500 g. + 100 − 100 _______ ______ ______ _________ _________ _________ _______ _______ Bal . 2,825 + 0 + 600 + 16,200 = 1,200 + 18,000 + 925 − 500 h. − 125 − 125 _______ ______ ______ _________ _________ _________ _______ _______ Bal . 2,700 + 0 + 600 + 16,200 = 1,200 + 18,000 + 925 − 625 i. + 930 + 930 _______ ______ ______ _________ _________ _________ _______ _______ Bal . 3,630 + 0 + 600 + 16,200 = 1,200 + 18,000 + 1,855 − 625 j. − 400 − 400 _______ ______ ______ _________ _________ _________ _______ _______ Bal . 3,230 + 0 + 600 + 16,200 = 800 + 18,000 + 1,855 − 625 k. − 900 − $ 900 _______ ______ ______ _________ _________ _________ ________ _______ _______ Bal . $ 2,330 + 0 + $ 600 + $ 16,200 = $ 800 + $ 18,000 − $ 900 + $1,855 − $625 _______ ______ ______ _________ _________ _________ ________ _______ _______ _______ ______ ______ _________ _________ _________ ________ _______ _______

[continued on next page]

2.

EXPRESSIONS Income Statement

For Month Ended August 31

Revenues

Haircutting services revenue . . . . . . . . . . . . . $ 1,855

Expenses

Rent expense . . . . . . . . . . . . . . . . . . . . . . . . . $ 500

Wages expense . . . . . . . . . . . . . . . . . . . . . . . 125

Total expenses . . . . . . . . . . . . . . . . . . . . . . . . 625

Net income . . . . . . . . . . . . . . . . . . . . . . . . . . . . . $ 1,230

3.

EXPRESSIONS Statement of Retained Earnings

For Month Ended August 31

Retained earnings, August 1* . . . . . . . . . . . . . $ 0

Plus: Net income . . . . . . . . . . . . . . . . . . . . . 1,230 1,230

Less: Dividends . . . . . . . . . . . . . . . . . . . . . . . 900

Retained earnings, August 31 . . . . . . . . . . . . . $ 330

* If Expressions had existed before August 1, the beginning retained earnings balance would equal the prior period’s ending balance.

Chapter 1 Accounting in Business 21

6. Return on assets = Net income

Average assets =

$1,230 ($18,000* + $19,130)∕2

= $1,230 $18,565

= 6.63%

*Uses the initial $18,000 investment as the beginning balance for the start-up period only.

4.

EXPRESSIONS Balance Sheet

August 31

Assets Liabilities Cash . . . . . . . . . . . . . . . . . . . . . $ 2,330 Accounts payable . . . . . . . . . . . . . . . . . . $ 800 Furniture . . . . . . . . . . . . . . . . . 600 Equity Store equipment . . . . . . . . . . . 16,200 Common stock . . . . . . . . . . . . . . . . . . . . . 18,000 Retained earnings . . . . . . . . . . . . . . . . . . 330 Total equity . . . . . . . . . . . . . . . . . . . . . . . . 18,330 Total assets . . . . . . . . . . . . . . . $19,130 Total liabilities and equity . . . . . . . . . . . . $19,130

5.

EXPRESSIONS Statement of Cash Flows

For Month Ended August 31

Cash flows from operating activities Cash received from customers . . . . . . . . . . . . . . . . . . . . . . . . . . . $ 1,855 Cash paid for expenditures ($500 + $125 + $400) . . . . . . . . . . (1,025) Net cash provided by operating activities . . . . . . . . . . . . . . . . . . $ 830 Cash flows from investing activities Cash paid for furniture . . . . . . . . . . . . . . . . . . . . . . . . . . . . . . . . . (600) Cash flows from financing activities Cash investments from shareholders . . . . . . . . . . . . . . . . . . . . . . 3,000 Cash dividends to shareholders . . . . . . . . . . . . . . . . . . . . . . . . . . (900) Net cash provided by financing activities . . . . . . . . . . . . . . . . . . 2,100 Net increase in cash . . . . . . . . . . . . . . . . . . . . . . . . . . . . . . . . . . . . . $2,330 Cash balance, August 1 . . . . . . . . . . . . . . . . . . . . . . . . . . . . . . . . . . 0 Cash balance, August 31 . . . . . . . . . . . . . . . . . . . . . . . . . . . . . . . . . . $2,330

APPENDIX

Return and Risk 1A This appendix covers return and risk analysis. Net income is often linked to return. Return on assets (ROA) is stated in ratio form as income divided by assets invested. For example, banks report return from a savings account in the form of an interest re- turn such as 2%. We also could invest in a company’s stock, or even start our own business. How do we decide among these options? The answer depends on our trade-off between return and risk. Risk is the uncertainty about the return we will earn. All business investments involve risk, but some in- vestments involve more risk than others. The lower the risk of an investment, the lower is our expected return. The reason that savings accounts pay such a low return is the low risk of not being repaid with interest (the government guarantees most savings accounts). If we buy a share of eBay or any other company, we might get a large return. However, we have no guarantee of any return; there is even the risk of loss. Exhibit 1A.1 shows recent returns for 10-year bonds with different risks. Bonds are written promises by organizations to repay amounts loaned with interest. U.S. Treasury bonds have a low expected return, but they also have low risk because they are backed by the U.S. govern- ment. High-risk corporate bonds have a much larger potential return but have much higher risk. The trade-off between return and risk is a normal part of business. Higher risk implies higher, but riskier, expected returns. To help us make better decisions, we use accounting information to assess both return and risk.

A3 Explain the relation between return and risk.

EXHIBIT 1A.1 Average Returns for Bonds with Different Risks

Annual Return

U.S. Treasury

Low-risk corporate

Medium-risk corporate

High-risk corporate

0% 4% 8% 12%

10.9%

8.3%

5.8%

2.5%

22 Chapter 1 Accounting in Business

APPENDIX

Business Activities1B This appendix explains how the accounting equation is linked to business activities. There are three major types of business activities: financing, investing, and operating. Each of these requires planning. Planning is defining an organization’s ideas, goals, and actions.

Financing Financing activities provide the resources organizations use to pay for assets such as land, buildings, and equipment. The two sources of financing are owner and nonowner. Owner financing refers to resources contributed by the owner along with any income the owner leaves in the organization. Nonowner (or creditor) financing refers to resources loaned by creditors (lenders).

Investing Investing activities are the acquiring and disposing of assets that an organization uses to buy and sell its products or services. Some organizations require land and factories to operate. Others need only an office. Invested amounts are referred to as assets. Creditor and owner financing hold claims on assets. Creditors’ claims are called liabilities, and the owner’s claim is called equity. This yields the accounting equation: Assets = Liabilities + Equity.

Operating Operating activities involve using resources to research, develop, purchase, produce, distribute, and market products and services. Sales and revenues are the inflow of assets from selling products and services. Costs and expenses are the outflow of assets to support operating activities. Exhibit 1B.1 summarizes business activities. Planning is part of each ac- tivity and gives them meaning and focus. Investing (assets) and financing (li- abilities and equity) are opposite each other because they always are equal. Operating activities are below to show that they are the result of investing and financing.

C5 Identify and describe the three major activities of organizations.

Point: Investing (assets) and financing (liabilities plus equity) totals are always equal.

EXHIBIT 1B.1 Activities of Organizations

Operating

Planning

P

la nn

in g

Planning

In vo

ic e

B ill

In vo

ic e

B ill Lones

Bes t Bu

y St ock

BANK

Investing BANKBANKBANKBANKBANK

Financing

ACCOUNTING USES External users: Do not directly run the organization and have limited access to its accounting information. Examples are lenders, shareholders, boards of directors, external auditors, nonexecutive employees, labor unions, regulators, voters, donors, suppliers, and customers. Internal users: Directly manage organization operations. Examples are the CEO and other executives, research and development managers, purchasing managers, production managers, and other managerial-level employees. Private accounting: Accounting employees working for businesses. Public accounting: Offering audit, tax, and accounting services to others.

ETHICS AND ACCOUNTING Fraud triangle: Factors that push a person to commit fraud. ∙ Opportunity: Must be able to commit fraud with a low risk of getting

caught. ∙ Pressure, or incentive: Must feel pressure or have incentive to commit

fraud. ∙ Rationalization, or attitude: Justifies fraud or does not see its criminal

nature.

Summary: Cheat Sheet

SYSTEM OF ACCOUNTS Assets: Resources a company owns or controls that are expected to yield future benefits. Liabilities: Creditors’ claims on assets. These are obligations to provide assets, products, or services to others. Equity: Shareholders’ claim on assets. It consists of:

Common stock reflects inflows of cash and other net assets from stockholders in exchange for stock.

Dividends are outflows of cash and other assets to stockholders that reduce equity.

Revenues increase equity (via net income) from sales of products and services to customers; examples are sales of products, consult- ing services provided, facilities rented to others, and commissions from services.

Expenses decrease equity (via net income) from costs of providing products and services to customers; examples are costs of employee time, use of supplies, advertising, utilities, and insurance fees.

Common Stock+ Dividends−

Revenues+

Expenses−

Common business entities: Sole Proprietorship Partnership

Number of owners 1 owner; easy to set up . 2 or more, called partners; easy to set up .

Business taxation No additional business income tax . No additional business income tax .

Owner liability Unlimited liability . Owner is personally liable for proprietorship debts .

Unlimited liability . Partners are jointly liable for partnership debts .

Legal entity Not a separate legal entity . Not a separate legal entity .

Business life Business ends with owner death or choice . Business ends with a partner death or choice .

Corporation Limited Liability Company (LLC)

Number of owners 1 or more, called stockholders; can get many investors by selling stock or shares of corporate ownership .

1 or more, called members .

Business taxation Additional corporate income tax . No additional business income tax .

Owner liability Limited liability . Owners, called stockholders (or share- holders), are not liable for corporate acts and debts .

Limited liability . Owners, called members, are not personally liable for LLC debts .

Legal entity A separate entity with the same rights and responsibili- ties as a person .

A separate entity with the same rights and responsibilities as a person .

Business life Indefinite . Indefinite .

Chapter 1 Accounting in Business 23

Accounting (3) Accounting equation (10) Assets (9) Audit (6) Auditors (6) Balance sheet (15) Bookkeeping (3) Business entity assumption (8) Common stock (8, 10) Conceptual framework (7) Contributed capital (10) Corporation (8) Cost-benefit constraint (8) Cost constraint (8) Cost principle (7) Dividends (10) Dodd-Frank Wall Street Reform and

Consumer Protection Act (6) Double taxation (9) Equity (9) Ethics (6) Events (11) Expanded accounting equation (10) Expense recognition principle (8)

Expenses (10) External transactions (11) External users (4) Financial accounting (4) Financial Accounting Standards

Board (FASB) (7) Full disclosure principle (8) Generally accepted accounting

principles (GAAP) (7) Going-concern assumption (8) Income statement (15) Internal controls (6) Internal transactions (11) Internal users (4) International Accounting Standards

Board (IASB) (7) International Financial Reporting

Standards (IFRS) (7) Liabilities (9) Limited liability company (LLC) (8) Managerial accounting (4) Matching principle (8) Measurement principle (7) Members (8)

Monetary unit assumption (8) Net income (15) Net loss (15) Owner investments (10) Partnership (8) Proprietorship (8) Recordkeeping (3) Retained earnings (10) Return (21) Return on assets (ROA) (18) Revenue recognition principle (7) Revenues (10) Risk (21) Sarbanes-Oxley Act (SOX) (6) Securities and Exchange

Commission (SEC) (7) Shareholders (8) Shares (8) Sole proprietorship (8) Statement of cash flows (15) Statement of retained earnings (15) Stock (8) Stockholders (8) Time period assumption (8)

Key Terms A list of key terms concludes each chapter (a complete glossary is also available)

Multiple Choice Quiz

1. A building is offered for sale at $500,000 but is currently as- sessed at $400,000. The purchaser of the building believes the building is worth $475,000, but ultimately purchases the building for $450,000. The purchaser records the building at:

a. $50,000. c. $450,000. e. $500,000. b. $400,000. d. $475,000.

Summary of transactions:

TRANSACTION ANALYSIS Accounting equation: Applies to all transactions and events, to all compa- nies and organizations, and to all points in time.

Assets = Liabilities + Equity

Assets = Liabilities + Equity

Cash + Accounts + Supplies + Equipment = Accounts + Common − Dividends + Revenues − Expenses Receivable Payable Stock (1) $30,000 = $30,000 (2) − 2,500 + $2,500 __________ ________ __________ Bal . 27,500 + 2,500 = 30,000 (3) −26,000 + $26,000 __________ ________ ____________ __________ Bal . 1,500 + 2,500 + 26,000 = 30,000 (4) + 7,100 = +$7,100 __________ ________ ____________ _________ __________ Bal . 1,500 + 9,600 + 26,000 = 7,100 + 30,000 (5) + 4,200 + $4,200 __________ ________ ____________ _________ __________ ________ Bal . 5,700 + 9,600 + 26,000 = 7,100 + 30,000 + 4,200 (6) − 1,000 − $1,000 __________ ________ ____________ _________ __________ ________ ________ Bal . 4,700 + 9,600 + 26,000 = 7,100 + 30,000 + 4,200 − 1,000 (7) − 700 − 700 __________ ________ ____________ _________ __________ ________ ________ Bal . 4,000 + 9,600 + 26,000 = 7,100 + 30,000 + 4,200 − 1,700 (8) + $1,900 + 1,600 + 300 __________ ________ ________ ____________ _________ __________ ________ ________ Bal . 4,000 + 1,900 + 9,600 + 26,000 = 7,100 + 30,000 6,100 − 1,700 (9) + 1,900 − 1,900 __________ ________ ________ ____________ _________ __________ ________ ________ Bal . 5,900 + 0 + 9,600 + 26,000 = 7,100 + 30,000 + 6,100 − 1,700 (10) − 900 − 900 __________ ________ ________ ____________ _________ __________ ________ ________ Bal . 5,000 + 0 + 9,600 + 26,000 = 6,200 + 30,000 + 6,100 − 1,700 (11) − 200 − $200 __________ ________ ________ ____________ _________ __________ ______ ________ ________ Bal . $ 4,800 + $ 0 + $ 9,600 + $ 26,000 = $ 6,200 + $ 30,000 − $ 200 + $6,100 − $ 1,700

Transaction 1: Investment by owner Transaction 2: Purchase supplies for cash Transaction 3: Purchase equipment for cash Transaction 4: Purchase supplies on credit Transaction 5: Provide services for cash Transactions 6 and 7: Payment of expenses in cash Transaction 8: Provide services and facilities for credit Transaction 9: Receipt of cash from accounts receivable Transaction 10: Payment of accounts payable Transaction 11: Payment of cash dividends

FINANCIAL STATEMENTS Financial Statement Layout Purpose

Income statement Describes a company’s revenues and expenses and computes net income or loss over a period of time.

Statement of retained earnings

Explains changes in retained earnings from net income (or loss) and any dividends over a period of time.

Balance sheet Describes a company’s financial position (types and amounts of assets, liabilities, and equity) at a point in time.

Statement of cash flows

Identifies cash inflows (receipts) and cash outflows (payments) over a period of time.

+/– Operating C.F. +/– Investing C.F. +/– Financing C.F.

Change in cash

Beg. retained earnings + Net income – Dividends

End. retained earnings

Assets = Liabilities + Equity

Revenue – Expenses

Net income

24 Chapter 1 Accounting in Business

2. On December 30 of the current year, KPMG signs a $150,000 contract to provide accounting services to one of its clients in the next year. KPMG has a December 31 year-end. Which accounting principle or assumption requires KPMG to record the accounting services revenue from this client in the next year and not in the current year? a. Business entity assumption b. Revenue recognition principle c. Monetary unit assumption d. Cost principle e. Going-concern assumption

3. If the assets of a company increase by $100,000 during the year and its liabilities increase by $35,000 during the same year, then the change in equity of the company during the year must have been: a. An increase of $135,000. d. An increase of $65,000. b. A decrease of $135,000. e. An increase of $100,000. c. A decrease of $65,000.

4. Brunswick borrows $50,000 cash from Third National Bank. How does this transaction affect the accounting equa- tion for Brunswick?

a. Assets increase by $50,000; liabilities increase by $50,000; no effect on equity.

b. Assets increase by $50,000; no effect on liabilities; equity increases by $50,000.

c. Assets increase by $50,000; liabilities decrease by $50,000; no effect on equity.

d. No effect on assets; liabilities increase by $50,000; eq- uity increases by $50,000.

e. No effect on assets; liabilities increase by $50,000; eq- uity decreases by $50,000.

5. Geek Squad performs services for a customer and bills the customer for $500. How would Geek Squad record this transaction? a. Accounts receivable increase by $500; revenues in-

crease by $500. b. Cash increases by $500; revenues increase by $500. c. Accounts receivable increase by $500; revenues

decrease by $500. d. Accounts receivable increase by $500; accounts payable

increase by $500. e. Accounts payable increase by $500; revenues increase

by $500. ANSWERS TO MULTIPLE CHOICE QUIZ

1. c; $450,000 is the actual cost incurred. 2. b; revenue is recorded when services are provided. 3. d;

4. a 5. a

Assets = Liabilities + Equity

+$100,000 = +$35,000 + ?

Change in equity = $100,000 − $35,000 = $65,000

A(B) Superscript letter A or B denotes assignments based on Appendix 1A or 1B.

Icon denotes assignments that involve decision making.

1. What is the purpose of accounting in society? 2. Technology is increasingly used to process accounting data.

Why then must we study and understand accounting? 3. Identify four kinds of external users and describe how

they use accounting information. 4. What are at least three questions business owners and

managers might be able to answer by looking at accounting information?

5. Identify three actual businesses that offer services and three actual businesses that offer products.

6. Describe the internal role of accounting for organizations. 7. Identify three types of services typically offered by ac-

counting professionals. 8. What type of accounting information might be useful

to the marketing managers of a business? 9. Why is accounting described as a service activity? 10. What are some accounting-related professions? 11. How do ethics rules affect auditors’ choice of clients?

12. What work do tax accounting professionals perform in ad- dition to preparing tax returns?

13. What does the concept of objectivity imply for information reported in financial statements?

14. A business reports its own office stationery on the balance sheet at its $400 cost, although it cannot be sold for more than $10 as scrap paper. Which accounting principle and/or assumption justifies this treatment?

15. Why is the revenue recognition principle needed? What does it demand?

16. Describe the four basic forms of business organization and their key attributes.

17. Define (a) assets, (b) liabilities, (c) equity, and (d) net assets.

18. What events or transactions change equity? 19. Identify the two main categories of accounting principles. 20. What do accountants mean by the term revenue? 21. Define net income and explain its computation.

Discussion Questions

Chapter 1 Accounting in Business 25

22. Identify the four basic financial statements of a business. 23. What information is reported in an income statement? 24. Give two examples of expenses a business might incur. 25. What is the purpose of the statement of retained earnings? 26. What information is reported in a balance sheet? 27. The statement of cash flows reports on what major activities? 28. Define and explain return on assets. 29.A Define return and risk. Discuss the trade-off between

them. 30.B Describe the three major business activities in organizations.

31.B Explain why investing (assets) and financing (liabilities and equity) totals are always equal.

32. Refer to Google’s financial statements in Appendix A near the end of the text. To what level of significance are dollar amounts rounded? What time period does its income statement cover?

33. Access the SEC EDGAR database (SEC.gov) and retrieve Apple’s 2017 10-K (filed November 3, 2017). Identify its auditor. What respon- sibility does its independent auditor claim regarding Apple’s financial statements?

GOOGLE

APPLE

QUICK STUDY

QS 1-1 Understanding accounting

C1

Choose the term or phrase below that best completes each statement. a. Accounting c. Recording e. Governmental g. Language of business b. Identifying d. Communicating f. Technology h. Recordkeeping (bookkeeping) 1. reduces the time, effort, and cost of recordkeeping while improving clerical accuracy. 2. requires that we input, measure, and log transactions and events. 3. is the recording of transactions and events, either manually or electronically.

Quick Study exercises offer a brief check of key points

Connect reproduces assignments online, in static or algorithmic mode, which allows instructors to monitor, promote, and assess student learning. It can be used for practice, homework, or exams

QS 1-2 Identifying accounting users

C2

Identify the following users as either external users (E) or internal users (I). a. Customers e. Managers i. Controllers b. Suppliers f. District attorney j. FBI and IRS c. External auditors g. Shareholders k. Consumer group d. Business press h. Lenders l. Directors

The fraud triangle asserts that the following three factors must exist for a person to commit fraud. A. Opportunity B. Pressure C. Rationalization Identify the fraud risk factor (A, B, or C) in each of the following situations.

1. The business has no cameras or security devices at its warehouse. 2. Managers are expected to grow business or be fired. 3. A worker sees other employees regularly take inventory for personal use. 4. No one matches the cash in the register to receipts when shifts end. 5. Officers are told to show rising income or risk layoffs. 6. A worker feels that fellow employees are not honest.

QS 1-3 Identifying ethical risks

C3

This icon highlights ethics-related assignments

QS 1-4 Identifying principles, assumptions, and constraints C4

Identify each of the following terms or phrases as an accounting (a) principle, (b) assumption, or (c) constraint.

1. Full disclosure 3. Going-concern 2. Time period 4. Revenue recognition

QS 1-5 Identifying attributes of businesses

C4

Complete the following table with either a yes or no regarding the attributes of a proprietorship, partner- ship, corporation, and limited liability company (LLC).

Attribute Present Proprietorship Partnership Corporation LLC

1 . Business taxed . . . . . . . . . . . . . . . . . .

2 . Limited liability . . . . . . . . . . . . . . . . . .

3 . Legal entity . . . . . . . . . . . . . . . . . . . . .

26 Chapter 1 Accounting in Business

QS 1-6 Identifying accounting principles and assumptions

C4

Identify the letter for the principle or assumption from A through F in the blank space next to each num- bered situation that it best explains or justifies. A. General accounting principle B. Measurement (cost) principle C. Business entity assumption

D. Revenue recognition principle E. Expense recognition (matching) principle F. Going-concern assumption

1. In December of this year, Chavez Landscaping received a customer’s order and cash prepay- ment to install sod at a house that would not be ready for installation until March of next year. Chavez should record the revenue from the customer order in March of next year, not in December of this year.

2. If $51,000 cash is paid to buy land, the land is reported on the buyer’s balance sheet at $51,000. 3. Mike Derr owns both Sailing Passions and Dockside Digs. In preparing financial statements

for Dockside Digs, Mike makes sure that the expense transactions of Sailing Passions are kept separate from Dockside Digs’s transactions and financial statements.

QS 1-7 Applying the accounting equation A1

a. Total assets of Charter Company equal $700,000 and its equity is $420,000. What is the amount of its liabilities?

b. Total assets of Martin Marine equal $500,000 and its liabilities and equity amounts are equal to each other. What is the amount of its liabilities? What is the amount of its equity?

This icon highlights assignments that enhance decision-making skills

QS 1-8 Applying the accounting equation

A1

1. Use the accounting equation to compute the missing financial statement amounts (a), (b), and (c).

A B DC

Company Assets1 2 3 4

$ 75,000

85,000 (b)

$ (a)

20,000 25,000

1

3 2

$ 40,000

(c) 70,000

Liabilities= + Equity

A D EB F GC

1 2 3 4

1 2 $ 80,000

$ 40,000 $ 32,000 $ 16,000

$ 44,000 $ 20,000

$ 18,000 $ 8,000

(b) $ 0

$ 24,000 (a)

Company Assets ExpensesRevenuesLiabilities Common

Stock Dividends

2. Use the expanded accounting equation to compute the missing financial statement amounts (a) and (b).

QS 1-9 Identifying and computing assets, liabilities, and equity

A1

Use Google’s December 31, 2017, financial statements, in Appendix A near the end of the text, to answer the following. a. Identify the amounts (in $ millions) of its 2017 (1) assets, (2) liabilities, and (3) equity. b. Using amounts from part a, verify that Assets = Liabilities + Equity.GOOGLE

QS 1-10 Identifying effects of transactions using accounting equation— Revenues and Expenses

P1

Create the following table similar to the one in Exhibit 1.9.

Assets = Liabilities + Equity

Cash + Accounts = Accounts + Common − Dividends + Revenues − Expenses Receivable Payable Stock

Then use additions and subtractions to show the dollar effects of each transaction on individual items of the accounting equation (identify each revenue and expense type, such as commissions revenue or rent expense). a. The company completed consulting work for a client and immediately collected $5,500 cash earned. b. The company completed commission work for a client and sent a bill for $4,000 to be received within

30 days. c. The company paid an assistant $1,400 cash as wages for the period. d. The company collected $1,000 cash as a partial payment for the amount owed by the client in transaction b. e. The company paid $700 cash for this period’s cleaning services.

Chapter 1 Accounting in Business 27

QS 1-11 Identifying effects of transactions using accounting equation— Assets and Liabilities

P1

Create the following table similar to the one in Exhibit 1.9.

Assets = Liabilities + Equity

Cash + Supplies + Equipment + Land = Accounts + Common − Dividends + Revenues − Expenses Payable Stock

Then use additions and subtractions to show the dollar effects of each transaction on individual items of the accounting equation. a. The owner invested $15,000 cash in the company in exchange for its common stock. b. The company purchased supplies for $500 cash. c. The owner invested $10,000 of equipment in the company in exchange for more common stock. d. The company purchased $200 of additional supplies on credit. e. The company purchased land for $9,000 cash.

QS 1-12 Identifying items with financial statements

P2

Indicate in which financial statement each item would most likely appear: income statement (I), balance sheet (B), or statement of cash flows (CF).

a. Assets b. Cash from operating activities c. Equipment d. Expenses

e. Liabilities f. Net decrease (or increase) in cash g. Revenues h. Total liabilities and equity

QS 1-13 Identifying income and equity accounts

P2

Classify each of the following items as revenues (R), expenses (EX), or dividends (D). 1. Cost of sales 2. Service revenue 3. Wages expense

4. Cash dividends 5. Rent expense 6. Rental revenue

7. Insurance expense 8. Consulting revenue

QS 1-14 Identifying assets, liabilities, and equity P2

Classify each of the following items as assets (A), liabilities (L), or equity (EQ). 1. Land 2. Common stock

3. Equipment 4. Accounts payable

5. Accounts receivable 6. Supplies

QS 1-15 Preparing an income statement

P2

On December 31, Hawkin’s records show the following accounts. Use this information to prepare a December income statement for Hawkin.

Equipment . . . . . . . . . . . $3,000 Accounts receivable . . . . . . . . . $ 600 Wages expense . . . . . . . . . . . $8,000

Cash . . . . . . . . . . . . . . . . 2,400 Services revenue . . . . . . . . . . . 16,000 Utilities expense . . . . . . . . . . 700

Rent expense . . . . . . . . . 1,500 Accounts payable . . . . . . . . . . . 6,000

QS 1-16 Computing and interpreting return on assets

A2

In a recent year’s financial statements, Home Depot reported the following results. Compute and interpret Home Depot’s return on assets (assume competitors average an 11.0% return on assets).

Sales . . . . . . . . . . . . . $95 billion Net income . . . . . . . . . . . . $8 billion Average total assets . . . . . . . $42 billion

QS 1-17 Identifying and computing assets, liabilities, and equity

A1

Use Samsung’s December 31, 2017, financial statements in Appendix A near the end of the text to an- swer the following. a. Identify the amounts (in millions of Korean won) of Samsung’s 2017 (1) assets, (2) liabilities, and

(3) equity. b. Using amounts from part a, verify that Assets = Liabilities + Equity.

Samsung

28 Chapter 1 Accounting in Business

EXERCISES

Exercise 1-1 Classifying activities reflected in the accounting system C1

Classify the following activities as part of the identifying (I), recording (R), or communicating (C) aspects of accounting.

1. Analyzing and interpreting reports. 2. Presenting financial information. 3. Keeping a log of service costs. 4. Measuring the costs of a product.

5. Preparing financial statements. 6. Acquiring knowledge of revenue transactions. 7. Observing transactions and events. 8. Registering cash sales of products sold.

Exercise 1-3 Describing accounting responsibilities

C2

Many accounting professionals work in one of the following three areas. A. Financial accounting B. Managerial accounting C. Tax accounting Identify the area of accounting that is most involved in each of the following responsibilities.

1. Internal auditing 2. External auditing 3. Cost accounting 4. Budgeting

5. Enforcing tax laws 6. Planning transactions to minimize taxes 7. Preparing external financial statements 8. Analyzing external financial reports

Exercise 1-4 Learning the language of business

C1 C2 C3

Match each of the numbered descriptions 1 through 5 with the term or phrase it best reflects. Indicate your answer by writing the letter A through H for the term or phrase in the blank provided. A. Audit B. GAAP

C. Ethics D. FASB

1. An assessment of whether financial statements follow GAAP. 2. Amount a business earns in excess of all expenses and costs associated with its sales and

revenues. 3. A group that sets accounting principles in the United States. 4. Accounting professionals who provide services to many clients. 5. Principles that determine whether an action is right or wrong.

G. Net income H. IASB

E. SEC F. Public accountants

Part A. Identify the following questions as most likely to be asked by an internal (I) or an external (E) user of accounting information.

1. Which inventory items are out of stock? 2. Should we make a five-year loan to that business? 3. What are the costs of our product’s ingredients? 4. Should we buy, hold, or sell a company’s stock? 5. Should we spend additional money for redesign of our product? 6. Which firm reports the highest sales and income? 7. What are the costs of our service to customers?

Part B. Identify the following users as either an internal (I) or an external (E) user. 1. Research and development executive 2. Human resources executive 3. Politician 4. Shareholder

5. Distribution manager 6. Creditor 7. Production supervisor 8. Purchasing manager

Exercise 1-2 Identifying accounting users and uses

C2

Match each of the numbered descriptions 1 through 7 with the term or phrase it best reflects. Indicate your answer by writing the letter A through G for the term or phrase in the blank provided. A. Ethics B. Fraud triangle C. Prevention

D. Internal controls E. Sarbanes-Oxley Act

1. Requires the SEC to pay whistleblowers. 2. Examines whether financial statements are prepared using GAAP; it does not ensure absolute

accuracy of the statements.

F. Audit G. Dodd-Frank Act

Exercise 1-5 Identifying ethical terminology

C3

Most Exercises and Quick Study assignments are supported with Guided Examples (“Hints”) in Connect using different numbers; an instructor can choose whether to make them available to students

Chapter 1 Accounting in Business 29

3. Requires documentation and verification of internal controls and increases emphasis on inter- nal control effectiveness.

4. Procedures set up to protect company property and equipment, ensure reliable accounting, promote efficiency, and encourage adherence to policies.

5. A less expensive and more effective means to stop fraud. 6. Three factors push a person to commit fraud: opportunity, pressure, and rationalization. 7. Beliefs that distinguish right from wrong.

Exercise 1-6 Distinguishing business organizations

C4

The following describe several different business organizations. Determine whether each description best refers to a sole proprietorship (SP), partnership (P), corporation (C), or limited liability company (LLC).

a. Micah and Nancy own Financial Services, which pays a business income tax. Micah and Nancy do not have personal responsibility for the debts of Financial Services.

b. Riley and Kay own Speedy Packages, a courier service. Both are personally liable for the debts of the business.

c. IBC Services does not have separate legal existence apart from the one person who owns it. d. Trent Company is owned by Trent Malone, who is personally liable for the company’s debts. e. Ownership of Zander Company is divided into 1,000 shares of stock. The company pays a

business income tax. f. Physio Products does not pay income taxes and has one owner. The owner has unlimited lia-

bility for business debt. g. AJ Company pays a business income tax and has two owners. h. Jeffy Auto is a separate legal entity from its owner, but it does not pay a business income tax.

Exercise 1-7 Identifying accounting principles and assumptions

C4

Enter the letter A through H for the principle or assumption in the blank space next to each numbered description that it best reflects. A. General accounting principle B. Measurement (cost) principle C. Business entity assumption D. Revenue recognition principle

E. Specific accounting principle F. Expense recognition (matching) principle G. Going-concern assumption H. Full disclosure principle

1. A company reports details behind financial statements that would impact users’ decisions. 2. Financial statements reflect the assumption that the business continues operating. 3. A company records the expenses incurred to generate the revenues reported. 4. Concepts, assumptions, and guidelines for preparing financial statements. 5. Each business is accounted for separately from its owner or owners. 6. Revenue is recorded when products and services are delivered. 7. Detailed rules used in reporting events and transactions. 8. Information is based on actual costs incurred in transactions.

Exercise 1-8 Using the accounting equation

A1

Determine the missing amount from each of the separate situations a, b, and c below.

A B C Assets1

2 3 4

(a)$

154,000 100,000

(c)

$ 20,000 34,000 (b)

$ 45,000

40,000

Liabilities= + Equity

Exercise 1-9 Using the accounting equation

A1

Answer the following questions. Hint: Use the accounting equation. a. At the beginning of the year, Addison Company’s assets are $300,000 and its equity is $100,000. During

the year, assets increase $80,000 and liabilities increase $50,000. What is the equity at year-end? b. Office Store Co. has assets equal to $123,000 and liabilities equal to $47,000 at year-end. What is the

equity for Office Store Co. at year-end? c. At the beginning of the year, Quaker Company’s liabilities equal $70,000. During the year, assets in-

crease by $60,000, and at year-end assets equal $190,000. Liabilities decrease $5,000 during the year. What are the beginning and ending amounts of equity?

Check (c) Beg. equity, $60,000

30 Chapter 1 Accounting in Business

Zen began a new consulting firm on January 5. Following is a financial summary, including balances, for each of the company’s first five transactions (using the accounting equation form).

Assets = Liabilities + Equity

Transaction Cash + Accounts + Office + Office = Accounts + Common + Revenues Receivable Supplies Furniture Payable Stock

1. $40,000 + $       0 + $        0 + $        0 = $      0 + $40,000 + $      0 2. 38,000 + 0 + 3,000 + 0 = 1,000 + 40,000 + 0 3. 30,000 + 0 + 3,000 + 8,000 = 1,000 + 40,000 + 0 4. 30,000 + 6,000 + 3,000 + 8,000 = 1,000 + 40,000 + 6,000 5. 31,000 + 6,000 + 3,000 + 8,000 = 1,000 + 40,000 + 7,000

Identify the explanation from a through j below that best describes each transaction 1 through 5 above and enter it in the blank space in front of each numbered transaction. a. The company purchased office furniture for $8,000 cash. b. The company received $40,000 cash from a bank loan. c. The owner invested $1,000 cash in the business in exchange for its common stock. d. The owner invested $40,000 cash in the business in exchange for its common stock. e. The company purchased office supplies for $3,000 by paying $2,000 cash and putting $1,000 on credit. f. The company billed a customer $6,000 for services provided. g. The company purchased office furniture worth $8,000 on credit. h. The company provided services for $1,000 cash. i. The company sold office supplies for $3,000 and received $2,000 cash and $1,000 on credit. j. The company provided services for $6,000 cash.

Identify the explanation from a through j below that best describes each transaction 1 through 5 and enter it in the blank space in front of each numbered transaction. a. The company purchased $1,000 of office supplies on credit. b. The company collected $1,900 cash from an account receivable. c. The company sold land for $4,000 cash. d. The company paid $1,000 cash in dividends to shareholders. e. The company purchased office supplies for $1,000 cash. f. The company purchased land for $4,000 cash. g. The company billed a client $1,900 for services provided. h. The company paid $1,000 cash toward an account payable. i. The owner invested $1,900 cash in the business in exchange for its common stock. j. The company sold office supplies for $1,900 on credit.

The following table shows the effects of transactions 1 through 5 on the assets, liabilities, and equity of Mulan’s Boutique.

Assets = Liabilities + Equity

Cash + Accounts + Office + Land = Accounts + Common + Revenues Receivable Supplies Payable Stock $ 21,000 + $         0 + $3,000 +   $19,000 = $           0 + $43,000 + $         0

1. − 4,000 + 4,000 2. +    1,000 +1,000    3. +     1,900 + 1,900 4. − 1,000 − 1,000    5. + 1,900 −    1,900                                                                                 

$ 17,900 + $         0 + $4,000 +   $23,000 = $           0 + $43,000 + $1,900

Exercise 1-10 Analysis using the accounting equation

P1

Exercise 1-11 Identifying effects of transactions on the accounting equation

P1

Chapter 1 Accounting in Business 31

For each transaction a through f, identify its impact on the accounting equation (select from 1 through 5 below). a. The company pays cash toward an account payable. b. The company purchases equipment on credit. c. The owner invests cash in the business in exchange for its common stock. d. The company pays cash dividends to shareholders. e. The company purchases supplies for cash. f. The company provides services for cash.

1. Decreases an asset and decreases equity. 2. Increases an asset and increases a liability. 3. Decreases an asset and decreases a liability.

4. Increases an asset and decreases an asset. 5. Increases an asset and increases equity.

On October 1, Ebony Ernst organized Ernst Consulting; on October 3, the owner contributed $84,000 in assets in exchange for its common stock to launch the business. On October 31, the company’s records show the fol- lowing items and amounts. Use this information to prepare an October income statement for the business.

Cash . . . . . . . . . . . . . . . . . . . . . . . . . $11,360 Cash dividends . . . . . . . . . . . . . . . . . . . . . . . $ 2,000

Accounts receivable . . . . . . . . . . . . 14,000 Consulting revenue . . . . . . . . . . . . . . . . . . . . 14,000

Office supplies . . . . . . . . . . . . . . . . 3,250 Rent expense . . . . . . . . . . . . . . . . . . . . . . . . . 3,550

Land . . . . . . . . . . . . . . . . . . . . . . . . . 46,000 Salaries expense . . . . . . . . . . . . . . . . . . . . . . 7,000

Office equipment . . . . . . . . . . . . . . 18,000 Telephone expense . . . . . . . . . . . . . . . . . . . . 760

Accounts payable . . . . . . . . . . . . . . 8,500 Miscellaneous expenses . . . . . . . . . . . . . . . . 580

Common stock . . . . . . . . . . . . . . . . 84,000

Exercise 1-15 Preparing an income statement

P2

Check Net income, $2,110

Exercise 1-12 Identifying effects of transactions on the accounting equation

P1

Swiss Group reports net income of $40,000 for 2019. At the beginning of 2019, Swiss Group had $200,000 in assets. By the end of 2019, assets had grown to $300,000. What is Swiss Group’s 2019 return on assets? How would you assess its performance if competitors average an 11% return on assets?

Exercise 1-14 Analyzing return on assets

A2

Ming Chen began a professional practice on June 1 and plans to prepare financial statements at the end of each month. During June, Ming Chen (the owner) completed these transactions. a. Owner invested $60,000 cash in the company along with equipment that had a $15,000 market value

in exchange for its common stock. b. The company paid $1,500 cash for rent of office space for the month. c. The company purchased $10,000 of additional equipment on credit (payment due within 30 days). d. The company completed work for a client and immediately collected the $2,500 cash earned. e. The company completed work for a client and sent a bill for $8,000 to be received within 30 days. f. The company purchased additional equipment for $6,000 cash. g. The company paid an assistant $3,000 cash as wages for the month. h. The company collected $5,000 cash as a partial payment for the amount owed by the client in transaction e. i. The company paid $10,000 cash to settle the liability created in transaction c. j. The company paid $1,000 cash in dividends to the owner (sole shareholder).

Required

Create the following table similar to the one in Exhibit 1.9.

Then use additions and subtractions to show the dollar effects of the transactions on individual items of the accounting equation. Show new balances after each transaction.

Check Ending balances: Cash, $46,000; Expenses, $4,500

Exercise 1-13 Identifying effects of transactions using the accounting equation

P1

Assets = Liabilities + Equity

Cash + Accounts + Equipment = Accounts + Common – Dividends + Revenues – Expenses Receivable Payable Stock

Use the information in Exercise 1-15 to prepare an October statement of retained earnings for Ernst Consulting.

Exercise 1-16 Preparing a statement of retained earnings P2

32 Chapter 1 Accounting in Business

Use the information in Exercise 1-15 to prepare an October 31 balance sheet for Ernst Consulting. Hint: The solution to Exercise 1-16 can help.

Exercise 1-17 Preparing a balance sheet P2

Indicate the section (O, I, or F) where transactions 1 through 8 would appear on the statement of cash flows. O. Cash flows from operating activity F. Cash flows from financing activity I. Cash flows from investing activity

1. Cash purchase of equipment 2. Cash paid for dividends 3. Cash paid for advertising 4. Cash paid for wages

5. Cash paid on account payable to supplier 6. Cash received from clients 7. Cash paid for rent 8. Cash investment from shareholders

Exercise 1-19 Identifying sections of the statement of cash flows

P2

Selling and administrative costs . . . . . . . . . . . . . $  12,196

Cost of sales . . . . . . . . . . . . . . . . . . . . . . . . . . . . . 126,584

Revenues . . . . . . . . . . . . . . . . . . . . . . . . . . . . . . . . $151,800

Other expenses . . . . . . . . . . . . . . . . . . . . . . . . . . . 8,413

Ford Motor Company, one of the world’s largest automakers, reports the following income statement accounts for the year ended December 31 ($ in millions). Use this information to prepare Ford’s income statement for the year ended December 31.

Exercise 1-20 Preparing an income statement for a company

P2

Match each transaction a through e to one of the following activities of an organization: financing activity (F), investing activity (I), or operating activity (O).

a. An owner contributes cash to the business in exchange for its common stock. b. An organization borrows money from a bank. c. An organization advertises a new product. d. An organization sells some of its land. e. An organization purchases equipment.

Exercise 1-21B Identifying business activities

C5

Revenues . . . . . . . . . . . . . . . . . . . . . . . . . . . . . . . . €75,350

Cost of sales . . . . . . . . . . . . . . . . . . . . . . . . . . . . . 60,946

Selling and administrative costs . . . . . . . . . . . . . €6,139

Other expenses . . . . . . . . . . . . . . . . . . . . . . . . . . 4,988

BMW Group, one of Europe’s largest manufacturers, reports the following income statement accounts for the year ended December 31 (euros in millions). Use this information to prepare BMW’s income state- ment for the year ended December 31.

Exercise 1-22 Preparing an income statement for a company

P2

Use the information in Exercise 1-15 to prepare an October 31 statement of cash flows for Ernst Consulting. Assume the following additional information. a. The owner’s initial investment consists of $38,000 cash and $46,000 in land in exchange for its common

stock. b. The company’s $18,000 equipment purchase is paid in cash. c. The accounts payable balance of $8,500 consists of the $3,250 office supplies purchase and $5,250 in

employee salaries yet to be paid. d. The company’s rent, telephone, and miscellaneous expenses are paid in cash. e. No cash has been collected on the $14,000 consulting fees earned.

Exercise 1-18 Preparing a statement of cash flows

P2

Check Net increase in cash, $11,360

Answer the following questions. Hint: Use the accounting equation. a. On January 1, Lumia Company’s liabilities are $60,000 and its equity is $40,000. On January 3, Lumia

purchases and installs solar panel assets costing $10,000. For the panels, Lumia pays $4,000 cash and promises to pay the remaining $6,000 in six months. What is the total of Lumia’s assets after the solar panel purchase?

b. On March 1, ABX Company’s assets are $100,000 and its liabilities are $30,000. On March 5, ABX is fined $15,000 for failing emission standards. ABX immediately pays the fine in cash. After the fine is paid, what is the amount of equity for ABX?

c. On August 1, Lola Company’s assets are $30,000 and its liabilities are $10,000. On August 4, Lola issues a sustainability report following SASB guidelines. Investors react positively to this report. On August 5, a new investor contributes $3,000 cash and $7,000 in equipment in exchange for ownership in Lola. After the investment, what is the amount of equity for Lola?

Exercise 1-23 Using the accounting equation

A1

This icon highlights sustainability-related assignments

Chapter 1 Accounting in Business 33

Identify how each of the following separate transactions 1 through 10 affects financial statements. For increases, place a “+” and the dollar amount in the column or columns. For decreases, place a “−” and the dollar amount in the column or columns. Some cells may contain both an increase (+) and a decrease (−) along with dollar amounts. The first transaction is completed as an example.

Required

a. For the balance sheet, identify how each transaction affects total assets, total liabilities, and total eq- uity. For the income statement, identify how each transaction affects net income.

b. For the statement of cash flows, identify how each transaction affects cash flows from operating ac- tivities, cash flows from investing activities, and cash flows from financing activities.

Problem Set B, located at the end of Problem Set A, is provided for each problem to reinforce the learning process

PROBLEM SET A

Problem 1-1A Identifying effects of transactions on financial statements

A1 P1

a. b.

Income Balance Sheet Statement Statement of Cash Flows

Total Total Total Net Operating Investing Financing Transaction Assets Liab. Equity Income Activities Activities Activities

1 Owner invests $900 cash in business in exchange for stock +900 +900 +900

2 Receives $700 cash for services provided

3 Pays $500 cash for employee wages

4 Buys $100 of equipment on credit

5 Purchases $200 of supplies on credit

6 Buys equipment for $300 cash

7 Pays $200 on accounts payable

8 Provides $400 services on credit

9 Pays $50 cash in dividends

10 Collects $400 cash on accounts receivable

[continued on next page]

The following financial statement information is from five separate companies. Problem 1-2A Computing missing information using accounting knowledge

A1 P1

Company Company Company Company Company A B C D E

December 31, 2018

Assets . . . . . . . . . . . . . . . . . . . . . . . . . . . . $55,000   $34,000   $24,000   $60,000 $119,000

Liabilities . . . . . . . . . . . . . . . . . . . . . . . . . .     24,500     21,500       9,000     40,000 ? December 31, 2019

Assets . . . . . . . . . . . . . . . . . . . . . . . . . . . .     58,000     40,000 ?     85,000   113,000 Liabilities . . . . . . . . . . . . . . . . . . . . . . . . . . ?     26,500     29,000     24,000     70,000 During year 2019

Stock issuances . . . . . . . . . . . . . . . . . . . .       6,000       1,400       9,750 ?       6,500 Net income (loss) . . . . . . . . . . . . . . . . . . .       8,500 ?       8,000     14,000     20,000 Cash dividends . . . . . . . . . . . . . . . . . . . . .       3,500       2,000       5,875              0     11,000

Required

1. Answer the following questions about Company A. a. What is the amount of equity on December 31, 2018? b. What is the amount of equity on December 31, 2019? c. What is the amount of liabilities on December 31, 2019? 2. Answer the following questions about Company B. a. What is the amount of equity on December 31, 2018? b. What is the amount of equity on December 31, 2019? c. What is net income for year 2019?

Check (1b) $41,500

(2c) $1,600

34 Chapter 1 Accounting in Business

3. Compute the amount of assets for Company C on December 31, 2019. 4. Compute the amount of stock issuances for Company D during year 2019. 5. Compute the amount of liabilities for Company E on December 31, 2018.

(3) $55,875

Problem 1-4A Preparing a statement of retained earnings P2

Use the information in Problem 1-3A to prepare a year-end statement of retained earnings for Armani Company.

Problem 1-5A Preparing a balance sheet

P2

Use the information in Problem 1-3A to prepare a year-end balance sheet for Armani Company.

Problem 1-7A Analyzing transactions and preparing financial statements

P1 P2

Gabi Gram started The Gram Co., a new business that began operations on May 1. The Gram Co. com- pleted the following transactions during its first month of operations.

May 1 G. Gram invested $40,000 cash in the company in exchange for its common stock. 1 The company rented a furnished office and paid $2,200 cash for May’s rent. 3 The company purchased $1,890 of office equipment on credit. 5 The company paid $750 cash for this month’s cleaning services. 8 The company provided consulting services for a client and immediately collected $5,400

cash. 12 The company provided $2,500 of consulting services for a client on credit. 15 The company paid $750 cash for an assistant’s salary for the first half of this month. 20 The company received $2,500 cash payment for the services provided on May 12. 22 The company provided $3,200 of consulting services on credit. 25 The company received $3,200 cash payment for the services provided on May 22. 26 The company paid $1,890 cash for the office equipment purchased on May 3. 27 The company purchased $80 of office equipment on credit. 28 The company paid $750 cash for an assistant’s salary for the second half of this month. 30 The company paid $300 cash for this month’s telephone bill. 30 The company paid $280 cash for this month’s utilities. 31 The company paid $1,400 cash in dividends to the owner (sole shareholder).

Problem 1-6A Preparing a statement of cash flows

P2

Following is selected financial information of Kia Company for the year ended December 31, 2019.

Cash used by investing activities . . . . . . . . . . $(2,000)

Net increase in cash . . . . . . . . . . . . . . . . . . . . 1,200 

Cash used by financing activities . . . . . . . . . . (2,800)

Cash from operating activities . . . . . . . . . . $6,000 

Cash, December 31, 2018 . . . . . . . . . . . . . 2,300

Required

Prepare the 2019 year-end statement of cash flows for Kia Company. Check Cash balance, Dec. 31, 2019, $3,500

Problem 1-3A Preparing an income statement

P2

As of December 31, 2019, Armani Company’s financial records show the following items and amounts.

Cash . . . . . . . . . . . . . . . . . . . . . . . . . . . . . . . . . . $10,000

Accounts receivable . . . . . . . . . . . . . . . . . . . . . 9,000

Supplies . . . . . . . . . . . . . . . . . . . . . . . . . . . . . . . 7,000

Equipment . . . . . . . . . . . . . . . . . . . . . . . . . . . . . 4,000

Accounts payable . . . . . . . . . . . . . . . . . . . . . . . 11,000

Common stock . . . . . . . . . . . . . . . . . . . . . . . . . 14,000

Retained earnings, Dec . 31, 2018 . . . . . . . . . . 3,000

Retained earnings, Dec . 31, 2019 . . . . . . . . . . . . $ 5,000

Dividends . . . . . . . . . . . . . . . . . . . . . . . . . . . . . . . 13,000

Consulting revenue . . . . . . . . . . . . . . . . . . . . . . . 33,000

Rental revenue . . . . . . . . . . . . . . . . . . . . . . . . . . . 22,000

Salaries expense . . . . . . . . . . . . . . . . . . . . . . . . . . 20,000

Rent expense . . . . . . . . . . . . . . . . . . . . . . . . . . . . 12,000

Selling and administrative expenses . . . . . . . . . . 8,000

Required

Prepare the 2019 year-end income statement for Armani Company.Check Net income, $15,000

Chapter 1 Accounting in Business 35

Required

1. Create the following table similar to the one in Exhibit 1.9.

Check (1) Ending balances: Cash, $42,780; Expenses, $5,030

Assets = Liabilities + Equity

Date Cash + Accounts + Office = Accounts + Common – Dividends + Revenues – Expenses Receivable Equipment Payable Stock

Enter the effects of each transaction on the accounts of the accounting equation by recording dollar increases and decreases in the appropriate columns. Do not determine new account balances after each transaction. Determine the final total for each account and verify that the equation is in balance.

2. Prepare the income statement and the statement of retained earnings for the month of May, and the balance sheet as of May 31.

3. Prepare the statement of cash flows for the month of May.

(2) Net income, $6,070; Total assets, $44,750

Assets = Liabilities + Equity

Cash + Accounts + Office + Office + Office = Accounts + Common – Dividends + Revenues – Expenses Receivable Supplies Equipment Suite Payable Stock

Use additions and subtractions within the table to show the dollar effects of each transaction on indi- vidual items of the accounting equation. Show new balances after each transaction.

2. Determine the company’s net income. (2) Net income, $4,500

Problem 1-8A Analyzing effects of transactions

A1 P1

Lita Lopez started Biz Consulting, a new business, and completed the following transactions during its first year of operations. a. Lita Lopez invested $70,000 cash and office equipment valued at $10,000 in the company in exchange

for its common stock. b. The company purchased an office suite for $40,000 cash. c. The company purchased office equipment for $15,000 cash. d. The company purchased $1,200 of office supplies and $1,700 of office equipment on credit. e. The company paid a local newspaper $500 cash for printing an announcement of the office’s opening. f. The company completed a financial plan for a client and billed that client $2,800 for the service. g. The company designed a financial plan for another client and immediately collected a $4,000 cash fee. h. The company paid $3,275 cash in dividends to the owner (sole shareholder). i. The company received $1,800 cash as partial payment from the client described in transaction f. j. The company made a partial payment of $700 cash on the equipment purchased in transaction d. k. The company paid $1,800 cash for the office secretary’s wages for this period.

Required

1. Create the following table similar to the one in Exhibit 1.9.

Check (1) Ending balances: Cash, $14,525; Expenses, $2,300; Accounts Payable, $2,200

Sanyu Sony started a new business and completed these transactions during December.

Dec. 1 Sanyu Sony transferred $65,000 cash from a personal savings account to a checking account in the name of Sony Electric in exchange for its common stock.

2 The company rented office space and paid $1,000 cash for the December rent. 3 The company purchased $13,000 of electrical equipment by paying $4,800 cash and agreeing to

pay the $8,200 balance in 30 days. 5 The company purchased office supplies by paying $800 cash. 6 The company completed electrical work and immediately collected $1,200 cash for these services. 8 The company purchased $2,530 of office equipment on credit. 15 The company completed electrical work on credit in the amount of $5,000. 18 The company purchased $350 of office supplies on credit. 20 The company paid $2,530 cash for the office equipment purchased on December 8. 24 The company billed a client $900 for electrical work completed; the balance is due in 30 days. 28 The company received $5,000 cash for the work completed on December 15. 29 The company paid the assistant’s salary of $1,400 cash for this month. 30 The company paid $540 cash for this month’s utility bill. 31 The company paid $950 cash in dividends to the owner (sole shareholder).

Problem 1-9A Analyzing transactions and preparing financial statements

C4 P1 P2

36 Chapter 1 Accounting in Business

Required

1. Create the following table similar to the one in Exhibit 1.9.

Assets = Liabilities + Equity

Date Cash + Accounts + Office + Office + Electrical = Accounts + Common − Dividends + Revenues − Expenses Receivable Supplies Equipment Equipment Payable Stock

Check (1) Ending balances: Cash, $59,180; Accounts Payable, $8,550

Use additions and subtractions within the table to show the dollar effects of each transaction on indi- vidual items of the accounting equation. Show new balances after each transaction.

2. Prepare the income statement and the statement of retained earnings for the current month, and the balance sheet as of the end of the month.

3. Prepare the statement of cash flows for the current month.

Analysis Component

4. Assume that the owner investment transaction on December 1 was $49,000 cash instead of $65,000 and that Sony Electric obtained another $16,000 in cash by borrowing it from a bank. Compute the dollar effect of this change on the month-end amounts for (a) total assets, (b) total liabilities, and (c) total equity.

(2) Net income, $4,160; Total assets, $76,760

Kyzera manufactures, markets, and sells cellular telephones. The average total assets for Kyzera is $250,000. In its most recent year, Kyzera reported net income of $65,000 on revenues of $475,000.

Required

1. What is Kyzera’s return on assets? 2. Does return on assets seem satisfactory for Kyzera given that its competitors average a 12% return on

assets? 3. What are total expenses for Kyzera in its most recent year? 4. What is the average total amount of liabilities plus equity for Kyzera?

Problem 1-10A Determining expenses, liabilities, equity, and return on assets

A1 A2

Check (3) $410,000

(4) $250,000

Coca-Cola and PepsiCo both produce and market beverages that are direct competitors. Key financial figures for these businesses for a recent year follow.

Key Figures ($ millions) Coca-Cola PepsiCo

Sales . . . . . . . . . . . . . . . . . . . . . . . . . . . .  $46,542  $66,504

Net income . . . . . . . . . . . . . . . . . . . . . . .      8,634      6,462

Average assets . . . . . . . . . . . . . . . . . . . .    76,448    70,518

Required

1. Compute return on assets for (a) Coca-Cola and (b) PepsiCo. 2. Which company is more successful in its total amount of sales to consumers? 3. Which company is more successful in returning net income from its assets invested?

Analysis Component

4. Write a one-paragraph memorandum explaining which company you would invest your money in and why. (Limit your explanation to the information provided.)

Problem 1-11A Computing and interpreting return on assets

A2

Check (1a) 11.3%; (1b) 9.2%

All business decisions involve aspects of risk and return. Rank order the following investment activities from 1 through 4, where “1” is most risky and “4” is least risky.

a. Lowest-risk corporate bond c. Company stock in a start-up b. Medium-risk corporate bond d. U.S. government Treasury bond

Problem 1-12AA Identifying risk and return

A3

A start-up company often engages in the following transactions during its first year of operations. Classify those transactions in one of the three major categories of an organization’s business activities. F. Financing I. Investing O. Operating

1. Shareholders investing in business 2. Purchasing a building 3. Purchasing land 4. Borrowing cash from a bank

Problem 1-13AB Describing business activities

C5 5. Purchasing equipment 6. Selling and distributing products 7. Paying for advertising 8. Paying employee wages

Chapter 1 Accounting in Business 37

An organization undertakes various activities in pursuit of business success. Identify an organization’s three major business activities, and describe each activity.

Problem 1-14AB Describing business activities C5

PROBLEM SET B

Problem 1-1B Identifying effects of transactions on financial statements

A1 P1

Identify how each of the following separate transactions 1 through 10 affects financial statements. For increases, place a “+” and the dollar amount in the column or columns. For decreases, place a “−” and the dollar amount in the column or columns. Some cells may contain both an increase (+) and a decrease (−) along with dollar amounts. The first transaction is completed as an example.

Required

a. For the balance sheet, identify how each transaction affects total assets, total liabilities, and total eq- uity. For the income statement, identify how each transaction affects net income.

b. For the statement of cash flows, identify how each transaction affects cash flows from operating ac- tivities, cash flows from investing activities, and cash flows from financing activities.

a. b.

Income Balance Sheet Statement Statement of Cash Flows

Total Total Total Net Operating Investing Financing Transaction Assets Liab. Equity Income Activities Activities Activities

1 Owner invests $800 cash in business in exchange for stock +800 +800 +800

2 Purchases $100 of supplies on credit

3 Buys equipment for $400 cash

4 Provides services for $900 cash

5 Pays $400 cash for rent incurred

6 Buys $200 of equipment on credit

7 Pays $300 cash for wages incurred

8 Pays $50 cash in dividends

9 Provides $600 services on credit

10 Collects $600 cash on accounts receivable

The following financial statement information is from five separate companies. Problem 1-2B Computing missing information using accounting knowledge

A1 P1

Company Company Company Company Company V W X Y Z

December 31, 2018

Assets . . . . . . . . . . . . . . . . . . . . . . . . . . . $54,000 $ 80,000 $141,500 $92,500 $144,000

Liabilities . . . . . . . . . . . . . . . . . . . . . . . . .   25,000   60,000     68,500   51,500 ? December 31, 2019

Assets . . . . . . . . . . . . . . . . . . . . . . . . . . .   59,000 100,000   186,500 ?   170,000 Liabilities . . . . . . . . . . . . . . . . . . . . . . . . .   36,000 ?     65,800   42,000     42,000 During year 2019

Stock issuances . . . . . . . . . . . . . . . . . . .     5,000   20,000 ?   48,100     60,000 Net income (or loss) . . . . . . . . . . . . . . . ?   40,000     18,500   24,000     32,000 Cash dividends . . . . . . . . . . . . . . . . . . .     5,500     2,000              0   20,000       8,000

Required

1. Answer the following questions about Company V. a. What is the amount of equity on December 31, 2018? b. What is the amount of equity on December 31, 2019? c. What is the net income or loss for the year 2019? [continued on next page]

Check (1b) $23,000

38 Chapter 1 Accounting in Business

2. Answer the following questions about Company W. a. What is the amount of equity on December 31, 2018? b. What is the amount of equity on December 31, 2019? c. What is the amount of liabilities on December 31, 2019? 3. Compute the amount of stock issuances for Company X during 2019. 4. Compute the amount of assets for Company Y on December 31, 2019. 5. Compute the amount of liabilities for Company Z on December 31, 2018.

(2c) $22,000

(4) $135,100

As of December 31, 2019, Audi Company’s financial records show the following items and amounts.Problem 1-3B Preparing an income statement

P2 Cash . . . . . . . . . . . . . . . . . . . . . . . . . . . . . . . . . . . . . $2,000

Accounts receivable . . . . . . . . . . . . . . . . . . . . . . . . 1,800

Supplies . . . . . . . . . . . . . . . . . . . . . . . . . . . . . . . . . . 1,200

Equipment . . . . . . . . . . . . . . . . . . . . . . . . . . . . . . . . 1,000

Accounts payable . . . . . . . . . . . . . . . . . . . . . . . . . . 3,600

Common stock . . . . . . . . . . . . . . . . . . . . . . . . . . . . 1,100

Retained earnings, Dec . 31, 2018 . . . . . . . . . . . . . 900

Retained earnings, Dec . 31, 2019 . . . . . . . . . . . . . . . $1,300

Dividends . . . . . . . . . . . . . . . . . . . . . . . . . . . . . . . . . . 2,600

Consulting revenue . . . . . . . . . . . . . . . . . . . . . . . . . . 6,600

Rental revenue . . . . . . . . . . . . . . . . . . . . . . . . . . . . . . 4,400

Salaries expense . . . . . . . . . . . . . . . . . . . . . . . . . . . . . 4,000

Rent expense . . . . . . . . . . . . . . . . . . . . . . . . . . . . . . . 2,400

Selling and administrative expenses . . . . . . . . . . . . . 1,600

Required

Prepare the 2019 year-end income statement for Audi Company.Check Net income, $3,000

Use the information in Problem 1-3B to prepare a year-end statement of retained earnings for Audi Company.

Problem 1-4B Preparing a statement of retained earnings P2

Use the information in Problem 1-3B to prepare a year-end balance sheet for Audi Company.Problem 1-5B Preparing a balance sheet P2

Nina Niko launched a new business, Niko’s Maintenance Co., that began operations on June 1. The fol- lowing transactions were completed by the company during that first month.

June 1 Nina Niko invested $130,000 cash in the company in exchange for its common stock. 2 The company rented a furnished office and paid $6,000 cash for June’s rent. 4 The company purchased $2,400 of equipment on credit. 6 The company paid $1,150 cash for this month’s advertising of the opening of the business. 8 The company completed maintenance services for a customer and immediately collected $850 cash. 14 The company completed $7,500 of maintenance services for City Center on credit. 16 The company paid $800 cash for an assistant’s salary for the first half of the month. 20 The company received $7,500 cash payment for services completed for City Center on June 14. 21 The company completed $7,900 of maintenance services for Paula’s Beauty Shop on credit. 24 The company completed $675 of maintenance services for Build-It Coop on credit. 25 The company received $7,900 cash payment from Paula’s Beauty Shop for the work completed

on June 21. 26 The company made payment of $2,400 cash for equipment purchased on June 4. 28 The company paid $800 cash for an assistant’s salary for the second half of this month. 29 The company paid $4,000 cash in dividends to the owner (sole shareholder). 30 The company paid $150 cash for this month’s telephone bill. 30 The company paid $890 cash for this month’s utilities.

Problem 1-7B Analyzing transactions and preparing financial statements

P1 P2

Problem 1-6B Preparing a statement of cash flows

P2

Selected financial information of Banji Company for the year ended December 31, 2019, follows.

Cash from investing activities . . . . . . . . . . . . . . . $1,600

Net increase in cash . . . . . . . . . . . . . . . . . . . . . . . 400

Cash from financing activities . . . . . . . . . . . . . . . 1,800

Cash used by operating activities . . . . . . . . . . $(3,000)

Cash, December 31, 2018 . . . . . . . . . . . . . . . . 1,300

Required

Prepare the 2019 year-end statement of cash flows for Banji Company.

Chapter 1 Accounting in Business 39

Required

1. Create the following table similar to the one in Exhibit 1.9.

Assets = Liabilities + Equity

Date Cash + Accounts + Equipment = Accounts + Common – Dividends + Revenues – Expenses Receivable Payable Stock

Enter the effects of each transaction on the accounts of the accounting equation by recording dollar increases and decreases in the appropriate columns. Do not determine new account balances after each transaction. Determine the final total for each account and verify that the equation is in balance.

2. Prepare the income statement and the statement of retained earnings for the month of June, and the balance sheet as of June 30.

3. Prepare the statement of cash flows for the month of June.

(2) Net income, $7,135; Total assets, $133,135

Check (1) Ending balances: Cash, $130,060; Expenses, $9,790

Problem 1-9B Analyzing transactions and preparing financial statements

C4 P1 P2

Rivera Roofing Company, owned by Reyna Rivera, began operations in July and completed these transac- tions during that first month of operations. July 1 Reyna Rivera invested $80,000 cash in the company in exchange for its common stock. 2 The company rented office space and paid $700 cash for the July rent. 3 The company purchased roofing equipment for $5,000 by paying $1,000 cash and agreeing to

pay the $4,000 balance in 30 days. 6 The company purchased office supplies for $600 cash. 8 The company completed work for a customer and immediately collected $7,600 cash for the work. 10 The company purchased $2,300 of office equipment on credit. 15 The company completed work for a customer on credit in the amount of $8,200. 17 The company purchased $3,100 of office supplies on credit. 23 The company paid $2,300 cash for the office equipment purchased on July 10. 25 The company billed a customer $5,000 for work completed; the balance is due in 30 days. 28 The company received $8,200 cash for the work completed on July 15. 30 The company paid an assistant’s salary of $1,560 cash for this month. 31 The company paid $295 cash for this month’s utility bill. 31 The company paid $1,800 cash in dividends to the owner (sole shareholder).

Problem 1-8B Analyzing effects of transactions

A1 P1

Neva Nadal started a new business, Nadal Computing, and completed the following transactions during its first year of operations. a. Neva Nadal invested $90,000 cash and office equipment valued at $10,000 in the company in

exchange for its common stock. b. The company purchased an office suite for $50,000 cash. c. The company purchased office equipment for $25,000 cash. d. The company purchased $1,200 of office supplies and $1,700 of office equipment on credit. e. The company paid a local newspaper $750 cash for printing an announcement of the office’s opening. f. The company completed a financial plan for a client and billed that client $2,800 for the service. g. The company designed a financial plan for another client and immediately collected a $4,000 cash fee. h. The company paid $11,500 cash in dividends to the owner (sole shareholder). i. The company received $1,800 cash from the client described in transaction f. j. The company made a payment of $700 cash on the equipment purchased in transaction d. k. The company paid $2,500 cash for the office secretary’s wages.

Required

1. Create the following table similar to the one in Exhibit 1.9.

Check (1) Ending balances: Cash, $5,350; Expenses, $3,250; Accounts Payable, $2,200

Assets = Liabilities + Equity

Cash + Accounts + Office + Office + Office = Accounts + Common – Dividends + Revenues – Expenses Receivable Supplies Equipment Suite Payable Stock

Use additions and subtractions within the table to show the dollar effects of each transaction on indi- vidual items of the accounting equation. Show new balances after each transaction.

2. Determine the company’s net income. (2) Net income, $3,550

40 Chapter 1 Accounting in Business

Required

1. Create the following table similar to the one in Exhibit 1.9.

Assets = Liabilities + Equity

Date Cash + Accounts + Office + Office + Roofing = Accounts + Common − Dividends + Revenues − Expenses Receivable Supplies Equipment Equipment Payable Stock

Use additions and subtractions within the table to show the dollar effects of each transaction on indi- vidual items of the accounting equation. Show new balances after each transaction.

2. Prepare the income statement and the statement of retained earnings for the month of July, and the balance sheet as of July 31.

3. Prepare the statement of cash flows for the month of July.

Analysis Component

4. Assume that the $5,000 purchase of roofing equipment on July 3 was financed from an owner investment of another $5,000 cash in the business in exchange for more common stock (instead of the purchase conditions described in the transaction above). Compute the dollar effect of this change on the month-end amounts for (a) total assets, (b) total liabilities, and (c) total equity.

Check (1) Ending balances: Cash, $87,545; Accounts Payable, $7,100

(2) Net income, $18,245; Total assets, $103,545

Ski-Doo Company manufactures, markets, and sells snowmobiles and snowmobile equipment and acces- sories. The average total assets for Ski-Doo is $3,000,000. In its most recent year, Ski-Doo reported net income of $201,000 on revenues of $1,400,000.

Required

1. What is Ski-Doo Company’s return on assets? 2. Does return on assets seem satisfactory for Ski-Doo given that its competitors average a 9.5% return

on assets? 3. What are the total expenses for Ski-Doo Company in its most recent year? 4. What is the average total amount of liabilities plus equity for Ski-Doo Company?

Problem 1-10B Determining expenses, liabilities, equity, and return on assets

A1 A2

Check (3) $1,199,000

(4) $3,000,000

Problem 1-11B Computing and interpreting return on assets

A2

Check (1a) 1.6%; (1b) 4.5%

AT&T and Verizon produce and market telecommunications products and are competitors. Key financial figures for these businesses for a recent year follow.

Key Figures ($ millions) AT&T Verizon

Sales . . . . . . . . . . . . . . . . . . . . . . . . . . . . $126,723 $110,875

Net income . . . . . . . . . . . . . . . . . . . . . . .       4,184     10,198

Average assets . . . . . . . . . . . . . . . . . . . .   269,868   225,233

Required

1. Compute return on assets for (a) AT&T and (b) Verizon. 2. Which company is more successful in the total amount of sales to consumers? 3. Which company is more successful in returning net income from its assets invested?

Analysis Component

4. Write a one-paragraph memorandum explaining which company you would invest your money in and why. (Limit your explanation to the information provided.)

Problem 1-12BA Identifying risk and return

A3

All business decisions involve aspects of risk and return. Rank order the following investment activities from 1 through 4, where “1” reflects the highest expected return and “4” the lowest expected return.

a. Low-risk corporate bond c. Money stored in a fireproof vault b. Stock of a successful company d. U.S. Treasury bond

Problem 1-13BB Describing business activities

C5

A start-up company often engages in the following activities during its first year of operations. Classify each of the following activities into one of the three major activities of an organization. F. Financing I. Investing O. Operating

1. Providing client services 2. Obtaining a bank loan 3. Purchasing machinery 4. Research for its products

5. Supervising workers 6. Shareholders investing in business 7. Renting office space 8. Paying utilities expenses

Chapter 1 Accounting in Business 41

Problem 1-14BB Describing business activities C5

Identify in outline format the three major business activities of an organization. For each of these activi- ties, identify at least two specific transactions or events normally undertaken by the business’s owners or its managers.

SERIAL PROBLEM Business Solutions

C4 P1

SP 1 On October 1, 2019, Santana Rey launched a computer services company, Business Solutions, that is organized as a corporation and provides consulting services, computer system installations, and custom program development.

Required

Create a table like the one in Exhibit 1.9 using the following headings for columns: Cash; Accounts Receivable; Computer Supplies; Computer System; Office Equipment; Accounts Payable; Common Stock; Dividends; Revenues; and Expenses. Then use additions and subtractions within the table to show the dollar effects for each of the following October transactions for Business Solutions on the individual items of the accounting equation. Show new balances after each transaction.

Oct. 1 S. Rey invested $45,000 cash, a $20,000 computer system, and $8,000 of office equipment in the company in exchange for its common stock.

3 The company purchased $1,420 of computer supplies on credit from Harris Office Products. 6 The company billed Easy Leasing $4,800 for services performed in installing a new web server. 8 The company paid $1,420 cash for the computer supplies purchased from Harris Office Prod-

ucts on October 3. 10 The company hired Lyn Addie as a part-time assistant for $125 per day, as needed. 12 The company billed Easy Leasing another $1,400 for services performed. 15 The company received $4,800 cash from Easy Leasing as partial payment toward its account. 17 The company paid $805 cash to repair computer equipment damaged when moving it. 20 The company paid $1,728 cash for advertisements published in the local newspaper. 22 The company received $1,400 cash from Easy Leasing toward its account. 28 The company billed IFM Company $5,208 for services performed. 31 The company paid $875 cash for Lyn Addie’s wages for seven days of work this month. 31 The company paid $3,600 cash in dividends to the owner (sole shareholder).

Serial Problem starts here and continues throughout the text

©Alexander Image/Shutterstock

Check Ending balances: Cash, $42,772; Revenues, $11,408; Expenses, $3,408

GENERAL LEDGER PROBLEM

Accounting professionals apply many technology tools to aid them in their everyday tasks and decision making. The General Ledger tool in Connect automates several of the procedural steps in the accounting cycle so the accounting professional can focus on the impacts of each transaction on the full set of finan- cial statements. Chapter 2 is the first chapter to use this tool in helping students see the advantages of technology and, in particular, the power of the General Ledger tool in accounting practice, including financial analysis and “what-if” scenarios.

GL

COMPANY ANALYSIS A1 A2

Accounting Analysis

AA 1-1 Key financial figures for Apple’s two most recent fiscal years follow.

Accounting Analysis (AA) is a section aimed to refine company analysis, comparative analysis, and global analysis skills; Accounting Analysis assignments are available in Connect.

$ millions Current Year Prior Year

Liabilities + Equity . . . . . . . . . . . . . . . . . $375,319 $321,686 Net income . . . . . . . . . . . . . . . . . . . . . . . 48,351 45,687

Revenues . . . . . . . . . . . . . . . . . . . . . . . . . 229,234 215,639

Required

1. What is the total amount of assets invested in Apple in the current year? 2. What is Apple’s return on assets for the current year? 3. How much are total expenses for Apple for the current year? 4. Is Apple’s current-year return on assets better or worse than competitors’ average of 10% return?

APPLE

42 Chapter 1 Accounting in Business

ETHICS CHALLENGE C3 C4

BTN 1-1 Tana Thorne works in a public accounting firm and hopes to eventually be a partner. The man- agement of Allnet Company invites Thorne to prepare a bid to audit Allnet’s financial statements. In dis- cussing the audit fee, Allnet’s management suggests a fee range in which the amount depends on the reported profit of Allnet. The higher its profit, the higher will be the audit fee paid to Thorne’s firm.

Required

1. Identify the parties potentially affected by this audit and the fee plan proposed. 2. What are the ethical factors in this situation? Explain. 3. Would you recommend that Thorne accept this audit fee arrangement? Why or why not? 4. Describe some ethical considerations guiding your recommendation.

Beyond the Numbers

Beyond the Numbers (BTN) is a special problem section aimed to refine communication, conceptual, analysis, and research skills. It includes many activities helpful in developing an active learning environment.

AA 1-2 Key comparative figures ($ millions) for both Apple and Google follow.COMPARATIVE ANALYSIS A1 A2

APPLE GOOGLE

Apple Google

Key Figures Current Year Prior Year Current Year Prior Year

Liabilities + Equity . . . . . . . . . . $375,319 $321,686 $197,295 $167,497 Net income . . . . . . . . . . . . . . . . 48,351 45,687 12,662 19,478

Revenues . . . . . . . . . . . . . . . . . . 229,234 215,639 110,855 90,272

Required

1. What is the total amount of assets invested for the current year in (a) Apple and (b) Google? 2. What is the current-year return on assets for (a) Apple and (b) Google? 3. How much are current-year expenses for (a) Apple and (b) Google? 4. Is the current-year return on assets better than the 10% return of competitors for (a) Apple and

(b) Google? 5. Relying only on return on assets, would we invest in Google or Apple?

Note: Reference to Google throughout the text refers to Alphabet Inc., as Google is a wholly owned subsidiary of Alphabet.

AA 1-3 Samsung is a leading global manufacturer that competes with Apple and Google. Key financial figures for Samsung follow.

Required

1. What is the return on assets for Samsung in the (a) current year and (b) prior year? 2. Does Samsung’s return on assets exhibit a favorable or unfavorable change? 3. Is Samsung’s current-year return on assets better or worse than that for (a) Apple and (b) Google?

GLOBAL ANALYSIS A1 A2

APPLE GOOGLE

Samsung Korean Won & USD Samsung* Apple Google in millions Current Year Prior Year Current Year Current Year

Average assets . . . . . . . . . . ₩281,963,207 ₩252,176,923 $348,503 $182,396

Net income . . . . . . . . . . . . . 42,186,747 22,726,092 48,351 12,662

Revenues . . . . . . . . . . . . . . . 239,575,376 201,866,745 229,234 110,855

*Figures prepared in accordance with International Financial Reporting Standards as adopted by the Republic of Korea.

BTN 1-2 Refer to this chapter’s opening feature about Apple. Assume that the owners, sometime during their first five years of business, desire to expand their computer product services to meet business demand regarding computing services. They eventually decide to meet with their banker to discuss a loan to allow Apple to expand and offer computing services.

COMMUNICATING IN PRACTICE C2 C4

APPLE

Chapter 1 Accounting in Business 43

Required

1. Prepare a half-page report outlining the information you would request from the owners if you were the loan officer.

2. Indicate whether the information you request and your loan decision are affected by the form of busi- ness organization for Apple.

BTN 1-3 Visit the EDGAR database at SEC.gov. Access the Form 10-K report of Rocky Mountain Chocolate Factory (ticker: RMCF) filed on May 23, 2017, covering its 2017 fiscal year.

Required

1. Item 6 of the 10-K report provides comparative financial highlights of RMCF for the years 2013–2017. Describe the revenue trend for RMCF over this five-year period.

2. Has RMCF been profitable (see net income) over this five-year period? Support your answer.

TAKING IT TO THE NET A2

BTN 1-4 Teamwork is important in today’s business world. Successful teams schedule convenient meet- ings, maintain regular communications, and cooperate with and support their members. This assignment aims to establish support/learning teams, initiate discussions, and set meeting times.

Required

1. Form teams and open a team discussion to determine a regular time and place for your team to meet between each scheduled class meeting. Notify your instructor via a memorandum or e-mail message as to when and where your team will hold regularly scheduled meetings.

2. Develop a list of telephone numbers, LinkedIn pages, and/or e-mail addresses of your teammates.

TEAMWORK IN ACTION C1

BTN 1-5 Refer to this chapter’s opening feature about Apple. Assume that the owners decide to open a new company with an innovative mobile app devoted to microblogging for accountants and those learning accounting. This new company will be called AccountApp.

Required

1. AccountApp obtains a $500,000 loan and the two owners contribute $250,000 in total from their own savings in exchange for ownership of the new company.

a. What is the new company’s total amount of liabilities plus equity? b. What is the new company’s total amount of assets? 2. If the new company earns $80,250 in net income in the first year of operation, compute its return on

assets (assume average assets equal $750,000). Assess its performance if competitors average a 10% return.

ENTREPRENEURIAL DECISION A1 A2

APPLE

Check (2) 10.7%

BTN 1-6 You are to interview a local business owner. (This can be a friend or relative.) Opening lines of communication with members of the business community can provide personal benefits of business net- working. If you do not know the owner, you should call ahead to introduce yourself and explain your position as a student and your assignment requirements. You should request a 30-minute appointment for a face-to-face or phone interview to discuss the form of organization and operations of the business. Be prepared to make a good impression.

Required

1. Identify and describe the main operating activities and the form of organization for this business. 2. Determine and explain why the owner(s) chose this particular form of organization. 3. Identify any special advantages and/or disadvantages the owner(s) experiences in operating with this

form of business organization.

HITTING THE ROAD C4

Design elements: Lightbulb: ©Chuhail/Getty Images; Blue globe: ©nidwlw/Getty Images and ©Dizzle52/Getty Images; Chess piece: ©Andrei Simonenko/Getty Images and ©Dizzle52/Getty Images; Mouse: ©Siede Preis/Getty Images; Global View globe: ©McGraw-Hill Education and ©Dizzle52/Getty Images; Sustainability: ©McGraw-Hill Education and ©Dizzle52/Getty Images

Learning Objectives

CONCEPTUAL C1 Explain the steps in processing

transactions and the role of source documents.

C2 Describe an account and its use in recording transactions.

C3 Describe a ledger and a chart of accounts.

PROCEDURAL P1 Record transactions in a journal and post

entries to a ledger.

P2 Prepare and explain the use of a trial balance.

P3 Prepare financial statements from business transactions.

C4 Define debits and credits and explain double-entry accounting.

ANALYTICAL A1 Analyze the impact of transactions on

accounts and financial statements.

A2 Compute the debt ratio and describe its use in analyzing financial condition.

Chapter Preview

2 Accounting for Business Transactions

NTK 2-4

TRIAL BALANCE

P2 Trial balance preparation and use

Error identification

NTK 2-5

FINANCIAL STATEMENTS

P3 Financial statement preparation

A2 Debt ratio

NTK 2-3

RECORDING TRANSACTIONS

P1 Journalizing and posting

A1 Processing transactions— Examples

NTK 2-1

SYSTEM OF ACCOUNTS

Using financial statements

C1 Source documents

C2 Types of accounts C3 General ledger

NTK 2-2

DEBITS AND CREDITS

T-account

C4 Debits and credits

Normal balance

45

“I’m always confident”—James Park

Have a Fit

SAN FRANCISCO—James Park and Eric Friedman created a wooden box with a circuit board inside. James recalls that to fix an antenna, he “literally took a piece of foam and put it on the circuit board.” Their device could be used to track fitness activ- ity, such as steps taken. The device James and Eric built would later be known as a Fitbit (Fitbit.com).

As Fitbit grew, the co-founders struggled to track sales and expenses. “It was pretty challenging,” recalls James. “I would just try to use the weekend to see if I could catch up.” James and Eric knew that having reliable accounting data would help “manage the ups and downs of running a company.”

To address this concern, the co-founders took action. They set up recordkeeping processes, transaction analysis, control procedures, and financial statement reporting. “You need to see the data,” insists James.

With accounting data, James says he “can uncover insights that weren’t possible or very practical before . . . and enable the discovery of new insights and trends.”

Eric offers the following advice to aspiring entrepreneurs unsure of how to unlock the potential of accounting data: “Get your hands dirty and do it yourself. You learn more that way.”

Sources: Fitbit website, January 2019; Wareable.com, September 2016; Business Wire, November 2015; Fortune, July 2015; Marketing Land, March 2015; Fast Company, March 2014

©Daniel Boczarski/Stringer/Fitbit/Getty Images

Business transactions and events are the starting points of financial statements. The process to go from transactions and events to financial statements includes the following. Identify each transaction and event from source documents. Analyze each transaction and event using the accounting equation. Record relevant transactions and events in a journal. Post journal information to ledger accounts. Prepare and analyze the trial balance and financial

statements.

Source Documents Source documents identify and describe transactions and events entering the accounting sys- tem. They can be in hard copy or electronic form. Examples are sales receipts, checks, purchase orders, bills from suppliers, payroll records, and bank statements. For example, cash registers record each sale on a tape or electronic file. This record is a source document for recording sales in the accounting system. Source documents are objective and reliable evidence about transac- tions and events and their amounts.

The “Account” Underlying Financial Statements An account is a record of increases and decreases in a specific asset, liability, equity, revenue, or expense. The general ledger, or simply ledger, is a record of all accounts used by a company. The ledger is often in electronic form. While most companies’ ledgers have similar accounts, a company often uses one or more unique accounts to match its type of operations. An unclassified balance sheet broadly groups accounts into assets, liabilities, and equity. Exhibit 2.1 shows common asset, liability, and equity accounts.

Asset Accounts Assets are resources owned or controlled by a company. Resources have expected future benefits. Most accounting systems include (at a minimum) separate accounts for the assets described here. Cash A Cash account shows a company’s cash balance. All increases and decreases in cash are recorded in the Cash account. It includes money and any funds that a bank accepts for de- posit (coins, checks, money orders, and checking account balances).

C2 Describe an account and its use in recording transactions.

BASIS OF FINANCIAL STATEMENTS C1 Explain the steps in process- ing transactions and the role of source documents.

Transactions analyzed with accounting equation

Transaction occurs with source documents

Z-Mart Stores

Transactions recorded and posted

Transactions reported in financial statements

Dec. 1 Cash C. Taylor, Capital

Receive investment by owner.

Account Titles and Explanation PRDate

30,000

Debit

30,000

Credit

307 101

General Journal

General Ledger

Dec. 1 G1 30,000 30,000

Date PR

Cash

Debit Credit Account no. 101

BalanceExplanation

Dec. 1 G1 30,000 30,000

Date PR

C. Taylor, Capital

Debit Credit Account no. 307

BalanceExplanation

4

3

1

2

Assets

Cash

Accoun ts paya

ble

Supplie s

Unearn ed con

sult. re venue

Prepai d insur

ance

Total li abilitie

s

Equipm ent

Liabilit ies

Equity

C. Tayl or, Cap

ital

Total a ssets

Total li abilitie

s and e quity

$

$

FASTF ORWA

RD

Income Statem

ent

For Mo nth En

ded De cembe

r 31,

Revenu es

+

Rental revenu

e

Total re venues

Expens es

Salarie s expe

nse

Rent ex pense

Utilitie s expe

nse

Total e xpense

s

Net inc ome

$

$

Plus: Investm

ents by owner

Net inc ome

Less: Withdr

awals b y owne

r

$FAST FORWA

RD

Statem ent of

Owner ’s Equi

ty

For Mo nth En

ded De cembe

r 31,

FASTF ORWA

RD

Balanc e Shee

t

Decem ber 31,

Point: Accounting records also are called accounting books or the books.

46 Chapter 2 Accounting for Business Transactions

Accounts Receivable Accounts receivable are held by a seller and are promises of payment from customers to sellers. Accounts receivable are increased by credit sales or sales on account (or on credit). They are decreased by customer payments. We record all increases and decreases in receivables in the Accounts Receivable account. When there are multiple customers, separate records are kept for each, titled Accounts Receivable—‘Customer Name’.

Note Receivable A note receivable, or promissory note, is a written promise of another en- tity to pay a specific sum of money on a specified future date to the holder of the note; the holder has an asset recorded in a Note (or Notes) Receivable account.

Prepaid Accounts Prepaid accounts (or prepaid expenses) are assets from prepayments of future expenses (expenses expected to be incurred in future accounting periods). When the ex- penses are later incurred, the amounts in prepaid accounts are transferred to expense accounts. Common examples of prepaid accounts are prepaid insurance, prepaid rent, and prepaid ser- vices. Prepaid accounts expire with the passage of time (such as with rent) or through use (such as with prepaid meal plans). When financial statements are prepared, (1) all expired and used prepaid accounts are recorded as expenses and (2) all unexpired and unused prepaid accounts are recorded as assets (reflecting future benefits). Chapter 3 covers prepaid accounts in detail.

Supplies Accounts Supplies are assets until they are used. When they are used up, their costs are reported as expenses. Unused supplies are recorded in a Supplies asset account. Supplies often are grouped by purpose—for example, office supplies and store supplies. Office supplies include paper and pens. Store supplies include packaging and cleaning materials.

Equipment Accounts Equipment is an asset. When equipment is used and wears down, its cost is gradually reported as an expense (called depreciation). Equipment often is grouped by its purpose—for example, office equipment and store equipment. Office equipment includes com- puters and desks. The Store Equipment account includes counters and cash registers.

Buildings Accounts Buildings such as stores, offices, warehouses, and factories are assets because they provide expected future benefits. When a building is used and wears down, its cost is reported as an expense (called depreciation). When several buildings are owned, separate ac- counts are sometimes kept for each of them.

Land The cost of land is recorded in a Land account. The cost of buildings located on the land is separately recorded in building accounts.

Point: Customers and others who owe a company are debtors.

Point: A note receivable is differ- ent than an account receivable because it comes from a formal contract called a promissory note. A note receivable usually requires interest, whereas an account receivable does not.

Point: At the beginning of the term, a prepaid college parking pass is an asset that allows a stu- dent to park on campus. Benefits of the parking pass expire as the term progresses. At term-end, prepaid parking (asset) equals zero as it has been entirely recorded as parking expense.

Point: Some assets are called intangible because they do not have physical existence. Coca-Cola reports billions in intangible assets.

Patents Land

Long-Term Notes Payable

= + Equity AccountsLiability Accounts

Accrued Liabilities

Unearned Revenue Short-Term Notes Payable

Accounts Payable

Dividends

Revenues Expenses

Common Stock

Buildings Equipment

Investment in Land Supplies

Prepaid Accounts Inventory

Notes Receivable Accounts Receivable

Cash

Asset Accounts

EXHIBIT 2.1 Accounts Organized by the Accounting Equation

Women Entrepreneurs Sara Blakely (in photo), the billionaire entrepreneur/owner of SPANX, has promised to do- nate half of her wealth to charity. The Center for Women’s Business Research reports the following for women-owned businesses.

• They total more than 11 million and employ nearly 20 million workers.

• They generate $2.5 trillion in annual sales and tend to embrace technology.

• They are philanthropic—70% of owners volunteer at least once per month. ■

Decision Insight

©Rob Kim/Getty Images

Chapter 2 Accounting for Business Transactions 47

Liability Accounts Liabilities are obligations to transfer assets or provide products or services to others. They are claims (by creditors) against assets. Creditors are individuals and organizations that have rights to receive payments from a company. Common liability accounts are described here.

Accounts Payable Accounts payable are promises to pay later. Payables can come from purchases of merchandise-for-resale, supplies, equipment, and services. We record all increases and decreases in payables in the Accounts Payable account. When there are multiple suppliers, separate records are kept for each, titled Accounts Payable—‘Supplier Name’.

Note Payable A note payable is a written promissory note to pay a future amount. It is re- corded as either a short-term note payable or a long-term note payable, depending on when it must be repaid. We explain short- and long-term classification in the next two chapters.

Unearned Revenue Accounts Unearned revenue is a liability that is settled in the future when a company delivers its products or services. When customers pay in advance for products or services (before revenue is earned), the seller records this receipt as unearned revenue. Examples of unearned revenue include magazine subscriptions collected in advance by a pub- lisher, rent collected in advance by a landlord, and season ticket sales by sports teams. The seller would record these in liability accounts such as Unearned Subscriptions and Unearned Rent. When products and services are later delivered, the earned portion of the unearned revenue is transferred to revenue accounts such as Subscription Fees Revenue and Rent Revenue.1

Accrued Liabilities Accrued liabilities are amounts owed that are not yet paid. Examples are wages payable, taxes payable, and interest payable. These often are recorded in separate liability accounts by the same title. If they are not a large amount, one or more ledger accounts can be added and reported as a single amount on the balance sheet. (Financial statements often report totals of several ledger accounts.)

Point: Accounts payable also are called trade payables.

Point: Two words that almost always identify liability accounts: “payable,” meaning liabilities that must be paid, and “unearned,” meaning liabilities that must be fulfilled.

1In practice, account titles vary. Subscription Fees Revenue is sometimes called Subscription Fees, Subscription Fees Earned, or Earned Subscription Fees. Rent Revenue is sometimes called Rent Earned, Rental Revenue, or Earned Rent Revenue. Titles can differ even within the same industry. Product sales are called net sales at Apple, revenues at Google, and revenue at Samsung. Revenues or fees is commonly used with service businesses, and net sales or sales is used with product businesses.

Unearned Revenue The Dallas Cowboys, Atlanta Falcons, New England Patriots, and most NFL teams have over $100 million in advance ticket sales in Unearned Revenue. When a team plays its home games, it settles this liability to its ticket holders and then transfers the amount earned to Ticket Revenue. Teams in other major sports such as the National Women’s Soccer League and the Women’s National Basketball Association also have unearned revenue. ■

Decision Insight

©Mike Zarrilli/Getty Images

Equity Accounts The owner’s claim on a company’s assets is called equity, stockholders’ equity, or shareholders’ equity. Equity is the owner’s residual interest in the assets of a business after subtracting liabilities. Equity is impacted by four types of accounts.

= – + –Equity Dividends Revenues ExpensesCommon stock

We show this in Exhibit 2.2 by expanding the accounting equation. We also organize assets and liabilities into subgroups that have similar attributes. An important subgroup for both assets and liabilities is the current items. Current items are expected to be either collected or owed within the next year. The next chapter explains this. At this point, know that a classified balance sheet groups accounts into classifications (such as land and buildings into Plant Assets) and it reports current assets before noncurrent assets and current liabilities before noncurrent liabilities.

Point: A note payable is different than an account payable because it comes from a formal contract called a promissory note and requires interest.

48 Chapter 2 Accounting for Business Transactions

Owner Investments When an owner invests in a company, it increases both assets and equity. The increase to equity is recorded in the account titled Common Stock. Owner invest- ments are not revenues of the business.

Owner Distributions When a corporation distributes assets to its owners, it decreases both company assets and total equity. The decrease to equity is recorded in an account titled Dividends. Dividends are not expenses of the business; they are simply the opposite of owner investments.

Revenue Accounts Amounts received from sales of products and services to customers are recorded in revenue accounts, which increase equity. Examples of revenue accounts are Sales, Commissions Earned, Professional Fees Earned, Rent Revenue, and Interest Revenue. Revenues always increase equity.

Expense Accounts Amounts used for costs of providing products and services are recorded in expense accounts, which decrease equity. Examples of expense accounts are Advertising Expense, Salaries Expense, Rent Expense, Utilities Expense, and Insurance Expense. Expenses always decrease equity. A variety of revenues and expenses are in the chart of accounts at the end of this book. (Different companies use different account titles to describe the same thing. For example, some use Interest Revenue instead of Interest Earned.)

Point: Dividends account can be viewed as a contra equity account because it reduces the normal balance of equity.

Revenues ExpensesDividends

Dividends Revenues Expenses

Common Stock

Common Stock

Patents

Land

Long-Term Notes Payable

= + Equity AccountsLiability Accounts

Accrued Liabilities

Unearned Revenue Short-Term Notes Payable

Accounts Payable

Dividends

Common Stock

Buildings Equipment

Investment in Land

Supplies Prepaid Accounts

Inventory Notes Receivable

Accounts Receivable Cash

Asset Accounts

Long-Term Investments

Plant Assets

Current Assets Current Liabilities

Long-Term Liabilities

Intangible Assets

Revenues Expenses

EXHIBIT 2.2 Accounts Classified by the Expanded Accounting Equation

Sporting Accounts The Cleveland Cavaliers, Boston Celtics, Golden State Warriors, and other NBA teams have revenue accounts that include Ticket Sales, Broadcast Fees, and Advertising Revenues. Expense accounts include Player Salaries, NBA Franchise Costs, and Promotional Costs. ■

Decision Insight

C3 Describe a ledger and a chart of accounts.

Ledger and Chart of Accounts The collection of all accounts and their balances is called a ledger (or general ledger). A com- pany’s size and diversity of operations affect the number of accounts needed. A small company

can have as few as 20 accounts; a large company can require thousands. The chart of accounts is a list of all ledger accounts and has an identification number assigned to each account. Exhibit 2.3 shows a common numbering system of accounts for a smaller business.

These account numbers have a three-digit code that is useful in record- keeping. In this example, the first digit of asset accounts is a 1, the first digit of liability accounts is a 2, and so on. The second and third digits relate to the accounts’ subcategories. Exhibit 2.4 shows a partial chart of accounts for FastForward.

Asset accounts Liability accounts Equity accounts Revenue accounts Expense accounts

Chart of Accounts

101–199 201–299 301–399 401–499 501–699

EXHIBIT 2.3 Typical Chart of Accounts for a Smaller Business

Chapter 2 Accounting for Business Transactions 49

EXHIBIT 2.4 Partial Chart of Accounts for FastForward

Chart of Accounts

Assets

101 Cash 106 Accounts receivable 126 Supplies 128 Prepaid insurance 167 Equipment

Liabilities

201 Accounts payable

236 Unearned consulting revenue

Revenues Expenses 403 Consulting revenue 622 Salaries expense

406 Rental revenue 637 Insurance expense

640 Rent expense

652 Supplies expense

690 Utilities expense

307 Common stock

318 Retained earnings

319 Dividends

Equity

Classify each of the following accounts as either an asset (A), liability (L), or equity (EQ) account.

Classifying Accounts

NEED-TO-KNOW 2-1

C1 C2 C3

1. Prepaid Rent 2. Common Stock 3. Note Receivable 4. Accounts Payable

5. Accounts Receivable 6. Equipment 7. Interest Payable 8. Unearned Revenue

9. Land 10. Prepaid Insurance 11. Wages Payable 12. Rent Payable

Solution

1. A 2. EQ 3. A 4. L 5. A 6. A 7. L 8. L 9. A 10. A 11. L 12. L Do More: QS 2-2, QS 2-3

Debits and Credits A T-account represents a ledger account and is used to show the effects of transactions. Its name comes from its shape like the letter T. The layout of a T-account is shown in Exhibit 2.5.

The left side of an account is called the debit side, or Dr. The right side is called the credit side, or Cr. To enter amounts on the left side of an account is to debit the account. To enter amounts on the right side is to credit the account. The term debit or credit, by itself, does not mean increase or decrease. Whether a debit or a credit is an increase or decrease depends on the account.

The difference between total debits and total credits for an account, including any beginning balance, is the account balance. When total debits exceed total credits, the account has a debit balance. It has a credit balance when total credits exceed total debits. When total debits equal total credits, the account has a zero balance.

Double-Entry System Double-entry accounting demands the accounting equation remain in balance, which means that for each transaction: At least two accounts are involved, with at least one debit and one credit. Total amount debited must equal total amount credited.

This means total debits must equal total credits for all entries, and total debit account balances in the ledger must equal total credit account balances. The system for recording debits and credits follows the accounting equation—see Exhibit 2.6.

DOUBLE-ENTRY ACCOUNTING

C4 Define debits and credits and explain double-entry accounting.

EXHIBIT 2.5 The T-Account (Left side) (Right side)

Debit Credit

Account Title

Point: Dr. and Cr. come from 18th-century English where terms debitor and creditor were used instead of debit and credit. Dr. and Cr. use the first and last let- ters of these terms, just as we still do for Saint (St.) and Doctor (Dr.).

“Total debits equal total credits for

each entry.”

50 Chapter 2 Accounting for Business Transactions

Net increases or decreases on one side have equal net effects on the other side. For exam- ple, a net increase in assets must include an equal net increase on the liabilities and equity side. Some transactions affect only one side of the equation, such as acquiring a land asset by giving up a cash asset, but their net effect on this one side is zero.

The left side is the normal balance side for assets; the right side is the normal balance side for liabilities and equity. This matches their layout in the accounting equation, where assets are on the left side and liabilities and equity are on the right.

Equity increases from revenues and owner investments (stock issuances), and it decreases from expenses and dividends. We see this by expanding the accounting equation to include debits and credits in double-entry form, as shown in Exhibit 2.7.

Point: Assets are on the left-hand side of the equation and thus in- crease on the left. Liabilities and equity are on the right-hand side of the equation and thus increase on the right.

Debit for increases

+

Credit for decreases

= +

– +

Debit for decreases

Credit for increases

– +

Debit for decreases

Credit for increases

Assets Liabilities Equity

Normal Normal Normal

EXHIBIT 2.6 Debits and Credits in the Accounting Equation

= + – + – Dr. for

increases Cr. for

decreases

Assets Liabilities Dividends Revenues ExpensesCommon Stock

+ +– +– – +– –+ Dr. for

decreases Cr. for

increases

+ Dr. for

increases Cr. for

decreases Dr. for

decreases Cr. for

increases Dr. for

increases Cr. for

decreases Dr. for

decreases Cr. for

increases

Equity

Normal Normal Normal Normal Normal Normal

EXHIBIT 2.7 Debit and Credit Effects for Component Accounts

Increases (credits) to common stock and revenues increase equity; increases (debits) to dividends and expenses decrease equity. The normal balance of each account is the side where increases are recorded.

The T-account for FastForward’s Cash account, reflecting its first 11 transactions (from Exhibit 1.9), is shown in Exhibit 2.8. The total increases (debits) in its Cash account are $36,100, and the total decreases (credits) are $31,300. Total debits exceed total credits by $4,800, result- ing in its ending debit balance of $4,800.

Point: DrEAD means debit (Dr) is the normal balance side for Expense, Asset, and Dividend accounts; credit the others.

Point: The ending balance is on the side with the larger dollar amount. Also, a plus (+) and minus (−) are not used in a T-account.

EXHIBIT 2.8 Computing the Balance for a T-Account

Cash

Receive investment by owner for stock 30,000 Purchase of supplies 2,500

Consulting services revenue earned 4,200 Purchase of equipment 26,000

Collection of account receivable 1,900 Payment of rent 1,000

Payment of salary 700

Payment of account payable 900

Payment of cash dividend 200

Balance 4,800

⎧ ⎨ ⎩

⎧ ⎨ ⎩

⎫ ⎪ ⎪ ⎬ ⎪ ⎪ ⎭

36,100

36,100 − 31,300

31,300

Identify the normal balance (debit [Dr] or credit [Cr]) for each of the following accounts.

Normal Account Balance

NEED-TO-KNOW 2-2 1. Prepaid Rent 2. Common Stock 3. Note Receivable 4. Accounts Payable

5. Accounts Receivable 6. Equipment 7. Interest Payable 8. Unearned Revenue

9. Land 10. Prepaid Insurance 11. Dividends 12. Utilities Expense

C4

Solution

1. Dr. 2. Cr. 3. Dr. 4. Cr. 5. Dr. 6. Dr. 7. Cr. 8. Cr. 9. Dr. 10. Dr. 11. Dr. 12. Dr. Do More: QS 2-4, QS 2-5,

QS 2-7, E 2-4

Point: Debit and credit are accounting directions for left and right.

Chapter 2 Accounting for Business Transactions 51

This section explains the analyzing, recording, and posting of transactions.

Journalizing and Posting Transactions The four steps of processing transactions are shown in Exhibit 2.9. Steps 1 and 2—transaction analysis and the accounting equation—already were covered. This section focuses on steps 3 and 4. Step 3 is to record each transaction chronologically in a journal. A journal is a complete record of each transaction in one place. It also shows debits and credits for each transaction. Recording transactions in a journal is called journalizing. Step 4 is to transfer (or post) entries from the journal to the ledger. Transferring journal entry information to the ledger is called posting.

ANALYZING AND PROCESSING TRANSACTIONS

P1 Record transactions in a journal and post entries to a ledger.

Services Contract

Deposit

TOTAL

1 30,000 Bank Statement

Client Billing Sales Receipt

Purchase Order Dec. 1 Cash Common Stock

30,000

2,500

30,000

2,500 SuppliesDec. 2 Cash

General Journal Debit for

increases

+

Credit for

decreases

= +

– 1 Debit for

decreases

Credit for

increases – + Debit for

decreases

Credit for

increasesAssets Liabilities

Equity

Assets = Liabilities + Equity

Step 1: Identify transactions and source documents.

Step 2: Analyze transactions using the accounting equation.

Step 3: Record journal entry.

General Journal

Ledger

Step 4: Post entry to ledger.

EXHIBIT 2.9 Steps in Processing Transactions

Journalizing Transactions Journalizing transactions requires an understanding of a journal. While companies can use various journals, every company uses a general journal. It can be used to record any transaction. Exhibit 2.10 shows how the first two transactions of FastForward are recorded in a general journal.

To record entries in a general journal, apply these steps; refer to Exhibit 2.10.

a Date the transaction: Enter the year at the top of the first column and the month and day on the first line of each journal entry.

b Enter titles of accounts debited and then enter amounts in the Debit column on the same line. Account titles are taken from the chart of accounts and are aligned with the left margin of the Account Titles and Explanation column.

c Enter titles of accounts credited and then enter amounts in the Credit column on the same line. Account titles are from the chart of accounts and are indented from the left margin of the Account Titles and Explanation column to separate them from debited accounts.

d Enter a brief explanation of the transaction on the line below the entry (it often references a source document). This explanation is indented about half as far as the credited account titles to avoid confusing it with accounts, and it is italicized.

A blank line is left between each journal entry for clarity. When a transaction is first recorded, the posting reference (PR) column is left blank (in a manual system). Later, when posting entries to the ledger, the identification numbers of the individual ledger accounts are entered in the PR column.

Balance Column Account T-accounts are simple and show how the accounting pro- cess works. However, actual accounting systems need more structure and therefore use a differ- ent formatting of T-accounts, called balance column accounts, shown in Exhibit 2.11.

Point: There are no exact rules for a journal entry explanation—it should be short yet describe why an entry is made.

Dec. 1

Dec. 2

Cash

Cash

Common Stock Receive investment by owner.

Purchase supplies for cash.

Supplies

Account Titles and Explanation PRDate

30,000

2,500

Debit

30,000

2,500

Credit 2019 a

d

General Journal

b c

EXHIBIT 2.10 Partial General Journal for FastForward

52 Chapter 2 Accounting for Business Transactions

Processing Transactions—An Example We use FastForward to show how double-entry accounting is used in analyzing and processing transactions. Analysis of each transaction follows the four steps of Exhibit 2.9.

Step 1 Identify the transaction and any source documents. Step 2 Analyze the transaction using the accounting equation. Step 3 Record the transaction in journal entry form applying double-entry accounting. Step 4 Post the entry (for simplicity, we use T-accounts to represent ledger accounts).

A1 Analyze the impact of transactions on accounts and financial statements.

Key:

Enter the debit account number from the ledger in the PR column of the journal (blue line).

C

D

A

B

Identify credit account in ledger: enter date, journal page, amount, and balance (gold line).

Enter the credit account number from the ledger in the PR column of the journal (green line).

Identify debit account in ledger: enter date, journal page, amount, and balance (red line).

Dec. 1 2019

Cash Common Stock

Receive investment by owner.

Account Titles and Explanation PRDate

30,000

Debit

30,000

Credit

307 101

General Journal

General Ledger

Dec. 1 G1 30,000 30,000

Date PR

Cash

Debit Credit Account No. 101

BalanceExplanation

Dec. 1 G1 30,000 30,000

Date PR

Common Stock

Debit Credit Account No. 307

BalanceExplanation

D

C

B

A

2019

2019

EXHIBIT 2.12 Posting an Entry to the Ledger

Dec. 2 Dec. 3 Dec. 10

Dec. 1 2019

G1 G1 G1 G1

30,000

4,200

2,500 26,000

30,000 27,500

1,500 5,700

Date PR Cash

General Ledger

Debit Credit Account No. 101

BalanceExplanation

EXHIBIT 2.11 Cash Account in Balance Column Format

Point: Posting is automatic with accounting software.

Point: The fundamental concepts of a manual system are identical to those of a computerized information system.

The balance column account format is similar to a T-account in having columns for debits and credits. It is different in including transaction date and explanation col- umns. It also has a column with the balance of the account after each entry is recorded. FastForward’s Cash account in Exhibit 2.11 is debited on December 1 for the $30,000 owner investment, yielding a $30,000 debit balance. The

account is credited on December 2 for $2,500, yielding a $27,500 debit balance. On December 3, it is credited for $26,000, and its debit balance is reduced to $1,500. The Cash account is debited for $4,200 on December 10, and its debit balance increases to $5,700; and so on.

The heading of the Balance column does not show whether it is a debit or credit balance. Instead, an account is assumed to have a normal balance. Unusual events can sometimes temporarily create an abnormal balance. An abnormal balance is a balance on the side where decreases are recorded. For example, a customer might mistakenly overpay a bill. This gives that customer’s account receiv- able an abnormal (credit) balance. An abnormal balance often is identified by setting it in brackets or entering it in red. A zero balance is shown by writing zero or a dash in the Balance column.

Posting Journal Entries Step 4 of processing transactions is to post journal entries to ledger accounts. All entries are posted to the ledger before financial statements are prepared so that account balances are up-to-date. When entries are posted to the ledger, the debits in journal entries are transferred into ledger accounts as debits, and credits are transferred into ledger ac- counts as credits. Exhibit 2.12 shows four parts to posting a journal entry. A Identify the led- ger account(s) that is debited in the entry. In the ledger, enter the entry date, the journal and page in its PR column, the debit amount, and the new balance of the ledger account. (G shows it came from the general journal.) B Enter the ledger account number in the PR col- umn of the journal. Parts C and D repeat the first two steps for credit entries and amounts. The posting process creates a link between the ledger and the journal entry. This link is a useful cross-reference for tracing an amount from one record to another.

Point: Explanations are included in ledger accounts only for unusual transactions or events.

Chapter 2 Accounting for Business Transactions 53

1. Receive Investment by Owner

2 Analyze Assets = Liabilities + Equity Common Cash Stock +30,000 = 0 +30,000

1 Identify FastForward receives $30,000 cash from Chas Taylor in exchange for common stock.

3 RecoRd (1) Cash 101 30,000 Common Stock 307 30,000

4 Post

(1) 30,000

Cash 101

(1) 30,000

Common Stock 307

Date Account Titles and Explanation PR Debit Credit

(2) 2,500

Supplies 126

(1) 30,000 (2) 2,500

Cash 101

4 Post

2. Purchase Supplies for Cash

1 Identify FastForward pays $2,500 cash for supplies. 2 analyze Assets = Liabilities + Equity

Cash Supplies −2,500 +2,500 = 0 + 0

Changes the composition of assets but not the total.

3 RecoRd (2) Supplies 126 2,500 Cash 101 2,500

Date Account Titles and Explanation PR Debit Credit

3. Purchase Equipment for Cash

1 Identify FastForward pays $26,000 cash for equipment.

Changes the composition of assets but not the total.

3 RecoRd (3) Equipment 167 26,000 Cash 101 26,000

(3) 26,000

Equipment 167 4 Post

(1) 30,000 (2) 2,500

(3) 26,000

Cash 101

2 analyze Assets = Liabilities + Equity Cash Equipment

−26,000 +26,000 = 0 + 0

Date Account Titles and Explanation PR Debit Credit

FASTForward

4. Purchase Supplies on Credit

1 Identify FastForward purchases $7,100 of supplies on credit from a supplier.

3 RecoRd (4) Supplies 126 7,100 Accounts Payable 201 7,100

(4) 7,100

Accounts Payable 201

4 Post

(2) 2,500

(4) 7,100

Supplies 126

2 analyze Assets = Liabilities + Equity Accounts Supplies Payable

+7,100 = +7,100 + 0

Date Account Titles and Explanation PR Debit Credit

Study each transaction before moving to the next. The first 11 transactions are from Chapter 1, and we analyze five additional December transactions of FastForward (numbered 12 through 16).

Point: In Need-to-Know 2-5, we show how to use balance column accounts for the ledger.

5. Provide Services for Cash

1 Identify FastForward provides consulting services and immediately collects $4,200 cash.

2 analyze Assets = Liabilities + Equity Consulting Cash Revenue +4,200 = 0 +4,200

3 RecoRd (5) Cash 101 4,200 Consulting Revenue 403 4,200

(1) 30,000 (2) 2,500

(5) 4,200 (3) 26,000

Cash 101

(5) 4,200

Consulting Revenue 403

4 Post

Date Account Titles and Explanation PR Debit Credit ©Adie Bush/Getty Images

54 Chapter 2 Accounting for Business Transactions

9. Receipt of Cash on Account

1 Identify FastForward receives $1,900 cash from the customer billed in Transaction 8.

2 analyze

3 RecoRd (9) Cash 101 1,900 Accounts Receivable 106 1,900

4 Post

(1) 30,000 (2) 2,500

(5) 4,200 (3) 26,000

(9) 1,900 (6) 1,000 (7) 700

Cash 101

(8) 1,900 (9) 1,900

Accounts Receivable 106

Assets = Liabilities + Equity Accounts

Cash Receivable +1,900 −1,900 = 0 + 0

Date Account Titles and Explanation PR Debit Credit

8. Provide Consulting and Rental Services on Credit

1 Identify FastForward provides consulting services of $1,600 and rents its test facilities for $300. The customer is billed $1,900 for these services.

2 analyze

3 RecoRd (8) Accounts Receivable 106 1,900 Consulting Revenue 403 1,600

Rental Revenue 406 300

4 Post

(8) 1,900

Accounts Receivable 106

(5) 4,200

(8) 1,600

Consulting Revenue 403

(8) 300

Rental Revenue 406

Assets = Liabilities + Equity Accounts Consulting Rental

Receivable Revenue Revenue +1,900 = 0 +1,600 +300

Date Account Titles and Explanation PR Debit Credit

Point: The revenue recognition principle requires revenue to be recognized when the company provides products and services to a customer. This is not necessarily the same time that the customer pays.

Point: Transaction 8 is a compound journal entry, which is an entry that affects three or more accounts. The rule that total debits equal total credits continues.

6. Payment of Expense in Cash

1 Identify FastForward pays $1,000 cash for December rent. 2 analyze Assets = Liabilities + Equity

Rent Cash Expense −1,000 = 0 −1,000

3 RecoRd (6) Rent Expense 640 1,000 Cash 101 1,000

4 Post

(6) 1,000

Rent Expense 640

(1) 30,000 (2) 2,500

(5) 4,200 (3) 26,000

(6) 1,000

Cash 101

Date Account Titles and Explanation PR Debit Credit

7. Payment of Expense in Cash

1 Identify FastForward pays $700 cash for employee salary. 2 analyze Assets = Liabilities + Equity

Salaries Cash Expense −700 = 0 −700

3 RecoRd (7) Salaries Expense 622 700 Cash 101 700

(1) 30,000 (2) 2,500

(5) 4,200 (3) 26,000

(6) 1,000

(7) 700

Cash 101

(7) 700

Salaries Expense 622 4 Post

Date Account Titles and Explanation PR Debit Credit

Point: Salary usually refers to compensation of a fixed amount for a given time period. Wages is compensation based on time worked.

Chapter 2 Accounting for Business Transactions 55

10. Partial Payment of Accounts Payable

1 Identify FastForward pays CalTech Supply $900 cash toward the payable of Transaction 4.

2 analyze

3 RecoRd (10) Accounts Payable 201 900 Cash 101 900

4 Post

(10) 900 (4) 7,100

Accounts Payable 201

(1) 30,000 (2) 2,500

(5) 4,200 (3) 26,000

(9) 1,900 (6) 1,000

(7) 700

(10) 900

Cash 101

Assets = Liabilities + Equity Cash Accounts Payable −900 = −900 + 0

Date Account Titles and Explanation PR Debit Credit

11. Payment of Cash Dividend

1 Identify FastForward pays a $200 cash dividend.

2 analyze

3 RecoRd (11) Dividends 319 200 Cash 101 200

4 Post

(11) 200

Dividends 319

(1) 30,000 (2) 2,500

(5) 4,200 (3) 26,000

(9) 1,900 (6) 1,000

(7) 700

(10) 900

(11) 200

Cash 101

Assets = Liabilities + Equity Cash Dividends −200 = 0 −200

Date Account Titles and Explanation PR Debit Credit

Point: Dividends always decrease equity.

12. Receipt of Cash for Future Services

1 Identify FastForward receives $3,000 cash in advance of providing consulting services to a customer.

2 analyze

Accepting $3,000 cash requires FastForward to perform future services and is a liability. No revenue is recorded until services are provided.

3 RecoRd (12) Cash 101 3,000 Unearned Consulting

Revenue 236 3,000 (12) 3,000

Unearned Consulting Revenue 236

(1) 30,000 (2) 2,500

(5) 4,200 (3) 26,000

(9) 1,900 (6) 1,000

(12) 3,000 (7) 700 (10) 900

(11) 200

Cash 101 4 Post

Assets = Liabilities + Equity Unearned

Cash Consulting Revenue +3,000 = +3,000 + 0

Date Account Titles and Explanation PR Debit Credit

Point: “Unearned” accounts are liabilities that must be fulfilled.

13. Pay Cash for Future Insurance Coverage

1 Identify FastForward pays $2,400 cash (insurance premium) for a 24-month insurance policy. Coverage begins on December 1.

2 Analyze

Changes the composition of assets from cash to prepaid insurance. Expense is recorded as insur- ance coverage expires.

3 RecoRd (13) Prepaid Insurance 128 2,400 Cash 101 2,400

(13) 2,400

Prepaid Insurance 128

(1) 30,000 (2) 2,500

(5) 4,200 (3) 26,000

(9) 1,900 (6) 1,000

(12) 3,000 (7) 700

(10) 900

(11) 200

(13) 2,400

Cash 101

4 Post

Assets = Liabilities + Equity Prepaid

Cash Insurance −2,400 +2,400 = 0 + 0

Date Account Titles and Explanation PR Debit Credit

56 Chapter 2 Accounting for Business Transactions

14. Purchase Supplies for Cash

1 Identify FastForward pays $120 cash for supplies.

2 analyze

3 RecoRd (14) Supplies 126 120 Cash 101 120

(2) 2,500

(4) 7,100

(14) 120

Supplies 126

(1) 30,000 (2) 2,500

(5) 4,200 (3) 26,000

(9) 1,900 (6) 1,000

(12) 3,000 (7) 700

(10) 900

(11) 200

(13) 2,400

(14) 120

Cash 101

4 Post

Assets = Liabilities + Equity Cash Supplies

−120 +120 = 0 + 0

Date Account Titles and Explanation PR Debit Credit

Point: Luca Pacioli, a 15th-century monk and famous mathematician, was the first to devise double- entry accounting.

15. Payment of Expense in Cash

1 Identify FastForward pays $305 cash for December utilities expense.

3 RecoRd (15) Utilities Expense 690 305 Cash 101 305

(15) 305

Utilities Expense 690

(1) 30,000 (2) 2,500

(5) 4,200 (3) 26,000

(9) 1,900 (6) 1,000

(12) 3,000 (7) 700

(10) 900

(11) 200

(13) 2,400

(14) 120

(15) 305

Cash 101

4 Post

2 Analyze Assets = Liabilities + Equity Utilities Cash Expense −305 = 0 −305

Date Account Titles and Explanation PR Debit Credit

16. Payment of Expense in Cash

2 Analyze Assets = Liabilities + Equity Salaries Cash Expense −700 = 0 −700

1 Identify FastForward pays $700 cash in employee salary for work performed in the latter part of December.

3 RecoRd (16) Salaries Expense 622 700 Cash 101 700

(7) 700

(16) 700

Salaries Expense 622

(1) 30,000 (2) 2,500

(5) 4,200 (3) 26,000

(9) 1,900 (6) 1,000

(12) 3,000 (7) 700

(10) 900

(11) 200

(13) 2,400

(14) 120

(15) 305

(16) 700

Cash 101

4 Post

Date Account Titles and Explanation PR Debit Credit

Chapter 2 Accounting for Business Transactions 57

Summarizing Transactions in a Ledger Exhibit 2.13 shows the ledger accounts (in T-account form) of FastForward after all 16 transac- tions are recorded and posted and the balances computed. The accounts are grouped into three columns following the accounting equation: assets, liabilities, and equity. Totals for the three columns obey the accounting equation:

Assets equal $42,395 ($4,275 + $0 + $9,720 + $2,400 + $26,000). Liabilities equal $9,200 ($6,200 + $3,000). Equity equals $33,195 ($30,000 − $200 + $5,800 + $300 − $1,400 − $1,000 − $305).

The accounting equation: $42,395 = $9,200 + $33,195. Common stock, dividends, revenue, and expense accounts reflect transactions that

change equity. Revenue and expense account balances are reported in the income statement.

Debit and Credit Rules Increase Accounts (normal bal.) Decrease

Asset . . . . . . . . . . . . Debit Credit Liability . . . . . . . . . . . Credit Debit Common Stock . . . . Credit Debit Dividends . . . . . . . . . Debit Credit Revenue . . . . . . . . . Credit Debit Expense . . . . . . . . . Debit Credit

EXHIBIT 2.13 Ledger for FastForward (in T-Account Form)

$42,395 = $9,200 + $33,195

Cash 101

(1) 30,000 (2) 2,500

(5) 4,200 (3) 26,000

(9) 1,900 (6) 1,000

(12) 3,000 (7) 700

(10) 900

(11) 200

(13) 2,400

(14) 120

(15) 305

(16) 700

Balance 4,275

Accounts Receivable 106

(8) 1,900 (9) 1,900

Balance 0

Supplies 126

(2) 2,500

(4) 7,100

(14) 120

Balance 9,720

Prepaid Insurance 128

(13) 2,400

Equipment 167

(3) 26,000

Accounts Payable 201

(10) 900 (4) 7,100

Balance 6,200

Unearned Consulting Revenue 236

(12) 3,000

Common Stock 307

(1) 30,000

Dividends 319

(11) 200

Consulting Revenue 403

(5) 4,200

(8) 1,600

Balance 5,800

Rental Revenue 406

(8) 300

Rent Expense 640

(6) 1,000

Utilities Expense 690

(15) 305

Accounts in this white area are on the income statement .

Salaries Expense 622

(7) 700

(16) 700

Balance 1,400

FASTForward

Assets = Liabilities + Equity General Ledger

58 Chapter 2 Accounting for Business Transactions

Assume Tata Company began operations on January 1 and completed the following transactions during its first month of operations. For each transaction, (a) analyze the transaction using the accounting equation, (b) record the transaction in journal entry form, and (c) post the entry using T-accounts to represent ledger accounts. Tata Company has the following (partial) chart of accounts—account numbers in parentheses: Cash (101); Accounts Receivable (106); Equipment (167); Accounts Payable (201); Common Stock (307); Dividends (319); Services Revenue (403); and Wages Expense (601).

Jan. 1 Jamsetji Tata invested $4,000 cash in the Tata Company in exchange for common stock. 5 Tata Company purchased $2,000 of equipment on credit. 14 Tata Company provided $540 of services for a client on credit.

Solution

Recording Transactions

NEED-TO-KNOW 2-3

P1 A1

Jan. 1 Receive Investment by Owner

a analyze Assets = Liabilities + Equity Common Cash Stock +4,000 = 0 +4,000

b RecoRd Jan . 1 Cash 101 4,000 Common Stock 307 4,000

c Post

Jan . 1 4,000

Cash 101

Jan . 1 4,000

Common Stock 307Date Account Titles and Explanation PR Debit Credit

Jan. 5 Purchase Equipment on Credit

b RecoRd Jan . 5 Equipment 167 2,000 Accounts Payable 201 2,000

Jan . 5 2,000

Accounts Payable 201

c Post

Jan . 5 2,000

Equipment 167 a analyze Assets = Liabilities + Equity

Accounts Equipment Payable

+2,000 = +2,000 + 0

Date Account Titles and Explanation PR Debit Credit

Do More: QS 2-6, E 2-7, E 2-9, E 2-11, E 2-12

Jan. 14 Provide Services on Credit

a analyze Assets = Liabilities + Equity Accounts Services Receivable Revenue +540 = 0 +540

b RecoRd Jan . 14 Accounts Receivable 106 540 Services Revenue 403 540

Jan . 14 540

Accounts Receivable 106

Jan . 14 540

Services Revenue 403

c Post

Date Account Titles and Explanation PR Debit Credit

A trial balance is a list of all ledger accounts and their balances at a point in time. Exhibit 2.14 shows the trial balance for FastForward after its 16 entries are posted to the ledger. (This is an unadjusted trial balance. Chapter 3 explains adjustments.)

Preparing a Trial Balance Preparing a trial balance has three steps.

1. List each account title and its amount (from the ledger) in the trial balance. If an account has a zero balance, list it with a zero in its normal balance column (or omit it).

2. Compute the total of debit balances and the total of credit balances. 3. Verify (prove) total debit balances equal total credit balances.

TRIAL BALANCE P2 Prepare and explain the use of a trial balance.

Chapter 2 Accounting for Business Transactions 59

FASTFORWARD Trial Balance

December 31, 2019

Debit Credit

Cash . . . . . . . . . . . . . . . . . . . . . . . . $ 4,275

Accounts receivable . . . . . . . . . . . 0

Supplies . . . . . . . . . . . . . . . . . . . . . 9,720

Prepaid insurance . . . . . . . . . . . . . 2,400

Equipment . . . . . . . . . . . . . . . . . . . 26,000

Accounts payable . . . . . . . . . . . . . $ 6,200

Unearned consulting revenue . . . 3,000

Common stock . . . . . . . . . . . . . . . 30,000

Dividends . . . . . . . . . . . . . . . . . . . 200

Consulting revenue . . . . . . . . . . . 5,800

Rental revenue . . . . . . . . . . . . . . . 300

Salaries expense . . . . . . . . . . . . . . 1,400

Rent expense . . . . . . . . . . . . . . . . 1,000

Utilities expense . . . . . . . . . . . . . . 305

Totals . . . . . . . . . . . . . . . . . . . . . . . $45,300 $45,300

The total of debit balances equals the total of credit balances for the trial bal- ance in Exhibit 2.14. Equality of these two totals does not guarantee that no errors were made. For example, the column totals will be equal when a debit or credit of a correct amount is made to a wrong account. Another error not identified with a trial balance is when equal debits and credits of an incor- rect amount are entered.

Searching for Errors If the trial balance does not balance (when its columns are not equal), the error(s) must be found and corrected. An efficient way to search for an error is to check the journalizing, post- ing, and trial balance preparation in reverse order. Step 1 is to verify that the trial bal- ance columns are correctly added. If step 1 does not find the error, step 2 is to verify that account balances are accurately entered from the ledger. Step 3 is to see whether a debit (or credit) balance is mistakenly listed in the trial balance as a credit (or debit). A clue to this error is when the difference between total debits and total credits equals twice the amount of the incorrect account balance. Step 4 is to recompute each account balance in the ledger. Step 5 is to verify that each journal entry is properly posted. Step 6 is to verify that the original journal entry has equal debits and credits. At this point, the errors should be uncovered.

Point: A trial balance is not a financial statement but a tool for checking equality of debits and credits in the ledger.

EXHIBIT 2.14 Trial Balance (Unadjusted)

Example: If a credit to Unearned Revenue was incorrectly posted to the Revenue ledger account, would the ledger still balance? Answer: The ledger would bal- ance, but liabilities would be understated, equity would be overstated, and income would be overstated.

Accounting Quality Recording valid and accurate transactions enhances the quality of financial statements. Roughly 30% of employees in IT report observing misconduct such as falsifying accounting data. They also report increased incidences of such misconduct in recent years. Source: KPMG. ■

Ethical Risk

Financial Statements Prepared from Trial Balance Financial Statements across Time How financial statements are linked in time is shown in Exhibit 2.15. A balance sheet reports an organization’s financial position at a point in time. The income statement, statement of retained earnings, and statement of cash flows report financial performance over a period of time. The three statements in the middle column of Exhibit 2.15 explain how financial position changes from the beginning to the end of a reporting period.

A one-year (annual) reporting period is common, as are semiannual, quarterly, and monthly periods. The one-year reporting period is called the accounting, or fiscal, year. Businesses whose accounting year begins on January 1 and ends on December 31 are called calendar-year companies.

Financial Statement Preparation This section shows how to prepare financial statements from the trial balance. (These are unadjusted statements. Chapter 3 explains adjust- ments.) We prepare these statements in the following order.

Beginning Balance Sheet Ending Balance SheetIncome Statement

Statement of Cash Flows Statement of Retained Earnings

Cash $30,000 Liabilities $ 0 Other assets 0 Equity 30,000 Total assets $30,000 Total $30,000

Cash $ 4,275 Liabilities $ 9,200 Other assets 38,120 Equity 33,195 Total assets $42,395 Total $42,395

Expenses Net income

2,705 $3,395

Revenues $6,100

Point in time Point in timePeriod of time

EXHIBIT 2.15 Links between Financial Statements across Time

P3 Prepare financial statements from business transactions.

FASTForward

60 Chapter 2 Accounting for Business Transactions

1 Income Statement An income statement reports revenues earned minus expenses incurred over a period of time. FastForward’s income statement for December is shown at the top right side of Exhibit 2.16. Information about revenues and expenses is taken from the trial balance on the left side. Net income of $3,395 is the bottom line for the income statement. Owner investments and dividends are not part of income.

2 Statement of Retained Earnings The statement of retained earnings reports how retained earnings changes over the reporting period. FastForward’s statement of retained earnings is the second report in Exhibit 2.16. It shows the $3,395 of net income, the $200 divi- dend, and the $3,195 end-of-period balance. (The beginning balance in the statement of retained earnings is rarely zero, except in the first period of operations. The beginning balance in January 2020 is $3,195, which is December 2019’s ending balance.)

3 Balance Sheet The balance sheet reports the financial position of a company at a point in time. FastForward’s balance sheet is the third report in Exhibit 2.16. This statement shows financial condition at the close of business on December 31. The left side of the balance

Point: An income statement also is called an earnings statement, a statement of operations, or a P&L (profit and loss) statement. A balance sheet also is called a statement of financial position.

Point: Revenues and expenses are not reported in detail in the statement of retained earnings. Instead, their effects are reflected through net income.

EXHIBIT 2.16 Financial Statements Prepared from Trial Balance

FASTFORWARD Income Statement

For Month Ended December 31, 2019

Revenues Consulting revenue ($4,200 + $1,600) . . . . . . . . . . . . . $5,800 Rental revenue . . . . . . . . . . . . . . . . . . . . . . . . . . . . . . . . . 300

Total revenues . . . . . . . . . . . . . . . . . . . . . . . . . . . . . . . . . . $ 6,100

Expenses

Salaries expense . . . . . . . . . . . . . . . . . . . . . . . . . . . . . . . . 1,400

Rent expense . . . . . . . . . . . . . . . . . . . . . . . . . . . . . . . . . . 1,000

Utilities expense . . . . . . . . . . . . . . . . . . . . . . . . . . . . . . . . 305

Total expenses . . . . . . . . . . . . . . . . . . . . . . . . . . . . . . . . . 2,705

Net income . . . . . . . . . . . . . . . . . . . . . . . . . . . . . . . . . . . . . . $ 3,395

FASTFORWARD Statement of Retained Earnings

For Month Ended December 31, 2019

Retained earnings, December 1, 2019 . . . . . . . . . . . . . . . . . . . . . . . . . . . . $ 0

Plus: Net income . . . . . . . . . . . . . . . . . . . . . . . . . . . . . . . . . . . . . . . . . . . . 3,395 3,395

Less: Cash dividends . . . . . . . . . . . . . . . . . . . . . . . . . . . . . . . . . . . . . . . . . 200

Retained earnings, December 31, 2019 . . . . . . . . . . . . . . . . . . . . . . . . . . . $ 3,195

FASTFORWARD Balance Sheet

December 31, 2019

Assets Liabilities Cash . . . . . . . . . . . . . . $ 4,275 Accounts payable . . . . . . . . . . . . $ 6,200

Supplies . . . . . . . . . . . 9,720 Unearned consulting revenue . . . 3,000

Prepaid insurance . . . 2,400 Total liabilities . . . . . . . . . . . . . . . 9,200

Equipment . . . . . . . . . 26,000 Equity Common stock . . . . . . . . . . . . . . . 30,000

Retained earnings . . . . . . . . . . . . . 3,195 Total equity . . . . . . . . . . . . . . . . . . 33,195

Total assets $42,395 Total liabilities and equity . . . . . . . $42,395

FASTFORWARD Trial Balance

December 31, 2019

Debit Credit

Cash . . . . . . . . . . . . . . . . . . . . . . . . $ 4,275

Accounts receivable . . . . . . . . . . . 0

Supplies . . . . . . . . . . . . . . . . . . . . . 9,720

Prepaid insurance . . . . . . . . . . . . . 2,400

Equipment . . . . . . . . . . . . . . . . . . . 26,000

Accounts payable . . . . . . . . . . . . . $ 6,200

Unearned consulting revenue . . . 3,000

Common stock . . . . . . . . . . . . . . . 30,000

Dividends . . . . . . . . . . . . . . . . . . . 200

Consulting revenue . . . . . . . . . . . 5,800

Rental revenue . . . . . . . . . . . . . . . 300

Salaries expense . . . . . . . . . . . . . . 1,400

Rent expense . . . . . . . . . . . . . . . . 1,000

Utilities expense . . . . . . . . . . . . . . 305

Totals . . . . . . . . . . . . . . . . . . . . . . . $45,300 $45,300

Each account on the trial balance is either an asset (to balance sheet), liability (to balance sheet), or equity (to income statement or to statement of retained earnings) .

Point: A statement’s heading lists the 3 W’s: Who—name of organization, What—name of statement, When—point in time or period of time.

Point: Arrow lines show how the statements are linked.

Point: To foot a column of numbers is to add them.

Chapter 2 Accounting for Business Transactions 61

sheet lists its assets: cash, supplies, prepaid insurance, and equipment. The liabilities section of the balance sheet shows that it owes $6,200 to creditors and $3,000 in services to customers who paid in advance. The equity section shows an ending balance of $33,195. Note the link between the ending balance of the statement of retained earnings and the retained earnings bal- ance. (This presentation of the balance sheet is called the account form: assets on the left and liabilities and equity on the right. Another presentation is the report form: assets on top, fol- lowed by liabilities and then equity. Either presentation is acceptable.)

Entrepreneur You open a wholesale business selling entertainment equipment to retail outlets. Most of your cus- tomers want to buy on credit. How can you use the balance sheets of customers to decide which ones to extend credit to? ■ Answer: We use the accounting equation (Assets = Liabilities + Equity) to identify risky customers to whom we would not want to extend credit. A balance sheet provides amounts for each of these key components. The lower a customer’s equity is relative to liabilities, the less likely you would be to extend credit. A low equity means the business already has many creditor claims to it.

Decision Maker

©REDPIXEL.PL/Shutterstock

Presentation Issues Dollar signs are not used in journals and ledgers. They do appear in financial statements and other reports such as trial balances. We usually put dollar signs be- side only the first and last numbers in a column. Apple’s financial statements in Appendix A show this. Companies commonly round amounts in reports to the nearest dollar, or even to a higher level. Apple, like many large companies, rounds its financial statement amounts to the nearest million. This decision is based on the impact of rounding for users’ decisions.

Prepare a trial balance for Apple using the following condensed data from its recent fiscal year ended September 30 ($ in millions).

Preparing Trial Balance

NEED-TO-KNOW 2-4

P2Common stock . . . . . . . . . . . . . . . . . . . . . . . . . . $ 35,867 Accounts payable . . . . . . . . . . . . . . . . . . . . . . . . 49,049

Other liabilities . . . . . . . . . . . . . . . . . . . . . . . . . . 192,223

Cost of sales (and other expenses) . . . . . . . . . . 141,048

Cash . . . . . . . . . . . . . . . . . . . . . . . . . . . . . . . . . . . 20,289

Revenues . . . . . . . . . . . . . . . . . . . . . . . . . . . . . . . 229,234

Dividends . . . . . . . . . . . . . . . . . . . . . . . . . . . . . . . . . . $ 47,169

Investments and other assets . . . . . . . . . . . . . . . . . . 303,373

Land and equipment . . . . . . . . . . . . . . . . . . . . . . . . . 33,783

Selling and other expense . . . . . . . . . . . . . . . . . . . . 39,835

Accounts receivable . . . . . . . . . . . . . . . . . . . . . . . . . 17,874

Retained earnings, beginning fiscal year . . . . . . . . . 96,998

Solution ($ in millions)

APPLE

APPLE Trial Balance

September 30

Do More: E 2-8, E 2-10

Debit Credit

Cash . . . . . . . . . . . . . . . . . . . . . . . . . . . . . . . . . . . . . . . . . . . $ 20,289

Accounts receivable . . . . . . . . . . . . . . . . . . . . . . . . . . . . . . 17,874

Land and equipment . . . . . . . . . . . . . . . . . . . . . . . . . . . . . . 33,783

Investments and other assets . . . . . . . . . . . . . . . . . . . . . . 303,373

Accounts payable . . . . . . . . . . . . . . . . . . . . . . . . . . . . . . . . $ 49,049

Other liabilities . . . . . . . . . . . . . . . . . . . . . . . . . . . . . . . . . . 192,223

Common stock . . . . . . . . . . . . . . . . . . . . . . . . . . . . . . . . . . 35,867

Retained earnings, beginning fiscal year . . . . . . . . . . . . . . 96,998

Dividends . . . . . . . . . . . . . . . . . . . . . . . . . . . . . . . . . . . . . . 47,169

Revenues . . . . . . . . . . . . . . . . . . . . . . . . . . . . . . . . . . . . . . . 229,234

Cost of sales (and other expenses) . . . . . . . . . . . . . . . . . . 141,048

Selling and other expense . . . . . . . . . . . . . . . . . . . . . . . . . 39,835

Totals . . . . . . . . . . . . . . . . . . . . . . . . . . . . . . . . . . . . . . . . . $603,371 $603,371

It is important to assess a company’s risk of failing to pay its debts. Companies finance their assets with either liabilities or equity. A company that finances a relatively large portion of its assets with liabilities is said to have higher financial leverage. Higher financial leverage means greater risk because liabilities must be repaid and often require regular interest payments (equity financing does not). One measure of the risk associated with liabilities is the debt ratio as defined in Exhibit 2.17.

Costco’s total liabilities, total assets, and debt ratio for the past three years are shown in Exhibit 2.18. Costco’s debt ratio ranges from a low of 0.63 to a high of 0.70. Its ratio exceeds Walmart’s in each of the last three years, suggesting a higher than average risk from financial leverage. So, is financial leverage good or bad for Costco? The answer: If Costco is making more money with this debt than it is paying the lenders, then it is successfully borrowing money to make more money. A company’s use of debt can turn unprofitable quickly if its return from that money drops below the rate it is paying lenders.

This problem extends Need-to-Know 1-6 from Chapter 1: Jasmine Worthy started a haircutting business called Expressions. The following events occurred during its first month. Aug. 1 Worthy invested $3,000 cash and $15,000 of equipment in Expressions in exchange for com-

mon stock. 2 Expressions paid $600 cash for furniture for the shop. 3 Expressions paid $500 cash to rent space in a strip mall for August. 4 Expressions purchased $1,200 of equipment on credit for the shop (recorded as accounts pay-

able). 15 Expressions opened for business on August 5. Cash received from haircutting services in the

first week and a half of business (ended August 15) was $825. 16 Expressions provided $100 of haircutting services on account. 17 Expressions received a $100 check for services previously rendered on account. 18 Expressions paid $125 to an assistant for hours worked for the grand opening. 31 Cash received from services provided during the second half of August was $930. 31 Expressions paid $400 cash toward the account payable entered into on August 4. 31 Expressions paid a $900 cash dividend to Worthy (sole shareholder).

Required

1. Open the following ledger accounts in balance column format (account numbers are in parentheses): Cash (101); Accounts Receivable (102); Furniture (161); Store Equipment (165); Accounts Payable (201); Common Stock (307); Dividends (319); Haircutting Services Revenue (403); Wages Expense (623); and Rent Expense (640). Prepare general journal entries for the transactions.

COMPREHENSIVE

Journalizing and Posting Transactions, Statement Preparation, and Debt Ratio

NEED-TO-KNOW 2-5

EXHIBIT 2.17 Debt Ratio Debt ratio =

Total liabilities Total assets

62 Chapter 2 Accounting for Business Transactions

A2 Compute the debt ratio and describe its use in analyzing financial condition.

Debt RatioDecision Analysis

Investor You consider buying stock in Converse. As part of your analysis, you compute the company’s debt ratio for 2017, 2018, and 2019 as 0.35, 0.74, and 0.94, respectively. Based on the debt ratio, is Converse a low-risk investment? Has the risk of buying Converse stock changed over this period? (The industry debt ratio averages 0.40.) ■ Answer: The debt ratio suggests that Converse’s stock is of higher risk than normal and that this risk is rising. The average industry ratio of 0.40 supports this conclusion. The 2019 debt ratio for Converse is twice the industry norm. Also, a debt ratio approaching 1.0 indicates little to no equity.

Decision Maker

EXHIBIT 2.18 Computation and Analysis of Debt Ratio

Company ($ millions) Current Year 1 Year Ago 2 Years Ago

Costco Total liabilities . . . . . . . . . . . . . . . . . . . . . . . . . $25,268 $20,831 $22,174

Total assets . . . . . . . . . . . . . . . . . . . . . . . . . . . $36,347 $33,163 $33,017

Debt ratio . . . . . . . . . . . . . . . . . . . . . . . . . . . . 0.70 0.63 0.67

Walmart Debt ratio . . . . . . . . . . . . . . . . . . . . . . . . . . . . 0 .59 0 .58 0 .58

Chapter 2 Accounting for Business Transactions 63

2. Post the journal entries from part 1 to the ledger accounts. 3. Prepare a trial balance as of August 31. 4. Prepare an income statement for August. 5. Prepare a statement of retained earnings for August. 6. Prepare a balance sheet as of August 31. 7. Determine the debt ratio as of August 31.

Extended Analysis

8. In the coming months, Expressions will have a greater variety of business transactions. Identify which accounts are debited and which are credited for the following transactions. Hint: We must use some accounts not opened in part 1.

a. Purchase supplies with cash. b. Pay cash for future insurance coverage. c. Receive cash for services to be provided in the future. d. Purchase supplies on account.

PLANNING THE SOLUTION Analyze each transaction and use the debit and credit rules to prepare a journal entry for each. Post each debit and each credit from journal entries to their ledger accounts and cross-reference each

amount in the posting reference (PR) columns of the journal and ledger. Calculate each account balance and list the accounts with their balances on a trial balance. Verify that total debits in the trial balance equal total credits. To prepare the income statement, identify revenues and expenses. List those items on the statement,

compute the difference, and label the result as net income or net loss. Use information in the ledger to prepare the statement of retained earnings. Use information in the ledger to prepare the balance sheet. Calculate the debt ratio by dividing total liabilities by total assets. Analyze the future transactions to identify the accounts affected and apply debit and credit rules.

SOLUTION 1. General journal entries.

[continued on next page]

General Journal Date Account Titles and Explanation PR Debit Credit

Aug . 1 Cash . . . . . . . . . . . . . . . . . . . . . . . . . . . . . . . . . . . . . . . . . . . . . . . . . . . . 101 3,000

Store Equipment . . . . . . . . . . . . . . . . . . . . . . . . . . . . . . . . . . . . . . . . . . . 165 15,000

Common Stock . . . . . . . . . . . . . . . . . . . . . . . . . . . . . . . . . . . . . . . 307 18,000

Owner’s investment in exchange for stock.

2 Furniture . . . . . . . . . . . . . . . . . . . . . . . . . . . . . . . . . . . . . . . . . . . . . . . . . 161 600

Cash . . . . . . . . . . . . . . . . . . . . . . . . . . . . . . . . . . . . . . . . . . . . . . . . 101 600

Purchased furniture for cash.

3 Rent Expense . . . . . . . . . . . . . . . . . . . . . . . . . . . . . . . . . . . . . . . . . . . . . 640 500

Cash . . . . . . . . . . . . . . . . . . . . . . . . . . . . . . . . . . . . . . . . . . . . . . . . 101 500

Paid rent for August.

4 Store Equipment . . . . . . . . . . . . . . . . . . . . . . . . . . . . . . . . . . . . . . . . . . . 165 1,200

Accounts Payable . . . . . . . . . . . . . . . . . . . . . . . . . . . . . . . . . . . . . 201 1,200

Purchased additional equipment on credit.

15 Cash . . . . . . . . . . . . . . . . . . . . . . . . . . . . . . . . . . . . . . . . . . . . . . . . . . . . 101 825

Haircutting Services Revenue . . . . . . . . . . . . . . . . . . . . . . . . . . . . 403 825

Cash receipts from first half of August.

64 Chapter 2 Accounting for Business Transactions

16 Accounts Receivable . . . . . . . . . . . . . . . . . . . . . . . . . . . . . . . . . . . . . . . 102 100

Haircutting Services Revenue . . . . . . . . . . . . . . . . . . . . . . . . . . . . 403 100

Record revenue for services provided on account.

17 Cash . . . . . . . . . . . . . . . . . . . . . . . . . . . . . . . . . . . . . . . . . . . . . . . . . . . . 101 100

Accounts Receivable . . . . . . . . . . . . . . . . . . . . . . . . . . . . . . . . . . . 102 100

Record cash received as payment on account.

18 Wages Expense . . . . . . . . . . . . . . . . . . . . . . . . . . . . . . . . . . . . . . . . . . . 623 125

Cash . . . . . . . . . . . . . . . . . . . . . . . . . . . . . . . . . . . . . . . . . . . . . . . . 101 125

Paid wages to assistant.

31 Cash . . . . . . . . . . . . . . . . . . . . . . . . . . . . . . . . . . . . . . . . . . . . . . . . . . . . 101 930

Haircutting Services Revenue . . . . . . . . . . . . . . . . . . . . . . . . . . . . 403 930

Cash receipts from second half of August.

31 Accounts Payable . . . . . . . . . . . . . . . . . . . . . . . . . . . . . . . . . . . . . . . . . . 201 400

Cash . . . . . . . . . . . . . . . . . . . . . . . . . . . . . . . . . . . . . . . . . . . . . . . . 101 400

Paid cash toward accounts payable.

31 Dividends . . . . . . . . . . . . . . . . . . . . . . . . . . . . . . . . . . . . . . . . . . . . . . . . 319 900

Cash . . . . . . . . . . . . . . . . . . . . . . . . . . . . . . . . . . . . . . . . . . . . . . . . 101 900

Paid a cash dividend.

[continued from previous page]

2. Post journal entries from part 1 to the ledger accounts (in balance column format).

Cash Account No. 101

Date PR Debit Credit Balance

Aug . 1 G1 3,000 3,000

2 G1 600 2,400

3 G1 500 1,900

15 G1 825 2,725

17 G1 100 2,825

18 G1 125 2,700

31 G1 930 3,630

31 G1 400 3,230

31 G1 900 2,330

Accounts Receivable Account No. 102

Date PR Debit Credit Balance

Aug . 16 G1 100 100

17 G1 100 0

Furniture Account No. 161

Date PR Debit Credit Balance

Aug . 2 G1 600 600

Store Equipment Account No. 165

Date PR Debit Credit Balance

Aug . 1 G1 15,000 15,000

4 G1 1,200 16,200

Accounts Payable Account No. 201

Date PR Debit Credit Balance

Aug . 4 G1 1,200 1,200 31 G1 400 800

Common Stock Account No. 307

Date PR Debit Credit Balance

Aug . 1 G1 18,000 18,000

Dividends Account No. 319

Date PR Debit Credit Balance

Aug . 31 G1 900 900

Haircutting Services Revenue Account No. 403

Date PR Debit Credit Balance

Aug . 15 G1 825 825 16 G1 100 925 31 G1 930 1,855

Wages Expense Account No. 623

Date PR Debit Credit Balance

Aug . 18 G1 125 125

Rent Expense Account No. 640

Date PR Debit Credit Balance

Aug . 3 G1 500 500

General Ledger

Chapter 2 Accounting for Business Transactions 65

EXPRESSIONS Balance Sheet

August 31

Assets Liabilities Cash . . . . . . . . . . . . . . . . . . . . . $ 2,330 Accounts payable . . . . . . . . . . . . . . . . . . $ 800

Furniture . . . . . . . . . . . . . . . . . 600 Equity Store equipment . . . . . . . . . . . . 16,200 Common stock . . . . . . . . . . . . . . . . . . . . . 18,000

Retained earnings . . . . . . . . . . . . . . . . . . 330

Total equity . . . . . . . . . . . . . . . . . . . . . . . . 18,330

Total assets . . . . . . . . . . . . . . . . $19,130 Total liabilities and equity . . . . . . . . . . . . $19,130

6.

EXPRESSIONS Income Statement

For Month Ended August 31

Revenues

Haircutting services revenue . . . . . . . . . . . . $1,855

Operating expenses

Rent expense . . . . . . . . . . . . . . . . . . . . . . . . $500

Wages expense . . . . . . . . . . . . . . . . . . . . . . . 125

Total operating expenses . . . . . . . . . . . . . . . 625

Net income . . . . . . . . . . . . . . . . . . . . . . . . . . . . $1,230

4.

EXPRESSIONS Statement of Retained Earnings

For Month Ended August 31

Retained earnings, August 1 . . . . . . . . . . . . . . . . . . . $ 0

Plus: Net income . . . . . . . . . . . . . . . . . . . . . . . . . . . 1,230

1,230

Less: Cash dividends . . . . . . . . . . . . . . . . . . . . . . . . 900

Retained earnings, August 31 . . . . . . . . . . . . . . . . . . $ 330

5.

3. Prepare a trial balance from the ledger—see how it feeds the financial statements.

EXPRESSIONS Trial Balance

August 31

Debit Credit Cash . . . . . . . . . . . . . . . . . . . . . . . . . . . . . . . . $ 2,330

Accounts receivable . . . . . . . . . . . . . . . . . . . 0

Furniture . . . . . . . . . . . . . . . . . . . . . . . . . . . . 600

Store equipment . . . . . . . . . . . . . . . . . . . . . . 16,200

Accounts payable . . . . . . . . . . . . . . . . . . . . . $ 800

Common stock . . . . . . . . . . . . . . . . . . . . . . . 18,000

Dividends . . . . . . . . . . . . . . . . . . . . . . . . . . . . 900

Haircutting services revenue . . . . . . . . . . . . 1,855

Wages expense . . . . . . . . . . . . . . . . . . . . . . . 125

Rent expense . . . . . . . . . . . . . . . . . . . . . . . . 500

Totals . . . . . . . . . . . . . . . . . . . . . . . . . . . . . . . $20,655 $20,655

7. Debt ratio = Total liabilities

Total assets =

$800 $19,130

= 4.18%

8a. Supplies debited 8c. Cash debited Cash credited Unearned Services Revenue credited 8b. Prepaid Insurance debited 8d. Supplies debited Cash credited Accounts Payable credited

Account (45) Account balance (49) Balance column account (51) Chart of accounts (48) Compound journal entry (54) Credit (49) Creditors (47) Debit (49)

Debt ratio (62) Debtors (46) Dividends (48) Double-entry accounting (49) General journal (51) General ledger (45) Journal (51) Journalizing (51)

Ledger (45) Posting (51) Posting reference (PR) column (51) Source documents (45) T-account (49) Trial balance (58) Unearned revenue (47)

Key Terms

66 Chapter 2 Accounting for Business Transactions

SYSTEM OF ACCOUNTS Asset Accounts Cash: A company’s cash balance. Accounts receivable: Held by a seller; promises of payment from custom- ers to sellers. Accounts receivable are increased by credit sales; often phrased as sales on account or on credit. Note receivable: Held by a lender; a borrower’s written promise to pay the lender a specific sum of money on a specified future date. Prepaid accounts (or expenses): Assets that arise from prepayment of future expenses. Examples are prepaid insurance and prepaid rent. More assets: Supplies, equipment, buildings, and land. Liability Accounts Accounts payable: Held by a buyer; a buyer’s promise to pay a seller later for goods or services received. More generally, payables arise from pur- chases of merchandise for resale, supplies, services, and other items. Note payable: Held by a borrower; a written promissory note to pay a future amount at a future date. Unearned revenue: A liability to be settled in the future when a company delivers its products or services. When a customer pays in advance for products or services (before revenue is earned), the seller records this receipt as unearned revenue. Accrued liabilities: Amounts owed that are not yet paid. Examples are wages payable, taxes payable, and interest payable. Equity Accounts Common stock: When an owner invests in a company in exchange for stock, the company increases both assets and equity. Dividends: When a company pays dividends, it decreases both company assets and total equity. Revenue: Amounts received from sales of products and services to cus- tomers. Revenue increases equity. Expenses: Costs of providing products and services. Expenses decrease equity.

DEBITS AND CREDITS The left side of an account is called the debit side, or Dr. The right side is called the credit side, or Cr. Double-entry accounting transaction rules: ∙ At least two accounts are involved, with at least one debit and one credit. ∙ Total amount debited must equal total amount credited. Debits and credits in accounting equation:

= + – + – Dr. for

increases Cr. for

decreases

Assets Liabilities Dividends Revenues ExpensesCommon Stock

+ +– +– – +– –+ Dr. for

decreases Cr. for

increases

+ Dr. for

increases Cr. for

decreases Dr. for

decreases Cr. for

increases Dr. for

increases Cr. for

decreases Dr. for

decreases Cr. for

increases

Equity

Normal Normal Normal Normal Normal Normal

Net increases or decreases on one side have equal net effects on the other side. Left side is the normal balance side for assets. Right side is the normal balance side for liabilities and equity.

Summary: Cheat Sheet

RECORDING TRANSACTIONS Receive owner investment for stock: (1) Cash 101 30,000 Common Stock 307 30,000

Date Account Titles and Explanation PR Debit Credit

(2) Supplies 126 2,500 Cash 101 2,500

Date Account Titles and Explanation PR Debit Credit

Purchase supplies for cash:

(3) Equipment 167 26,000 Cash 101 26,000

Date Account Titles and Explanation PR Debit Credit

Purchase equipment for cash:

(4) Supplies 126 7,100 Accounts Payable 201 7,100

Date Account Titles and Explanation PR Debit Credit

Purchase supplies on credit:

Provide services for cash:

(5) Cash 101 4,200 Consulting Revenue 403 4,200

Date Account Titles and Explanation PR Debit Credit

FINANCIAL STATEMENTS

(6) Rent Expense 640 1,000 Cash 101 1,000

Date Account Titles and Explanation PR Debit Credit

(7) Salaries Expense 622 700 Cash 101 700

Date Account Titles and Explanation PR Debit Credit

(15) Utilities Expense 690 305 Cash 101 305

Date Account Titles and Explanation PR Debit Credit

Payment of expenses in cash:

(8) Accounts Receivable 106 1,900 Consulting Revenue 403 1,600 Rental Revenue 406 300

Date Account Titles and Explanation PR Debit Credit

Provide consulting and rental services on credit:

(9) Cash 101 1,900 Accounts Receivable 106 1,900

Date Account Titles and Explanation PR Debit Credit

Receipt of cash on account:

(10) Accounts Payable 201 900 Cash 101 900

Date Account Titles and Explanation PR Debit Credit

Partial payment of accounts payable:

(11) Dividends 319 200 Cash 101 200

Date Account Titles and Explanation PR Debit Credit

Payment of cash dividend:

(12) Cash 101 3,000 Unearned Consulting Revenue 236 3,000

Date Account Titles and Explanation PR Debit Credit

Receipt of cash for future services:

(13) Prepaid Insurance 128 2,400 Cash 101 2,400

Date Account Titles and Explanation PR Debit Credit

Pay cash for future insurance coverage:

FASTFORWARD Income Statement

For Month Ended December 31, 2019

Revenues Consulting revenue ($4,200 + $1,600) . . . . . . . . . . . . . $5,800 Rental revenue . . . . . . . . . . . . . . . . . . . . . . . . . . . . . . . . . 300

Total revenues . . . . . . . . . . . . . . . . . . . . . . . . . . . . . . . . . . $ 6,100

Expenses

Salaries expense . . . . . . . . . . . . . . . . . . . . . . . . . . . . . . . . 1,400

Rent expense . . . . . . . . . . . . . . . . . . . . . . . . . . . . . . . . . . 1,000

Utilities expense . . . . . . . . . . . . . . . . . . . . . . . . . . . . . . . . 305

Total expenses . . . . . . . . . . . . . . . . . . . . . . . . . . . . . . . . . 2,705

Net income . . . . . . . . . . . . . . . . . . . . . . . . . . . . . . . . . . . . . . $ 3,395

FASTFORWARD Statement of Retained Earnings

For Month Ended December 31, 2019

Retained earnings, December 1, 2019 . . . . . . . . . . . . . . . . . . . . . . . . . . . . $ 0

Plus: Net income . . . . . . . . . . . . . . . . . . . . . . . . . . . . . . . . . . . . . . . . . . . . 3,395 3,395

Less: Cash dividends . . . . . . . . . . . . . . . . . . . . . . . . . . . . . . . . . . . . . . . . . 200

Retained earnings, December 31, 2019 . . . . . . . . . . . . . . . . . . . . . . . . . . . $ 3,195

FASTFORWARD Balance Sheet

December 31, 2019

Assets Liabilities Cash . . . . . . . . . . . . . . $ 4,275 Accounts payable . . . . . . . . . . . . $ 6,200

Supplies . . . . . . . . . . . 9,720 Unearned consulting revenue . . . 3,000

Prepaid insurance . . . 2,400 Total liabilities . . . . . . . . . . . . . . . 9,200

Equipment . . . . . . . . . 26,000 Equity Common stock . . . . . . . . . . . . . . . 30,000

Retained earnings . . . . . . . . . . . . . 3,195 Total equity . . . . . . . . . . . . . . . . . . 33,195

Total assets $42,395 Total liabilities and equity . . . . . . . $42,395

FASTFORWARD Trial Balance

December 31, 2019

Debit Credit

Cash . . . . . . . . . . . . . . . . . . . . . . . . $ 4,275

Accounts receivable . . . . . . . . . . . 0

Supplies . . . . . . . . . . . . . . . . . . . . . 9,720

Prepaid insurance . . . . . . . . . . . . . 2,400

Equipment . . . . . . . . . . . . . . . . . . . 26,000

Accounts payable . . . . . . . . . . . . . $ 6,200

Unearned consulting revenue . . . 3,000

Common stock . . . . . . . . . . . . . . . 30,000

Dividends . . . . . . . . . . . . . . . . . . . 200

Consulting revenue . . . . . . . . . . . . 5,800

Rental revenue . . . . . . . . . . . . . . . 300

Salaries expense . . . . . . . . . . . . . . 1,400

Rent expense . . . . . . . . . . . . . . . . 1,000

Utilities expense . . . . . . . . . . . . . . 305

Totals . . . . . . . . . . . . . . . . . . . . . . . $45,300 $45,300

Each account on the trial balance is either an asset (to balance sheet), liability (to balance sheet), or equity (to income statement or to statement of retained earnings) .

Chapter 2 Accounting for Business Transactions 67

Multiple Choice Quiz

1. Amalia Company received its utility bill for the current pe- riod of $700 and immediately paid it. Its journal entry to record this transaction includes a a. Credit to Utility Expense for $700. b. Debit to Utility Expense for $700. c. Debit to Accounts Payable for $700. d. Debit to Cash for $700. e. Credit to Accounts Receivable for $700.

2. On May 1, Mattingly Lawn Service collected $2,500 cash from a customer in advance of five months of lawn ser- vice. Mattingly’s journal entry to record this transaction includes a a. Credit to Unearned Lawn Service Fees for $2,500. b. Debit to Lawn Service Fees Earned for $2,500. c. Credit to Cash for $2,500. d. Debit to Unearned Lawn Service Fees for $2,500. e. Credit to Accounts Payable for $2,500.

3. Liang Shue contributed $250,000 cash and land worth $500,000 to open his new business, Shue Consulting. Which of the following journal entries does Shue Consulting make to record this transaction? a. Cash Assets . . . . . . . . . . . . . 750,000 Common Stock . . . . . . . . 750,000 b. Common Stock . . . . . . . . . . 750,000 Assets. . . . . . . . . . . . . . . . 750,000

c. Cash . . . . . . . . . . . . . . . . . . . 250,000 Land . . . . . . . . . . . . . . . . . . . 500,000 Common Stock . . . . . . . . 750,000 d. Common Stock . . . . . . . . . . 750,000 Cash . . . . . . . . . . . . . . . . . 250,000 Land . . . . . . . . . . . . . . . . . 500,000

4. A trial balance prepared at year-end shows total credits ex- ceed total debits by $765. This discrepancy could have been caused by a. An error in the general journal where a $765 increase in

Accounts Payable was recorded as a $765 decrease in Accounts Payable.

b. The ledger balance for Accounts Payable of $7,650 be- ing entered in the trial balance as $765.

c. A general journal error where a $765 increase in Accounts Receivable was recorded as a $765 increase in Cash.

d. The ledger balance of $850 in Accounts Receivable was entered in the trial balance as $85.

e. An error in recording a $765 increase in Cash as a credit.

5. Bonaventure Company has total assets of $1,000,000, lia- bilities of $400,000, and equity of $600,000. What is its debt ratio (rounded to a whole percent)? a. 250% c. 67% e. 40% b. 167% d. 150%

ANSWERS TO MULTIPLE CHOICE QUIZ

1. b; debit Utility Expense for $700, and credit Cash for $700. 2. a; debit Cash for $2,500 and credit Unearned Lawn Service Fees

for $2,500. 3. c; debit Cash for $250,000, debit Land for $500,000, and credit

Common Stock for $750,000.

4. d 5. e; Debt ratio = $400,000/$1,000,000 = 40%

Icon denotes assignments that involve decision making.

1. Provide the names of two (a) asset accounts, (b) liability accounts, and (c) equity accounts.

2. What is the difference between a note payable and an ac- count payable?

3. Discuss the steps in processing business transactions. 4. What kinds of transactions can be recorded in a general

journal? 5. Are debits or credits typically listed first in general journal

entries? Are the debits or the credits indented? 6. Should a transaction be recorded first in a journal or the

ledger? Why? 7. If assets are valuable resources and asset accounts have debit

balances, why do expense accounts also have debit balances? 8. Why does the recordkeeper prepare a trial balance?

9. If an incorrect amount is journalized and posted to the ac- counts, how should the error be corrected?

10. Identify the four financial statements of a business. 11. What information is reported in a balance sheet? 12. What information is reported in an income statement? 13. Why does the user of an income statement need to

know the time period that it covers? 14. Define (a) assets, (b) liabilities, and (c) equity. 15. Which financial statement is sometimes called the state-

ment of financial position?

16. Review the Apple balance sheet in Appendix A. Identify three accounts on its bal- ance sheet that carry debit balances and three accounts on its balance sheet that carry credit balances.

Discussion Questions

APPLE

68 Chapter 2 Accounting for Business Transactions

17. Review the Google balance sheet in Appendix A. Identify an asset with the word receivable in its account title and a liability with the word payable in its account title.

18. Review the Samsung balance sheet in Appendix A. Identify three current liabili- ties and three noncurrent liabilities in its balance sheet.

SamsungGOOGLE

QUICK STUDY

QS 2-1 Identifying source documents C1

Identify the items from the following list that are likely to serve as source documents. a. Sales receipt d. Prepaid insurance account g. Income statement b. Trial balance e. Invoice from supplier h. Bank statement c. Balance sheet f. Company revenue account i. Telephone bill

QS 2-2 Identifying financial statement accounts

C2

Classify each of the following accounts as an asset (A), liability (L), or equity (EQ) account. a. Cash d. Prepaid Insurance g. Accounts Payable b. Prepaid Rent e. Office Equipment h. Unearned Rent Revenue c. Office Supplies f. Common Stock i. Dividends

QS 2-3 Reading a chart of accounts

C3

A chart of accounts is a list of all ledger accounts and an identification number for each. One example of a chart of accounts is near the end of the book on pages CA and CA-1. Using that chart, identify the fol- lowing accounts as either an asset (A), liability (L), equity (EQ), revenue (R), or expense (E) account, along with its identification number. a. Advertising Expense d. Machinery g. Notes Payable b. Rent Revenue e. Accounts Payable h. Common Stock c. Rent Receivable f. Furniture i. Utilities Expense

QS 2-4 Identifying normal balance

C4

Identify the normal balance (debit or credit) for each of the following accounts. a. Fees Earned (Revenues) d. Wages Expense g. Wages Payable b. Office Supplies e. Accounts Receivable h. Building c. Dividends f. Prepaid Rent i. Common Stock

QS 2-5 Linking debit or credit with normal balance

C4

Indicate whether a debit or credit decreases the normal balance of each of the following accounts. a. Interest Payable e. Common Stock i. Dividends b. Service Revenue f. Prepaid Insurance j. Unearned Revenue c. Salaries Expense g. Buildings k. Accounts Payable d. Accounts Receivable h. Interest Revenue l. Land

QS 2-6 Analyzing transactions and preparing journal entries

P1

For each transaction, (1) analyze the transaction using the accounting equation, (2) record the transaction in journal entry form, and (3) post the entry using T-accounts to represent ledger accounts. Use the follow- ing (partial) chart of accounts—account numbers in parentheses: Cash (101); Accounts Receivable (106); Office Supplies (124); Trucks (153); Equipment (167); Accounts Payable (201); Unearned Landscaping Revenue (236); Common Stock (307); Dividends (319); Landscaping Revenue (403); Wages Expense (601), and Landscaping Expense (696). a. On May 15, DeShawn Tyler opens a landscaping company called Elegant Lawns by investing $7,000

in cash along with equipment having a $3,000 value in exchange for common stock. b. On May 21, Elegant Lawns purchases office supplies on credit for $500. c. On May 25, Elegant Lawns receives $4,000 cash for performing landscaping services. d. On May 30, Elegant Lawns receives $1,000 cash in advance of providing landscaping services to

a customer.

Identify whether a debit or credit results in the indicated change for each of the following accounts. a. To increase Land f. To decrease Prepaid Rent b. To decrease Cash g. To increase Notes Payable c. To increase Fees Earned (Revenues) h. To decrease Accounts Receivable d. To increase Salaries Expense i. To increase Common Stock e. To decrease Unearned Revenue j. To increase Store Equipment

QS 2-7 Analyzing debit or credit by account

A1

Chapter 2 Accounting for Business Transactions 69

Indicate the financial statement on which each of the following items appears. Use I for income statement, E for statement of retained earnings, and B for balance sheet. a. Services Revenue e. Equipment i. Dividends b. Interest Payable f. Prepaid Insurance j. Office Supplies c. Accounts Receivable g. Buildings k. Interest Expense d. Salaries Expense h. Rental Revenue l. Insurance Expense

QS 2-9 Classifying accounts in financial statements

P3

Prepare general journal entries for the following transactions of Green Energy Company. Use the follow- ing (partial) chart of accounts: Cash; Accounts Receivable; Supplies; Accounts Payable; Consulting Revenue; and Utilities Expense.

May 1 The company billed a customer $2,000 in consulting revenue for sustainable proposals. 3 The company purchased $300 of energy-efficient supplies on credit. 9 The company collected $500 cash as partial payment of the May 1 consulting revenue. 20 The company paid $300 cash toward the payable for energy-efficient supplies. 31 The company paid $100 cash for May’s renewable energy utilities.

QS 2-11 Preparing journal entries

P1

Determine the ending balance of each of the following T-accounts. QS 2-10 Computing T-account balance

C4 Cash

100 50 300 60 20

Accounts Payable

2,000 8,000 2,700

Supplies

10,000 3,800 1,100

a. b. c.

Accounts Receivable

600 150 150 150 100

Wages Payable

700 700

Cash

11,000 4,500 800 6,000 100 1,300

d. e. f.

A trial balance has total debits of $20,000 and total credits of $24,500. Which one of the following errors would create this imbalance? Explain. a. A $2,250 debit to Utilities Expense in a journal entry was incorrectly posted to the ledger as a $2,250

credit, leaving the Utilities Expense account with a $3,000 debit balance. b. A $4,500 debit to Salaries Expense in a journal entry was incorrectly posted to the ledger as a $4,500

credit, leaving the Salaries Expense account with a $750 debit balance. c. A $2,250 credit to Consulting Fees Earned (Revenues) in a journal entry was incorrectly posted to the

ledger as a $2,250 debit, leaving the Consulting Fees Earned account with a $6,300 credit balance. d. A $2,250 debit posting to Accounts Receivable was posted mistakenly to Land. e. A $4,500 debit posting to Equipment was posted mistakenly to Cash. f. An entry debiting Cash and crediting Accounts Payable for $4,500 was mistakenly not posted.

QS 2-8 Identifying a posting error

P2

QS 2-12 Preparing an income statement

P3

Liu Zhang operates Lawson Consulting, which began operations on June 1. On June 30, the company’s records show the following selected accounts and amounts for the month of June. Prepare a June income statement for the business.

Cash . . . . . . . . . . . . . . . . . . . . $5,000

Accounts receivable . . . . . . . 4,500

Equipment . . . . . . . . . . . . . . . . 6,500

Accounts payable . . . . . . . . . . $ 3,000

Common stock . . . . . . . . . . . . 10,500

Dividends . . . . . . . . . . . . . . . . 1,500

Service revenue . . . . . . . . . $12,000

Rent expense . . . . . . . . . . . 2,000

Wages expense . . . . . . . . . 6,000

QS 2-13 Preparing a statement of retained earnings P3

Use the information in QS 2-12 to prepare a June statement of retained earnings for Lawson Consulting. The Retained Earnings account balance at June 1 was $0. Hint: Net income for June is $4,000.

70 Chapter 2 Accounting for Business Transactions

QS 2-14 Preparing a balance sheet P3

Use the information in QS 2-12 and QS 2-13 to prepare a June 30 balance sheet for Lawson Consulting. Hint: The ending Retained Earnings account balance as of June 30 is $2,500.

Exercise 2-2 Identifying and classifying accounts

C2

Enter the number for the item that best completes each of the descriptions below. 1. Asset 2. Equity 3. Account 4. Liability 5. Three a. Balance sheet accounts are arranged into general categories. b. Common Stock and Dividends are examples of accounts. c. Accounts Payable and Note Payable are examples of accounts. d. Accounts Receivable, Prepaid Accounts, Supplies, and Land are examples of accounts. e. A(n) is a record of increases and decreases in a specific asset, liability, equity, revenue, or

expense item.

Exercise 2-3 Identifying a ledger and chart of accounts

C3

Enter the number for the item that best completes each of the descriptions below. 1. Chart 2. General ledger 3. Journal 4. Account 5. Source document a. A(n) of accounts is a list of all accounts a company uses, not including account balances. b. The is a record containing all accounts used by a company, including account balances. c. A(n) describes transactions entering an accounting system, such as a purchase order. d. Increases and decreases in a specific asset, liability, equity, revenue, or expense are recorded in

a(n) . e. A(n) has a complete record of every transaction recorded.

Exercise 2-4 Identifying type and normal balances of accounts

C4

For each of the following, (1) identify the type of account as an asset, liability, equity, revenue, or expense; (2) identify the normal balance of the account; and (3) enter debit (Dr.) or credit (Cr.) to identify the kind of entry that would increase the account balance. a. Land e. Accounts Receivable i. Fees Earned b. Cash f. Dividends j. Equipment c. Legal Expense g. License Fee Revenue k. Notes Payable d. Prepaid Insurance h. Unearned Revenue l. Common Stock

QS 2-15 Computing and using the debt ratio A2

In a recent year’s financial statements, Home Depot reported the following: Total liabilities = $38,633 million and Total assets = $42,966 million. Compute and interpret Home Depot’s debt ratio (assume com- petitors average a 60.0% debt ratio).

EXERCISES

Exercise 2-1 Steps in analyzing and recording transactions C1

Order the following steps in the accounting process that focus on analyzing and recording transactions. a. Prepare and analyze the trial balance. b. Analyze each transaction from source documents. c. Record relevant transactions in a journal. d. Post journal information to ledger accounts.

Groro Co. bills a client $62,000 for services provided and agrees to accept the following three items in full payment: (1) $10,000 cash, (2) equipment worth $80,000, and (3) to assume responsibility for a $28,000 note payable related to the equipment. For this transaction, (a) analyze the transaction using the account- ing equation, (b) record the transaction in journal entry form, and (c) post the entry using T-accounts to represent ledger accounts. Use the following (partial) chart of accounts—account numbers in parentheses: Cash (101); Supplies (124); Equipment (167); Accounts Payable (201); Note Payable (245); Common Stock (307); and Revenue (404).

Exercise 2-5 Analyzing effects of a compound entry

A1

Use the information in each of the following separate cases to calculate the unknown amount. a. Corentine Co. had $152,000 of accounts payable on September 30 and $132,500 on October 31. Total

purchases on account during October were $281,000. Determine how much cash was paid on accounts payable during October.

b. On September 30, Valerian Co. had a $102,500 balance in Accounts Receivable. During October, the company collected $102,890 from its credit customers. The October 31 balance in Accounts Receivable was $89,000. Determine the amount of sales on account that occurred in October.

Exercise 2-6 Analyzing account entries and balances

A1

[continued on next page]

Chapter 2 Accounting for Business Transactions 71

Prepare general journal entries for the following transactions of a new company called Pose-for-Pics. Use the following (partial) chart of accounts: Cash; Office Supplies; Prepaid Insurance; Photography Equipment; Common Stock; Photography Fees Earned; and Utilities Expense.

Aug. 1 Madison Harris, the owner, invested $6,500 cash and $33,500 of photography equipment in the company in exchange for common stock.

2 The company paid $2,100 cash for an insurance policy covering the next 24 months. 5 The company purchased office supplies for $880 cash. 20 The company received $3,331 cash in photography fees earned. 31 The company paid $675 cash for August utilities.

Exercise 2-7 Preparing general journal entries

P1

Use the information in Exercise 2-7 to prepare a trial balance for Pose-for-Pics. Begin by opening these T-accounts: Cash; Office Supplies; Prepaid Insurance; Photography Equipment; Common Stock; Photography Fees Earned; and Utilities Expense. Then, (1) post the general journal entries to these T-accounts (which will serve as the ledger) and (2) prepare the August 31 trial balance.

Exercise 2-8 Preparing T-accounts (ledger) and a trial balance P2

Prepare general journal entries to record the transactions below for Spade Company by using the follow- ing accounts: Cash; Accounts Receivable; Office Supplies; Office Equipment; Accounts Payable; Common Stock; Dividends; Fees Earned; and Rent Expense. Use the letters beside each transaction to identify entries. After recording the transactions, post them to T-accounts, which serve as the general ledger for this assignment. Determine the ending balance of each T-account. a. Kacy Spade, owner, invested $100,750 cash in the company in exchange for common stock. b. The company purchased office supplies for $1,250 cash. c. The company purchased $10,050 of office equipment on credit. d. The company received $15,500 cash as fees for services provided to a customer. e. The company paid $10,050 cash to settle the payable for the office equipment purchased in

transaction c. f. The company billed a customer $2,700 as fees for services provided. g. The company paid $1,225 cash for the monthly rent. h. The company collected $1,125 cash as partial payment for the account receivable created in

transaction f. i. The company paid a $10,000 cash dividend to the owner (sole shareholder).

Exercise 2-9 Recording effects of transactions in T-accounts

A1

Check Cash ending balance, $94,850

After recording the transactions of Exercise 2-9 in T-accounts and calculating the balance of each account, prepare a trial balance. Use May 31 as its report date.

Exercise 2-10 Preparing a trial balance P2

c. During October, Alameda Company had $102,500 of cash receipts and $103,150 of cash disburse- ments. The October 31 Cash balance was $18,600. Determine how much cash the company had at the close of business on September 30.

1. Prepare general journal entries for the following transactions of Valdez Services. a. The company paid $2,000 cash for payment on a 6-month-old account payable for office supplies. b. The company paid $1,200 cash for the just completed two-week salary of the receptionist. c. The company paid $39,000 cash for equipment purchased. d. The company paid $800 cash for this month’s utilities. e. The company paid a $4,500 cash dividend to the owner (sole shareholder).

2. Transactions a, c, and e did not result in an expense. Match each transaction (a, c, and e) with one of the following reasons for not recording an expense.

This transaction is a distribution of cash to the owner. Even though equity decreased, that decrease did not occur in the process of providing goods or services to customers. This transaction decreased cash in settlement of a previously existing liability (equity did not change). Supplies expense is recorded when assets are used, not necessarily when cash is paid. This transaction involves the purchase of an asset. The form of the company’s assets changed, but total assets did not (and neither did equity).

Exercise 2-11 Analyzing and journalizing transactions involving cash payments

P1

72 Chapter 2 Accounting for Business Transactions

1. Prepare general journal entries for the following transactions of Valdez Services. a. Brina Valdez invested $20,000 cash in the company in exchange for common stock. b. The company provided services to a client and immediately received $900 cash. c. The company received $10,000 cash from a client in payment for services to be provided next year. d. The company received $3,500 cash from a client in partial payment of accounts receivable. e. The company borrowed $5,000 cash from the bank by signing a note payable.

2. Transactions a, c, d, and e did not yield revenue. Match each transaction (a, c, d, and e) with one of the following reasons for not recording revenue.

This transaction changed the form of an asset from a receivable to cash. Total assets were not increased (revenue was recognized when the services were originally provided). This transaction brought in cash (increased assets), and it also increased a liability by the same amount (represented by the signing of a note to repay the amount). This transaction brought in cash, but this is an owner investment. This transaction brought in cash, but it created a liability to provide services to the client in the next year.

Exercise 2-12 Analyzing and journalizing transactions involving receipt of cash

P1

Fill in each of the following T-accounts for Belle Co.’s seven transactions listed here. The T-accounts repre- sent Belle Co.’s general ledger. Code each entry with transaction number 1 through 7 (in order) for reference. 1. D. Belle created a new business and invested $6,000 cash, $7,600 of equipment, and $12,000 in web servers

in exchange for common stock. 2. The company paid $4,800 cash in advance for prepaid insurance coverage. 3. The company purchased $900 of supplies on account. 4. The company paid $800 cash for selling expenses. 5. The company received $4,500 cash for services provided. 6. The company paid $900 cash toward accounts payable. 7. The company paid $3,400 cash for equipment.

Exercise 2-13 Entering transactions into T-accounts

A1

Cash

Equipment

Common Stock

Supplies

Web Servers

Services Revenue

Prepaid Insurance

Accounts Payable

Selling Expenses

Exercise 2-14 Preparing general journal entries P1

Use information from Exercise 2-13 to prepare the general journal entries for Belle Co.’s first seven transactions.

Determine net income or net loss for the business during the year for each of the following separate cases. a. Owner made no investments in the business, and no dividends were paid during the year. b. Owner made no investments in the business, but dividends were $1,250 cash per month. c. No dividends were paid during the year, but the owner did invest an additional $55,000 cash in

exchange for common stock. d. Dividends were $1,250 cash per month, and the owner invested an additional $35,000 cash in exchange

for common stock.

A corporation had the following assets and liabilities at the beginning and end of this year.

Assets Liabilities

Beginning of the year . . . . . . . . . . . . $ 60,000 $20,000

End of the year . . . . . . . . . . . . . . . . . 105,000 36,000

Exercise 2-15 Computing net income

A1

Carmen Camry operates a consulting firm called Help Today, which began operations on August 1. On August 31, the company’s records show the following selected accounts and amounts for the month of August. Use this information to prepare an August income statement for the business.

Exercise 2-16 Preparing an income statement C3 P3

Chapter 2 Accounting for Business Transactions 73

Use the information in Exercise 2-16 to prepare an August statement of retained earnings for Help Today. The Retained Earnings account balance at August 1 was $0. Hint: Net income for August is $10,470.

Exercise 2-17 Preparing a statement of retained earnings P3

Use the information in Exercise 2-16 to prepare an August 31 balance sheet for Help Today. Hint: The ending Retained Earnings account balance as of August 31 is $4,470.

Exercise 2-18 Preparing a balance sheet P3

Compute the missing amount for each of the following separate companies in columns B through E. Exercise 2-19 Analyzing changes in a company’s equity

P3

110,000 $ 0

? 22,000

104,000

? $ 0

(47,000) 90,000 85,000

87,000 $ 0

(10,000) (4,000)

?

210,000 $ 0

(55,000) ?

110,000

2 1

3 4 5 6

A B C D E

CBS ABC CNN NBC

Equity, beginning of year

Equity, end of year

Owner investments during the year Dividends during the year Net income (loss) for the year

Check Net income, $10,470

Cash . . . . . . . . . . . . . . . . . . $25,360 Accounts receivable . . . . . . 22,360 Office supplies . . . . . . . . . . 5,250 Land . . . . . . . . . . . . . . . . . . 44,000 Office equipment . . . . . . . . 20,000

Accounts payable . . . . . . . . . . . $ 10,500 Common stock . . . . . . . . . . . . . 102,000 Dividends . . . . . . . . . . . . . . . . . 6,000 Consulting fees earned . . . . . . 27,000 Rent expense . . . . . . . . . . . . . . 9,550

Salaries expense . . . . . . . . . . $5,600 Telephone expense . . . . . . . . 860 Miscellaneous expenses . . . . 520

Posting errors are identified in the following table. In column (1), enter the amount of the difference be- tween the two trial balance columns (debit and credit) due to the error. In column (2), identify the trial balance column (debit or credit) with the larger amount if they are not equal. In column (3), identify the account(s) affected by the error. In column (4), indicate the amount by which the account(s) in column (3) is under- or overstated. Item (a) is completed as an example.

Exercise 2-20 Identifying effects of posting errors on the trial balance A1 P2

Exercise 2-21 Analyzing a trial balance error

P1 P2

You are told the column totals in a trial balance are not equal. After careful analysis, you discover only one error. Specifically, a correctly journalized credit purchase of an automobile for $18,950 is posted from the journal to the ledger with an $18,950 debit to Automobiles and another $18,950 debit to Accounts Payable. The Automobiles account has a debit balance of $37,100 on the trial balance. (1) Answer each of the following questions and (2) compute the dollar amount of any misstatement for parts a through d. a. Is the Debit column total of the trial balance overstated, understated, or correctly stated? b. Is the Credit column total of the trial balance overstated, understated, or correctly stated? c. Is the Automobiles account balance overstated, understated, or correctly stated in the trial balance? d. Is the Accounts Payable account balance overstated, understated, or correctly stated in the trial balance? e. If the Debit column total of the trial balance is $200,000 before correcting the error, what is the total

of the Credit column before correction?

(1) (2) (3) (4) Difference between Column with Identify Amount That Debit and Credit the Larger Account(s) Account(s) Is Over- or Description of Posting Error Columns Total Incorrectly Stated Understated

a . $3,600 debit to Rent Expense is $2,260 Credit Rent Expense Rent Expense posted as a $1,340 debit . understated $2,260

b . $6,500 credit to Cash is posted twice as two credits to Cash .

c . $10,900 debit to the Dividends account is debited to Common Stock .

d . $2,050 debit to Prepaid Insurance is posted as a debit to Insurance Expense .

e . $38,000 debit to Machinery is posted as a debit to Accounts Payable .

f . $5,850 credit to Services Revenue is posted as a $585 credit .

g . $1,390 debit to Store Supplies is not posted .

74 Chapter 2 Accounting for Business Transactions

Exercise 2-23 Preparing journal entries

P1

Prepare general journal entries for the following transactions of Sustain Company. Use the following (par- tial) chart of accounts: Cash; Prepaid Insurance; Accounts Receivable; Furniture; Accounts Payable; Unearned Revenue; Fees Earned; and Common Stock.

June 1 T. James, owner, invested $11,000 cash in Sustain Company in exchange for common stock. 2 The company purchased $4,000 of furniture made from reclaimed wood on credit. 3 The company paid $600 cash for a 12-month insurance policy on the reclaimed furniture. 4 The company billed a customer $3,000 in fees earned from preparing a sustainability report. 12 The company paid $4,000 cash toward the payable from the June 2 furniture purchase. 20 The company collected $3,000 cash for fees billed on June 4. 21 T. James invested an additional $10,000 cash in Sustain Company in exchange for common stock. 30 The company received $5,000 cash in advance of providing sustainability services to a customer.

Exercise 2-22 Calculating and interpreting the debt ratio

A2

a. Compute the debt ratio for each of the three companies. b. Which company has the most financial leverage?

67,000 $22,000

12,000 150,000

$ 40,000

68,000 27,000

$19,000

5,000 147,000

$ 30,000

17,000

ExpensesCompany Total Assets Net Income Total Liabilities

DreamWorks Pixar Universal

Aracel Engineering completed the following transactions in the month of June. a. Jenna Aracel, the owner, invested $100,000 cash, office equipment with a value of $5,000, and

$60,000 of drafting equipment to launch the company in exchange for common stock. b. The company purchased land worth $49,000 for an office by paying $6,300 cash and signing a long-

term note payable for $42,700. c. The company purchased a portable building with $55,000 cash and moved it onto the land acquired in b. d. The company paid $3,000 cash for the premium on an 18-month insurance policy. e. The company completed and delivered a set of plans for a client and collected $6,200 cash. f. The company purchased $20,000 of additional drafting equipment by paying $9,500 cash and signing

a long-term note payable for $10,500.

Problem 2-2A Preparing and posting journal entries; preparing a trial balance

C3 C4 A1 P1 P2

PROBLEM SET A

Problem 2-1A Preparing and posting journal entries; preparing a trial balance

C3 C4 A1 P1 P2

Karla Tanner opened a web consulting business called Linkworks and completed the following transac- tions in its first month of operations.

Apr. 1 Tanner invested $80,000 cash along with office equipment valued at $26,000 in the company in exchange for common stock.

2 The company prepaid $9,000 cash for 12 months’ rent for office space. Hint: Debit Prepaid Rent for $9,000.

3 The company made credit purchases for $8,000 in office equipment and $3,600 in office sup- plies. Payment is due within 10 days.

6 The company completed services for a client and immediately received $4,000 cash. 9 The company completed a $6,000 project for a client, who must pay within 30 days. 13 The company paid $11,600 cash to settle the account payable created on April 3. 19 The company paid $2,400 cash for the premium on a 12-month insurance policy. Hint: Debit

Prepaid Insurance for $2,400. 22 The company received $4,400 cash as partial payment for the work completed on April 9. 25 The company completed work for another client for $2,890 on credit. 28 The company paid a $5,500 cash dividend. 29 The company purchased $600 of additional office supplies on credit. 30 The company paid $435 cash for this month’s utility bill.

Required

1. Prepare general journal entries to record these transactions (use account titles listed in part 2). 2. Open the following ledger accounts—their account numbers are in parentheses (use the balance col-

umn format): Cash (101); Accounts Receivable (106); Office Supplies (124); Prepaid Insurance (128); Prepaid Rent (131); Office Equipment (163); Accounts Payable (201); Common Stock (307); Dividends (319); Services Revenue (403); and Utilities Expense (690). Post journal entries from part 1 to the ledger accounts and enter the balance after each posting.

3. Prepare a trial balance as of April 30.

Check (2) Ending balances: Cash, $59,465; Accounts Receivable, $4,490; Accounts Payable, $600

(3) Total debits, $119,490

Chapter 2 Accounting for Business Transactions 75

Denzel Brooks opened a web consulting business called Venture Consultants and completed the following transactions in March.

Mar. 1 Brooks invested $150,000 cash along with $22,000 in office equipment in the company in exchange for common stock.

2 The company prepaid $6,000 cash for six months’ rent for an office. Hint: Debit Prepaid Rent for $6,000.

3 The company made credit purchases of office equipment for $3,000 and office supplies for $1,200. Payment is due within 10 days.

6 The company completed services for a client and immediately received $4,000 cash. 9 The company completed a $7,500 project for a client, who must pay within 30 days. 12 The company paid $4,200 cash to settle the account payable created on March 3. 19 The company paid $5,000 cash for the premium on a 12-month insurance policy. Hint: Debit

Prepaid Insurance for $5,000. 22 The company received $3,500 cash as partial payment for the work completed on March 9. 25 The company completed work for another client for $3,820 on credit. 29 The company paid a $5,100 cash dividend. 30 The company purchased $600 of additional office supplies on credit. 31 The company paid $500 cash for this month’s utility bill.

Required

1. Prepare general journal entries to record these transactions (use the account titles listed in part 2). 2. Open the following ledger accounts—their account numbers are in parentheses (use the balance col-

umn format): Cash (101); Accounts Receivable (106); Office Supplies (124); Prepaid Insurance (128); Prepaid Rent (131); Office Equipment (163); Accounts Payable (201); Common Stock (307); Dividends (319); Services Revenue (403); and Utilities Expense (690). Post the journal entries from part 1 to the ledger accounts and enter the balance after each posting.

3. Prepare a trial balance as of the end of March.

Check (2) Ending balances: Cash, $136,700; Accounts Receivable, $7,820; Accounts Payable, $600

(3) Total debits, $187,920

Problem 2-3A Preparing and posting journal entries; preparing a trial balance

C3 C4 A1 P1 P2

g. The company completed $14,000 of engineering services for a client. This amount is to be received in 30 days.

h. The company purchased $1,150 of additional office equipment on credit. i. The company completed engineering services for $22,000 on credit. j. The company received a bill for rent of equipment that was used on a recently completed job. The

$1,333 rent cost must be paid within 30 days. k. The company collected $7,000 cash in partial payment from the client described in transaction g. l. The company paid $1,200 cash for wages to a drafting assistant. m. The company paid $1,150 cash to settle the account payable created in transaction h. n. The company paid $925 cash for minor maintenance of its drafting equipment. o. The company paid a $9,480 cash dividend. p. The company paid $1,200 cash for wages to a drafting assistant. q. The company paid $2,500 cash for advertisements on the web during June.

Required

1. Prepare general journal entries to record these transactions (use the account titles listed in part 2). 2. Open the following ledger accounts—their account numbers are in parentheses (use the balance col-

umn format): Cash (101); Accounts Receivable (106); Prepaid Insurance (108); Office Equipment (163); Drafting Equipment (164); Building (170); Land (172); Accounts Payable (201); Notes Payable (250); Common Stock (307); Dividends (319); Engineering Fees Earned (402); Wages Expense (601); Equipment Rental Expense (602); Advertising Expense (603); and Repairs Expense (604). Post the journal entries from part 1 to the accounts and enter the balance after each posting.

3. Prepare a trial balance as of the end of June.

Check (2) Ending balances: Cash, $22,945; Accounts Receivable, $29,000; Accounts Payable, $1,333

(3) Trial balance totals, $261,733

Problem 2-4A Recording transactions; posting to ledger; preparing a trial balance

C3 A1 P1 P2

Business transactions completed by Hannah Venedict during the month of September are as follows. a. Venedict invested $60,000 cash along with office equipment valued at $25,000 in a new business

named HV Consulting in exchange for common stock. b. The company purchased land valued at $40,000 and a building valued at $160,000. The purchase is

paid with $30,000 cash and a long-term note payable for $170,000. c. The company purchased $2,000 of office supplies on credit. [continued on next page]

76 Chapter 2 Accounting for Business Transactions

Problem 2-5A Computing net income from equity analysis, preparing a balance sheet, and computing the debt ratio

C2 A1 A2 P3

The accounting records of Nettle Distribution show the following assets and liabilities as of December 31, 2018 and 2019.

December 31 2018 2019

Cash . . . . . . . . . . . . . . . . . . . . . . . . $ 64,300 $ 15,640

Accounts receivable . . . . . . . . . . . . 26,240 19,100

Office supplies . . . . . . . . . . . . . . . . 3,160 1,960

Office equipment . . . . . . . . . . . . . . 44,000 44,000

Trucks . . . . . . . . . . . . . . . . . . . . . . . 148,000 157,000

December 31 2018 2019

Building . . . . . . . . . . . . . . . . . . . . . . $ 0 $80,000

Land . . . . . . . . . . . . . . . . . . . . . . . . 0 60,000

Accounts payable . . . . . . . . . . . . . . 3,500 33,500

Note payable . . . . . . . . . . . . . . . . . 0 40,000

Required

1. Prepare balance sheets for the business as of December 31, 2018 and 2019. Hint: Report only total equity on the balance sheet and remember that total equity equals the difference between assets and liabilities.

2. Compute net income for 2019 by comparing total equity amounts for these two years and using the following information: During 2019, the owner invested $35,000 additional cash in the business (in exchange for common stock) and the company paid a $19,000 cash dividend.

3. Compute the 2019 year-end debt ratio (in percent and rounded to one decimal).

Check (2) Net income, $6,000

(3) Debt ratio, 19.5%

Yi Min started an engineering firm called Min Engineering. He began operations and completed seven transactions in May, which included his initial investment of $18,000 cash. After those seven transactions, the ledger included the following accounts with normal balances.

Problem 2-6A Analyzing account balances and reconstructing transactions

C1 C3 A1 P2 Cash . . . . . . . . . . . . . . . . . $37,600 Office supplies . . . . . . . . . 890

Prepaid insurance . . . . . . . 4,600

Office equipment . . . . . . . $12,900

Accounts payable . . . . . . . 12,900

Common stock . . . . . . . . . 18,000

Dividends . . . . . . . . . . . . . . . . . . . $ 3,370

Engineering fees earned . . . . . . . 36,000

Rent expense . . . . . . . . . . . . . . . . 7,540

Required

1. Prepare a trial balance for this business as of the end of May. 2. The following seven transactions produced the account balances shown above. a. Y. Min invested $18,000 cash in the business in exchange for common stock. b. Paid $7,540 cash for monthly rent expense for May. c. Paid $4,600 cash in advance for the annual insurance premium beginning the next period.

Check (1) Trial balance totals, $66,900 (2) Ending Cash balance, $37,600

Check (2) Ending balances: Cash, $12,665; Office Equipment, $50,900

(3) Trial balance totals, $291,350

d. Venedict invested her personal automobile in the company in exchange for more common stock. The automobile has a value of $16,500 and is to be used exclusively in the business.

e. The company purchased $5,600 of additional office equipment on credit. f. The company paid $1,800 cash salary to an assistant. g. The company provided services to a client and collected $8,000 cash. h. The company paid $635 cash for this month’s utilities. i. The company paid $2,000 cash to settle the account payable created in transaction c. j. The company purchased $20,300 of new office equipment by paying $20,300 cash. k. The company completed $6,250 of services for a client, who must pay within 30 days. l. The company paid $1,800 cash salary to an assistant. m. The company received $4,000 cash in partial payment on the receivable created in transaction k. n. The company paid a $2,800 cash dividend.

Required

1. Prepare general journal entries to record these transactions (use account titles listed in part 2). 2. Open the following ledger accounts—their account numbers are in parentheses (use the balance col-

umn format): Cash (101); Accounts Receivable (106); Office Supplies (108); Office Equipment (163); Automobiles (164); Building (170); Land (172); Accounts Payable (201); Notes Payable (250); Common Stock (307); Dividends (319); Fees Earned (402); Salaries Expense (601); and Utilities Expense (602). Post the journal entries from part 1 to the ledger accounts and enter the balance after each posting.

3. Prepare a trial balance as of the end of September.

[continued from previous page]

Chapter 2 Accounting for Business Transactions 77

Angela Lopez owns and manages a consulting firm called Metrix, which began operations on March 1. On March 31, Metrix shows the following selected accounts and amounts for the month of March.

Problem 2-7A Preparing an income statement, statement of retained earnings, and balance sheet

P3

Equipment . . . . . . . . . . . . . . $ 4,000

Salaries expense . . . . . . . . 3,000

Consulting revenue . . . . . . 12,000

Cash . . . . . . . . . . . . . . . . . . 8,000

Utilities expense . . . . . . . . . 200

Note payable . . . . . . . . . . . 2,400

Accounts receivable . . . . . . . . $ 3,500

Common stock . . . . . . . . . . . . 11,600

Dividends . . . . . . . . . . . . . . . . 2,000

Office supplies . . . . . . . . . . . . 1,500

Rental revenue . . . . . . . . . . . . 500

Advertising expense . . . . . . . . 400

Prepaid insurance . . . . . . . $1,000

Accounts payable . . . . . . . 1,300

Note receivable . . . . . . . . 2,500

Rent expense . . . . . . . . . . 2,000

Unearned revenue . . . . . . 300

Required

1. Prepare a March income statement for the business. 2. Prepare a March statement of retained earnings. The Retained Earnings account balance at March 1 was

$0, and the owner invested $11,600 cash in the company on March 2 in exchange for common stock. 3. Prepare a March 31 balance sheet. Hint: Use the Retained Earnings account balance calculated in part 2.

PROBLEM SET B

Problem 2-1B Preparing and posting journal entries; preparing a trial balance

C3 C4 A1 P1 P2

Humble Management Services opened for business and completed these transactions in September.

Sep. 1 Henry Humble, the owner, invested $38,000 cash along with office equipment valued at $15,000 in the company in exchange for common stock.

2 The company prepaid $9,000 cash for 12 months’ rent for office space. Hint: Debit Prepaid Rent for $9,000.

4 The company made credit purchases for $8,000 in office equipment and $2,400 in office sup- plies. Payment is due within 10 days.

8 The company completed work for a client and immediately received $3,280 cash. 12 The company completed a $15,400 project for a client, who must pay within 30 days. 13 The company paid $10,400 cash to settle the payable created on September 4. 19 The company paid $1,900 cash for the premium on an 18-month insurance policy. Hint: Debit

Prepaid Insurance for $1,900. 22 The company received $7,700 cash as partial payment for the work completed on September 12. 24 The company completed work for another client for $2,100 on credit. 28 The company paid a $5,300 cash dividend. 29 The company purchased $550 of additional office supplies on credit. 30 The company paid $860 cash for this month’s utility bill.

Required

1. Prepare general journal entries to record these transactions (use account titles listed in part 2). 2. Open the following ledger accounts—their account numbers are in parentheses (use the balance col-

umn format): Cash (101); Accounts Receivable (106); Office Supplies (124); Prepaid Insurance (128); Prepaid Rent (131); Office Equipment (163); Accounts Payable (201); Common Stock (307); Dividends (319); Services Revenue (401); and Utilities Expense (690). Post journal entries from part 1 to the ledger accounts and enter the balance after each posting.

3. Prepare a trial balance as of the end of September.

Check (2) Ending balances: Cash, $21,520; Accounts Receivable, $9,800; Accounts Payable, $550

(3) Total debits, $74,330

d. Purchased office supplies for $890 cash. e. Purchased $12,900 of office equipment on credit (with accounts payable). f. Received $36,000 cash for engineering services provided in May. g. The company paid a $3,370 cash dividend. Prepare a Cash T-account, enter the cash effects (if any) of each transaction, and compute the ending

Cash balance. Code each entry in the T-account with one of the transaction codes a through g.

At the beginning of April, Bernadette Grechus launched a custom computer solutions company called Softworks. The company had the following transactions during April. a. Bernadette Grechus invested $65,000 cash, office equipment with a value of $5,750, and $30,000 of

computer equipment in the company in exchange for common stock. b. The company purchased land worth $22,000 for an office by paying $5,000 cash and signing a long-

term note payable for $17,000.

Problem 2-2B Preparing and posting journal entries; preparing a trial balance

C3 C4 A1 P1 P2

[continued on next page]

78 Chapter 2 Accounting for Business Transactions

Zucker Management Services opened for business and completed these transactions in November.

Nov. 1 Matt Zucker, the owner, invested $30,000 cash along with $15,000 of office equipment in the company in exchange for common stock.

2 The company prepaid $4,500 cash for six months’ rent for an office. Hint: Debit Prepaid Rent for $4,500.

4 The company made credit purchases of office equipment for $2,500 and of office supplies for $600. Payment is due within 10 days.

8 The company completed work for a client and immediately received $3,400 cash. 12 The company completed a $10,200 project for a client, who must pay within 30 days. 13 The company paid $3,100 cash to settle the payable created on November 4. 19 The company paid $1,800 cash for the premium on a 24-month insurance policy. 22 The company received $5,200 cash as partial payment for the work completed on November 12. 24 The company completed work for another client for $1,750 on credit. 28 The company paid a $5,300 cash dividend. 29 The company purchased $249 of additional office supplies on credit. 30 The company paid $831 cash for this month’s utility bill.

Required

1. Prepare general journal entries to record these transactions (use account titles listed in part 2). 2. Open the following ledger accounts—their account numbers are in parentheses (use the balance col-

umn format): Cash (101); Accounts Receivable (106); Office Supplies (124); Prepaid Insurance (128); Prepaid Rent (131); Office Equipment (163); Accounts Payable (201); Common Stock (307); Dividends (319); Services Revenue (403); and Utilities Expense (690). Post the journal entries from part 1 to the ledger accounts and enter the balance after each posting.

3. Prepare a trial balance as of the end of November.

Check (2) Ending balances: Cash, $23,069; Accounts Receivable, $6,750; Accounts Payable, $249

(3) Total debits, $60,599

Problem 2-3B Preparing and posting journal entries; preparing a trial balance

C3 C4 A1 P1 P2

c. The company purchased a portable building with $34,500 cash and moved it onto the land acquired in b.

d. The company paid $5,000 cash for the premium on a two-year insurance policy. e. The company provided services to a client and immediately collected $4,600 cash. f. The company purchased $4,500 of additional computer equipment by paying $800 cash and signing a

long-term note payable for $3,700. g. The company completed $4,250 of services for a client. This amount is to be received within

30 days. h. The company purchased $950 of additional office equipment on credit. i. The company completed client services for $10,200 on credit. j. The company received a bill for rent of a computer testing device that was used on a recently com-

pleted job. The $580 rent cost must be paid within 30 days. k. The company collected $5,100 cash in partial payment from the client described in transaction i. l. The company paid $1,800 cash for wages to an assistant. m. The company paid $950 cash to settle the payable created in transaction h. n. The company paid $608 cash for minor maintenance of the company’s computer equipment. o. The company paid a $6,230 cash dividend. p. The company paid $1,800 cash for wages to an assistant. q. The company paid $750 cash for advertisements on the web during April.

Required

1. Prepare general journal entries to record these transactions (use account titles listed in part 2). 2. Open the following ledger accounts—their account numbers are in parentheses (use the balance col-

umn format): Cash (101); Accounts Receivable (106); Prepaid Insurance (108); Office Equipment (163); Computer Equipment (164); Building (170); Land (172); Accounts Payable (201); Notes Payable (250); Common Stock (307); Dividends (319); Fees Earned (402); Wages Expense (601); Computer Rental Expense (602); Advertising Expense (603); and Repairs Expense (604). Post the journal entries from part 1 to the accounts and enter the balance after each posting.

3. Prepare a trial balance as of the end of April.

Check (2) Ending balances: Cash, $17,262; Accounts Receivable, $9,350; Accounts Payable, $580

(3) Trial balance totals, $141,080

[continued from previous page]

Chapter 2 Accounting for Business Transactions 79

Problem 2-4B Recording transactions; posting to ledger; preparing a trial balance

C3 A1 P1 P2

Nuncio Consulting completed the following transactions during June. a. Armand Nuncio, the owner, invested $35,000 cash along with office equipment valued at $11,000 in

the new company in exchange for common stock. b. The company purchased land valued at $7,500 and a building valued at $40,000. The purchase is paid

with $15,000 cash and a long-term note payable for $32,500. c. The company purchased $500 of office supplies on credit. d. A. Nuncio invested his personal automobile in the company in exchange for more common stock. The

automobile has a value of $8,000 and is to be used exclusively in the business. e. The company purchased $1,200 of additional office equipment on credit. f. The company paid $1,000 cash salary to an assistant. g. The company provided services to a client and collected $3,200 cash. h. The company paid $540 cash for this month’s utilities. i. The company paid $500 cash to settle the payable created in transaction c. j. The company purchased $3,400 of new office equipment by paying $3,400 cash. k. The company completed $4,200 of services for a client, who must pay within 30 days. l. The company paid $1,000 cash salary to an assistant. m. The company received $2,200 cash in partial payment on the receivable created in transaction k. n. The company paid a $1,100 cash dividend.

Required

1. Prepare general journal entries to record these transactions (use account titles listed in part 2). 2. Open the following ledger accounts—their account numbers are in parentheses (use the balance col-

umn format): Cash (101); Accounts Receivable (106); Office Supplies (108); Office Equipment (163); Automobiles (164); Building (170); Land (172); Accounts Payable (201); Notes Payable (250); Common Stock (307); Dividends (319); Fees Earned (402); Salaries Expense (601); and Utilities Expense (602). Post the journal entries from part 1 to the ledger accounts and enter the balance after each posting.

3. Prepare a trial balance as of the end of June.

Check (2) Ending balances: Cash, $17,860; Office Equipment, $15,600

(3) Trial balance totals, $95,100

Problem 2-5B Computing net income from equity analysis, preparing a balance sheet, and computing the debt ratio

C2 A1 A2 P3

The accounting records of Tama Co. show the following assets and liabilities as of December 31, 2018 and 2019.

December 31 2018 2019

Cash . . . . . . . . . . . . . . . . . . . . . . . . $30,000 $ 5,000

Accounts receivable . . . . . . . . . . . . 35,000 25,000

Office supplies . . . . . . . . . . . . . . . . 8,000 13,500

Office equipment . . . . . . . . . . . . . . 40,000 40,000

Machinery . . . . . . . . . . . . . . . . . . . . 28,000 28,500

December 31 2018 2019

Building . . . . . . . . . . . . . . . . . . $ 0 $250,000

Land . . . . . . . . . . . . . . . . . . . . 0 50,000

Accounts payable . . . . . . . . . . 4,000 12,000

Note payable . . . . . . . . . . . . . 0 250,000

Required

1. Prepare balance sheets for the business as of December 31, 2018 and 2019. Hint: Report only total equity on the balance sheet and remember that total equity equals the difference between assets and liabilities.

2. Compute net income for 2019 by comparing total equity amounts for these two years and using the following information: During 2019, the owner invested $5,000 additional cash in the business (in exchange for common stock) and the company paid a $3,000 cash dividend.

3. Compute the December 31, 2019, debt ratio (in percent and rounded to one decimal).

Check (2) Net income, $11,000

(3) Debt ratio, 63.6%

Roshaun Gould started a web consulting firm called Gould Solutions. He began operations and completed seven transactions in April that resulted in the following accounts, which all have normal balances.

Problem 2-6B Analyzing account balances and reconstructing transactions

C1 C3 A1 P2 Cash . . . . . . . . . . . . . . . . . $20,000

Office supplies . . . . . . . . . 750

Prepaid rent . . . . . . . . . . . 1,800

Office equipment . . . . . . . $12,250

Accounts payable . . . . . . . 12,250

Common stock . . . . . . . . . 15,000

Dividends . . . . . . . . . . . . . . . . . . . $ 5,200

Consulting fees earned . . . . . . . . 20,400

Miscellaneous expenses . . . . . . . 7,650

80 Chapter 2 Accounting for Business Transactions

Required

1. Prepare a trial balance for this business as of the end of April. 2. The following seven transactions produced the account balances shown above. a. Gould invested $15,000 cash in the business in exchange for common stock. b. Paid $1,800 cash in advance for next month’s rent expense. c. Paid $7,650 cash for miscellaneous expenses. d. Purchased office supplies for $750 cash. e. Purchased $12,250 of office equipment on credit (with accounts payable). f. Received $20,400 cash for consulting services provided in April. g. The company paid a $5,200 cash dividend. Prepare a Cash T-account, enter the cash effects (if any) of each transaction, and compute the ending

Cash balance. Code each entry in the T-account with one of the transaction codes a through g.

Check (1) Trial balance totals, $47,650 (2) Ending Cash balance, $20,000

Victoria Rivera owns and manages a consulting firm called Prisek, which began operations on July 1. On July 31, the company’s records show the following selected accounts and amounts for the month of July.

Problem 2-7B Preparing an income statement, statement of retained earnings, and balance sheet

P3

Equipment . . . . . . . . . . . . . $12,000 Salaries expense . . . . . . . 9,000 Consulting revenue . . . . . 36,000 Cash . . . . . . . . . . . . . . . . . 24,000 Utilities expense . . . . . . . . 600 Note payable . . . . . . . . . . 7,200

Accounts receivable . . . . . . . . $10,500 Common stock . . . . . . . . . . . . 34,800 Dividends . . . . . . . . . . . . . . . . 6,000 Office supplies . . . . . . . . . . . . 4,500 Rental revenue . . . . . . . . . . . . 1,500 Advertising expense . . . . . . . . 1,200

Prepaid insurance . . . . . . . $3,000 Accounts payable . . . . . . . 3,900 Note receivable . . . . . . . . 7,500 Rent expense . . . . . . . . . . 6,000 Unearned revenue . . . . . . 900

Required

1. Prepare a July income statement for the business. 2. Prepare a July statement of retained earnings. The Retained Earnings account balance at July 1 was $0,

and the owner invested $34,800 cash in the company on July 2 in exchange for common stock. 3. Prepare a July 31 balance sheet. Hint: Use the Retained Earnings account balance calculated in part 2.

SERIAL PROBLEM Business Solutions

A1 P1 P2

This serial problem started in Chapter 1 and continues through most of the chapters. If the Chapter 1 seg- ment was not completed, the problem can begin at this point.

SP 2 On October 1, 2019, Santana Rey launched a computer services company called Business Solutions, which provides consulting services, computer system installations, and custom program devel- opment. Rey adopts the calendar year for reporting purposes and expects to prepare the company’s first set of financial statements on December 31, 2019. The company’s initial chart of accounts follows.

Account No. Account No.

Cash . . . . . . . . . . . . . . . . . . . . . . . . . . 101 Common Stock . . . . . . . . . . . . . . . . . . . . . . . . 307

Accounts Receivable . . . . . . . . . . . . 106 Dividends . . . . . . . . . . . . . . . . . . . . . . . . . . . . 319

Computer Supplies . . . . . . . . . . . . . . 126 Computer Services Revenue . . . . . . . . . . . . . 403

Prepaid Insurance . . . . . . . . . . . . . . . 128 Wages Expense . . . . . . . . . . . . . . . . . . . . . . . 623

Prepaid Rent . . . . . . . . . . . . . . . . . . . 131 Advertising Expense . . . . . . . . . . . . . . . . . . . . 655

Office Equipment . . . . . . . . . . . . . . . 163 Mileage Expense . . . . . . . . . . . . . . . . . . . . . . 676

Computer Equipment . . . . . . . . . . . . 167 Miscellaneous Expenses . . . . . . . . . . . . . . . . 677

Accounts Payable . . . . . . . . . . . . . . . 201 Repairs Expense—Computer . . . . . . . . . . . . . 684 ©Alexander Image/Shutterstock

Required

1. Prepare journal entries to record each of the following transactions for Business Solutions.

Oct. 1 S. Rey invested $45,000 cash, a $20,000 computer system, and $8,000 of office equipment in the company in exchange for common stock.

2 The company paid $3,300 cash for four months’ rent. Hint: Debit Prepaid Rent for $3,300. 3 The company purchased $1,420 of computer supplies on credit from Harris Office Products. 5 The company paid $2,220 cash for one year’s premium on a property and liability insurance

policy. Hint: Debit Prepaid Insurance for $2,220. 6 The company billed Easy Leasing $4,800 for services performed in installing a new web server.

Chapter 2 Accounting for Business Transactions 81

8 The company paid $1,420 cash for the computer supplies purchased from Harris Office Prod- ucts on October 3.

10 The company hired Lyn Addie as a part-time assistant. 12 The company billed Easy Leasing another $1,400 for services performed. 15 The company received $4,800 cash from Easy Leasing as partial payment on its account. 17 The company paid $805 cash to repair computer equipment that was damaged when moving it. 20 The company paid $1,728 cash for advertisements published in the local newspaper. 22 The company received $1,400 cash from Easy Leasing on its account. 28 The company billed IFM Company $5,208 for services performed. 31 The company paid $875 cash for Lyn Addie’s wages for seven days’ work. 31 The company paid a $3,600 cash dividend. Nov. 1 The company reimbursed S. Rey in cash for business automobile mileage allowance (Rey

logged 1,000 miles at $0.32 per mile). 2 The company received $4,633 cash from Liu Corporation for computer services performed. 5 The company purchased computer supplies for $1,125 cash from Harris Office Products. 8 The company billed Gomez Co. $5,668 for services performed. 13 The company agreed to perform future services for Alex’s Engineering Co. No work has yet

been performed. 18 The company received $2,208 cash from IFM Company as partial payment of the October 28 bill. 22 The company paid $250 cash for miscellaneous expenses. Hint: Debit Miscellaneous Expenses

for $250. 24 The company completed work and sent a bill for $3,950 to Alex’s Engineering Co. 25 The company sent another bill to IFM Company for the past-due amount of $3,000. 28 The company reimbursed S. Rey in cash for business automobile mileage (1,200 miles at $0.32

per mile). 30 The company paid $1,750 cash for Lyn Addie’s wages for 14 days’ work. 30 The company paid a $2,000 cash dividend. 2. Open ledger accounts (in balance column format) and post the journal entries from part 1 to them. 3. Prepare a trial balance as of the end of November.

Check (2) Cash, Nov. 30 bal., $38,264 (3) Trial bal. totals, $98,659

GENERAL LEDGER PROBLEM

Using transactions from the following assignments along with the General Ledger tool, prepare journal entries for each transaction and identify the financial statement impact of each entry. The financial state- ments are automatically generated based on the journal entries recorded.

GL 2-1 Transactions from the FastForward illustration in this chapter GL 2-2 Based on Exercise 2-9 GL 2-3 Based on Exercise 2-12 GL 2-4 Based on Problem 2-1A Using transactions from the following assignments, record journal entries, create financial statements, and assess the impact of each transaction on financial statements.

GL 2-5 Based on Problem 2-2A GL 2-7 Based on Problem 2-4A GL 2-6 Based on Problem 2-3A GL 2-8 Based on the Serial Problem SP 2

GL

COMPANY ANALYSIS A1 A2

Accounting Analysis

AA 2-1 Refer to Apple’s financial statements in Appendix A for the following questions.

Required

1. What amount of total liabilities does Apple report for each of the fiscal years ended (a) September 30, 2017, and (b) September 24, 2016?

2. What amount of total assets does it report for each of the fiscal years ended (a) September 30, 2017, and (b) September 24, 2016?

3. Compute its debt ratio for each of the fiscal years ended (a) September 30, 2017, and (b) September 24, 2016. (Report ratio in percent and round it to one decimal.)

4. In which fiscal year did it employ more financial leverage: September 30, 2017, or September 24, 2016? Explain.

APPLE

82 Chapter 2 Accounting for Business Transactions

AA 2-2 Key comparative figures for Apple and Google follow.COMPARATIVE ANALYSIS A1 A2 Apple Google

$ millions Current Year Prior Year Current Year Prior Year

Total liabilities . . . . . . . . . . . . . . . . . . . $241,272 $193,437 $ 44,793 $ 28,461 Total assets . . . . . . . . . . . . . . . . . . . . . 375,319 321,686 197,295 167,497

1. What is the debt ratio for Apple in the current year and for the prior year? 2. What is the debt ratio for Google in the current year and for the prior year? 3. Which of the two companies has the higher degree of financial leverage in the current year?

APPLE GOOGLE

BTN 2-2 Lila Corentine is an aspiring entrepreneur and your friend. She is having difficulty understand- ing the purposes of financial statements and how they fit together across time.

Required

Write a one-page memorandum to Corentine explaining the purposes of the four financial statements and how they are linked across time.

COMMUNICATING IN PRACTICE C1 C2 A1 P3

AA 2-3 Key comparative figures for Apple, Google, and Samsung follow.GLOBAL ANALYSIS A2 Samsung Apple Google

In millions Current Year Prior Year Current Year Current Year

Total liabilities . . . . . . . . . . . ₩ 87,260,662 ₩ 69,211,291 $241,272 $ 44,793

Total assets . . . . . . . . . . . . . 301,752,090 262,174,324 375,319 197,295

APPLE GOOGLE Samsung

Required

1. Compute Samsung’s debt ratio for the current year and prior year. 2. Is Samsung on a trend toward increased or decreased financial leverage? 3. Looking at the current-year debt ratio, is Samsung a more risky or less risky investment than (a) Apple

and (b) Google?

ETHICS CHALLENGE C1

BTN 2-1 Assume that you are a cashier and your manager requires that you immediately enter each sale when it occurs. Recently, lunch hour traffic has increased and the assistant manager asks you to avoid delays by taking customers’ cash and making change without entering sales. The assistant manager says she will add up cash and enter sales after lunch. She says that, in this way, customers will be happy and the register record will always match the cash amount when the manager arrives at three o’clock.

The advantages to the process proposed by the assistant manager include improved customer service, fewer delays, and less work for you. The disadvantage is that the assistant manager could steal cash by simply recording less sales than the cash received and then pocketing the excess cash. You decide to reject her suggestion without the manager’s approval and to confront her on the ethics of her suggestion.

Required

Propose and evaluate two other courses of action you might consider, and explain why.

Beyond the Numbers

BTN 2-3 Access EDGAR online (SEC.gov) and locate the 2016 10-K report of Amazon.com (ticker: AMZN) filed on February 10, 2017. Review its financial statements reported for years ended 2016, 2015, and 2014 to answer the following questions.

Required

1. What are the amounts of Amazon’s net income or net loss reported for each of these three years? 2. Do Amazon’s operating activities provide cash or use cash for each of these three years? Hint: See the

statement of cash flows. 3. If Amazon has 2016 net income of $2,371 million and 2016 operating cash flows of $16,443 million,

how is it possible that its cash balance at December 31, 2016, increases by only $3,444 million relative to its balance at December 31, 2015?

TAKING IT TO THE NET A1

Chapter 2 Accounting for Business Transactions 83

BTN 2-4 The expanded accounting equation consists of assets, liabilities, common stock, dividends, revenues, and expenses. It can be used to reveal insights into changes in a company’s financial position.

Required

1. Form learning teams of six (or more) members. Each team member must select one of the six compo- nents, and each team must have at least one expert on each component: (a) assets, (b) liabilities, (c) common stock, (d ) dividends, (e) revenues, and ( f ) expenses.

2. Form expert teams of individuals who selected the same component in part 1. Expert teams are to draft a report that each expert will present to his or her learning team addressing the following: a. Identify for its component the (i) increase and decrease side of the account and (ii) normal balance

side of the account. b. Describe a transaction, with amounts, that increases its component. c. Using the transaction and amounts in (b), verify the equality of the accounting equation and then

explain any effects on the income statement and statement of cash flows. d. Describe a transaction, with amounts, that decreases its component. e. Using the transaction and amounts in (d), verify the equality of the accounting equation and then

explain any effects on the income statement and statement of cash flows. 3. Each expert should return to his/her learning team. In rotation, each member presents his/her expert

team’s report to the learning team. Team discussion is encouraged.

TEAMWORK IN ACTION C1 C2 C4 A1

BTN 2-5 Assume that James Park and Eric Friedman of Fitbit plan on expanding their business to accommodate more product lines. They are considering financing expansion in one of two ways: (1) con- tributing more of their own funds to the business or (2) borrowing the funds from a bank.

Required

Identify at least two issues that James and Eric should consider when trying to decide on the method for financing their expansion.

ENTREPRENEURIAL DECISION A1 A2 P3

BTN 2-6 Angel Martin is a young entrepreneur who operates Martin Music Services, offering singing lessons and instruction on musical instruments. Martin wishes to expand but needs a $30,000 loan. The bank requests that Martin prepare a balance sheet and key financial ratios. Martin has not kept formal records but is able to provide the following accounts and their amounts as of December 31.

ENTREPRENEURIAL DECISION A1 A2 P3

Cash . . . . . . . . . . . . . . . . . . $ 3,600 Accounts receivable . . . . . . $ 9,600 Prepaid insurance . . . . . . . $ 1,500 Prepaid rent . . . . . . . . . . . . 9,400 Store supplies . . . . . . . . . . . 6,600 Equipment . . . . . . . . . . . . . 50,000 Accounts payable . . . . . . . 2,200 Unearned lesson fees . . . . . 15,600 Total equity* . . . . . . . . . . . . 62,900 Annual net income . . . . . . . 40,000

*The total equity amount reflects all owner investments, dividends, revenues, and expenses as of December 31.

Required

1. Prepare a balance sheet as of December 31 for Martin Music Services. (Report only the total equity amount on the balance sheet.)

2. Compute Martin’s debt ratio and its return on assets (the latter ratio is defined in Chapter 1). Assume average assets equal its ending balance.

3. Do you believe the prospects of a $30,000 bank loan are good? Why or why not?

BTN 2-7 Obtain a recent copy of the most prominent newspaper distributed in your area. Research the classified section and prepare a report answering the following questions (attach relevant printouts to your report). Alternatively, you may want to search the web for the required information. One suitable website is CareerOneStop (CareerOneStop.org). For documentation, print copies of websites accessed. 1. Identify the number of listings for accounting positions and the various accounting job titles. 2. Identify the number of listings for other job titles, with examples, that require or prefer accounting

knowledge/experience but are not specifically accounting positions. 3. Specify the salary range for the accounting and accounting-related positions if provided. 4. Indicate the job that appeals most to you, the reason for its appeal, and its requirements.

HITTING THE ROAD C1

Design elements: Lightbulb: ©Chuhail/Getty Images; Blue globe: ©nidwlw/Getty Images and ©Dizzle52/Getty Images; Chess piece: ©Andrei Simonenko/Getty Images and ©Dizzle52/Getty Images; Mouse: ©Siede Preis/Getty Images; Global View globe: ©McGraw-Hill Education and ©Dizzle52/Getty Images; Sustainability: ©McGraw-Hill Education and ©Dizzle52/Getty Images

Learning Objectives

CONCEPTUAL C1 Explain the importance of periodic

reporting and the role of accrual accounting.

C2 Identify steps in the accounting cycle.

C3 Explain and prepare a classified balance sheet.

ANALYTICAL A1 Compute profit margin and describe its

use in analyzing company performance.

A2 Compute the current ratio and describe what it reveals about a company’s financial condition.

P6 Prepare financial statements from an adjusted trial balance.

P7 Describe and prepare closing entries.

P8 Explain and prepare a post-closing trial balance.

P9 Appendix 3A—Explain the alternatives in accounting for prepaids.

P10 Appendix 3B—Prepare a work sheet and explain its usefulness.

P11 Appendix 3C—Prepare reversing entries and explain their purpose.

PROCEDURAL P1 Prepare adjusting entries for deferral of

expenses.

P2 Prepare adjusting entries for deferral of revenues.

P3 Prepare adjusting entries for accrued expenses.

P4 Prepare adjusting entries for accrued revenues.

P5 Explain and prepare an adjusted trial balance.

Chapter Preview

3 Adjusting Accounts for Financial Statements

NTK 3-4

ACCRUED REVENUE

P4 Framework Examples

Summary

NTK 3-3

ACCRUED EXPENSE

P3 Framework Examples

NTK 3-2

DEFERRAL OF REVENUE

P2 Framework Examples

DEFERRAL OF EXPENSE

C1 Timing Accrual vs. cash

3-Step process

P1 Framework Examples

NTK 3-1 NTK 3-5, 6

REPORTING

P5 Adjusted trial balance

P6 Financial statements

P7 Closing process

P8 Post-closing trial balance

NTK 3-7

CLASSIFICATION AND ANALYSIS

C2 Accounting cycle C3 Classified

balance sheet

A1 Profit margin A2 Current ratio

85

“Creativity creates value”—Evan Spiegel

Snap!

VENICE, CA—Evan Spiegel met his future co-founder Bobby Murphy in college. “We weren’t cool,” recalls Bobby, “so we tried to build things to be cool!” One of their cool projects was an app that could send messages that disappeared after a few seconds. This app would later be called Snapchat (Snapchat.com).

The first headquarters of Snapchat was the home of Evan’s dad. However, within a matter of months, their app had over a million users.

As Snapchat grew, Evan and Bobby knew an effective ac- counting system was key to attracting investors. “One of the things I did underestimate,” admits Evan, “was how much more important communication becomes [when seeking investors].”

Investors wanted to know revenues, costs, assets, and liabilities for Snapchat. “You really need to explain . . . how your business works,” insists Evan.

To communicate “the Snap story,” the entrepreneurs learned how to defer and accrue revenues and expenses and to prepare financial statements for investors. This included learning the

accounting cycle. With accounting reports in hand, Evan and Bobby were able to secure additional financing. Exclaims Evan: “That was the greatest feeling of all time!”

Sources: Snapchat website, January 2019; Vanity Fair, October 2017; LA Times, March 2017; Forbes, January 2014

The Accounting Period The value of information is linked to its timeliness. Useful information must reach decision makers frequently. To provide timely information, accounting systems prepare reports at regular intervals. The time period assumption presumes that an organization’s activities can be divided into specific time periods such as a month, a three-month quarter, a six- month interval, or a year. Exhibit 3.1 shows various accounting, or reporting, periods. Most organizations use a year as their primary accounting period. Reports covering a one-year period are known as annual finan- cial statements. Many organizations also prepare interim financial statements covering one, three, or six months of activity.

TIMING AND REPORTING

C1 Explain the importance of periodic reporting and the role of accrual accounting.

$0 2019 2018 2017 2016 2015

$100

Millions Ratio

$200 $300 $400 $500 $600 $700

$900

15%

0.0%

30%

45%

$800

Apple

“Apple announces annual income of . . .”

Jan. Mar. May June July Aug. Sept. Oct. Nov. TimeDec.

1

1 2 3 4

2 3 4 5 6 7 8 9 10 11 12 Monthly

Quarterly

1 2 Semiannually

1 Annually

Feb. Apr.

EXHIBIT 3.1 Accounting Periods

The annual reporting period is not always a calendar year ending on December 31. An organization can use a fiscal year consisting of any 12 consecutive months or 52 weeks. For example, Gap’s fiscal year consistently ends the final week of January or the first week of February each year.

©J. Emilio Flores/Corbis/Getty Images

86 Chapter 3 Adjusting Accounts for Financial Statements

Companies with little seasonal variation in sales often use the calendar year as their fiscal year. Facebook uses calendar-year reporting. Companies that have seasonal variations in sales often use a natural business year end, which is when sales are at their lowest level for the year. The natural business year for retailers such as Target and Dick’s Sporting Goods ends around January 31, after the holidays.

Accrual Basis versus Cash Basis After external transactions and events are recorded, several accounts require adjustments before their balances appear in financial statements. This is needed because internal transactions and events are not yet recorded. Accrual basis accounting records revenues when services and products are delivered and

records expenses when incurred (matched with revenues). Cash basis accounting records revenues when cash is received and records expenses when

cash is paid. Cash basis income is cash receipts minus cash payments.

Most agree that accrual accounting better reflects business performance than cash basis accounting. Accrual accounting also increases the comparability of financial statements from period to period.

Accrual Basis To compare these two systems, let’s consider FastForward’s Prepaid Insurance account. FastForward paid $2,400 for 24 months of insurance coverage that began on December 1, 2019. Accrual accounting requires that $100 of insurance expense be reported each month, from December 2019 through November 2021. (This means expenses are $100 in 2019, $1,200 in 2020, and $1,100 in 2021.) Exhibit 3.2 shows this allocation of insurance cost across the three years. Any unexpired premium is reported as a Prepaid Insurance asset on the accrual basis balance sheet.

©Vixit/Shutterstock

Insurance Expense 2021 Jan

$100

May $100

Sept $100

Feb $100

June $100

Oct $100

Mar $100

July $100

Nov $100

Apr $100

Aug $100

Dec $0

Insurance Expense 2019

2019 2020 2021

Jan $0

May $0

Sept $0

Feb $0

June $0

Oct $0

Mar $0

July $0

Nov $0

Apr $0

Aug $0

Dec $100

Insurance Expense 2020 Jan

$100

May $100

Sept $100

Feb $100

June $100

Oct $100

Mar $100

July $100

Nov $100

Apr $100

Aug $100

Dec $100

Paid $2,400 for 24 months’ insurance beginning

Dec. 1, 2019

Transaction:

EXHIBIT 3.2 Accrual Accounting for Allocating Prepaid Insurance to Expense

Cash Basis A cash basis income statement for December 2019 reports insurance ex- pense of $2,400, as shown in Exhibit 3.3. The cash basis income statements for years 2020 and 2021 report no insurance expense. The cash basis balance sheet never reports a prepaid insurance asset because it is immediately expensed. Also, cash basis income for 2019–2021 does not match the cost of insurance with the insurance benefits received for those years and months.

Insurance Expense 2021 Jan $0

May $0

Sept $0

Feb $0

June $0

Oct $0

Mar $0

July $0

Nov $0

Apr $0

Aug $0

Dec $0

Insurance Expense 2020 Jan $0

May $0

Sept $0

Feb $0

June $0

Oct $0

Mar $0

July $0

Nov $0

Apr $0

Aug $0

Dec $0

Insurance Expense 2019 Jan $0

May $0

Sept $0

Feb $0

June $0

Oct $0

Mar $0

July $0

Nov $0

Apr $0

Aug $0

Dec $2,400

2019 2020 2021

Paid $2,400 for 24 months’ insurance beginning

Dec. 1, 2019

Transaction:

EXHIBIT 3.3 Cash Accounting for Allocating Prepaid Insurance to Expense

Recognizing Revenues and Expenses We divide a company’s activities into time periods, but not all activities are complete when financial statements are prepared. Thus, adjustments are required to get proper account balances.

Point: Annual income statements for Exhibit 3.3 follow: Cash Basis 2019 2020 2021

Revenues . . . . . . . $ # $# $# Insurance exp . . . . $2,400 $0 $0

Point: Annual income statements for Exhibit 3.2 follow: Accrual Basis 2019 2020 2021

Revenues . . . . . . . $ # $ # $ # Insurance exp . . . . $100 $1,200 $1,100

Chapter 3 Adjusting Accounts for Financial Statements 87

We use two principles in the adjusting process: revenue recognition and expense recognition. Revenue recognition principle requires that revenue be recorded when goods or services

are provided to customers and at an amount expected to be received from customers. Adjustments ensure revenue is recognized (reported) in the time period when those services and products are provided.

Expense recognition (or matching) principle requires that expenses be recorded in the same accounting period as the revenues that are recognized as a result of those expenses.

Point: Recording revenue early overstates current-period income; recording it late understates current-period income.

Point: Recording expense early understates current-period in- come; recording it late overstates current-period income.

Framework for Adjustments Four types of adjustments exist for transactions and events that extend over more than one period.

Deferral of expense Deferral of revenue Accrued expense Accrued revenue

Adjustments are made using a 3-step process, as shown in Exhibit 3.4.

Clawbacks from Accounting Fraud Former executives at Saba Software, a cloud-based talent management sys- tem, were charged with accounting fraud by the SEC for falsifying revenue to boost income. This alleged overstate- ment of income led to a payback of millions of dollars to the company by the former CEO and former CFO. See SEC release 2015–28. ■

Ethical Risk

©Marco Marchi/Getty Images

Step 1: Determine what the current account balance equals. Step 2: Determine what the current account balance should equal. Step 3: Record an adjusting entry to get from step 1 to step 2.

EXHIBIT 3.4 Three-Step Process for Adjusting Entries

Each adjusting entry made at the end of an accounting period reflects a transaction or event that is not yet recorded. An adjusting entry affects one or more income statement accounts and one or more balance sheet accounts (but never the Cash account).

Prepaid expenses, or deferred expenses, are assets paid for in advance of receiving their bene- fits. When these assets are used, those advance payments become expenses.

Framework Adjusting entries for prepaid expenses increase expenses and decrease assets, as shown in the T-accounts of Exhibit 3.5. This adjustment shows the using up of prepaid ex- penses. To demonstrate accounting for pre- paid expenses, we look at prepaid insurance, supplies, and depreciation. In each case we decrease an asset (balance sheet) account and increase an expense (income statement) account.

Prepaid Insurance Prepaid insurance expires with time. We use our three-step process.

Step 1: We determine that the current balance of FastForward’s prepaid insurance is equal to its $2,400 payment for 24 months of insurance benefits that began on December 1, 2019. Step 2: As time passes, the benefits of the insurance gradually expire and a portion of the Prepaid Insurance asset becomes expense. For instance, one month’s insurance coverage expires by December 31, 2019. This expense is $100, or 1/24 of $2,400, which leaves $2,300.

DEFERRAL OF EXPENSE P1 Prepare adjusting entries for deferral of expenses.

Asset

Unadjusted balance

overstated

Expense

Dr. Expense… # Cr. Asset….. #

Decrease it Increase it EXHIBIT 3.5 Adjusting for Prepaid Expenses (decrease an asset and record an expense)

Insurance Dec. 1 Pay insurance premium and record asset Prepaid Insurance....... 2,400 Cash......................... 2,400

Dec. 31 Coverage expires and record expense

Two-Year Insurance Policy Total cost is $2,400 Monthly cost is $100

88 Chapter 3 Adjusting Accounts for Financial Statements

Supplies We count supplies at period-end and make an adjusting entry.

Step 1: FastForward purchased $9,720 of supplies in December, some of which were used during that same month. When financial statements are prepared at December 31, the cost of supplies used during December is expensed. Step 2: When FastForward computes (physically counts) its remaining unused supplies at December 31, it finds $8,670 of supplies remaining of the $9,720 total supplies. The $1,050 difference between these two amounts is December’s supplies expense. Step 3: The adjusting entry to record this expense and reduce the Supplies asset account, along with T-account postings, follows.

Step 3: The adjusting entry to record this expense and reduce the asset, along with T-account postings, follows.

Explanation After adjusting and posting, the $100 balance in Insurance Expense and the $2,300 balance in Prepaid Insurance are ready for reporting in financial statements. Not making the adjustment on or before December 31 would Understate expenses by $100 for the December income statement. Overstate prepaid insurance (assets) by $100 in the December 31 balance sheet.

The following highlights the adjustment for prepaid insurance.

Prepaid Insurance = $2,400

Reports $2,400 policy for 24 months’ coverage .

Deduct $100 from Prepaid Insurance Add $100 to Insurance Expense

Record current month’s $100 insurance expense and $100 reduction in prepaid .

Prepaid Insurance = $2,300

Reports $2,300 in coverage for remaining 23 months .

Before Adjustment Adjustment After Adjustment

Supplies Dec. 2,6,26 Purchase supplies and record asset

Dec. 31 Physical count Dec. 31 Record expense

Explanation The balance of the Supplies account is $8,670 after posting—equaling the cost of the remaining supplies. Not making the adjustment on or before December 31 would Understate expenses by $1,050 for the December income statement. Overstate supplies by $1,050 in the December 31 balance sheet.

Assets = Liabilities + Equity −100 −100

Adjustment (a) Dec . 31 Insurance Expense . . . . . . . . . . . . . . . . . . . . . . . . . . . . . . . . . . 100

Prepaid Insurance . . . . . . . . . . . . . . . . . . . . . . . . . . . . . . . 100

Record first month’s expired insurance.

Dec. 31 100

Insurance Expense 637

Dec . 1 2,400

Balance 2,300

Dec. 31 100

Prepaid Insurance 128

Assets = Liabilities + Equity −1,050 −1,050

Adjustment (b) Dec . 31 Supplies Expense . . . . . . . . . . . . . . . . . . . . . . . . . . . . . . . . . . . 1,050

Supplies . . . . . . . . . . . . . . . . . . . . . . . . . . . . . . . . . . . . . . 1,050

Record supplies used.

Dec. 31 1,050

Supplies Expense 652

Dec . 2 2,500

6 7,100

26 120

Balance 8,670

Dec. 31 1,050

Supplies 126

Chapter 3 Adjusting Accounts for Financial Statements 89

The following highlights the adjustment for supplies.

Supplies = $9,720

Reports $9,720 in supplies .

Deduct $1,050 from Supplies Add $1,050 to Supplies Expense

Record $1,050 in supplies used and $1,050 as supplies expense .

Supplies = $8,670

Reports $8,670 in supplies .

Before Adjustment Adjustment After Adjustment

Other Prepaid Expenses Other prepaid expenses, such as Prepaid Rent and Prepaid Advertising, are accounted for exactly as insurance and supplies are.

Some prepaid expenses are both paid for and fully used up within a single period. One exam- ple is when a company pays monthly rent on the first day of each month. In this case, we record the cash paid with a debit to Rent Expense instead of an asset account.

Investor A publisher signs an Olympic skier to write a book. The company pays the skier $500,000 to sign plus future book royalties. A note to the company’s financial statements says that “prepaid expenses include $500,000 in author signing fees to be matched against future expected sales.” How does this affect your analysis? ■ Answer: Prepaid expenses are assets paid for in advance of receiving their benefits–they are expensed as they are used up. As an investor, you are concerned about the risk of future book sales. The riskier the likelihood of future book sales is, the more likely your analysis is to treat the $500,000, or a portion of it, as an expense, not a prepaid expense (asset).

Decision Maker

©Don Hammond/Design Pics

Depreciation A special category of prepaid expenses is plant assets, which are long-term tangible assets used to produce and sell products and services. Plant assets provide benefits for more than one period. Examples of plant assets are buildings, machines, vehicles, and fixtures. All plant assets (exclud- ing land) eventually wear out or become less useful. The costs of plant assets are gradually reported as expenses in the income statement over the assets’ useful lives (benefit periods). Depreciation is the allocation of the costs of these assets over their expected useful lives. Depre- ciation expense is recorded with an adjusting entry similar to that for other prepaid expenses.

Step 1: FastForward purchased equipment for $26,000 in early December to use in earning revenue. This equipment’s cost must be depreciated. Step 2: The equipment is expected to have a useful life (benefit period) of five years and to be worth about $8,000 at the end of five years. This means the net cost of this equipment over its useful life is $18,000 ($26,000 − $8,000). FastForward depreciates it using straight-line depreciation, which allocates equal amounts of the asset’s net cost to depreciation during its useful life. Dividing the $18,000 net cost by the 60 months (5 years) in the asset’s useful life gives a monthly cost of $300 ($18,000∕60). Step 3: The adjusting entry to record monthly depreciation expense, along with T-account postings, follows.

Point: Plant assets are also called Plant & Equipment or Property, Plant & Equipment (PP&E ).

Point: Depreciation does not necessarily measure decline in market value.

Point: An asset’s expected value at the end of its useful life is called salvage value.

Depreciation Dec. 3 Purchase equipment and record asset Equipment........... 26,000 Cash................ 26,000

Dec. 31 Allocate asset cost and record depreciation

Explanation After posting the adjustment, the Equipment account ($26,000) minus its Accumulated Depreciation ($300) account equals the $25,700 net cost. The $300 balance in the

Assets = Liabilities + Equity −300 −300

Adjustment (c) Dec . 31 Depreciation Expense . . . . . . . . . . . . . . . . . . . . . . . . . . . . . . . . 300

Accumulated Depreciation—Equipment . . . . . . . . . . . . 300

Record monthly equipment depreciation.

Dec. 31 300

Depreciation Expense—Equipment 612

Dec . 3 26,000

Equipment 167

Dec. 31 300

Accumulated Depreciation—Equipment 168

90 Chapter 3 Adjusting Accounts for Financial Statements

Depreciation Expense account is reported in the December income statement. Not making the adjustment at December 31 would Understate expenses by $300 for the December income statement. Overstate assets by $300 in the December 31 balance sheet.

The following highlights the adjustment for depreciation.

Equipment, net = $26,000

Reports $26,000 in equipment .

Deduct $300 from Equipment, net Add $300 to Depreciation Expense

Record $300 in depreciation and $300 as accumulated depreciation .

Equipment, net = $25,700

Reports $25,700 in equipment, net of accumulated depreciation .

Before Adjustment Adjustment After Adjustment

Dec . 3 26,000

Equipment 167 Accumulated Depreciation—Equipment 168

Dec . 31 300

Jan . 31 300

Feb . 28 300

Balance 900

EXHIBIT 3.6 Accounts after Three Months of Depreciation Adjustments

Accumulated Depreciation is a separate contra account. A contra account is an account linked with another account, it has an opposite normal balance, and it is reported as a subtraction from that other account’s balance. FastForward’s contra account of Accumulated Depreciation— Equipment is subtracted from the Equipment account in the balance sheet.

The Accumulated Depreciation contra account includes total depreciation expense for all prior periods for which the asset was used. To demonstrate, on February 28, 2020, after three months of adjusting entries, the Equipment and Accumulated Depreciation accounts appear as in Exhibit 3.6. The $900 balance in the Accumulated Depreciation account is subtracted from its related $26,000 asset cost. The difference ($25,100) between these two balances is called book value, or net amount, which is the asset’s costs minus its accumulated depreciation.

Point: Accumulated Depreciation has a normal credit balance; it decreases the asset’s reported value.

Point: The net cost of equipment is also called depreciable basis.

These account balances are reported in the assets section of the February 28 balance sheet in Exhibit 3.7. This presentation shows the full cost of assets and accumulated depreciation.

EXHIBIT 3.7 Equipment and Accumulated Depreciation on February 28 Balance Sheet

Assets (at February 28, 2020)

Cash $ .... Equipment $26,000 Less accumulated depreciation 900 25,100 Total Assets $

Commonly titled Equipment, net

For each separate case below, follow the three-step process for adjusting the prepaid asset account at December 31. Assume no other adjusting entries are made during the year.

1. Prepaid Insurance. The Prepaid Insurance account has a $5,000 debit balance to start the year, and no insurance payments were made during the year. A review of insurance policies shows that $1,000 of unexpired insurance remains at its December 31 year-end.

2. Prepaid Rent. On October 1 of the current year, the company prepaid $12,000 for one year of rent for facilities being occupied from that day forward. The company debited Prepaid Rent and credited Cash for $12,000. December 31 year-end statements must be prepared.

3. Supplies. The Supplies account has a $1,000 debit balance to start the year. Supplies of $2,000 were purchased during the current year and debited to the Supplies account. A December 31 physical count shows $500 of supplies remaining.

Prepaid Expenses

NEED-TO-KNOW 3-1

P1

Chapter 3 Adjusting Accounts for Financial Statements 91

4. Accumulated Depreciation. The company has only one fixed asset (equipment) that it purchased at the start of this year. That asset had cost $38,000, had an estimated life of 10 years, and is expected to be valued at $8,000 at the end of the 10-year life. December 31 year-end statements must be prepared.

Solution

1. Step 1: Prepaid Insurance equals $5,000 (before adjustment) Step 2: Prepaid Insurance should equal $1,000 (the unexpired part) Step 3: Adjusting entry to get from step 1 to step 2

Dec . 31 Insurance Expense . . . . . . . . . . . . . . . . . . . . . . . . . . . . . . . . . . . . . . . . . . . . . . . . . . 4,000

Prepaid Insurance . . . . . . . . . . . . . . . . . . . . . . . . . . . . . . . . . . . . . . . . . . . . . . 4,000

Record expired insurance coverage ($5,000 − $1,000).

Dec . 31 Rent Expense . . . . . . . . . . . . . . . . . . . . . . . . . . . . . . . . . . . . . . . . . . . . . . . . . . . . . . 3,000

Prepaid Rent . . . . . . . . . . . . . . . . . . . . . . . . . . . . . . . . . . . . . . . . . . . . . . . . . . 3,000

Record expired prepaid rent. *$12,000 − $3,000 = $9,000, where $3,000 is from: ($12,000∕12 months) × 3 months

2. Step 1: Prepaid Rent equals $12,000 (before adjustment) Step 2: Prepaid Rent should equal $9,000 (the unexpired part)* Step 3: Adjusting entry to get from step 1 to step 2

3. Step 1: Supplies equal $3,000 (from $1,000 + $2,000; before adjustment) Step 2: Supplies should equal $500 (what’s left) Step 3: Adjusting entry to get from step 1 to step 2*

4. Step 1: Accumulated Depreciation equals $0 (before adjustment) Step 2: Accumulated Depreciation should equal $3,000 (after current-period depreciation of $3,000)* Step 3: Adjusting entry to get from step 1 to step 2

Dec . 31 Depreciation Expense—Equipment . . . . . . . . . . . . . . . . . . . . . . . . . . . . . . . . . . . . . 3,000

Accumulated Depreciation—Equipment . . . . . . . . . . . . . . . . . . . . . . . . . . . . . 3,000

Record depreciation for period. *($38,000 − $8,000)∕10 years Do More: QS 3-5, QS 3-6,

QS 3-7, QS 3-8, QS 3-9

Unearned revenue is cash received in advance of providing products and services. Unearned revenues, or deferred revenues, are liabilities. When cash is accepted, an obligation to provide products or services is accepted.

Framework As products or services are provided, the liability decreases and the unearned revenues become earned reve- nues. Adjusting entries for unearned reve- nue decrease the unearned revenue (balance sheet) account and increase the revenue (income statement) account, as shown in Exhibit 3.8.

DEFERRAL OF REVENUE P2 Prepare adjusting entries for deferral of revenues.

Point: To defer is to postpone. We postpone reporting amounts received as revenues until the product or service is provided.

Dr. Liability….. # Cr. Revenue… #

Liability Revenue

Decrease it Increase it EXHIBIT 3.8 Adjusting for Unearned Revenues (decrease a liability and record revenue)

Dec . 31 Supplies Expense . . . . . . . . . . . . . . . . . . . . . . . . . . . . . . . . . . . . . . . . . . . . . . . . . . . 2,500

Supplies . . . . . . . . . . . . . . . . . . . . . . . . . . . . . . . . . . . . . . . . . . . . . . . . . . . . . . 2,500

Record supplies used. *$1,000 + $2,000 purchased − $ 2,500 supplies used = $500 remaining

92 Chapter 3 Adjusting Accounts for Financial Statements

Unearned revenues are common in sporting and concert events. When the Boston Celtics receive cash from advance ticket sales, they record it in an unearned revenue account called Deferred Game Revenues. The Celtics record revenue as games are played.

Unearned Consulting Revenue FastForward has unearned revenues. The company agreed on December 26 to provide consult- ing services to a client for 60 days for a fixed fee of $3,000.

Step 1: On December 26, the client paid the 60-day fee in advance, covering the period December 27 to February 24. The entry to record the cash received in advance is

This advance payment increases cash and creates a liability to do consulting work over the next 60 days (5 days this year and 55 days next year). Step 2: As time passes, FastForward earns this payment through consulting. By December 31, it has provided five days’ service and earned 5∕60 of the $3,000 unearned revenue. This amounts to $250 ($3,000 × 5∕60). The revenue recognition principle requires that $250 of unearned revenue be reported as revenue on the December income statement. Step 3: The adjusting entry to reduce the liability account and recognize earned revenue, along with T-account postings, follows.

Unearned Revenues

Thanks for cash in advance. I’ll work now

through Feb. 24

Dec. 26 Cash received in advance and record liability

Dec. 31 Provided 5 days of services and record revenue

Assets = Liabilities + Equity +3,000 +3,000

Dec . 26 Cash . . . . . . . . . . . . . . . . . . . . . . . . . . . . . . . . . . . . . . . . . . . . . . 3,000

Unearned Consulting Revenue . . . . . . . . . . . . . . . . . . . . 3,000

Received advance payment for services over the next 60 days.

Explanation The adjusting entry transfers $250 from unearned revenue (a liability account) to a revenue account. Not making the adjustment Understates revenue by $250 in the December income statement. Overstates unearned revenue by $250 on the December 31 balance sheet.

The following highlights the adjustment for unearned revenue.

Unearned Consulting Revenue = $3,000

Reports $3,000 in unearned revenue for consulting services promised for

60 days ($50 per day) .

Deduct $250 from Unearned Consulting Revenue

Add $250 to Consulting Revenue

Record 5 days of earned consulting revenue, which is 5∕60 of

unearned amount .

Unearned Consulting Revenue = $2,750

Reports $2,750 in unearned revenue for consulting services owed over next

55 days (55 days × $50 = $2,750) .

Before Adjustment Adjustment After Adjustment

Assets = Liabilities + Equity −250 +250

Adjustment (d) Dec . 31 Unearned Consulting Revenue . . . . . . . . . . . . . . . . . . . . . . . . . 250

Consulting Revenue . . . . . . . . . . . . . . . . . . . . . . . . . . . . . 250

Record earned revenue that was received in advance ($3,000 × 5∕60).

Dec . 5 4,200

12 1,600

31 250

Balance 6,050

Consulting Revenue 403

Dec. 31 250 Dec . 26 3,000

Balance 2,750

Unearned Consulting Revenue 236

Chapter 3 Adjusting Accounts for Financial Statements 93

For each separate case below, follow the three-step process for adjusting the unearned revenue liability account at December 31. Assume no other adjusting entries are made during the year.

a. Unearned Rent Revenue. The company collected $24,000 rent in advance on September 1, debiting Cash and crediting Unearned Rent Revenue. The tenant was paying 12 months’ rent in advance and moved in on September 1.

b. Unearned Services Revenue. The company charges $100 per month to spray a house for insects. A customer paid $600 on November 1 in advance for six treatments, which was recorded with a debit to Cash and a credit to Unearned Services Revenue. At year-end, the company has applied two treatments for the customer.

Solution

a. Step 1: Unearned Rent Revenue equals $24,000 (before adjustment) Step 2: Unearned Rent Revenue should equal $16,000 (current-period earned revenue is $8,000*) Step 3: Adjusting entry to get from step 1 to step 2

Unearned Revenues

NEED-TO-KNOW 3-2

P2

b. Step 1: Unearned Services Revenue equals $600 (before adjustment) Step 2: Unearned Services Revenue should equal $400 (current-period earned revenue is $200*) Step 3: Adjusting entry to get from step 1 to step 2

Dec . 31 Unearned Services Revenue . . . . . . . . . . . . . . . . . . . . . . . . 200

Services Revenue . . . . . . . . . . . . . . . . . . . . . . . . . . . . 200

Record earned portion of revenue received in advance. *$100 × 2 treatments = Services revenue Do More: QS 3-10, QS 3-11

Dec . 31 Unearned Rent Revenue . . . . . . . . . . . . . . . . . . . . . . . . . . . 8,000

Rent Revenue . . . . . . . . . . . . . . . . . . . . . . . . . . . . . . . 8,000

Record earned portion of rent received in advance. *($24,000∕12 months) × 4 months’ rental usage

Accrued expenses are costs that are incurred in a period that are both unpaid and unrecorded. Accrued expenses are reported on the income statement for the period when incurred.

Framework Adjusting entries for recording accrued expenses increase the expense (income statement) account and increase a liability (balance sheet) account, as shown in Exhibit 3.9. This adjustment recognizes expenses in- curred in a period but not yet paid. Common examples of accrued expenses are salaries, interest, rent, and taxes. We use salaries and interest to show how to adjust accounts for accrued expenses.

Accrued Salaries Expense FastForward’s employee earns $70 per day, or $350 for a five-day workweek beginning on Monday and ending on Friday. Step 1: Its employee is paid every two weeks on Friday. On December 12 and 26, the wages are paid, recorded in the journal, and posted to the ledger. Step 2: The calendar in Exhibit 3.10 shows three working days after the December 26 payday (29, 30, and 31). This means the employee has earned three days’ salary by the close of business on Wednesday, December 31, yet this salary cost has not been paid or recorded. FastForward must report the added expense and liability for unpaid salary from December 29, 30, and 31.

ACCRUED EXPENSE

Point: Accrued expenses are also called accrued liabilities.

Dr. Expense….. # Cr. Liability… #

Expense Liability

Increase it Increase it EXHIBIT 3.9 Adjusting for Accrued Expenses (increase a liability and record an expense)

P3 Prepare adjusting entries for accrued expenses.

94 Chapter 3 Adjusting Accounts for Financial Statements

Step 3: The adjusting entry for accrued salaries, along with T-account postings, follows.

PaydayPaydaySalary expense incurred

Pay period begins

S M T W T F S 1 2 3 4 5 6

7 8 9 10 11 12 13

14 15 16 17 18 19 20

21 22 23 24 25 26 27

28 29 30 31

S M T W T F S 1 2 3

4 5 6 7 8 9 10

11 12 13 14 15 16 17

18 19 20 21 22 23 24

25 26 27 28 29 30 31

December January

EXHIBIT 3.10 Salary Accrual and Paydays

Salaries Payable = $0

Reports $0 from employee salaries incurred but not yet paid in cash .

Add $210 to Salaries Payable Add $210 to Salaries Expense

Record 3 days’ salaries owed, but not yet paid, at $70 per day .

Salaries Payable = $210

Reports $210 salaries payable to employee but not yet paid .

Before Adjustment Adjustment After Adjustment

Explanation Salaries expense of $1,610 is reported on the December income statement, and $210 of salaries payable (liability) is reported in the balance sheet. Not making the adjustment Understates salaries expense by $210 in the December income statement. Understates salaries payable by $210 on the December 31 balance sheet.

The following highlights the adjustment for salaries incurred.

Accrued Interest Expense Companies accrue interest expense on notes payable (loans) and other long-term liabilities at the end of a period. Interest expense is incurred as time passes. Unless interest is paid on the last day of an accounting period, we need to adjust for interest expense incurred but not yet paid. This means we must accrue interest cost from the most recent payment date up to the end of the period. The formula for computing accrued interest is

Principal amount owed × Annual interest rate × Fraction of year since last payment

If a company has a $6,000 loan from a bank at 5% annual interest, then 30 days’ accrued interest expense is $25—computed as $6,000 × 0.05 × 30∕360. The adjusting entry debits Interest Expense for $25 and credits Interest Payable for $25.

Future Cash Payment of Accrued Expenses Accrued expenses at the end of one accounting period result in cash payment in a future period(s). Recall that FastForward recorded accrued salaries of $210. On January 9, the first

Point: Interest computations use a 360-day year, called the bankers’ rule.

Assets = Liabilities + Equity +210 −210

Adjustment (e) Dec . 31 Salaries Expense . . . . . . . . . . . . . . . . . . . . . . . . . . . . . . . . . . . . 210

Salaries Payable . . . . . . . . . . . . . . . . . . . . . . . . . . . . . . . . 210

Record three days’ accrued salary (3 × $70).

Dec . 12 700

26 700

31 210

Balance 1,610

Salaries Expense 622

Dec. 31 210

Salaries Payable 209

©Plus One Pix/Alamy Stock Photo

Chapter 3 Adjusting Accounts for Financial Statements 95

payday of the next period, the following entry settles the accrued liability (salaries payable) and records salaries expense for seven days of work in January.

The $210 debit is the payment of the liability for the three days’ salary accrued on December 31. The $490 debit records the salary for January’s first seven working days (including the New Year’s Day holiday) as an expense of the new accounting period. The $700 credit records the total amount of cash paid to the employee.

For each separate case below, follow the three-step process for adjusting the accrued expense account at December 31. Assume no other adjusting entries are made during the year.

a. Salaries Payable. At year-end, salaries expense of $5,000 has been incurred by the company but is not yet paid to employees.

b. Interest Payable. At its December 31 year-end, the company holds a mortgage payable that has incurred $1,000 in annual interest that is neither recorded nor paid. The company intends to pay the interest on January 3 of the next year.

Solution

a. Step 1: Salaries Payable equals $0 (before adjustment) Step 2: Salaries Payable should equal $5,000 (not yet recorded) Step 3: Adjusting entry to get from step 1 to step 2

Accrued Expenses

NEED-TO-KNOW 3-3

P3

b. Step 1: Interest Payable equals $0 (before adjustment) Step 2: Interest Payable should equal $1,000 (not yet recorded) Step 3: Adjusting entry to get from step 1 to step 2

Dec . 31 Salaries Expense . . . . . . . . . . . . . . . . . . . . . . . . . . . . . . . . . . . . 5,000

Salaries Payable . . . . . . . . . . . . . . . . . . . . . . . . . . . . . . . . 5,000

Record employee salaries earned but not yet paid.

Do More: QS 3-12, QS 3-13

Dec . 31 Interest Expense . . . . . . . . . . . . . . . . . . . . . . . . . . . . . . . . . . . . 1,000

Interest Payable . . . . . . . . . . . . . . . . . . . . . . . . . . . . . . . . 1,000

Record interest incurred but not yet paid.

Accrued revenues are revenues earned in a period that are both unrecorded and not yet received in cash (or other assets). An example is a technician who bills customers after the job is done. If one-third of a job is complete by the end of a period, then the technician must record one-third of the expected billing as revenue in that period—even though there is no billing or collection.

Framework The adjusting entries for ac- crued revenues increase a revenue (income statement) account and increase an asset (balance sheet) account, as shown in Exhibit 3.11. Accrued revenues usually come from services, products, interest, and rent. We use service fees and interest to show how to adjust for accrued revenues.

ACCRUED REVENUE P4 Prepare adjusting entries for accrued revenues.

Point: Accrued revenues are also called accrued assets.

Dr. Asset…........ # Cr. Revenue… #

Asset Revenue

Increase it Increase it EXHIBIT 3.11 Adjusting for Accrued Revenues (increase an asset and record revenue)

Jan . 9 Salaries Payable (3 days at $70 per day) . . . . . . . . . . . . . . . . 210

Salaries Expense (7 days at $70 per day) . . . . . . . . . . . . . . . . 490

Cash . . . . . . . . . . . . . . . . . . . . . . . . . . . . . . . . . . . . . . . . . 700

Paid two weeks’ salary including three days accrued.

Assets = Liabilities + Equity −700 −210 −490

96 Chapter 3 Adjusting Accounts for Financial Statements

Accrued Services Revenue Accrued revenues are recorded when adjusting entries are made at the end of the accounting period. These accrued revenues are earned but unrecorded because either the buyer has not yet paid or the seller has not yet billed the buyer. FastForward provides an example.

Step 1: In the second week of December, FastForward agreed to provide 30 days of con- sulting services to a fitness club for a fixed fee of $2,700 (or $90 per day). FastForward will provide services from December 12 through January 10, or 30 days of service. The club agrees to pay FastForward $2,700 on January 10 when the service is complete. Step 2: At December 31, 20 days of services have already been provided. Because the con- tracted services have not yet been entirely provided, FastForward has neither billed the club nor recorded the services already provided. Still, FastForward has earned two-thirds of the 30-day fee, or $1,800 ($2,700 × 20∕30). The revenue recognition principle requires FastForward to report the $1,800 on the December income statement. The balance sheet reports that the club owes FastForward $1,800. Step 3: The adjusting entry for accrued services, along with T-account postings, follows.

Accrued Revenues

Jan. 10 Receive cash and reduce receivable

Dec. 31 Record revenue and receivable for services provided but unbilled

Pay me when I'm done

Explanation Accounts receivable are reported on the balance sheet at $1,800, and the $7,850 total of consulting revenue is reported on the income statement. Not making the adjustment Understates consulting revenue by $1,800 in the December income statement. Understates accounts receivable by $1,800 on the December 31 balance sheet.

The following highlights the adjustment for accrued revenue.

Example: What is the adjusting entry if the 30-day consulting period began on December 22? Answer: One-third of the fee is earned: Accounts Receivable . . . . . . . . 900 Consulting Revenue . . . . . . 900

Accounts Receivable = $0

Reports $0 from revenue earned but not yet received in cash .

Add $1,800 to Accounts Receivable Add $1,800 to Consulting Revenue

Record 20 days of earned revenue, which is 20∕30 of total contract .

Accounts Receivable = $1,800

Reports $1,800 in accounts receivable from services provided .

Before Adjustment Adjustment After Adjustment

Accrued Interest Revenue If a company is holding notes receivable that produce interest revenue, we must adjust the accounts to record any earned and yet uncollected interest revenue. The adjusting entry is similar to the one for accruing services revenue. Specifically, debit Interest Receivable (asset) and credit Interest Revenue.

Future Cash Receipt of Accrued Revenues Accrued revenues at the end of one accounting period result in cash receipts in a future period(s). Recall that FastForward made an adjusting entry for $1,800 to record 20 days’ accrued revenue earned from its consulting contract. When FastForward receives $2,700 cash on January 10 for the entire contract amount, it makes the following entry to remove the accrued asset (accounts receivable) and record revenue earned in January. The $2,700 debit is the cash received. The $1,800 credit is the removal of the receivable, and the $900 credit is revenue earned in January.

Assets = Liabilities + Equity +1,800 +1,800

Adjustment (f ) Dec . 31 Accounts Receivable . . . . . . . . . . . . . . . . . . . . . . . . . . . . . . . . . 1,800

Consulting Revenue . . . . . . . . . . . . . . . . . . . . . . . . . . . . . 1,800

Record 20 days’ accrued revenue.

Dec . 12 1,900

31 1,800

Balance 1,800

Dec . 22 1,900

Accounts Receivable 106

Dec . 5 4,200

12 1,600

31 250

31 1,800

Balance 7,850

Consulting Revenue 403

Chapter 3 Adjusting Accounts for Financial Statements 97

Loan Officer The owner of a home theater store applies for a business loan. The store’s financial statements reveal large increases in current-year revenues and income. Increases are due to a promotion that let consumers buy now and pay nothing until January 1 of next year. The store recorded these sales as accrued revenue. Does your analysis raise any concerns? ■ Answer: While increased revenues and income are fine, your concern is with collectibility of these promotional sales. If the store sold products to customers with poor records of paying bills, then collectibility of these sales is low. Your analysis must assess this possibility and estimate losses.

Decision Maker

For each separate case below, follow the three-step process for adjusting the accrued revenue account at December 31. Assume no other adjusting entries are made during the year.

a. Accounts Receivable. At year-end, the company has completed services of $1,000 for a client, but the client has not yet been billed for those services.

b. Interest Receivable. At year-end, the company has earned, but not yet recorded, $500 of interest earned from its investments in government bonds.

Solution

a. Step 1: Accounts Receivable equals $0 (before adjustment) Step 2: Accounts Receivable should equal $1,000 (not yet recorded) Step 3: Adjusting entry to get from step 1 to step 2

Accrued Revenues

NEED-TO-KNOW 3-4

P4

Dec . 31 Accounts Receivable . . . . . . . . . . . . . . . . . . . . . . . . . . . . . . . . . 1,000 Services Revenue . . . . . . . . . . . . . . . . . . . . . . . . . . . . . . . 1,000 Record services revenue earned but not yet received.

b. Step 1: Interest Receivable equals $0 (before adjustment) Step 2: Interest Receivable should equal $500 (not yet recorded) Step 3: Adjusting entry to get from step 1 to step 2

Do More: QS 3-3, QS 3-14

Dec . 31 Interest Receivable . . . . . . . . . . . . . . . . . . . . . . . . . . . . . . . . . . 500 Interest Revenue . . . . . . . . . . . . . . . . . . . . . . . . . . . . . . . 500 Record interest earned but not yet received.

Links to Financial Statements Exhibit 3.12 summarizes the four adjustments. Each adjusting entry affects one or more income statement (revenue or expense) accounts and one or more balance sheet (asset or liability) accounts, but never the Cash account.

Jan . 10 Cash . . . . . . . . . . . . . . . . . . . . . . . . . . . . . . . . . . . . . . . . . . . . . 2,700 Accounts Receivable (20 days at $90 per day) . . . . . . . 1,800 Consulting Revenue (10 days at $90 per day) . . . . . . . 900 Received cash for accrued asset and recorded earned

consulting revenue for January.

Assets = Liabilities + Equity +2,700 +900 −1,800

©Yin Yang/Getty Images

Paid (or received) cash before expense

(or revenue) recognized

Accrued expense

Accruals

Accrued revenue

Deferral of revenue†

Deferral of expense†

Deferrals

Dr. (increase) Expense Cr. (decrease) Asset*

Dr. (decrease) Liability Cr. (increase) Revenue

Dr. (increase) Expense Cr. (increase) Liability

Dr. (increase) Asset Cr. (increase) Revenue

Adjusting Entry

*For depreciation, the credit is to Accumulated Depreciation (contra asset). †Exhibit assumes that deferred expenses are initially recorded as assets and that deferred revenues are initially recorded as liabilities.

Paid (or received) cash after expense

(or revenue) recognized

Expense understated Asset overstated

Expense understated Liability understated

Liability overstated Revenue understated

Asset understated Revenue understated

BEFORE AdjustingFour Adjustments

EXHIBIT 3.12 Summary of Adjustments and Financial Statement Links

98 Chapter 3 Adjusting Accounts for Financial Statements

Adjusted Trial Balance An unadjusted trial balance is a list of accounts and balances before adjustments are recorded. An adjusted trial balance is a list of accounts and balances after adjusting entries have been recorded and posted to the ledger.

Exhibit 3.13 shows both the unadjusted and the adjusted trial balances for FastForward at December 31, 2019. The order of accounts in the trial balance usually matches the order in the chart of accounts. Several new accounts usually arise from adjusting entries.

Each adjustment (see middle columns) has a letter that links it to an adjusting entry explained earlier. Each amount in the Adjusted Trial Balance columns is computed by taking that account’s amount from the Unadjusted Trial Balance columns and adding or subtracting any adjustment(s). To demonstrate, Supplies has a $9,720 Dr. balance in the unadjusted columns. Subtracting the $1,050 Cr. amount shown in the Adjustments columns equals an adjusted $8,670 Dr. balance for Supplies. An account can have more than one adjustment, such as for Consulting Revenue. Also, some accounts might not require adjustment for this period, such as Accounts Payable.

TRIAL BALANCE AND FINANCIAL STATEMENTS P5 Explain and prepare an adjusted trial balance.

Dr. Cr. Dr. Cr.Cr. Dr.

FASTFORWARD Trial Balances

December 31, 2019

Unadjusted Trial Balance Adjustments

Adjusted Trial Balance

(f) $1,800

(d) 250

$ 0

0

0 0

6,200

3,000 30,000

$45,300

5,800

300

0 $ 4,275

9,720 2,400

26,000

0

0

0

1,400

1,000

305 $45,300 $3,710

(c) 300 (e) 210 (a) 100

(b) 1,050

1,800 $ 4,275

8,670 2,300

26,000

300 1,610

100 1,000 1,050

305 $47,610

(b) $1,050 (a) 100

(c) 300

(e) 210

$3,710

(d) 250 (f) 1,800

$ 300 6,200

210 2,750

30,000

$47,610

7,850

300

Acct. No.

Cash Account Title

Accounts receivable Supplies Prepaid insurance Equipment Accumulated depreciation—Equip. Accounts payable Salaries payable Unearned consulting revenue Common stock

101 106 126 128 167 168 201 209 236 307 318 319 403

406 612 622 637 640 652 690

Retained earnings Dividends 200 200 Consulting revenue

Rental revenue Depreciation expense—Equip. Salaries expense Insurance expense Rent expense Supplies expense Utilities expense Totals

EXHIBIT 3.13 Unadjusted and Adjusted Trial Balances

FASTForward

Financial Officer At year-end, the president instructs you, the financial officer, not to record accrued expenses until next year because they will not be paid until then. The president also directs you to record in current-year sales a recent purchase order from a customer that requires merchandise to be delivered two weeks after the year-end. Your company would report a net income instead of a net loss if you follow these instructions. What do you do? ■ Answer: Omitting accrued expenses and recognizing revenue early mislead financial statement users. One action is to explain to the president what is required. If the president persists, you might talk to lawyers and any auditors involved.

Decision Ethics

Information for some adjustments is not available until after the period-end. This means that some adjusting and closing entries are recorded later than, but dated as of, the last day of the period. One example is a company that receives a December utility bill on January 10. When it receives the bill, the company records the expense and the payable as of December 31. The income statement and balance sheet include these adjustments even though amounts were not known at period-end.

Chapter 3 Adjusting Accounts for Financial Statements 99

Preparing Financial Statements We can prepare financial statements directly from information in the adjusted trial balance. Exhibit 3.14 shows how revenue and expense balances are transferred from the adjusted trial balance to the income statement (red lines). The net income and dividends amounts are then used to prepare the statement of retained earnings (black lines). Asset and liability balances are

Steps to Prepare Financial Statements Prepare income statement using revenue and expense accounts from trial balance

Prepare balance sheet using asset and liability accounts along with common stock from trial balance; pull updated retained earnings from step 2

Prepare statement of retained earnings using retained earnings and dividends from trial balance; pull net income from step 1

Prepare statement of cash flows from changes in cash flows for the period (illustrated later in the book)

Step 1

Step 2

Step 4

Step 3

$47,610

Acct. No. Account Title Debit

8,670 2,300

1,800 $ 4,275

26,000

300 1,610

100 1,000 1,050

305

101 Cash ........................................................... Accounts receivable .............................. Supplies .................................................... Prepaid insurance .................................. Equipment ................................................ Accumulated depreciation—Equip..... Accounts payable .................................. Salaries payable .....................................

106 126 128 167 168 201 209

Unearned consulting revenue ............236

Consulting revenue ...............................403 Rental revenue ........................................ 406 Depreciation expense—Equip. ..........612 Salaries expense ....................................622 Insurance expense ................................637 Rent expense ..........................................640 Supplies expense ...................................652 Utilities expense ..................................... Totals .........................................................

690

$ 300

210 6,200

2,750 30,000

300 7,850

$47,610

Credit

Common stock ....................................... Retained earnings .................................

307 318 0

200Dividends .................................................319

Step 2 Prepare statement of retained earnings

$ 3,785

200 $3,585

3,785 Less: Dividends .................................. . . . Retained earnings, December 31 ......

Retained earnings, December 1......... Plus: Net income ................................. . . ..

0

Assets

Liabilities

Equity

Cash ..................................................... $ 4,275

Accounts payable .............................

Accounts receivable ........................ 1,800

Unearned consulting revenue ....... 2,750 Salaries payable ................................ 210

8,670 2,300

Supplies .............................................. Prepaid insurance .............................

Total assets ........................................ $ 42,745 25,700

$26,000

6,200

Equipment .......................................... 300Less accumulated depreciation......

Common stock ................................... Retained earnings .............................. Total equity ......................................... Total liabilities and equity ...............

3,585

Total liabilities ....................................... 9,160

$

30,000

$ 42,745 33,585

Step 1 Prepare income statement

Revenues Consulting revenue ............................. $7,850

300 Rental revenue ..................................... Total revenues ......................................

Depreciation expense—Equip.......... Expenses

Salaries expense................................... Insurance expense............................... Rent expense......................................... Supplies expense ................................. Utilities expense.................................... Total expenses........................................ Net income................................................

300 1,610 100

305

1,000 1,050

4,365 $3,785

$8,150

Step 3 Prepare balance sheet

FASTFORWARD Balance Sheet

December 31, 2019

FASTFORWARD Statement of Retained Earnings

For Month Ended December 31, 2019

FASTFORWARD Income Statement

For Month Ended December 31, 2019

FASTFORWARD Adjusted Trial Balance

December 31, 2019

EXHIBIT 3.14 Preparing Financial Statements (Adjusted Trial Balance from Exhibit 3.13)FASTForward

P6 Prepare financial statements from an adjusted trial balance.

100 Chapter 3 Adjusting Accounts for Financial Statements

then transferred to the balance sheet (blue lines). The ending retained earnings is computed in the statement of retained earnings and transferred to the balance sheet (green line).

We prepare financial statements in the following order: (1) income statement, (2) state- ment of retained earnings, and (3) balance sheet. This order makes sense because the balance sheet uses information from the statement of retained earnings, which in turn uses information from the income statement. The statement of cash flows is usually the final statement prepared.

Point: Each trial balance amount is used in only one financial statement.

Use the following adjusted trial balance of Magic Company to prepare its December 31 year-end (1) in- come statement, (2) statement of retained earnings, and (3) balance sheet (unclassified). The Retained Earnings account balance was $45,000 on December 31 of the prior year.Preparing Financial

Statements from a Trial Balance

NEED-TO-KNOW 3-5

P6

Do More: QS 3-22, E 3-8, P 3-4

Solution

MAGIC COMPANY Adjusted Trial Balance

December 31

Account Title Debit Credit

Cash . . . . . . . . . . . . . . . . . . . . . . . . . . . . . . . . . . . . . . $ 13,000 Accounts receivable . . . . . . . . . . . . . . . . . . . . . . . . . 17,000 Land . . . . . . . . . . . . . . . . . . . . . . . . . . . . . . . . . . . . . . 85,000 Accounts payable . . . . . . . . . . . . . . . . . . . . . . . . . . . $ 12,000 Long-term notes payable . . . . . . . . . . . . . . . . . . . . . 33,000 Common stock . . . . . . . . . . . . . . . . . . . . . . . . . . . . . . 30,000 Retained earnings . . . . . . . . . . . . . . . . . . . . . . . . . . . 45,000 Dividends . . . . . . . . . . . . . . . . . . . . . . . . . . . . . . . . . . 20,000 Fees earned . . . . . . . . . . . . . . . . . . . . . . . . . . . . . . . . 79,000 Salaries expense . . . . . . . . . . . . . . . . . . . . . . . . . . . . 56,000 Office supplies expense . . . . . . . . . . . . . . . . . . . . . . 8,000 Totals . . . . . . . . . . . . . . . . . . . . . . . . . . . . . . . . . . . . . $199,000 $199,000

MAGIC COMPANY Income Statement

For Year Ended December 31

Fees earned . . . . . . . . . . . . . . . . . . . . . . . . $79,000 Expenses Salaries expense . . . . . . . . . . . . . . . . . . $56,000 Office supplies expense . . . . . . . . . . . . 8,000 Total expenses . . . . . . . . . . . . . . . . . . . . 64,000 Net income . . . . . . . . . . . . . . . . . . . . . . . . . $15,000

Step 1

MAGIC COMPANY Statement of Retained Earnings

For Year Ended December 31

Retained earnings, December 31 prior year-end . . . . . . $45,000 Add: Net income . . . . . . . . . . . . . . . . . . . . . . . . . . . . . . . . 15,000 60,000 Less: Dividends . . . . . . . . . . . . . . . . . . . . . . . . . . . . . . . . . 20,000 Retained earnings, December 31 current year-end . . . . $40,000

Step 2

MAGIC COMPANY Balance Sheet December 31

Assets

Cash . . . . . . . . . . . . . . . . . . . . . . . . . . . . $ 13,000 Accounts receivable . . . . . . . . . . . . . . . . 17,000 Land . . . . . . . . . . . . . . . . . . . . . . . . . . . . 85,000 Total assets . . . . . . . . . . . . . . . . . . . . . . $115,000

Liabilities

Accounts payable . . . . . . . . . . . . . . . . . $ 12,000 Long-term notes payable . . . . . . . . . . . . 33,000 Total liabilities . . . . . . . . . . . . . . . . . . . . 45,000

Equity

Common stock . . . . . . . . . . . . . . . . . . . . 30,000 Retained earnings . . . . . . . . . . . . . . . . . . 40,000 Total equity . . . . . . . . . . . . . . . . . . . . . . . 70,000 Total liabilities and equity . . . . . . . . . . . $115,000

Step 3

The closing process occurs at the end of an accounting period after financial statements are completed. In the closing process we (1) identify accounts for closing, (2) record and post the closing entries, and (3) prepare a post-closing trial balance. The closing process has two pur- poses. First, it resets revenue, expense, and dividends account balances to zero at the end of each

CLOSING PROCESS P7 Describe and prepare closing entries.

Chapter 3 Adjusting Accounts for Financial Statements 101

period (which updates the Retained Earnings account for inclusion on the balance sheet). This is done so that these accounts can properly measure income and divi- dends for the next period. Second, it helps summarize a period’s revenues and expenses. This section explains the closing process.

Temporary and Permanent Accounts Temporary accounts relate to one accounting period. They include all income statement accounts, the dividends account, and the Income Sum- mary account. They are temporary because the accounts are opened at the beginning of a period, used to record transactions and events for that period, and then closed at the end of the period. The closing process applies only to temporary accounts.

Permanent accounts report on activities related to one or more future accounting periods. They include asset, liability, and equity accounts (all balance sheet accounts). Permanent accounts are not closed each period and carry their ending balance into future periods.

Recording Closing Entries Closing entries transfer the end-of-period balances in revenue, expense, and dividends accounts to the permanent Retained Earnings account. Closing entries are necessary at the end of each period after financial statements are prepared because Revenue, expense, and dividends accounts must begin each period with zero balances. Retained Earnings must reflect prior periods’ revenues, expenses, and dividends.

An income statement reports revenues and expenses for a specific accounting period. Divi- dends are also for a specific accounting period. Because revenue, expense, and dividends accounts record information separately for each period, they must start each period with zero balances.

Exhibit 3.15 uses the adjusted account balances of FastForward (from the Adjusted Trial Balance columns of Exhibit 3.14 or from the left side of Exhibit 3.16) to show the four steps to close its temporary accounts.

1 2 To close revenue and expense accounts, we transfer their balances to Income Summary. Income Summary is a temporary account only used for the closing process that contains a credit for total revenues (and gains) and a debit for total expenses (and losses).

Point: If Apple did not make closing entries, prior-year revenue from iPhone sales would be in- cluded with current-year revenue.

TEMPORARY

PERMANEN T

Revenues Expenses Dividends Income Summary

Temporary Accounts (closed at period-end)

Assets Liabilities Common Stock Retained Earnings

Permanent Accounts (not closed at period-end)

Consulting Revenue

Rental Revenue

Balance 7,850

Balance 300

4,365 8,150

Balance 3,785

7,850

300

Balance 200 200

Close income statement credit balances1

Close income statement debit balances2

Close Income Summary account3

Close dividends account4

Four-Step Closing Process

Balance 30,000

200 3,785

Balance 33,585

Income Summary

Dividends

Revenue Accounts

Retained Earnings

1

3

4

Expense Accounts Depreciation Expense—Equip.

Balance

Salaries Expense

Balance

Insurance Expense

Balance

Rent Expense

Balance

Supplies Expense

Balance

Utilities Expense

Balance

300

1,610

100

1,000

1,050

305

2

3,785

300

1,610

100

1,000

1,050

305

Point: Retained Earnings is the only permanent account in Exhibit 3.15— meaning it is not closed, but it does have Income Summary closed to it.

EXHIBIT 3.15 Four-Step Closing Process

102 Chapter 3 Adjusting Accounts for Financial Statements

$47,610

8,670 2,300

1,800 $ 4,275

26,000

300 1,610

100 1,000 1,050

305

Cash . . . . . . . . . . . . . . . . . . . . . . . . . . . . . Accounts receivable . . . . . . . . . . . . . . . Supplies . . . . . . . . . . . . . . . . . . . . . . . . . . Prepaid insurance . . . . . . . . . . . . . . . . . Equipment . . . . . . . . . . . . . . . . . . . . . . . . Accumulated depreciation—Equip. . . Accounts payable . . . . . . . . . . . . . . . . .

Common stock . . . . . . . . . . . . . . . . . . . . Retained earnings . . . . . . . . . . . . . . . .

200Dividends . . . . . . . . . . . . . . . . . . . . . . . Consulting revenue . . . . . . . . . . . . . . . . Rental revenue . . . . . . . . . . . . . . . . . . . . Depreciation expense—Equip. . . . . . . Salaries expense . . . . . . . . . . . . . . . . . . Insurance expense . . . . . . . . . . . . . . . . Rent expense . . . . . . . . . . . . . . . . . . . . . Supplies expense . . . . . . . . . . . . . . . . . Utilities expense . . . . . . . . . . . . . . . . . . Totals . . . . . . . . . . . . . . . . . . . . . . . . . . . .

FASTFORWARD Adjusted Trial Balance

December 31, 2019 Dec. 31 Consulting Revenue . . . . . . . . . . . . . . . . . Rental Revenue . . . . . . . . . . . . . . . . . . . . . Income Summary . . . . . . . . . . . . . . . . .

Close revenue accounts.

Close Income Summary account.

7,850 300

1,610

8,150

Step 1:

Dec. 31 Income Summary . . . . . . . . . . . . . . . . . . .

Salaries Expense . . . . . . . . . . . . . . . . . Depreciation Expense—Equip. . . . . .

Insurance Expense . . . . . . . . . . . . . . . Rent Expense . . . . . . . . . . . . . . . . . . . . Supplies Expense . . . . . . . . . . . . . . . . Utilities Expense . . . . . . . . . . . . . . . . .

4,365 300

100 1,000 1,050

305

Step 2:

Dec. 31 Income Summary . . . . . . . . . . . . . . . . . . . Retained Earnings . . . . . . . . . . . . . . . .

3,785 3,785

Step 3:

$ 300 6,200

Salaries payable . . . . . . . . . . . . . . . . . . . 210 Unearned consulting revenue . . . . . . . 2,750

30,000 0

300 7,850

$47,610

Close expense accounts.

CreditDebit

Retained Earnings . . . . . . . . . . . . . . . . . . .

Close dividends account.

Dec. 31 Dividends . . . . . . . . . . . . . . . . . . . . . . .

200 200

Step 4:

General Journal

3 The Income Summary balance, which equals net income or net loss, is transferred to the Retained Earnings account.

4 The Dividends account balance is transferred to the Retained Earnings account. After closing entries are posted, the revenue, expense, dividends, and Income Summary accounts have zero bal- ances and are said to be closed or cleared.

Exhibit 3.16 shows the four closing journal entries to apply the closing process of Exhibit 3.15. EXHIBIT 3.16 Preparing Closing Entries

Step 1: Close Credit Balances in Revenue Accounts to Income Summary The first closing entry transfers credit balances in revenue (and gain) accounts to the Income Summary account. We bring accounts with credit balances to zero by debiting them. For FastForward, this is step 1 in Exhibit 3.16. The $8,150 credit entry to Income Summary equals total revenues for the period. This leaves revenue accounts with zero balances, and they are now ready to record revenues for next period.

Step 2: Close Debit Balances in Expense Accounts to Income Summary The second closing entry transfers debit balances in expense (and loss) accounts to the Income Summary account. We bring expense accounts’ debit balances to zero by crediting them. With a balance of zero, these accounts are ready to record expenses for next period. This second clos- ing entry for FastForward is step 2 in Exhibit 3.16.

Step 3: Close Income Summary to Retained Earnings After steps 1 and 2, the balance of Income Summary equals December net income of $3,785 ($8,150 credit less $4,365 debit). The third closing entry transfers the balance of the Income Summary account to the Retained Earnings account. This entry closes the Income Summary account—see step 3 in Exhibit 3.16. (If a net loss occurred because expenses exceeded revenues, the third entry is re- versed: debit Retained Earnings and credit Income Summary.)

Step 4: Close Dividends Account to Retained Earnings The fourth closing entry transfers any debit balance in the Dividends account to the Retained Earnings account— see step 4 in Exhibit 3.16. This entry gives the Dividends account a zero balance, and the ac- count is now ready to record next period’s dividends.

Exhibit 3.17 shows the entire ledger of FastForward as of December 31 after adjusting and closing entries are posted. The temporary accounts (revenues, expenses, and dividends) have ending balances equal to zero.

103

Asset Accounts

Cash Acct . No . 101

Date Explan . PR Debit Credit Balance

2019

Dec . 1 (1) G1 30,000 30,000

2 (2) G1 2,500 27,500

3 (3) G1 26,000 1,500

5 (5) G1 4,200 5,700

6 (13) G1 2,400 3,300

12 (6) G1 1,000 2,300

12 (7) G1 700 1,600

22 (9) G1 1,900 3,500

24 (10) G1 900 2,600

24 (11) G1 200 2,400

26 (12) G1 3,000 5,400

26 (14) G1 120 5,280

26 (15) G1 305 4,975

26 (16) G1 700 4,275

Supplies Acct . No . 126

Date Explan . PR Debit Credit Balance

2019

Dec . 2 (2) G1 2,500 2,500

6 (4) G1 7,100 9,600

26 (14) G1 120 9,720

31 Adj.(b) G1 1,050 8,670

Accounts Receivable Acct . No . 106

Date Explan . PR Debit Credit Balance

2019

Dec . 12 (8) G1 1,900 1,900

22 (9) G1 1,900 0

31 Adj.(f) G1 1,800 1,800

Accumulated Depreciation— Equipment Acct . No . 168

Date Explan . PR Debit Credit Balance

2019

Dec. 31 Adj.(c) G1 300 300

Equipment Acct . No . 167

Date Explan . PR Debit Credit Balance

2019

Dec . 3 (3) G1 26,000 26,000

Prepaid Insurance Acct . No . 128

Date Explan . PR Debit Credit Balance

2019

Dec . 6 (13) G1 2,400 2,400

31 Adj.(a) G1 100 2,300

Revenue and Expense Accounts (Including Income Summary)

Depreciation Expense— Equipment Acct . No . 612

Date Explan . PR Debit Credit Balance

2019

Dec. 31 Adj.(c) G1 300 300 31 Clos.(2) G1 300   0

Rental Revenue Acct . No . 406

Date Explan . PR Debit Credit Balance

2019

Dec . 12 (8) G1 300 300 31 Clos.(1) G1 300   0

Consulting Revenue Acct . No . 403

Date Explan . PR Debit Credit Balance

2019

Dec . 5 (5) G1 4,200 4,200

12 (8) G1 1,600 5,800

31 Adj.(d) G1 250 6,050 31 Adj.(f) G1 1,800 7,850 31 Clos.(1) G1 7,850         0

Rent Expense Acct . No . 640

Date Explan . PR Debit Credit Balance

2019

Dec . 12 (6) G1 1,000 1,000 31 Clos.(2) G1 1,000   0

Insurance Expense Acct . No . 637

Date Explan . PR Debit Credit Balance

2019

Dec. 31 Adj.(a) G1 100 100 31 Clos.(2) G1 100 0

Salaries Expense Acct . No . 622

Date Explan . PR Debit Credit Balance

2019

Dec . 12 (7) G1 700 700

26 (16) G1 700 1,400

31 Adj.(e) G1 210 1,610 31 Clos.(2) G1 1,610 0

Income Summary Acct . No . 901

Date Explan . PR Debit Credit Balance

2019

Dec. 31 Clos.(1) G1 8,150 8,150 31 Clos.(2) G1 4,365 3,785 31 Clos.(3) G1 3,785           0

Utilities Expense Acct . No . 690

Date Explan . PR Debit Credit Balance

2019

Dec . 26 (15) G1 305 305 31 Clos.(2) G1 305   0

Supplies Expense Acct . No . 652

Date Explan . PR Debit Credit Balance

2019

Dec. 31 Adj.(b) G1 1,050 1,050 31 Clos.(2) G1 1,050         0

EXHIBIT 3.17 General Ledger after the Closing Process for FastForward

FASTForward

Liability and Equity Accounts

Unearned Consulting Revenue Acct . No . 236

Date Explan . PR Debit Credit Balance

2019

Dec . 26 (12) G1 3,000 3,000

31 Adj.(d) G1 250 2,750

Accounts Payable Acct . No . 201

Date Explan . PR Debit Credit Balance

2019

Dec . 6 (4) G1 7,100 7,100

24 (10) G1 900 6,200

Salaries Payable Acct . No . 209

Date Explan . PR Debit Credit Balance

2019

Dec. 31 Adj.(e) G1 210 210

Common Stock Acct . No . 307

Date Explan . PR Debit Credit Balance

2019

Dec . 1 (1) G1 30,000 30,000

Dividends Acct . No . 319

Date Explan . PR Debit Credit Balance

2019

Dec . 24 (11) G1 200 200

31 Clos.(4) G1 200 0

Retained Earnings Acct . No . 318

Date Explan . PR Debit Credit Balance

2019

Dec. 31 Clos.(3) G1 3,785 3,785 31 Clos.(4) G1 200 3,585

104 Chapter 3 Adjusting Accounts for Financial Statements

Post-Closing Trial Balance A post-closing trial balance is a list of permanent accounts and their balances after all closing entries. It lists the balances for all accounts not closed. A post-closing trial balance verifies that (1) total debits equal total credits for permanent accounts and (2) all temporary accounts have zero balances. FastForward’s post-closing trial balance is in Exhibit 3.18 and often is the last step in the accounting process.

P8 Explain and prepare a post-closing trial balance.

Point: Only balance sheet (permanent) accounts are on a post-closing trial balance.

EXHIBIT 3.18 Post-Closing Trial Balance

FASTFORWARD Post-Closing Trial Balance

December 31, 2019 Debit Credit

Cash . . . . . . . . . . . . . . . . . . . . . . . . . . . . . . . . . . . . . . $ 4,275 Accounts receivable . . . . . . . . . . . . . . . . . . . . . . . . . 1,800 Supplies . . . . . . . . . . . . . . . . . . . . . . . . . . . . . . . . . . . 8,670 Prepaid insurance . . . . . . . . . . . . . . . . . . . . . . . . . . . 2,300 Equipment . . . . . . . . . . . . . . . . . . . . . . . . . . . . . . . . . 26,000 Accumulated depreciation—Equipment . . . . . . . . . . $ 300 Accounts payable . . . . . . . . . . . . . . . . . . . . . . . . . . . 6,200 Salaries payable . . . . . . . . . . . . . . . . . . . . . . . . . . . . 210 Unearned consulting revenue . . . . . . . . . . . . . . . . . 2,750 Common stock . . . . . . . . . . . . . . . . . . . . . . . . . . . . . . 30,000 Retained earnings . . . . . . . . . . . . . . . . . . . . . . . . . . . 3,585 Totals . . . . . . . . . . . . . . . . . . . . . . . . . . . . . . . . . . . . . $43,045 $43,045

©IM_photo/Shutterstock

Staff Accountant A friend shows you the post-closing trial balance she is working on. You review the statement and see a line item for rent expense. How do you know that an error exists? ■ Answer: This error is apparent in a post-closing trial balance because Rent Expense is a temporary account. Post-closing trial balances only contain permanent accounts.

Decision Maker

C2 Identify steps in the accounting cycle.

The accounting cycle is the steps in preparing financial statements. It is called a cycle because the steps are repeated each reporting period. Exhibit 3.19 shows the 10 steps in the cycle. Steps 1 through 3 occur regularly as a company enters into transactions. Steps 4 through 9 are done at the end of a period. Reversing entries in step 10 are optional and are explained in Appendix 3C.

ACCOUNTING CYCLE

Use the adjusted trial balance solution for Magic Company from Need-to-Know 3-5 to prepare its closing entries—the accounts are also listed here for convenience.

Cash . . . . . . . . . . . . . . . . . . . . . . . . . . . . . . $13,000 Dr . Accounts receivable . . . . . . . . . . . . . . . . . 17,000 Dr . Land . . . . . . . . . . . . . . . . . . . . . . . . . . . . . . 85,000 Dr . Accounts payable . . . . . . . . . . . . . . . . . . . 12,000 Cr . Long-term notes payable . . . . . . . . . . . . . 33,000 Cr . Common stock . . . . . . . . . . . . . . . . . . . . . 30,000 Cr .

Retained earnings . . . . . . . . . . . . . . . . . . . . $45,000 Cr . Dividends . . . . . . . . . . . . . . . . . . . . . . . . . . . 20,000 Dr . Fees earned . . . . . . . . . . . . . . . . . . . . . . . . . 79,000 Cr . Salaries expense . . . . . . . . . . . . . . . . . . . . . 56,000 Dr . Office supplies expense . . . . . . . . . . . . . . . 8,000 Dr .

Solution

Dec . 31 Fees Earned . . . . . . . . . . . . . . . . . . . . . . . 79,000 Income Summary . . . . . . . . . . . . . . 79,000 Close revenue account. Dec . 31 Income Summary . . . . . . . . . . . . . . . . . . . 64,000 Salaries Expense . . . . . . . . . . . . . . . 56,000 Office Supplies Expense . . . . . . . . . 8,000 Close expense accounts.

Dec . 31 Income Summary . . . . . . . . . . . . . . . . . . . 15,000 Retained Earnings . . . . . . . . . . . . . . 15,000 Close Income Summary. Dec . 31 Retained Earnings . . . . . . . . . . . . . . . . . . 20,000 Dividends . . . . . . . . . . . . . . . . . . . . 20,000 Close Dividends account.

Closing Entries

NEED-TO-KNOW 3-6

P7

Do More: QS 3-18, E 3-9, E 3-10

Chapter 3 Adjusting Accounts for Financial Statements 105

EXHIBIT 3.19 Steps in the Accounting Cycle*

$

33,785

$33,585 200Less: Withdrawals by owner .............

Retained earnings, December 31 .......

Retained earnings, December 1.......... Plus: Investments by owner ............... $30,000

Net income .................................. 3,785

0

FASTFORWARD Statement of Retained Earnings

For Month Ended December 31, 2019

Assets Liabilities Equity

Cash

+ $1,500Old Bal. +

+

+

+

+ Supplies Equipment =

=

Accounts Payable

Common Stock

+ +

+

+

$2,500 $26,000 =

=

$30,000

(4) 7,100 +$7,100 New Bal. $9,600 $ 7,100$1,500 $26,000 $30,000

Date Account Titles and Explanation PR Debit Credit

(4) Supplies 126 7,100 Accounts Payable 201 7,100

(4) 7,100

201

2,500 (4) 7,100

Supplies 126

General Ledger

Accounts Payable

(2)

Cash Accounts receivable Supplies Prepaid insurance Equipment Accounts payable Unearned consulting revenue

6,200 3,000

FASTFORWARD Trial Balance

December 31, 2019

Debit Credit

$

4,275 0

9,720 2,400

26,000

$

Adjustment (b) Supplies Expense . . . . . . . . . . . . . . . . . . . . . . . . . . . . . .

Supplies . . . . . . . . . . . . . . . . . . . . . . . . . . . . . . . . . .

Dec. 31 1,050

652

Dec. 2 2,500 6 7,100

26 120

Balance 8,670

Dec. 31 1,050

126Supplies Expense Supplies

Dec. 31 1,050 1,050

Record supplies used.

Dr. Cr. Dr. Cr.Cr. Dr.

FASTFORWARD Trial Balances

December 31, 2019

Unadjusted Trial Balance Adjustments

Adjusted Trial Balance

(f) $1,800

$ 0

0 6,200

0 $ 4,275

9,720 2,400

26,000

1,800 $ 4,275

8,670 2,300

26,000

(b) $1,050 (a) 100

(c) 300

(e) 210

$ 300 6,200

210

Acct. No.

101 106 126 128 167 168 201 209

Cash Account Title

Accounts receivable Supplies Prepaid insurance Equipment Accumulated depreciation—Equip. Accounts payable Salaries payable

Dec. 31 Consulting Revenue................................ Rental Revenue........................................ Income Summary................................ Close revenue accounts.

7,850 300

1,610

8,150

Step 1:

Dec. 31 Income Summary.....................................

Salaries Expense................................ Depreciation Expense—Equip. .....

Insurance Expense............................ Rent Expense...................................... Supplies Expense.............................. Utilities Expense.................................

4,365 300

100 1,000 1,050

305

Step 2:

Step 3: Close expense accounts.

General Journal

FASTFORWARD Post-Closing Trial Balance

December 31, 2019

Debit Credit

Cash . . . . . . . . . . . . . . . . . . . . . . . . . . . . . . . . . . . . . Accounts receivable . . . . . . . . . . . . . . . . . . . . . . . . Supplies . . . . . . . . . . . . . . . . . . . . . . . . . . . . . . . . . Prepaid insurance . . . . . . . . . . . . . . . . . . . . . . . . . Equipment . . . . . . . . . . . . . . . . . . . . . . . . . . . . . . . Accumulated depreciation—Equipment . . . . . . Accounts payable . . . . . . . . . . . . . . . . . . . . . . . . . S l i bl

$ 4,275 1,800 8,670 2,300

26,000 $ 300

6,200 210

Reversing entry recorded on Jan. 1, 2020

Salaries Expense

Salaries Payable

Date 2020

(e)

Expl. Debit BalanceCredit

Date 2019 Dec. 31 2020 Jan. 1

210210

210 0

Expl. Debit BalanceCredit

Salaries Payable 210 Salaries Expense 210

Jan. 1 210 210

Revenues Consulting revenue .............................. $7,850

300 Rental revenue ....................................... Total revenues .......................................

Depreciation expense—Equip............ Expenses

Salaries expense.................................... Insurance expense................................ Rent expense.......................................... Supplies expense................................... Utilities expense..................................... Total exenses

300 1,610 100

305

1,000 1,050

$8,150

FASTFORWARD Income Statement

For Month Ended December 31, 2019

Assets Cash ........................................................ $ 4,275 Accounts receivable ........................... 1,800

8,670 2,300

Supplies ................................................. Prepaid insurance ...............................

$

FASTFORWARD Balance Sheet

December 31, 2019

Explanations   1. Analyze transactions Analyze transactions to prepare for journalizing.   2. Journalize Record accounts, including debits and credits, in a journal.   3. Post Transfer debits and credits from the journal to the ledger.   4. Prepare unadjusted trial balance Summarize unadjusted ledger accounts and amounts.   5. Adjust and post Record adjustments to bring account balances up to date; journalize and post adjustments.   6. Prepare adjusted trial balance Summarize adjusted ledger accounts and amounts.   7. Prepare financial statements Use adjusted trial balance to prepare financial statements.   8. Close accounts Journalize and post entries to close temporary accounts.   9. Prepare post-closing trial balance Test clerical accuracy of the closing procedures. 10. Reverse and post (optional step) Reverse certain adjustments in the next period—optional step; see Appendix 3C.

* Steps 4, 6, and 9 can be done on a work sheet. A work sheet is useful in planning adjustments, but adjustments (step 5) must always be journalized and posted. Steps 3, 4, 6, and 9 are automatic with a computerized system.

Accounting Cycle

5. Adjust and post accounts

6. Prepare adjusted trial balance7. Prepare financial statements8. Close accounts

9. Prepare post-closing trial balance

10. Reverse and post (optional)

2. Journalize 3. Post

4. Prepare unadjusted trial balance

1. Analyze transactions

C3 Explain and prepare a classified balance sheet.

This section describes a classified balance sheet. An unclassified balance sheet broadly groups accounts into assets, liabilities, and equity. One example is FastForward’s balance sheet in Exhibit 3.14. A classified balance sheet organizes assets and liabilities into subgroups.

Classification Structure A classified balance sheet typically contains the categories in Exhibit 3.20 (there is no required layout). An important classification is the separation between current and noncurrent for both

CLASSIFIED BALANCE SHEET

106 Chapter 3 Adjusting Accounts for Financial Statements

assets and liabilities. Current items are expected to come due (either collected or owed) within one year or the company’s operating cycle, whichever is longer. The operating cycle is the time span from when cash is used to acquire goods and services until cash is received from the sale

of goods and services. Most operating cycles are less than one year, which means most companies use a one-year period to classify current and noncurrent items. To make it easy, assume an operating cycle of one year, unless we say otherwise.

A balance sheet lists current assets before noncurrent assets and cur- rent liabilities before noncurrent liabilities. Current assets and current liabilities are listed in order of how quickly they will be converted to, or paid in, cash.

Classification Categories The balance sheet for Snowboarding Components in Exhibit 3.21 shows the typical categories. Its assets are classified as either current or noncurrent. Its noncurrent assets include three main categories: long-term investments, plant assets, and intangible assets. Its liabilities are classified as either current or long-term. Not all companies use the same categories. Jarden, a producer of snowboards, reported a balance sheet with five asset classes: current assets; property, plant, and equipment; goodwill; intangibles; and other assets.

Current Assets Current assets are cash and other resources that are expected to be sold, collected, or used within one year or the company’s operating cycle, whichever is longer. Examples are cash, short-term investments, accounts receivable, short-term notes receivable, goods for sale (called merchandise or inventory), and prepaid expenses.

Long-Term Investments Long-term (or noncurrent) investments include notes re- ceivable and investments in stocks and bonds when they are expected to be held for more than the longer of one year or the operating cycle. Land held for future expansion is a long-term in- vestment because it is not used in operations.

EXHIBIT 3.20 Typical Categories in a Classified Balance Sheet

Assets Liabilities and Equity

Current assets Current liabilities Noncurrent assets Noncurrent liabilities Long-term investments Equity Plant assets Intangible assets

©Sean Sullivan/Getty Images

Point: Current is also called short- term, and noncurrent is also called long-term.

SNOWBOARDING COMPONENTS Balance Sheet

January 31, 2019

Liabilities Current liabilities Accounts payable . . . . . . . . . . . . . . . . . . . . . . . . $15,300

Wages payable . . . . . . . . . . . . . . . . . . . . . . . . . . 3,200

Notes payable (due within one year) . . . . . . . . . 3,000

Current portion of long-term liabilities . . . . . . . . 7,500

Total current liabilities . . . . . . . . . . . . . . . . . . . . . $ 29,000

Long-term liabilities (net of current portion) . . . . . 150,000 Total liabilities . . . . . . . . . . . . . . . . . . . . . . . . . . . . . . 179,000

Equity Common stock . . . . . . . . . . . . . . . . . . . . . . . . . . . . . 50,000 Retained earnings . . . . . . . . . . . . . . . . . . . . . . . . . .   114,800 Total equity . . . . . . . . . . . . . . . . . . . . . . . . . . . . . . . .   164,800

Total liabilities and equity . . . . . . . . . . . . . . . . . . . . $343,800

Assets Current assets Cash . . . . . . . . . . . . . . . . . . . . . . . . . . . . . . . . . $ 6,500

Short-term investments . . . . . . . . . . . . . . . . . 2,100

Accounts receivable, net . . . . . . . . . . . . . . . . 4,400

Merchandise inventory . . . . . . . . . . . . . . . . . . 27,500

Prepaid expenses . . . . . . . . . . . . . . . . . . . . . . 2,400

Total current assets . . . . . . . . . . . . . . . . . . . . . $ 42,900

Long-term investments Notes receivable (due in three years) . . . . . . 1,500

Investments in stocks and bonds . . . . . . . . . . 18,000

Land held for future expansion . . . . . . . . . . . 48,000

Total long-term investments . . . . . . . . . . . . . . 67,500

Plant assets Equipment and buildings . . . . . . . . . . . . . . . . 203,200

Less accumulated depreciation . . . . . . . . . . . 53,000

Equipment and buildings, net . . . . . . . . . . . . 150,200

Land . . . . . . . . . . . . . . . . . . . . . . . . . . . . . . . . . 73,200

Total plant assets . . . . . . . . . . . . . . . . . . . . . . 223,400

Intangible assets . . . . . . . . . . . . . . . . . . . . . . . . 10,000 Total assets . . . . . . . . . . . . . . . . . . . . . . . . . . . . . $343,800

EXHIBIT 3.21 Example of a Classified Balance Sheet

Chapter 3 Adjusting Accounts for Financial Statements 107

Plant Assets Plant assets are tangible assets that are both long-lived and used to produce or sell products and services. Examples are equipment, machinery, buildings, and land that are used to produce or sell products and services.

Intangible Assets Intangible assets are long-term assets that benefit business op- erations but lack physical form. Examples are patents, trademarks, copyrights, franchises, and goodwill. Their value comes from the privileges or rights granted to or held by the owner.

Current Liabilities Current liabilities are liabilities due to be paid or settled within one year or the operating cycle, whichever is longer. They usually are settled by paying out cash. Current liabilities include accounts payable, notes payable, wages payable, taxes payable, inter- est payable, and unearned revenues. Also, any portion of a long-term liability due to be paid within one year or the operating cycle, whichever is longer, is a current liability. Unearned rev- enues are current liabilities when products or services are to be provided within one year or the operating cycle, whichever is longer.

Long-Term Liabilities Long-term liabilities are liabilities not due within one year or the operating cycle, whichever is longer. Notes payable, mortgages payable, bonds payable, and lease obligations are common long-term liabilities. If a company has both short- and long-term items in each of these categories, they are commonly separated into two accounts in the ledger.

Equity Equity is the owner’s claim on assets. For a corporation, this claim is reported in the equity section as common stock and retained earnings.

©Johannes Simon/Getty Images

Point: Only assets and liabilities (not equity) are classified as current or noncurrent.

Use the following account balances for Magic Company from Need-To-Know 3-5 to prepare its classified balance sheet as of December 31.

Solution

MAGIC COMPANY Balance Sheet December 31

Assets Current assets

Cash . . . . . . . . . . . . . . . . . . . . . . . . . . . . . . $ 13,000

Accounts receivable . . . . . . . . . . . . . . . . . 17,000

Total current assets . . . . . . . . . . . . . . . . . . 30,000

Plant assets

Land . . . . . . . . . . . . . . . . . . . . . . . . . . . . . . 85,000

Total plant assets . . . . . . . . . . . . . . . . . . . . 85,000

Total assets . . . . . . . . . . . . . . . . . . . . . . . . . . $115,000

Liabilities Current liabilities

Accounts payable . . . . . . . . . . . . . . . . . . . . . . $ 12,000

Total current liabilities . . . . . . . . . . . . . . . . . . 12,000

Long-term notes payable . . . . . . . . . . . . . . . . . . 33,000

Total liabilities . . . . . . . . . . . . . . . . . . . . . . . . . . . 45,000

Equity

Common stock . . . . . . . . . . . . . . . . . . . . . . . . . . 30,000

Retained earnings . . . . . . . . . . . . . . . . . . . . . . . 40,000

Total equity . . . . . . . . . . . . . . . . . . . . . . . . . . . . . 70,000

Total liabilities and equity . . . . . . . . . . . . . . . . . $115,000

Cash . . . . . . . . . . . . . . . . . . . . . . . . . . . . . . . . . $13,000 Dr .

Accounts receivable . . . . . . . . . . . . . . . . . . . . 17,000 Dr .

Land . . . . . . . . . . . . . . . . . . . . . . . . . . . . . . . . . 85,000 Dr .

Accounts payable . . . . . . . . . . . . . . . . . . . . . . 12,000 Cr .

Long-term notes payable . . . . . . . . . . . . . . . . 33,000 Cr .

Common stock . . . . . . . . . . . . . . . . . . . . . . . . 30,000 Cr .

Retained earnings . . . . . . . . . . . . . . . . . . . . . . $40,000 Cr .

Dividends . . . . . . . . . . . . . . . . . . . . . . . . . . . . . 20,000 Dr .

Fees earned . . . . . . . . . . . . . . . . . . . . . . . . . . . 79,000 Cr .

Salaries expense . . . . . . . . . . . . . . . . . . . . . . . 56,000 Dr .

Office supplies expense . . . . . . . . . . . . . . . . . 8,000 Dr .

Classified Balance Sheet

NEED-TO-KNOW 3-7

C3

Do More: QS 3-21, QS 3-23, E 3-12, P 3-7

Point: Plant assets are also called fixed assets; property, plant and equipment (PP&E); or long-lived assets.

Profit Margin A useful measure of a company’s operating results is the ratio of its net income to net sales. This ratio is called profit margin, or return on sales, and is computed as in Exhibit 3.22. This ratio shows the percent of profit in each dollar of sales.

A1 Compute profit margin and describe its use in analyzing company performance.

CFO Your health care equipment company consistently reports a 9% profit margin, which is similar to that of com- petitors. The treasurer argues that profit margin can be increased to 20% if the company cuts marketing expenses. Do you cut those expenses? ■ Answer: Cutting those expenses increases profit margin in the short run. However, over the long run, cutting such expenses can hurt current and future sales. You must explain that the company can cut the “fat” (expenses that do not create sales) but should be careful if cutting those that create sales.

Decision Maker

Profit margin = Net income

Net sales

EXHIBIT 3.22 Profit Margin

Visa’s profit margins are shown in Exhibit 3.23. Visa’s profit margin is superior to Mastercard’s in each of the last three years. For Mastercard to improve its profit margin, it must either reduce expenses or increase revenues at a relatively greater amount than expenses.

108 Chapter 3 Adjusting Accounts for Financial Statements

Profit Margin and Current RatioDecision Analysis

EXHIBIT 3.23 Computation and Analysis using Profit Margin

Company Figure ($ millions) Current Year 1 Year Ago 2 Years Ago

Visa Net income . . . . . . . . . . . . . . . . . $ 6,699 $ 5,991 $ 6,328 Net sales . . . . . . . . . . . . . . . . . . $18,358 $15,082 $13,880

Profit margin . . . . . . . . . . . . . . . 36% 40% 46% Mastercard Profit margin . . . . . . . . . . . . . . . 31% 38% 39%

Company Figure ($ millions) Current Year 1 Year Ago 2 Years Ago

Costco Current assets . . . . . . . . . . . . . . . $17,317 $15,218 $16,779 Current liabilities . . . . . . . . . . . . $17,495 $15,575 $16,539

Current ratio . . . . . . . . . . . . . . . 0.99 0.98 1.01 Walmart Current ratio . . . . . . . . . . . . . . . . 0 .86 0 .93 0 .97

Current Ratio An important use of financial statements is to help assess a company’s ability to pay its debts in the near future. Such analysis affects decisions by suppliers when allowing a company to buy on credit. It also affects decisions by creditors when lending money to a company, including loan terms such as interest rate and due date. The current ratio is one measure of a company’s ability to pay its short-term obligations. It is defined in Exhibit 3.24.

A2 Compute the current ratio and describe what it reveals about a company’s financial condition.

Current ratio = Current assets

Current liabilities

EXHIBIT 3.24 Current Ratio

Costco’s current ratio for each of the last three years is in Exhibit 3.25. A current ratio of over 1.0 means that current obligations can be covered with current assets. For the recent two years, Costco’s current ratio was slightly below 1.0. This means Costco could face challenges in covering current liabilities. Although Costco has a better ratio than Walmart in each of the last three years, management must continue to monitor current assets and liabilities.

EXHIBIT 3.25 Computation and Analysis using Current Ratio

Analyst You are analyzing a dirt bike company’s ability to meet upcoming loan payments. You compute its current ratio as 1.2. You find that a major portion of accounts receivable is due from one client who has not made any pay- ments in the past 12 months. Removing this receivable from current assets lowers the current ratio to 0.7. What do you conclude? ■ Answer: A current ratio of 1.2 suggests that current assets are sufficient to cover current liabilities. Removing the past-due receiv- able reduces the current ratio to 0.7. You conclude that the company will have difficulty meeting its loan payments.

Decision Maker

©sgpage902/Getty Images

Chapter 3 Adjusting Accounts for Financial Statements 109

The following information relates to Fanning’s Electronics on December 31, 2019. The company, which uses the calendar year as its annual reporting period, initially records prepaid and unearned items in bal- ance sheet accounts (assets and liabilities, respectively).

a. The company’s weekly payroll is $8,750, paid each Friday for a five-day workweek. Assume December 31, 2019, falls on a Monday, but the employees will not be paid their wages until Friday, January 4, 2020.

b. Eighteen months earlier, on July 1, 2018, the company purchased equipment that cost $20,000. Its useful life is predicted to be five years, at which time the equipment is expected to be worthless (zero salvage value).

c. On October 1, 2019, the company agreed to work on a new housing development. The company is paid $120,000 on October 1 in advance of future installation of similar alarm systems in 24 new homes. That amount was credited to the Unearned Services Revenue account. Between October 1 and December 31, work on 20 homes was completed.

d. On September 1, 2019, the company purchased a 12-month insurance policy for $1,800. The transac- tion was recorded with an $1,800 debit to Prepaid Insurance.

e. On December 29, 2019, the company completed a $7,000 service that has not been billed or recorded as of December 31, 2019.

Required

1. Prepare any necessary adjusting entries on December 31, 2019, in relation to transactions and events a through e.

2. Prepare T-accounts for the accounts affected by adjusting entries, and post the adjusting entries. Determine the adjusted balances for the Unearned Revenue and the Prepaid Insurance accounts.

3. Complete the following table and determine the amounts and effects of your adjusting entries on the year 2019 income statement and the December 31, 2019, balance sheet. Use up (down) arrows to indicate an increase (decrease) in the Effect columns.

COMPREHENSIVE 1

Preparing Year-End Accounting Adjustments

NEED-TO-KNOW 3-8

Amount in Effect on Effect on Effect on Effect on Entry the Entry Net Income Total Assets Total Liabilities Total Equity

PLANNING THE SOLUTION Analyze each situation to determine which accounts need to be updated with an adjustment. Calculate the amount of each adjustment and prepare the necessary journal entries. Show the amount of each adjustment in the designated accounts, determine the adjusted balance, and

identify the balance sheet classification of the account. Determine each entry’s effect on net income for the year and on total assets, total liabilities, and total

equity at the end of the year.

SOLUTION 1. Adjusting journal entries.

(a) Dec . 31 Wages Expense . . . . . . . . . . . . . . . . . . . . . . . . . . . . . . . . . . . . . 1,750 Wages Payable . . . . . . . . . . . . . . . . . . . . . . . . . . . . . . . . . 1,750 Accrue wages for last day of year ($8,750 × 1∕5) . (b) Dec . 31 Depreciation Expense—Equipment . . . . . . . . . . . . . . . . . . . . . 4,000 Accumulated Depreciation—Equipment . . . . . . . . . . . . . 4,000 Record depreciation expense for year

($20,000/5 years = $4,000 per year) . (c) Dec . 31 Unearned Services Revenue . . . . . . . . . . . . . . . . . . . . . . . . . . 100,000 Services Revenue . . . . . . . . . . . . . . . . . . . . . . . . . . . . . . . 100,000 Record revenue earned ($120,000 × 20∕24) . (d) Dec . 31 Insurance Expense . . . . . . . . . . . . . . . . . . . . . . . . . . . . . . . . . . 600 Prepaid Insurance . . . . . . . . . . . . . . . . . . . . . . . . . . . . . . . 600 Adjust for expired portion of insurance ($1,800 × 4∕12) . (e) Dec . 31 Accounts Receivable . . . . . . . . . . . . . . . . . . . . . . . . . . . . . . . . . 7,000 Services Revenue . . . . . . . . . . . . . . . . . . . . . . . . . . . . . . . 7,000 Record services revenue earned.

110 Chapter 3 Adjusting Accounts for Financial Statements

2. T-accounts for adjusting journal entries a through e.

(a) 1,750

Wages ExpenseWages Payable

(a) 1,750

Accounts Receivable

(e) 7,000

(d ) 600

Insurance ExpenseUnearned Services Revenue

Unadj . Bal . 120,000

(c) 100,000

Adj . Bal . 20,000

Unadj . Bal . 1,800

(d ) 600

Adj . Bal . 1,200

Prepaid Insurance

(b) 4,000

Depreciation Expense — EquipmentServices Revenue

(c) 100,000

(e) 7,000

Adj . Bal . 107,000

Accumulated Depreciation — Equipment

(b) 4,000

3. Financial statement effects of adjusting journal entries.

Amount in Effect on Effect on Effect on Effect on Entry the Entry Net Income Total Assets Total Liabilities Total Equity

a $ 1,750 $ 1,750 ↓ No effect $ 1,750 ↑ $ 1,750 ↓ b 4,000 4,000 ↓ $4,000 ↓ No effect 4,000 ↓ c 100,000 100,000 ↑ No effect $100,000 ↓ 100,000 ↑ d 600 600 ↓ $ 600 ↓ No effect 600 ↓ e 7,000 7,000 ↑ $7,000 ↑ No effect 7,000 ↑

Use the following year-end adjusted trial balance to answer questions 1–3.

COMPREHENSIVE 2

Preparing Financial Statements from Adjusted Account Balances

NEED-TO-KNOW 3-9 CHOI COMPANY

Adjusted Trial Balance December 31

Debit Credit

Cash . . . . . . . . . . . . . . . . . . . . . . . . . . . . . . . . . . . . . . . . . . . . . . . . $ 3,050 Accounts receivable . . . . . . . . . . . . . . . . . . . . . . . . . . . . . . . . . . . 400 Prepaid insurance . . . . . . . . . . . . . . . . . . . . . . . . . . . . . . . . . . . . . 910 Equipment . . . . . . . . . . . . . . . . . . . . . . . . . . . . . . . . . . . . . . . . . . . 217,200 Accumulated depreciation—Equipment . . . . . . . . . . . . . . . . . . . $ 29,100 Interest payable . . . . . . . . . . . . . . . . . . . . . . . . . . . . . . . . . . . . . . 4,480 Unearned rent . . . . . . . . . . . . . . . . . . . . . . . . . . . . . . . . . . . . . . . . 460 Long-term notes payable . . . . . . . . . . . . . . . . . . . . . . . . . . . . . . . 150,000 Common stock . . . . . . . . . . . . . . . . . . . . . . . . . . . . . . . . . . . . . . . . 10,000 Retained earnings . . . . . . . . . . . . . . . . . . . . . . . . . . . . . . . . . . . . . 30,340 Dividends . . . . . . . . . . . . . . . . . . . . . . . . . . . . . . . . . . . . . . . . . . . . 21,000 Rent earned . . . . . . . . . . . . . . . . . . . . . . . . . . . . . . . . . . . . . . . . . . 57,500 Wages expense . . . . . . . . . . . . . . . . . . . . . . . . . . . . . . . . . . . . . . . 25,000 Utilities expense . . . . . . . . . . . . . . . . . . . . . . . . . . . . . . . . . . . . . . 1,900 Insurance expense . . . . . . . . . . . . . . . . . . . . . . . . . . . . . . . . . . . . 3,450 Depreciation expense—Equipment . . . . . . . . . . . . . . . . . . . . . . . 5,970 Interest expense . . . . . . . . . . . . . . . . . . . . . . . . . . . . . . . . . . . . . . 3,000 Totals . . . . . . . . . . . . . . . . . . . . . . . . . . . . . . . . . . . . . . . . . . . . . . . $281,880 $281,880

Chapter 3 Adjusting Accounts for Financial Statements 111

1. Prepare the annual income statement from the adjusted trial bal- ance of Choi Company.

Answer:

CHOI COMPANY Income Statement

For Year Ended December 31

Revenues Rent earned . . . . . . . . . . . . . . . . . . . . . . . . . . . . . . . . . $57,500 Expenses Wages expense . . . . . . . . . . . . . . . . . . . . . . . . . . . . . . $25,000 Utilities expense . . . . . . . . . . . . . . . . . . . . . . . . . . . . . 1,900 Insurance expense . . . . . . . . . . . . . . . . . . . . . . . . . . . 3,450 Depreciation expense—Equipment . . . . . . . . . . . . . . 5,970 Interest expense . . . . . . . . . . . . . . . . . . . . . . . . . . . . . 3,000 Total expenses . . . . . . . . . . . . . . . . . . . . . . . . . . . . . . . 39,320 Net income . . . . . . . . . . . . . . . . . . . . . . . . . . . . . . . . . . . . $18,180

2. Prepare a statement of retained earnings from the adjusted trial balance of Choi Company.

Answer:

CHOI COMPANY Statement of Retained Earnings

For Year Ended December 31

Retained earnings, December 31 prior year-end . . . . . . $30,340 Plus: Net income . . . . . . . . . . . . . . . . . . . . . . . . . . . . . . 18,180

48,520 Less: Dividends . . . . . . . . . . . . . . . . . . . . . . . . . . . . . . . 21,000 Retained earnings, December 31 current year-end . . . . $27,520

3. Prepare a balance sheet (unclassified) from the adjusted trial balance of Choi Company.

Answer:

CHOI COMPANY Balance Sheet December 31

Assets Cash . . . . . . . . . . . . . . . . . . . . . . . . . . . . . . . . . $ 3,050 Accounts receivable . . . . . . . . . . . . . . . . . . . . 400 Prepaid insurance . . . . . . . . . . . . . . . . . . . . . . 910 Equipment . . . . . . . . . . . . . . . . . . . . . . . . . . . . $217,200 Less accumulated depreciation . . . . . . . . . . . . 29,100 188,100 Total assets . . . . . . . . . . . . . . . . . . . . . . . . . . . $192,460

Liabilities Interest payable . . . . . . . . . . . . . . . . . . . . . . . . $ 4,480 Unearned rent . . . . . . . . . . . . . . . . . . . . . . . . . 460 Long-term notes payable . . . . . . . . . . . . . . . . 150,000 Total liabilities . . . . . . . . . . . . . . . . . . . . . . . . . 154,940

Equity Common stock . . . . . . . . . . . . . . . . . . . . . . . . . 10,000 Retained earnings . . . . . . . . . . . . . . . . . . . . . . . 27,520 Total equity . . . . . . . . . . . . . . . . . . . . . . . . . . . . 37,520 Total liabilities and equity . . . . . . . . . . . . . . . . . $192,460

APPENDIX

Alternative Accounting for Prepayments 3A This appendix explains alternative accounting for deferred expenses and deferred revenues.

RECORDING PREPAYMENT OF EXPENSES IN EXPENSE ACCOUNTS An alternative method is to record all prepaid expenses with debits to expense accounts. If any prepaids remain unused or unexpired at the end of an accounting period, then adjusting entries transfer the cost of the unused portions from expense accounts to prepaid expense (asset) accounts. The financial statements are identical under either method, but the adjusting entries are different. To demonstrate the differences between these two methods, let’s look at FastForward’s cash payment on December 1 for 24 months of insurance coverage beginning on December 1. FastForward recorded that payment with a debit to an as- set account, but it could have recorded a debit to an expense account. These alternatives are shown in Exhibit 3A.1.

P9 Explain the alternatives in accounting for prepaids.

Payment Recorded as Asset

Dec . 1 Prepaid Insurance . . . . . . . 2,400 Cash . . . . . . . . . . . . . . 2,400

Payment Recorded as Expense

Dec . 1 Insurance Expense . . . . . . 2,400 Cash . . . . . . . . . . . . . 2,400

EXHIBIT 3A.1 Alternative Initial Entries for Prepaid Expenses

112 Chapter 3 Adjusting Accounts for Financial Statements

Payment Recorded as Asset

Dec . 31 Insurance Expense . . . . . . . . 100 Prepaid Insurance . . . . 100

Payment Recorded as Expense

Dec . 31 Prepaid Insurance . . . . . . . 2,300 Insurance Expense . . 2,300

EXHIBIT 3A.2 Adjusting Entry for Prepaid Expenses for the Two Alternatives

When these entries are posted, we see in Exhibit 3A.3 that the two methods give identical results.

EXHIBIT 3A.3 Account Balances under Two Alternatives for Recording Prepaid Expenses

Payment Recorded as Asset

Prepaid Insurance 128

Dec . 1 2,400 Dec . 31 100

Balance 2,300

Prepaid Insurance 128

Dec . 31 2,300

Payment Recorded as Expense

Insurance Expense 637

Dec . 31 100

Insurance Expense 637

Dec . 1 2,400 Dec . 31 2,300

Balance 100

RECORDING PREPAYMENT OF REVENUES IN REVENUE ACCOUNTS An alternative method is to record all unearned revenues with credits to revenue accounts. If any revenues are unearned at the end of an accounting period, then adjusting entries transfer the unearned portions from revenue accounts to unearned revenue (liability) accounts. The adjusting entries are different for these two alternatives, but the financial statements are identical. To demonstrate the differences between these two methods, let’s look at FastForward’s December 26 receipt of $3,000 for consulting services covering the period December 27 to February 24. FastForward recorded this transaction with a credit to a liability account. The alternative is to record it with a credit to a revenue account, as shown in Exhibit 3A.4.

Receipt Recorded as Liability

Dec . 26 Cash . . . . . . . . . . . . . . . . . . . . . . . . . . . . 3,000 Unearned Consulting Revenue . . . 3,000

Receipt Recorded as Revenue

Dec . 26 Cash . . . . . . . . . . . . . . . . . . . 3,000 Consulting Revenue . . 3,000

EXHIBIT 3A.4 Alternative Initial Entries for Unearned Revenues

By the end of its accounting period on December 31, FastForward has earned $250 of this revenue. This means $250 of the liability has been satisfied. Depending on how the initial receipt is recorded, the adjust- ing entry is as shown in Exhibit 3A.5.

After adjusting entries are posted, the two alternatives give identical results, as shown in Exhibit 3A.6.

Receipt Recorded as Revenue

Dec . 31 Consulting Revenue . . . . . . . . . . . . . . . 2,750 Unearned Consulting Revenue . . 2,750

Receipt Recorded as Liability

Dec . 31 Unearned Consulting Revenue . 250 Consulting Revenue . . . . . . 250

EXHIBIT 3A.5 Adjusting Entry for Unearned Revenues for the Two Alternatives

EXHIBIT 3A.6 Account Balances under Two Alternatives for Recording Unearned Revenues

Unearned Consulting Revenue 236

Dec . 31 2,750

Unearned Consulting Revenue 236

Dec . 31 250 Dec . 26 3,000

Balance 2,750

Receipt Recorded as RevenueReceipt Recorded as Liability

Consulting Revenue 403

Dec . 31 2,750 Dec . 26 3,000

Balance 250

Consulting Revenue 403

Dec . 31 250

At the end of its accounting period on December 31, insurance protection for one month has expired. This means $100 ($2,400∕24) of insurance coverage expired and is an expense for December. The adjusting entry depends on how the original payment was recorded. This is shown in Exhibit 3A.2.

Chapter 3 Adjusting Accounts for Financial Statements 113

APPENDIX

Work Sheet as a Tool 3B Benefits of a Work Sheet (Spreadsheet) A work sheet is a document that is used internally by companies to help with adjusting and closing accounts and with preparing financial state- ments. It is an internal accounting aid and is not a substitute for journals, ledgers, or financial statements. A work sheet

Helps in preparing financial statements. Reduces the risk of errors when working with many accounts and adjustments. Links accounts and adjustments to financial statements. Shows the effects of proposed or “what-if” transactions.

Use of a Work Sheet When a work sheet is used to prepare financial statements, it is con- structed at the end of a period before the adjusting process. The complete work sheet includes a list of the accounts, their balances and adjustments, and their sorting into financial statement columns. It provides two columns each for the unadjusted trial balance, the adjustments, the adjusted trial balance, the income statement, and the balance sheet. To describe and interpret the work sheet, we use the information from FastForward. Preparing the work sheet has five steps.

1 Step 1. Enter Unadjusted Trial Balance Refer to Exhibit 3B.1—green section. The first step in preparing a work sheet is to list the title of every account and its account number that appears on its financial statements. This includes all accounts in the ledger plus any new ones from adjusting entries. The unadjusted balance for each account is then entered in the correct Debit or Credit column of the Unadjusted Trial Balance columns. The totals of these two columns must be equal. The light green section of Exhibit 3B.1 shows FastForward’s work sheet after completing this first step (dark green rows show accounts that arise because of the adjustments). Sometimes an account can require more than one adjustment, such as for Consulting Revenue. The addi- tional adjustment can be added to a blank line below (as in Exhibit 3B.1), squeezed on one line, or com- bined into one adjustment amount.

2 Step 2. Enter Adjustments Exhibit 3B.1—yellow section. The second step is to enter adjustments in the Adjustments columns. The adjustments shown are the same ones shown in Exhibit 3.13. An identifying letter links the debit and credit of each adjustment. This is called keying the adjustments. After preparing a work sheet, adjust- ments must still be entered in the journal and posted to the ledger. The Adjustments columns provide the information for adjusting entries in the journal.

3 Step 3. Prepare Adjusted Trial Balance Exhibit 3B.1—blue section. The adjusted trial balance is prepared by combining the adjustments with the unadjusted balances for each account. As an example, the Prepaid Insurance account has a $2,400 debit balance in the Unadjusted Trial Balance columns. This $2,400 debit is combined with the $100 credit in the Adjustments columns to give Prepaid Insurance a $2,300 debit in the Adjusted Trial Balance columns. The totals of the Adjusted Trial Balance columns confirm debits and credits are equal.

4 Step 4. Sort Adjusted Trial Balance Amounts to Financial Statements Exhibit 3B.1—orange section. This step involves sorting account balances from the adjusted trial balance to their proper financial statement columns. Expenses go to the Income Statement Debit column and rev- enues to the Income Statement Credit column. Assets and dividends go to the Balance Sheet Debit col- umn. Liabilities, retained earnings, and common stock go to the Balance Sheet Credit column.

5 Step 5. Total Statement Columns, Compute Income or Loss, and Balance Columns Exhibit 3B.1—purple section. Each financial statement column (from step 4) is totaled. The differ- ence between the Debit and Credit column totals of the Income Statement columns is net income or net loss. This occurs because revenues are entered in the Credit column and expenses in the Debit column. If the Credit total exceeds the Debit total, there is net income. If the Debit total exceeds the Credit total, there is a net loss. For FastForward, the Credit total exceeds the Debit total, giving a $3,785 net income.

P10 Prepare a work sheet and explain its usefulness.

FASTForward

114 Chapter 3 Adjusting Accounts for Financial Statements

8,150

8,150

7,850

300

4,365 3,785

8,150

300 1,610

100 1,000 1,050

305

4

43,245

43,245

4,275 1,800 8,670 2,300

26,000

200

39,460 3,785

43,245

300 6,200

210 2,750

30,000 0

300 6,200

210 2,750

30,000 0

7,850 200

300

47,610

4,275 1,800 8,670 2,300

26,000

300 1,610

100 1,000 1,050

305 47,610

3

(b) (a)

(c)

(e)

1,050 100

300

210

(d) (f)

250 1,800

3,710

(f)

(d)

2

1,800

250

(c) (e) (a)

(b)

300 210 100

1,050

3,710

0

0

Account

Unadjusted Trial Balance

Adjusted Trial Balance

Dr.

Adjustments

Cr. Dr. Cr.

Income Statement

Dr. Cr.

Balance Sheet

Dr. Cr.Dr. Cr.No.

1 1 2 3 4 5 6 7 8 9 10 11 12 13 14 15 16 17 18 19 20 21 22 23 24 25 26 27 28 29 30 31

Enter all amounts available from ledger accounts. Column totals must be equal.

A work sheet organizes information used to prepare adjusting entries, financial statements, and closing entries.

5

List all accounts from the ledger; accounts necessary to make accounting adjustments are shaded in dark green.

1a 1b 4a 4b

5c

3

5a 2

5b

Enter adjustment amounts and use letters to cross-reference debit and credit adjustments. Column totals must be equal.

Combine unadjusted trial balance amounts with the adjustments to get the adjusted trial balance amounts. Column totals must be equal.

Extend all revenue and expense amounts to the income statement columns.

Extend all asset, liability, equity, and dividends amounts to these columns.

Enter two new lines for the (1) Net income or loss. (2) Totals.

Net income (loss) is extended to the credit (debit) column.

First “Totals” row for income statement columns dier by the amount of net income or net loss.

Cash Accounts receivable Supplies Prepaid insurance Equipment Accumulated depreciation—Equip. Accounts payable Salaries payable Unearned consulting revenue Common stock Retained earnings Dividends Consulting revenue

Rental revenue Depreciation expense—Equip. Salaries expense Insurance expense Rent expense Supplies expense Utilities expense Totals Net income Totals

101 106 126 128 167

168 201

209 236 307 318 319 403

406 612 622 637 640 652 690

5d Ending balance of retained earnings is computed in the statement of retained earnings.

FastForward Work Sheet

For Month Ended December 31, 2019

A B C D E F G H I J K L M

6,200

3,000

30,000 0

5,800

300

45,300

4,275

9,720 2,400

26,000

200

1,400

1,000

305 45,300

EXHIBIT 3B.1 Work Sheet with Five-Step Process for Completion FASTForward

Chapter 3 Adjusting Accounts for Financial Statements 115

The net income from the Income Statement columns is then entered in the Balance Sheet Credit col- umn. Adding net income to the last Credit column means that it is to be added to retained earnings. If a loss occurs, it is added to the Debit column. This means that it is to be subtracted from retained earnings. The ending balance of retained earnings does not appear in the last two columns as a single amount, but it is computed in the statement of retained earnings using these account balances. When net in- come or net loss is added to the proper Balance Sheet column, the totals of the last two columns must balance. If they do not, one or more errors have occurred.

Work Sheet Applications and Analysis A work sheet does not substitute for financial statements. It is a tool we use to help prepare financial statements. FastForward’s financial statements are shown in Exhibit 3.14. Its income statement amounts are taken from the Income Statement columns of the work sheet. Amounts for its balance sheet and its statement of retained earnings are taken from the Balance Sheet columns of the work sheet. Work sheets are also useful in analyzing the effects of proposed, or what-if, transactions. This is done by entering financial statement amounts in the Unadjusted (what-if) columns. Proposed transactions are then entered in the Adjustments columns. We then compute “adjusted” amounts from these proposed transactions. The extended amounts in the financial statement columns produce pro forma financial statements because they show the statements as if the proposed transactions had occurred.

APPENDIX

Reversing Entries 3C Reversing entries are optional. They are recorded in response to accrued assets and accrued liabilities that were created by adjusting entries at the end of a reporting period. Reversing entries simplify re- cordkeeping. Exhibit 3C.1 shows an example of FastForward’s reversing entries. The top of the exhibit shows the adjusting entry FastForward recorded on December 31 for its employee’s earned but unpaid salary. The entry recorded three days’ salary of $210, which increased December’s total salary expense to $1,610. The entry also recognized a liability of $210. The expense is reported on December’s income statement. The expense account is then closed. The ledger on January 1, 2020, shows a $210 liability and a zero balance in the Salaries Expense account. At this point, the choice is made between using or not using reversing entries.

Accounting without Reversing Entries The path down the left side of Exhibit 3C.1 is described in the chapter. To summarize, when the next payday occurs on January 9, we record payment with a compound entry that debits both the expense and liability accounts and credits Cash. Posting that entry creates a $490 balance in the expense account and reduces the liability account balance to zero because the payable has been settled.

Accounting with Reversing Entries The right side of Exhibit 3C.1 shows reversing entries. A reversing entry is the exact opposite of an adjusting entry. For FastForward, the Salaries Payable liability account is debited for $210, meaning that this account now has a zero balance after the entry is posted on January 1. The Salaries Payable account temporarily understates the liability, but this is not a problem because financial statements are not prepared before the liability is settled on January 9. The credit to the Salaries Expense account is unusual because it gives the account an abnormal credit balance. We highlight an abnormal balance by circling it. Because of the reversing entry, the January 9 entry to record payment debits the Salaries Expense account and credits Cash for the full $700 paid. It is the same as all other entries made to record 10 days’ salary for the employee. We see that after the payment entry is posted, the Salaries Expense account has a $490 balance that reflects seven days’ salary of $70 per day (see the lower right side of Exhibit 3C.1). The zero balance in the Salaries Payable account is now correct. The lower section of Exhibit 3C.1 shows that the expense and liability accounts have exactly the same balances whether reversing entries are used or not.

P11 Prepare reversing entries and explain their purpose.

Point: Adjusting entries that cre- ate new asset or liability accounts likely require reversing.

116 Chapter 3 Adjusting Accounts for Financial Statements

Insurance Expense . . . . . . . . . . . . . . . . . . . . . . . . . . . . . 100 Prepaid Insurance . . . . . . . . . . . . . . . . . . . . . . . . . . 100

Prepaid insurance expires:

Supplies are used up:

Supplies Expense . . . . . . . . . . . . . . . . . . . . . . . . . . . . . . 1,050 Supplies . . . . . . . . . . . . . . . . . . . . . . . . . . . . . . . . . 1,050

Depreciation of assets:

Depreciation Expense . . . . . . . . . . . . . . . . . . . . . . . . . . . 300 Accumulated Depreciation—Equipment . . . . . . . . . 300

Salaries Expense

Accrue salaries expense on December 31, 2019

No reversing entry recorded on Jan. 1, 2020

WITHOUT Reversing Entries WITH Reversing Entries

Under both approaches, the expense and liability accounts have identical balances after the cash payment on January 9.

210 Salaries Payable 210

Salaries Expense

Salaries Payable

Date 2019 Dec. 12 700 700(7)

26 31

700 1,400 1,610

(16) 210(e)

(e)

Expl. Debit BalanceCredit

Date 2019 Dec. 31 210210

Expl. Debit BalanceCredit

Salaries Expense $490 Salaries Payable $ 0

Reversing entry recorded on Jan. 1, 2020— OR —

*Circled numbers in the Balance column indicate abnormal balances.

Salaries Expense 490 Salaries Payable 210

Cash 700 Salaries Expense

Salaries Payable

Date 2020 Jan. 9 490 490

(e)

Expl. Debit BalanceCredit

Date 2019 Dec. 31 210210

Expl. Debit BalanceCredit

NO ENTRY

Salaries Expense

Salaries Payable

Date 2020

(e)

Expl. Debit BalanceCredit

Date 2019 Dec. 31 2020

210210

2020 Jan. 9 0210

Expl. Debit BalanceCredit

Salaries Expense 700 Cash 700

Salaries Expense*

Salaries Payable

Date 2020 Jan. 1 Jan. 9 700 490

(e)

Expl. Debit

210

BalanceCredit

Date 2019 Dec. 31 210210 2020 Jan. 1 0210

Expl. Debit BalanceCredit

Salaries Expense*

Salaries Payable

Date 2020

(e)

Expl. Debit BalanceCredit

Date 2019 Dec. 31 2020 Jan. 1

210210

210 0

Expl. Debit BalanceCredit

Salaries Payable 210 Salaries Expense 210

Jan. 1 210

210

210

Pay the accrued and current salaries on January 9, the first payday in 2020

EXHIBIT 3C.1 Reversing Entries for an Accrued Expense

DEFERRAL OF EXPENSE Prepaid expenses: Assets paid for in advance of receiving their benefits. When these assets are used, the advance payments become expenses.

Summary: Cheat Sheet

Accumulated depreciation: A separate contra account. A contra account is an account linked with another account. It has an opposite normal bal- ance and is a subtraction from that other account’s balance.

Chapter 3 Adjusting Accounts for Financial Statements 117

ACCRUED EXPENSE Accrued expenses: Costs incurred in a period that are both unpaid and unrecorded. They are reported on the income statement for the period when incurred.

Salaries Expense . . . . . . . . . . . . . . . . . . . . . . . . . . . . . . . 210 Salaries Payable . . . . . . . . . . . . . . . . . . . . . . . . . . . 210

Salaries expense owed but not yet paid:

Record unearned revenue (cash received in advance):

Cash . . . . . . . . . . . . . . . . . . . . . . . . . . . . . . . . . . . . . . . . . 3,000 Unearned Consulting Revenue . . . . . . . . . . . . . . . . 3,000

Reduce unearned revenue (products or services are provided):

Unearned Consulting Revenue . . . . . . . . . . . . . . . . . . . . 250 Consulting Revenue . . . . . . . . . . . . . . . . . . . . . . . . 250

DEFERRAL OF REVENUE Unearned revenue: Cash received in advance of providing products and services. When cash is accepted, the company has a liability to provide products or services.

Accrued interest formula:

Principal amount owed × Annual interest rate × Fraction of year since last payment

Salaries Payable (3 days at $70 per day) . . . . . . . . . . . . 210 Salaries Expense (7 days at $70 per day) . . . . . . . . . . . 490 Cash . . . . . . . . . . . . . . . . . . . . . . . . . . . . . . . . . . . . 700

Payment of accrued expenses:

REPORTING AND ANALYSIS Unadjusted trial balance: A list of ledger accounts and balances before adjustments are recorded. Adjusted trial balance: A list of accounts and balances after adjusting entries have been recorded and posted to the ledger.

Revenue earned but not received in cash:

Accounts Receivable . . . . . . . . . . . . . . . . . . . . . . . . . . . . 1,800 Consulting Revenue . . . . . . . . . . . . . . . . . . . . . . . . 1,800

Receipt of accrued revenue:

Cash . . . . . . . . . . . . . . . . . . . . . . . . . . . . . . . . . . . . . . . . . 2,700 Accounts Receivable (20 days at $90 per day) . . . 1,800 Consulting Revenue (10 days at $90 per day) . . . 900

ACCRUED REVENUE Accrued revenues: Revenues earned in a period that are both unrecorded and not yet received in cash.

Steps to Prepare Financial Statements Prepare income statement using revenue and expense accounts from trial balance

Prepare balance sheet using asset and liability accounts along with common stock from trial balance; pull updated retained earnings from step 2

Prepare statement of retained earnings using retained earnings and dividends from trial balance; pull net income from step 1

Prepare statement of cash flows from changes in cash flows for the period (illustrated later in the book)

Step 1

Step 2

Step 4

Step 3

$47,610

Acct. No. Account Title Debit

8,670 2,300

1,800 $ 4,275

26,000

300 1,610

100 1,000 1,050

305

101 Cash ........................................................... Accounts receivable .............................. Supplies .................................................... Prepaid insurance .................................. Equipment ................................................ Accumulated depreciation—Equip..... Accounts payable .................................. Salaries payable .....................................

106 126 128 167 168 201 209

Unearned consulting revenue ............236

Consulting revenue ...............................403 Rental revenue ........................................ 406 Depreciation expense—Equip. ..........612 Salaries expense ....................................622 Insurance expense ................................637 Rent expense ..........................................640 Supplies expense ...................................652 Utilities expense ..................................... Totals .........................................................

690

$ 300

210 6,200

2,750 30,000

300 7,850

$47,610

Credit

Common stock ....................................... Retained earnings .................................

307 318 0

200Dividends .................................................319

Step 2 Prepare statement of retained earnings

$ 3,785

200 $3,585

3,785 Less: Dividends .................................. . . . Retained earnings, December 31 ......

Retained earnings, December 1......... Plus: Net income ................................... . . .

0

Assets

Liabilities

Equity

Cash ..................................................... $ 4,275

Accounts payable .............................

Accounts receivable ........................ 1,800

Unearned consulting revenue ....... 2,750 Salaries payable ................................ 210

8,670 2,300

Supplies .............................................. Prepaid insurance .............................

Total assets ........................................ $ 42,745 25,700

$26,000

6,200

Equipment .......................................... 300Less accumulated depreciation......

Common stock ................................... Retained earnings .............................. Total equity ......................................... Total liabilities and equity ...............

3,585

Total liabilities ....................................... 9,160

$

30,000

$ 42,745 33,585

Step 1 Prepare income statement

Revenues Consulting revenue ............................. $7,850

300 Rental revenue ..................................... Total revenues ......................................

Depreciation expense—Equip.......... Expenses

Salaries expense................................... Insurance expense............................... Rent expense......................................... Supplies expense ................................. Utilities expense.................................... Total expenses........................................ Net income................................................

300 1,610 100

305

1,000 1,050

4,365 $3,785

$8,150

Step 3 Prepare balance sheet

FASTFORWARD Balance Sheet

December 31, 2019

FASTFORWARD Statement of Retained Earnings

For Month Ended December 31, 2019

FASTFORWARD Income Statement

For Month Ended December 31, 2019

FASTFORWARD Adjusted Trial Balance

December 31, 2019

Preparing financial statements from adjusted trial balance:

CLOSING PROCESS Closing process: Occurs at period-end after financial statements have been prepared. Resets revenue, expense, and dividends balances to zero. Temporary accounts: Closed at period-end. They consist of revenue, expense, dividends, and Income Summary. Permanent accounts: Not closed at period-end. They consist of asset, liability, common stock, and retained earnings (all balance sheet accounts). Income Summary: A temporary account only used for the closing process that has a credit for total revenues and a debit for total expenses.

Balance 8,150 8,150

Expense Accounts

4,365 8,150

Balance 3,785

Income Summary

3,785

Balance 30,000

200 3,785

Balance 33,585

Retained Earnings 3 Close income statement credit balances1

Close income statement debit balances2

Close Income Summary account3

Close dividends account4

Four-Step Closing Process

Balance 200 200

Dividends

4

Balance 4,365 4,365 2

Revenue Accounts

1

Closing Process Journal Entries by Step

1

2

Consulting Revenue . . . . . . . . . . . . . 7,850 Rental Revenue . . . . . . . . . . . . . . . . . 300 Income Summary . . . . . . . . . . . . . 8,150

Income Summary . . . . . . . . . . . . . . . . 4,365 Depreciation Expense—Equip . . . . 300 Salaries Expense . . . . . . . . . . . . . . 1,610 Insurance Expense . . . . . . . . . . . . 100 Rent Expense . . . . . . . . . . . . . . . . 1,000 Supplies Expense . . . . . . . . . . . . . 1,050 Utilities Expense . . . . . . . . . . . . . . 305

Income Summary . . . . . . . 3,785 Retained Earnings . . . . 3,785

Retained Earnings . . . . . . 200 Dividends . . . . . . . . . . . 200

3

4

Post-closing trial balance: A list of permanent accounts (assets, liabili- ties, equity) and their balances after all closing entries.

118 Chapter 3 Adjusting Accounts for Financial Statements

CLASSIFIED BALANCE SHEET Classified balance sheet: Organizes assets and liabilities into meaningful subgroups. Current vs. long-term classification: Current items are to be col- lected or owed within one year. Long-term items are expected after one year. Current assets: Assets to be sold, collected, or used within one year. Examples are cash, short-term investments, accounts receivable, short- term notes receivable, merchandise, inventory, and prepaid expenses. Long-term investments: Assets to be held for more than one year. Examples are notes receivable, long-term investments in stock and bonds, and land held for future expansion. Plant assets: Tangible assets used to produce or sell products and services. Examples are equipment, machinery, buildings, and land used in operations. Intangible assets: Long-term assets that lack physical form. Examples are patents, trademarks, copyrights, franchises, and goodwill.

Current liabilities: Liabilities to be paid or settled within one year. Examples are accounts payable, wages payable, taxes payable, interest pay- able, unearned revenues, and current portions of notes or long-term debt. Long-term liabilities: Liabilities not due within one year. Examples are notes payable, mortgages payable, bonds payable, and lease obligations. Equity: The owner’s claim on assets. For a corporation, this is common stock and retained earnings.

Common Layout of Classified Balance Sheet

Assets Liabilities and Equity

Current assets Current liabilities Noncurrent assets Noncurrent liabilities Long-term investments Plant assets Equity Intangible assets

Accounting cycle (104) Accounting period (85) Accrual basis accounting (86) Accrued expenses (93) Accrued revenues (95) Accumulated depreciation (90) Adjusted trial balance (98) Adjusting entry (87) Annual financial statements (85) Book value (90) Cash basis accounting (86) Classified balance sheet (105) Closing entries (101) Closing process (100) Contra account (90)

Current assets (106) Current liabilities (107) Current ratio (108) Depreciation (89) Expense recognition (or matching)

principle (87) Fiscal year (85) Income Summary (101) Intangible assets (107) Interim financial statements (85) Long-term investments (106) Long-term liabilities (107) Natural business year (86) Operating cycle (106) Permanent accounts (101)

Plant assets (89) Post-closing trial balance (104) Prepaid expenses (87) Pro forma financial statements (115) Profit margin (108) Revenue recognition principle (87) Reversing entries (115) Straight-line depreciation (89) Temporary accounts (101) Time period assumption (85) Unadjusted trial balance (98) Unclassified balance sheet (105) Unearned revenue (91) Work sheet (113)

Key Terms

Multiple Choice Quiz

1. A company forgot to record accrued and unpaid employee wages of $350,000 at period-end. This oversight would a. Understate net income by $350,000. b. Overstate net income by $350,000. c. Have no effect on net income. d. Overstate assets by $350,000. e. Understate assets by $350,000.

2. Prior to recording adjusting entries, the Supplies account has a $450 debit balance. A physical count of supplies shows $125 of unused supplies still available. The required adjusting entry is

a. Debit Supplies $125; credit Supplies Expense $125. b. Debit Supplies $325; credit Supplies Expense $325. c. Debit Supplies Expense $325; credit Supplies $325. d. Debit Supplies Expense $325; credit Supplies $125. e. Debit Supplies Expense $125; credit Supplies $125.

3. On May 1 of the current year, a two-year insurance policy was purchased for $24,000 with coverage to begin immedi- ately. What is the amount of insurance expense that appears on the company’s income statement for the current year ended December 31? a. $4,000 c. $12,000 e. $24,000 b. $8,000 d. $20,000

Chapter 3 Adjusting Accounts for Financial Statements 119

A(B,C) Superscript letter A, B, or C denotes assignments based on Appendix 3A, 3B, or 3C.

Icon denotes assignments that involve decision making.

1. What is the difference between the cash basis and the accrual basis of accounting?

2. Why is the accrual basis of accounting generally preferred over the cash basis?

3. What type of business is most likely to select a fiscal year that corresponds to its natural business year instead of the calendar year?

4. What is a prepaid expense and where is it reported in the financial statements?

5. What contra account is used when recording and re- porting the effects of depreciation? Why is it used?

6. What is an accrued revenue? Give an example. 7. What are the steps in recording closing entries? 8. What is the purpose of the Income Summary account? 9. Explain whether an error has occurred if a post-closing

trial balance includes a Depreciation Expense account. 10. What is a company’s operating cycle? 11. What classes of assets and liabilities are shown on a typical

classified balance sheet? 12. How is unearned revenue classified on the balance sheet?

13.A If a company initially records prepaid expenses with debits to expense accounts, what type of account is debited in the adjusting entries for those prepaid expenses?

14.C If a company recorded accrued salaries expense of $500 at the end of its fiscal year, what reversing entry could be made? When would it be made?

15. Refer to Apple’s most recent balance sheet in Appendix A. What five main noncurrent as- set categories are used on its classified balance sheet?

16. Refer to Google’s most recent balance sheet in Appendix A. Identify the six accounts listed as current liabilities.

17. Review Google’s balance sheet in Appendix A. Identify the amount for prop- erty and equipment. What adjusting entry is necessary (no numbers required) for this account when preparing finan- cial statements?

18. Refer to Samsung’s financial state- ments in Appendix A. What journal entry was likely recorded as of December 31, 2017, to close its Income Summary account?

Discussion Questions

APPLE

Samsung

GOOGLE

GOOGLE

4. On November 1, Stockton Co. receives $3,600 cash from Hans Co. for consulting services to be provided evenly over the period November 1 to April 30—at which time Stockton credits $3,600 to Unearned Consulting Fees. The adjusting entry on December 31 (Stockton’s year-end) would include a a. Debit to Unearned Consulting Fees for $1,200. b. Debit to Unearned Consulting Fees for $2,400. c. Credit to Consulting Fees Earned for $2,400. d. Debit to Consulting Fees Earned for $1,200. e. Credit to Cash for $3,600.

5. The following information is available for a company before closing the accounts. After all of the closing entries are made, what will be the balance in the Retained Earnings account?

a. $360,000 d. $150,000 b. $250,000 e. $60,000 c. $160,000

Total revenues . . $300,000

Total expenses . . 195,000

Retained earnings . . . $100,000

Dividends . . . . . . . . . . 45,000

ANSWERS TO MULTIPLE CHOICE QUIZ

1. b; the forgotten adjusting entry is: dr. Wages Expense, cr. Wages Payable.

2. c; Supplies used = $450 − $125 = $325 3. b; Insurance expense = $24,000 × (8∕24) = $8,000; adjusting entry

is: dr. Insurance Expense for $8,000, cr. Prepaid Insurance for $8,000.

4. a; Consulting fees earned = $3,600 × (2∕6) = $1,200; adjusting entry is: dr. Unearned Consulting Fees for $1,200, cr. Consulting Fees Earned for $1,200.

5. c; $100,000 + $300,000 − $195,000 − $45,000

QUICK STUDY

QS 3-1 Periodic reporting

C1

Choose from the following list of terms and phrases to best complete the statements below. a. Fiscal year c. Accrual basis accounting e. Cash basis accounting b. Timeliness d. Annual financial statements f. Time period assumption 1. presumes that an organization’s activities can be divided into specific time periods. 2. Financial reports covering a one-year period are known as . 3. A(n) consists of any 12 consecutive months. 4. records revenues when services are provided and records expenses when incurred. 5. The value of information is often linked to its .

120 Chapter 3 Adjusting Accounts for Financial Statements

QS 3-2 Computing accrual and cash income

C1

In its first year of operations, Roma Company reports the following.

∙ Earned revenues of $45,000 ($37,000 cash received from customers). ∙ Incurred expenses of $25,500 ($20,250 cash paid toward them). ∙ Prepaid $6,750 cash for costs that will not be expensed until next year.

Compute Roma’s first-year net income under the cash basis and the accrual basis of accounting.

QS 3-3 Identifying accounting adjustments

P1 P2 P3 P4

Classify the following adjusting entries as involving prepaid expenses (PE), unearned revenues (UR), accrued expenses (AE), or accrued revenues (AR).

a. To record revenue earned that was previously received as cash in advance. b. To record wages expense incurred but not yet paid (nor recorded). c. To record revenue earned but not yet billed (nor recorded). d. To record expiration of prepaid insurance. e. To record annual depreciation expense.

QS 3-4 Concepts of adjusting entries

P1 P2 P3 P4

During the year, a company recorded prepayments of expenses in asset accounts and cash receipts of unearned revenues in liability accounts. At the end of its annual accounting period, the company must make three adjusting entries. (1) Accrue salaries expense. . . . . . . . . . . . . . . . . . . . . . . . . . . . . . . . . . . . . . . . . . . . . . . . Dr. ___ Cr. ___ (2) Adjust the Unearned Services Revenue account to recognize earned revenue. . . . . . . Dr. ___ Cr. ___ (3) Record services revenue earned for which cash will be received the following period.. . Dr. ___ Cr. ___

For each of the adjusting entries (1), (2), and (3), indicate the account to be debited and the account to be credited—from a through i below. a. Prepaid Insurance d. Unearned Services Revenue g. Accounts Receivable b. Cash e. Salaries Expense h. Accounts Payable c. Salaries Payable f. Services Revenue i. Depreciation Expense

QS 3-5 Prepaid (deferred) expenses adjustments

P1

For each separate case below, follow the three-step process for adjusting the prepaid asset account at December 31. Step 1: Determine what the current account balance equals. Step 2: Determine what the current account balance should equal. Step 3: Record the December 31 adjusting entry to get from step 1 to step 2. Assume no other adjusting entries are made during the year. a. Prepaid Insurance. The Prepaid Insurance account has a $4,700 debit balance to start the year. A

review of insurance policies shows that $900 of unexpired insurance remains at year-end. b. Prepaid Insurance. The Prepaid Insurance account has a $5,890 debit balance at the start of the year.

A review of insurance policies shows $1,040 of insurance has expired by year-end. c. Prepaid Rent. On September 1 of the current year, the company prepaid $24,000 for two years of rent

for facilities being occupied that day. The company debited Prepaid Rent and credited Cash for $24,000.

QS 3-6 Prepaid (deferred) expenses adjustments

P1

For each separate case below, follow the three-step process for adjusting the Supplies asset account at December 31. Step 1: Determine what the current account balance equals. Step 2: Determine what the current account balance should equal. Step 3: Record the December 31 adjusting entry to get from step 1 to step 2. Assume no other adjusting entries are made during the year. a. Supplies. The Supplies account has a $300 debit balance to start the year. No supplies were purchased

during the current year. A December 31 physical count shows $110 of supplies remaining. b. Supplies. The Supplies account has an $800 debit balance to start the year. Supplies of $2,100 were

purchased during the current year and debited to the Supplies account. A December 31 physical count shows $650 of supplies remaining.

c. Supplies. The Supplies account has a $4,000 debit balance to start the year. During the current year, supplies of $9,400 were purchased and debited to the Supplies account. The inventory of supplies available at December 31 totaled $2,660.

QS 3-7 Adjusting prepaid (deferred) expenses

P1

For each separate case, record the necessary adjusting entry. a. On July 1, Lopez Company paid $1,200 for six months of insurance coverage. No adjustments have

been made to the Prepaid Insurance account, and it is now December 31. Prepare the year-end adjust- ing entry to reflect expiration of the insurance as of December 31.

Chapter 3 Adjusting Accounts for Financial Statements 121

QS 3-8 Accumulated depreciation adjustments

P1

For each separate case below, follow the three-step process for adjusting the Accumulated Depreciation account at December 31. Step 1: Determine what the current account balance equals. Step 2: Determine what the current account balance should equal. Step 3: Record the December 31 adjusting entry to get from step 1 to step 2. Assume no other adjusting entries are made during the year. a. Accumulated Depreciation. The Krug Company’s Accumulated Depreciation account has a $13,500

balance to start the year. A review of depreciation schedules reveals that $14,600 of depreciation expense must be recorded for the year.

b. Accumulated Depreciation. The company has only one fixed asset (truck) that it purchased at the start of this year. That asset had cost $44,000, had an estimated life of five years, and is expected to have zero value at the end of the five years.

c. Accumulated Depreciation. The company has only one fixed asset (equipment) that it purchased at the start of this year. That asset had cost $32,000, had an estimated life of seven years, and is expected to be valued at $4,000 at the end of the seven years.

QS 3-9 Adjusting for depreciation

P1

For each separate case, record an adjusting entry (if necessary). a. Barga Company purchases $20,000 of equipment on January 1. The equipment is expected to last five

years and be worth $2,000 at the end of that time. Prepare the entry to record one year’s depreciation expense of $3,600 for the equipment as of December 31.

b. Welch Company purchases $10,000 of land on January 1. The land is expected to last forever. What depreciation adjustment, if any, should be made with respect to the Land account as of December 31?

QS 3-10 Unearned (deferred) revenues adjustments

P2

For each separate case below, follow the three-step process for adjusting the unearned revenue liability account at December 31. Step 1: Determine what the current account balance equals. Step 2: Determine what the current account balance should equal. Step 3: Record the December 31 adjusting entry to get from step 1 to step 2. Assume no other adjusting entries are made during the year. a. Unearned Rent Revenue. The Krug Company collected $6,000 rent in advance on November 1, deb-

iting Cash and crediting Unearned Rent Revenue. The tenant was paying 12 months’ rent in advance and occupancy began November 1.

b. Unearned Services Revenue. The company charges $75 per insect treatment. A customer paid $300 on October 1 in advance for four treatments, which was recorded with a debit to Cash and a credit to Unearned Services Revenue. At year-end, the company has applied three treatments for the customer.

c. Unearned Rent Revenue. On September 1, a client paid the company $24,000 cash for six months of rent in advance (the client leased a building and took occupancy immediately). The company recorded the cash as Unearned Rent Revenue.

QS 3-11 Adjusting for unearned (deferred) revenues

P2

For each separate case, record the necessary adjusting entry. a. Tao Co. receives $10,000 cash in advance for four months of evenly planned legal services beginning

on October 1. Tao records it by debiting Cash and crediting Unearned Revenue both for $10,000. It is now December 31, and Tao has provided legal services as planned. What adjusting entry should Tao make to account for the work performed from October 1 through December 31?

b. Caden started a new publication called Contest News. Its subscribers pay $24 to receive 12 monthly issues. With every new subscriber, Caden debits Cash and credits Unearned Subscription Revenue for the amounts received. The company has 100 new subscribers as of July 1. It sends Contest News to each of these subscribers every month from July through December. Assuming no changes in sub- scribers, prepare the year-end journal entry that Caden must make as of December 31 to adjust the Subscription Revenue account and the Unearned Subscription Revenue account.

b. Zim Company has a Supplies account balance of $5,000 at the beginning of the year. During the year, it purchases $2,000 of supplies. As of December 31, a physical count of supplies shows $800 of supplies available. Prepare the adjusting journal entry to correctly report the balance of the Supplies account and the Supplies Expense account as of December 31.

For each separate case below, follow the three-step process for adjusting the accrued expense account at December 31. Step 1: Determine what the current account balance equals. Step 2: Determine what the current account balance should equal. Step 3: Record the December 31 adjusting entry to get from step 1 to step 2. Assume no other adjusting entries are made during the year. a. Salaries Payable. At year-end, salaries expense of $15,500 has been incurred by the company but is

not yet paid to employees.

QS 3-12 Accrued expenses adjustments

P3

[continued on next page]

122 Chapter 3 Adjusting Accounts for Financial Statements

Molly Mocha employs one college student every summer in her coffee shop. The student works the five weekdays and is paid on the following Monday. (For example, a student who works Monday through Friday, June 1 through June 5, is paid for that work on Monday, June 8.) The coffee shop adjusts its books monthly, if needed, to show salaries earned but unpaid at month-end. The student works the last week of July, which is Monday, July 28, through Friday, August 1. If the student earns $100 per day, what adjust- ing entry must the coffee shop make on July 31 to correctly record accrued salaries expense for July?

QS 3-13 Accruing salaries

P3

For each separate case below, follow the three-step process for adjusting the accrued revenue account at December 31. Step 1: Determine what the current account balance equals. Step 2: Determine what the current account balance should equal. Step 3: Record the December 31 adjusting entry to get from step 1 to step 2. Assume no other adjusting entries are made during the year. a. Accounts Receivable. At year-end, the L. Cole Company has completed services of $19,000 for a

client, but the client has not yet been billed for those services. b. Interest Receivable. At year-end, the company has earned, but not yet recorded, $390 of interest

earned from its investments in government bonds. c. Accounts Receivable. A painting company bills customers when jobs are complete. The work for one

job is now complete. The customer has not yet been billed for the $1,300 of work.

QS 3-14 Accrued revenues adjustments

P4

Adjusting entries affect at least one balance sheet account and at least one income statement account. For the entries below, identify the account to be debited and the account to be credited from the following ac- counts: Cash; Accounts Receivable; Prepaid Insurance; Equipment; Accumulated Depreciation; Wages Payable; Unearned Revenue; Revenue; Wages Expense; Insurance Expense; and Depreciation Expense. Indicate which of the accounts is the income statement account and which is the balance sheet account. a. Entry to record revenue earned that was previously received as cash in advance. b. Entry to record wage expenses incurred but not yet paid (nor recorded). c. Entry to record revenue earned but not yet billed (nor recorded). d. Entry to record expiration of prepaid insurance. e. Entry to record annual depreciation expense.

QS 3-15 Recording and analyzing adjusting entries

P1 P2 P3 P4

In making adjusting entries at the end of its accounting period, Chao Consulting mistakenly forgot to record: 1. $3,200 of insurance coverage that had expired (this $3,200 cost had been initially debited to the

Prepaid Insurance account). 2. $2,000 of accrued salaries expense. As a result of these two oversights, the financial statements for the reporting period will [choose one]: a. Understate assets by $3,200. c. Understate net income by $2,000. b. Understate expenses by $5,200. d. Overstate liabilities by $2,000.

QS 3-16 Determining effects of adjusting entries

P1 P3

Following are unadjusted balances along with year-end adjustments for Quinlan Company. Complete the adjusted trial balance by entering the adjusted balance for each of the following accounts.

QS 3-17 Preparing an adjusted trial balance

P5 $8,000

2,000 4,500

5,500 0

3,000 6,000

11,000

101 106 126 209 307 318 403 622 652

Cash Accounts receivable Supplies Salaries payable Common stock Retained earnings Consulting revenue Salaries expense Supplies expense

Account Title Dr.

$4,000

400 2,500

Dr.

$2,500

400

4,000

Cr.Cr.No. Unadjusted Trial Balance

Dr. Cr. Adjusted Trial BalanceAdjustments

$ 0

The ledger of Mai Company includes the following accounts with normal balances as of December 31: Common Stock $9,000; Dividends $800; Services Revenue $13,000; Wages Expense $8,400; and Rent Expense $1,600. Prepare its December 31 closing entries.

QS 3-18 Preparing closing entries from the ledger P7

b. Interest Payable. At its December 31 year-end, the company owes $250 of interest on a line-of-credit loan. That interest will not be paid until sometime in January of the next year.

c. Interest Payable. At its December 31 year-end, the company holds a mortgage payable that has in- curred $875 in annual interest that is neither recorded nor paid. The company intends to pay the inter- est on January 7 of the next year.

Chapter 3 Adjusting Accounts for Financial Statements 123

Identify which of the following accounts would be included in a post-closing trial balance. a. Accounts Receivable c. Goodwill e. Income Tax Expense b. Salaries Expense d. Land f. Salaries Payable

QS 3-19 Identifying post-closing accounts P8

The following are common categories on a classified balance sheet. A. Current assets C. Plant assets E. Current liabilities B. Long-term investments D. Intangible assets F. Long-term liabilities

For each of the following items, select the letter that identifies the balance sheet category where the item typically would best appear.

1. Land held for future expansion 5. Accounts payable 2. Notes payable (due in five years) 6. Store equipment 3. Accounts receivable 7. Wages payable 4. Trademarks 8. Cash

QS 3-21 Classifying balance sheet items

C3

List the following steps of the accounting cycle in their proper order. a. Posting the journal entries. b. Journalizing and posting adjusting entries. c. Preparing the adjusted trial balance. d. Journalizing and posting closing entries. e. Analyzing transactions and events.

QS 3-20 Identifying the accounting cycle

C2

f. Preparing the financial statements. g. Preparing the unadjusted trial balance. h. Journalizing transactions and events. i. Preparing the post-closing trial balance.

Use the following adjusted trial balance of Sierra Company to prepare its (1) income statement and (2) statement of retained earnings for the year ended December 31. The Retained Earnings account bal- ance was $5,500 on December 31 of the prior year.

QS 3-22 Preparing financial statements

P6 Cash Prepaid insurance Notes receivable (due in 5 years) Buildings Accumulated depreciation—Buildings Accounts payable Notes payable (due in 3 years) Common stock Retained earnings Dividends Consulting revenue Wages expense Depreciation expense—Buildings Insurance expense

Adjusted Trial Balance Debit Credit

$12,000 2,500 3,000 5,000 5,500

9,500

$37,500

$ 5,000 500

4,000 20,000

3,500 2,000 1,500

$37,500

1,000

Totals

Damita Company reported net income of $48,025 and net sales of $425,000 for the current year. Calculate the company’s profit margin and interpret the result. Assume that its competitors earn an average profit margin of 15%.

QS 3-24 Analyzing profit margin

A1

Use the information in the adjusted trial balance reported in QS 3-22 to prepare Sierra Company’s classi- fied balance sheet as of December 31.

QS 3-23 Preparing a classified balance sheet C3

Compute Chavez Company’s current ratio using the following information. QS 3-25 Identifying current accounts and computing the current ratio

A2

Accounts receivable . . . . . . . . . . . . $18,000 Long-term notes payable . . . . . . . . . . . . . . . $21,000

Accounts payable . . . . . . . . . . . . . . 11,000 Office supplies . . . . . . . . . . . . . . . . . . . . . . . 2,800

Buildings . . . . . . . . . . . . . . . . . . . . . 45,000 Prepaid insurance . . . . . . . . . . . . . . . . . . . . . 3,560

Cash . . . . . . . . . . . . . . . . . . . . . . . . . 7,000 Unearned services revenue . . . . . . . . . . . . . 3,000

124 Chapter 3 Adjusting Accounts for Financial Statements

QS 3-26A Preparing adjusting entries

P9

Garcia Company had the following selected transactions during the year. (A partial chart of accounts fol- lows: Cash; Accounts Receivable; Prepaid Insurance; Wages Payable; Unearned Revenue; Revenue; Wages Expense; Insurance Expense; Depreciation Expense.)

Jan. 1 The company paid $6,000 cash for 12 months of insurance coverage beginning immediately. Aug. 1 The company received $2,400 cash in advance for 6 months of contracted services beginning on

August 1 and ending on January 31. Dec. 31 The company prepared any necessary year-end adjusting entries related to insurance coverage

and services performed.

a. Record journal entries for these transactions assuming Garcia follows the usual practice of recording a prepayment of an expense in an asset account and recording a prepayment of revenue received in a liability account.

b. Record journal entries for these transactions assuming Garcia follows the alternative practice of re- cording a prepayment of an expense in an expense account and recording a prepayment of revenue received in a revenue account.

QS 3-28C Reversing entries

P11

On December 31, Yates Co. prepared an adjusting entry for $12,000 of earned but unrecorded consulting revenue. On January 16, Yates received $26,700 cash as payment in full for consulting work it provided that began on December 18 and ended on January 16. The company uses reversing entries. a. Prepare the December 31 adjusting entry. c. Prepare the January 16 cash receipt entry. b. Prepare the January 1 reversing entry.

The Adjusted Trial Balance columns of a 10-column work sheet for Planta Company follow. Complete the work sheet by extending the account balances into the appropriate financial statement columns and by entering the amount of net income for the reporting period.

QS 3-27B Extending accounts in a work sheet P10

$ 7,000 27,200 42,000

32,000

15,400

6,500 38,000 13,000 8,700

$189,800

$ 17,500

15,000 4,200 3,600

20,000 45,500

84,000

$189,800

101 106 153 154 183 201 209 233 307 318 319 401 611 622 640 677

Cash Accounts receivable Trucks Accumulated depreciation—Trucks Land Accounts payable Salaries payable Unearned fees Common stock Retained earnings Dividends Plumbing fees earned Depreciation expense—Trucks Salaries expense Rent expense Miscellaneous expenses Totals Net income Totals

Account Title

Unadjusted Trial Balance

Adjusted Trial Balance

Dr. Adjustments

Cr. Dr. Cr.

Income Statement

Dr. Cr. Balance Sheet

Dr. Cr.Dr. Cr.No.

Check Net income, $17,800

EXERCISES

Exercise 3-1 Preparing adjusting entries

P1 P2 P3

Prepare adjusting journal entries for the year ended (date of) December 31 for each of these separate situ- ations. Entries can draw from the following partial chart of accounts: Cash; Accounts Receivable; Supplies; Prepaid Insurance; Prepaid Rent; Equipment; Accumulated Depreciation—Equipment; Wages Payable; Unearned Revenue; Revenue; Wages Expense; Supplies Expense; Insurance Expense; Rent Expense; and Depreciation Expense—Equipment. a. Depreciation on the company’s equipment for the year is computed to be $18,000. b. The Prepaid Insurance account had a $6,000 debit balance at December 31 before adjusting for the

costs of any expired coverage. An analysis of the company’s insurance policies showed that $1,100 of unexpired insurance coverage remains.

c. The Supplies account had a $700 debit balance at the beginning of the year; and $3,480 of supplies were purchased during the year. The December 31 physical count showed $300 of supplies available.

Check (c) Dr. Supplies Expense, $3,880

Chapter 3 Adjusting Accounts for Financial Statements 125

Exercise 3-2 Adjusting and paying accrued wages

P3

Pablo Management has five employees, each of whom earns $250 per day. They are paid on Fridays for work completed Monday through Friday of the same week. Near year-end, the five employees worked Monday, December 31, and Wednesday through Friday, January 2, 3, and 4. New Year’s Day (January 1) was an unpaid holiday. a. Prepare the year-end adjusting entry for wages expense. b. Prepare the journal entry to record payment of the employees’ wages on Friday, January 4.

Exercise 3-3 Adjusting and paying accrued expenses

P3

The following three separate situations require adjusting journal entries to prepare financial statements as of April 30. For each situation, present both: ∙ The April 30 adjusting entry. ∙ The subsequent entry during May to record payment of the accrued expenses. Entries can draw from the following partial chart of accounts: Cash; Accounts Receivable; Salaries Payable; Interest Payable; Legal Services Payable; Unearned Revenue; Revenue; Salaries Expense; Interest Expense; Legal Services Expense; and Depreciation Expense. a. On April 1, the company hired an attorney for a flat monthly fee of $3,500. Payment for April legal

services was made by the company on May 12. b. As of April 30, $3,000 of interest expense has accrued on a note payable. The full interest payment of

$9,000 on the note is due on May 20. c. Total weekly salaries expense for all employees is $10,000. This amount is paid at the end of the day

on Friday of each five-day workweek. April 30 falls on a Tuesday, which means that the employees had worked two days since the last payday. The next payday is May 3.

Check (b) May 20, Dr. Interest Expense, $6,000

Exercise 3-4 Preparing adjusting entries

P1 P3 P4

For each of the following separate cases, prepare adjusting entries required of financial statements for the year ended (date of) December 31. Entries can draw from the following partial chart of accounts: Cash; Interest Receivable; Supplies; Prepaid Insurance; Equipment; Accumulated Depreciation—Equipment; Wages Payable; Interest Payable; Unearned Revenue; Interest Revenue; Wages Expense; Supplies Expense; Insurance Expense; Interest Expense; and Depreciation Expense—Equipment. a. Wages of $8,000 are earned by workers but not paid as of December 31. b. Depreciation on the company’s equipment for the year is $18,000. c. The Supplies account had a $240 debit balance at the beginning of the year. During the year, $5,200 of

supplies are purchased. A physical count of supplies at December 31 shows $440 of supplies available. d. The Prepaid Insurance account had a $4,000 balance at the beginning of the year. An analysis of insur-

ance policies shows that $1,200 of unexpired insurance benefits remain at December 31. e. The company has earned (but not recorded) $1,050 of interest revenue for the year ended December

31. The interest payment will be received 10 days after the year-end on January 10. f. The company has a bank loan and has incurred (but not recorded) interest expense of $2,500 for the

year ended December 31. The company will pay the interest five days after the year-end on January 5.

Check (d) Dr. Insurance Expense, $2,800

(e) Cr. Interest Revenue, $1,050

Exercise 3-5 Preparing adjusting entries—accrued revenues and expenses

P3 P4

Prepare year-end adjusting journal entries for M&R Company as of December 31 for each of the following separate cases. Entries can draw from the following partial chart of accounts: Cash; Accounts Receivable; Interest Receivable; Equipment; Wages Payable; Salary Payable; Interest Payable; Lawn Services Payable; Unearned Revenue; Revenue; Interest Revenue; Wages Expense; Salary Expense; Supplies Expense; Lawn Services Expense; and Interest Expense. a. M&R Company provided $2,000 in services to customers in December, which are not yet recorded.

Those customers are expected to pay the company in January following the company’s year-end. b. Wage expenses of $1,000 have been incurred but are not paid as of December 31. c. M&R Company has a $5,000 bank loan and has incurred (but not recorded) 8% interest expense of

$400 for the year ended December 31. The company will pay the $400 interest in cash on January 2 following the company’s year-end.

d. M&R Company hired a firm that provided lawn services during December for $500. M&R will pay for December lawn services on January 15 following the company’s year-end.

e. M&R Company has earned $200 in interest revenue from investments for the year ended December 31. The interest revenue will be received on January 15 following the company’s year-end.

f. Salary expenses of $900 have been earned by supervisors but not paid as of December 31.

d. Two-thirds of the work related to $15,000 of cash received in advance was performed this period. e. The Prepaid Rent account had a $6,800 debit balance at December 31 before adjusting for the costs of

expired prepaid rent. An analysis of the rental agreement showed that $5,800 of prepaid rent had expired. f. Wage expenses of $3,200 have been incurred but are not paid as of December 31.

(e) Dr. Rent Expense, $5,800

126 Chapter 3 Adjusting Accounts for Financial Statements

For each of the following separate cases, prepare the required December 31 year-end adjusting entries. Entries can draw from this partial chart of accounts: Interest Receivable; Prepaid Insurance; Accumulated Depreciation—Equipment; Wages Payable; Unearned Revenue; Consulting Revenue; Interest Revenue; Wages Expense; Insurance Expense; Interest Expense; and Depreciation Expense—Equipment. a. Depreciation on the company’s wind turbine equipment for the year is $5,000. b. The Prepaid Insurance account for the solar panels had a $2,000 debit balance at December 31 before

adjusting for the costs of any expired coverage. Analysis of prepaid insurance shows that $600 of un- expired insurance coverage remains at year-end.

c. The company received $3,000 cash in advance for sustainability consulting work. As of December 31, one-third of the sustainability consulting work had been performed.

d. As of December 31, $1,200 in wages expense for the organic produce workers has been incurred but not yet paid.

e. As of December 31, the company has earned, but not yet recorded, $400 of interest revenue from in- vestments in socially responsible bonds. The interest revenue is expected to be received on January 12.

Exercise 3-6 Preparing adjusting entries

P1 P2 P3 P4

Income Statements For Year Ended December 31

Unadjusted Adjustments Adjusted

Revenues

Fees earned . . . . . . . . . . . . . . . . . . . . . . . . . . . . . . . . . . . . . $18,000 a . $25,000

Commissions earned . . . . . . . . . . . . . . . . . . . . . . . . . . . . . 36,500 36,500

Total revenues . . . . . . . . . . . . . . . . . . . . . . . . . . . . . . . . . . . 54,500 61,500

Expenses

Depreciation expense—Computers . . . . . . . . . . . . . . . . . . 0 b . 1,600

Depreciation expense—Office furniture . . . . . . . . . . . . . . . 0 c . 1,850

Salaries expense . . . . . . . . . . . . . . . . . . . . . . . . . . . . . . . . . 13,500 d . 15,750

Insurance expense . . . . . . . . . . . . . . . . . . . . . . . . . . . . . . . 0 e . 1,400

Rent expense . . . . . . . . . . . . . . . . . . . . . . . . . . . . . . . . . . . . 3,800 3,800

Office supplies expense . . . . . . . . . . . . . . . . . . . . . . . . . . . 0 f . 580

Advertising expense . . . . . . . . . . . . . . . . . . . . . . . . . . . . . . 2,500 2,500

Utilities expense . . . . . . . . . . . . . . . . . . . . . . . . . . . . . . . . . 1,245 g . 1,335

Total expenses . . . . . . . . . . . . . . . . . . . . . . . . . . . . . . . . . . . 21,045 28,815

Net income . . . . . . . . . . . . . . . . . . . . . . . . . . . . . . . . . . . . . . . . $33,455 $32,685

Following are two income statements for Alexis Co. for the year ended December 31. The left number col- umn is prepared before adjusting entries are recorded, and the right column is prepared after adjusting en- tries. Analyze the statements and prepare the seven adjusting entries a through g that likely were recorded. Hint: The entry for a refers to fees that have been earned but not yet billed. None of the entries involve cash.

Exercise 3-7 Analyzing and preparing adjusting entries

P5

Following are the accounts and balances (in random order) from the adjusted trial balance of Stark Company. Prepare the (1) income statement and (2) statement of retained earnings for the year ended December 31 and (3) balance sheet at December 31. The Retained Earnings account balance was $14,800 on December 31 of the prior year.

Exercise 3-8 Preparing financial statements from a trial balance

P6 Notes payable . . . . . . . . . . . . . . . . . . . . . . . . . $11,000 Accumulated depreciation—Buildings . . . . . . . . . . . . $15,000 Prepaid insurance . . . . . . . . . . . . . . . . . . . . . 2,500 Accounts receivable . . . . . . . . . . . . . . . . . . . . . . . . . . 4,000

Interest expense . . . . . . . . . . . . . . . . . . . . . . 500 Utilities expense . . . . . . . . . . . . . . . . . . . . . . . . . . . . . 1,300

Accounts payable . . . . . . . . . . . . . . . . . . . . . 1,500 Interest payable . . . . . . . . . . . . . . . . . . . . . . . . . . . . . 100

Wages payable . . . . . . . . . . . . . . . . . . . . . . . 400 Unearned revenue . . . . . . . . . . . . . . . . . . . . . . . . . . . 800

Cash . . . . . . . . . . . . . . . . . . . . . . . . . . . . . . . . 10,000 Supplies expense . . . . . . . . . . . . . . . . . . . . . . . . . . . . 200

Wages expense . . . . . . . . . . . . . . . . . . . . . . . 7,500 Buildings . . . . . . . . . . . . . . . . . . . . . . . . . . . . . . . . . . . 40,000

Insurance expense . . . . . . . . . . . . . . . . . . . . 1,800 Dividends . . . . . . . . . . . . . . . . . . . . . . . . . . . . . . . . . . 3,000

Common stock . . . . . . . . . . . . . . . . . . . . . . . . 10,000 Depreciation expense—Buildings . . . . . . . . . . . . . . . 2,000

Retained earnings . . . . . . . . . . . . . . . . . . . . . 14,800 Supplies . . . . . . . . . . . . . . . . . . . . . . . . . . . . . . . . . . . . 800

Services revenue . . . . . . . . . . . . . . . . . . . . . . 20,000

Chapter 3 Adjusting Accounts for Financial Statements 127

Net sales . . . . . . . . . . . . . . . . . . . . . . . . . . . ¥504,459

Cost of sales . . . . . . . . . . . . . . . . . . . . . . . . 283,494

Advertising expense . . . . . . . . . . . . . . . . . . . . . ¥ 46,636

Other expense, net . . . . . . . . . . . . . . . . . . . . . . 157,811

Following are Nintendo’s revenue and expense accounts for a recent March 31 fiscal year-end (yen in millions). Prepare the company’s closing entries for (1) its revenues and (2) its expenses.

Exercise 3-9 Preparing closing entries

P7

No. Account Title Debit Credit

101 Cash . . . . . . . . . . . . . . . . . . . . . . . . . . . . . . . . . . . . . . . . . . $19,000

126 Supplies . . . . . . . . . . . . . . . . . . . . . . . . . . . . . . . . . . . . . . . 13,000

128 Prepaid insurance . . . . . . . . . . . . . . . . . . . . . . . . . . . . . . . 3,000

167 Equipment . . . . . . . . . . . . . . . . . . . . . . . . . . . . . . . . . . . . . 24,000

168 Accumulated depreciation—Equipment . . . . . . . . . . . . . $   7,500

307 Common stock . . . . . . . . . . . . . . . . . . . . . . . . . . . . . . . . . 10,000

318 Retained earnings . . . . . . . . . . . . . . . . . . . . . . . . . . . . . . . 37,600

319 Dividends . . . . . . . . . . . . . . . . . . . . . . . . . . . . . . . . . . . . . . 7,000

404 Services revenue . . . . . . . . . . . . . . . . . . . . . . . . . . . . . . . 44,000

612 Depreciation expense—Equipment . . . . . . . . . . . . . . . . . 3,000

622 Salaries expense . . . . . . . . . . . . . . . . . . . . . . . . . . . . . . . . 22,000

637 Insurance expense . . . . . . . . . . . . . . . . . . . . . . . . . . . . . . 2,500

640 Rent expense . . . . . . . . . . . . . . . . . . . . . . . . . . . . . . . . . . 3,400

652 Supplies expense . . . . . . . . . . . . . . . . . . . . . . . . . . . . . . . 2,200

Totals . . . . . . . . . . . . . . . . . . . . . . . . . . . . . . . . . . . . . . . . . $99,100 $99,100

The following adjusted trial balance contains the accounts and year-end balances of Cruz Company as of December 31. (1) Prepare the December 31 closing entries for Cruz Company. Assume the ac- count number for Income Summary is 901. (2) Prepare the December 31 post-closing trial balance for Cruz Company. Note: The Retained Earnings account balance was $37,600 on December 31 of the prior year.

Exercise 3-10 Preparing closing entries and a post-closing trial balance

P7 P8

Use the following adjusted year-end trial balance at December 31 of Wilson Trucking Company to pre- pare the (1) income statement and (2) statement of retained earnings for the year ended December 31. The Retained Earnings account balance was $155,000 at December 31 of the prior year.

Exercise 3-11 Preparing financial statements P6

Account Title Debit Credit

Cash . . . . . . . . . . . . . . . . . . . . . . . . . . . . . . . . . . . . . . . $ 8,000

Accounts receivable . . . . . . . . . . . . . . . . . . . . . . . . . . 17,500

Office supplies . . . . . . . . . . . . . . . . . . . . . . . . . . . . . . 3,000

Trucks . . . . . . . . . . . . . . . . . . . . . . . . . . . . . . . . . . . . . 172,000

Accumulated depreciation—Trucks . . . . . . . . . . . . . . $ 36,000

Land . . . . . . . . . . . . . . . . . . . . . . . . . . . . . . . . . . . . . . . 85,000

Accounts payable . . . . . . . . . . . . . . . . . . . . . . . . . . . . 12,000

Interest payable . . . . . . . . . . . . . . . . . . . . . . . . . . . . . 4,000

Long-term notes payable . . . . . . . . . . . . . . . . . . . . . . 58,000

Common stock . . . . . . . . . . . . . . . . . . . . . . . . . . . . . . 15,000

Retained earnings . . . . . . . . . . . . . . . . . . . . . . . . . . . . 155,000

Dividends . . . . . . . . . . . . . . . . . . . . . . . . . . . . . . . . . . 20,000

Trucking fees earned . . . . . . . . . . . . . . . . . . . . . . . . . 130,000

Depreciation expense—Trucks . . . . . . . . . . . . . . . . . 23,500

Salaries expense . . . . . . . . . . . . . . . . . . . . . . . . . . . . . 61,000

Office supplies expense . . . . . . . . . . . . . . . . . . . . . . . 8,000

Repairs expense—Trucks . . . . . . . . . . . . . . . . . . . . . . 12,000

Totals . . . . . . . . . . . . . . . . . . . . . . . . . . . . . . . . . . . . . . $410,000 $410,000

128 Chapter 3 Adjusting Accounts for Financial Statements

Use the following information to compute profit margin for each separate company a through e. Which of the five companies is the most profitable according to the profit margin ratio? Interpret the profit margin ratio for company c.

Exercise 3-13 Computing and interpreting profit margin

A1 Net Income Net Sales Net Income Net Sales

a. $ 4,361 $ 44,500 d. $65,646 $1,458,800 b. 97,706 398,800 e. 80,132 435,500 c. 111,281 257,000

Ricardo Construction began operations on December 1. In setting up its accounting procedures, the com- pany decided to debit expense accounts when it prepays its expenses and to credit revenue accounts when customers pay for services in advance. Prepare journal entries for items a through d and the adjusting en- tries as of its December 31 period-end for items e through g. Entries can draw from the following partial chart of accounts: Cash; Accounts Receivable; Interest Receivable; Supplies; Prepaid Insurance; Unearned Remodeling Fees; Remodeling Fees Earned; Supplies Expense; Insurance Expense; and Interest Expense. a. Supplies are purchased on December 1 for $2,000 cash. b. The company prepaid its insurance premiums for $1,540 cash on December 2. c. On December 15, the company receives an advance payment of $13,000 cash from a customer for re-

modeling work. d. On December 28, the company receives $3,700 cash from another customer for remodeling work to be

performed in January. e. A physical count on December 31 indicates that the company has $1,840 of supplies available. f. An analysis of insurance policies in effect on December 31 shows that $340 of insurance coverage had

expired. g. As of December 31, only one remodeling project has been worked on and completed. The $5,570 fee

for this project had been received in advance and recorded as remodeling fees earned.

Check (f ) Cr. Insurance Expense, $1,200

(g) Dr. Remodeling Fees Earned, $11,130

Exercise 3-15A Adjusting for prepaids recorded as expenses and unearned revenues recorded as revenues

P9

Use the information in the adjusted trial balance reported in Exercise 3-11 to prepare Wilson Trucking Company’s classified balance sheet as of December 31.

Exercise 3-12 Preparing a classified balance sheet C3

Calculate the current ratio for each of the following companies (round the ratio to two decimals). Identify the company with the strongest liquidity position. (These companies are competitors in the same industry.)

Current Assets Current Liabilities

Edison . . . . . . . . . . $ 79,040 $ 32,000 MAXT . . . . . . . . . . . 104,880 76,000

Chatter . . . . . . . . . 45,080 49,000

TRU . . . . . . . . . . . . 85,680 81,600

Gleeson . . . . . . . . . 61,000 100,000

Exercise 3-14 Computing and analyzing the current ratio

A2

1. Enter the accounts in proper order and enter their balances in the correct Debit or Credit column of the Unadjusted Trial Balance columns of the 10-column work sheet.

The following data are taken from the unadjusted trial balance of the Westcott Company at December 31. Each account carries a normal balance. Set up a 10-column work sheet to answer the requirements.

Exercise 3-16B Preparing unadjusted and adjusted trial balances, including the adjustments

P10 Accounts Payable . . . . . . . . . . . . . . . . . . . . $ 6 Prepaid Insurance . . . . . . $18 Retained Earnings . . . . . . . . . . . . $32

Accounts Receivable . . . . . . . . . . . . . . . . . 12 Revenue . . . . . . . . . . . . . . 75 Dividends . . . . . . . . . . . . . . . . . . . 6

Accumulated Depreciation—Equip . . . . . . . 15 Salaries Expense . . . . . . . 18 Unearned Revenue . . . . . . . . . . . 12

Cash . . . . . . . . . . . . . . . . . . . . . . . . . . . . . . 21 Supplies . . . . . . . . . . . . . . 24 Utilities Expense . . . . . . . . . . . . . 12

Equipment . . . . . . . . . . . . . . . . . . . . . . . . . 39 Common Stock . . . . . . . . 10

[continued on next page]

Chapter 3 Adjusting Accounts for Financial Statements 129

2. Use the following adjustment information to complete the Adjustments columns of the work sheet from part 1.

a. Depreciation on equipment, $3 d. Supplies available at December 31, $15 b. Accrued salaries, $6 e. Expired insurance, $15 c. The $12 of unearned revenue has been earned 3. Extend the balances in the Adjusted Trial Balance columns of the work sheet to the proper financial

statement columns. Compute totals for those columns, including net income.

PROBLEM SET A

Problem 3-1A Identifying adjusting entries with explanations

P1 P2 P3 P4

For journal entries 1 through 12, enter the letter of the explanation that most closely describes it in the space beside each entry. You can use letters more than once. A. To record receipt of unearned revenue. B. To record this period’s earning of prior

unearned revenue. C. To record payment of an accrued expense. D. To record receipt of an accrued revenue.

E. To record an accrued expense. F. To record an accrued revenue. G. To record this period’s use of a prepaid expense. H. To record payment of a prepaid expense. I. To record this period’s depreciation expense.

______ 1. Interest Expense . . . . . . . . . . . . . . . 1,000 Interest Payable . . . . . . . . . . . 1,000

______ 2. Depreciation Expense . . . . . . . . . . 4,000 Accumulated Depreciation . . 4,000

______ 3. Unearned Professional Fees . . . . . 3,000 Professional Fees Earned . . . 3,000

______ 4. Insurance Expense . . . . . . . . . . . . . 4,200 Prepaid Insurance . . . . . . . . . 4,200

______ 5. Salaries Payable . . . . . . . . . . . . . . . 1,400 Cash . . . . . . . . . . . . . . . . . . . . 1,400

______ 6. Prepaid Rent . . . . . . . . . . . . . . . . . . 4,500 Cash . . . . . . . . . . . . . . . . . . . . 4,500

______ 7. Salaries Expense . . . . . . . . . . . . . . . . . . . . . . . . 6,000 Salaries Payable . . . . . . . . . . . . . . . . . . . . 6,000

______ 8. Interest Receivable . . . . . . . . . . . . . . . . . . . . . . 5,000 Interest Revenue . . . . . . . . . . . . . . . . . . . 5,000

______ 9. Cash . . . . . . . . . . . . . . . . . . . . . . . . . . . . . . . . . . 9,000 Accounts Receivable (from consulting) . . 9,000

______ 10. Cash . . . . . . . . . . . . . . . . . . . . . . . . . . . . . . . . . . 7,500 Unearned Professional Fees . . . . . . . . . . 7,500

______ 11. Cash . . . . . . . . . . . . . . . . . . . . . . . . . . . . . . . . . . 2,000 Interest Receivable . . . . . . . . . . . . . . . . . . 2,000

______ 12. Rent Expense . . . . . . . . . . . . . . . . . . . . . . . . . . . 2,000 Prepaid Rent . . . . . . . . . . . . . . . . . . . . . . . 2,000

The following two events occurred for Trey Co. on October 31, the end of its fiscal year. a. Trey rents a building from its owner for $2,800 per month. By a prearrangement, the company de-

layed paying October’s rent until November 5. On this date, the company paid the rent for both October and November.

b. Trey rents space in a building it owns to a tenant for $850 per month. By prearrangement, the tenant delayed paying the October rent until November 8. On this date, the tenant paid the rent for both October and November.

Required

1. Prepare adjusting entries that the company must record for these events as of October 31. 2. Assuming Trey does not use reversing entries, prepare journal entries to record Trey’s payment of rent

on November 5 and the collection of the tenant’s rent on November 8. 3. Assuming that the company uses reversing entries, prepare reversing entries on November 1 and the

journal entries to record Trey’s payment of rent on November 5 and the collection of the tenant’s rent on November 8.

Exercise 3-17C Preparing reversing entries

P11

Arnez Company’s annual accounting period ends on December 31, 2019. The following information con- cerns the adjusting entries to be recorded as of that date. Entries can draw from the following partial chart of accounts: Cash; Rent Receivable; Office Supplies; Prepaid Insurance; Building; Accumulated Depreciation—Building; Salaries Payable; Unearned Rent; Rent Earned; Salaries Expense; Office Supplies Expense; Insurance Expense; and Depreciation Expense—Building.

Problem 3-2A Preparing adjusting and subsequent journal entries

P1 P2 P3 P4

[continued on next page]

130 Chapter 3 Adjusting Accounts for Financial Statements

Wells Technical Institute (WTI), a school owned by Tristana Wells, provides training to individuals who pay tuition directly to the school. WTI also offers training to groups in off-site locations. Its unadjusted trial balance as of December 31 follows, along with descriptions of items a through h that require adjust- ing entries on December 31.

Additional Information

a. An analysis of WTI’s insurance policies shows that $2,400 of coverage has expired. b. An inventory count shows that teaching supplies costing $2,800 are available at year-end. c. Annual depreciation on the equipment is $13,200. d. Annual depreciation on the professional library is $7,200. e. On September 1, WTI agreed to do five courses for a client for $2,500 each. Two courses will start

immediately and finish before the end of the year. Three courses will not begin until next year. The client paid $12,500 cash in advance for all five courses on September 1, and WTI credited Unearned Training Fees.

f. On October 15, WTI agreed to teach a four-month class (beginning immediately) for an executive with payment due at the end of the class. At December 31, $7,500 of the tuition has been earned by WTI.

g. WTI’s two employees are paid weekly. As of the end of the year, two days’ salaries have accrued at the rate of $100 per day for each employee.

h. The balance in the Prepaid Rent account represents rent for December.

Problem 3-3A Preparing adjusting entries, adjusted trial balance, and financial statements

P1 P2 P3 P4 P5 P6

Policy Date of Purchase Months of Coverage Cost

A . . . . . . April 1, 2017 24 $14,400

B . . . . . . April 1, 2018 36 12,960

C . . . . . . August 1, 2019 12 2,400

c. The company has 15 employees, who earn a total of $1,960 in salaries each working day. They are paid each Monday for their work in the five-day workweek ending on the previous Friday. Assume that December 31, 2019, is a Tuesday, and all 15 employees worked the first two days of that week. Because New Year’s Day is a paid holiday, they will be paid salaries for five full days on Monday, January 6, 2020.

d. The company purchased a building on January 1, 2019. It cost $960,000 and is expected to have a $45,000 salvage value at the end of its predicted 30-year life. Annual depreciation is $30,500.

e. Since the company is not large enough to occupy the entire building it owns, it rented space to a ten- ant at $3,000 per month, starting on November 1, 2019. The rent was paid on time on November 1, and the amount received was credited to the Rent Earned account. However, the tenant has not paid the December rent. The company has worked out an agreement with the tenant, who has promised to pay both December and January rent in full on January 15. The tenant has agreed not to fall behind again.

f. On November 1, the company rented space to another tenant for $2,800 per month. The tenant paid five months’ rent in advance on that date. The payment was recorded with a credit to the Unearned Rent account.

Required

1. Use the information to prepare adjusting entries as of December 31, 2019. 2. Prepare journal entries to record the first subsequent cash transaction in 2020 for parts c and e.

Check (1b) Dr. Insurance Expense, $7,120 (1d) Dr. Depreciation Expense, $30,500

a. The Office Supplies account started the year with a $4,000 balance. During 2019, the company pur- chased supplies for $13,400, which was added to the Office Supplies account. The inventory of sup- plies available at December 31, 2019, totaled $2,554.

b. An analysis of the company’s insurance policies provided the following facts. The total premium for each policy was paid in full (for all months) at the purchase date, and the Prepaid Insurance account was debited for the full cost. (Year-end adjusting entries for Prepaid Insurance were properly recorded in all prior years.)

Chapter 3 Adjusting Accounts for Financial Statements 131

WELLS TECHNICAL INSTITUTE Unadjusted Trial Balance

December 31

Cash Accounts receivable Teaching supplies Prepaid insurance Prepaid rent Professional library Accumulated depreciation—Professional library Equipment Accumulated depreciation—Equipment Accounts payable Salaries payable Unearned training fees

Tuition fees earned Training fees earned Depreciation expense—Professional library Depreciation expense—Equipment Salaries expense Insurance expense Rent expense Teaching supplies expense Advertising expense Utilities expense Totals

Common stock Retained earnings 80,000

10,000

Dividends

Debit $ 34,000

0 8,000

12,000 3,000

35,000

80,000

50,000

0 0

50,000 0

33,000 0

6,000 6,400

$317,400

Credit

$317,400

$ 10,000

15,000 26,000

0 12,500

123,900 40,000

Required

1. Prepare T-accounts (representing the ledger) with balances from the unadjusted trial balance. 2. Prepare the necessary adjusting journal entries for items a through h and post them to the T-accounts.

Assume that adjusting entries are made only at year-end. 3. Update balances in the T-accounts for the adjusting entries and prepare an adjusted trial balance. 4. Prepare Wells Technical Institute’s income statement and statement of retained earnings for the year

and prepare its balance sheet as of December 31. The Retained Earnings account balance was $80,000 on December 31 of the prior year.

Check (2e) Cr. Training Fees Earned, $5,000 (2f) Cr. Tuition Fees Earned, $7,500 (3) Adj. trial balance totals, $345,700 (4) Net income, $49,600

The adjusted trial balance for Chiara Company as of December 31 follows. Problem 3-4A Preparing financial statements from the adjusted trial balance

P6

Debit Credit

Cash . . . . . . . . . . . . . . . . . . . . . . . . . . . . . . . . . . . . . . . . . . . . . . $ 30,000 Accounts receivable . . . . . . . . . . . . . . . . . . . . . . . . . . . . . . . . . 52,000 Interest receivable . . . . . . . . . . . . . . . . . . . . . . . . . . . . . . . . . . . 18,000 Notes receivable (due in 90 days) . . . . . . . . . . . . . . . . . . . . . . 168,000 Office supplies . . . . . . . . . . . . . . . . . . . . . . . . . . . . . . . . . . . . . . 16,000 Automobiles . . . . . . . . . . . . . . . . . . . . . . . . . . . . . . . . . . . . . . . . 168,000 Accumulated depreciation—Automobiles . . . . . . . . . . . . . . . . $ 50,000 Equipment . . . . . . . . . . . . . . . . . . . . . . . . . . . . . . . . . . . . . . . . . 138,000 Accumulated depreciation—Equipment . . . . . . . . . . . . . . . . . . 18,000 Land . . . . . . . . . . . . . . . . . . . . . . . . . . . . . . . . . . . . . . . . . . . . . . 78,000 Accounts payable . . . . . . . . . . . . . . . . . . . . . . . . . . . . . . . . . . . 96,000 Interest payable . . . . . . . . . . . . . . . . . . . . . . . . . . . . . . . . . . . . . 20,000 Salaries payable . . . . . . . . . . . . . . . . . . . . . . . . . . . . . . . . . . . . 19,000 Unearned fees . . . . . . . . . . . . . . . . . . . . . . . . . . . . . . . . . . . . . . 30,000 Long-term notes payable . . . . . . . . . . . . . . . . . . . . . . . . . . . . . 138,000 Common stock . . . . . . . . . . . . . . . . . . . . . . . . . . . . . . . . . . . . . . 20,000 Retained earnings . . . . . . . . . . . . . . . . . . . . . . . . . . . . . . . . . . . . 235,800 Dividends . . . . . . . . . . . . . . . . . . . . . . . . . . . . . . . . . . . . . . . . . . 46,000

[continued on next page]

132 Chapter 3 Adjusting Accounts for Financial Statements

Required

Use the information in the adjusted trial balance to prepare (a) the income statement for the year ended December 31; (b) the statement of retained earnings for the year ended December 31 [Note: Retained Earnings at December 31 of the prior year was $235,800]; and (c) the balance sheet as of December 31.

Check Total assets, $600,000

Fees earned . . . . . . . . . . . . . . . . . . . . . . . . . . . . . . . . . . . . . . . . 484,000 Interest earned . . . . . . . . . . . . . . . . . . . . . . . . . . . . . . . . . . . . . 24,000 Depreciation expense—Automobiles . . . . . . . . . . . . . . . . . . . . 26,000 Depreciation expense—Equipment . . . . . . . . . . . . . . . . . . . . . 18,000 Salaries expense . . . . . . . . . . . . . . . . . . . . . . . . . . . . . . . . . . . . 188,000 Wages expense . . . . . . . . . . . . . . . . . . . . . . . . . . . . . . . . . . . . . 40,000 Interest expense . . . . . . . . . . . . . . . . . . . . . . . . . . . . . . . . . . . . 32,000 Office supplies expense . . . . . . . . . . . . . . . . . . . . . . . . . . . . . . 34,000 Advertising expense . . . . . . . . . . . . . . . . . . . . . . . . . . . . . . . . . 58,000 Repairs expense—Automobiles . . . . . . . . . . . . . . . . . . . . . . . . 24,800 Totals . . . . . . . . . . . . . . . . . . . . . . . . . . . . . . . . . . . . . . . . . . . . . $1,134,800 $1,134,800

[continued from previous page]

On April 1, Jiro Nozomi created a new travel agency, Adventure Travel. The following transactions occurred during the company’s first month.

Apr. 1 Nozomi invested $30,000 cash and computer equipment worth $20,000 in the company in exchange for common stock.

2 The company rented furnished office space by paying $1,800 cash for the first month’s (April) rent.

3 The company purchased $1,000 of office supplies for cash. 10 The company paid $2,400 cash for the premium on a 12-month insurance policy. Coverage

begins on April 11. 14 The company paid $1,600 cash for two weeks’ salaries earned by employees. 24 The company collected $8,000 cash for commissions earned. 28 The company paid $1,600 cash for two weeks’ salaries earned by employees. 29 The company paid $350 cash for minor repairs to the company’s computer. 30 The company paid $750 cash for this month’s telephone bill. 30 The company paid $1,500 cash in dividends.

The company’s chart of accounts follows.

Problem 3-5A Applying the accounting cycle

P1 P2 P3 P4 P5 P6 P7 P8

Required

1. Use the balance column format to set up each ledger account listed in its chart of accounts. 2. Prepare journal entries to record the transactions for April and post them to the ledger accounts. The

company records prepaid and unearned items in balance sheet accounts. 3. Prepare an unadjusted trial balance as of April 30. 4. Use the following information to journalize and post adjusting entries for the month: a. Prepaid insurance of $133 has expired this month. b. At the end of the month, $600 of office supplies are still available. c. This month’s depreciation on the computer equipment is $500. d. Employees earned $420 of unpaid and unrecorded salaries as of month-end. e. The company earned $1,750 of commissions that are not yet billed at month-end. 5. Prepare the adjusted trial balance as of April 30. Prepare the income statement and the statement of

retained earnings for the month of April and the balance sheet at April 30. 6. Prepare journal entries to close the temporary accounts and post these entries to the ledger. 7. Prepare a post-closing trial balance.

Check (3) Unadj. trial balance totals, $58,000

(4a) Dr. Insurance Expense, $133

(5) Net income, $2,197; Total assets, $51,117

101 Cash 106 Accounts Receivable 124 Office Supplies 128 Prepaid Insurance 167 Computer Equipment 168 Accumulated Depreciation—Computer Equip . 209 Salaries Payable

307 Common Stock 318 Retained Earnings 319 Dividends 405 Commissions Earned 612 Depreciation Expense — Computer Equip . 622 Salaries Expense 637 Insurance Expense

640 Rent Expense 650 Office Supplies Expense 684 Repairs Expense 688 Telephone Expense 901 Income Summary

(7) P-C trial balance totals, $51,617

Chapter 3 Adjusting Accounts for Financial Statements 133

The adjusted trial balance for Tybalt Construction as of December 31, 2019, follows. O. Tybalt invested $5,000 cash in the business in exchange for common stock during year 2019. The December 31, 2018, credit balance of the Retained Earnings account was $121,400.

Problem 3-6A Preparing closing entries and financial statements

P6 P7Adjusted Trial Balance December 31, 2019

No. Account Title Debit Credit

101 Cash . . . . . . . . . . . . . . . . . . . . . . . . . . . . . . . . . . . . . . . . . $ 5,000 104 Short-term investments . . . . . . . . . . . . . . . . . . . . . . . . . . 23,000 126 Supplies . . . . . . . . . . . . . . . . . . . . . . . . . . . . . . . . . . . . . . 8,100 128 Prepaid insurance . . . . . . . . . . . . . . . . . . . . . . . . . . . . . . 7,000 167 Equipment . . . . . . . . . . . . . . . . . . . . . . . . . . . . . . . . . . . . . 40,000 168 Accumulated depreciation—Equipment . . . . . . . . . . . . . $ 20,000 173 Building . . . . . . . . . . . . . . . . . . . . . . . . . . . . . . . . . . . . . . . 150,000 174 Accumulated depreciation—Building . . . . . . . . . . . . . . . 50,000 183 Land . . . . . . . . . . . . . . . . . . . . . . . . . . . . . . . . . . . . . . . . . 55,000 201 Accounts payable . . . . . . . . . . . . . . . . . . . . . . . . . . . . . . . 16,500 203 Interest payable . . . . . . . . . . . . . . . . . . . . . . . . . . . . . . . . 2,500 208 Rent payable . . . . . . . . . . . . . . . . . . . . . . . . . . . . . . . . . . . 3,500 210 Wages payable . . . . . . . . . . . . . . . . . . . . . . . . . . . . . . . . . 2,500 213 Property taxes payable . . . . . . . . . . . . . . . . . . . . . . . . . . 900 233 Unearned professional fees . . . . . . . . . . . . . . . . . . . . . . . 7,500 244 Current portion of long-term note payable . . . . . . . . . . . 7,000 251 Long-term notes payable . . . . . . . . . . . . . . . . . . . . . . . . . 60,000 307 Common stock . . . . . . . . . . . . . . . . . . . . . . . . . . . . . . . . . 5,000 318 Retained earnings . . . . . . . . . . . . . . . . . . . . . . . . . . . . . . 121,400 319 Dividends . . . . . . . . . . . . . . . . . . . . . . . . . . . . . . . . . . . . . 13,000 401 Professional fees earned . . . . . . . . . . . . . . . . . . . . . . . . . 97,000 406 Rent earned . . . . . . . . . . . . . . . . . . . . . . . . . . . . . . . . . . . 14,000 407 Dividends earned . . . . . . . . . . . . . . . . . . . . . . . . . . . . . . . 2,000 409 Interest earned . . . . . . . . . . . . . . . . . . . . . . . . . . . . . . . . . 2,100 606 Depreciation expense—Building . . . . . . . . . . . . . . . . . . . 11,000 612 Depreciation expense—Equipment . . . . . . . . . . . . . . . . . 6,000 623 Wages expense . . . . . . . . . . . . . . . . . . . . . . . . . . . . . . . . 32,000 633 Interest expense . . . . . . . . . . . . . . . . . . . . . . . . . . . . . . . . 5,100 637 Insurance expense . . . . . . . . . . . . . . . . . . . . . . . . . . . . . . 10,000 640 Rent expense . . . . . . . . . . . . . . . . . . . . . . . . . . . . . . . . . . 13,400 652 Supplies expense . . . . . . . . . . . . . . . . . . . . . . . . . . . . . . . 7,400 682 Postage expense . . . . . . . . . . . . . . . . . . . . . . . . . . . . . . . 4,200 683 Property taxes expense . . . . . . . . . . . . . . . . . . . . . . . . . . 5,000 684 Repairs expense . . . . . . . . . . . . . . . . . . . . . . . . . . . . . . . . 8,900 688 Telephone expense . . . . . . . . . . . . . . . . . . . . . . . . . . . . . 3,200 690 Utilities expense . . . . . . . . . . . . . . . . . . . . . . . . . . . . . . . . 4,600 Totals . . . . . . . . . . . . . . . . . . . . . . . . . . . . . . . . . . . . . . . . . $411,900 $411,900

Required

1. Prepare the income statement and the statement of retained earnings for calendar-year 2019 and the classified balance sheet at December 31, 2019.

2. Prepare the necessary closing entries at December 31, 2019.

Check (1) Total assets (12/31/2019), $218,100; Net income, $4,300

In the blank space beside each numbered balance sheet item, enter the letter of its balance sheet classifica- tion. If the item should not appear on the balance sheet, enter a Z in the blank. A. Current assets B. Long-term investments C. Plant assets

Problem 3-7A Determining balance sheet classifications

C3 D. Intangible assets E. Current liabilities

F. Long-term liabilities G. Equity

1. Long-term investment in stock 2. Depreciation expense—Building 3. Prepaid rent (2 months of rent) 4. Interest receivable 5. Taxes payable (due in 5 weeks) 6. Automobiles 7. Notes payable (due in 3 years) 8. Accounts payable 9. Cash 10. Common stock

11. Unearned services revenue 12. Accumulated depreciation—Trucks 13. Prepaid insurance (expires in 5 months) 14. Buildings 15. Store supplies 16. Office equipment 17. Land (used in operations) 18. Repairs expense 19. Office supplies 20. Current portion of long-term note payable

134 Chapter 3 Adjusting Accounts for Financial Statements

PROBLEM SET B

Problem 3-1B Identifying adjusting entries with explanations

P1 P2 P3 P4

For each of the following journal entries 1 through 12, enter the letter of the explanation that most closely describes it in the space beside each entry. You can use letters more than once. A. To record payment of a prepaid expense. B. To record this period’s use of a prepaid expense. C. To record this period’s depreciation expense. D. To record receipt of unearned revenue. E. To record this period’s earning of prior

unearned revenue.

F. To record an accrued expense. G. To record payment of an accrued expense. H. To record an accrued revenue. I. To record receipt of accrued revenue.

______ 1. Interest Receivable . . . . . . . . . . . . . 3,500 Interest Revenue . . . . . . . . . . 3,500

______ 2. Salaries Payable . . . . . . . . . . . . . . . 9,000 Cash . . . . . . . . . . . . . . . . . . . . 9,000

______ 3. Depreciation Expense . . . . . . . . . . 8,000 Accumulated Depreciation . . 8,000

______ 4. Cash . . . . . . . . . . . . . . . . . . . . . . . . . 9,000 Unearned Professional Fees . 9,000

______ 5. Insurance Expense . . . . . . . . . . . . . 4,000 Prepaid Insurance . . . . . . . . . 4,000

______ 6. Interest Expense . . . . . . . . . . . . . . . 5,000 Interest Payable . . . . . . . . . . . 5,000

______ 7. Cash . . . . . . . . . . . . . . . . . . . . . . . . . . . . . . . . . . . 1,500 Accounts Receivable (from services) . . . . 1,500

______ 8. Salaries Expense . . . . . . . . . . . . . . . . . . . . . . . . . 7,000 Salaries Payable . . . . . . . . . . . . . . . . . . . . 7,000

______ 9. Cash . . . . . . . . . . . . . . . . . . . . . . . . . . . . . . . . . . . 1,000 Interest Receivable . . . . . . . . . . . . . . . . . . 1,000

______ 10. Prepaid Rent . . . . . . . . . . . . . . . . . . . . . . . . . . . . 3,000 Cash . . . . . . . . . . . . . . . . . . . . . . . . . . . . . . 3,000

______ 11. Rent Expense . . . . . . . . . . . . . . . . . . . . . . . . . . . 7,500 Prepaid Rent . . . . . . . . . . . . . . . . . . . . . . . 7,500

______ 12. Unearned Professional Fees . . . . . . . . . . . . . . . 6,000 Professional Fees Earned . . . . . . . . . . . . . 6,000

Problem 3-2B Preparing adjusting and subsequent journal entries

P1 P2 P3 P4

Natsu Company’s annual accounting period ends on October 31, 2019. The following information con- cerns the adjusting entries that need to be recorded as of that date. Entries can draw from the following partial chart of accounts: Cash; Rent Receivable; Office Supplies; Prepaid Insurance; Building; Accumulated Depreciation—Building; Salaries Payable; Unearned Rent; Rent Earned; Salaries Expense; Office Supplies Expense; Insurance Expense; and Depreciation Expense—Building. a. The Office Supplies account started the fiscal year with a $600 balance. During the fiscal year, the

company purchased supplies for $4,570, which was added to the Office Supplies account. The sup- plies available at October 31, 2019, totaled $800.

b. An analysis of the company’s insurance policies provided the following facts. The total premium for each policy was paid in full (for all months) at the purchase date, and the Prepaid Insurance account was debited for the full cost. (Year-end adjusting entries for Prepaid Insurance were properly recorded in all prior fiscal years.)

Policy Date of Purchase Months of Coverage Cost

A . . . . . . April 1, 2018 24 $6,000 B . . . . . . April 1, 2019 36 7,200 C . . . . . . August 1, 2019 12 1,320

c. The company has four employees, who earn a total of $1,000 for each workday. They are paid each Monday for their work in the five-day workweek ending on the previous Friday. Assume that October 31, 2019, is a Monday, and all four employees worked the first day of that week. They will be paid salaries for five full days on Monday, November 7, 2019.

d. The company purchased a building on November 1, 2016, that cost $175,000 and is expected to have a $40,000 salvage value at the end of its predicted 25-year life. Annual depreciation is $5,400.

e. Because the company does not occupy the entire building it owns, it rented space to a tenant at $1,000 per month, starting on September 1, 2019. The rent was paid on time on September 1, and the amount received was credited to the Rent Earned account. However, the October rent has not been paid. The company has worked out an agreement with the tenant, who has promised to pay both October and November rent in full on November 15. The tenant has agreed not to fall behind again.

f. On September 1, the company rented space to another tenant for $725 per month. The tenant paid five months’ rent in advance on that date. The payment was recorded with a credit to the Unearned Rent account.

Required

1. Use the information to prepare adjusting entries as of October 31, 2019. 2. Prepare journal entries to record the first subsequent cash transaction in November 2019 for parts c and e.

Check (1b) Dr. Insurance Expense, $4,730 (1d) Dr. Depreciation Expense, $5,400

Chapter 3 Adjusting Accounts for Financial Statements 135

Following is the unadjusted trial balance for Alonzo Institute as of December 31. The Institute provides one-on-one training to individuals who pay tuition directly to the business and offers extension training to groups in off-site locations. Shown after the trial balance are items a through h that require adjusting entries as of December 31.

Problem 3-3B Preparing adjusting entries, adjusted trial balance, and financial statements

P1 P2 P3 P4 P5 P6 ALONZO INSTITUTE

Unadjusted Trial Balance December 31

Cash Accounts receivable Teaching supplies Prepaid insurance Prepaid rent Professional library Accumulated depreciation—Professional library Equipment Accumulated depreciation—Equipment Accounts payable Salaries payable Unearned training fees Common stock

Tuition fees earned Training fees earned Depreciation expense—Professional library Depreciation expense—Equipment Salaries expense Insurance expense Rent expense Teaching supplies expense Advertising expense Utilities expense Totals

0

0 0

0

$ 60,000 0

70,000 19,000 3,800

12,000

40,000

20,000Dividends

44,200

29,600

19,000 13,400

$331,000

Debit

$331,000

$ 2,500

20,000 11,200

0 28,600 11,000

129,200 68,000

Credit

Retained earnings 60,500

Additional Information

a. An analysis of the Institute’s insurance policies shows that $9,500 of coverage has expired. b. An inventory count shows that teaching supplies costing $20,000 are available at year-end. c. Annual depreciation on the equipment is $5,000. d. Annual depreciation on the professional library is $2,400. e. On November 1, the Institute agreed to do a special two-month course (starting immediately) for

a client. The contract calls for a $14,300 monthly fee, and the client paid the two months’ fees in advance. When the cash was received, the Unearned Training Fees account was credited.

f. On October 15, the Institute agreed to teach a four-month class (beginning immediately) to an execu- tive with payment due at the end of the class. At December 31, $5,750 of the tuition has been earned by the Institute.

g. The Institute’s only employee is paid weekly. As of the end of the year, three days’ salaries have accrued at the rate of $150 per day.

h. The balance in the Prepaid Rent account represents rent for December.

Required

1. Prepare T-accounts (representing the ledger) with balances from the unadjusted trial balance. 2. Prepare the necessary adjusting journal entries for items a through h, and post them to the T-accounts.

Assume that adjusting entries are made only at year-end. 3. Update balances in the T-accounts for the adjusting entries and prepare an adjusted trial balance. 4. Prepare the company’s income statement and statement of retained earnings for the year, and prepare its

balance sheet as of December 31. The Retained Earnings account balance was $60,500 on December 31 of the prior year.

Check (2e) Cr. Training Fees Earned, $28,600 (2f ) Cr. Tuition Fees Earned, $5,750 (3) Adj. trial balance totals, $344,600 (4) Net income, $54,200

136 Chapter 3 Adjusting Accounts for Financial Statements

Problem 3-4B Preparing financial statements from adjusted trial balance

P6

The adjusted trial balance for Speedy Courier as of December 31 follows.

Debit Credit

Cash . . . . . . . . . . . . . . . . . . . . . . . . . . . . . . . . . . . . . . . . . $ 58,000 Accounts receivable . . . . . . . . . . . . . . . . . . . . . . . . . . . . 120,000 Interest receivable . . . . . . . . . . . . . . . . . . . . . . . . . . . . . . 7,000 Notes receivable (due in 90 days) . . . . . . . . . . . . . . . . . 210,000 Office supplies . . . . . . . . . . . . . . . . . . . . . . . . . . . . . . . . . 22,000 Trucks . . . . . . . . . . . . . . . . . . . . . . . . . . . . . . . . . . . . . . . . 134,000 Accumulated depreciation—Trucks . . . . . . . . . . . . . . . . $ 58,000 Equipment . . . . . . . . . . . . . . . . . . . . . . . . . . . . . . . . . . . . 270,000 Accumulated depreciation—Equipment . . . . . . . . . . . . . 200,000 Land . . . . . . . . . . . . . . . . . . . . . . . . . . . . . . . . . . . . . . . . . 100,000 Accounts payable . . . . . . . . . . . . . . . . . . . . . . . . . . . . . . 134,000 Interest payable . . . . . . . . . . . . . . . . . . . . . . . . . . . . . . . . 20,000 Salaries payable . . . . . . . . . . . . . . . . . . . . . . . . . . . . . . . 28,000 Unearned delivery fees . . . . . . . . . . . . . . . . . . . . . . . . . . 120,000 Long-term notes payable . . . . . . . . . . . . . . . . . . . . . . . . 200,000 Common stock . . . . . . . . . . . . . . . . . . . . . . . . . . . . . . . . . 15,000 Retained earnings . . . . . . . . . . . . . . . . . . . . . . . . . . . . . . . 110,000 Dividends . . . . . . . . . . . . . . . . . . . . . . . . . . . . . . . . . . . . . 50,000 Delivery fees earned . . . . . . . . . . . . . . . . . . . . . . . . . . . . 611,800 Interest earned . . . . . . . . . . . . . . . . . . . . . . . . . . . . . . . . 34,000 Depreciation expense—Trucks . . . . . . . . . . . . . . . . . . . . 29,000 Depreciation expense—Equipment . . . . . . . . . . . . . . . . 48,000 Salaries expense . . . . . . . . . . . . . . . . . . . . . . . . . . . . . . . 74,000 Wages expense . . . . . . . . . . . . . . . . . . . . . . . . . . . . . . . . 300,000 Interest expense . . . . . . . . . . . . . . . . . . . . . . . . . . . . . . . 15,000 Office supplies expense . . . . . . . . . . . . . . . . . . . . . . . . . 31,000 Advertising expense . . . . . . . . . . . . . . . . . . . . . . . . . . . . 27,200 Repairs expense—Trucks . . . . . . . . . . . . . . . . . . . . . . . . 35,600 Totals . . . . . . . . . . . . . . . . . . . . . . . . . . . . . . . . . . . . . . . . . $1,530,800 $1,530,800

Required

Use the information in the adjusted trial balance to prepare (a) the income statement for the year ended December 31; (b) the statement of retained earnings for the year ended December 31 [Note: Retained Earnings at Dec. 31 of the prior year was $110,000]; and (c) the balance sheet as of December 31.

Check Total assets, $663,000

101 Cash 307 Common Stock 640 Rent Expense 106 Accounts Receivable 318 Retained Earnings 650 Office Supplies Expense 124 Office Supplies 319 Dividends 684 Repairs Expense 128 Prepaid Insurance 401 Storage Fees Earned 688 Telephone Expense 173 Buildings 606 Depreciation Expense—Buildings 901 Income Summary 174 Accumulated Depreciation—Buildings 622 Salaries Expense 209 Salaries Payable 637 Insurance Expense

On July 1, Lula Plume created a new self-storage business, Safe Storage Co. The following transactions occurred during the company’s first month.

July 1 Plume invested $30,000 cash and buildings worth $150,000 in the company in exchange for common stock.

2 The company rented equipment by paying $2,000 cash for the first month’s (July) rent. 5 The company purchased $2,400 of office supplies for cash. 10 The company paid $7,200 cash for the premium on a 12-month insurance policy. Coverage

begins on July 11. 14 The company paid an employee $1,000 cash for two weeks’ salary earned. 24 The company collected $9,800 cash for storage fees from customers. 28 The company paid $1,000 cash for two weeks’ salary earned by an employee. 29 The company paid $950 cash for minor repairs to a leaking roof. 30 The company paid $400 cash for this month’s telephone bill. 31 The company paid $2,000 cash in dividends.

The company’s chart of accounts follows.

Problem 3-5B Applying the accounting cycle

P1 P2 P3 P4 P5 P6 P7 P8

Chapter 3 Adjusting Accounts for Financial Statements 137

Required

1. Use the balance column format to set up each ledger account listed in its chart of accounts. 2. Prepare journal entries to record the transactions for July and post them to the ledger accounts. Record

prepaid and unearned items in balance sheet accounts. 3. Prepare an unadjusted trial balance as of July 31. 4. Use the following information to journalize and post adjusting entries for the month: a. Prepaid insurance of $400 has expired this month. b. At the end of the month, $1,525 of office supplies are still available. c. This month’s depreciation on the buildings is $1,500. d. An employee earned $100 of unpaid and unrecorded salary as of month-end. e. The company earned $1,150 of storage fees that are not yet billed at month-end. 5. Prepare the adjusted trial balance as of July 31. Prepare the income statement and the statement of

retained earnings for the month of July and the balance sheet at July 31. 6. Prepare journal entries to close the temporary accounts and post these entries to the ledger. 7. Prepare a post-closing trial balance.

Check (3) Unadj. trial balance totals, $189,800

(4a) Dr. Insurance Expense, $400

(5) Net income, $2,725; Total assets, $180,825

(7) P-C trial balance totals, $182,325

The adjusted trial balance for Anara Co. as of December 31, 2019, follows. P. Anara invested $40,000 cash in the business in exchange for common stock during year 2019. The December 31, 2018, credit bal- ance of the Retained Earnings account was $52,800.

Problem 3-6B Preparing closing entries and financial statements

P6 P7 Adjusted Trial Balance December 31, 2019

No. Account Title Debit Credit

101 Cash . . . . . . . . . . . . . . . . . . . . . . . . . . . . . . . . . . . . . . . . . $ 7,400 104 Short-term investments . . . . . . . . . . . . . . . . . . . . . . . . . . 11,200 126 Supplies . . . . . . . . . . . . . . . . . . . . . . . . . . . . . . . . . . . . . . 4,600 128 Prepaid insurance . . . . . . . . . . . . . . . . . . . . . . . . . . . . . . 1,000 167 Equipment . . . . . . . . . . . . . . . . . . . . . . . . . . . . . . . . . . . . . 24,000 168 Accumulated depreciation—Equipment . . . . . . . . . . . . . $ 4,000 173 Building . . . . . . . . . . . . . . . . . . . . . . . . . . . . . . . . . . . . . . . 100,000 174 Accumulated depreciation—Building . . . . . . . . . . . . . . . 10,000 183 Land . . . . . . . . . . . . . . . . . . . . . . . . . . . . . . . . . . . . . . . . . 30,500 201 Accounts payable . . . . . . . . . . . . . . . . . . . . . . . . . . . . . . . 3,500 203 Interest payable . . . . . . . . . . . . . . . . . . . . . . . . . . . . . . . . 1,750 208 Rent payable . . . . . . . . . . . . . . . . . . . . . . . . . . . . . . . . . . . 400 210 Wages payable . . . . . . . . . . . . . . . . . . . . . . . . . . . . . . . . . 1,280 213 Property taxes payable . . . . . . . . . . . . . . . . . . . . . . . . . . 3,330 233 Unearned professional fees . . . . . . . . . . . . . . . . . . . . . . . 750 244 Current portion of long-term notes payable . . . . . . . . . . 8,400 251 Long-term notes payable . . . . . . . . . . . . . . . . . . . . . . . . . 31,600 307 Common stock . . . . . . . . . . . . . . . . . . . . . . . . . . . . . . . . . 40,000 318 Retained earnings . . . . . . . . . . . . . . . . . . . . . . . . . . . . . . 52,800 319 Dividends . . . . . . . . . . . . . . . . . . . . . . . . . . . . . . . . . . . . . 8,000 401 Professional fees earned . . . . . . . . . . . . . . . . . . . . . . . . . 59,600 406 Rent earned . . . . . . . . . . . . . . . . . . . . . . . . . . . . . . . . . . . 4,500 407 Dividends earned . . . . . . . . . . . . . . . . . . . . . . . . . . . . . . . 1,000 409 Interest earned . . . . . . . . . . . . . . . . . . . . . . . . . . . . . . . . . 1,320 606 Depreciation expense—Building . . . . . . . . . . . . . . . . . . . 2,000 612 Depreciation expense—Equipment . . . . . . . . . . . . . . . . . 1,000 623 Wages expense . . . . . . . . . . . . . . . . . . . . . . . . . . . . . . . . 18,500 633 Interest expense . . . . . . . . . . . . . . . . . . . . . . . . . . . . . . . . 1,550 637 Insurance expense . . . . . . . . . . . . . . . . . . . . . . . . . . . . . . 1,525 640 Rent expense . . . . . . . . . . . . . . . . . . . . . . . . . . . . . . . . . . 3,600 652 Supplies expense . . . . . . . . . . . . . . . . . . . . . . . . . . . . . . . 1,000 682 Postage expense . . . . . . . . . . . . . . . . . . . . . . . . . . . . . . . 410 683 Property taxes expense . . . . . . . . . . . . . . . . . . . . . . . . . . 4,825 684 Repairs expense . . . . . . . . . . . . . . . . . . . . . . . . . . . . . . . . 679 688 Telephone expense . . . . . . . . . . . . . . . . . . . . . . . . . . . . . 521 690 Utilities expense . . . . . . . . . . . . . . . . . . . . . . . . . . . . . . . . 1,920 Totals . . . . . . . . . . . . . . . . . . . . . . . . . . . . . . . . . . . . . . . . . $224,230 $224,230

138 Chapter 3 Adjusting Accounts for Financial Statements

SERIAL PROBLEM Business Solutions

P1 P2 P3 P4 P5 P6 P7 P8

This serial problem began in Chapter 1 and continues through most of the book. If previous chapter seg- ments were not completed, the serial problem can begin at this point.

SP 3 After the success of the company’s first two months, Santana Rey continues to operate Business Solutions. (Transactions for the first two months are described in the Chapter 2 serial problem.) The November 30, 2019, unadjusted trial balance of Business Solutions (reflecting its transactions for October and November of 2019) follows.

No. Account Title Debit Credit

101 Cash . . . . . . . . . . . . . . . . . . . . . . . . . . . . . . . . . . . . . . . . . . . . . . . . . . . $38,264 106 Accounts receivable . . . . . . . . . . . . . . . . . . . . . . . . . . . . . . . . . . . . . . . 12,618 126 Computer supplies . . . . . . . . . . . . . . . . . . . . . . . . . . . . . . . . . . . . . . . . 2,545 128 Prepaid insurance . . . . . . . . . . . . . . . . . . . . . . . . . . . . . . . . . . . . . . . . . 2,220 131 Prepaid rent . . . . . . . . . . . . . . . . . . . . . . . . . . . . . . . . . . . . . . . . . . . . . 3,300 163 Office equipment . . . . . . . . . . . . . . . . . . . . . . . . . . . . . . . . . . . . . . . . . 8,000 164 Accumulated depreciation—Office equipment . . . . . . . . . . . . . . . . . . $ 0 167 Computer equipment . . . . . . . . . . . . . . . . . . . . . . . . . . . . . . . . . . . . . 20,000 168 Accumulated depreciation—Computer equipment . . . . . . . . . . . . . . 0 201 Accounts payable . . . . . . . . . . . . . . . . . . . . . . . . . . . . . . . . . . . . . . . . 0 210 Wages payable . . . . . . . . . . . . . . . . . . . . . . . . . . . . . . . . . . . . . . . . . . 0 236 Unearned computer services revenue . . . . . . . . . . . . . . . . . . . . . . . . 0 307 Common stock . . . . . . . . . . . . . . . . . . . . . . . . . . . . . . . . . . . . . . . . . . . 73,000 318 Retained earnings . . . . . . . . . . . . . . . . . . . . . . . . . . . . . . . . . . . . . . . . . 0 319 Dividends . . . . . . . . . . . . . . . . . . . . . . . . . . . . . . . . . . . . . . . . . . . . . . . 5,600 403 Computer services revenue . . . . . . . . . . . . . . . . . . . . . . . . . . . . . . . . 25,659 612 Depreciation expense—Office equipment . . . . . . . . . . . . . . . . . . . . . 0 613 Depreciation expense—Computer equipment . . . . . . . . . . . . . . . . . . 0 623 Wages expense . . . . . . . . . . . . . . . . . . . . . . . . . . . . . . . . . . . . . . . . . . 2,625 637 Insurance expense . . . . . . . . . . . . . . . . . . . . . . . . . . . . . . . . . . . . . . . 0 640 Rent expense . . . . . . . . . . . . . . . . . . . . . . . . . . . . . . . . . . . . . . . . . . . . 0 652 Computer supplies expense . . . . . . . . . . . . . . . . . . . . . . . . . . . . . . . . 0 655 Advertising expense . . . . . . . . . . . . . . . . . . . . . . . . . . . . . . . . . . . . . . . 1,728 676 Mileage expense . . . . . . . . . . . . . . . . . . . . . . . . . . . . . . . . . . . . . . . . . 704 677 Miscellaneous expenses . . . . . . . . . . . . . . . . . . . . . . . . . . . . . . . . . . . 250 684 Repairs expense—Computer . . . . . . . . . . . . . . . . . . . . . . . . . . . . . . . 805 Totals . . . . . . . . . . . . . . . . . . . . . . . . . . . . . . . . . . . . . . . . . . . . . . . . . . $98,659 $98,659

Required

1. Prepare the income statement and the statement of retained earnings for calendar-year 2019 and the classified balance sheet at December 31, 2019.

2. Prepare the necessary closing entries at December 31, 2019.

Check (1) Total assets (12/31/2019), $164,700; Net income, $28,890

In the blank space beside each numbered balance sheet item, enter the letter of its balance sheet classifica- tion. If the item should not appear on the balance sheet, enter a Z in the blank. A. Current assets D. Intangible assets F. Long-term liabilities B. Long-term investments E. Current liabilities G. Equity C. Plant assets

Problem 3-7B Determining balance sheet classifications

C3

1. Commissions earned 2. Interest receivable 3. Long-term investment in stock 4. Prepaid insurance (4 months of rent) 5. Machinery 6. Notes payable (due in 15 years) 7. Copyrights 8. Current portion of long-term

note payable 9. Accumulated depreciation—Trucks 10. Office equipment

11. Rent receivable 12. Salaries payable 13. Income taxes payable

(due in 11 weeks) 14. Common stock 15. Office supplies 16. Interest payable 17. Rent revenue 18. Notes receivable (due in 120 days) 19. Land (used in operations) 20. Depreciation expense—Trucks

©Alexander Image/Shutterstock

Chapter 3 Adjusting Accounts for Financial Statements 139

Business Solutions had the following transactions and events in December 2019.

Dec. 2 Paid $1,025 cash to Hillside Mall for Business Solutions’s share of mall advertising costs. 3 Paid $500 cash for minor repairs to the company’s computer. 4 Received $3,950 cash from Alex’s Engineering Co. for the receivable from November. 10 Paid cash to Lyn Addie for six days of work at the rate of $125 per day. 14 Notified by Alex’s Engineering Co. that Business Solutions’s bid of $7,000 on a proposed proj-

ect has been accepted. Alex’s paid a $1,500 cash advance to Business Solutions. 15 Purchased $1,100 of computer supplies on credit from Harris Office Products. 16 Sent a reminder to Gomez Co. to pay the fee for services recorded on November 8. 20 Completed a project for Liu Corporation and received $5,625 cash. 22–26 Took the week off for the holidays. 28 Received $3,000 cash from Gomez Co. on its receivable. 29 Reimbursed S. Rey for business automobile mileage (600 miles at $0.32 per mile). 31 The company paid $1,500 cash in dividends.

The following additional facts are collected for use in making adjusting entries prior to preparing financial statements for the company’s first three months. a. The December 31 inventory count of computer supplies shows $580 still available. b. Three months have expired since the 12-month insurance premium was paid in advance. c. As of December 31, Lyn Addie has not been paid for four days of work at $125 per day. d. The computer system, acquired on October 1, is expected to have a four-year life with no salvage value. e. The office equipment, acquired on October 1, is expected to have a five-year life with no salvage value. f. Three of the four months’ prepaid rent have expired.

Required

1. Prepare journal entries to record each of the December transactions and events for Business Solutions. Post those entries to the accounts in the ledger.

2. Prepare adjusting entries to reflect a through f. Post those entries to the accounts in the ledger. 3. Prepare an adjusted trial balance as of December 31, 2019. 4. Prepare an income statement for the three months ended December 31, 2019. 5. Prepare a statement of retained earnings for the three months ended December 31, 2019. 6. Prepare a balance sheet as of December 31, 2019. 7. Record and post the necessary closing entries as of December 31, 2019. 8. Prepare a post-closing trial balance as of December 31, 2019.

Check (3) Adjusted trial balance totals, $109,034

(6) Total assets, $83,460

(8) Post-closing trial balance totals, $85,110

The General Ledger tool in Connect allows students to immediately see the financial statements as of a specific date. Each of the following questions begins with an unadjusted trial balance. Using transactions from the following assignment, prepare the necessary adjustments and determine the impact each adjust- ment has on net income. The financial statements are automatically populated.

GL 3-1 Based on the FastForward illustration in this chapter

Using transactions from the following assignments, prepare the necessary adjustments, create the finan- cial statements, and determine the impact each adjustment has on net income.

GL 3-2 Based on Problem 3-3A

GL 3-3 Extension of Problem 2-1A

GL 3-4 Extension of Problem 2-2A

GL 3-5 Based on Serial Problem SP 3

GENERAL LEDGER PROBLEM

GL

COMPANY ANALYSIS A1 P7

Accounting Analysis

AA 3-1 Use Apple’s financial statements in Appendix A to answer the following. 1. Compute Apple’s profit margin for fiscal years ended (a) September 30, 2017, and (b) September 24, 2016. 2. Is the change in Apple’s profit margin favorable or unfavorable? 3. In 2017, did Apple’s profit margin outperform or underperform the industry (assumed) average of 12%? 4. For the fiscal year ended September 30, 2017, what is the balance of its Income Summary account

before it is closed? APPLE

140 Chapter 3 Adjusting Accounts for Financial Statements

GLOBAL ANALYSIS A1

APPLE Samsung

GOOGLE

AA 3-3 Key comparative figures for Samsung, Apple, and Google follow.

Required

1. Compute profit margin for Samsung, Apple, and Google. 2. Which company has the highest profit margin?

In millions Samsung Apple Google

Net income . . . . . . . . . . . W 42,186,747 $ 48,351 $ 12,662

Net sales . . . . . . . . . . . . 239,575,376 229,234 110,855

Required

1. Compute profit margins for (a) Apple and (b) Google for the two years of data reported above. 2. In the current year, which company is more successful on the basis of profit margin? 3. Compute current ratios for (a) Apple and (b) Google for the two years reported above. 4. In the current year, which company has the better ability to pay short-term obligations according to the

current ratio?

AA 3-2 Key figures for the recent two years of both Apple and Google follow.

Apple Google

$ millions Current Year Prior Year Current Year Prior Year

Net income . . . . . . . . . . . . . . . . . . $ 48,351 $ 45,687 $ 12,662 $ 19,478

Net sales . . . . . . . . . . . . . . . . . . . 229,234 215,639 110,855 90,272

Current assets . . . . . . . . . . . . . . . 128,645 106,869 124,308 105,408

Current liabilities . . . . . . . . . . . . . 100,814 79,006 24,183 16,756

COMPARATIVE ANALYSIS A1 A2

APPLE GOOGLE

ETHICS CHALLENGE P4 P6

BTN 3-1 On January 20, 2019, Tamira Nelson, the accountant for Picton Enterprises, is feeling pressure to complete the annual financial statements. The company president has said he needs up-to-date financial statements to share with the bank on January 21 at a dinner meeting that has been called to discuss Picton’s obtaining loan financing for a special building project. Tamira knows that she will not be able to gather all the needed information in the next 24 hours to prepare the entire set of adjusting entries. Those entries must be posted before the financial statements accurately portray the company’s performance and financial position for the fiscal period ended December 31, 2018. Tamira ultimately decides to estimate several expense accruals at the last minute. When deciding on estimates for the expenses, she uses low estimates because she does not want to make the financial statements look worse than they are. Tamira finishes the financial statements before the deadline and gives them to the president without mentioning that several account balances are estimates that she provided.

Required

1. Identify several courses of action that Tamira could have taken instead of the one she took. 2. If you were in Tamira’s situation, what would you have done? Briefly justify your response.

Beyond the Numbers

BTN 3-2 One of your classmates states that a company’s books should be ongoing and therefore not closed until that business is terminated. Write a half-page memo to this classmate explaining the concept of the closing process by drawing analogies between (1) a scoreboard for an athletic event and the revenue and expense accounts of a business or (2) a sports team’s record book and the retained earnings account. Hint: Think about what would happen if the scoreboard were not cleared before the start of a new game.

COMMUNICATING IN PRACTICE P7 P8

Chapter 3 Adjusting Accounts for Financial Statements 141

BTN 3-4 Four types of adjustments are described in the chapter: (1) prepaid expenses, (2) unearned revenues, (3) accrued expenses, and (4) accrued revenues.

Required

1. Form learning teams of four (or more) members. Each team member must select one of the four adjustments as an area of expertise (each team must have at least one expert in each area).

2. Form expert teams from the individuals who have selected the same area of expertise. Expert teams are to discuss and write a report that each expert will present to his or her learning team addressing the following: a. Description of the adjustment and why it’s necessary. b. Example of a transaction or event, with dates and amounts, that requires adjustment. c. Adjusting entry(ies) for the example in requirement b. d. Status of the affected account(s) before and after the adjustment in requirement c. e. Effects on financial statements of not making the adjustment.

3. Each expert should return to his or her learning team. In rotation, each member should present his or her expert team’s report to the learning team. Team discussion is encouraged.

TEAMWORK IN ACTION P1 P2 P3 P4

BTN 3-3 Access EDGAR online (SEC.gov) and locate the 10-K report of The Gap, Inc. (ticker: GPS), filed on March 20, 2017. Review its financial statements reported for the year ended January 28, 2017, to answer the following questions.

Required

1. What are Gap’s main brands? 2. When is Gap’s fiscal year-end? 3. What is Gap’s net sales for the period ended January 28, 2017? 4. What is Gap’s net income for the period ended January 28, 2017? 5. Compute Gap’s profit margin for the year ended January 28, 2017. 6. Do you believe Gap’s decision to use a year-end of late January or early February relates to its natural

business year? Explain.

TAKING IT TO THE NET A1

Design elements: Lightbulb: ©Chuhail/Getty Images; Blue globe: ©nidwlw/Getty Images and ©Dizzle52/Getty Images; Chess piece: ©Andrei Simonenko/Getty Images and ©Dizzle52/Getty Images; Mouse: ©Siede Preis/Getty Images; Global View globe: ©McGraw-Hill Education and ©Dizzle52/Getty Images; Sustainability: ©McGraw-Hill Education and ©Dizzle52/Getty Images

BTN 3-5 Review this chapter’s opening feature involving Evan and Bobby and Snapchat. 1. Explain how a classified balance sheet can help Evan and Bobby know what bills are due when and

whether they have the resources to pay those bills. 2. Why is it important for Evan and Bobby to match costs and revenues in a specific time period? How

do closing entries help them in this regard? 3. What objectives are met when Evan and Bobby apply closing procedures each fiscal year-end?

ENTREPRENEURIAL DECISION C3 P7

BTN 3-6 Select a company that you can visit in person or interview on the telephone. Call ahead to the company to arrange a time when you can interview an employee (preferably an accountant) who helps prepare the annual financial statements. Inquire about the following aspects of its accounting cycle: 1. Does the company prepare interim financial statements? What time period(s) is used for interim

statements? 2. Does the company use the cash or accrual basis of accounting? 3. Does the company use a work sheet in preparing financial statements? Why or why not? 4. Does the company use a spreadsheet program? If so, which software program is used? 5. How long does it take after the end of its reporting period to complete annual statements?

HITTING THE ROAD C1 C2

Learning Objectives

CONCEPTUAL C1 Describe merchandising activities and

identify income components for a merchandising company.

C2 Identify and explain the inventory asset and cost flows of a merchandising company.

ANALYTICAL A1 Compute the acid-test ratio and explain

its use to assess liquidity.

A2 Compute the gross margin ratio and explain its use to assess profitability.

P5 Appendix 4A—Record and compare merchandising transactions using both periodic and perpetual inventory systems.

P6 Appendix 4B—Prepare adjustments for discounts, returns, and allowances per revenue recognition rules.

P7 Appendix 4C—Record and compare merchandising transactions using the gross method and net method.

PROCEDURAL P1 Analyze and record transactions for

merchandise purchases using a perpetual system.

P2 Analyze and record transactions for merchandise sales using a perpetual system.

P3 Prepare adjustments and close accounts for a merchandising company.

P4 Define and prepare multiple-step and single-step income statements.

Chapter Preview

4 Accounting for Merchandising Operations

MERCHANDISING PURCHASES

P1 Accounting for: Purchases discounts

Purchases returns and allowances

Transportation costs

MERCHANDISING SALES

P2 Accounting for: Sales of merchandise

Sales discounts

Sales returns and allowances

MERCHANDISING ACTIVITIES

C1 Income and inventory for merchandisers

C2 Operating cycle Inventory cost flows

NTK 4-1 NTK 4-2 NTK 4-3

MERCHANDISER REPORTING

P3 Adjusting and closing P4 Multiple-step and

single-step income statements

A1 Acid-test analysis A2 Gross margin analysis

NTK 4-4, 4-5

143

“Understand what matters”—Maxine Clark

Bear Up

ST. LOUIS—“When I graduated from college,” explains Maxine Clark, “I felt the retail world had lost its spark. I wanted to be more creative.” Maxine was determined to start a business that would be different. Then she went shopping with the young daughter of a friend. “When we couldn’t find anything new, Katie picked up a Beanie Baby and said we could make one,” recalls Maxine. “Her words gave me the idea to create a company that would allow people to create their own customized stuffed animals.” Build-A-Bear Workshop (BuildaBear.com) was born!

“I did some research and began putting together a plan,” says Maxine. The Build-A-Bear Workshops were an instant success.

As her company grew, Maxine says accounting data on her merchandising operations fell short. “We can’t give up!” was her view. In response, Maxine set up an accounting system to mea- sure, track, summarize, and report on merchandising transac- tions, especially purchases.

Maxine computerized the accounting system, prepared monthly financial statements per store, developed annual bud- gets, and tracked all bank accounts and payables.

Build-A-Bear’s successful use of accounting data has made Maxine a self-made woman. She insists, however, it is not about

the financial rewards. “We’re a family business,” explains Maxine. “It’s important to set an example for children by being a company that does good things and cares about the well-being of others.”

Sources: Build-A-Bear website, January 2019; Fortune, March 2012; LEADERS, April 2011; CSRwire, August 2008

©Monty Brinton/CBS/Getty Images

Previous chapters covered accounting for service companies. A merchandising company’s activities differ from those of a service company. Merchandise refers to products, also called goods, that a company buys to resell. A merchandiser earns net income by buying and selling merchandise. Merchandisers are wholesalers or retailers. A wholesaler buys products from manufacturers and sells them to retailers. A retailer buys products from manufacturers or wholesalers and sells them to consumers.

Reporting Income for a Merchandiser Net income for a merchandiser equals revenues from selling merchandise minus both the cost of merchandise sold and other expenses—see Exhibit 4.1. Revenue from selling merchandise is called sales, and the expense of buying and preparing merchandise is called cost of goods sold. (Some service companies use the term sales instead of revenues; cost of goods sold is also called cost of sales.)

MERCHANDISING ACTIVITIES C1 Describe merchandising activities and identify income components for a merchandising company.

EqualsMinusEqualsMinus Expenses

Net income

Net sales

Merchandiser

Expenses Net

income Revenues

Service Company

Minus Equals

Gross profit

Cost of goods sold

EXHIBIT 4.1 Computing Income for a Merchandising Company versus a Service Company

Point: SuperValu and SYSCO are wholesalers. Target and Walmart are retailers.

144 Chapter 4 Accounting for Merchandising Operations

EXHIBIT 4.2 Income Statement for a Service Company and a Merchandising Company

LIBERTY TAX Income Statement ($ millions)

Revenues . . . . . . . . . . . . . . . . . . . . . . . . . . . . . . $174

Expenses . . . . . . . . . . . . . . . . . . . . . . . . . . . . . . 162

Net income . . . . . . . . . . . . . . . . . . . . . . . . . . . . $ 12

Service Company Merchandising Company

NORDSTROM INC. Income Statement ($ millions)

Net sales . . . . . . . . . . . . . . . . . . . . . . . . . . . $14,757

Cost of goods sold . . . . . . . . . . . . . . . . . . 9,440 Gross profit . . . . . . . . . . . . . . . . . . . . . . . . 5,317 Expenses . . . . . . . . . . . . . . . . . . . . . . . . . . . 4,963

Net income . . . . . . . . . . . . . . . . . . . . . . . . . $ 354

Reporting Inventory for a Merchandiser A merchandiser’s balance sheet has a current asset called merchandise inventory, an item not on a service company’s balance sheet. Merchandise inventory, or simply inventory, refers to products that a company owns and intends to sell. Inventory cost includes the cost to buy the goods, ship them to the store, and make them ready for sale.

Operating Cycle for a Merchandiser Exhibit 4.3 shows an operating cycle for a merchandiser with credit sales. The cycle moves from (a) cash purchases of merchandise to (b) inventory for sale to (c) credit sales to (d) accounts receivable to (e) receipt of cash. The length of an operating cycle differs across the types of businesses. Department stores often have operating cycles of two to five months. Operating cycles for grocery stores are usually from two to eight weeks. Companies try to keep their operating cycles short because assets tied up in inventory and receivables are not productive. Cash sales shorten operating cycles.

Inventory Systems Exhibit 4.4 shows that a company’s merchandise available for sale consists of what it begins with (beginning inventory) and what it purchases (net purchases). The merchandise available for sale is either sold (cost of goods sold) or kept for future sales (ending inventory).

Companies account for inventory in one of two ways: perpetual system or periodic system. Perpetual inventory system updates account-

ing records for each purchase and each sale of inventory.

Periodic inventory system updates account- ing records for purchases and sales of inven- tory only at the end of a period.

Technology has dramatically increased the use of the perpetual system. It gives managers immediate access to information on sales and inventory levels, which allows them to strategi- cally react and increase profit. (Some compa-

nies use a hybrid system where the perpetual system is used for tracking units available and the periodic system is used to compute cost of sales.)

C2 Identify and explain the inventory asset and cost flows of a merchandising company.

Cash

(b) Merchandise inventory

(d) Accounts receivable

(a) Purchases

(e ) C

as h

co lle

ctio n

(c) Credit sale s

54 6

1

W9797 Cherry Rd. Antigo, WI 54409

See reverse for terms of sale and returns.

Net of Discount $490

Invoice Date Number

11/2/17 4657-2

P.O. Date Salesperson Terms Freight Ship

Subtotal 500

500

Shipping Tax

Total

7

6 Freight terms Goods7 Total invoice amount8 Net amount9

1 32 54Seller Invoice date Purchaser Order date Credit termsKey:

INVOICE

CH015 SD099

Toddler–Challenger X7 Boys/Girls–Speed Demon

1 1

150 350

150 350

10/30/17 #141 2/10, n/30 FOB Destination Via FedEx

Model No. Description Quantity Price Amount

8

SOLD TO

9

Z-Mart

Tom Novak, Purchasing Agent

10 Michigan Street

Chicago

Illinois Zip

3 Firm Name

Attention of

Address

City

State 605212

EXHIBIT 4.3 Merchandiser’s Operating Cycle

Merchandise Inventory

Beg. inventory # Net purchases #

Merchandise avail. for sale # COGS #

End. inventory #

Beginning inventory

Net purchases

= Merchandise available for sale

Cost of goods sold

Ending inventory

+

+

EXHIBIT 4.4 Merchandiser’s Cost Flow for a Single Time Period

Point: Merchandise avail. for sale: MAS = EI + COGS,

which can be rewritten as MAS − EI = COGS, or MAS − COGS = EI.

The income statements for a service company, Liberty Tax, and for a merchandiser, Nordstrom, are in Exhibit 4.2. We see that the merchandiser, Nordstrom, reports cost of goods sold, which is not reported by the service company. The merchandiser also reports gross profit, or gross margin, which is net sales minus cost of goods sold.

Chapter 4 Accounting for Merchandising Operations 145

Use the following information (in random order) from a merchandising company and from a service com- pany to complete the requirements. Hint: Not all information may be necessary for the solutions.

C1 C2

Merchandise Accounts and Computations

NEED-TO-KNOW 4-1

1. For the merchandiser only, compute (a) goods available for sale, (b) cost of goods sold, and (c) gross profit. 2. Compute net income for each company.

SaveCo Merchandiser

Supplies . . . . . . . . . . . . . $ 10 Beginning inventory . . . 100 Ending inventory . . . . . . 50

Expenses . . . . . . . $ 20 Net purchases . . . 80 Net sales . . . . . . . 190

Hi-Tech Services

Expenses . . . . . $170 Revenues . . . . . 200 Cash . . . . . . . . . 10

Prepaid rent . . . . . . . . $25 Accounts payable . . . 35 Supplies . . . . . . . . . . . 65

Solution

1. a. Computation of goods available for sale (SaveCo).

b. Computation of cost of goods sold (SaveCo).

c. Computation of gross profit (SaveCo).

2. Computation of net income for each company.

SaveCo Merchandiser

Net sales . . . . . . . . . . . . . . . . . . . . . . . . . . . . . . . . . . $190 Less: Cost of goods sold (from part 1b) . . . . . . . . . . 130 Gross profit . . . . . . . . . . . . . . . . . . . . . . . . . . . . . . . . 60 Less: Expenses . . . . . . . . . . . . . . . . . . . . . . . . . . . . . 20 Net income . . . . . . . . . . . . . . . . . . . . . . . . . . . . . . . . $ 40

Hi-Tech Services

Revenues . . . . . . . . . . . . . . . . . . . . . . $200

Less: Expenses . . . . . . . . . . . . . . . . . 170 Net income . . . . . . . . . . . . . . . . . . . . $ 30 Do More: QS 4-3, E 4-1, E 4-2

Beginning inventory . . . . . . . . . . $100 Plus: Net purchases . . . . . . . . . . 80 Goods available for sale . . . . . . $180

Beginning inventory . . . . . . . . . . $100 Plus: Net purchases . . . . . . . . . . 80 Goods available for sale . . . . . . 180 Less: Ending inventory . . . . . . . . 50 Cost of goods sold . . . . . . . . . . . $130

Net sales . . . . . . . . . . . . . . . . . . $190 Less: Cost of goods sold (from part b) . . . . . . . . . 130 Gross profit . . . . . . . . . . . . . . . . $ 60

This section explains how we record purchases under different purchase terms.

Purchases without Cash Discounts Z-Mart records a $500 cash purchase of merchandise on November 2 as follows.

ACCOUNTING FOR MERCHANDISE PURCHASES P1 Analyze and record transactions for merchandise purchases using a perpetual system.

Assets = Liabilities + Equity +500 −500

Nov . 2 Merchandise Inventory . . . . . . . . . . . . . . . . . . . . . . . . . . . . . . . 500

Cash . . . . . . . . . . . . . . . . . . . . . . . . . . . . . . . . . . . . . . . . . . 500

Purchased goods for cash.

If these goods are instead purchased on credit, and no discounts are offered for early payment, Z-Mart makes the same entry except that Accounts Payable is credited instead of Cash.

Point: Costs recorded in Merchandise Inventory are called inventoriable costs.

Trade Discounts When a manufacturer or wholesaler prepares a catalog of items for sale, each item has a list price, or catalog price. However, an item’s selling price equals list price minus a percent called a trade discount. A wholesaler buying in large quantities gets a larger discount than a retailer buying in small quantities. A buyer re- cords the net amount of list price minus trade discount. If a supplier of Z-Mart lists an item at $625 and gives Z-Mart a 20% trade discount, Z-Mart’s purchase price is $500, computed as $625 − (20% × $625). ■

Decision Insight

Point: Trade discounts are not journalized; purchases are recorded based on the invoice amount.

Purchases with Cash Discounts The purchase of goods on credit requires credit terms. Credit terms include the amounts and timing of payments from a buyer to a seller. To demonstrate, when sellers require payment within 10 days after the end of the month (EOM) of the invoice date, credit terms are “n∕10

146 Chapter 4 Accounting for Merchandising Operations

EOM.” When sellers require payment within 30 days after the invoice date, credit terms are “n∕30,” meaning net 30 days.

Credit Terms Exhibit 4.5 explains credit terms. The amount of time allowed before full pay- ment is due is the credit period. Sellers can grant a cash discount to encourage buyers to pay earlier. A buyer views a cash discount as a purchases discount. A seller views a cash discount as a sales discount. Any cash discounts are described on the invoice. For example, credit terms of “2∕10, n∕60” mean that full payment is due within a 60-day credit period, but the buyer can deduct 2% of the invoice amount if payment is made within 10 days of the invoice date. This reduced payment is only for the discount period.

Amount Due

Due: Invoice priceDue: Invoice price minus discount

Discount period

Credit period

Invoice date

Credit Terms

Time

EXHIBIT 4.5 Credit Terms

Invoice On November 2, Z-Mart purchases $500 of merchandise on credit with terms of 2∕10, n∕30. The invoice for this purchase is shown in Exhibit 4.6. This is a purchase invoice for Z-Mart (buyer) and a sales invoice for Trex (seller). The amount recorded for merchandise in- ventory includes its purchase cost, shipping fees, taxes, and any other costs necessary to make it ready for sale.

54 6

1

W9797 Cherry Rd. Antigo, WI 54409

See reverse for terms of sale and returns.

Net of Discount $490

Invoice Date Number

11/2/18 4657-2 2

P.O. Date Salesperson Terms Freight Ship

Subtotal 500

500

Shipping Tax

Total

7

INVOICE

CH015 SD099

Toddler–Challenger X7 Boys/Girls–Speed Demon

1 1

150 350

150 350

10/30/18 #141 2/10, n/30 FOB Destination Via FedEx

Model No. Description Quantity Price Amount

8

Firm Name SOLD TO

Attention of

Address

City

State Zip

Tom Novak, Purchasing Agent

10 Michigan Street

Z-Mart

Chicago

Illinois 60521

3

6 Freight terms

Goods7

Total invoice amount8

Net amount9

1 Seller

2 Invoice date

3 Purchaser

5

4 Order date

Credit terms

Key:

9

EXHIBIT 4.6 Invoice

Point: The invoice date sets the discount and credit periods.

Gross Method Z-Mart purchases $500 of merchandise on credit terms of 2∕10, n∕30. The November 2 invoice offers a 2% discount if paid within 10 days; if not, Z-Mart must pay the full amount within 30 days. The buyer has two options. Pay within discount period (Nov. 2 through Nov. 12): Due = $490.

or Pay after discount period (Nov. 13 through Dec. 2): Due = $500.

The $490 equals the $500 invoice minus $10 discount (computed as $500 × 2%).

Chapter 4 Accounting for Merchandising Operations 147

On the purchase date, we do not know if payment will occur within the discount period. The gross method records the purchase at its gross (full) invoice amount. For Z-Mart, the purchase of $500 of merchandise with terms of 2∕10, n∕30 is recorded at $500. The gross method is used here because it is (1) used more in practice, (2) easier to apply, and (3) less costly.

Purchases on Credit Z-Mart’s entry to record the November 2 purchase of $500 of merchandise on credit follows. (For recording, it can help to add the name to the payable, such as Accounts Payable—Trex.)

Payment within Discount Period Good cash management means that invoices are not paid until the last day of the discount or credit period. This is because the buyer can use that money until payment is required. If Z-Mart pays the amount due on (or before) November 12, the entry is

Nov . 2 500

Bal . 490

Nov . 12 10

Merchandise Inventory

Nov . 12 490

Cash

Nov . 12 500 Nov . 2 500

Bal . 0

Accounts Payable

The Merchandise Inventory account equals the $490 net cost of purchases after these entries, and the Accounts Payable account has a zero balance.

Payment after Discount Period If the invoice is paid after November 12, the dis- count is lost. If Z-Mart pays the gross (full) amount due on December 2 (the n∕30 due date), the entry is

Point: Appendix 4A repeats jour- nal entries a through g using the periodic system.

Purchases with Returns and Allowances Purchases returns are merchandise a buyer purchases but then returns. Purchases allowances refer to a seller granting a price reduction (allowance) to a buyer of defective or unacceptable merchandise.

Purchases Allowances On November 5, Z-Mart (buyer) agrees to a $30 allowance from Trex for defective merchandise (assume allowance is $30 whether paid within the discount period or not). Z-Mart’s entry to update Merchandise Inventory and record the allowance follows. Z-Mart’s allowance for defective merchandise reduces its account payable to the seller. If cash is refunded, Cash is debited instead of Accounts Payable.

Point: When a buyer returns or takes an allowance on merchan- dise, the buyer issues a debit memorandum. This informs the seller of a debit made to the seller’s account payable in the buyer’s records.

(a) Nov . 2 Merchandise Inventory . . . . . . . . . . . . . . . . . . . . . . . . . . . . . . . 500 Accounts Payable . . . . . . . . . . . . . . . . . . . . . . . . . . . . . . . 500

Purchased goods, terms 2∕10, n∕30.

Assets = Liabilities + Equity +500 +500

(b1) Nov . 12 Accounts Payable . . . . . . . . . . . . . . . . . . . . . . . . . . . . . . . . . . . . 500 Merchandise Inventory . . . . . . . . . . . . . . . . . . . . . . . . . . . 10

Cash* . . . . . . . . . . . . . . . . . . . . . . . . . . . . . . . . . . . . . . . . . 490

Paid for goods within discount period. *$500 × (100% − 2%)

Assets = Liabilities + Equity −490 −500 − 10

(b2) Dec . 2 Accounts Payable . . . . . . . . . . . . . . . . . . . . . . . . . . . . . . . . . . . . 500 Cash . . . . . . . . . . . . . . . . . . . . . . . . . . . . . . . . . . . . . . . . . . 500

Paid for goods outside discount period.

Assets = Liabilities + Equity −500 −500

(c1) Nov . 5 Accounts Payable . . . . . . . . . . . . . . . . . . . . . . . . . . . . . . . . . . . . 30 Merchandise Inventory . . . . . . . . . . . . . . . . . . . . . . . . . . . 30

Allowance for defective goods.

Assets = Liabilities + Equity −30 −30

Purchases Returns Returns of inventory are recorded at the amount charged for that inventory. On June 1, Z-Mart purchases $250 of merchandise with terms 2∕10, n∕60—see en- tries below. On June 3, Z-Mart returns $50 of those goods. When Z-Mart pays on June 11, it

148 Chapter 4 Accounting for Merchandising Operations

takes the 2% discount only on the $200 remaining balance ($250 − $50). When goods are re- turned, a buyer takes a discount on only the remaining balance. This means the discount is $4 (computed as $200 × 2%) and the cash payment is $196 (computed as $200 − $4).

Point: Credit terms apply to both partial and full payments.

Assets = Liabilities + Equity +250 +250

Assets = Liabilities + Equity −50 −50

Assets = Liabilities + Equity −196 −200 − 4

     June 1 Merchandise Inventory . . . . . . . . . . . . . . . . . . . . . . . . . . . . . . . 250

Accounts Payable . . . . . . . . . . . . . . . . . . . . . . . . . . . . . . . 250

Purchased goods, terms 2∕10, n∕60. (c2) June 3 Accounts Payable . . . . . . . . . . . . . . . . . . . . . . . . . . . . . . . . . . . . 50 Merchandise Inventory . . . . . . . . . . . . . . . . . . . . . . . . . . . 50

Returned goods to seller.

       June 11 Accounts Payable . . . . . . . . . . . . . . . . . . . . . . . . . . . . . . . . . . . . 200

Merchandise Inventory . . . . . . . . . . . . . . . . . . . . . . . . . . . 4

Cash . . . . . . . . . . . . . . . . . . . . . . . . . . . . . . . . . . . . . . . . . . 196

Paid for $200 of goods less $4 discount.

These T-accounts show the final $196 in inventory, the zero balance in Accounts Payable, and the $196 cash payment.

Example: If on June 20, Z-Mart returns all goods paid for on June 11, the entry is Cash . . . . . . . . . . . . . . . . . . . . 196 Merchandise Inventory . . . 196

Jun . 1 250

Bal . 196

Jun . 3 50 Jun . 11 4

Merchandise Inventory

Jun . 3 50 Jun . 11 200

Jun . 1 250

Bal . 0

Accounts Payable

Jun . 11 196

Cash

Purchases and Transportation Costs The buyer and seller must agree on who is responsible for paying freight (shipping) costs and who has the risk of loss during transit. This is the same as asking at what point ownership trans- fers from the seller to the buyer. The point of transfer is called the FOB (free on board) point.

Exhibit 4.7 covers two alternative points of transfer.

1. FOB shipping point means the buyer accepts ownership when the goods depart the seller’s place of business. The buyer pays shipping costs and has the risk of loss in transit. The goods are part of the buyer’s inventory when they are in transit because ownership has transferred to the buyer. 1-800-Flowers.com, a floral merchandiser, uses FOB shipping point.

2. FOB destination means ownership of goods transfers to the buyer when the goods arrive at the buyer’s place of business. The seller pays shipping charges and has the risk of loss in transit. The seller does not record revenue until the goods arrive at the destination.

©Michael DeYoung/Blend Images

Point: When the party not respon- sible for shipping pays shipping cost, it either bills the other party responsible or adjusts its account payable or account receivable with the other party. Freight pay- ments are not applied in comput- ing discounts.

Destination

Ownership Transfers at

Goods in Transit Owned by

FOB shipping point Shipping point

Transportation Costs Paid byShipping Terms

Goods in transit Shipping point

Seller Buyer

FOB destination Destination

Buyer

Seller

Buyer Merchandise Inventory . . . # Cash . . . . . . . . . . . . . . . #

Seller Delivery Expense . . . . . . . . # Cash . . . . . . . . . . . . . . . . #

EXHIBIT 4.7 Ownership Transfer and Transportation Costs

What’s Your Policy? Return policies are a competitive advantage for businesses. REI offers a 1-year return policy on nearly every product it sells. Amazon picks up returned items at your door. On the other hand, some stores like Best Buy allow only 14 days to return products. ■

Decision Insight

Chapter 4 Accounting for Merchandising Operations 149

When a buyer is responsible for paying transportation costs, the payment is made to a carrier or directly to the seller. The cost principle requires that transportation costs of a buyer (often called transportation-in or freight-in) be part of the cost of merchandise inventory. Z-Mart’s entry to record a $75 freight charge from UPS for merchandise purchased FOB shipping point is

Point: If we place an order online and receive free shipping, we have terms FOB destination.

(d) Nov . 24 Merchandise Inventory . . . . . . . . . . . . . . . . . . . . . . . . . . . . . . . 75 Cash . . . . . . . . . . . . . . . . . . . . . . . . . . . . . . . . . . . . . . . . . . 75

Paid freight costs on goods.

Assets = Liabilities + Equity +75 −75

When a seller is responsible for paying shipping costs, it records these costs in a Delivery Expense account. Delivery expense, also called transportation-out or freight-out, is reported as a selling expense in the seller’s income statement.

Itemized Costs of Purchases In summary, purchases are recorded as debits to Merchandise Inventory (or Inventory). Purchases discounts, returns, and allowances are credited to (subtracted from) Merchandise Inventory. Transportation-in is debited (added) to Merchandise Inventory. Z-Mart’s itemized costs of merchandise purchases for the year are in Exhibit 4.8.

The accounting system described here does not provide separate records (accounts) for total purchases, total pur- chases discounts, total purchases returns and allowances, and total transportation-in. Many companies collect this information in supple- mentary records to evaluate these costs. Supplementary records, or supplemental records, refer to information outside the usual ledger accounts.

Point: INcoming freight costs are charged to INventory. When inventory EXits, freight costs are charged to EXpense.

Itemized Costs of Merchandise Purchases

Invoice cost of merchandise purchases . . . . . . . . . $ 235,800

Less: Purchases discounts received . . . . . . . . . . . . (4,200)

Purchases returns and allowances . . . . . . . . . (1,500)

Add: Costs of transportation-in . . . . . . . . . . . . . . . .       2,300

Total net cost of merchandise purchases . . . . . . $232,400

EXHIBIT 4.8 Itemized Costs of Merchandise Purchases

Point: Some companies have separate accounts for purchases discounts, returns and allowances, and transportation-in. These accounts are then transferred to Merchandise Inventory at period- end. This is a hybrid system of perpetual and periodic. That is, Merchandise Inventory is updated on a perpetual basis but only for purchases and cost of goods sold.

Payables Manager As a new accounts payable manager, you are being trained by the outgoing manager. She explains that the system prepares checks for amounts net of favorable cash discounts, and the checks are dated the last day of the discount period. She tells you that checks are not mailed until five days later, adding that “the company gets free use of cash for an extra five days, and our department looks better.” Do you continue this policy? ■ Answer: One point of view is that the late payment policy is unethical. A deliberate plan to make late payments means the company lies when it pretends to make payment within the discount period. Another view is that the late payment policy is acceptable. Some believe attempts to take discounts through late payments are accepted as “price negotiation.”

Decision Ethics

Prepare journal entries to record each of the following purchases transactions of a merchandising com- pany. Assume a perpetual inventory system using the gross method for recording purchases.

Oct. 1 Purchased $1,000 of goods. Terms of the sale are 4∕10, n∕30, and FOB shipping point; the in- voice is dated October 1.

3 Paid $30 cash for freight charges from UPS for the October 1 purchase. 7 Returned $50 of the $1,000 of goods from the October 1 purchase and received full credit. 11 Paid the amount due from the October 1 purchase (less the return on October 7). 31 Assume the October 11 payment was never made. Instead, payment of the amount due, less the

return on October 7, occurred on October 31.

Solution

P1 Merchandise Purchases

NEED-TO-KNOW 4-2

Oct . 1 Merchandise Inventory . . . . . . . . . . . . . . . . . . . . . . . . . . . . . . . 1,000

Accounts Payable . . . . . . . . . . . . . . . . . . . . . . . . . . . . . . . 1,000

Purchased goods, terms 4∕10, n∕30. Oct . 3 Merchandise Inventory . . . . . . . . . . . . . . . . . . . . . . . . . . . . . . . 30

Cash . . . . . . . . . . . . . . . . . . . . . . . . . . . . . . . . . . . . . . . . . . 30

Paid freight on purchases FOB shipping point.

[continued on next page]

150 Chapter 4 Accounting for Merchandising Operations

The perpetual accounting system requires that each sales transaction for a merchandiser, whether for cash or on credit, has two entries: one for revenue and one for cost.

1. Revenue received (and asset increased) from the customer. 2. Cost of goods sold incurred (and asset decreased) to the customer.

Sales without Cash Discounts Revenue Side: Inflow of Assets Z-Mart sold $1,000 of merchandise on credit terms n∕60 on November 12. The revenue part of this transaction is recorded as follows. This entry shows an increase in Z-Mart’s assets in the form of accounts receivable. It also shows the in- crease in revenue (Sales). If the sale is for cash, debit Cash instead of Accounts Receivable.

Oct . 7 Accounts Payable . . . . . . . . . . . . . . . . . . . . . . . . . . . . . . . . . . . . 50

Merchandise Inventory . . . . . . . . . . . . . . . . . . . . . . . . . . . 50

Returned goods.

Oct . 11 Accounts Payable . . . . . . . . . . . . . . . . . . . . . . . . . . . . . . . . . . . . 950

Merchandise Inventory* . . . . . . . . . . . . . . . . . . . . . . . . . . 38

Cash† . . . . . . . . . . . . . . . . . . . . . . . . . . . . . . . . . . . . . . . . . 912

Paid for goods within discount period. *$950 × 4% †$950 − ($950 × 4%)

Oct . 31 Accounts Payable‡ . . . . . . . . . . . . . . . . . . . . . . . . . . . . . . . . . . . 950

Cash . . . . . . . . . . . . . . . . . . . . . . . . . . . . . . . . . . . . . . . . . . 950

Paid for goods outside discount period. ‡$1,000 − $50

[continued from previous page]

Do More: QS 4-5, QS 4-6, QS 4-7, E 4-3, E 4-5

Merchandising companies must account for sales, sales discounts, sales returns and allowances, and cost of goods sold. Z-Mart has these items in its gross profit computation—see Exhibit 4.9. This shows that customers paid $314,700 for merchandise that cost Z-Mart $230,400, yielding a gross profit of $84,300.

ACCOUNTING FOR MERCHANDISE SALES P2 Analyze and record transac- tions for merchandise sales using a perpetual system.

EXHIBIT 4.9 Gross Profit Computation

Computation of Gross Profit

Net sales (net of discounts, returns, and allowances) . . . . . . . . . . . . $314,700

Cost of goods sold . . . . . . . . . . . . . . . . . . . . . . . . . . . . . . . . . . . . . . . . 230,400

Gross profit . . . . . . . . . . . . . . . . . . . . . . . . . . . . . . . . . . . . . . . . . . . . . $ 84,300

Cost Side: Outflow of Assets The cost side of each sale requires that Merchandise Inventory decrease by that item’s cost. The cost of the merchandise Z-Mart sold on November 12 is $300, and the entry to record the cost part of this transaction follows.

Point: Gross profit on Nov. 12 sale:

Net sales . . . . . . . . . . . . . $1,000 Cost of goods sold . . . . . . 300 Gross profit . . . . . . . . . . . $ 700

Assets = Liabilities + Equity +1,000 +1,000

Nov . 12 Accounts Receivable . . . . . . . . . . . . . . . . . . . . . . . . . . . . . . . . . 1,000

Sales . . . . . . . . . . . . . . . . . . . . . . . . . . . . . . . . . . . . . . . . . 1,000

Sold goods on credit.

Assets = Liabilities + Equity −300 −300

Nov . 12 Cost of Goods Sold . . . . . . . . . . . . . . . . . . . . . . . . . . . . . . . . . . 300

Merchandise Inventory . . . . . . . . . . . . . . . . . . . . . . . . . . . 300

Record cost of Nov. 12 sale.

Chapter 4 Accounting for Merchandising Operations 151

Sales with Cash Discounts Offering discounts on credit sales benefits a seller through earlier cash receipts and reduced col- lection efforts. We use the gross method, which records sales at the full amount and records sales discounts if, and when, they are taken. The gross method requires a period-end adjusting entry to estimate future sales discounts. (The net method records sales at the net amount, which assumes all discounts are taken. This method requires an adjusting entry to estimate future dis- counts lost. See Appendix 4C.)

Sales on Credit Z-Mart makes a credit sale for $1,000 on November 12 with terms of 2∕10, n∕45 (cost of the merchandise sold is $300). The entries to record this sale follow.

Future Demands Large merchandising companies, such as Amazon, bombard suppliers with demands. These in- clude discounts for bar coding and technology support systems and fines for shipping errors. Merchandisers’ goals are to reduce inventories, shorten lead times, and eliminate errors. Colleges offer programs in supply chain manage- ment and logistics to train future employees to help merchandisers meet such goals. ■

Decision Insight

©Polaris/Newscom

Assets = Liabilities + Equity +1,000 +1,000

Assets = Liabilities + Equity −300 −300

Nov . 12 Accounts Receivable . . . . . . . . . . . . . . . . . . . . . . . . . . . . . . . . . 1,000

Sales . . . . . . . . . . . . . . . . . . . . . . . . . . . . . . . . . . . . . . . . . 1,000

Sold goods, terms 2∕10, n∕45. Nov . 12 Cost of Goods Sold . . . . . . . . . . . . . . . . . . . . . . . . . . . . . . . . . . 300

Merchandise Inventory . . . . . . . . . . . . . . . . . . . . . . . . . . . 300

Record cost of Nov. 12 sale.

Buyer Pays within Discount Period One option is for the buyer to pay $980 within the 10-day discount period ending November 22. The $20 sales discount is computed as $1,000 × 2%. If the customer pays on (or before) November 22, Z-Mart records the cash receipt as follows. Sales Discounts is a contra revenue account, meaning the Sales Discounts account is subtracted from the Sales account when computing net sales. The Sales Discounts account has a normal debit balance because it is subtracted from Sales, which has a normal credit balance.

Point: Net sales is the amount received from the customer.

Sales . . . . . . . . . . . . . . . . . $1,000 Sales discounts . . . . . . . . (20 ) Net sales . . . . . . . . . . . . . $ 980

Buyer Pays after Discount Period The customer’s second option is to wait 45 days until December 27 (or at least until after the discount period) and then pay $1,000. Z-Mart records that cash receipt as

Sales with Returns and Allowances If a customer is unhappy with a purchase, many sellers allow the customer to either return the merchandise for a full refund (sales return) or keep the merchandise along with a partial refund (sales allowance). Most sellers can reliably estimate returns and allowances (abbreviated R&A).

Buyer Returns Goods—Revenue Side When a buyer returns goods, it impacts the seller’s revenue and cost sides. When a return occurs, the seller debits Sales Returns and

Assets = Liabilities + Equity + 980 −20 −1,000

Nov . 22 Cash* . . . . . . . . . . . . . . . . . . . . . . . . . . . . . . . . . . . . . . . . . . . . . 980

Sales Discounts . . . . . . . . . . . . . . . . . . . . . . . . . . . . . . . . . . . . . 20

Accounts Receivable . . . . . . . . . . . . . . . . . . . . . . . . . . . . . 1,000

Received payment on Nov. 12 sale less discount. *$1,000 − ($1,000 × 2%)

Assets = Liabilities + Equity +1,000 −1,000

Dec . 27 Cash . . . . . . . . . . . . . . . . . . . . . . . . . . . . . . . . . . . . . . . . . . . . . . 1,000

Accounts Receivable . . . . . . . . . . . . . . . . . . . . . . . . . . . . . 1,000

Received payment on Nov. 12 sale after discount period.

152 Chapter 4 Accounting for Merchandising Operations

Buyer Returns Goods—Cost Side When a return occurs, the seller must reduce the cost of sales. Continuing the example where the returned items sold for $15 and cost $9, the cost-side entry depends on whether the goods are defective.

Returned Goods Not Defective. If the merchandise returned is not defective and can be resold, there is a cost-side entry. The seller adds the cost of the returned goods back to inven- tory and reduces cost of goods sold as follows. This entry reverses the cost-side entry of November 12 for only $9 of goods returned.

(e1) Nov . 26 Sales Returns and Allowances . . . . . . . . . . . . . . . . . . . . . . . . . 15 Cash . . . . . . . . . . . . . . . . . . . . . . . . . . . . . . . . . . . . . . . . . . 15

Goods returned from Nov. 12 sale.

Assets = Liabilities + Equity −15 −15

Allowances, a contra revenue account to Sales. Assume that a customer returns merchandise on November 26 that sold for $15 and cost $9; the revenue-side returns entry is

Assets = Liabilities + Equity +9 +9

(e2) Nov . 26 Merchandise Inventory . . . . . . . . . . . . . . . . . . . . . . . . . . . . . . . 9 Cost of Goods Sold . . . . . . . . . . . . . . . . . . . . . . . . . . . . . . 9

Returned goods are added back to inventory.

If the seller has already collected cash for the sale, the seller could give the price reduction in cash. For example, instead of crediting the buyer’s Accounts Receivable in the entry above, the seller can credit Cash for $10.

Point: When a seller accepts returns or grants an allowance, the seller issues a credit memo- randum. This informs the buyer of a credit made to the buyer’s account in the seller’s records.

Prepare journal entries to record each of the following sales transactions of a merchandising company. Assume a perpetual inventory system and use of the gross method (beginning inventory equals $9,000).

June 1 Sold 50 units of merchandise to a customer for $150 per unit under credit terms of 2∕10, n∕30, FOB shipping point, and the invoice is dated June 1. The 50 units of merchandise had cost $100 per unit.

7 The customer returns 2 units purchased on June 1 because those units did not fit its needs. The seller restores those units to its inventory (as they are not defective) and credits Accounts Re- ceivable from the customer.

11 The seller receives the balance due from the June 1 sale to the customer less returns and allowances. 14 The customer discovers that 10 units have minor damage but keeps them because the seller

sends a $50 cash payment allowance to compensate.

Merchandise Sales

NEED-TO-KNOW 4-3

P2

Buyer Granted Allowances If a buyer is not satisfied with the goods, the seller might offer a price reduction for the buyer to keep the goods. There is no cost-side entry in this case as the inventory is not returned. On the revenue side, the seller debits Sales Returns and Allowances and credits Cash or Accounts Receivable depending on what’s agreed. Assume that $40 of mer- chandise previously sold is defective. The seller gives a price reduction and credits the buyer’s accounts receivable for $10. The seller records this allowance as follows.

Returned Goods Are Defective. If the merchandise returned is defective, the returned inventory is recorded at its estimated value, not its cost. The following entry assumes the returned goods costing $9 are defective and are worth $2.

Nov . 26 Merchandise Inventory . . . . . . . . . . . . . . . . . . . . . . . . . . . . . . . 2

Loss from Defective Merchandise . . . . . . . . . . . . . . . . . . . . . . . 7

Cost of Goods Sold . . . . . . . . . . . . . . . . . . . . . . . . . . . . . . 9

Returned defective goods to inventory and record loss.

Assets = Liabilities + Equity +2 −7 +9

(f ) Nov . 24 Sales Returns and Allowances . . . . . . . . . . . . . . . . . . . . . . . . . 10 Accounts Receivable . . . . . . . . . . . . . . . . . . . . . . . . . . . . . 10

Sales allowance granted.

Assets = Liabilities + Equity −10 −10

Chapter 4 Accounting for Merchandising Operations 153

June 1 Accounts Receivable . . . . . . . . . . . . . . . . . . . . . . . . . . . . . . . . . 7,500

Sales . . . . . . . . . . . . . . . . . . . . . . . . . . . . . . . . . . . . . . . . . 7,500

Sold goods. 50 units × $150 June 1 Cost of Goods Sold . . . . . . . . . . . . . . . . . . . . . . . . . . . . . . . . . . 5,000

Merchandise Inventory . . . . . . . . . . . . . . . . . . . . . . . . . . . 5,000

Cost of sale. 50 units × $100 June 7 Sales Returns and Allowances . . . . . . . . . . . . . . . . . . . . . . . . . 300

Accounts Receivable . . . . . . . . . . . . . . . . . . . . . . . . . . . . . 300

Returns accepted. 2 units × $150 June 7 Merchandise Inventory . . . . . . . . . . . . . . . . . . . . . . . . . . . . . . . 200

Cost of Goods Sold . . . . . . . . . . . . . . . . . . . . . . . . . . . . . . 200

Returns added to inventory. 2 units × $100 June 11 Cash . . . . . . . . . . . . . . . . . . . . . . . . . . . . . . . . . . . . . . . . . . . . . . 7,056

Sales Discounts* . . . . . . . . . . . . . . . . . . . . . . . . . . . . . . . . . . . . 144

Accounts Receivable . . . . . . . . . . . . . . . . . . . . . . . . . . . . 7,200

Received payment. *($7,500 − $300) × 2% June 14 Sales Returns and Allowances . . . . . . . . . . . . . . . . . . . . . . . . . 50

Cash . . . . . . . . . . . . . . . . . . . . . . . . . . . . . . . . . . . . . . . . . . 50

Recorded allowance on goods. Do More: QS 4-8, E 4-4,

E 4-6, E 4-7

Exhibit 4.10 shows the flow of merchandising costs during a period and where these costs are reported at period-end. Specifically, beginning inventory plus the net cost of purchases is the merchandise available for sale. As inventory is sold, its cost is recorded in cost of goods sold on the income statement; what remains is ending inventory on the balance sheet. A period’s ending inventory is the next period’s beginning inventory.

ADJUSTING AND CLOSING FOR MERCHANDISERS

Solution

Beginning inventory

From supplier

Net purchases

Ending inventory

Period 2Period 1

Cost of goods sold

Merchandise available for sale

To Income Statement

To Balance Sheet

Beginning inventory

From supplier

Net purchases

Ending inventory

Cost of goods sold

Merchandise available for sale

To Income Statement

To Balance Sheet

EXHIBIT 4.10 Merchandising Cost Flow in the Accounting Cycle

Adjusting Entries for Merchandisers Each of the steps in the accounting cycle described in the prior chapter applies to a merchan- diser. We expand upon three steps of the accounting cycle for a merchandiser—adjustments, statement preparation, and closing.

Inventory Shrinkage—Adjusting Entry A merchandiser using a perpetual inven- tory system makes an adjustment to Merchandise Inventory for any loss of merchandise, includ- ing theft and deterioration. Shrinkage is the loss of inventory, and it is computed by comparing a physical count of inventory with recorded amounts.

P3 Prepare adjustments and close accounts for a mer- chandising company.

154 Chapter 4 Accounting for Merchandising Operations

Z-Mart’s Merchandise Inventory account at the end of the year has a balance of $21,250, but a physical count shows only $21,000 of inventory exists. The adjusting entry to record this $250 shrinkage is

Dec . 31 Cost of Goods Sold . . . . . . . . . . . . . . . . . . . . . . . . . . . . . . . . . . 250

Merchandise Inventory . . . . . . . . . . . . . . . . . . . . . . . . . . . 250

Adjust for $250 shrinkage.

Assets = Liabilities + Equity −250 −250

Sales Discounts, Returns, and Allowances—Adjusting Entries Revenue recognition rules require sales to be reported at the amount expected to be received. This means that period-end adjusting entries are commonly made for Expected sales discounts. Expected returns and allowances (revenue side). Expected returns and allowances (cost side).

These three adjustments produce three new accounts: Allowance for Sales Discounts, Sales Refund Payable, and Inventory Returns Estimated. Appendix 4B covers these accounts and the adjusting entries.

Preparing Financial Statements The financial statements of a merchandiser are similar to those for a service company described in prior chapters. The income statement mainly differs by the addition of cost of goods sold and gross profit. Net sales is affected by discounts, returns and allowances, and some additional expenses such as delivery expense and loss from defective merchandise. The balance sheet dif- fers by the addition of merchandise inventory as part of current assets. (Appendix 4B explains inventory returns estimated as part of current assets and sales refund payable as part of current liabilities.) The statement of retained earnings is unchanged.

Closing Entries for Merchandisers Closing entries are similar for service companies and merchandising companies. The difference is that we close some new temporary accounts that come from merchandising activities. Z-Mart has temporary accounts unique to merchandisers: Sales (of goods), Sales Discounts, Sales Returns and Allowances, and Cost of Goods Sold. The third and fourth closing entries are identical for a mer- chandiser and a service company. The differences are in red in the closing entries of Exhibit 4.11.

EXHIBIT 4.11 Closing Entries for a Merchandiser

Step 1: Close Credit Balances in Temporary Accounts to Income Summary.

Step 2: Close Debit Balances in Temporary Accounts to Income Summary.

Dec . 31 Income Summary . . . . . . . . . . . . . . . . . . . . . . . . . . . . . . . . . . . . 308,100 Sales Discounts . . . . . . . . . . . . . . . . . . . . . . . . . . . . . . . . 4,300 Sales Returns and Allowances . . . . . . . . . . . . . . . . . . . 2,000 Cost of Goods Sold . . . . . . . . . . . . . . . . . . . . . . . . . . . . . 230,400 Depreciation Expense . . . . . . . . . . . . . . . . . . . . . . . . . . . . 3,700 Salaries Expense . . . . . . . . . . . . . . . . . . . . . . . . . . . . . . . 43,800 Insurance Expense . . . . . . . . . . . . . . . . . . . . . . . . . . . . . . 600 Rent Expense . . . . . . . . . . . . . . . . . . . . . . . . . . . . . . . . . . 9,000 Supplies Expense . . . . . . . . . . . . . . . . . . . . . . . . . . . . . . . 3,000 Advertising Expense . . . . . . . . . . . . . . . . . . . . . . . . . . . . 11,300 Close debit balances in temporary accounts.

Step 3: Close Income Summary.

Dec . 31 Income Summary . . . . . . 12,900 Retained Earnings . . 12,900

Step 4: Close Dividends.

Dec . 31 Retained Earnings . . . . . . . . . 4,000 Dividends . . . . . . . . . . . . 4,000

Dec . 31 Sales . . . . . . . . . . . . . . . . . . . . . . . . . . . . . . . . . . . . . . . . . . . . . . 321,000 Income Summary . . . . . . . . . . . . . . . . . . . . . . . . . . . . . . . 321,000 Close credit balances in temporary accounts.

Chapter 4 Accounting for Merchandising Operations 155

Sales, having a normal credit balance, is debited in step 1. Sales Discounts, Sales Returns and Allowances, and Cost of Goods Sold, having normal debit balances, are credited in step 2.

Summary of Merchandising Entries Exhibit 4.12 summarizes the adjusting and closing entries of a merchandiser (using a perpetual inventory system).

EXHIBIT 4.12 Summary of Key Merchandising Entries (using perpetual system and gross method)

Merchandising Transactions Merchandising Entries Dr. Cr.

Purchasing merchandise for Merchandise Inventory . . . . . . . . . . . . . . . . . . . . . # resale . Cash or Accounts Payable . . . . . . . . . . . . . . #

Paying freight costs on Merchandise Inventory . . . . . . . . . . . . . . . . . . . . . # purchases; FOB shipping point . Cash . . . . . . . . . . . . . . . . . . . . . . . . . . . . . . . #

Paying within discount period . Accounts Payable . . . . . . . . . . . . . . . . . . . . . . . . . # Purchases Merchandise Inventory . . . . . . . . . . . . . . . . . # Cash . . . . . . . . . . . . . . . . . . . . . . . . . . . . . . . #

Paying outside discount period . Accounts Payable . . . . . . . . . . . . . . . . . . . . . . . . . # Cash . . . . . . . . . . . . . . . . . . . . . . . . . . . . . . . #

Recording purchases returns or Cash or Accounts Payable . . . . . . . . . . . . . . . . . . . # allowances . Merchandise Inventory . . . . . . . . . . . . . . . . . #

Selling merchandise . Cash or Accounts Receivable . . . . . . . . . . . . . . . . # Sales . . . . . . . . . . . . . . . . . . . . . . . . . . . . . . . #

Cost of Goods Sold . . . . . . . . . . . . . . . . . . . . . . . . # Merchandise Inventory . . . . . . . . . . . . . . . . . #

Receiving payment within Cash . . . . . . . . . . . . . . . . . . . . . . . . . . . . . . . . . . . . #

discount period . Sales Discounts . . . . . . . . . . . . . . . . . . . . . . . . . . . # Accounts Receivable . . . . . . . . . . . . . . . . . . #

Sales Receiving payment outside Cash . . . . . . . . . . . . . . . . . . . . . . . . . . . . . . . . . . . . #

discount period . Accounts Receivable . . . . . . . . . . . . . . . . . . #

Receiving sales returns Sales Returns and Allowances . . . . . . . . . . . . . . . # of nondefective inventory . Cash or Accounts Receivable . . . . . . . . . . . #

Merchandise Inventory . . . . . . . . . . . . . . . . . . . . . # Cost of Goods Sold . . . . . . . . . . . . . . . . . . . #

Recognizing sales allowances . Sales Returns and Allowances . . . . . . . . . . . . . . . # Cash or Accounts Receivable . . . . . . . . . . . #

Paying freight costs on sales; Delivery Expense . . . . . . . . . . . . . . . . . . . . . . . . . . # FOB destination . Cash . . . . . . . . . . . . . . . . . . . . . . . . . . . . . . . #

Merchandising Events Adjusting and Closing Entries

Adjustment for shrinkage Cost of Goods Sold . . . . . . . . . . . . . . . . . . . . . . . . # (occurs when recorded amount Merchandise Inventory . . . . . . . . . . . . . . . . . # larger than physical inventory) .

Period-end adjustment for Sales Discounts . . . . . . . . . . . . . . . . . . . . . . . . . . . # Adjusting expected sales discounts .* Allowance for Sales Discounts . . . . . . . . . . . #

Period-end adjustment for expected Sales Returns and Allowances . . . . . . . . . . . . . . . # returns—both revenue side and Sales Refund Payable . . . . . . . . . . . . . . . . . . # cost side .* Inventory Returns Estimated . . . . . . . . . . . . . . . . . # Cost of Goods Sold . . . . . . . . . . . . . . . . . . . . #

Closing temporary accounts Sales . . . . . . . . . . . . . . . . . . . . . . . . . . . . . . . . . . . # with credit balances . Income Summary . . . . . . . . . . . . . . . . . . . . . #

Closing temporary accounts Income Summary . . . . . . . . . . . . . . . . . . . . . . . . . # Closing with debit balances . Sales Returns and Allowances . . . . . . . . . . . # Sales Discounts . . . . . . . . . . . . . . . . . . . . . . # Cost of Goods Sold . . . . . . . . . . . . . . . . . . . # Delivery Expense . . . . . . . . . . . . . . . . . . . . . # “Other Expenses” . . . . . . . . . . . . . . . . . . . . . #

Merchandise Inventory

Beginning inventory Purchases Pur. returns Freight-in (FOB shp pt) Pur. allowances Pur. discounts Shrinkage

Goods avail. for sale Customer returns COGS

Ending inventory

* Period-end adjustments depend on unadjusted balances, which can reverse the debit and credit in the adjusting entries shown; these three entries are covered in Appendix 4B.

156 Chapter 4 Accounting for Merchandising Operations

A merchandising company’s ledger on May 31, its fiscal year-end, includes the following accounts that have normal balances (it uses the perpetual inventory system). A physical count of its May 31 year-end inventory reveals that the cost of the merchandise inventory still available is $656. (a) Prepare the entry to record any inventory shrinkage. (b) Prepare the four closing entries as of May 31.

Recording Shrinkage and Closing Entries

NEED-TO-KNOW 4-4

P3

Solution

This section covers two income statement formats: multiple-step and single-step. The classified balance sheet of a merchandiser also is covered.

Multiple-Step Income Statement A multiple-step income statement details net sales and expenses and reports subtotals for various types of items. Exhibit 4.13 shows a multiple-step income statement. The statement has three main parts: (1) gross profit, which is net sales minus cost of goods sold; (2) income from operations, which is gross profit minus operating expenses; and (3) net income, which is income from operations plus or minus nonoperating items.

Operating expenses are separated into two sections. Selling expenses are the expenses of advertising merchandise, making sales, and delivering goods to customers. General and admin- istrative expenses support a company’s overall operations and include expenses related to accounting, human resources, and finance. Expenses are allocated between sections when they contribute to more than one. Z-Mart allocates rent expense of $9,000 from its store building between two sections: $8,100 to selling expense and $900 to general and administrative expenses.

Nonoperating activities consist of other expenses, revenues, losses, and gains that are un related to a company’s operations. Other revenues and gains commonly include interest revenue, divi- dend revenue, rent revenue, and gains from asset disposals. Other expenses and losses commonly include interest expense, losses from asset disposals, and casualty losses. When there are no reportable nonoperating activities, its income from operations is simply labeled net income.

MORE ON FINANCIAL STATEMENT FORMATS P4 Define and prepare multiple-step and single- step income statements.

Example: Sometimes interest rev- enue and interest expense are netted and reported on the in- come statement as Interest, net.

Do More: QS 4-9, QS 4-10, E 4-10, E 4-12, P 4-4

May 31 Cost of Goods Sold . . . . . . . . . . . . . . . . . . . . . . . . . . . . . . . . . . 100 Merchandise Inventory . . . . . . . . . . . . . . . . . . . . . . . . . . . 100 Adjust for shrinkage ($756 − $656).

a.

May 31 Sales . . . . . . . . . . . . . . . . . . . . . . . . . . . . . . . . . . . . . . . . . . . . . . 4,300 Income Summary . . . . . . . . . . . . . . . . . . . . . . . . . . . . . . . 4,300 Close temporary accounts with credit balances.

May 31 Income Summary . . . . . . . . . . . . . . . . . . . . . . . . . . . . . . . . . . . . 3,800 Sales Discounts . . . . . . . . . . . . . . . . . . . . . . . . . . . . . . . . . 50 Sales Returns and Allowances . . . . . . . . . . . . . . . . . . . . . 250 Cost of Goods Sold* . . . . . . . . . . . . . . . . . . . . . . . . . . . . . 2,200 Depreciation Expense . . . . . . . . . . . . . . . . . . . . . . . . . . . . 400 Salaries Expense . . . . . . . . . . . . . . . . . . . . . . . . . . . . . . . . 600 Other Operating Expenses . . . . . . . . . . . . . . . . . . . . . . . . 300 Close temporary accounts with debit balances.

*$2,100 (Unadj. bal.) + $100 (Shrinkage)

May 31 Income Summary . . . . . . . . . . . . . . . . . . . . . . . . . . . . . . . . . . . . 500 Retained Earnings . . . . . . . . . . . . . . . . . . . . . . . . . . . . . . . 500 Close Income Summary account.

May 31 Retained Earnings . . . . . . . . . . . . . . . . . . . . . . . . . . . . . . . . . . . 150 Dividends . . . . . . . . . . . . . . . . . . . . . . . . . . . . . . . . . . . . . 150 Close Dividends account.

b.

Merchandise inventory . . . . $ 756 Sales . . . . . . . . . . . . . . . . . . . . . $4,300 Depreciation expense . . . . . . . . . $400 Common stock . . . . . . . . . . . 1,000 Sales discounts . . . . . . . . . . . . . 50 Salaries expense . . . . . . . . . . . . . 600 Retained earnings . . . . . . . . 1,300 Other operating expenses . . . . 300 Sales returns and allowances . . . 250 Dividends . . . . . . . . . . . . . . . 150 Cost of goods sold . . . . . . . . . . 2,100

Chapter 4 Accounting for Merchandising Operations 157

Single-Step Income Statement A single-step income statement is shown in Exhibit 4.14. It lists cost of goods sold as another expense and shows only one subtotal for total expenses. Expenses are grouped into few, if any, categories. Many companies use formats that combine features of both single- and multiple-step statements. Management chooses the format that best informs users.

Z-MART Income Statement

For Year Ended December 31, 2019

Sales . . . . . . . . . . . . . . . . . . . . . . . . . . . . . . . . . . . . . . . . . . . . . . . . $321,000

Less: Sales discounts . . . . . . . . . . . . . . . . . . . . . . . . . . . . . . . . . . . $ 4,300

Sales returns and allowances . . . . . . . . . . . . . . . . . . . . . . . . 2,000 6,300

Net sales . . . . . . . . . . . . . . . . . . . . . . . . . . . . . . . . . . . . . . . . . . . 314,700

Cost of goods sold* . . . . . . . . . . . . . . . . . . . . . . . . . . . . . . . . . . . . 230,400

Gross profit . . . . . . . . . . . . . . . . . . . . . . . . . . . . . . . . . . . . . . . . . . 84,300 Operating expenses

Selling expenses

Depreciation expense—Store equipment . . . . . . . . . . . . . . 3,000

Sales salaries expense . . . . . . . . . . . . . . . . . . . . . . . . . . . . . 18,500

Rent expense—Selling space . . . . . . . . . . . . . . . . . . . . . . . . 8,100

Store supplies expense . . . . . . . . . . . . . . . . . . . . . . . . . . . . . 1,200

Advertising expense . . . . . . . . . . . . . . . . . . . . . . . . . . . . . . . 11,300

Total selling expenses . . . . . . . . . . . . . . . . . . . . . . . . . . . . . 42,100

General and administrative expenses

Depreciation expense—Office equipment . . . . . . . . . . . . . 700

Office salaries expense . . . . . . . . . . . . . . . . . . . . . . . . . . . . 25,300

Insurance expense . . . . . . . . . . . . . . . . . . . . . . . . . . . . . . . . 600

Rent expense—Office space . . . . . . . . . . . . . . . . . . . . . . . . 900

Office supplies expense . . . . . . . . . . . . . . . . . . . . . . . . . . . . 1,800

Total general and administrative expenses . . . . . . . . . . . . . 29,300

Total operating expenses . . . . . . . . . . . . . . . . . . . . . . . . . . . . . 71,400

Income from operations . . . . . . . . . . . . . . . . . . . . . . . . . . . . . . . . 12,900 Other revenues and gains (expenses and losses)

Interest revenue . . . . . . . . . . . . . . . . . . . . . . . . . . . . . . . . . . . . 1,000

Gain on sale of building . . . . . . . . . . . . . . . . . . . . . . . . . . . . . . 2,500

Interest expense . . . . . . . . . . . . . . . . . . . . . . . . . . . . . . . . . . . . (1,500)

Total other revenues and gains (expenses and losses) . . . . . . 2,000

Net income . . . . . . . . . . . . . . . . . . . . . . . . . . . . . . . . . . . . . . . . . . $ 14,900

Nonoperating activities computation

Income from operations computation

Gross profit computation

Beginning inventory. . . . . . $ 19,000 Net cost of purchases . . . . 232,400 Goods available for sale . . . 251,400 Less ending inventory . . . . 21,000 Cost of goods sold . . . . . . . $230,400

*Cost of goods sold:

EXHIBIT 4.13 Multiple-Step Income Statement

EXHIBIT 4.14 Single-Step Income Statement

Z-MART Income Statement

For Year Ended December 31, 2019

Revenues Net sales . . . . . . . . . . . . . . . . . . . . . . . . . . . . . . . . . . . $314,700

Interest revenue . . . . . . . . . . . . . . . . . . . . . . . . . . . . . 1,000

Gain on sale of building . . . . . . . . . . . . . . . . . . . . . . 2,500

Total revenues . . . . . . . . . . . . . . . . . . . . . . . . . . . . . . 318,200

Expenses Cost of goods sold . . . . . . . . . . . . . . . . . . . . . . . . . . . $230,400

Selling expenses . . . . . . . . . . . . . . . . . . . . . . . . . . . . 42,100

General and administrative expenses . . . . . . . . . . . 29,300

Interest expense . . . . . . . . . . . . . . . . . . . . . . . . . . . . 1,500

Total expenses . . . . . . . . . . . . . . . . . . . . . . . . . . . . . . 303,300

Net income . . . . . . . . . . . . . . . . . . . . . . . . . . . . . . . . . . $ 14,900 Point: Net income is identical under the single-step and multiple-step formats.

Point: Z-Mart did not have any nonoperating activities. Exhibit 4.13 includes some for illustrative purposes.

158 Chapter 4 Accounting for Merchandising Operations

Classified Balance Sheet The classified balance sheet reports merchandise inventory as a current asset, usually after accounts receivable, according to how quickly they can be converted to cash. Inventory is converted less quickly to cash than accounts receivable because inventory first must be sold before cash can be received. Exhibit 4.15 shows the current asset section of Z-Mart’s classified balance sheet (other sections are similar to the previous chapter).

Z-MART Balance Sheet (partial)

December 31, 2019

Current assets Cash . . . . . . . . . . . . . . . . . . . . . . . . . $ 8,200 Accounts receivable . . . . . . . . . . . . . 11,200 Merchandise inventory . . . . . . . . . 21,000 Office supplies . . . . . . . . . . . . . . . . . 550 Store supplies . . . . . . . . . . . . . . . . . . 250 Prepaid insurance . . . . . . . . . . . . . . . 300 Total current assets . . . . . . . . . . . . . $41,500

EXHIBIT 4.15 Classified Balance Sheet (partial) of a Merchandiser

Shenanigans Accurate invoices are important to both sellers and buyers. Merchandisers use invoices to make sure they receive full payment for products provided. To achieve this, controls are set up. Still, failures occur. A survey reports that 30% of employees in sales and marketing witnessed false or misleading invoices sent to customers. Another 29% observed employees violating contract terms with customers (KPMG). ■

Ethical Risk

Taret’s adjusted trial balance on April 30, its fiscal year-end, is shown here (accounts in random order). (a) Prepare a multiple-step income statement that begins with gross sales and includes separate categories for net sales, cost of goods sold, selling expenses, and gen- eral and administrative expenses. (b) Prepare a single-step income statement that begins with net sales and includes these expense cat- egories: cost of goods sold, selling expenses, and general and administrative expenses.

Multiple- and Single-Step Income Statements

NEED-TO-KNOW 4-5

P4

Solution

a. Multiple-step income statement.

TARET Income Statement

For Year Ended April 30

Sales . . . . . . . . . . . . . . . . . . . . . . . . . . . . . . . . . . . . . . . . . . . . . $9,500 Less: Sales discounts . . . . . . . . . . . . . . . . . . . . . . . . . . . . . . . $260 Sales returns and allowances . . . . . . . . . . . . . . . . . . . . 240 500 Net sales . . . . . . . . . . . . . . . . . . . . . . . . . . . . . . . . . . . . . . . . . 9,000 Cost of goods sold . . . . . . . . . . . . . . . . . . . . . . . . . . . . . . . . . 6,500 Gross profit . . . . . . . . . . . . . . . . . . . . . . . . . . . . . . . . . . . . . . . 2,500 Operating expenses Selling expenses Sales salaries expense . . . . . . . . . . . . . . . . . . . . . . . . . . 450 Rent expense—Selling space . . . . . . . . . . . . . . . . . . . . . 400 Store supplies expense . . . . . . . . . . . . . . . . . . . . . . . . . 30 Advertising expense . . . . . . . . . . . . . . . . . . . . . . . . . . . . 20 Total selling expenses . . . . . . . . . . . . . . . . . . . . . . . . . . 900 General and administrative expenses Office salaries expense . . . . . . . . . . . . . . . . . . . . . . . . . 420 Rent expense—Office space . . . . . . . . . . . . . . . . . . . . . 72 Office supplies expense . . . . . . . . . . . . . . . . . . . . . . . . 8 Total general and administrative expenses . . . . . . . . . 500 Total operating expenses . . . . . . . . . . . . . . . . . . . . . . . . . 1,400 Net income . . . . . . . . . . . . . . . . . . . . . . . . . . . . . . . . . . . . . . . . $1,100

TARET Income Statement

For Year Ended April 30

Net sales . . . . . . . . . . . . . . . . . . . . . . . . . . . . . . . . . . . . . . $9,000 Expenses Cost of goods sold . . . . . . . . . . . . . . . . . . . . . . . . . . $6,500 Selling expenses . . . . . . . . . . . . . . . . . . . . . . . . . . . . 900 General and administrative expenses . . . . . . . . . . . 500 Total expenses . . . . . . . . . . . . . . . . . . . . . . . . . . . . . 7,900 Net income . . . . . . . . . . . . . . . . . . . . . . . . . . . . . . . . . . . . . $1,100

b. Single-step income statement.

Adjusted Trial Balance Debit Credit

Merchandise inventory . . . . . . . . . . . . . . . . . . . $ 800 Other (noninventory) assets . . . . . . . . . . . . . . . 2,600 Total liabilities . . . . . . . . . . . . . . . . . . . . . . . . . . $ 500 Common stock . . . . . . . . . . . . . . . . . . . . . . . . . 400 Retained earnings . . . . . . . . . . . . . . . . . . . . . . . 1,700 Dividends . . . . . . . . . . . . . . . . . . . . . . . . . . . . . 300 Sales . . . . . . . . . . . . . . . . . . . . . . . . . . . . . . . . . 9,500 Sales discounts . . . . . . . . . . . . . . . . . . . . . . . . . 260 Sales returns and allowances . . . . . . . . . . . . . 240 Cost of goods sold . . . . . . . . . . . . . . . . . . . . . . 6,500 Sales salaries expense . . . . . . . . . . . . . . . . . . . 450 Rent expense—Selling space . . . . . . . . . . . . . . 400 Store supplies expense . . . . . . . . . . . . . . . . . . 30 Advertising expense . . . . . . . . . . . . . . . . . . . . 20 Office salaries expense . . . . . . . . . . . . . . . . . . 420 Rent expense—Office space . . . . . . . . . . . . . . 72 Office supplies expense . . . . . . . . . . . . . . . . . 8 Totals . . . . . . . . . . . . . . . . . . . . . . . . . . . . . . . . . $12,100 $12,100

Do More: QS 4-11, E 4-11, E 4-15, P 4-3

Chapter 4 Accounting for Merchandising Operations 159

Acid-Test and Gross Margin Ratios Decision Analysis

Acid-Test Ratio One measure of a merchandiser’s ability to pay its current liabilities (referred to as its liquidity) is the acid- test ratio. The acid-test ratio, also called quick ratio, is defined as quick assets (cash, short-term invest- ments, and current receivables) divided by current liabilities—see Exhibit 4.16. It differs from the current ratio by excluding less liquid current assets such as inventory and prepaid expenses that take longer to be converted to cash.

Exhibit 4.17 shows both the acid-test and current ratios of Nike and Under Armour for three recent years. Nike’s acid-test ratio implies that it has enough quick assets to cover current liabilities. It is also on par with its competitor, Under Armour. Nike’s current ratio suggests it has more than enough current assets to cover current liabilities. Analysts might argue that Nike could invest some current assets in more productive assets. An acid-test ratio less than 1.0 means that current liabilities exceed quick assets. A rule of thumb is that the acid-test ratio should have a value near, or higher than, 1.0. Less than 1.0 raises liquid- ity concerns unless a company can get enough cash from sales or if liabilities are not due until late in the next period.

A1 Compute the acid-test ratio and explain its use to as- sess liquidity.

Acid-test ratio = Cash and cash equivalents + Short-term investments + Current receivables

Current liabilities

EXHIBIT 4.16 Acid-Test (Quick) Ratio

Company $ millions Current Year 1 Year Ago 2 Years Ago

Nike Total quick assets . . . . . . . . . . . . . . $ 9,856 $ 8,698 $ 9,282 Total current assets . . . . . . . . . . . . . $16,061 $15,025 $15,587

Total current liabilities . . . . . . . . . . . $ 5,474 $ 5,358 $ 6,332

Acid-test ratio . . . . . . . . . . . . . . . . . 1.8 1.6 1.5 Current ratio . . . . . . . . . . . . . . . . . . 2.9 2.8 2.5 Under Armour Acid-test ratio . . . . . . . . . . . . . . . . . 0 .9 1 .3 1 .2 Current ratio . . . . . . . . . . . . . . . . . . 2 .2 2 .9 3 .1

EXHIBIT 4.17 Acid-Test and Current Ratios for two competitors

Gross Margin Ratio Without enough gross profit, a merchandiser can fail. The gross margin ratio helps understand this link. It differs from the profit margin ratio in that it excludes all costs except cost of goods sold. The gross margin ratio (or gross profit ratio) is defined as gross margin (net sales minus cost of goods sold) divided by net sales—see Exhibit 4.18.

Supplier A retailer requests to purchase supplies on credit from your company. You have no prior experience with this retailer. The retailer’s current ratio is 2.1, its acid-test ratio is 0.5, and inventory makes up most of its current assets. Do you extend credit? ■ Answer: A current ratio of 2.1 suggests sufficient current assets to cover current liabilities. An acid-test ratio of 0.5 suggests, however, that quick assets can cover only about one-half of current liabilities. The retailer depends on money from sales of inventory to pay current liabilities. If sales decline, the likelihood that this retailer will default on its payments increases. You probably do not extend credit.

Decision Maker

Point: Successful use of a just-in- time inventory system can narrow the gap between the acid-test ratio and the current ratio.

A2 Compute the gross margin ratio and explain its use to assess profitability.

EXHIBIT 4.18 Gross Margin RatioGross margin ratio =

Net sales − Cost of goods sold Net sales

Exhibit 4.19 shows the gross margin ratio of Nike for three recent years. For Nike, each $1 of sales in the current year yielded about 44.6¢ in gross margin to cover all expenses and still produce a net income. This 44.6¢ margin is down from 46.2¢ in the prior year. This decrease is unfavorable.

$ millions Current Year 1 Year Ago 2 Years Ago

Gross margin . . . . . . . . . . . . . . . . . . . . . . . . . . . . . . . . . . . . . $15,312 $14,971 $14,067

Net sales . . . . . . . . . . . . . . . . . . . . . . . . . . . . . . . . . . . . . . . . $34,350 $32,376 $30,601

Gross margin ratio . . . . . . . . . . . . . . . . . . . . . . . . . . . . . . . 44.6% 46.2% 46.0%

EXHIBIT 4.19 Nike’s Gross Margin Ratio

160 Chapter 4 Accounting for Merchandising Operations

Use the following adjusted trial balance and additional information to complete the requirements.

COMPREHENSIVE 1

Single- and Multiple-Step Income Statements, Closing Entries, and Analysis Using Acid-Test and Gross Margin

NEED-TO-KNOW 4-6 KC ANTIQUES

Adjusted Trial Balance December 31

Debit Credit

Cash . . . . . . . . . . . . . . . . . . . . . . . . . . . . . . . . . . . . . . $ 7,000

Accounts receivable . . . . . . . . . . . . . . . . . . . . . . . . . 13,000

Merchandise inventory (ending) . . . . . . . . . . . . . . . 60,000

Store supplies . . . . . . . . . . . . . . . . . . . . . . . . . . . . . 1,500

Equipment . . . . . . . . . . . . . . . . . . . . . . . . . . . . . . . . 45,600

Accumulated depreciation—Equipment . . . . . . . . . $ 16,600

Accounts payable . . . . . . . . . . . . . . . . . . . . . . . . . . . 9,000

Salaries payable . . . . . . . . . . . . . . . . . . . . . . . . . . . 2,000

Common stock . . . . . . . . . . . . . . . . . . . . . . . . . . . . . 20,000

Retained earnings . . . . . . . . . . . . . . . . . . . . . . . . . . . 59,000

Dividends . . . . . . . . . . . . . . . . . . . . . . . . . . . . . . . . . 10,000

Sales . . . . . . . . . . . . . . . . . . . . . . . . . . . . . . . . . . . . . 343,250

Sales discounts . . . . . . . . . . . . . . . . . . . . . . . . . . . . . 5,000

Sales returns and allowances . . . . . . . . . . . . . . . . . 6,000

Cost of goods sold . . . . . . . . . . . . . . . . . . . . . . . . . . 159,900

Depreciation expense—Store equipment . . . . . . . 4,100

Depreciation expense—Office equipment . . . . . . . 1,600

Sales salaries expense . . . . . . . . . . . . . . . . . . . . . . 30,000

Office salaries expense . . . . . . . . . . . . . . . . . . . . . . 34,000

Insurance expense . . . . . . . . . . . . . . . . . . . . . . . . . . 11,000

Rent expense—Selling space . . . . . . . . . . . . . . . . . . 16,800

Rent expense—Office space . . . . . . . . . . . . . . . . . . 7,200

Store supplies expense . . . . . . . . . . . . . . . . . . . . . . 5,750

Advertising expense . . . . . . . . . . . . . . . . . . . . . . . . 31,400

Totals . . . . . . . . . . . . . . . . . . . . . . . . . . . . . . . . . . . . $449,850 $449,850

160 Chapter 4 Accounting for Merchandising Operations

Financial Officer Your company has a 36% gross margin ratio and a 17% net profit margin ratio. Industry averages are 44% for gross margin and 16% for net profit margin. Do these comparative results concern you? ■ Answer: Your company’s net profit margin is about equal to the industry average. However, gross margin shows that your company is paying far more in cost of goods sold or receiving far less in sales price than competitors. You should try to find the problem with cost of goods sold, sales, or both.

Decision Maker

KC Antiques’s supplementary records for the year reveal the following itemized costs for merchandising activities.

Invoice cost of merchandise purchases . . . . . . . $150,000

Purchases discounts received . . . . . . . . . . . . . . . 2,500 Purchases returns and allowances . . . . . . . . . . . $2,700

Cost of transportation-in . . . . . . . . . . . . . . . . . . . 5,000

Required

1. Use the supplementary records to compute the total cost of merchandise purchases for the year. 2. Prepare a multiple-step income statement for the year. (Beginning inventory was $70,100.) 3. Prepare a single-step income statement for the year. 4. Prepare closing entries for KC Antiques at December 31. 5. Compute the acid-test ratio and the gross margin ratio. Explain the meaning of each ratio and interpret

them for KC Antiques.

Chapter 4 Accounting for Merchandising Operations 161

PLANNING THE SOLUTION Compute the total cost of merchandise purchases for the year. To prepare the multiple-step statement, first compute net sales. Then, to compute cost of goods sold,

add the net cost of merchandise purchases for the year to beginning inventory and subtract the cost of ending inventory. Subtract cost of goods sold from net sales to get gross profit. Then classify expenses as selling expenses or general and administrative expenses.

To prepare the single-step income statement, begin with net sales. Then list and subtract the expenses. The first closing entry debits all temporary accounts with credit balances and opens the Income

Summary account. The second closing entry credits all temporary accounts with debit balances. The third entry closes the Income Summary account to the Retained Earnings account, and the fourth entry closes the Dividends account to the Retained Earnings account.

Identify the quick assets on the adjusted trial balance. Compute the acid-test ratio by dividing quick assets by current liabilities. Compute the gross margin ratio by dividing gross profit by net sales.

SOLUTION 1.

Invoice cost of merchandise purchases . . . . . . . . . . . . $150,000

Less: Purchases discounts received . . . . . . . . . . . . . . . 2,500

Purchases returns and allowances . . . . . . . . . . . 2,700

Add: Cost of transportation-in . . . . . . . . . . . . . . . . . . . . 5,000

Total cost of merchandise purchases . . . . . . . . . . . . . . $149,800

2. Multiple-step income statement. 3. Single-step income statement.

KC ANTIQUES Income Statement

For Year Ended December 31

Sales . . . . . . . . . . . . . . . . . . . . . . . . . . . . . . . . . . . . . . . $343,250

Less: Sales discounts . . . . . . . . . . . . . . . . . . . . . . . . . . $ 5,000

Sales returns and allowances . . . . . . . . . . . . . . 6,000 11,000

Net sales . . . . . . . . . . . . . . . . . . . . . . . . . . . . . . . . . . 332,250

Cost of goods sold* . . . . . . . . . . . . . . . . . . . . . . . . . . . 159,900

Gross profit . . . . . . . . . . . . . . . . . . . . . . . . . . . . . . . . . . 172,350

Expenses

Selling expenses

Depreciation expense—Store equipment . . . . . 4,100

Sales salaries expense . . . . . . . . . . . . . . . . . . . . 30,000

Rent expense—Selling space . . . . . . . . . . . . . . . 16,800

Store supplies expense . . . . . . . . . . . . . . . . . . . 5,750

Advertising expense . . . . . . . . . . . . . . . . . . . . . . 31,400

Total selling expenses . . . . . . . . . . . . . . . . . . . . . 88,050

General and administrative expenses

Depreciation expense—Office equipment . . . . . 1,600

Office salaries expense . . . . . . . . . . . . . . . . . . . . 34,000

Insurance expense . . . . . . . . . . . . . . . . . . . . . . . 11,000

Rent expense—Office space . . . . . . . . . . . . . . . . 7,200

Total general and administrative expenses . . . . 53,800

Total operating expenses . . . . . . . . . . . . . . . . . . . . 141,850

Net income . . . . . . . . . . . . . . . . . . . . . . . . . . . . . . . . . . $ 30,500

*Cost of goods sold also can be directly computed: Beginning merchandise inventory . . . . . . . . . . . . . . . . . . . . . . . $ 70,100 Total cost of merchandise purchases (from part 1) . . . . . . . . . . 149,800 Goods available for sale . . . . . . . . . . . . . . . . . . . . . . . . . . . . . . 219,900 Ending merchandise inventory . . . . . . . . . . . . . . . . . . . . . . . . . 60,000 Cost of goods sold . . . . . . . . . . . . . . . . . . . . . . . . . . . . . . . . . . . $159,900

Tax expense for a corporation appears immediately before Net income in its own category .

KC ANTIQUES Income Statement

For Year Ended December 31

Net sales . . . . . . . . . . . . . . . . . . . . . . . . . . . . . $332,250

Expenses

Cost of goods sold . . . . . . . . . . . . . . . . . . . $159,900

Selling expenses . . . . . . . . . . . . . . . . . . . . 88,050

General and administrative expenses . . . 53,800

Total expenses . . . . . . . . . . . . . . . . . . . . . . 301,750

Net income . . . . . . . . . . . . . . . . . . . . . . . . . . . $ 30,500

162 Chapter 4 Accounting for Merchandising Operations

5. Acid-test ratio = (Cash and equivalents + Short-term investments + Current receivables)∕ Current liabilities

= (Cash + Accounts receivable)∕(Accounts payable + Salaries payable)

= ($7,000 + $13,000)∕($9,000 + $2,000) = $20,000∕$11,000 = 1.82 Gross margin ratio = Gross profit∕Net sales = $172,350∕$332,250 = 0.52 (or 52%)

KC Antiques has a healthy acid-test ratio of 1.82. This means it has $1.82 in liquid assets to sat- isfy each $1.00 in current liabilities. The gross margin of 0.52 shows that KC Antiques spends 48¢ ($1.00 − $0.52) of every dollar of net sales on the costs of acquiring the merchandise it sells. This leaves 52¢ of every dollar of net sales to cover other expenses incurred in the business and to provide a net profit.

4. Dec . 31 Sales . . . . . . . . . . . . . . . . . . . . . . . . . . . . . . . . . . . . . . . . . . . . . 343,250

Income Summary . . . . . . . . . . . . . . . . . . . . . . . . . . . . . . . 343,250

Close credit balances in temporary accounts.

Dec . 31 Income Summary . . . . . . . . . . . . . . . . . . . . . . . . . . . . . . . . . . . 312,750

Sales Discounts . . . . . . . . . . . . . . . . . . . . . . . . . . . . . . . . 5,000

Sales Returns and Allowances . . . . . . . . . . . . . . . . . . . . 6,000

Cost of Goods Sold . . . . . . . . . . . . . . . . . . . . . . . . . . . . . 159,900

Depreciation Expense—Store Equipment . . . . . . . . . . . . 4,100

Depreciation Expense—Office Equipment . . . . . . . . . . . 1,600

Sales Salaries Expense . . . . . . . . . . . . . . . . . . . . . . . . . . 30,000

Office Salaries Expense . . . . . . . . . . . . . . . . . . . . . . . . . . 34,000

Insurance Expense . . . . . . . . . . . . . . . . . . . . . . . . . . . . . . 11,000

Rent Expense—Selling Space . . . . . . . . . . . . . . . . . . . . . . 16,800

Rent Expense—Office Space . . . . . . . . . . . . . . . . . . . . . . 7,200

Store Supplies Expense . . . . . . . . . . . . . . . . . . . . . . . . . . 5,750

Advertising Expense . . . . . . . . . . . . . . . . . . . . . . . . . . . . 31,400

Close debit balances in temporary accounts.

Dec . 31 Income Summary . . . . . . . . . . . . . . . . . . . . . . . . . . . . . . . . . . . . 30,500

Retained Earnings . . . . . . . . . . . . . . . . . . . . . . . . . . . . . . . 30,500

Close Income Summary account.

Dec . 31 Retained Earnings . . . . . . . . . . . . . . . . . . . . . . . . . . . . . . . . . . . 10,000

Dividends . . . . . . . . . . . . . . . . . . . . . . . . . . . . . . . . . . . . . 10,000

Close Dividends account.

Prepare journal entries for the following transactions for both the seller (BMX) and buyer (Sanuk).

May 4 BMX sold $1,500 of merchandise on account to Sanuk, terms FOB shipping point, n∕45, in- voice dated May 4. The cost of the merchandise was $900.

6 Sanuk paid transportation charges of $30 on the May 4 purchase from BMX. 8 BMX sold $1,000 of merchandise on account to Sanuk, terms FOB destination, n∕15, in-

voice dated May 8. The cost of the merchandise was $700. This sale permitted returns for 30 days.

10 BMX paid transportation costs of $50 for delivery of merchandise sold to Sanuk on May 8. 16 BMX issued Sanuk a $200 credit memorandum for merchandise returned. The merchan-

dise was purchased by Sanuk on account on May 8. The cost of the merchandise returned was $140.

18 BMX received payment from Sanuk for the May 8 purchase. 21 BMX sold $2,400 of merchandise on account to Sanuk, terms FOB shipping point, 2∕10,

n∕EOM. The cost of the merchandise was $1,440. This sale permitted returns for 90 days. 31 BMX received payment from Sanuk for the May 21 purchase, less discount.

COMPREHENSIVE 2

Recording Merchandising Transactions—Both Seller and Buyer

NEED-TO-KNOW 4-7

Chapter 4 Accounting for Merchandising Operations 163

Solution BMX (Seller) Sanuk (Buyer)

May 4 Accounts Receivable—Sanuk . . . . . . . . . . 1,500 Merchandise Inventory . . . . . . . . . . . . . . 1,500

Sales . . . . . . . . . . . . . . . . . . . . . . . . . 1,500 Accounts Payable—BMX . . . . . . . . . 1,500

Cost of Goods Sold . . . . . . . . . . . . . . . . . . . 900

Merchandise Inventory . . . . . . . . . . . 900

6 No entry . Merchandise Inventory . . . . . . . . . . . . . . 30

Cash . . . . . . . . . . . . . . . . . . . . . . . . . 30

8 Accounts Receivable—Sanuk . . . . . . . . . . 1,000 Merchandise Inventory . . . . . . . . . . . . . . 1,000

Sales . . . . . . . . . . . . . . . . . . . . . . . . . 1,000 Accounts Payable—BMX . . . . . . . . . 1,000

Cost of Goods Sold . . . . . . . . . . . . . . . . . . 700

Merchandise Inventory . . . . . . . . . . . 700

10 Delivery Expense . . . . . . . . . . . . . . . . . . . . 50 No entry .

Cash . . . . . . . . . . . . . . . . . . . . . . . . . . 50

16 Sales Returns & Allowances . . . . . . . . . . . 200 Accounts Payable—BMX . . . . . . . . . . . . . 200

Accounts Receivable—Sanuk . . . . . . 200 Merchandise Inventory . . . . . . . . . . 200

Merchandise Inventory . . . . . . . . . . . . . . . 140

Cost of Goods Sold . . . . . . . . . . . . . . 140

18 Cash . . . . . . . . . . . . . . . . . . . . . . . . . . . . . . 800 Accounts Payable—BMX . . . . . . . . . . . . . 800

Accounts Receivable—Sanuk . . . . . . 800 Cash . . . . . . . . . . . . . . . . . . . . . . . . . 800

21 Accounts Receivable—Sanuk . . . . . . . . . . . 2,400 Merchandise Inventory . . . . . . . . . . . . . . 2,400

Sales . . . . . . . . . . . . . . . . . . . . . . . . . 2,400 Accounts Payable—BMX . . . . . . . . . 2,400

Cost of Goods Sold . . . . . . . . . . . . . . . . . . . 1,440

Merchandise Inventory . . . . . . . . . . . 1,440

31 Cash . . . . . . . . . . . . . . . . . . . . . . . . . . . . . . 2,352 Accounts Payable—BMX . . . . . . . . . . . . . 2,400

Sales Discounts . . . . . . . . . . . . . . . . . . . . . 48 Merchandise Inventory . . . . . . . . . . 48

Accounts Receivable—Sanuk . . . . . . 2,400 Cash . . . . . . . . . . . . . . . . . . . . . . . . . 2,352

APPENDIX

Periodic Inventory System 4A A periodic inventory system requires updating the inventory account only at the end of a period. During the period, the Merchandise Inventory balance remains unchanged and cost of merchandise is recorded in a temporary Purchases account. When a company sells merchandise, it records revenue but not the cost of the goods sold. At the end of the period, it takes a physical count of inventory to get ending inventory. The cost of goods sold is then computed as cost of merchandise available for sale minus ending inventory.

Recording Merchandise Purchases Under a periodic system, the purchases, purchases returns and allowances, purchases discounts, and transportation-in transactions are recorded in separate temporary accounts. At period-end, each of these temporary accounts is closed, which updates the Merchandise Inventory account. To demonstrate, journal entries under the periodic inventory system are shown for the most common transactions (codes a through d link these transactions to those in the chap- ter). For comparison, perpetual system journal entries are shown to the right of each periodic entry. Differences are highlighted.

Credit Purchases with Cash Discounts The periodic system uses a temporary Purchases account that accumulates the cost of all purchase transactions during each period. The Purchases account has a normal debit balance, as it increases the cost of merchandise available for sale. Z-Mart’s November 2 entry to record the purchase of merchandise for $500 on credit with terms of 2∕10, n∕30 is

P5 Record and compare merchandising transactions using both periodic and perpetual inventory systems.

(a) Periodic Perpetual

Purchases . . . . . . . . . . . . . . . . . . 500 Merchandise Inventory . . . . . . . . . . . 500

Accounts Payable . . . . . . . 500 Accounts Payable . . . . . . . . . . . 500

164 Chapter 4 Accounting for Merchandising Operations

Payment of Purchases The periodic system uses a temporary Purchases Discounts account that accumu- lates discounts taken during the period. If payment for transaction a is made within the discount period, the entry is

Purchases Allowances The buyer and seller agree to a $30 purchases allowance for defective goods (whether paid within the discount period or not). In the periodic system, the temporary Purchases Returns and Allowances account accumulates the cost of all returns and allowances during a period. The buyer records the $30 allowance as

If payment for transaction a is made after the discount period expires, the entry is

(b1) Periodic Perpetual

Accounts Payable . . . . . . . . . . . . 500 Accounts Payable . . . . . . . . . . . . . . . . 500

Purchases Discounts* . . . . . 10 Merchandise Inventory* . . . . . . . 10

Cash . . . . . . . . . . . . . . . . . . 490 Cash . . . . . . . . . . . . . . . . . . . . . . 490 *$500 × 2% *$500 × 2%

(b2) Periodic Perpetual

Accounts Payable . . . . . . . . . . . . 500 Accounts Payable . . . . . . . . . . . . . . . . 500

Cash . . . . . . . . . . . . . . . . . . 500 Cash . . . . . . . . . . . . . . . . . . . . . . 500

(c1) Periodic Perpetual

Accounts Payable . . . . . . . . . . . . 30 Accounts Payable . . . . . . . . . . . . . . . . 30

Purchases Returns and Allowances . . . . . . . . . . 30 Merchandise Inventory . . . . . . . 30

(c2) Periodic Perpetual

Accounts Payable . . . . . . . . . . . . 50 Accounts Payable . . . . . . . . . . . . . . . . 50

Purchases Returns and Allowances . . . . . . . . . . 50 Merchandise Inventory . . . . . . . 50

(d) Periodic Perpetual

Transportation-In . . . . . . . . . . . . 75 Merchandise Inventory . . . . . . . . . . . 75

Cash . . . . . . . . . . . . . . . . . . . 75 Cash . . . . . . . . . . . . . . . . . . . . . . 75

Purchases Returns The buyer returns $50 of merchandise within the discount period. The entry is

Point: Purchases Discounts and Purchases Returns and Allowances are contra purchases accounts and have normal credit balances, as they both decrease the cost of merchandise available for sale.

Transportation-In The buyer paid a $75 freight charge to transport goods with terms FOB destination. In the periodic system, this cost is recorded in a temporary Transportation-In account, which has a normal debit balance as it increases the cost of merchandise available for sale.

Recording Merchandise Sales Journal entries under the periodic system are shown for the most common transactions (codes e through h link these transactions to those in the chapter). Perpetual system entries are shown to the right of each periodic entry. Differences are highlighted.

Credit Sales and Receipt of Payments Both the periodic and perpetual systems record sales entries simi- larly, using the gross method. The same holds for entries related to payment of receivables from sales both during and after the discount period. However, under the periodic system, the cost of goods sold is not recorded at the time of each sale (whereas it is under the perpetual system). The entry to record $1,000 in credit sales (costing $300) is

Periodic Perpetual

Accounts Receivable . . . . . . . . . . 1,000 Accounts Receivable . . . . . . . . . . . . . . 1,000

Sales . . . . . . . . . . . . . . . . . . 1,000 Sales . . . . . . . . . . . . . . . . . . . . . . 1,000

Cost of Goods Sold . . . . . . . . . . . . . . . 300

No cost-side entry . . . . . . . Merchandise Inventory . . . . . . . 300

Chapter 4 Accounting for Merchandising Operations 165

Returns Received by Seller A customer returned merchandise for a cash refund. The goods sell for $15 and cost $9. (Recall: The periodic system records only the revenue effect, not the cost effect, for sales transactions.) The entry for the seller to take back the return is

Allowances Granted by Seller The seller gives a price reduction and credits the buyer’s accounts receivable for $10. The entry is identical under the periodic and perpetual systems. The seller records this allowance as

Recording Adjusting Entries Shrinkage—Adjusting Entry Adjusting (and closing) entries for the two systems are in Exhibit 4A.1. The $250 shrinkage is only recorded under the perpetual system—see entry z in Exhibit 4A.1. Shrinkage in cost of goods is unknown using a periodic system because inventory is not continually updated and therefore cannot be compared to the physical count.

Periodic Perpetual

(f ) Sales Returns and Allowances . . . . 10 Sales Returns and Allowances . . . . . . 10 Accounts Receivable . . . . . . . 10 Accounts Receivable . . . . . . . . . 10

Periodic Perpetual

(e1) Sales Returns and Allowances . . . . 15 Sales Returns and Allowances . . . . . . 15 Cash . . . . . . . . . . . . . . . . . . . . 15 Cash . . . . . . . . . . . . . . . . . . . . . . 15

(e2) Merchandise Inventory . . . . . . . . . . . 9 No entry . . . . . . . . . . . . . . . . . Cost of Goods Sold . . . . . . . . . . 9

EXHIBIT 4A.1 Comparison of Adjusting and Closing Entries— Periodic and Perpetual

Periodic Perpetual Adjusting Entries Adjusting Entries

(z) None Cost of Goods Sold . . . . . . . . . . . . . . 250 Merchandise Inventory . . . . . . 250

(g) Sales Discounts . . . . . . . . . . . . . . . . 50 Sales Discounts . . . . . . . . . . . . . . . . . 50 Allowance for Sales Discounts 50 Allowance for Sales Discounts . . 50

(h1) Sales Returns and Allowances . . . . 900 Sales Returns and Allowances . . . . . . 900 Sales Refund Payable . . . . . . . 900 Sales Refund Payable . . . . . . . . 900

(h2) Inventory Returns Estimated . . . . . . 300 Inventory Returns Estimated . . . . . . . 300 Purchases . . . . . . . . . . . . . . . . 300 Cost of Goods Sold . . . . . . . . . . 300

Entries in gray are covered in Appendix 4B. Entries in gray are covered in Appendix 4B.

Periodic Perpetual Closing Entries Closing Entries

(1) Sales . . . . . . . . . . . . . . . . . . . . . . . . 321,000 Sales . . . . . . . . . . . . . . . . . . . . . . . . . . 321,000 Merchandise Inventory (ending) 21,000 Purchases Discounts . . . . . . . . . . 4,200 Purchases Returns and Allowances 1,500 Income Summary . . . . . . . . . . 347,700 Income Summary . . . . . . . . . . . 321,000

(2) Income Summary . . . . . . . . . . . . . . 334,800 Income Summary . . . . . . . . . . . . . . . . 308,100 Sales Discounts . . . . . . . . . . . 4,300 Sales Discounts . . . . . . . . . . . . 4,300 Sales Returns and Allowances 2,000 Sales Returns and Allowances . . 2,000 Merch. Inven. (beginning) . . . 19,000 Purchases . . . . . . . . . . . . . . . 235,800 Cost of Goods Sold . . . . . . . . . 230,400 Transportation-In . . . . . . . . . 2,300 Depreciation Expense . . . . . . 3,700 Depreciation Expense . . . . . . . 3,700 Salaries Expense . . . . . . . . . . 43,800 Salaries Expense . . . . . . . . . . . 43,800 Insurance Expense . . . . . . . . 600 Insurance Expense . . . . . . . . . . 600 Rent Expense . . . . . . . . . . . . . 9,000 Rent Expense . . . . . . . . . . . . . . 9,000 Supplies Expense . . . . . . . . . 3,000 Supplies Expense . . . . . . . . . . . 3,000 Advertising Expense . . . . . . . 11,300 Advertising Expense . . . . . . . . . 11,300

(3) Income Summary . . . . . . . . . . . . . . 12,900 Income Summary . . . . . . . . . . . . . . . . 12,900 Retained Earnings . . . . . . . . . 12,900 Retained Earnings . . . . . . . . . . . 12,900

(4) Retained Earnings . . . . . . . . . . . . . 4,000 Retained Earnings . . . . . . . . . . . . . . . 4,000 Dividends . . . . . . . . . . . . . . . . 4,000 Dividends . . . . . . . . . . . . . . . . . . 4,000

166 Chapter 4 Accounting for Merchandising Operations

Expected Sales Discounts—Adjusting Entry Both the periodic and perpetual methods make a period-end adjusting entry under the gross method to estimate the $50 sales discounts arising from current-period sales that are likely to be taken in future periods. Z-Mart made the period-end adjusting entry g in Exhibit 4A.1 for expected sales discounts.

Expected Returns and Allowances—Adjusting Entry Both the periodic and perpetual inventory systems estimate returns and allowances arising from current-period sales that will occur in future periods. The adjusting entry for both systems is identical for the sales side, but slightly different for the cost side. The period-end entries h1 and h2 in Exhibit 4A.1 are used to record the updates to expected sales refunds of $900 and the cost side of $300. Under both systems, the seller sets up a Sales Refund Payable account, which is a current liability reflecting the amount expected to be refunded to customers, and an Inventory Returns Estimated account, which is a current asset reflecting the inventory estimated to be returned.

Recording Closing Entries Periodic and perpetual inventory systems have slight differ- ences in closing entries. The period-end Merchandise Inventory balance (unadjusted) is $19,000 under the periodic system. Because the periodic system does not update the Merchandise Inventory balance during the period, the $19,000 amount is the beginning inventory. A physical count of inventory taken at the end of the period reveals $21,000 of merchandise available. The adjusting and closing entries for the two systems are in Exhibit 4A.1. Recording the periodic inventory balance is a two-step process. The ending inventory balance of $21,000 is entered by debiting the inventory account in the first closing entry. The beginning inventory balance of $19,000 is deleted by crediting the inventory account in the second closing entry.1 By updating Merchandise Inventory and closing Purchases, Purchases Discounts, Purchases Returns and Allowances, and Transportation-In, the periodic system transfers the cost of sales amount to Income Summary. Review the periodic side of Exhibit 4A.1 and see that the red items affect Income Summary as follows.

1This approach is called the closing entry method. An alternative approach, referred to as the adjusting entry method, would not make any entries to Merchandise Inventory in the closing entries of Exhibit 4A.1, but instead would make two adjusting entries. Using Z-Mart data, the two adjusting entries would be (1) Dr. Income Summary and Cr. Merchandise Inventory for $19,000 each and (2) Dr. Merchandise Inventory and Cr. Income Summary for $21,000 each. The first entry removes the beginning balance of Merchandise Inventory, and the second entry records the actual ending balance.

Credit to Income Summary in the first closing entry includes amounts from

Merchandise inventory (ending) . . . . . . . . . . . . . . . . . . . . . . . . . . . . . . . . . . . . . . . . . . . . . . . . . . . . . . $  21,000

Purchases discounts . . . . . . . . . . . . . . . . . . . . . . . . . . . . . . . . . . . . . . . . . . . . . . . . . . . . . . . . . . . . . . . 4,200

Purchases returns and allowances . . . . . . . . . . . . . . . . . . . . . . . . . . . . . . . . . . . . . . . . . . . . . . . . . . . . 1,500

Debit to Income Summary in the second closing entry includes amounts from

Merchandise inventory (beginning) . . . . . . . . . . . . . . . . . . . . . . . . . . . . . . . . . . . . . . . . . . . . . . . . . . . . (19,000)

Purchases . . . . . . . . . . . . . . . . . . . . . . . . . . . . . . . . . . . . . . . . . . . . . . . . . . . . . . . . . . . . . . . . . . . . . . . . (235,800)

Transportation-in . . . . . . . . . . . . . . . . . . . . . . . . . . . . . . . . . . . . . . . . . . . . . . . . . . . . . . . . . . . . . . . . . . (2,300)

Net effect on Income Summary (net debit = cost of goods sold) . . . . . . . . . . . . . . . . . . . . . . . . . . . . . $(230,400)

This $230,400 effect on Income Summary is the cost of goods sold amount (which is equal to cost of goods sold reported in a perpetual inventory system). The periodic system transfers cost of goods sold to the Income Summary account but without using a Cost of Goods Sold account. Also, the periodic system does not separately measure shrinkage. Instead, it computes cost of goods available for sale, subtracts the cost of ending inventory, and defines the difference as cost of goods sold, which includes shrinkage.

Preparing Financial Statements The financial statements of a merchandiser using the periodic system are similar to those for a service company described in prior chapters. The income statement mainly differs by the inclusion of cost of goods sold and gross profit—of course, net sales is affected by dis- counts, returns, and allowances. The cost of goods sold section under the periodic system follows. The balance sheet mainly differs by the inclusion of merchandise inventory, inventory returns estimated, allowance for sales discounts, and sales refund payable. Visit the Additional Student Resource section of the Connect ebook to view sample chart of accounts for periodic and perpetual systems.

Calculation of Cost of Goods Sold

Beginning inventory . . . . . . . . . . . . . . . . . . . $ 19,000

Net cost of purchases . . . . . . . . . . . . . . . . . . 232,400

Cost of goods available for sale . . . . . . . . . 251,400

Less ending inventory . . . . . . . . . . . . . . . . . 21,000

Cost of goods sold . . . . . . . . . . . . . . . . . . . . $230,400

Chapter 4 Accounting for Merchandising Operations 167

APPENDIX

Adjusting Entries under New Revenue Recognition Rules 4B

Expected Sales Discounts—Adjusting Entry New revenue recognition rules require sales to be reported at the amount expected to be received. This means that a period-end adjusting entry is made to estimate sales discounts for current-period sales that are expected to be taken in future periods. To demonstrate, assume Z-Mart has the following unadjusted balances.

P6 Prepare adjustments for discounts, returns, and allowances per revenue recognition rules.

Accounts Receivable . . . . . . . . . . $11,250 Allowance for Sales Discounts . . . . . . . . . . $0

Of the $11,250 of receivables, $2,500 of them are within the 2% discount period for which we expect buy- ers to take $50 in future-period discounts (computed as $2,500 × 2%) arising from this period’s sales. The adjusting entry for the $50 update to Allowance for Sales Discounts is

Allow. for Sales Discounts

Beg . bal . 0 Req . adj . 50

Est . bal . 50

Allowance for Sales Discounts is a contra asset account and is reported on the balance sheet as a reduc- tion to the Accounts Receivable asset account. The Allowance for Sales Discounts account has a normal credit balance because it reduces Accounts Receivable, which has a normal debit balance. This adjusting entry results in both accounts receivable and sales being reported at expected amounts.*

Expected Returns and Allowances—Adjusting Entries To avoid overstatement of sales and cost of sales, sellers estimate sales returns and allowances in the period of the sale. Estimating returns and allowances requires companies to maintain the following two balance sheet accounts that are set up with adjusting entries. Two adjusting entries are made: one for the revenue side and one for the cost side.

Current Asset→Inventory Returns Estimated Current Liability→Sales Refund Payable

Balance Sheet—partial

Accounts receivable . . . . . . . . . . . . . . . . $11,250 Less allowance for sales discounts . . . . 50 Accounts receivable, net . . . . . . . . . . . . $11,200

Income Statement—partial

Sales . . . . . . . . . . . . . . . . . . . . . . . . . . . . . . . . . . . . $321,000 Less sales discounts, returns & allowances . . . . . 6,300 Net sales . . . . . . . . . . . . . . . . . . . . . . . . . . . . . . . . . $314,700

*Next Period Adjustment The Allowance for Sales Discounts balance remains unchanged during a period except for the period-end adjusting entry. At next period-end, assume that Z-Mart computes an $80 balance for the Allowance for Sales Discounts. Using our three-step adjusting process we get: Step 1: Current bal. is $50 credit in Allowance for Sales Discounts. Step 2: Current bal. should be $80 credit in Allowance for Sales Discounts. Step 3: Record entry to get from step 1 to step 2. Sales Discounts . . . . . . . . . . . . . . . . . . 30 Allowance for Sales Discounts. . . . . 30

Revenue Side for Expected R&A When returns and allowances are expected, a seller sets up a Sales Refund Payable account, which is a current liability showing the amount expected to be refunded to customers. Assume that on December 31 the company estimates future sales refunds to be $1,200. Assume also that the unadjusted balance in Sales Refund Payable is a $300 credit. The adjusting entry for the $900 update to Sales Refund Payable follows. The Sales Refund Payable account is updated only during the adjusting entry process. Its balance remains unchanged during the period when actual returns and allowances are recorded.

Assets = Liabilities + Equity −50 −50

(g) Dec . 31 Sales Discounts . . . . . . . . . . . . . . . . . . . . . . . . . . . . . . . . . . . . . 50 Allowance for Sales Discounts . . . . . . . . . . . . . . . . . . . . . 50 Adjustment for future discounts.

Assets = Liabilities + Equity +900 −900

Sales Refund Payable

Beg . bal . 300 Req . adj . 900

Est . bal . 1,200

(h1) Dec . 31 Sales Returns and Allowances . . . . . . . . . . . . . . . . . . . . . . . . . 900 Sales Refund Payable . . . . . . . . . . . . . . . . . . . . . . . . . . 900 Expected refund of sales.*

*This entry uses our three-step adjusting process: Step 1: Current bal. is $300 credit for Sales Refund Payable. Step 2: Current bal. should be $1,200 credit for Sales Refund Payable. Step 3: Record entry to get from step 1 to step 2.

168 Chapter 4 Accounting for Merchandising Operations

Cost Side for Expected R&A On the cost side, some inventory is expected to be returned, which means that cost of goods sold recorded at the time of sale is overstated due to expected returns. A seller sets up an Inventory Returns Estimated account, which is a current asset showing the inventory estimated to be returned. Extending the example above, assume that the company estimates future inventory returns to be $500 (which is the cost side of the $1,200 expected returns and allowances above). Assume also that the (beginning) unadjusted balance in Inventory Returns Estimated is a $200 debit. The adjusting entry for the $300 update to expected returns follows. The Inventory Returns Estimated account is updated only during the adjusting entry process. Its balance remains unchanged during the period when actual returns and allowances are recorded.

Point: If estimates of returns and allowances prove too high or too low, we adjust future estimates accordingly.

Assets = Liabilities + Equity +300 +300

Inventory Returns Est.

Beg . bal . 200 Req . adj . 300

Est . bal . 500

(h2) Dec . 31 Inventory Returns Estimated . . . . . . . . . . . . . . . . . . . . . . . . . 300 Cost of Goods Sold . . . . . . . . . . . . . . . . . . . . . . . . . . . . . . 300

Expected return of inventory.*

*This entry uses our three-step adjusting process: Step 1: Current bal. is $200 debit for Inventory Returns Estimated. Step 2: Current bal. should be $500 debit for Inventory Returns Estimated. Step 3: Record entry to get from step 1 to step 2.

At the current year-end, a company shows the following unadjusted balances for selected accounts.

P6

Estimating Discounts, Returns, and Allowances

NEED-TO-KNOW 4-8 Allowance for Sales Discounts . . . . . . . . . . . . . . $  75 credit Sales Discounts . . . . . . . . . . . . . . . . . . . . . . $1,850 debit Sales Refund Payable . . . . . . . . . . . . . . . . . . . . . 800 credit Sales Returns and Allowances . . . . . . . . . . 4,825 debit Inventory Returns Estimated . . . . . . . . . . . . . . . 450 debit Cost of Goods Sold . . . . . . . . . . . . . . . . . . . 9,875 debit

a. After an analysis of future sales discounts, the company estimates that the Allowance for Sales Discounts account should have a $275 credit balance. Prepare the current year-end adjusting journal entry for future sales discounts.

b. After an analysis of future sales returns and allowances, the company estimates that the Sales Refund Payable account should have an $870 credit balance (revenue side).

c. After an analysis of future inventory returns, the company estimates that the Inventory Returns Estimated account should have a $500 debit balance (cost side).

Solution

Dec . 31 Sales Discounts . . . . . . . . . . . . . . . . . . . . . . . . . . . . . . . . . . . . . 200 Allowance for Sales Discounts . . . . . . . . . . . . . . . . . . . . . 200 Adjustment for future discounts. $275 Cr. − $75 Cr. Dec . 31 Sales Returns and Allowances . . . . . . . . . . . . . . . . . . . . . . . . . 70 Sales Refund Payable . . . . . . . . . . . . . . . . . . . . . . . . . . . . 70 Adjustment for future sales refund. $870 Cr. − $800 Cr. Dec . 31 Inventory Returns Estimated . . . . . . . . . . . . . . . . . . . . . . . . . . . 50 Cost of Goods Sold . . . . . . . . . . . . . . . . . . . . . . . . . . . . . . 50 Adjustment for future inventory returns. $500 Dr. − $450 Dr.

Do More: QS 4-19, QS 4-20, E 4-20, E 4-21, E 4-22

APPENDIX

Net Method for Merchandising4C The net method records an invoice at its net amount (net of any cash discount). The gross method, cov- ered earlier in the chapter, initially records an invoice at its gross (full) amount. This appendix records merchandising transactions using the net method. Differences with the gross method are highlighted. When invoices are recorded at net amounts, any cash discounts are deducted from the balance of the Merchandise Inventory account when initially recorded. This assumes that all cash discounts will be taken. If any discounts are later lost, they are recorded in a Discounts Lost expense account reported on the income statement.

P7 Record and compare merchandising transactions using the gross method and net method.

Chapter 4 Accounting for Merchandising Operations 169

If the invoice is paid on (or before) November 12 within the discount period, it records Gross Method—Perpetual Net Method—Perpetual

Accounts Payable . . . . . . . . . . . . 500 Accounts Payable . . . . . . . . . . . . . . . . 490

Merchandise Inventory . . . 10

Cash . . . . . . . . . . . . . . . . . . 490 Cash . . . . . . . . . . . . . . . . . . . . . . 490

Gross Method—Perpetual Net Method—Perpetual

Accounts Payable . . . . . . . . . . . . 500 Accounts Payable . . . . . . . . . . . . . . . . 490 Discounts Lost* . . . . . . . . . . . . . . . . . . 10

Cash . . . . . . . . . . . . . . . . . . 500 Cash . . . . . . . . . . . . . . . . . . . . . . 500

*For simplicity, we record Discounts Lost on the payment date.

Cost of Goods Sold . . . . . . . . . . . 200 Cost of Goods Sold . . . . . . . . . . . . . . . 200

Merchandise Inventory . . . 200 Merchandise Inventory . . . . . . . 200

Gross Method—Perpetual Net Method—Perpetual

Accounts Receivable . . . . . . . . . . 500 Accounts Receivable . . . . . . . . . . . . . . 490 Sales . . . . . . . . . . . . . . . . . . 500 Sales . . . . . . . . . . . . . . . . . . . . . . 490

If the invoice is paid after the discount period, it records

Gross Method—Perpetual Net Method—Perpetual

Merchandise Inventory . . . . . . . . 500 Merchandise Inventory . . . . . . . . . . . . 490

Accounts Payable . . . . . . . . 500 Accounts Payable . . . . . . . . . . . . 490

SALES—Perpetual A company sells merchandise on November 2 at a $500 invoice price ($490 net) with terms of 2∕10, n∕30. The goods cost $200. Its November 2 entries are

If cash is received on (or before) November 12 within the discount period, it records Gross Method—Perpetual Net Method—Perpetual

Cash . . . . . . . . . . . . . . . . . . . . . . . 490 Cash . . . . . . . . . . . . . . . . . . . . . . . . . . . 490

Sales Discounts . . . . . . . . . . . . . . 10

Accounts Receivable . . . . . 500 Accounts Receivable . . . . . . . . . 490

Perpetual Inventory System PURCHASES—Perpetual A company purchases merchandise on November 2 at a $500 invoice price ($490 net) with terms of 2∕10, n∕30. Its November 2 entries under the gross and net methods are

If cash is received after the discount period, it records Gross Method—Perpetual Net Method—Perpetual

Cash . . . . . . . . . . . . . . . . . . . . . . . 500 Cash . . . . . . . . . . . . . . . . . . . . . . . . . . . 500 Interest Revenue . . . . . . . . . . . . 10

Accounts Receivable . . . . . 500 Accounts Receivable . . . . . . . . . 490

Periodic Inventory System PURCHASES—Periodic Under the periodic system, the balance of the Merchandise Inventory account remains unchanged during the period and is updated at period-end. During the period, three accounts are used to record purchases of inventory: Purchases; Purchases Discounts; and Purchases Returns and Allowances. The entries below are identical to the perpetual system except that Merchandise Inventory is substituted for each of the three purchases accounts. To demonstrate, we apply the periodic system to purchases transactions. On November 2, a buyer pur- chases goods ($500 gross; $490 net) with terms of 2∕10, n∕30. Its November 2 entries under the gross and net methods are

Gross Method—Periodic Net Method—Periodic

Purchases . . . . . . . . . . . . . . . . . . 500 Purchases . . . . . . . . . . . . . . . . . . . . . . 490 Accounts Payable . . . . . . . . 500 Accounts Payable . . . . . . . . . . . . 490

170 Chapter 4 Accounting for Merchandising Operations

If the invoice is paid on (or before) November 12 within the discount period, it records

Gross Method—Periodic Net Method—Periodic

Accounts Payable . . . . . . . . . . . . 500 Accounts Payable . . . . . . . . . . . . . . . . 490

Purchases Discounts . . . . . 10

Cash . . . . . . . . . . . . . . . . . . 490 Cash . . . . . . . . . . . . . . . . . . . . . . 490

Gross Method—Periodic Net Method—Periodic

Accounts Payable . . . . . . . . . . . . 500 Accounts Payable . . . . . . . . . . . . . . . . 490

Discounts Lost . . . . . . . . . . . . . . . . . . . 10

Cash . . . . . . . . . . . . . . . . . . 500 Cash . . . . . . . . . . . . . . . . . . . . . . 500

If the invoice is paid after the discount period, it records

SALES—Periodic For sales transactions, the perpetual and periodic entries are identical except that under the periodic system the cost-side entries are not made at the time of each sale nor for any subsequent returns. Instead, the cost of goods sold is computed at period-end based on a physical count of inventory. This entry is shown in Exhibit 4A.1.

APPENDIX

Work Sheet—Perpetual System4D This appendix along with assignments is available online.

MERCHANDISING ACTIVITIES Merchandise: Goods a company buys to resell. Cost of goods sold: Costs of merchandise sold. Gross profit (gross margin): Net sales minus cost of goods sold. Computing net income (service company vs. merchandiser):

EqualsMinusEqualsMinus Expenses Netincome

Net sales

Merchandiser

Expenses NetincomeRevenues

Service Company Minus Equals

Gross profit

Cost of goods sold

Inventory: Costs of merchandise owned, but not yet sold. It is a current asset on the balance sheet. Merchandise Cost Flows:

Net purchases

Merchandise available for sale

Cost of goods sold

Ending inventory

Beginning inventory

Perpetual inventory system: Updates accounting records for each pur- chase and each sale of inventory. Periodic inventory system: Updates accounting records for purchases and sales of inventory only at the end of a period.

Summary: Cheat Sheet

MERCHANDISING PURCHASES Cash discount: A purchases discount on the price paid by the buyer; or, a sales discount on amount received for the seller. Credit terms example: “2/10, n/60” means full payment is due within 60 days, but the buyer can deduct 2% of the invoice amount if payment is made within 10 days. Gross method: Initially record purchases at gross (full) invoice amounts. Purchasing Merchandise for Resale Entries:

Transportation Costs and Ownership Transfer Rules:

Purchasing merchandise Merchandise Inventory . . . . . . . . 500 on credit Accounts Payable . . . . . . . . 500

Ownership Transfers at

Goods in Transit Owned by

FOB shipping point Shipping point

Transportation Costs Paid byShipping Terms

FOB destination Destination

Buyer

Seller

Buyer Merchandise Inventory . . . # Cash . . . . . . . . . . . . . . . #

Seller Delivery Expense . . . . . . . . # Cash . . . . . . . . . . . . . . . . #

Paying within discount period Accounts Payable . . . . . . . . . . . . 500 (Inventory reduced by Merchandise Inventory . . . 10 discount taken) Cash . . . . . . . . . . . . . . . . . . 490

Paying outside discount Accounts Payable . . . . . . . . . . . . 500 period Cash . . . . . . . . . . . . . . . . . . 500

Recording purchases Cash or Accounts Payable . . . . . 30 returns or allowances Merchandise Inventory . . . 30

Chapter 4 Accounting for Merchandising Operations 171

MERCHANDISING SALES

Sales Discounts: A contra revenue account, meaning Sales Discounts is subtracted from Sales when computing net sales.

If goods are defective, Inventory is debited for estimated value. A loss is recorded for the difference between cost of merchandise and estimated value.

MERCHANDISER REPORTING Inventory shrinkage: An adjusting entry to account for the loss of inven- tory due to theft or deterioration. It is computed by comparing a physical count of inventory with recorded amounts.

Steps 3 and 4: Same entries as those for service companies. Multiple-step income statement: Three parts: (1) gross profit; (2) income from operations, which is gross profit minus operating expenses; and (3) net income, which is income from operations plus or minus nonoperating items. Operating expenses: Separated into selling expenses and general & administrative expenses. Selling expenses: Expenses of advertising merchandise, making sales, and delivering goods to customers. General & administrative expenses: Expenses that support a company’s overall operations, including accounting and human resources. Nonoperating activities: Consist of expenses, revenues, losses, and gains that are unrelated to a company’s main operations. Multiple-Step Income Statement Example

Closing Entries: Differences between merchandisers and service companies in red.

Step 1: Close Credit Balances Sales . . . . . . . . . . . . . . . . . . . . . . . . . 321,000 in Temporary Accounts to Income Summary . . . . . . . . . . . . . 321,000 Income Summary

Adjustment for shrinkage Cost of Goods Sold . . . . . . . . . . . . . . . 250 (occurs when recorded amount Merchandise Inventory . . . . . . . . . 250 larger than physical inventory)

Acid-test ratio (159) Allowance for Sales Discounts (167) Cash discount (146) Cost of goods sold (143) Credit memorandum (152) Credit period (146) Credit terms (145) Debit memorandum (147) Discount period (146)

Discounts Lost (168) EOM (145) FOB (148) General and administrative expenses (156) Gross margin (144) Gross margin ratio (159) Gross method (147, 168) Gross profit (144) Inventory (144)

Inventory Returns Estimated (166) List price (145) Merchandise (143) Merchandise inventory (144) Merchandiser (143) Multiple-step income statement (156) Net method (151, 168) Periodic inventory system (144) Perpetual inventory system (144)

Key Terms

Step 2: Close Debit Balances Income Summary . . . . . . . . . . . . . . . 308,100 in Temporary Accounts to Sales Discounts . . . . . . . . . . . . . 4,300 Income Summary Sales Returns and Allowances . . 2,000 Cost of Goods Sold . . . . . . . . . . . 230,400 Other Expenses . . . . . . . . . . . . . . 71,400

Selling merchandise Accounts Receivable . . . . . . . . . . 1,000 on credit Sales . . . . . . . . . . . . . . . . . . . . 1,000

Cost of Goods Sold . . . . . . . . . . . 300 Merchandise Inventory . . . . . 300

Receiving payment within Cash . . . . . . . . . . . . . . . . . . . . . . . 980 discount period Sales Discounts . . . . . . . . . . . . . . 20 Accounts Receivable . . . . . . . 1,000

Receiving payment outside Cash . . . . . . . . . . . . . . . . . . . . . . . 1,000 discount period Accounts Receivable . . . . . . . 1,000

Customer Merchandise Returns Entries:

Receiving sales returns of Sales Returns and Allowances . . 15 nondefective inventory Cash or Accounts Receivable 15

Merchandise Inventory . . . . . . . . 9 Cost of Goods Sold . . . . . . . . . 9

Receiving sales returns of Merchandise Inventory . . . . . . . . . . . 2 defective inventory Loss from Defective Merchandise . . . 7

Cost of Goods Sold . . . . . . . . . . . . 9

Recognizing sales Sales Returns and Allowances . . 10 allowances Cash or Accounts Receivable 10

Sales . . . . . . . . . . . . . . . . . . . . . . . . . . . . . . . . . . . . . . . . . . . . . $321,000 Less: Sales discounts . . . . . . . . . . . . . . . . . . . . . . . . . . . . . . . . $4,300 Sales returns and allowances . . . . . . . . . . . . . . . . . . . . . 2,000 6,300 Net sales . . . . . . . . . . . . . . . . . . . . . . . . . . . . . . . . . . . . . . . . 314,700 Cost of goods sold . . . . . . . . . . . . . . . . . . . . . . . . . . . . . . . . . . 230,400 Gross profit . . . . . . . . . . . . . . . . . . . . . . . . . . . . . . . . . . . . . . . 84,300 Operating Expenses Selling expenses† General and administrative expenses† Total operating expenses . . . . . . . . . . . . . . . . . . . . . . . . . . 71,400 Income from operations . . . . . . . . . . . . . . . . . . . . . . . . . . . . 12,900 Total other revenues and gains (expenses and losses) . . . . 2,000 Net income . . . . . . . . . . . . . . . . . . . . . . . . . . . . . . . . . . . . . . . $ 14,900

†Must list all individual expenses and amounts—see Exhibit 4.13 (not done here for brevity).

Single-Step Income Statement Example

Revenues Total revenues* . . . . . . . . . . . . . . . . . . . . . . . . . . . . . . . . . . . . . . . $318,200 Expenses Total expenses* . . . . . . . . . . . . . . . . . . . . . . . . . . . . . . . . . . . . . . . 303,300 Net income . . . . . . . . . . . . . . . . . . . . . . . . . . . . . . . . . . . . . . . . . . . . $ 14,900

*Must list all individual items and amounts—see Exhibit 4.14 (not done here for brevity).

Sales allowance: A price reduction agreed to with the buyer if they are unsatisfied with the goods.

172 Chapter 4 Accounting for Merchandising Operations

Purchases discount (146) Retailer (143) Sales discount (146) Sales Refund Payable (166)

Sales Returns and Allowances (151) Selling expenses (156) Shrinkage (153) Single-step income statement (157)

Supplementary records (149) Trade discount (145) Wholesaler (143)

Multiple Choice Quiz

1. A company has $550,000 in net sales and $193,000 in gross profit. This means its cost of goods sold equals a. $743,000. c. $357,000. e. $(193,000). b. $550,000. d. $193,000.

2. A company purchased $4,500 of merchandise on May 1 with terms of 2∕10, n∕30. On May 6, it returned $250 of that merchandise. On May 8, it paid the balance owed for merchandise, taking any discount it is entitled to. The cash paid on May 8 is a. $4,500. c. $4,160. e. $4,410. b. $4,250. d. $4,165.

3. A company has cash sales of $75,000, credit sales of $320,000, sales returns and allowances of $13,700, and sales discounts of $6,000. Its net sales equal

a. $395,000. c. $300,300. e. $414,700. b. $375,300. d. $339,700.

4. A company’s quick assets are $37,500, its current assets are $80,000, and its current liabilities are $50,000. Its acid-test ratio equals a. 1.600. c. 0.625. e. 0.469. b. 0.750. d. 1.333.

5. A company’s net sales are $675,000, its cost of goods sold is $459,000, and its net income is $74,250. Its gross margin ratio equals a. 32%. c. 47%. e. 34%. b. 68%. d. 11%.

ANSWERS TO MULTIPLE CHOICE QUIZ

1. c; Gross profit = $550,000 − $193,000 = $357,000 2. d; ($4,500 − $250) × (100% − 2%) = $4,165 3. b; Net sales = $75,000 + $320,000 − $13,700 − $6,000 = $375,300

4. b; Acid-test ratio = $37,500∕$50,000 = 0.75 5. a; Gross margin ratio = ($675,000 − $459,000)∕$675,000 = 32%

A(B,C) Superscript letter A, B, or C denotes assignments based on Appendix 4A, 4B, or 4C.

Icon denotes assignments that involve decision making.

1. What items appear in financial statements of merchan- dising companies but not in the statements of service companies?

2. In comparing the accounts of a merchandising company with those of a service company, what additional accounts would the merchandising company likely use, assuming it employs a perpetual inventory system?

3. Explain how a business can earn a positive gross profit on its sales and still have a net loss.

4. Why do companies offer a cash discount? 5. How does a company that uses a perpetual inventory system

determine the amount of inventory shrinkage? 6. Distinguish between cash discounts and trade discounts for

purchases. Is the amount of a trade discount on purchased merchandise recorded in the accounts?

7. What is the difference between a sales discount and a pur- chases discount?

8. Why would a company’s manager be concerned about the quantity of its purchases returns if its suppliers allow unlimited returns?

Discussion Questions

9. Does the sender (maker) of a debit memorandum record a debit or a credit in the recipient’s account? What entry (debit or credit) does the recipient record?

10. What is the difference between the single-step and multiple- step income statement formats?

11. Refer to Apple’s balance sheet and income statement in Appendix A. What does the com- pany title its inventory account? Does the company present a detailed calculation of its cost of goods sold?

12. Refer to Google’s income statement in Appendix A. What title does it use for cost of goods sold?

13. Refer to Samsung’s income statement in Appendix A. What does Samsung title its cost of goods sold account?

14. Refer to Samsung’s income statement in Appendix A. Does its income statement report a gross profit figure? If yes, what is the amount?

15. Buyers negotiate purchase contracts with suppliers. What type of shipping terms should a buyer attempt to negotiate to minimize freight-in costs?

APPLE

Samsung

Samsung

GOOGLE

Chapter 4 Accounting for Merchandising Operations 173

QUICK STUDY

QS 4-1 Applying merchandising terms

C1 P1

Enter the letter for each term in the blank space beside the definition that it most closely matches. A. Sales discount D. FOB destination G. Merchandise inventory B. Credit period E. FOB shipping point H. Purchases discount C. Discount period F. Gross profit

1. Goods a company owns and expects to sell to its customers. 2. Time period that can pass before a customer’s full payment is due. 3. Seller’s description of a cash discount granted to buyers in return for early payment. 4. Ownership of goods is transferred when the seller delivers goods to the carrier. 5. Purchaser’s description of a cash discount received from a supplier of goods. 6. Difference between net sales and the cost of goods sold. 7. Time period in which a cash discount is available. 8. Ownership of goods is transferred when delivered to the buyer’s place of business.

Costs of $5,000 were incurred to acquire goods and make them ready for sale. The goods were shipped to the buyer (FOB shipping point) for a cost of $200. Additional necessary costs of $400 were incurred to acquire the goods. No other incentives or discounts were available. What is the buyer’s total cost of mer- chandise inventory? a. $5,000 b. $5,200 c. $5,400 d. $5,600

QS 4-2 Identifying inventory costs

C2

Use the following information (in random order) from a merchandising company and from a service com- pany. Hint: Not all information may be necessary for the solutions. a. For the merchandiser only, compute (1) goods available for sale, (2) cost of goods sold, and (3) gross profit. b. Compute net income for each company.

QS 4-3 Merchandise accounts and computations

C2

Kleiner Merchandising Company

Accumulated depreciation . . . $ 700 Expenses . . . . . . . . $1,450

Beginning inventory . . . . . . . . 5,000 Net purchases . . . . 3,900

Ending inventory . . . . . . . . . . . 1,700 Net sales . . . . . . . . 9,500

Krug Service Company

Expenses . . . . . . . . . . . . $12,500 Prepaid rent . . . . . . . . $ 800

Revenues . . . . . . . . . . . . 14,000 Accounts payable . . . . 200

Cash . . . . . . . . . . . . . . . . 700 Equipment . . . . . . . . . . 1,300

Compute the amount to be paid for each of the four separate invoices assuming that all invoices are paid within the discount period.

Merchandise (gross) Terms Merchandise (gross) Terms a. $5,000 2∕10, n∕60 c. $75,000 1∕10, n∕30 b. $20,000 1∕15, EOM d. $10,000 3∕15, n∕45

QS 4-4 Computing net invoice amounts

P1

Prepare journal entries to record each of the following transactions of a merchandising company. The company uses a perpetual inventory system and the gross method.

Nov. 5 Purchased 600 units of product at a cost of $10 per unit. Terms of the sale are 2∕10, n∕60; the invoice is dated November 5.

7 Returned 25 defective units from the November 5 purchase and received full credit. 15 Paid the amount due from the November 5 purchase, minus the return on November 7.

QS 4-5 Recording purchases, returns, and discounts taken

P1

Prepare journal entries to record each of the following transactions. The company records purchases using the gross method and a perpetual inventory system.

Aug. 1 Purchased merchandise with an invoice price of $60,000 and credit terms of 3∕10, n∕30. 11 Paid supplier the amount owed from the August 1 purchase.

QS 4-6 Recording purchases and discounts taken

P1

Prepare journal entries to record each of the following transactions. The company records purchases using the gross method and a perpetual inventory system.

Sep. 15 Purchased merchandise with an invoice price of $35,000 and credit terms of 2∕5, n∕15. 29 Paid supplier the amount owed on the September 15 purchase.

QS 4-7 Recording purchases and discounts missed

P1

174 Chapter 4 Accounting for Merchandising Operations

QS 4-8 Recording sales, returns, and discounts taken

P2

Prepare journal entries to record each of the following sales transactions of a merchandising company. The company uses a perpetual inventory system and the gross method.

Apr. 1 Sold merchandise for $3,000, with credit terms n∕30; invoice dated April 1. The cost of the merchandise is $1,800.

4 The customer in the April 1 sale returned $300 of merchandise for full credit. The merchandise, which had cost $180, is returned to inventory.

8 Sold merchandise for $1,000, with credit terms of 1∕10, n∕30; invoice dated April 8. Cost of the merchandise is $700.

11 Received payment for the amount due from the April 1 sale less the return on April 4.

QS 4-10 Closing entries P3

Refer to QS 4-9 and prepare journal entries to close the balances in temporary revenue and expense ac- counts. Remember to consider the entry for shrinkage from QS 4-9.

QS 4-11 Multiple-step income statement

P4

For each item below, indicate whether the statement describes a multiple-step income statement or a single-step income statement. a. Multiple-step income statement b. Single-step income statement

1. Commonly reports detailed computations of net sales and other costs and expenses. 2. Statement limited to two main categories (revenues and expenses). 3. Reports gross profit on a separate line. 4. Separates income from operations from the other revenues and gains.

QS 4-9 Accounting for shrinkage— perpetual system

P3

Nix’It Company’s ledger on July 31, its fiscal year-end, includes the following selected accounts that have normal balances (Nix’It uses the perpetual inventory system).

Merchandise inventory . . . . . . . . . . $ 37,800 Sales returns and allowances . . . . . . . . $ 6,500

Retained earnings . . . . . . . . . . . . . . 115,300 Cost of goods sold . . . . . . . . . . . . . . . . . 105,000

Dividends . . . . . . . . . . . . . . . . . . . . . 7,000 Depreciation expense . . . . . . . . . . . . . . 10,300

Sales . . . . . . . . . . . . . . . . . . . . . . . . . 160,200 Salaries expense . . . . . . . . . . . . . . . . . . 32,500

Sales discounts . . . . . . . . . . . . . . . . 4,700 Miscellaneous expenses . . . . . . . . . . . . 5,000

A physical count of its July 31 year-end inventory discloses that the cost of the merchandise inventory still available is $35,900. Prepare the entry to record any inventory shrinkage.

Sales discounts . . . . . . . . . . . . . . . . . . . . $ 750 Office supplies expense . . . . . . . . . . . . . . . $ 500

Office salaries expense . . . . . . . . . . . . . 2,000 Cost of goods sold . . . . . . . . . . . . . . . . . . . 9,000

Rent expense—Office space . . . . . . . . . 1,500 Sales . . . . . . . . . . . . . . . . . . . . . . . . . . . . . . 20,000

Advertising expense . . . . . . . . . . . . . . . . 500 Insurance expense . . . . . . . . . . . . . . . . . . . 1,000

Sales returns and allowances . . . . . . . . 250 Sales staff salaries . . . . . . . . . . . . . . . . . . . 2,500

QS 4-12 Preparing a multiple-step income statement

P4

Save-the-Earth Co. reports the following income statement accounts for the year ended December 31. Prepare a multiple-step income statement that includes separate categories for net sales, cost of goods sold, selling expenses, and general and administrative expenses. Categorize the following accounts as sell- ing expenses: Sales Staff Salaries and Advertising Expense. Categorize the remaining expenses as general and administrative.

Buildings . . . . . . . . . . . . . . . . . . . . . . . . $25,000 Notes payable (due in 7 years) . . . . . . . . . . $30,000

Accounts receivable . . . . . . . . . . . . . . . 2,000 Office supplies . . . . . . . . . . . . . . . . . . . . . . . 1,000

Land . . . . . . . . . . . . . . . . . . . . . . . . . . . . 11,000 Common stock . . . . . . . . . . . . . . . . . . . . . . . 10,000

Merchandise inventory . . . . . . . . . . . . . 7,000 Retained earnings . . . . . . . . . . . . . . . . . . . . 6,000

Accounts payable . . . . . . . . . . . . . . . . . 5,000 Wages payable . . . . . . . . . . . . . . . . . . . . . . . 3,000

Cash . . . . . . . . . . . . . . . . . . . . . . . . . . . . 8,000

QS 4-13 Preparing a classified balance sheet for a merchandiser

P4

Clear Water Co. reports the following balance sheet accounts as of December 31. Prepare a classified bal- ance sheet.

Chapter 4 Accounting for Merchandising Operations 175

Compute net sales, gross profit, and the gross margin ratio for each of the four separate companies. Interpret the gross margin ratio for Carrier.

Carrier Lennox Trane York

Sales . . . . . . . . . . . . . . . . . . . . . . . . . . . . . . . . $150,000 $550,000 $38,700 $255,700

Sales discounts . . . . . . . . . . . . . . . . . . . . . . . . 5,000 17,500 600 4,800

Sales returns and allowances . . . . . . . . . . . . 20,000 6,000 5,100 900

Cost of goods sold . . . . . . . . . . . . . . . . . . . . . 79,750 329,589 24,453 126,500

QS 4-15 Computing and analyzing gross margin ratio

A2

Identify whether each description best applies to a periodic or a perpetual inventory system. a. Updates the inventory account only at period-end. b. Requires an adjusting entry to record inventory shrinkage. c. Returns immediately affect the account balance of Merchandise Inventory. d. Records cost of goods sold each time a sales transaction occurs. e. Provides more timely information to managers.

QS 4-16A Contrasting periodic and perpetual systems

P5

Refer to QS 4-5 and prepare journal entries to record each of the merchandising transactions assuming that the company records purchases using the gross method and a periodic inventory system.

QS 4-17A Recording purchases, returns, and discounts— periodic & gross methods P5

Refer to QS 4-8 and prepare journal entries to record each of the merchandising transactions assuming that the company records purchases using the gross method and a periodic inventory system.

QS 4-18A Recording sales, returns, and discounts—periodic & gross methods P5

ProBuilder has the following June 30 fiscal-year-end unadjusted balances: Allowance for Sales Discounts, $0; and Accounts Receivable, $10,000. Of the $10,000 of receivables, $2,000 are within a 3% discount period, meaning that it expects buyers to take $60 in future discounts arising from this period’s sales. a. Prepare the June 30 fiscal-year-end adjusting journal entry for future sales discounts. b. Assume the same facts above and that there is a $10 fiscal-year-end unadjusted credit balance in the

Allowance for Sales Discounts. Prepare the June 30 fiscal-year-end adjusting journal entry for future sales discounts.

QS 4-19B Recording estimates of future discounts

P6

QS 4-20B Recording estimates of future returns

P6

ProBuilder reports merchandise sales of $50,000 and cost of merchandise sales of $20,000 in its first year of operations ending June 30. It makes fiscal-year-end adjusting entries for estimated future returns and allowances equal to 2% of sales, or $1,000, and 2% of cost of sales, or $400. a. Prepare the June 30 fiscal-year-end adjusting journal entry for future returns and allowances related to sales. b. Prepare the June 30 fiscal-year-end adjusting journal entry for future returns and allowances related to

cost of sales.

QS 4-21C Recording purchases, returns, and discounts—net & perpetual methods P7

Refer to QS 4-5 and prepare journal entries to record each of the merchandising transactions assuming that the company records purchases using the net method and a perpetual inventory system.

QS 4-22C Recording sales, returns, and discounts—net & perpetual methods P7

Refer to QS 4-8 and prepare journal entries to record each of the merchandising transactions assuming that the company records purchases using the net method and a perpetual inventory system.

Use the following information on current assets and current liabilities to compute and interpret the acid- test ratio. Explain what the acid-test ratio of a company measures.

Cash . . . . . . . . . . . . . . . . . . . . . . . . . . . . $1,490 Prepaid expenses . . . . . . . . . . . . . . . . . . . $ 700

Accounts receivable . . . . . . . . . . . . . . . 2,800 Accounts payable . . . . . . . . . . . . . . . . . . . 5,750

Inventory . . . . . . . . . . . . . . . . . . . . . . . . 6,000 Other current liabilities . . . . . . . . . . . . . . . 850

QS 4-14 Computing and interpreting acid-test ratio

A1

176 Chapter 4 Accounting for Merchandising Operations

QS 4-23 Sales transactions

P2

Prepare journal entries to record each of the following sales transactions of EcoMart Merchandising. EcoMart uses a perpetual inventory system and the gross method. Oct. 1 Sold fair trade merchandise for $1,500, with credit terms n∕30, invoice dated October 1. The

cost of the merchandise is $900. 6 The customer in the October 1 sale returned $150 of fair trade merchandise for full credit. The

merchandise, which had cost $90, is returned to inventory. 9 Sold recycled leather merchandise for $700, with credit terms of 1∕10, n∕30, invoice dated

October 9. Cost of the merchandise is $450. 11 Received payment for the amount due from the October 1 sale less the return on October 6.

Exercise 4-2 Operating cycle for merchandiser

C2

The operating cycle of a merchandiser with credit sales includes the following five activities. Starting with merchandise acquisition, identify the chronological order of these five activities.

a. Prepare merchandise for sale. d. Purchase merchandise. b. Collect cash from customers on account. e. Monitor and service accounts receivable. c. Make credit sales to customers.

Exercise 4-4 Recording sales, sales returns, and sales allowances

P2

Allied Merchandisers was organized on May 1. Macy Co. is a major customer (buyer) of Allied (seller) products. Prepare journal entries to record the following transactions for Allied assuming it uses a per- petual inventory system and the gross method. May 3 Allied made its first and only purchase of inventory for the period on May 3 for 2,000 units at a

price of $10 cash per unit (for a total cost of $20,000). 5 Allied sold 1,500 of the units in inventory for $14 per unit (invoice total: $21,000) to Macy Co.

under credit terms 2∕10, n∕60. The goods cost Allied $15,000. 7 Macy returns 125 units because they did not fit the customer’s needs (invoice amount: $1,750).

Allied restores the units, which cost $1,250, to its inventory. 8 Macy discovers that 200 units are scuffed but are still of use and, therefore, keeps the units.

Allied gives a price reduction (allowance) and credits Macy’s accounts receivable for $300 to compensate for the damage.

15 Allied receives payment from Macy for the amount owed on the May 5 purchase; payment is net of returns, allowances, and any cash discount.

Exercise 4-3 Recording purchases, purchases returns, and purchases allowances

P1

Prepare journal entries to record the following transactions for a retail store. The company uses a perpet- ual inventory system and the gross method. Apr. 2 Purchased $4,600 of merchandise from Lyon Company with credit terms of 2∕15, n∕60, invoice

dated April 2, and FOB shipping point. 3 Paid $300 cash for shipping charges on the April 2 purchase. 4 Returned to Lyon Company unacceptable merchandise that had an invoice price of $600. 17 Sent a check to Lyon Company for the April 2 purchase, net of the discount and the returned mer-

chandise. 18 Purchased $8,500 of merchandise from Frist Corp. with credit terms of 1∕10, n∕30, invoice

dated April 18, and FOB destination. 21 After negotiations over scuffed merchandise, received from Frist a $500 allowance toward the

$8,500 owed on the April 18 purchase. 28 Sent check to Frist paying for the April 18 purchase, net of the allowance and the discount.

Check Apr. 28, Cr. Cash, $7,920

EXERCISES

Exercise 4-1 Computing revenues, expenses, and income

C1 C2

Fill in the blanks in the following separate income statements a through e. Identify any negative amount by putting it in parentheses.

a b c d e

Sales . . . . . . . . . . . . . . . . . . . . . . . . . . . . . . . . . . . . . . . . . . $62,000 $43,500 $46,000 $ ? $25,600 Cost of goods sold Merchandise inventory (beginning) . . . . . . . . . . . . . . . . 8,000 17,050 7,500 8,000 4,560 Total cost of merchandise purchases . . . . . . . . . . . . . . 38,000 ? ? 32,000 6,600 Merchandise inventory (ending) . . . . . . . . . . . . . . . . . . ? (3,000) (9,000) (6,600) ? Cost of goods sold . . . . . . . . . . . . . . . . . . . . . . . . . . . . 34,050 16,000 ? ? 7,000 Gross profit . . . . . . . . . . . . . . . . . . . . . . . . . . . . . . . . . . . . . ? ? 3,750 45,600 ? Expenses . . . . . . . . . . . . . . . . . . . . . . . . . . . . . . . . . . . . . . 10,000 10,650 12,150 3,600 6,000 Net income (loss) . . . . . . . . . . . . . . . . . . . . . . . . . . . . . . . . $ ? $16,850 $  (8,400) $42,000 $ ?

Chapter 4 Accounting for Merchandising Operations 177

Exercise 4-5 Recording purchases, purchases returns, and purchases allowances P1

Refer to Exercise 4-4 and prepare journal entries for Macy Co. to record each of the May transactions. Macy is a retailer that uses the gross method and a perpetual inventory system; it purchases these units for resale.

Exercise 4-7 Recording sales, purchases, shipping, and returns—buyer and seller

P1 P2

Sydney Retailing (buyer) and Troy Wholesalers (seller) enter into the following transactions. Both Sydney and Troy use a perpetual inventory system and the gross method. May 11 Sydney accepts delivery of $40,000 of merchandise it purchases for resale from Troy: invoice

dated May 11, terms 3∕10, n∕90, FOB shipping point. The goods cost Troy $30,000. Sydney pays $345 cash to Express Shipping for delivery charges on the merchandise.

12 Sydney returns $1,400 of the $40,000 of goods to Troy, who receives them the same day and restores them to its inventory. The returned goods had cost Troy $1,050.

20 Sydney pays Troy for the amount owed. Troy receives the cash immediately. 1. Prepare journal entries that Sydney Retailing (buyer) records for these three transactions. 2. Prepare journal entries that Troy Wholesalers (seller) records for these three transactions.

Check (1) May 20, Cr. Cash, $37,442

Exercise 4-8 Inventory and cost of sales transactions in T-accounts

P1 P2

The following summarizes Tesla’s merchandising activities for the year. Set up T-accounts for Merchandise Inventory and for Cost of Goods Sold. Enter each line item into one of the two T-accounts and compute the T-account balances.

Check Ending Merch. Inventory, $20,000

Cost of merchandise sold to customers . . . . . . . . . . . . . . . . . . . . . . . . . . . . . . . $196,000 Merchandise inventory, beginning-year . . . . . . . . . . . . . . . . . . . . . . . . . . . . . . . 25,000 Cost of merchandise purchases, gross amount . . . . . . . . . . . . . . . . . . . . . . . . . 192,500 Shrinkage on merchandise as of year-end . . . . . . . . . . . . . . . . . . . . . . . . . . . . . 800 Cost of transportation-in for merchandise purchases . . . . . . . . . . . . . . . . . . . . 2,900 Cost of merchandise returned by customers and restored to inventory . . . . . . 2,100 Discounts received from suppliers on merchandise purchases . . . . . . . . . . . . . 1,700 Returns to and allowances from suppliers on merchandise purchases . . . . . . 4,000

Exercise 4-9 Recording purchases, sales, returns, and shipping

P1 P2

Prepare journal entries for the following merchandising transactions of Dollar Store assuming it uses a perpetual inventory system and the gross method. Nov. 1 Dollar Store purchases merchandise for $1,500 on terms of 2∕5, n∕30, FOB shipping point,

invoice dated November 1. 5 Dollar Store pays cash for the November 1 purchase. 7 Dollar Store discovers and returns $200 of defective merchandise purchased on November 1,

and paid for on November 5, for a cash refund. 10 Dollar Store pays $90 cash for transportation costs for the November 1 purchase. 13 Dollar Store sells merchandise for $1,600 with terms n∕30. The cost of the merchandise is $800. 16 Merchandise is returned to the Dollar Store from the November 13 transaction. The returned items

are priced at $160 and cost $80; the items were not damaged and were returned to inventory.

Exercise 4-6 Recording sales, purchases, and cash discounts—buyer and seller

P1 P2

Santa Fe Retailing purchased merchandise “as is” (with no returns) from Mesa Wholesalers with credit terms of 3∕10, n∕60 and an invoice price of $24,000. The merchandise had cost Mesa $16,000. Assume that both buyer and seller use a perpetual inventory system and the gross method. 1. Prepare entries that the buyer records for the (a) purchase, (b) cash payment within the discount

period, and (c) cash payment after the discount period. 2. Prepare entries that the seller records for the (a) sale, (b) cash collection within the discount period,

and (c) cash collection after the discount period.

Exercise 4-10 Preparing adjusting and closing entries for a merchandiser

P3

The following list includes selected permanent accounts and all of the temporary accounts from the December 31 unadjusted trial balance of Emiko Co., a business owned by Kumi Emiko. Use these account balances along with the additional information to journalize (a) adjusting entries and (b) closing entries. Emiko Co. uses a perpetual inventory system.

[continued on next page]

Debit Credit

Merchandise inventory . . . . . . . . . . $30,000 Prepaid selling expenses . . . . . . . . . 5,600 Dividends . . . . . . . . . . . . . . . . . . . . . 33,000 Sales . . . . . . . . . . . . . . . . . . . . . . . . . $529,000 Sales returns and allowances . . . . . 17,500 Sales discounts . . . . . . . . . . . . . . . . . 5,000

Debit Credit

Cost of goods sold . . . . . . . . . . $212,000 Sales salaries expense . . . . . . . 48,000 Utilities expense . . . . . . . . . . . . 15,000 Selling expenses . . . . . . . . . . . . 36,000 Administrative expenses . . . . . 105,000

178 Chapter 4 Accounting for Merchandising Operations

Exercise 4-11 Computing net sales for multiple-step income statement

P4

A company reports the following sales-related information. Compute and prepare the net sales portion only of this company’s multiple-step income statement.

Sales, gross . . . . . . . . . . . . . . . . $200,000 Sales returns and allowances . . . . . . . . . . . . $16,000 Sales discounts . . . . . . . . . . . . . 4,000 Sales salaries expense . . . . . . . . . . . . . . . . . 10,000

Additional Information

Accrued and unpaid sales salaries amount to $1,700. Prepaid selling expenses of $3,000 have expired. A physical count of year-end merchandise inventory is taken to determine shrinkage and shows $28,700 of goods still available.

Check Dr. $84,500 to close Income Summary

Exercise 4-12 Impacts of inventory error on key accounts

P3

A retailer completed a physical count of ending merchandise inventory. When counting inventory, employ- ees did not include $3,000 of incoming goods shipped by a supplier on December 31 under FOB shipping point. These goods had been recorded in Merchandise Inventory, but they were not included in the physical count because they were in transit. This means shrinkage was incorrectly overstated by $3,000.

Compute the amount of overstatement or understatement for each of the following amounts for this period. a. Ending inventory b. Total assets c. Net income d. Total equity

Exercise 4-13 Physical count error and profits A2

Refer to the information in Exercise 4-12 and indicate whether the failure to include in-transit inventory as part of the physical count results in an overstatement, understatement, or no effect on the following ratios. a. Gross margin ratio b. Profit margin ratio c. Acid-test ratio d. Current ratio

Exercise 4-14 Computing and analyzing acid-test and current ratios

A1

Compute the current ratio and acid-test ratio for each of the following separate cases. (Round ratios to two decimals.) Which company is in the best position to meet short-term obligations? Explain.

Camaro GTO Torino

Cash . . . . . . . . . . . . . . . . . . . . . . . . . . . . $2,000 $      110 $1,000 Short-term investments . . . . . . . . . . . . 50 0 580 Current receivables . . . . . . . . . . . . . . . 350 470 700 Inventory . . . . . . . . . . . . . . . . . . . . . . . . 2,600 2,420 4,230 Prepaid expenses . . . . . . . . . . . . . . . . . 200 500 900 Total current assets . . . . . . . . . . . . . . . $5,200 $3,500 $7,410

Current liabilities . . . . . . . . . . . . . . . . . . $2,000 $1,000 $3,800

Exercise 4-15 Preparing a multiple-step income statement

P4

Fit-for-Life Foods reports the following income statement accounts for the year ended December 31. Prepare a multiple-step income statement that includes separate categories for net sales; cost of goods sold; selling expenses; general and administrative expenses; and other revenues, gains, expenses, and losses. Categorize the following accounts as selling expenses: Sales Staff Wages, Rent Expense—Selling Space, TV Advertising Expense, and Sales Commission Expense. Categorize the remaining expenses as general and administrative.

Gain on sale of equipment . . . . . . . . . . . . $ 6,250 Depreciation expense—Office copier . . . . . . . . . . . $ 500 Office supplies expense . . . . . . . . . . . . . . 700 Sales discounts . . . . . . . . . . . . . . . . . . . . . . . . . . . . 16,000 Insurance expense . . . . . . . . . . . . . . . . . . 1,300 Sales returns and allowances . . . . . . . . . . . . . . . . . 4,000 Sales . . . . . . . . . . . . . . . . . . . . . . . . . . . . . 220,000 TV advertising expense . . . . . . . . . . . . . . . . . . . . . . 2,000 Office salaries expense . . . . . . . . . . . . . . 32,500 Interest revenue . . . . . . . . . . . . . . . . . . . . . . . . . . . 750 Rent expense—Selling space . . . . . . . . . . 10,000 Cost of goods sold . . . . . . . . . . . . . . . . . . . . . . . . . . 90,000 Sales staff wages . . . . . . . . . . . . . . . . . . . 23,000 Sales commission expense . . . . . . . . . . . . . . . . . . . 13,000

Exercise 4-16 Preparing a classified balance sheet for a merchandiser

P4

Adams Co. reports the following balance sheet accounts as of December 31. Prepare a classified balance sheet.

Salaries payable . . . . . . . . . . . . . . . . . . . . $ 6,000 Retained earnings . . . . . . . . . . . . . . . . . . . . . . . . . . $50,000 Buildings . . . . . . . . . . . . . . . . . . . . . . . . . . 55,000 Notes payable (due in 9 years) . . . . . . . . . . . . . . . . 30,000 Prepaid rent . . . . . . . . . . . . . . . . . . . . . . . 7,000 Office supplies . . . . . . . . . . . . . . . . . . . . . . . . . . . . . 2,000 Merchandise inventory . . . . . . . . . . . . . . . 14,000 Land . . . . . . . . . . . . . . . . . . . . . . . . . . . . . . . . . . . . . 22,000 Accounts payable . . . . . . . . . . . . . . . . . . . 10,000 Accumulated depreciation—Building . . . . . . . . . . . 5,000 Prepaid insurance . . . . . . . . . . . . . . . . . . . 3,000 Mortgages payable (due in 5 years) . . . . . . . . . . . . 12,000 Accounts receivable . . . . . . . . . . . . . . . . . 4,000 Cash . . . . . . . . . . . . . . . . . . . . . . . . . . . . . . . . . . . . . 16,000 Common stock . . . . . . . . . . . . . . . . . . . . . 10,000

Chapter 4 Accounting for Merchandising Operations 179

Exercise 4-17A Recording purchases, returns, and allowances— periodic P5

Refer to Exercise 4-3 and prepare journal entries to record each of the merchandising transactions assum- ing that the buyer uses the periodic inventory system and the gross method.

Exercise 4-18A Recording sales, purchases, and discounts: buyer and seller—periodic P5

Refer to Exercise 4-6 and prepare journal entries to record each of the merchandising transactions assum- ing that the periodic inventory system and the gross method are used by both the buyer and the seller.

Exercise 4-19A Recording sales, purchases, shipping, and returns: buyer and seller—periodic P5

Refer to Exercise 4-7 and prepare journal entries to record each of the merchandising transactions assum- ing that the periodic inventory system and the gross method are used by both the buyer and the seller.

Exercise 4-20B Recording estimates of future discounts

P6

Med Labs has the following December 31 year-end unadjusted balances: Allowance for Sales Discounts, $0; and Accounts Receivable, $5,000. Of the $5,000 of receivables, $1,000 are within a 2% discount pe- riod, meaning that it expects buyers to take $20 in future-period discounts arising from this period’s sales. a. Prepare the December 31 year-end adjusting journal entry for future sales discounts. b. Assume the same facts above and that there is a $5 year-end unadjusted credit balance in Allowance for

Sales Discounts. Prepare the December 31 year-end adjusting journal entry for future sales discounts. c. Is Allowance for Sales Discounts a contra asset or a contra liability account?

Exercise 4-21B Recording estimates of future returns

P6

Chico Company allows its customers to return merchandise within 30 days of purchase. ∙ At December 31, the end of its first year of operations, Chico estimates future-period merchandise

returns of $60,000 (cost of $22,500) related to its current-year sales. ∙ A few days later, on January 3, a customer returns merchandise with a selling price of $2,000 for a cash

refund; the returned merchandise cost $750 and is returned to inventory as it is not defective. a. Prepare the December 31 year-end adjusting journal entry for estimated future sales returns and allow-

ances (revenue side). b. Prepare the December 31 year-end adjusting journal entry for estimated future inventory returns and

allowances (cost side). c. Prepare the January 3 journal entries to record the merchandise returned.

Exercise 4-22B Recording estimates of future returns

P6

Lopez Company reports unadjusted first-year merchandise sales of $100,000 and cost of merchandise sales of $30,000. a. Compute gross profit (using the unadjusted numbers above). b. The company expects future returns and allowances equal to 5% of sales and 5% of cost of sales. 1. Prepare the year-end adjusting entry to record the sales expected to be refunded. 2. Prepare the year-end adjusting entry to record the cost side of sales returns and allowances. 3. Recompute gross profit using the adjusted numbers from parts 1 and 2. c. Is Sales Refund Payable an asset, liability, or equity account? d. Is Inventory Returns Estimated an asset, liability, or equity account?

Exercise 4-23C Recording sales, purchases, shipping, and returns: buyer and seller—perpetual and net method P7

Refer to Exercise 4-7 and prepare journal entries to record each of the merchandising transactions assum- ing that the perpetual inventory system and the net method are used by both the buyer and the seller.

Exercise 4-24C Recording purchases, sales, returns, and discounts: buyer and seller—perpetual and both net & gross methods

P7

Piere Imports uses the perpetual system in accounting for merchandise inventory and had the following transactions during the month of October. Prepare entries to record these transactions assuming that Piere Imports records invoices (a) at gross amounts and (b) at net amounts.

Oct. 2 Purchased merchandise at a $3,000 price ($2,940 net), invoice dated October 2, terms 2∕10, n∕30. 10 Returned $500 ($490 net) of merchandise purchased on October 2 and debited its account pay-

able for that amount. 17 Purchased merchandise at a $5,400 price ($5,292 net), invoice dated October 17, terms 2∕10, n∕30. 27 Paid for the merchandise purchased on October 17, less the discount. 31 Paid for the merchandise purchased on October 2.

180 Chapter 4 Accounting for Merchandising Operations

Exercise 4-25 Purchasing transactions

P1

Prepare journal entries to record the following transactions of Recycled Fashion retail store. Recycled Fashion uses a perpetual inventory system and the gross method. Mar. 3 Purchased $1,150 of merchandise made from recycled material from GreenWorld Company

with credit terms of 2∕15, n∕60, invoice dated March 3, and FOB shipping point. 4 Paid $75 cash for shipping charges on the March 3 purchase. 5 Returned to GreenWorld unacceptable merchandise that had an invoice price of $150. 18 Paid GreenWorld for the March 3 purchase, net of the discount and the returned merchandise. 19 Purchased $425 of fair trade merchandise from PeopleFirst Corp. with credit terms of 1∕10,

n∕30, invoice dated March 19, and FOB destination. 21 After negotiations, received from PeopleFirst a $25 allowance (for scuffed merchandise) toward

the $425 owed on the March 19 purchase. 29 Sent check to PeopleFirst paying for the March 19 purchase, net of the allowance and the discount.

PROBLEM SET A

Problem 4-1A Preparing journal entries for merchandising activities—perpetual system

P1 P2

Prepare journal entries to record the following merchandising transactions of Cabela’s, which uses the perpetual inventory system and the gross method. Hint: It will help to identify each receivable and pay- able; for example, record the purchase on July 1 in Accounts Payable—Boden. July 1 Purchased merchandise from Boden Company for $6,000 under credit terms of 1∕15, n∕30,

FOB shipping point, invoice dated July 1. 2 Sold merchandise to Creek Co. for $900 under credit terms of 2∕10, n∕60, FOB shipping point,

invoice dated July 2. The merchandise had cost $500. 3 Paid $125 cash for freight charges on the purchase of July 1. 8 Sold merchandise that had cost $1,300 for $1,700 cash. 9 Purchased merchandise from Leight Co. for $2,200 under credit terms of 2∕15, n∕60, FOB

destination, invoice dated July 9. 11 Returned $200 of merchandise purchased on July 9 from Leight Co. and debited its account

payable for that amount. 12 Received the balance due from Creek Co. for the invoice dated July 2, net of the discount. 16 Paid the balance due to Boden Company within the discount period. 19 Sold merchandise that cost $800 to Art Co. for $1,200 under credit terms of 2∕15, n∕60, FOB

shipping point, invoice dated July 19. 21 Gave a price reduction (allowance) of $100 to Art Co. for merchandise sold on July 19 and

credited Art’s accounts receivable for that amount. 24 Paid Leight Co. the balance due, net of discount. 30 Received the balance due from Art Co. for the invoice dated July 19, net of discount. 31 Sold merchandise that cost $4,800 to Creek Co. for $7,000 under credit terms of 2∕10, n∕60,

FOB shipping point, invoice dated July 31.

July 24, Cr. Cash, $1,960 July 30, Dr. Cash, $1,078

Check July 12, Dr. Cash, $882 July 16, Cr. Cash, $5,940

Problem 4-2A Preparing journal entries for merchandising activities—perpetual system

P1 P2

Prepare journal entries to record the following merchandising transactions of Lowe’s, which uses the per- petual inventory system and the gross method. Hint: It will help to identify each receivable and payable; for example, record the purchase on August 1 in Accounts Payable—Aron. Aug. 1 Purchased merchandise from Aron Company for $7,500 under credit terms of 1∕10, n∕30, FOB

destination, invoice dated August 1. 5 Sold merchandise to Baird Corp. for $5,200 under credit terms of 2∕10, n∕60, FOB destination,

invoice dated August 5. The merchandise had cost $4,000. 8 Purchased merchandise from Waters Corporation for $5,400 under credit terms of 1∕10, n∕45,

FOB shipping point, invoice dated August 8. 9 Paid $125 cash for shipping charges related to the August 5 sale to Baird Corp. 10 Baird returned merchandise from the August 5 sale that had cost Lowe’s $400 and was sold for

$600. The merchandise was restored to inventory. 12 After negotiations with Waters Corporation concerning problems with the purchases on

August 8, Lowe’s received a price reduction from Waters of $400 off the $5,400 of goods pur- chased. Lowe’s debited accounts payable for $400.

14 At Aron’s request, Lowe’s paid $200 cash for freight charges on the August 1 purchase, reduc- ing the amount owed (accounts payable) to Aron.

15 Received balance due from Baird Corp. for the August 5 sale less the return on August 10. 18 Paid the amount due Waters Corporation for the August 8 purchase less the price allowance

from August 12.

Check Aug. 9, Dr. Delivery Expense, $125

Aug. 18, Cr. Cash, $4,950

[continued on next page]

Chapter 4 Accounting for Merchandising Operations 181

19 Sold merchandise to Tux Co. for $4,800 under credit terms of n∕10, FOB shipping point, in- voice dated August 19. The merchandise had cost $2,400.

22 Tux requested a price reduction on the August 19 sale because the merchandise did not meet specifications. Lowe’s gave a price reduction (allowance) of $500 to Tux and credited Tux’s accounts receivable for that amount.

29 Received Tux’s cash payment for the amount due from the August 19 sale less the price allow- ance from August 22.

30 Paid Aron Company the amount due from the August 1 purchase.

Aug. 29, Dr. Cash, $4,300

Problem 4-3A Computing merchandising amounts and formatting income statements

C2 P4

Valley Company’s adjusted trial balance on August 31, its fiscal year-end, follows. It categorizes the fol- lowing accounts as selling expenses: Sales Salaries Expense, Rent Expense—Selling Space, Store Supplies Expense, and Advertising Expense. It categorizes the remaining expenses as general and administrative.

Debit Credit

Merchandise inventory (ending) . . . . . . . . . . $ 41,000 Other (noninventory) assets . . . . . . . . . . . . . 130,400 Total liabilities . . . . . . . . . . . . . . . . . . . . . . . . . $ 25,000 Common stock . . . . . . . . . . . . . . . . . . . . . . . . 10,000 Retained earnings . . . . . . . . . . . . . . . . . . . . . . 94,550 Dividends . . . . . . . . . . . . . . . . . . . . . . . . . . . . 8,000 Sales . . . . . . . . . . . . . . . . . . . . . . . . . . . . . . . . 225,600 Sales discounts . . . . . . . . . . . . . . . . . . . . . . . . 2,250 Sales returns and allowances . . . . . . . . . . . . 12,000 Cost of goods sold . . . . . . . . . . . . . . . . . . . . . 74,500 Sales salaries expense . . . . . . . . . . . . . . . . . . 32,000 Rent expense—Selling space . . . . . . . . . . . . . 8,000 Store supplies expense . . . . . . . . . . . . . . . . . 1,500 Advertising expense . . . . . . . . . . . . . . . . . . . . 13,000 Office salaries expense . . . . . . . . . . . . . . . . . 28,500 Rent expense—Office space . . . . . . . . . . . . . 3,600 Office supplies expense . . . . . . . . . . . . . . . . . 400 Totals . . . . . . . . . . . . . . . . . . . . . . . . . . . . . . . . $355,150 $355,150

Beginning merchandise inventory was $25,400. Supplementary records of merchandising activities for the year ended August 31 reveal the following itemized costs.

Invoice cost of merchandise purchases . . . . . . . . . $92,000 Purchases returns and allowances . . . . . . . . . . $ 4,500

Purchases discounts received . . . . . . . . . . . . . . . . . 2,000 Costs of transportation-in . . . . . . . . . . . . . . . . . 4,600

Required

1. Compute the company’s net sales for the year. 2. Compute the company’s total cost of merchandise purchased for the year. 3. Prepare a multiple-step income statement that includes separate categories for net sales, cost of goods

sold, selling expenses, and general and administrative expenses. 4. Prepare a single-step income statement that includes these expense categories: cost of goods sold, sell-

ing expenses, and general and administrative expenses.

Check (2) $90,100

(3) Gross profit, $136,850; Net income, $49,850

(4) Total expenses, $161,500

Use the data for Valley Company in Problem 4-3A to complete the following requirement.

Required

Prepare closing entries as of August 31 (the perpetual inventory system is used).

Problem 4-4A Preparing closing entries and interpreting information about discounts and returns C2 P3

The following unadjusted trial balance is prepared at fiscal year-end for Nelson Company. Nelson Company uses a perpetual inventory system. It categorizes the following accounts as selling expenses: Depreciation Expense—Store Equipment, Sales Salaries Expense, Rent Expense—Selling Space, Store Supplies Expense, and Advertising Expense. It categorizes the remaining expenses as general and administrative.

Problem 4-5A Preparing adjusting entries and income statements; computing gross margin, acid-test, and current ratios

A1 A2 P3 P4

182 Chapter 4 Accounting for Merchandising Operations

Cash Merchandise inventory Store supplies Prepaid insurance Store equipment Accumulated depreciation—Store equipment Accounts payable Common stock Retained earnings

Dividends Sales

1,000 12,500 5,800 2,400

42,900

2,200

2,000 2,200

$

$

NELSON COMPANY Unadjusted Trial Balance

January 31 Debit Credit

169,200

15,250 10,000 5,000

27,000

111,950

$

Cost of goods sold Depreciation expense—Store equipment

Sales salaries expense O�ce salaries expense Insurance expense Rent expense—Selling space

Rent expense—O�ce space Store supplies expense

Advertising expense

Totals

38,400 0

17,500 17,500

0 7,500

7,500 0

9,800

169,200 $

Sales discounts Sales returns and allowances

Required

1. Prepare adjusting journal entries to reflect each of the following: a. Store supplies still available at fiscal year-end amount to $1,750. b. Expired insurance, an administrative expense, is $1,400 for the fiscal year. c. Depreciation expense on store equipment, a selling expense, is $1,525 for the fiscal year. d. To estimate shrinkage, a physical count of ending merchandise inventory is taken. It shows $10,900

of inventory is still available at fiscal year-end. 2. Prepare a multiple-step income statement for the year ended January 31 that begins with gross sales

and includes separate categories for net sales, cost of goods sold, selling expenses, and general and administrative expenses.

3. Prepare a single-step income statement for the year ended January 31. 4. Compute the current ratio, acid-test ratio, and gross margin ratio as of January 31. (Round ratios to

two decimals.)

Check (2) Gross profit, $67,750

(3) Total expenses, $106,775; Net income, $975

PROBLEM SET B

Problem 4-1B Preparing journal entries for merchandising activities—perpetual system

P1 P2

Prepare journal entries to record the following merchandising transactions of IKEA, which uses the per- petual inventory system and gross method. Hint: It will help to identify each receivable and payable; for example, record the purchase on May 2 in Accounts Payable—Havel.

May 2 Purchased merchandise from Havel Co. for $10,000 under credit terms of 1∕15, n∕30, FOB shipping point, invoice dated May 2.

4 Sold merchandise to Rath Co. for $11,000 under credit terms of 2∕10, n∕60, FOB shipping point, invoice dated May 4. The merchandise had cost $5,600.

5 Paid $250 cash for freight charges on the purchase of May 2. 9 Sold merchandise that had cost $2,000 for $2,500 cash. 10 Purchased merchandise from Duke Co. for $3,650 under credit terms of 2∕15, n∕60, FOB des-

tination, invoice dated May 10. 12 Returned $650 of merchandise purchased on May 10 from Duke Co. and debited its account

payable for that amount. 14 Received the balance due from Rath Co. for the invoice dated May 4, net of the discount. 17 Paid the balance due to Havel Co. within the discount period.

Check May 14, Dr. Cash, $10,780 May 17, Cr. Cash, $9,900

[continued on next page]

Chapter 4 Accounting for Merchandising Operations 183

Problem 4-2B Preparing journal entries for merchandising activities—perpetual system

P1 P2

Prepare journal entries to record the following merchandising transactions of Menards, which applies the perpetual inventory system and gross method. Hint: It will help to identify each receivable and payable; for example, record the purchase on July 3 in Accounts Payable—OLB.

July 3 Purchased merchandise from OLB Corp. for $15,000 under credit terms of 1∕10, n∕30, FOB destination, invoice dated July 3.

7 Sold merchandise to Brill Co. for $11,500 under credit terms of 2∕10, n∕60, FOB destination, invoice dated July 7. The merchandise had cost $7,750.

10 Purchased merchandise from Rupert Co. for $14,200 under credit terms of 1∕10, n∕45, FOB shipping point, invoice dated July 10.

11 Paid $300 cash for shipping charges related to the July 7 sale to Brill Co. 12 Brill returned merchandise from the July 7 sale that had cost Menards $1,450 and been sold for

$2,000. The merchandise was restored to inventory. 14 After negotiations with Rupert Co. concerning problems with the merchandise purchased on

July 10, Menards received a price reduction from Rupert of $1,200. Menards debited accounts payable for $1,200.

15 At OLB’s request, Menards paid $200 cash for freight charges on the July 3 purchase, reducing the amount owed (accounts payable) to OLB.

17 Received balance due from Brill Co. for the July 7 sale less the return on July 12. 20 Paid the amount due Rupert Co. for the July 10 purchase less the price reduction granted on July 14. 21 Sold merchandise to Brown for $11,000 under credit terms of 1∕10, n∕30, FOB shipping point,

invoice dated July 21. The merchandise had cost $7,000. 24 Brown requested a price reduction on the July 21 sale because the merchandise did not meet

specifications. Menards gave a price reduction (allowance) of $1,000 to Brown and credited Brown’s accounts receivable for that amount.

30 Received Brown’s cash payment for the amount due from the July 21 sale less the price allow- ance from July 24.

31 Paid OLB Corp. the amount due from the July 3 purchase.

Check July 17, Dr. Cash, $9,310

July 30, Dr. Cash, $9,900

July 31, Cr. Cash, $14,800

May 30, Dr. Cash, $2,450

20 Sold merchandise that cost $1,450 to Tamer Co. for $2,800 under credit terms of 2∕15, n∕60, FOB shipping point, invoice dated May 20.

22 Gave a price reduction (allowance) of $300 to Tamer Co. for merchandise sold on May 20 and credited Tamer’s accounts receivable for that amount.

25 Paid Duke Co. the balance due, net of the discount. 30 Received the balance due from Tamer Co. for the invoice dated May 20, net of discount and

allowance. 31 Sold merchandise that cost $3,600 to Rath Co. for $7,200 under credit terms of 2∕10, n∕60,

FOB shipping point, invoice dated May 31.

Problem 4-3B Computing merchandising amounts and formatting income statements

C1 C2 P4

Barkley Company’s adjusted trial balance on March 31, its fiscal year-end, follows. It categorizes the fol- lowing accounts as selling expenses: Sales Salaries Expense, Rent Expense—Selling Space, Store Supplies Expense, and Advertising Expense. It categorizes the remaining expenses as general and administrative.

Debit Credit

Merchandise inventory (ending) . . . . . . . . . . $ 56,500 Other (noninventory) assets . . . . . . . . . . . . . . 202,600 Total liabilities . . . . . . . . . . . . . . . . . . . . . . . . . $ 42,500 Common stock . . . . . . . . . . . . . . . . . . . . . . . . 10,000 Retained earnings . . . . . . . . . . . . . . . . . . . . . . 154,425 Dividends . . . . . . . . . . . . . . . . . . . . . . . . . . . . 3,000 Sales . . . . . . . . . . . . . . . . . . . . . . . . . . . . . . . . 332,650 Sales discounts . . . . . . . . . . . . . . . . . . . . . . . . 5,875 Sales returns and allowances . . . . . . . . . . . . 20,000 Cost of goods sold . . . . . . . . . . . . . . . . . . . . . 115,600 Sales salaries expense . . . . . . . . . . . . . . . . . . 44,500 Rent expense—Selling space . . . . . . . . . . . . . 16,000 Store supplies expense . . . . . . . . . . . . . . . . . 3,850 Advertising expense . . . . . . . . . . . . . . . . . . . . 26,000 Office salaries expense . . . . . . . . . . . . . . . . . 40,750 Rent expense—Office space . . . . . . . . . . . . . 3,800 Office supplies expense . . . . . . . . . . . . . . . . . 1,100 Totals . . . . . . . . . . . . . . . . . . . . . . . . . . . . . . . . $539,575 $539,575

184 Chapter 4 Accounting for Merchandising Operations

Beginning merchandise inventory was $37,500. Supplementary records of merchandising activities for the year ended March 31 reveal the following itemized costs.

Invoice cost of merchandise purchases . . . . . . . . . $138,500 Purchases returns and allowances . . . . . . . . . . . . $6,700

Purchases discounts received . . . . . . . . . . . . . . . . . 2,950 Costs of transportation-in . . . . . . . . . . . . . . . . . . . 5,750

Required

1. Compute the company’s net sales for the year. 2. Compute the company’s total cost of merchandise purchased for the year. 3. Prepare a multiple-step income statement that includes separate categories for net sales, cost of goods

sold, selling expenses, and general and administrative expenses. 4. Prepare a single-step income statement that includes these expense categories: cost of goods sold, sell-

ing expenses, and general and administrative expenses.

Check (2) $134,600

(3) Gross profit, $191,175; Net income, $55,175

(4) Total expenses, $251,600

Problem 4-4B Preparing closing entries and interpreting information about discounts and returns C2 P3

Use the data for Barkley Company in Problem 4-3B to complete the following requirement.

Required

Prepare closing entries as of March 31 (the perpetual inventory system is used).

Problem 4-5B Preparing adjusting entries and income statements; computing gross margin, acid-test, and current ratios

P3 P4 A1 A2

The following unadjusted trial balance is prepared at fiscal year-end for Foster Products Company. Foster Products Company uses a perpetual inventory system. It categorizes the following accounts as selling expenses: Depreciation Expense—Store Equipment, Sales Salaries Expense, Rent Expense—Selling Space, Store Supplies Expense, and Advertising Expense. It categorizes the remaining expenses as general and administrative.

Cash Merchandise inventory Store supplies Prepaid insurance Store equipment Accumulated depreciation—Store equipment Accounts payable Common stock

Dividends Sales

7,400 24,000

9,700 6,600

81,800

2,000

1,000 5,000

$

FOSTER PRODUCTS COMPANY Unadjusted Trial Balance

October 31 Debit Credit

$320,100

32,000 18,000 3,000

Retained earnings 40,000

227,100

$

Cost of goods sold Depreciation expense—Store equipment

Sales salaries expense O�ce salaries expense Insurance expense Rent expense—Selling space

Rent expense—O�ce space Store supplies expense Advertising expense

Totals

75,800 0

31,500 31,500

0 13,000

13,000 0

17,800

$320,100

Sales discounts Sales returns and allowances

Required

1. Prepare adjusting journal entries to reflect each of the following: a. Store supplies still available at fiscal year-end amount to $3,700. b. Expired insurance, an administrative expense, is $2,800 for the fiscal year. [continued on next page]

Chapter 4 Accounting for Merchandising Operations 185

Check (2) Gross profit, $142,600

(3) Total expenses, $197,100; Net income, $24,000

SERIAL PROBLEM Business Solutions

P1 P2 P3 P4

This serial problem began in Chapter 1 and continues through most of the book. If previous chapter seg- ments were not completed, the serial problem can begin at this point.

SP 4 Santana Rey created Business Solutions on October 1, 2019. The company has been successful, and its list of customers has grown. To accommodate the growth, the accounting system is modified to set up separate accounts for each customer. The following chart of accounts includes the account number used for each account and any balance as of December 31, 2019. Santana Rey decided to add a fourth digit with a decimal point to the 106 account number that had been used for the single Accounts Receivable account. This change allows the company to continue using the existing chart of accounts.

c. Depreciation expense on store equipment, a selling expense, is $3,000 for the fiscal year. d. To estimate shrinkage, a physical count of ending merchandise inventory is taken. It shows $21,300

of inventory is still available at fiscal year-end. 2. Prepare a multiple-step income statement for the year ended October 31 that begins with gross sales

and includes separate categories for net sales, cost of goods sold, selling expenses, and general and administrative expenses.

3. Prepare a single-step income statement for the year ended October 31. 4. Compute the current ratio, acid-test ratio, and gross margin ratio as of October 31. (Round ratios to

two decimals.)

No. Account Title Dr. Cr.

210 Wages payable . . . . . . . . . . . . . . . . . . . . . . . . . . . . . . . . $    500

236 Unearned computer services revenue . . . . . . . . . . . . . . 1,500

307 Common stock . . . . . . . . . . . . . . . . . . . . . . . . . . . . . . . . . 73,000

318 Retained earnings . . . . . . . . . . . . . . . . . . . . . . . . . . . . . . . 7,360

319 Dividends . . . . . . . . . . . . . . . . . . . . . . . . . . . . . . . . . . . . . $0

403 Computer services revenue . . . . . . . . . . . . . . . . . . . . . . 0

413 Sales . . . . . . . . . . . . . . . . . . . . . . . . . . . . . . . . . . . . . . . . . 0

414 Sales returns and allowances . . . . . . . . . . . . . . . . . . . . . 0

415 Sales discounts . . . . . . . . . . . . . . . . . . . . . . . . . . . . . . . . 0

502 Cost of goods sold . . . . . . . . . . . . . . . . . . . . . . . . . . . . . . 0

612 Depreciation expense—Office equipment . . . . . . . . . . . 0

613 Depreciation expense—Computer equipment . . . . . . . . 0

623 Wages expense . . . . . . . . . . . . . . . . . . . . . . . . . . . . . . . . 0

637 Insurance expense . . . . . . . . . . . . . . . . . . . . . . . . . . . . . 0

640 Rent expense . . . . . . . . . . . . . . . . . . . . . . . . . . . . . . . . . . 0

652 Computer supplies expense . . . . . . . . . . . . . . . . . . . . . . 0

655 Advertising expense . . . . . . . . . . . . . . . . . . . . . . . . . . . . 0

676 Mileage expense . . . . . . . . . . . . . . . . . . . . . . . . . . . . . . . 0

677 Miscellaneous expenses . . . . . . . . . . . . . . . . . . . . . . . . . 0

684 Repairs expense—Computer . . . . . . . . . . . . . . . . . . . . . 0

No. Account Title Dr. Cr.

101 Cash . . . . . . . . . . . . . . . . . . . . . . . . . . . . $48,372

106 .1 Alex’s Engineering Co . . . . . . . . . . . . . . . 0

106 .2 Wildcat Services . . . . . . . . . . . . . . . . . . . 0

106 .3 Easy Leasing . . . . . . . . . . . . . . . . . . . . . . 0

106 .4 IFM Co . . . . . . . . . . . . . . . . . . . . . . . . . . . 3,000

106 .5 Liu Corp . . . . . . . . . . . . . . . . . . . . . . . . . . 0

106 .6 Gomez Co . . . . . . . . . . . . . . . . . . . . . . . . 2,668

106 .7 Delta Co . . . . . . . . . . . . . . . . . . . . . . . . . . 0

106 .8 KC, Inc . . . . . . . . . . . . . . . . . . . . . . . . . . . 0

106 .9 Dream, Inc . . . . . . . . . . . . . . . . . . . . . . . . 0

119 Merchandise inventory . . . . . . . . . . . . . . 0

126 Computer supplies . . . . . . . . . . . . . . . . . 580

128 Prepaid insurance . . . . . . . . . . . . . . . . . 1,665

131 Prepaid rent . . . . . . . . . . . . . . . . . . . . . . 825

163 Office equipment . . . . . . . . . . . . . . . . . . 8,000

164 Accumulated depreciation— Office equipment . . . . . . . . . . . . . . . . $ 400

167 Computer equipment . . . . . . . . . . . . . . . 20,000

168 Accumulated depreciation— Computer equipment . . . . . . . . . . . . . 1,250

201 Accounts payable . . . . . . . . . . . . . . . . . . 1,100

In response to requests from customers, S. Rey will begin selling computer software. The company will extend credit terms of 1∕10, n∕30, FOB shipping point, to all customers who purchase this merchandise. However, no cash discount is available on consulting fees. Additional accounts (Nos. 119, 413, 414, 415, and 502) are added to its general ledger to accommodate the company’s new merchandising activities. Its transactions for January through March follow.

Jan. 4 The company paid cash to Lyn Addie for five days’ work at the rate of $125 per day. Four of the five days relate to wages payable that were accrued in the prior year.

5 Santana Rey invested an additional $25,000 cash in the company in exchange for more common stock.

7 The company purchased $5,800 of merchandise from Kansas Corp. with terms of 1∕10, n∕30, FOB shipping point, invoice dated January 7.

9 The company received $2,668 cash from Gomez Co. as full payment on its account. 11 The company completed a five-day project for Alex’s Engineering Co. and billed it $5,500,

which is the total price of $7,000 less the advance payment of $1,500. The company debited Unearned Computer Services Revenue for $1,500.

©Alexander Image/Shutterstock

186 Chapter 4 Accounting for Merchandising Operations

13 The company sold merchandise with a retail value of $5,200 and a cost of $3,560 to Liu Corp., invoice dated January 13.

15 The company paid $600 cash for freight charges on the merchandise purchased on January 7. 16 The company received $4,000 cash from Delta Co. for computer services provided. 17 The company paid Kansas Corp. for the invoice dated January 7, net of the discount. 20 The company gave a price reduction (allowance) of $500 to Liu Corp. and credited Liu’s ac-

counts receivable for that amount. 22 The company received the balance due from Liu Corp., net of the discount and the allowance. 24 The company returned defective merchandise to Kansas Corp. and accepted a credit against

future purchases (debited accounts payable). The defective merchandise invoice cost, net of the discount, was $496.

26 The company purchased $9,000 of merchandise from Kansas Corp. with terms of 1∕10, n∕30, FOB destination, invoice dated January 26.

26 The company sold merchandise with a $4,640 cost for $5,800 on credit to KC, Inc., invoice dated January 26.

31 The company paid cash to Lyn Addie for 10 days’ work at $125 per day. Feb. 1 The company paid $2,475 cash to Hillside Mall for another three months’ rent in advance. 3 The company paid Kansas Corp. for the balance due, net of the cash discount, less the $496

credit from merchandise returned on January 24. 5 The company paid $600 cash to Facebook for an advertisement to appear on February 5 only. 11 The company received the balance due from Alex’s Engineering Co. for fees billed on January 11. 15 The company paid a $4,800 cash dividend. 23 The company sold merchandise with a $2,660 cost for $3,220 on credit to Delta Co., invoice

dated February 23. 26 The company paid cash to Lyn Addie for eight days’ work at $125 per day. 27 The company reimbursed Santana Rey $192 cash for business automobile mileage. The com-

pany recorded the reimbursement as “Mileage Expense.” Mar. 8 The company purchased $2,730 of computer supplies from Harris Office Products on credit

with terms of n∕30, FOB destination, invoice dated March 8. 9 The company received the balance due from Delta Co. for merchandise sold on February 23. 11 The company paid $960 cash for minor repairs to the company’s computer. 16 The company received $5,260 cash from Dream, Inc., for computing services provided. 19 The company paid the full amount due of $3,830 to Harris Office Products, consisting of

amounts created on December 15 (of $1,100) and March 8. 24 The company billed Easy Leasing for $9,047 of computing services provided. 25 The company sold merchandise with a $2,002 cost for $2,800 on credit to Wildcat Services,

invoice dated March 25. 30 The company sold merchandise with a $1,048 cost for $2,220 on credit to IFM Company, in-

voice dated March 30. 31 The company reimbursed Santana Rey $128 cash for business automobile mileage. The com-

pany recorded the reimbursement as “Mileage Expense.”

The following additional facts are available for preparing adjustments on March 31 prior to financial state- ment preparation. a. The March 31 amount of computer supplies still available totals $2,005. b. Prepaid insurance coverage of $555 expired during this three-month period. c. Lyn Addie has not been paid for seven days of work at the rate of $125 per day. d. Prepaid rent of $2,475 expired during this three-month period. e. Depreciation on the computer equipment for January 1 through March 31 is $1,250. f. Depreciation on the office equipment for January 1 through March 31 is $400. g. The March 31 amount of merchandise inventory still available totals $704.

Required

1. Prepare journal entries to record each of the January through March transactions. 2. Post the journal entries in part 1 to the accounts in the company’s general ledger. Note: Begin with the

ledger’s post-closing adjusted balances as of December 31, 2019. 3. Prepare a 6-column work sheet (similar to the one shown in Exhibit 3.13) that includes the unadjusted

trial balance, the March 31 adjustments (a) through (g), and the adjusted trial balance. Do not prepare closing entries and do not journalize the adjustments or post them to the ledger.

Check (2) Ending balances at March 31: Cash, $68,057; Sales, $19,240 (3) Unadj. TB totals, $151,557; Adj. TB totals, $154,082

[continued on next page]

Chapter 4 Accounting for Merchandising Operations 187

GENERAL LEDGER PROBLEM

The General Ledger tool in Connect automates several of the procedural steps in the accounting cycle so that the accounting professional can focus on the impacts of each transaction on the various financial reports. The following General Ledger questions highlight the operating cycle of a merchandising com- pany. In each case, the trial balance is automatically updated from the journal entries recorded.

GL 4-1 Based on Problem 4-1A GL 4-3 Based on Problem 4-5A

GL 4-2 Based on Problem 4-2A

GL

4. Prepare an income statement (from the adjusted trial balance in part 3) for the three months ended March 31, 2020. (a) Use a single-step format. List all expenses without differentiating between selling expenses and general and administrative expenses. (b) Use a multiple-step format that begins with gross sales (service revenues plus gross product sales) and includes separate categories for net sales, cost of goods sold, selling expenses, and general and administrative expenses. Categorize the follow- ing accounts as selling expenses: Wages Expense, Mileage Expense, and Advertising Expense. Categorize the remaining expenses as general and administrative.

5. Prepare a statement of retained earnings (from the adjusted trial balance in part 3) for the three months ended March 31, 2020.

6. Prepare a classified balance sheet (from the adjusted trial balance) as of March 31, 2020. (6) Total assets, $120,268

COMPANY ANALYSIS A1

Accounting Analysis

AA 4-1 Refer to Apple’s financial statements in Appendix A to answer the following.

Required

1. Assume that the amounts reported for inventories and cost of sales reflect items purchased in a form ready for resale. Compute the net cost of goods purchased for the year ended September 30, 2017.

2. Compute the current ratio and acid-test ratio as of September 30, 2017, and September 24, 2016. 3. Does Apple’s 2017 current ratio outperform or underperform the (assumed) industry average of 1.5? 4. Does Apple’s 2017 acid-test ratio outperform or underperform the (assumed) industry average of 1.0?

APPLE

AA 4-2 Key comparative figures for Apple and Google follow.

Required

1. Compute the amount of gross margin and the gross margin ratio for the two years shown for each of these companies.

2. Which company earns more in gross margin for each dollar of net sales for the current year? 3. Do (a) Apple’s and (b) Google’s current-year gross margins underperform or outperform the industry

(assumed) average of 35.0%? 4. Are (a) Apple’s and (b) Google’s current-year gross margins on a favorable or unfavorable trend?

Apple Google

$ millions Current Year Prior Year Current Year Prior Year

Net sales . . . . . . . . . . . . . . . $229,234 $215,639 $110,855 $90,272

Cost of sales . . . . . . . . . . . . . 141,048 131,376 45,583 35,138

COMPARATIVE ANALYSIS A2

APPLE GOOGLE

AA 4-3 Key comparative figures for Samsung, Apple, and Google follow.

In millions Net Sales Cost of Sales

Samsung . . . . . . . . . . . . . . W239,575,376 W129,290,661

Apple . . . . . . . . . . . . . . . . . $ 229,234 $ 141,048

Google . . . . . . . . . . . . . . . . $ 110,855 $ 45,583

GLOBAL ANALYSIS A2 P4

APPLE GOOGLE Samsung

(4) Net income, $18,833

188 Chapter 4 Accounting for Merchandising Operations

ETHICS CHALLENGE C1 P2

BTN 4-1 Amy Martin is a student who plans to attend approximately four professional events a year at her college. Each event necessitates a financial outlay of $100 to $200 for a new suit and accessories. After incurring a major hit to her savings for the first event, Amy developed a different approach. She buys the suit on credit the week before the event, wears it to the event, and returns it the next week to the store for a full refund on her charge card.

Required

1. Comment on the ethics exhibited by Amy and possible consequences of her actions. 2. How does the merchandising company account for the suits that Amy returns?

Beyond the Numbers

Required

1. Compute the gross margin ratio for each of the three companies. 2. Is Samsung’s gross margin ratio better or worse than (a) Apple’s ratio? (b) Google’s? 3. Do (a) Apple, (b) Google, and (c) Samsung use single-step or multiple-step income statements?

BTN 4-2 You are the financial officer for Music Plus, a retailer that sells goods for home entertainment needs. The business owner, Vic Velakturi, recently reviewed the annual financial statements you prepared and sent you an e-mail stating that he thinks you overstated net income. He explains that although he has invested a great deal in security, he is sure shoplifting and other forms of inventory shrinkage have occurred, but he does not see any deduction for shrinkage on the income statement. The store uses a per- petual inventory system.

Required

Prepare a brief memorandum that responds to the owner’s concerns.

COMMUNICATING IN PRACTICE C2 P3 P5

BTN 4-3 Access the SEC’s EDGAR database (SEC.gov) and obtain the March 21, 2017, filing of its fiscal 2017 10-K report (for year ended January 28, 2017) for J. Crew Group, Inc. (ticker: JCG).

Required

Prepare a table that reports the gross margin ratios for J. Crew using the revenues and cost of goods sold data from J. Crew’s income statement for each of its most recent three years. Analyze and comment on the trend in its gross margin ratio.

TAKING IT TO THE NET C1 A2

BTN 4-4 Official Brands’s general ledger and supplementary records at the end of its current period reveal the following.

TEAMWORK IN ACTION C1 C2

Sales, gross . . . . . . . . . . . . . . . . . . . . $600,000 Merchandise inventory (beginning of period) . . . . . . . $ 98,000

Sales returns & allowances . . . . . . . . 20,000 Invoice cost of merchandise purchases . . . . . . . . . . . . 360,000

Sales discounts . . . . . . . . . . . . . . . . . 13,000 Purchases discounts received . . . . . . . . . . . . . . . . . . . 9,000

Cost of transportation-in . . . . . . . . . . 22,000 Purchases returns and allowances . . . . . . . . . . . . . . . . 11,000

Operating expenses . . . . . . . . . . . . . 50,000 Merchandise inventory (end of period) . . . . . . . . . . . . 84,000

Required

1. Each member of the team is to assume responsibility for computing one of the following items. You are not to duplicate your teammates’ work. Get any necessary amounts to compute your item from the appropriate teammate. Each member is to explain his or her computation to the team in preparation for reporting to the class.

a. Net sales d. Gross profit b. Total cost of merchandise purchases e. Net income c. Cost of goods sold 2. Check your net income with the instructor. If correct, proceed to step 3. 3. Assume that a physical inventory count finds that actual ending inventory is $76,000. Discuss how this

affects previously computed amounts in step 1.

Point: In teams of four, assign the same student a and e. Rotate teams for reporting on a different computation and the analysis in step 3.

Chapter 4 Accounting for Merchandising Operations 189

BTN 4-5 Refer to the opening feature about Build-A-Bear Workshop and its founder Maxine Clark. Assume the business reports current annual sales at approximately $1 million and prepares the following income statement.

ENTREPRENEURIAL DECISION C1 C2 P1

BUILD-A-BEAR WORKSHOP Income Statement

For Year Ended January 31, 2018

Net sales . . . . . . . . . . . . . . . . . . . . . . . . . . . . . . . . . . $1,000,000

Cost of sales . . . . . . . . . . . . . . . . . . . . . . . . . . . . . . . . 610,000

Expenses (other than cost of sales) . . . . . . . . . . . . . 200,000

Net income . . . . . . . . . . . . . . . . . . . . . . . . . . . . . . . . . $      190,000

Assume the business sells to individuals and retailers, ranging from small shops to large chains. Assume that they currently offer credit terms of 1∕15, n∕60, and ship FOB destination. To improve their cash flow, they are considering changing credit terms to 3∕10, n∕30. In addition, they propose to change shipping terms to FOB shipping point. They expect that the increase in discount rate will increase net sales by 9%, but the gross margin ratio (and ratio of cost of sales divided by net sales) is expected to remain unchanged. They also expect that delivery expenses will be zero under this proposal; thus, expenses other than cost of sales are expected to increase only 6%.

Required

1. Prepare a forecasted income statement for the year ended January 31, 2019, based on the proposal. 2. Based on the forecasted income statement alone (from your part 1 solution), do you recommend that

the business implement the new sales policies? Explain. 3. What else should the business consider before deciding whether to implement the new policies?

Explain.

BTN 4-6 Arrange an interview (in person or by phone) with the manager of a retail shop in a mall or in the downtown area of your community. Explain to the manager that you are a student studying merchan- dising activities and the accounting for sales returns and sales allowances. Ask the manager what the store policy is regarding returns. Also find out if sales allowances are ever negotiated with customers. Inquire whether management perceives that customers are abusing return policies and what actions management takes to counter potential abuses. Be prepared to discuss your findings in class.

HITTING THE ROAD C1 P2

Point: This activity complements the Ethics Challenge assignment.

Design elements: Lightbulb: ©Chuhail/Getty Images; Blue globe: ©nidwlw/Getty Images and ©Dizzle52/Getty Images; Chess piece: ©Andrei Simonenko/Getty Images and ©Dizzle52/Getty Images; Mouse: ©Siede Preis/Getty Images; Global View globe: ©McGraw-Hill Education and ©Dizzle52/Getty Images; Sustainability: ©McGraw-Hill Education and ©Dizzle52/Getty Images

Learning Objectives

CONCEPTUAL C1 Identify the items making up

merchandise inventory.

C2 Identify the costs of merchandise inventory.

ANALYTICAL A1 Analyze the effects of inventory

methods for both financial and tax reporting.

P2 Compute the lower of cost or market amount of inventory.

P3 Appendix 5A—Compute inventory in a periodic system using the methods of specific identification, FIFO, LIFO, and weighted average.

P4 Appendix 5B—Apply both the retail inventory and gross profit methods to estimate inventory.

A2 Analyze the effects of inventory errors on current and future financial statements.

A3 Assess inventory management using both inventory turnover and days’ sales in inventory.

PROCEDURAL P1 Compute inventory in a perpetual

system using the methods of specific identification, FIFO, LIFO, and weighted average.

Chapter Preview

5 Inventories and Cost of Sales

NTK 5-3, 5-4

INVENTORY VALUATION, ERRORS, AND ANALYSIS

P2 Lower of cost or market A2 Effects of inventory errors A3 Inventory management P3 Appendix: Periodic system P4 Appendix: Inventory

estimation

NTK 5-1

INVENTORY BASICS

C1 Determining inventory items C2 Determining inventory costs

Control of inventory

Physical count

NTK 5-2

INVENTORY COSTING

P1 Cost flow assumptions: Specific identification

First-in, first-out

Last-in, first-out

Weighted average

A1 Effects on financial statements

191

“Show guests you care”—Danny Meyer

Shake It Up

NEW YORK—Danny Meyer opened his first Shake Shack (ShakeShack.com) restaurant in Madison Square Park. The first Shake Shack was a hot dog stand! While much has changed since the first Shack, Danny’s commitment to high-quality ingre- dients has not.

“We call it fine-casual,” explains Danny. “Shake Shack . . . is proving that people don’t want to go backwards in terms of how their food was sourced, how it was cooked.”

Managing this “modern-day roadside burger stand” was not easy. Danny’s Shack grew from “$5,000 worth of hamburgers” to “$30,000-plus” of hamburgers per day. Danny needed an accounting system to track everything.

“The thinking back then was, to have a successful restaurant, the owner had to be there 24/7,” says Danny. To expand Shake Shack, that had to change. Danny put in an inventory system for each of his Shacks. “Great companies,” insists Danny, “figured [inventory] out.”

To ensure fresh sourced ingredients were available at the Shacks, Danny set up an inventory tracking system. He pre- pared and read inventory reports and applied inventory man- agement tools. His inventory system tracks all transactions, and he regularly reviews accounting data in making key decisions.

“You need to get your ducks in a line,” asserts Danny. This means that Shake Shack must successfully manage its inven- tory, even as growth continues.

To be successful, Danny insists that “the numbers add up.” Once your financial house is in order, explains Danny, “you need to take more risk.” He adds, “The best start-ups are businesses that find a unique way to solve problems for people—sometimes problems that people didn’t even know they had.”

Sources: Shake Shack website, January 2019; Fool.com, December 2016; Eater.com, September 2016; Inc.com, May 2015

©Monica Schipper/NYCWFF/Getty Images

Determining Inventory Items Merchandise inventory includes all goods that a company owns and holds for sale. This is true regardless of where the goods are located when inventory is counted. Special atten- tion is directed at goods in transit, goods on consignment, and goods that are damaged or obsolete.

Goods in Transit Does a buyer’s inventory include goods in transit from a supplier? If ownership has passed to the buyer, the goods are included in the buyer’s inventory. We deter- mine this by reviewing shipping terms. FOB shipping point—goods are included in buyer’s inventory once they are shipped. FOB destination—goods are included in buyer’s inventory after arrival at their destination.

Goods on Consignment Goods on consignment are goods shipped by the owner, called the consignor, to another party, the consignee. A consignee sells goods for the owner. The consignor owns the consigned goods and reports them in its inventory. For example, Upper Deck pays sports celebrities such as Russell Wilson of the Seattle Seahawks to sign memorabilia, which are offered to card shops on consignment. Upper Deck, the consignor, reports these items in its inventory until sold. The consignee never reports consigned goods in inventory.

Goods Damaged or Obsolete Damaged, obsolete (out-of-date), and deteriorated goods are not reported in inventory if they cannot be sold. If these goods can be sold at a

INVENTORY BASICS

C1 Identify the items making up merchandise inventory.

192 Chapter 5 Inventories and Cost of Sales

Eyes in the Sky One of the largest builders, Homex, was accused of faking the construction and sale of 100,000 homes. How were they caught? When the SEC used satellite imagery to confirm the existence of homes, they found nothing but bare soil. SEC 2017-60 ■

Ethical Risk

©Aleksandar Georgiev/Getty Images

Determining Inventory Costs Merchandise inventory includes costs to bring an item to a salable condition and location. Inventory costs include invoice cost minus any discount, plus any other costs. Other costs include shipping, storage, import duties, and insurance. The expense recognition principle says that inventory costs are expensed as cost of goods sold when inventory is sold.

Internal Controls and Taking a Physical Count Events can cause the Inventory account balance to be different than the actual inventory avail- able. Such events include theft, loss, damage, and errors. Thus, nearly all companies take a physical count of inventory at least once each year. This physical count is used to adjust the Inventory account balance to the actual inventory available.

C2 Identify the costs of merchandise inventory.

Fraud: Auditors observe employ- ees as they count inventory. Auditors also take their own count to ensure accuracy.

In Control A company applies internal controls when taking a physical count of inventory that usually include the following to minimize fraud and to increase reliability.

• Prenumbered inventory tickets are distributed to counters—each ticket must be accounted for. • Counters of inventory are assigned and do not include those responsible for inventory. • Counters confirm the existence, amount, and condition of inventory. • A second count is taken by a different counter. • A manager confirms all inventories are ticketed once, and only once. ■

Decision Insight

Point: The Inventory account has subsidiary ledgers that contain a separate record (units and costs) for each separate product.

1. A master carver of wooden birds operates her business out of a garage. At the end of the current period, the carver has 17 units (carvings) in her garage, 3 of which were damaged by water and cannot be sold. She also has another 5 units in her truck, ready to deliver per a customer order, terms FOB destination, and another 11 units out on consignment at retail stores. How many units does she include in the busi- ness’s period-end inventory?

2. A distributor of artistic iron-based fixtures acquires a piece for $1,000, terms FOB shipping point. Additional costs in obtaining it and offering it for sale include $150 for transportation-in, $300 for import duties, $100 for insurance during shipment, $200 for advertising, a $50 voluntary gratuity to the delivery person, $75 for enhanced store lighting, and $250 for sales staff salaries. For computing inventory, what cost is assigned to this artistic piece?

Solutions

1. 2.

Inventory Items and Costs

NEED-TO-KNOW 5-1

C1 C2

Do More: QS 5-1, QS 5-2, QS 5-23, E 5-1, E 5-2

Units in ending inventory

Units in storage . . . . . . . . . . . . . . . . . . . . . . . . . 17 units

Less damaged (unsalable) units . . . . . . . . . . . . (3)

Plus units in transit . . . . . . . . . . . . . . . . . . . . . . 5

Plus units on consignment . . . . . . . . . . . . . . . . 11

Total units in ending inventory . . . . . . . . . . . . . 30 units

Merchandise cost . . . . . . . . . . . . . . . . $1,000

Plus:

Transportation-in . . . . . . . . . . . . . . 150

Import duties . . . . . . . . . . . . . . . . . 300

Insurance . . . . . . . . . . . . . . . . . . . . 100

Total inventory cost . . . . . . . . . . . . . . . $1,550

lower price, they are included in inventory at net realizable value. Net realizable value is sales price minus the cost of making the sale. A loss is recorded when the damage or obso- lescence occurs.

Chapter 5 Inventories and Cost of Sales 193

INVENTORY COSTING UNDER A PERPETUAL SYSTEM

*Includes specific identification.

FIFO 48%

LIFO 28%

Weighted Average 20%

Other* 4%

EXHIBIT 5.1 Frequency in Use of Inventory Methods

When identical items are purchased at different costs, we must decide which amounts to record in cost of goods sold and which amounts remain in inventory. Four methods are used to assign costs to inventory and to cost of goods sold: (1) specific identification; (2) first-in, first-out (FIFO); (3) last-in, first-out (LIFO); and (4) weighted average. Exhibit 5.1 shows the frequency in use of these methods.

Each method has a pattern for how costs flow through inventory. The cost flow assumption does not have to match the actual physical flow of goods. For example, Kroger’s grocery chain sells food first-in, first-out, meaning they sell the oldest food in inventory first. However, Kroger can use last-in, first-out to assign costs to food sold. With the exception of specific identifica- tion, the physical flow and cost flow do not have to be the same.

Inventory Cost Flow Assumptions To show inventory cost flow assumptions, assume that three identical units are purchased separately at the following three dates and costs: May 1 at $45, May 3 at $65, and May 6 at $70. One unit is then sold on May 7 for $100. Exhibit 5.2 shows the flow of costs to either cost of goods sold on the income statement or inventory reported on the balance sheet for FIFO, LIFO, and weighted average.

Point: Cost of goods sold is abbre- viated COGS.

$180 3

= $60 each

2. Last-in, first-out (LIFO) Costs flow in the reverse

order incurred.

3. Weighted average Costs flow at an average

of costs available.

1. First-in, first-out (FIFO) Costs flow in the order

incurred.

× 2× 1

Income Statement Net sales.................... $100 Cost of goods sold.. 45 Gross profit................ $ 55

Balance Sheet Inventory.................... $135

Income Statement Net sales.................... $100 Cost of goods sold.. 70 Gross profit................ $ 30

Balance Sheet Inventory.................... $1 10

Income Statement Net sales.................... $100 Cost of goods sold.. 60 Gross profit................ $ 40

Balance Sheet Inventory.................... $120

$65 May 3

$45 May 1 G

o o d s

s o l d

G o o d s

s o l d

G o o d s

s o l d

G o o d s

l e f t

G o o d s

l e f t

G o o d s

l e f t

$70 May 6

$65 May 3

$45 May 1

$70 May 6

$70 May 6

$65 May 3

$45 May 1

EXHIBIT 5.2 Cost Flow Assumptions

(1) FIFO assumes costs flow in the order incurred. The unit purchased on May 1 for $45 is the earliest cost incurred—it is sent to cost of goods sold on the income statement first. The remaining two units ($65 and $70) are reported in inventory on the balance sheet.

(2) LIFO assumes costs flow in the reverse order incurred. The unit purchased on May 6 for $70 is the most recent cost incurred—it is sent to cost of goods sold on the income statement. The remaining two units ($45 and $65) are reported in inventory on the balance sheet.

(3) Weighted average assumes costs flow at an average of the costs available. The units available at the May 7 sale average $60 in cost, computed as ($45 + $65 + $70)/3. One unit’s $60 average cost is sent to cost of goods sold on the income statement. The remaining two units’ average costs are reported in inventory at $120 on the balance sheet.

Cost flow assumptions impact gross profit and inventory numbers. Exhibit 5.2 shows that gross profit ranges from $30 to $55 due to the cost flow assumption.

Point: Recall inventory cost flow.

+

= Merchandise available for sale

Cost of goods sold

Net purchases

Beginning inventory

Ending inventory

+

The following sections on inventory costing use the perpetual system. Appendix 5A uses the periodic system. An instructor can choose to cover either one or both systems. If the perpetual system is skipped, then read Appendix 5A and return to the “Valuing Inventory at LCM and the Effects of Inventory Errors” section.

194 Chapter 5 Inventories and Cost of Sales

Inventory Costing Illustration This section demonstrates inventory costing methods. We use information from Trekking, a sporting goods store. Among its products, Trekking sells one type of mountain bike whose sales are directed at resorts that provide inexpensive bikes for guest use. We use Trekking’s data from August. Its mountain bike (unit) inventory at the beginning of August and its purchases and sales during August are in Exhibit 5.3. It ends August with 12 bikes in inventory.

P1 Compute inventory in a perpetual system using the methods of specific identification, FIFO, LIFO, and weighted average.

EXHIBIT 5.3 Purchases and Sales of Goods

Date Activity Units Acquired at Cost Units Sold at Retail Unit Inventory

Aug . 1 Beginning inventory . . . . . . . 10 units @ $ 91 = $ 910 10 units Aug . 3 Purchases . . . . . . . . . . . . . . . 15 units @ $106 = $ 1,590 25 units Aug . 14 Sales . . . . . . . . . . . . . . . . . . . 20 units @ $130 5 units

Aug . 17 Purchases . . . . . . . . . . . . . . . 20 units @ $115 = $ 2,300 25 units Aug . 28 Purchases . . . . . . . . . . . . . . . 10 units @ $119 = $ 1,190 35 units Aug . 30 Sales . . . . . . . . . . . . . . . . . . . 23 units @ $150 12 units Totals . . . . . . . . . . . . . . . . . . 55 units $5,990 43 units

Units available for sale Goods available for sale Units sold Units left

Trekking uses the perpetual inventory system, which means that its Merchandise Inventory account is updated for each purchase and sale of inventory. (Appendix 5A describes the assign- ment of costs to inventory using a periodic system.) Regardless of what inventory method is used, cost of goods available for sale must be allocated between cost of goods sold and ending inventory.

Specific Identification When each item in inventory can be matched with a specific purchase and invoice, we can use specific identification or SI to assign costs. We also need sales records that identify exactly which items were sold and when. Trekking’s internal documents show the following specific unit sales.

August 14 Sold 8 bikes costing $91 each and 12 bikes costing $106 each. Total cost = $2,000. August 30 Sold 2 bikes costing $91 each, 3 bikes costing $106 each, 15 bikes costing $115

each, and 3 bikes costing $119 each. Total cost = $2,582.

Exhibit 5.4 begins with the $5,990 in total units available for sale. For the 20 units sold on August 14, the total cost of sales is $2,000. Next, for the 23 units sold on August 30, the total cost of sales is $2,582. The total cost of sales for the period is $4,582. We then subtract this $4,582 in cost of goods sold from the $5,990 in cost of goods available to get $1,408 in ending inventory.

EXHIBIT 5.4 Specific Identification Computations

Total cost of 55 units available for sale (from Exhibit 5 .3) . . . . . . . . . . . . . . . . . . . . . . . . . . . . . $ 5,990

Cost of goods sold Aug . 14 (8 @ $91) + (12 @ $106) . . . . . . . . . . . . . . . . . . . . . . . . . . . . . . . . . . . . . . . . . $2,000 Aug . 30 (2 @ $91) + (3 @ $106) + (15 @ $115) + (3 @ $119) . . . . . . . . . . . . . . . . . . 2,582 4,582 Ending inventory . . . . . . . . . . . . . . . . . . . . . . . . . . . . . . . . . . . . . . . . . . . . . . . . . . . . . . . . . $1,408

Merchandise Inventory (SI)

Aug. 1 910 Aug. 3 1,590 Aug. 14 2,000 Aug. 17 2,300 Aug. 28 1,190 Aug. 30 2,582

Aug. 31 1,408

Trekking’s cost of goods sold reported on the income statement is $4,582, and ending inventory reported on the balance sheet is $1,408. The following graphic shows this flow of costs.

Point: Specific identification is common for custom-made inven- tory. Examples include jewelers and fashion designers.

LAST BoughtFIRST Bought

SI Inventory $1,408

SI COGS $4,582

©Michael DeYoung/Blend Images

Chapter 5 Inventories and Cost of Sales 195

First-In, First-Out First-in, first-out (FIFO) assumes that inventory items are sold in the order acquired. When sales occur, the costs of the earliest units acquired are charged to cost of goods sold. This leaves the costs from the most recent purchases in ending inventory.

Exhibit 5.5 starts with beginning inventory of 10 bikes at $91 each.

August 3 Purchased 15 bikes costing $106 each for $1,590. Inventory now consists of 10 bikes at $91 each and 15 bikes at $106 each, for a total of $2,500.

August 14 Sold 20 bikes—applying FIFO, the first 10 sold cost $91 each and the next 10 sold cost $106 each, for a total cost of $1,970. This leaves 5 bikes costing $106 each, or $530, in inventory.

August 17 Purchased 20 bikes costing $115 each, and on August 28, purchased another 10 bikes costing $119 each, for a total of 35 bikes costing $4,020 in inventory.

August 30 Sold 23 bikes—applying FIFO, the first 5 bikes sold cost $106 each and the next 18 sold cost $115 each, for a total of $2,600. This leaves 12 bikes costing $1,420 in ending inventory.

Point: “Goods Purchased” column is identical for all methods.

Trekking’s cost of goods sold reported on its income statement is $4,570 ($1,970 + $2,600), and its ending inventory reported on the balance sheet is $1,420.

LAST BoughtFIRST Bought

FIFO COGS $4,570 FIFO Inventory $1,420

EXHIBIT 5.5 FIFO Computations— Perpetual System

Date Goods Purchased Cost of Goods Sold Inventory Balance

Aug. 1 Beginning balance 10 @ $ 91 = $ 910

Aug. 3 15 @ $106 = $1,590 10 @ $ 91 15 @ $106

= $ 2,500

Aug. 14 10 @ $ 91 = $ 910 10 @ $106 = $1,060

= $1,970 5 @ $106 = $ 530

Aug. 17 20 @ $115 = $2,300 5 @ $106 20 @ $115

= $ 2,830

Aug. 28 10 @ $119 = $1,190 5 @ $106 20 @ $115 = $ 4,020 10 @ $119

Aug. 30 5 @ $106 = $ 530 2 @ $115 18 @ $115 = $2,070

= $2,600 10 @ $119

= $1,420

$4,570

⎧ ⎨ ⎩

⎧ ⎨ ⎩

⎧ ⎨ ⎩

⎧ ⎨ ⎩

⎧ ⎪ ⎨ ⎪ ⎩

⎧ ⎨ ⎩

Merchandise Inventory (FIFO)

Aug. 1 910 Aug. 3 1,590 Aug. 14 1,970 Aug. 17 2,300 Aug. 28 1,190 Aug. 30 2,600

Aug. 31 1,420

Last-In, First-Out Last-in, first-out (LIFO) assumes that the most recent purchases are sold first. These more recent costs are charged to the goods sold, and the costs of the earliest purchases are assigned to inventory.

Point: By assigning costs from the most recent purchases to cost of goods sold, LIFO comes closest to matching current costs of goods sold with revenues.

196 Chapter 5 Inventories and Cost of Sales

Exhibit 5.6 starts with beginning inventory of 10 bikes at $91 each.

August 3 Purchased 15 bikes costing $106 each for $1,590. Inventory now consists of 10 bikes at $91 each and 15 bikes at $106 each, for a total of $2,500.

August 14 Sold 20 bikes—applying LIFO, the first 15 sold are from the most recent purchase costing $106 each, and the next 5 sold are from the next most recent purchase costing $91 each, for a total of $2,045. This leaves 5 bikes costing $91 each, or $455, in inventory.

August 17 Purchased 20 bikes costing $115 each, and on August 28, purchased another 10 bikes costing $119 each, for a total of 35 bikes costing $3,945 in inventory.

August 30 Sold 23 bikes—applying LIFO, the first 10 bikes sold are from the most recent purchase costing $119 each, and the next 13 sold are from the next most recent purchase costing $115 each, for a total of $2,685. This leaves 12 bikes costing $1,260 in ending inventory.

Date Goods Purchased Cost of Goods Sold Inventory Balance

Aug. 1 Beginning balance 10 @ $ 91 = $ 910

Aug. 3 15 @ $106 = $1,590 10 @ $ 91 15 @ $106

= $ 2,500

Aug. 14 15 @ $106 = $1,590 5 @ $ 91 = $ 455

= $2,045 5 @ $ 91 = $ 455

Aug. 17 20 @ $115 = $2,300 5 @ $ 91 20 @ $115

= $ 2,755

Aug. 28 10 @ $119 = $1,190 5 @ $ 91 20 @ $115 = $ 3,945 10 @ $119

Aug. 30 10 @ $119 = $1,190 5 @ $ 91 13 @ $115 = $1,495

= $2,685 7 @ $115

= $1,260

$4,730

EXHIBIT 5.6 LIFO Computations— Perpetual System

⎧ ⎨ ⎩

⎧ ⎨ ⎩

⎧ ⎨ ⎩

⎧ ⎨ ⎩

⎧ ⎪ ⎨ ⎪ ⎩

⎧ ⎨ ⎩

Trekking’s cost of goods sold reported on the income statement is $4,730 ($2,045 + $2,685), and its ending inventory reported on the balance sheet is $1,260.

Merchandise Inventory (LIFO)

Aug. 1 910 Aug. 3 1,590 Aug. 14 2,045 Aug. 17 2,300 Aug. 28 1,190 Aug. 30 2,685

Aug. 31 1,260

LAST Bought*LIFO perpetual applied at each sale date.FIRST Bought

LIFO Inventory $1,260

*LIFO COGS $4,730

Weighted Average Weighted average or WA (also called average cost) requires that we use the weighted average cost per unit of inventory at the time of each sale.

Weighted average cost per unit at time of each sale =

Cost of goods available for sale (at each sale) Number of units available for sale (at each sale)

Exhibit 5.7 starts with beginning inventory of 10 bikes at $91 each.

August 3 Purchased 15 bikes costing $106 each for $1,590. Inventory now consists of 10 bikes at $91 each and 15 bikes at $106 each, for a total of $2,500. The average cost per bike for that inventory is $100, computed as $2,500/(10 bikes + 15 bikes).

Chapter 5 Inventories and Cost of Sales 197

August 14 Sold 20 bikes—applying WA, the 20 sold are assigned the $100 average cost, for a total of $2,000. This leaves 5 bikes with an average cost of $100 each, or $500, in inventory.

August 17 Purchased 20 bikes costing $2,300, and on August 28, purchased another 10 bikes costing $1,190, for a total of 35 bikes costing $3,990 in inventory at August 28. The average cost per bike for the August 28 inventory is $114, computed as $3,990/35 bikes.

August 30 Sold 23 bikes—applying WA, the 23 sold are assigned the $114 average cost, for a total of $2,622. This leaves 12 bikes costing $1,368 in ending inventory.

Date Goods Purchased Cost of Goods Sold Inventory Balance

Aug . 1 Beginning balance 10 @ $ 91 = $ 910 (10 @ $ 91 per unit)

Aug . 3 15 @ $106 = $1,590 10 @ $ 91 15 @ $106

= $ 2,500 (25 @ $100 per unit)a

Aug . 14 20 @ $100 = $2,000 5 @ $100 = $ 500 ( 5 @ $100 per unit)b

Aug . 17 20 @ $115 = $2,300 5 @ $100 20 @ $115

= $ 2,800 (25 @ $112 per unit)c

Aug . 28 10 @ $119 = $1,190 25 @ $112 10 @ $119

= $ 3,990 (35 @ $114 per unit)d

Aug . 30 23 @ $114 = $2,622 12 @ $114 = $1,368 (12 @ $114 per unit)e

$4,622

a$100 per unit = ($2,500 inventory balance ÷ 25 units in inventory). b$100 per unit = ($500 inventory balance ÷ 5 units in inventory). c$112 per unit = ($2,800 inventory balance ÷ 25 units in inventory).

d$114 per unit = ($3,990 inventory balance ÷ 35 units in inventory). e$114 per unit = ($1,368 inventory balance ÷ 12 units in inventory).

EXHIBIT 5.7 Weighted Average Computations—Perpetual System

Merchandise Inventory (WA)

Aug. 1 910 Aug. 3 1,590 Aug. 14 2,000 Aug. 17 2,300 Aug. 28 1,190 Aug. 30 2,622

Aug. 31 1,368

Trekking’s cost of goods sold reported on the income statement is $4,622 ($2,000 + $2,622), and its ending inventory reported on the balance sheet is $1,368.

LAST BoughtFIRST Bought

WA COGS $4,622WA Inventory $1,368

Point: WA perpetual applied at each sale date.

Kickbacks and Invoice Fraud Inventory safeguards include restricted access, use of authorized requisitions, and security measures. Proper accounting includes matching inventory received with purchase order terms and quality requirements, preventing misstatements, and controlling access to records. A study reports that 35% of employees in purchasing and procurement observed improper kickbacks or gifts from suppliers. ■

Ethical Risk

Financial Statement Effects of Costing Methods When purchase prices do not change, each inventory costing method assigns the same cost amounts to inventory and to cost of goods sold. When purchase prices are different, the methods assign different cost amounts. We show these differences in Exhibit 5.8 using Trekking’s data.

Rising Costs When purchase costs regularly rise, as in Trekking’s case, the following occurs. FIFO reports the lowest cost of goods sold—yielding the highest gross profit and net income. LIFO reports the highest cost of goods sold—yielding the lowest gross profit and net income. Weighted average yields results between FIFO and LIFO.

A1 Analyze the effects of inventory methods for both financial and tax reporting.

198 Chapter 5 Inventories and Cost of Sales

Falling Costs When costs regularly decline, the reverse occurs for FIFO and LIFO. FIFO gives the highest cost of goods sold—yielding the lowest gross profit and income. LIFO gives the lowest cost of goods sold—yielding the highest gross profit and income.

Method Advantages Each method offers advantages. FIFO—inventory on the balance sheet approximates its current cost; it also follows the actual

flow of goods for most businesses. LIFO—cost of goods sold on the income statement approximates its current cost; it also bet-

ter matches current costs with revenues. Weighted average—smooths out erratic changes in costs. Specific identification—matches the costs of items with the revenues they generate.

Tax Effects of Costing Methods Inventory costs affect net income and have potential tax effects. Exhibit 5.8 shows that Trekking gains a temporary tax advantage by using LIFO because it has less income to be taxed. Many companies use LIFO for this reason. The IRS requires that when LIFO is used for tax reporting, it also must be used for financial reporting—called LIFO conformity rule.

Point: LIFO inventory is often less than the inventory’s replacement cost because LIFO inventory is valued using the oldest inventory purchase costs.

Trekking Company Specific Weighted For Month Ended August 31 Identification FIFO LIFO Average

Income Statement Sales . . . . . . . . . . . . . . . . . . . . . . . . . . . . . . $ 6,050 $ 6,050 $ 6,050 $ 6,050

Cost of goods sold . . . . . . . . . . . . . . . . . . 4,582 4,570 4,730 4,622 Gross profit . . . . . . . . . . . . . . . . . . . . . . 1,468 1,480 1,320 1,428

Expenses . . . . . . . . . . . . . . . . . . . . . . . . . . 450 450 450 450

Income before taxes . . . . . . . . . . . . . . . . . 1,018 1,030 870 978

Income tax expense (30%) . . . . . . . . . . . . 305 309 261 293

Net income . . . . . . . . . . . . . . . . . . . . . . . . $ 713 $   721 $   609 $ 685

Balance Sheet Inventory . . . . . . . . . . . . . . . . . . . . . . . . . . $1,408 $1,420 $1,260 $1,368

EXHIBIT 5.8 Financial Statement Effects of Inventory Costing Methods

Inventory Manager Your compensation as inventory manager includes a bonus plan based on gross profit. Your superior asks your opinion on changing the inventory costing method from FIFO to LIFO. As costs are expected to continue to rise, your superior predicts that LIFO would match higher current costs against sales, thereby lowering taxable income (and gross profit). What do you recommend? ■ Answer: It seems your company can save (or at least postpone) taxes by switching to LIFO, but the switch is likely to reduce bonus money that you believe you have earned and deserve. Your best decision is to tell your superior about the tax savings with LIFO. You should discuss your bonus plan and how this is likely to hurt you unfairly.

Decision Ethics

A company reported the following December purchase and sales data for its only product.

Perpetual SI, FIFO, LIFO, and WA

NEED-TO-KNOW 5-2

P1

Date Activities Units Acquired at Cost Units Sold at Retail

Dec . 1 Beginning inventory . . . . . . . . . . . . . . 5 units @ $3 .00 = $ 15 .00 Dec . 8 Purchase . . . . . . . . . . . . . . . . . . . . . . . 10 units @ $4 .50 = 45 .00 Dec . 9 Sales . . . . . . . . . . . . . . . . . . . . . . . . . . . 8 units @ $7 .00

Dec . 19 Purchase . . . . . . . . . . . . . . . . . . . . . . . 13 units @ $5 .00 = 65 .00 Dec . 24 Sales . . . . . . . . . . . . . . . . . . . . . . . . . . . 18 units @ $8 .00

Dec . 30 Purchase . . . . . . . . . . . . . . . . . . . . . . . 8 units @ $5 .30 = 42 .40 Totals . . . . . . . . . . . . . . . . . . . . . . . . . . . . . . . 36 units $167 .40 26 units

The company uses a perpetual inventory system. Determine the cost assigned to ending inventory and to cost of goods sold using (a) specific identification, (b) FIFO, (c) LIFO, and (d) weighted average. (Round per unit costs and inventory amounts to cents.)

Chapter 5 Inventories and Cost of Sales 199

For specific identification, ending inventory consists of 10 units, where 8 are from the December 30 purchase and 2 are from the December 8 purchase. Specific unit sales follow.

Dec. 9 Sold 2 units costing $3.00 each and 6 units costing $4.50 each. Total cost = $33.00. Dec. 24 Sold 3 units costing $3.00 each, 2 units costing $4.50 each, and 13 units costing $5.00

each. Total cost = $83.00.

Solutions

a. Specific identification: Ending inventory—eight units from December 30 purchase and two units from December 8 purchase.

b. FIFO—Perpetual.

Date Goods Purchased Cost of Goods Sold Inventory Balance

12/1 5 @ $3 .00 = $15 .00

12/8 10 @ $4 .50 5 @ $3 .00

10 @ $4 .50 = $60 .00

12/9 5 @ $3 .00 7 @ $4 .50 = $31 .50

3 @ $4 .50 = $ 28 .50

12/19 13 @ $5 .00 7 @ $4 .50

13 @ $5 .00 = $96 .50

12/24 7 @ $4 .50

11 @ $5 .00 = $ 86 .50 2 @ $5 .00 = $10 .00

12/30 8 @ $5 .30 2 @ $5 .00

$115 .00

8 @ $5 .30 = $52 .40

Merchandise Inventory (FIFO)

Beg. inventory 15.00 Dec. 8 45.00 Dec. 9 28.50 Dec. 19 65.00 Dec. 24 86.50 Dec. 30 42.40

End. inventory 52.40

Ending Cost of Specific Identification Inventory Goods Sold

(8 × $5 .30) + (2 × $4 .50) . . . . . . . . . . . . . . . . . . . . . . . . . . . . . . . . . . . . . . . . . . . . . $51 .40 (5 × $3 .00) + (8 × $4 .50) + (13 × $5 .00) + (0 × $5 .30) or $167 .40 [Total Goods Available] − $51 .40 [Ending Inventory] . . . . . . . . . . . . $116 .00

Merchandise Inventory (SI)

Beg. inventory 15.00 Dec. 8 45.00 Dec. 9 33.00 Dec. 19 65.00 Dec. 24 83.00 Dec. 30 42.40

End. inventory 51.40

OR “short-cut” FIFO—Perpetual.

Ending Cost of FIFO Inventory Goods Sold

(8 × $5 .30) + (2 × $5 .00) . . . . . . . . . . . . . . . . . . . . . . . . . . . . . . . . . . . . . . . . . . . . . $52 .40 (5 × $3 .00) + (10 × $4 .50) + (11 × $5 .00) or $167 .40 [Total Goods Available] − $52 .40 [Ending Inventory] . . . . . . . . . . . . $115 .00

c. LIFO—Perpetual.

Merchandise Inventory (LIFO)

Beg. inventory 15.00 Dec. 8 45.00 Dec. 9 36.00 Dec. 19 65.00 Dec. 24 83.00 Dec. 30 42.40

End. inventory 48.40

Date Goods Purchased Cost of Goods Sold Inventory Balance

12/1 5 @ $3 .00 = $15 .00

12/8 10 @ $4 .50 5 @ $3 .00

10 @ $4 .50 = $60 .00

12/9 5 @ $3 .00

8 @ $4 .50 = $ 36 .00

2 @ $4 .50 = $24 .00

12/19 13 @ $5 .00 5 @ $3 .00

2 @ $4 .50 = $89 .00 13 @ $5 .00

12/24 13 @ $5 .00

2 @ $4 .50 = $ 83 .00 2 @ $3 .00 = $ 6 .00 3 @ $3 .00

12/30 8 @ $5 .30 2 @ $3 .00

$119 .00

8 @ $5 .30 = $48 .40

200 Chapter 5 Inventories and Cost of Sales

Merchandise Inventory (WA)

Beg. inventory 15.00 Dec. 8 45.00 Dec. 9 32.00 Dec. 19 65.00 Dec. 24 83.70 Dec. 30 42.40

End. inventory 51.70

Date Goods Purchased Cost of Goods Sold Inventory Balance

12/1 5 @ $3 .00 = $15 .00 (5 @ $3 .00 per unit)

12/8 10 @ $4 .50 5 @ $3 .00

10 @ $4 .50 = $60 .00

($60 .00∕15 units = $4 .00 avg . cost)

12/9 8 @ $4 .00 = $ 32 .00 7 @ $4 .00 = $28 .00 (7 @ $4 .00 per unit)

12/19 13 @ $5 .00 7 @ $4 .00

13 @ $5 .00 = $93 .00

($93 .00∕20 units = $4 .65 avg . cost)

12/24 18 @ $4 .65 = $ 83 .70 2 @ $4 .65 = $ 9 .30 (2 @ $4 .65 per unit)

12/30 8 @ $5 .30 2 @ $4 .65

$115 .70

8 @ $5 .30 = $51 .70

($51 .70∕10 units = $5 .17 avg . cost)

Do More: QS 5-3, QS 5-4, QS 5-5, QS 5-6, QS 5-10, QS 5-11, QS 5-12, E 5-3

d. Weighted Average—Perpetual.

VALUING INVENTORY AT LCM AND THE EFFECTS OF INVENTORY ERRORS This section covers how market value and inventory errors impact financial statements.

Lower of Cost or Market After companies apply one of four costing methods (FIFO, LIFO, weighted average, or spe- cific identification), inventory is reviewed to ensure it is reported at the lower of cost or market (LCM).

Computing the Lower of Cost or Market Market in the term LCM is replace- ment cost for LIFO, but net realizable value for the other three methods—advanced courses cover specifics. A decline in market value means a loss of value in inventory. When market value is lower than cost of inventory, a loss is recorded. When market value is higher than cost of inventory, no adjustment is made.

LCM is applied in one of three ways: (1) to each individual item separately, (2) to major cat- egories of items, or (3) to the whole of inventory. With the increasing use of technology and inventory tracking, companies increasingly apply LCM to each individual item separately. Accordingly, we show that method only; advanced courses cover other methods. To demonstrate LCM, we apply it to the ending inventory of a motorsports retailer in Exhibit 5.9.

P2 Compute the lower of cost or market amount of inventory.

Point: LCM applied to each individual item always yields the lowest inventory.

Per Unit Inventory Total Total LCM Applied Items Units Cost Market Cost Market to Items

Roadster . . . . . . . . . . . . . 20 $8,500 $7,000 $170,000 $140,000 $   140,000

Sprint . . . . . . . . . . . . . . . 10 5,000 6,000 50,000 60,000 50,000

Totals . . . . . . . . . . . . . . . $220,000 $190,000

EXHIBIT 5.9 Lower of Cost or Market Computations

The amount of $190,000 is lower than the $220,000 recorded cost .

$140,000 is the lower of $170,000 or $140,000 .

For Roadster, $140,000 is the lower of the $170,000 cost and the $140,000 market. For Sprint, $50,000 is the lower of the $50,000 cost and the $60,000 market. This yields a $190,000 reported inventory, computed from $140,000 for Roadster plus $50,000 for Sprint.

Recording the Lower of Cost or Market Inventory is adjusted downward when total “LCM applied to items” is less than total cost of inventory. To demonstrate, if LCM is

Chapter 5 Inventories and Cost of Sales 201

applied in Exhibit 5.9, the Merchandise Inventory account must be adjusted from the $220,000 recorded cost down to the $190,000 LCM amount as follows.

Cost of Goods Sold . . . . . . . . . . . . . . . . . . . . . . . . . . . . . . . 30,000 Merchandise Inventory . . . . . . . . . . . . . . . . . . . . . . . 30,000 Adjust inventory cost to market.

Financial Statement Effects of Inventory Errors An inventory error causes misstatements in cost of goods sold, gross profit, net income, current assets, and equity. It also causes misstatements in the next period’s statements because ending inventory of one period is the beginning inventory of the next. As we consider financial state- ment effects, we recall the following inventory relation.

A2 Analyze the effects of inventory errors on current and future financial statements.

Beginning inventory

Net purchases

Ending inventory

Cost of goods sold+ – =

Income Statement Effects Exhibit 5.10 shows the effects of inventory errors in the current and next period’s income statements. Row 1, Year 1. Understating ending inventory overstates cost of goods sold. This is because

we subtract a smaller ending inventory in computing cost of goods sold. A higher cost of goods sold yields a lower income.

Row 1, Year 2. Understated ending inventory for Year 1 becomes an understated beginning in- ventory for Year 2. If beginning inventory is understated, cost of goods sold is understated (be- cause we are starting with a smaller amount). A lower cost of goods sold yields a higher income.

Row 2, Year 1. Overstating ending inventory understates cost of goods sold. A lower cost of goods sold yields a higher income.

Row 2, Year 2. Overstated ending inventory for Year 1 becomes an overstated beginning in- ventory for Year 2. If beginning inventory is overstated, cost of goods sold is overstated. A higher cost of goods sold yields a lower income.

LCM Method

NEED-TO-KNOW 5-3

P2

A company has the following products in its ending inventory, along with cost and market values. (a) Compute the lower of cost or market for its inventory when applied separately to each product. (b) If the market amount is less than the recorded cost of the inventory, then record the December 31 LCM adjustment to the Merchandise Inventory account.

Do More: QS 5-19, E 5-10

b. Dec . 31 Cost of Goods Sold . . . . . . . . . . . . . . . . . . . . . . . . . . . . . . . . . . . . . . . 1,000 Merchandise Inventory . . . . . . . . . . . . . . . . . . . . . . . . . . . . . . . . 1,000 Adjust inventory cost to market ($11,000 − $10,000).

Units Cost per Unit Market per Unit

Road bikes . . . . . . . . . . . . . . 5 $1,000 $800 Mountain bikes . . . . . . . . . . . 4 500 600 Town bikes . . . . . . . . . . . . . . 10 400 450

Solution

a. Cost Market Total Total Inventory Items Units per Unit per Unit Cost Market LCM Items

Road bikes . . . . . . . . . . . . . . . . . . 5 $1,000 $800 $ 5,000 $4,000 $ 4,000 Mountain bikes . . . . . . . . . . . . . . . 4 500 600 2,000 2,400 2,000 Town bikes . . . . . . . . . . . . . . . . . . 10 400 450 4,000 4,500 4,000 Totals . . . . . . . . . . . . . . . . . . . . . . . $11,000 $ 10,000

LCM applied to each product . . . . $10,000

202 Chapter 5 Inventories and Cost of Sales

EXHIBIT 5.10 Effects of Inventory Errors on the Income Statement

Year 1 Year 2

Ending Inventory Cost of Goods Sold Net Income Cost of Goods Sold Net Income

Understated  . . . . . . . . . . . Overstated  Understated  Understated  Overstated 

Overstated  . . . . . . . . . . . . . Understated  Overstated  Overstated  Understated 

Inventory Error Example Consider an inventory error for a company with $100,000 in sales for each of Year 1, Year 2, and Year 3. If this company has a steady $20,000 inventory level and makes $60,000 in purchases in each year, its cost of goods sold is $60,000 and its gross profit is $40,000.

Year 1 Understated Inventory: Year 1 Impact Assume the company makes an error in comput- ing its Year 1 ending inventory and reports $16,000 instead of the correct amount of $20,000. The effects of this error are in Exhibit 5.11. The $4,000 understatement of Year 1 ending inven- tory causes a $4,000 overstatement in Year 1 cost of goods sold and a $4,000 understatement in both gross profit and net income for Year 1.

Correct income is $30,000 for each year .

Income Statements Year 1 Year 2 Year 3

Sales . . . . . . . . . . . . . . . . . . . . . . . . . . $100,000 $100,000 $100,000

Cost of goods sold

Beginning inventory . . . . . . . . . . . . $20,000 $16,000* $20,000 Cost of goods purchased . . . . . . . . 60,000 60,000 60,000

Goods available for sale . . . . . . . . 80,000 76,000 80,000

Ending inventory . . . . . . . . . . . . . . 16,000* 20,000 20,000 Cost of goods sold . . . . . . . . . . . . . 64,000† 56,000† 60,000 Gross profit . . . . . . . . . . . . . . . . . . . . . 36,000 44,000 40,000

Expenses . . . . . . . . . . . . . . . . . . . . . . . 10,000 10,000 10,000

Net income . . . . . . . . . . . . . . . . . . . . . $ 26,000 $ 34,000 $ 30,000

*Correct amount is $20,000. †Correct amount is $60,000.

EXHIBIT 5.11 Effects of Inventory Errors on Three Periods’ Income Statements

Year 1 Understated Inventory: Year 2 Impact The Year 1 understated ending inventory becomes the Year 2 understated beginning inventory. This error causes an understatement in Year 2 cost of goods sold and a $4,000 overstatement in both gross profit and net income for Year 2.

Year 1 Understated Inventory: Year 3 Impact The Year 1 ending inventory error affects only that period and the next. It does not affect Year 3 results or any period thereafter.

Balance Sheet Effects Understating ending inventory understates both current and total assets. An understatement in ending inventory also yields an understatement in equity

because of the understatement in net income. Exhibit 5.12 shows the effects of inventory errors on the current period’s balance sheet amounts.

Example: If Year 1 ending inventory in Exhibit 5.11 is overstated by $3,000, cost of goods sold is understated by $3,000 in Year 1 and overstated by $3,000 in Year 2. Net income is overstated in Year 1 and understated in Year 2. Assets and equity are overstated in Year 1.

EXHIBIT 5.12 Effects of Inventory Errors on Current Period’s Balance Sheet

Ending Inventory Assets Equity

Understated . . . . Understated Understated

Overstated . . . . . Overstated Overstated

A company had $10,000 of sales, and it purchased merchandise costing $7,000 in each of Year 1, Year 2, and Year 3. It also maintained a $2,000 physical inventory from the beginning to the end of that three-year period. In accounting for inventory, it made an error at the end of Year 1 that caused its Year 1 ending inventory to appear on its statements as $1,600 rather than the correct $2,000. (a) Determine the correct amount of the company’s gross profit in each of Year 1, Year 2, and Year 3. (b) Prepare comparative in- come statements as in Exhibit 5.11 to show the effect of this error on the company’s cost of goods sold and gross profit for each of Year 1, Year 2, and Year 3.

Solution

a. Correct gross profit = $10,000 − $7,000 = $3,000 (for each year). b. Cost of goods sold and gross profit figures follow.

Effects of Inventory Errors

NEED-TO-KNOW 5-4

A2

Chapter 5 Inventories and Cost of Sales 203

Do More: QS 5-20, E 5-12

Year 1 Year 2 Year 3

Sales . . . . . . . . . . . . . . . . . . . . . . . . . . $10,000 $10,000 $10,000

Cost of goods sold

Beginning inventory . . . . . . . . . . . . $2,000 $1,600 $2,000 Cost of purchases . . . . . . . . . . . . . 7,000 7,000 7,000

Goods available for sale . . . . . . . . 9,000 8,600 9,000

Ending inventory . . . . . . . . . . . . . . 1,600 2,000 2,000 Cost of goods sold . . . . . . . . . . . . . 7,400 6,600 7,000 Gross profit . . . . . . . . . . . . . . . . . . . . . $ 2,600 $ 3,400 $ 3,000

Combined income for the 3 years is $9,000 ($2,600 + $3,400 + $3,000), which is correct, meaning the inventory error is “self-correcting” (even though individual years’ inventory amounts are in error).

Inventory Turnover and Days’ Sales in Inventory Decision Analysis

Inventory Turnover Inventory turnover, also called merchandise inventory turnover, is defined in Exhibit 5.13. Inventory turnover tells how many times a company turns over (sells) its inventory in a period. It is used to assess whether management is doing a good job controlling the amount of inventory. A low ratio means the com- pany may have more inventory than it needs. A very high ratio means inventory might be too low. This can cause lost sales if customers must back-order merchandise. Inventory turnover has no simple rule except to say a high ratio is preferable if inventory is adequate to meet demand.

EXHIBIT 5.13 Inventory TurnoverInventory turnover =

Cost of goods sold Average inventory

Days’ Sales in Inventory Days’ sales in inventory is a ratio that shows how much inventory is available in terms of the number of days’ sales. It can be interpreted as the number of days one can sell from existing inventory if no new items are purchased. This ratio reveals the buffer against out-of-stock inventory and is useful in evaluating how quickly inventory is being sold. It is defined in Exhibit 5.14. Days’ sales in inventory uses ending inventory, whereas inventory turnover uses average inventory.

Point: Low inventory turnover can reveal obsolescence.

Point: Inventory turnover is higher and days’ sales in inventory is lower for industries such as foods.

A3 Assess inventory management using both inventory turnover and days’ sales in inventory.

EXHIBIT 5.14 Days’ Sales in InventoryDays’ sales in inventory =

Ending inventory Cost of goods sold

× 365

Analysis of Inventory Management Merchandisers must plan and control inventory purchases and sales. Costco’s inventory at the end of the current year was $9,834 million. This inventory was 57% of its current assets and 27% of its total assets. We apply the analysis tools in this section to Costco and Walmart, as shown in Exhibit 5.15.

EXHIBIT 5.15 Inventory Turnover and Days’ Sales in Inventory for Costco and Walmart

Costco’s current year inventory turnover of 11.9 times means that it turns over its inventory 11.9 times per year. Costco’s inventory turnover exceeded Walmart’s turnover in each of the last three years. This is a positive for Costco, as we prefer inventory turnover to be high provided inventory is not out of stock and the company is not losing customers. Days’ sales in inventory of 32.1 days means that Costco is carrying 32.1 days of sales in inventory. This inventory buffer seems sufficient. As long as Costco is not at risk of running out of stock, it prefers its assets not be tied up in inventory.

Point: Take care when comparing turnover ratios across companies that use different costing meth- ods (such as FIFO and LIFO).

Company Figure ($ millions) Current Year 1 Year Ago 2 Years Ago

Costco Cost of goods sold . . . . . . . . . . . . . . $111,882 $102,901 $101,065 Ending inventory . . . . . . . . . . . . . . . . $ 9,834 $ 8,969 $ 8,908

Inventory turnover . . . . . . . . . . . . . 11.9 times 11.5 times 11.6 times Days’ sales in inventory . . . . . . . . . 32.1 days 31.8 days 32.2 days Walmart Inventory turnover . . . . . . . . . . . . . . 8 .3 times 8 .1 times 8 .1 times Days’ sales in inventory . . . . . . . . . . 43 .5 days 45 .0 days 45 .1 days

Craig Company buys and sells one product. Its beginning inventory, purchases, and sales during calendar- year 2019 follow.

COMPREHENSIVE 1

NEED-TO-KNOW 5-5

Perpetual Method: Computing Inventory Using LIFO, FIFO, WA, and SI; Financial Statement Impacts; and Inventory Errors

204 Chapter 5 Inventories and Cost of Sales

Entrepreneur Your retail store has an inventory turnover of 5.0 and a days’ sales in inventory of 73 days. The indus- try norm for inventory turnover is 4.4 and for days’ sales in inventory is 74 days. What is your assessment of inventory management? ■ Answer: Your inventory turnover is higher than the norm, whereas days’ sales in inventory approximates the norm. Because your turnover is already 14% better than average, you should probably direct attention to days’ sales in inventory. You should see if you can reduce the level of inventory while maintaining service to customers. Given your higher turnover, you should be able to hold less inventory.

Decision Maker

Date Activity Units Acquired at Cost Units Sold at Retail Unit Inventory

Jan . 1 Beg . inventory . . . . . . . . . . . 400 units @ $14 = $ 5,600 400 units Jan . 15 Sale . . . . . . . . . . . . . . . . . . . 200 units @ $30 200 units Mar . 10 Purchase . . . . . . . . . . . . . . . 200 units @ $15 = $ 3,000 400 units Apr . 1 Sale . . . . . . . . . . . . . . . . . . . 200 units @ $30 200 units May 9 Purchase . . . . . . . . . . . . . . . 300 units @ $16 = $ 4,800 500 units Sep . 22 Purchase . . . . . . . . . . . . . . . 250 units @ $20 = $ 5,000 750 units Nov . 1 Sale . . . . . . . . . . . . . . . . . . . 300 units @ $35 450 units Nov . 28 Purchase . . . . . . . . . . . . . . . 100 units @ $21 = $ 2,100 550 units Totals . . . . . . . . . . . . . . . . . . 1,250 units $20,500 700 units

Additional tracking data for specific identification: (1) January 15 sale—200 units @ $14, (2) April 1 sale—200 units @ $15, and (3) November 1 sale—200 units @ $14 and 100 units @ $20.

Required

1. Compute the cost of goods available for sale. 2. Apply the four methods of inventory costing (FIFO, LIFO, weighted average, and specific identifica-

tion) to compute ending inventory and cost of goods sold under each method using the perpetual system. 3. Compute gross profit earned by the company for each of the four costing methods in part 2. Also,

report the inventory amount reported on the balance sheet for each of the four methods. 4. In preparing financial statements for year 2019, the financial officer was instructed to use FIFO but

failed to do so and instead computed cost of goods sold according to LIFO, which led to a $1,400 over- statement in cost of goods sold from using LIFO. Determine the impact on year 2019’s income from the error. Also determine the effect of this error on year 2020’s income. Assume no income taxes.

5. Management wants a report that shows how changing from FIFO to another method would change net income. Prepare a table showing (1) the cost of goods sold amount under each of the four methods, (2) the amount by which each cost of goods sold total is different from the FIFO cost of goods sold, and (3) the effect on net income if another method is used instead of FIFO.

PLANNING THE SOLUTION Compute cost of goods available for sale by multiplying the units of beginning inventory and each

purchase by their unit costs to determine the total cost of goods available for sale. Prepare a perpetual FIFO table starting with beginning inventory and showing how inventory changes

after each purchase and after each sale (see Exhibit 5.5). Prepare a perpetual LIFO table starting with beginning inventory and showing how inventory changes

after each purchase and after each sale (see Exhibit 5.6). Make a table of purchases and sales recalculating the average cost of inventory prior to each sale to

arrive at the weighted average cost of ending inventory. Total the average costs associated with each sale to determine cost of goods sold (see Exhibit 5.7).

Prepare a table showing the computation of cost of goods sold and ending inventory using the specific identification method (see Exhibit 5.4).

Compare the year-end 2019 inventory amounts under FIFO and LIFO to determine the misstatement of year 2019 income that results from using LIFO. The errors for years 2019 and 2020 are equal in amount but opposite in effect.

Create a table showing cost of goods sold under each method and how net income would differ from FIFO net income if an alternate method were adopted.

Chapter 5 Inventories and Cost of Sales 205

SOLUTION 1. Cost of goods available for sale (this amount is the same for all methods).

Date Units Unit Cost Cost

Jan . 1 Beg . inventory . . . . . . . . . . . . . 400 $14 $ 5,600

Mar . 10 Purchase . . . . . . . . . . . . . . . . . 200 15 3,000

May 9 Purchase . . . . . . . . . . . . . . . . . 300 16 4,800

Sep . 22 Purchase . . . . . . . . . . . . . . . . . 250 20 5,000

Nov . 28 Purchase . . . . . . . . . . . . . . . . . 100 21 2,100

Total goods available for sale . . . . . . . . . . . . 1,250 $20,500

2a. FIFO perpetual method.

Date Goods Purchased Cost of Goods Sold Inventory Balance

Jan . 1 Beginning balance 400 @ $14 = $ 5,600

Jan . 15 200 @ $14 = $2,800 200 @ $14 = $ 2,800

Mar . 10 200 @ $15 = $3,000 200 @ $14 = $ 5,800

200 @ $15

Apr . 1 200 @ $14 = $2,800 200 @ $15 = $ 3,000

May 9 300 @ $16 = $4,800 200 @ $15 = $ 7,800

300 @ $16

Sep . 22 250 @ $20 = $5,000 200 @ $15 300 @ $16 = $ 12,800 250 @ $20

Nov . 1 200 @ $15 = $3,000 200 @ $16 = $ 8,200

100 @ $16 = $1,600 250 @ $20

Nov . 28 100 @ $21 = $2,100 200 @ $16 250 @ $20 = $10,300 100 @ $21

Total cost of goods sold $10,200 ⎧ ⎨ ⎩

⎧ ⎨ ⎩

⎧ ⎨ ⎩

⎧ ⎪ ⎨ ⎪ ⎩

⎧ ⎪ ⎨ ⎪ ⎩

Note: In a classroom situation, once we compute cost of goods available for sale, we can compute the amount for either cost of goods sold or ending inventory—it is a matter of preference. In practice, the costs of items sold are identified as sales are made and immediately transferred from the Inventory account to the Cost of Goods Sold account. The previous solution showing the line-by-line approach illustrates actual ap- plication in practice. The following alternate solutions illustrate that, once the concepts are understood, other solution approaches are available. Although this is only shown for FIFO, it could be shown for all methods.

Alternate Methods to Compute FIFO Perpetual Numbers

Cost of goods available for sale (from part 1) . . . . . $ 20,500

Ending inventory*

Nov . 28 Purchase (100 @ $21) . . . . . . . . . . . $2,100

Sep . 22 Purchase (250 @ $20) . . . . . . . . . . . 5,000

May 9 Purchase (200 @ $16) . . . . . . . . . . . 3,200

Ending inventory . . . . . . . . . . . . . . . . . . . . . . . . . . . 10,300 Cost of goods sold . . . . . . . . . . . . . . . . . . . . . . . . . $10,200

* FIFO assumes that the earlier costs are the first to flow out; thus, we determine ending inventory by assigning the most recent costs to the remaining items.

[FIFO Alternate No . 1: Computing ending inventory first] [FIFO Alternate No . 2: Computing cost of goods sold first]

Cost of goods available for sale (from part 1) . . . . . . . . $ 20,500

Cost of goods sold Jan . 15 Sold (200 @ $14) . . . . . . . . . . . . . . . . . $2,800

Apr . 1 Sold (200 @ $14) . . . . . . . . . . . . . . . . . 2,800

Nov . 1 Sold (200 @ $15 and 100 @ $16) . . . . 4,600 10,200 Ending inventory . . . . . . . . . . . . . . . . . . . . . . . . . . . . . . $10,300

206 Chapter 5 Inventories and Cost of Sales

2b. LIFO perpetual method.

Date Goods Purchased Cost of Goods Sold Inventory Balance

Jan . 1 Beginning balance 400 @ $14 = $ 5,600

Jan . 15 200 @ $14 = $2,800 200 @ $14 = $ 2,800

Mar . 10 200 @ $15 = $3,000 200 @ $14 = $ 5,800

200 @ $15

Apr . 1 200 @ $15 = $3,000 200 @ $14 = $ 2,800

May 9 300 @ $16 = $4,800 200 @ $14 = $ 7,600

300 @ $16

Sep . 22 250 @ $20 = $5,000 200 @ $14 300 @ $16 = $12,600 250 @ $20

Nov . 1 250 @ $20 = $5,000 200 @ $14 = $ 6,800

50 @ $16 = $ 800 250 @ $16

Nov . 28 100 @ $21 = $2,100 200 @ $14 250 @ $16 = $ 8,900 100 @ $21

Total cost of goods sold $11,600

⎧ ⎨ ⎩

⎧ ⎨ ⎩

⎧ ⎨ ⎩

⎧ ⎪ ⎨ ⎪ ⎩

⎧ ⎪ ⎨ ⎪ ⎩

2c. Weighted average perpetual method.

Date Goods Purchased Cost of Goods Sold Inventory Balance

Jan . 1 Beginning balance 400 @ $14 .00 = $     5,600 ($5,600/400 units = $14 .00 avg . cost)

Jan . 15 200 @ $14 .00 = $ 2,800 200 @ $14 .00 = $    2,800

Mar . 10 200 @ $15 .00 = $3,000 200 @ $14 .00 = $     5,800

200 @ $15 .00

($5,800/400 units = $14 .50 avg . cost)

Apr . 1 200 @ $14 .50 = $ 2,900 200 @ $14 .50 = $    2,900

May 9 300 @ $16 .00 = $4,800 200 @ $14 .50 = $    7,700

300 @ $16 .00

($7,700/500 units = $15 .40 avg . cost)

Sep . 22 250 @ $20 .00 = $5,000 500 @ $15 .40 250 @ $20 .00

= $ 12,700

($12,700/750 units = $16 .93† avg . cost)

Nov . 1 300 @ $16 .93 = $ 5,079 450 @ $16 .93 = $ 7,618 .50

Nov . 28 100 @ $21 .00 = $2,100 450 @ $16 .93 = $9,718.50

100 @ $21 .00

($9,718 .50/550 units = $17 .67 avg . cost)

Total cost of goods sold* $10,779

*Cost of goods sold ($10,779) plus ending inventory ($9,718.50) is $2.50 less †Rounded to 2 decimal places. than the cost of goods available for sale ($20,500) due to rounding.

Chapter 5 Inventories and Cost of Sales 207

2d. Specific identification method.

Cost of goods available for sale (from part 1) . . . . . . . . . . . . . . . $ 20,500

Ending inventory*

May 9 Purchase (300 @ $16) . . . . . . . . . . . . . . . . . . . . . $4,800

Sep . 22 Purchase (150 @ $20) . . . . . . . . . . . . . . . . . . . . . 3,000

Nov . 28 Purchase (100 @ $21) . . . . . . . . . . . . . . . . . . . . . 2,100

Ending inventory . . . . . . . . . . . . . . . . . . . . . . . . . . . . . . . . . . . . . 9,900 Cost of goods sold . . . . . . . . . . . . . . . . . . . . . . . . . . . . . . . . . . . $10,600

*The additional tracking data provided are used to identify the items in ending inventory.

3.

Weighted Specific FIFO LIFO Average Identification

Income Statement Sales* . . . . . . . . . . . . . . . . . . . . . . . . . . . $ 22,500 $22,500 $ 22,500 $22,500

Cost of goods sold . . . . . . . . . . . . . . . . 10,200 11,600 10,779 10,600 Gross profit . . . . . . . . . . . . . . . . . . . . . $ 12,300 $10,900 $ 11,721 $11,900

Balance Sheet Inventory . . . . . . . . . . . . . . . . . . . . . . . . $10,300 $ 8,900 $9,718.50 $ 9,900

*Sales = (200 units × $30) + (200 units × $30) + (300 units × $35) = $22,500

4. Mistakenly using LIFO when FIFO should have been used overstates cost of goods sold in year 2019 by $1,400, which is the difference between the FIFO and LIFO amounts of ending inventory. It under- states income in 2019 by $1,400. In year 2020, income is overstated by $1,400 because of the under- statement in beginning inventory.

5. Analysis of the effects of alternative inventory methods.

Difference from Effect on Net FIFO Cost of Income If Adopted Cost of Goods Sold Goods Sold Instead of FIFO

FIFO . . . . . . . . . . . . . . . . . . . . . . . . $10,200 — —

LIFO . . . . . . . . . . . . . . . . . . . . . . . . 11,600 +$1,400 $1,400 lower Weighted average . . . . . . . . . . . . 10,779 + 579 579 lower Specific identification . . . . . . . . . . 10,600 + 400 400 lower

Craig Company buys and sells one product. Its beginning inventory, purchases, and sales during calendar- year 2019 follow.

COMPREHENSIVE 2

NEED-TO-KNOW 5-6

Periodic Method: Computing Inventory Using LIFO, FIFO, WA, and SI; Financial Statement Impacts; and Inventory Errors

Date Activity Units Acquired at Cost Units Sold at Retail Unit Inventory

Jan . 1 Beg . inventory . . . . . . . . . . . 400 units @ $14 = $  5,600 400 units Jan . 15 Sale . . . . . . . . . . . . . . . . . . . 200 units @ $30 200 units

Mar . 10 Purchase . . . . . . . . . . . . . . . 200 units @ $15 = $  3,000 400 units Apr . 1 Sale . . . . . . . . . . . . . . . . . . . 200 units @ $30 200 units

May 9 Purchase . . . . . . . . . . . . . . . 300 units @ $16 = $  4,800 500 units Sep . 22 Purchase . . . . . . . . . . . . . . . 250 units @ $20 = $  5,000 750 units Nov . 1 Sale . . . . . . . . . . . . . . . . . . . 300 units @ $35 450 units

Nov . 28 Purchase . . . . . . . . . . . . . . . 100 units @ $21 = $  2,100 550 units Totals . . . . . . . . . . . . . . . . . . 1,250 units $20,500 700 units

Additional tracking data for specific identification: (1) January 15 sale—200 units @ $14, (2) April 1 sale—200 units @ $15, and (3) November 1 sale—200 units @ $14 and 100 units @ $20.

208 Chapter 5 Inventories and Cost of Sales

Required

1. Compute the cost of goods available for sale. 2. Apply the four methods of inventory costing (FIFO, LIFO, weighted average, and specific iden-

tification) to compute ending inventory and cost of goods sold under each method using the periodic system.

3. Compute gross profit earned by the company for each of the four costing methods in part 2. Also, report the inventory amount reported on the balance sheet for each of the four methods.

4. In preparing financial statements for year 2019, the financial officer was instructed to use FIFO but failed to do so and instead computed cost of goods sold according to LIFO. Determine the impact of the error on year 2019’s income. Also determine the effect of this error on year 2020’s income. Assume no income taxes.

PLANNING THE SOLUTION Compute cost of goods available for sale by multiplying the units of beginning inventory and each

purchase by their unit costs to determine the total cost of goods available for sale. Prepare a periodic FIFO computation starting with cost of units available and subtracting FIFO ending

inventory amounts to obtain FIFO cost of goods sold (see Exhibit 5A.3). Prepare a periodic LIFO computation starting with cost of units available and subtracting LIFO ending

inventory amounts to obtain LIFO cost of goods sold (see Exhibit 5A.4). Compute weighted average ending inventory and cost of goods sold using the three-step process illus-

trated in Exhibits 5A.5a and 5A.5b. Prepare a table showing the computation of cost of goods sold and ending inventory using the specific

identification method (see Exhibit 5A.2). Compare the year-end 2019 inventory amounts under FIFO and LIFO to determine the misstatement of

year 2019 income that results from using LIFO. The errors for years 2019 and 2020 are equal in amount but opposite in effect.

SOLUTION 1. Cost of goods available for sale (this amount is the same for all methods).

Date Units Unit Cost Cost

Jan . 1 Beg . inventory . . . . . . . . . . . . . 400 $14 $ 5,600

Mar . 10 Purchase . . . . . . . . . . . . . . . . . 200 15 3,000

May 9 Purchase . . . . . . . . . . . . . . . . . 300 16 4,800

Sep . 22 Purchase . . . . . . . . . . . . . . . . . 250 20 5,000

Nov . 28 Purchase . . . . . . . . . . . . . . . . . 100 21 2,100

Total goods available for sale . . . . . . . . . . . . 1,250 $20,500

2a. FIFO periodic method.

Cost of goods available for sale (from part 1) . . . . $ 20,500

Ending inventory*

Nov . 28 Purchase (100 @ $21) . . . . . . . . . . $2,100

Sep . 22 Purchase (250 @ $20) . . . . . . . . . . 5,000

May 9 Purchase (200 @ $16) . . . . . . . . . . 3,200

Ending inventory . . . . . . . . . . . . . . . . . . . . . . . . . . 10,300 Cost of goods sold . . . . . . . . . . . . . . . . . . . . . . . . $10,200

* FIFO assumes that the earlier costs are the first to flow out; thus, we determine ending inventory by assigning the most recent costs to the remaining items.

2b. LIFO periodic method.

Cost of goods available for sale (from part 1) . . . . $ 20,500

Ending inventory†

Jan . 1 Beg . inventory (400 @ $14) . . . $5,600

Mar . 10 Purchase (150 @ $15) . . . . . . . . 2,250

Ending inventory . . . . . . . . . . . . . . . . . . . . . . . . . . 7,850 Cost of goods sold . . . . . . . . . . . . . . . . . . . . . . . . $12,650

† LIFO assumes that the most recent (newest) costs are the first to flow out; thus, we determine ending inventory by assigning the earliest (oldest) costs to the remaining items.

Chapter 5 Inventories and Cost of Sales 209

2c. Weighted average periodic method.

Step 1: 400 units @ $14 = $ 5,600 200 units @ $15 =  3,000 300 units @ $16 = 4,800 250 units @ $20 = 5,000 100 units @ $21 = 2,100 1,250 units $20,500

Step 2: $20,500/1,250 units = $16.40 weighted average cost per unit Step 3: Total cost of 1,250 units available for sale . . . . . . . . . . . . . . $ 20,500 Less ending inventory priced on a weighted average cost basis: 550 units at $16 .40 each . . . . . . . . . . . . . . . . . . . 9,020 Cost of goods sold (700 units at $16.40 each) . . . . . . . . . $11,480

2d. Specific identification method.

Cost of goods available for sale (from part 1) . . . $  20,500 Ending inventory* May 9 Purchase (300 @ $16) . . . . . . . $4,800 Sep . 22 Purchase (150 @ $20) . . . . . . . 3,000 Nov . 28 Purchase (100 @ $21) . . . . . . . 2,100 Ending inventory . . . . . . . . . . . . . . . . . . . . . . . . . 9,900 Cost of goods sold . . . . . . . . . . . . . . . . . . . . . . . $10,600

*The additional tracking data provided are used to identify the items in ending inventory.

3.

Weighted Specific FIFO LIFO Average Identification

Income Statement Sales* . . . . . . . . . . . . . . . . . . . . . . . . . . . $ 22,500 $22,500 $ 22,500 $22,500

Cost of goods sold . . . . . . . . . . . . . . . . 10,200 12,650 11,480 10,600 Gross profit . . . . . . . . . . . . . . . . . . . . . $ 12,300 $ 9,850 $ 11,020 $11,900

Balance Sheet Inventory . . . . . . . . . . . . . . . . . . . . . . . . $10,300 $  7,850 $ 9,020 $   9,900

*Sales = (200 units × $30) + (200 units × $30) + (300 units × $35) = $22,500

4. Mistakenly using LIFO, when FIFO should have been used, overstates cost of goods sold in year 2019 by $2,450, which is the difference between the FIFO and LIFO amounts of ending inventory. It under- states income in 2019 by $2,450. In year 2020, income is overstated by $2,450 because of the under- statement in beginning inventory.

APPENDIX

Inventory Costing under a Periodic System 5A This section demonstrates inventory costing methods. We use information from Trekking, a sporting goods store. Among its many products, Trekking sells one type of mountain bike whose sales are directed at resorts that provide inexpensive bikes for guest use. We use Trekking’s data from August. Its mountain bike (unit) inventory at the beginning of August and its purchases and sales during August are shown in Exhibit 5A.1. It ends August with 12 bikes remaining in inventory.

P3 Compute inventory in a periodic system using the methods of specific identification, FIFO, LIFO, and weighted average.

Date Activity Units Acquired at Cost Units Sold at Retail Unit Inventory

Aug . 1 Beginning inventory . . . . . . . 10 units @ $ 91 = $ 910 10 units Aug . 3 Purchases . . . . . . . . . . . . . . . 15 units @ $106 = $ 1,590 25 units Aug . 14 Sales . . . . . . . . . . . . . . . . . . . 20 units @ $130 5 units

Aug . 17 Purchases . . . . . . . . . . . . . . . 20 units @ $115 = $ 2,300 25 units Aug . 28 Purchases . . . . . . . . . . . . . . . 10 units @ $119 = $ 1,190 35 units Aug . 30 Sales . . . . . . . . . . . . . . . . . . . 23 units @ $150 12 units Totals . . . . . . . . . . . . . . . . . . 55 units $5,990 43 units

Units available for sale Goods available for sale Units sold Units left

EXHIBIT 5A.1 Purchases and Sales of Goods

210 Chapter 5 Inventories and Cost of Sales

Trekking uses the periodic inventory system, which means that its Merchandise Inventory account is updated at the end of each period (monthly for Trekking) to reflect purchases and sales. Regardless of what inventory method is used, cost of goods available for sale must be allocated between cost of goods sold and ending inventory. (Many companies use the periodic system for tracking costs [not so much for sales]. Reasons include the use of standard costs by some companies and dollar-value LIFO by others. Also, the methods of specific identification and FIFO, used by a majority of companies, give the same result under the periodic and the perpetual systems.)

Specific Identification When each item in inventory can be matched with a specific purchase and invoice, we can use specific identification or SI to assign costs. We also need sales records that iden- tify exactly which items were sold and when. Trekking’s internal documents show the following specific unit sales.

August 14 Sold 8 bikes costing $91 each and 12 bikes costing $106 each. Total cost = $2,000. August 30 Sold 2 bikes costing $91 each, 3 bikes costing $106 each, 15 bikes costing $115 each, and

3 bikes costing $119 each. Total cost = $2,582.

Exhibit 5A.2 begins with the $5,990 in total units available for sale. For the 20 units sold on August 14, the total cost of sales is $2,000. Next, for the 23 units sold on August 30, the total cost of sales is $2,582. The total cost of sales for the period is $4,582. We then subtract this $4,582 in cost of goods sold from the $5,990 in cost of goods available to get $1,408 in ending inventory.

EXHIBIT 5A.2 Specific Identification Computations

Total cost of 55 units available for sale (from Exhibit 5A .1) . . . . . . . . . . . . . . . . . . . . . . . . . $ 5,990

Cost of goods sold Aug . 14 (8 @ $91) + (12 @ $106) . . . . . . . . . . . . . . . . . . . . . . . . . . . . . . . . . . . . . . . . . $2,000 Aug . 30 (2 @ $91) + (3 @ $106) + (15 @ $115) + (3 @ $119) . . . . . . . . . . . . . . . . . . 2,582 4,582 Ending inventory . . . . . . . . . . . . . . . . . . . . . . . . . . . . . . . . . . . . . . . . . . . . . . . . . . . . . . . . . $1,408

Trekking’s cost of goods sold reported on the income statement is $4,582, and ending inventory re- ported on the balance sheet is $1,408. The following graphic shows these cost flows.

Point: Specific identification is common for custom-made inven- tory. Examples include jewelers and fashion designers.

LAST BoughtFIRST Bought

SI Inventory $1,408

SI COGS $4,582

Point: SI yields identical results un- der both periodic and perpetual.

First-In, First-Out First-in, first-out (FIFO) assumes that inventory items are sold in the order acquired. When sales occur, the costs of the earliest units acquired are charged to cost of goods sold. This leaves the costs from the most recent purchases in ending inventory. Exhibit 5A.3 starts with $5,990 in total units available for sale. Applying FIFO, the 12 units in ending inventory are reported at the cost of the most recent 12 purchases. Reviewing purchases in reverse order, we assign costs to the 12 bikes in ending inventory as follows: $119 cost to 10 bikes and $115 cost to 2 bikes. This yields $1,420 in ending inventory. We subtract this $1,420 in ending inventory from $5,990 in cost of goods available to get $4,570 in cost of goods sold.

Point: For FIFO, COGS and ending inventory are the same for peri- odic and perpetual.

Chapter 5 Inventories and Cost of Sales 211

Last-In, First-Out Last-in, first-out (LIFO) assumes that the most recent purchases are sold first. These more recent costs are charged to goods sold, and the costs of the earliest purchases are assigned to inventory. Exhibit 5A.4 starts with $5,990 in total units available for sale. Applying LIFO, the 12 units in ending inventory are reported at the cost of the earliest 12 purchases. Reviewing the earliest purchases in order, we assign costs to the 12 bikes in ending inventory as follows: $91 cost to 10 bikes and $106 cost to 2 bikes. This yields $1,122 in ending inventory. We subtract this $1,122 in ending inventory from $5,990 in cost of goods available to get $4,868 in cost of goods sold.

Point: By assigning costs from the most recent purchases to cost of goods sold, LIFO comes closest to matching current costs of goods sold with revenues.

LAST BoughtFIRST Bought

FIFO COGS $4,570 FIFO Inventory $1,420

Total cost of 55 units available for sale (from Exhibit 5A .1) . . . . . . . . . . . . . . $ 5,990

Less ending inventory priced using FIFO

10 units from August 28 purchase at $119 each . . . . . . . . . . . . . . . . . . . . $1,190

2 units from August 17 purchase at $115 each . . . . . . . . . . . . . . . . . . . . . 230

Ending inventory . . . . . . . . . . . . . . . . . . . . . . . . . . . . . . . . . . . . . . . . . . . . . . . 1,420 Cost of goods sold . . . . . . . . . . . . . . . . . . . . . . . . . . . . . . . . . . . . . . . . . . . . . . $4,570

Exhibit 5A .1 shows that the 12 units in ending inventory consist of 10 units from the latest purchase on Aug . 28 and 2 units from the next latest purchase on Aug . 17 .

EXHIBIT 5A.3 FIFO Computations— Periodic System

Trekking’s ending inventory reported on the balance sheet is $1,420, and its cost of goods sold re- ported on the income statement is $4,570.

Trekking’s ending inventory reported on the balance sheet is $1,122, and its cost of goods sold re- ported on the income statement is $4,868.

LAST BoughtFIRST Bought

LIFO Inventory $1,122

LIFO COGS $4,868

Total cost of 55 units available for sale (from Exhibit 5A .1) . . . . . . . . . . . . . . . $ 5,990

Less ending inventory priced using LIFO

10 units in beginning inventory at $91 each . . . . . . . . . . . . . . . . . . . . . . . . $910

2 units from August 3 purchase at $106 each . . . . . . . . . . . . . . . . . . . . . . . 212

Ending inventory . . . . . . . . . . . . . . . . . . . . . . . . . . . . . . . . . . . . . . . . . . . . . . . . 1,122 Cost of goods sold . . . . . . . . . . . . . . . . . . . . . . . . . . . . . . . . . . . . . . . . . . . . . . $4,868

EXHIBIT 5A.4 LIFO Computations— Periodic System

Exhibit 5A .1 shows that the 12 units in ending inventory consist of 10 units from the earliest purchase (beg . inv .) and 2 units from the next earliest purchase on Aug . 3 .

Weighted Average Weighted average or WA (also called average cost) requires that we use the average cost per unit of inventory at the end of the period. Weighted average cost per unit equals the cost

212 Chapter 5 Inventories and Cost of Sales

Trekking’s ending inventory reported on the balance sheet is $1,307, and its cost of goods sold reported on the income statement is $4,683.

EXHIBIT 5A.5a Weighted Average Cost per Unit

Step 1: 10 units @ $ 91 = $       910 15 units @ $106 = 1,590 20 units @ $115 = 2,300 10 units @ $119 = 1,190 55 $5,990

Step 2: $5,990/55 units = $108.91 weighted average cost per unit

EXHIBIT 5A.5b Weighted Average Computations—Periodic

Step 3: Total cost of 55 units available for sale (from Exhibit 5A .1) . . . . . . . . . . . . . . . $ 5,990 Less ending inventory priced on a weighted average cost basis: 12 units at $108 .91 each (from Exhibit 5A .5a) . . . . . . . . . . . . . . . . 1,307 Cost of goods sold (43 units at $108.91 each) . . . . . . . . . . . . . . . . . . . . . . . $4,683

Step 3 uses the weighted average cost per unit to assign costs to ending inventory and to cost of goods sold, as shown in Exhibit 5A.5b.

of goods available for sale divided by the units available. The weighted average method has three steps. The first two steps are shown in Exhibit 5A.5a. Step 1 in Exhibit 5A.5a multiplies the per unit cost for beginning inventory and each purchase by the number of units (from Exhibit 5A.1). Step 2 adds these amounts and divides by the total number of units available for sale to find the weighted average cost per unit.

LAST BoughtFIRST Bought

WA Inventory $1,307

WA COGS $4,683

Financial Statement Effects of Costing Methods When purchase prices do not change, each inventory costing method assigns the same cost amounts to inventory and to cost of goods sold. When purchase prices are different, the methods assign different cost amounts. We show these dif- ferences in Exhibit 5A.6 using Trekking’s data.

Rising Costs When purchase costs regularly rise, as in Trekking’s case, the following occurs.

FIFO reports the lowest cost of goods sold—yielding the highest gross profit and net income. LIFO reports the highest cost of goods sold—yielding the lowest gross profit and net income. Weighted average yields results between FIFO and LIFO.

EXHIBIT 5A.6 Financial Statement Effects of Inventory Costing Methods

Trekking Company Specific Weighted For Month Ended August 31 Identification FIFO LIFO Average

Income Statement Sales . . . . . . . . . . . . . . . . . . . . . . . . . . . . . . . . . $ 6,050 $ 6,050 $ 6,050 $ 6,050

Cost of goods sold . . . . . . . . . . . . . . . . . . . . . 4,582 4,570 4,868 4,683 Gross profit . . . . . . . . . . . . . . . . . . . . . . . . . 1,468 1,480 1,182 1,367

Expenses . . . . . . . . . . . . . . . . . . . . . . . . . . . . . 450 450 450 450

Income before taxes . . . . . . . . . . . . . . . . . . . . 1,018 1,030 732 917

Income tax expense (30%) . . . . . . . . . . . . . . . 305 309 220 275

Net income . . . . . . . . . . . . . . . . . . . . . . . . . . . $   713 $   721 $   512 $   642 Balance Sheet Inventory . . . . . . . . . . . . . . . . . . . . . . . . . . . . . $1,408 $1,420 $1,122 $1,307

Chapter 5 Inventories and Cost of Sales 213

Falling Costs When costs regularly decline, the reverse occurs for FIFO and LIFO. FIFO gives the highest cost of goods sold—yielding the lowest gross profit and income. LIFO gives the lowest cost of goods sold—yielding the highest gross profit and income.

Method Advantages Each method offers advantages.

FIFO—inventory on the balance sheet approximates its current cost; it also follows the actual flow of goods for most businesses.

LIFO—cost of goods sold on the income statement approximates its current cost; it also better matches current costs with revenues.

Weighted average—smooths out erratic changes in costs. Specific identification—matches the costs of items with the revenues they generate.

Point: LIFO inventory is often less than the inventory’s replacement cost because LIFO inventory is valued using the oldest inventory purchase costs.

Periodic SI, FIFO, LIFO, and WA

NEED-TO-KNOW 5-7

P3

A company reported the following December purchases and sales data for its only product.

The company uses a periodic inventory system. Determine the cost assigned to ending inventory and to cost of goods sold using (a) specific identification, (b) FIFO, (c) LIFO, and (d) weighted average. (Round per unit costs and inventory amounts to cents.) For specific identification, ending inventory consists of 10 units, where 8 are from the December 30 purchase and 2 are from the December 8 purchase.

Solutions

a. Specific identification: Ending inventory—eight units from December 30 purchase and two units from December 8 purchase.

Date Activities Units Acquired at Cost Units Sold at Retail

Dec . 1 Beginning inventory . . . . . . . . . . . . . . 5 units @ $3 .00 = $ 15 .00 Dec . 8 Purchase . . . . . . . . . . . . . . . . . . . . . . . 10 units @ $4 .50 = 45 .00 Dec . 9 Sales . . . . . . . . . . . . . . . . . . . . . . . . . . . 8 units @ $7 .00

Dec . 19 Purchase . . . . . . . . . . . . . . . . . . . . . . . 13 units @ $5 .00 = 65 .00 Dec . 24 Sales . . . . . . . . . . . . . . . . . . . . . . . . . . . 18 units @ $8 .00

Dec . 30 Purchase . . . . . . . . . . . . . . . . . . . . . . . 8 units @ $5 .30 = 42 .40 Totals . . . . . . . . . . . . . . . . . . . . . . . . . . . . . . . 36 units $167 .40 26 units

b. FIFO—Periodic.

c. LIFO—Periodic.

Ending Cost of Specific Identification Inventory Goods Sold

(8 × $5 .30) + (2 × $4 .50) . . . . . . . . . . . . . . . . . . . . . . . . . . . . . . . . . . . . . . . . . . . . . . . . . . . $51 .40 (5 × $3 .00) + (8 × $4 .50) + (13 × $5 .00) + (0 × $5 .30) or $167 .40 [Total Goods Available] − $51 .40 [Ending Inventory] . . . . . . . . . . . . . . . . . . $116 .00

Ending Cost of FIFO Inventory Goods Sold

(8 × $5 .30) + (2 × $5 .00) . . . . . . . . . . . . . . . . . . . . . . . . . . . . . . . . . . . . . . . . . . . . . . . . . . $52 .40 (5 × $3 .00) + (10 × $4 .50) + (11 × $5 .00) or $167 .40 [Total Goods Available] − $52 .40 [Ending Inventory] . . . . . . . . . . . . . . . . . $115 .00

Ending Cost of LIFO Inventory Goods Sold

(5 × $3 .00) + (5 × $4 .50) . . . . . . . . . . . . . . . . . . . . . . . . . . . . . . . . . . . . . . . . . . . . . . . . . . $37 .50 (8 × $5 .30) + (13 × $5 .00) + (5 × $4 .50) or $167 .40 [Total Goods Available] − $37 .50 [Ending Inventory] . . . . . . . . . . . . . . . . . $129 .90

214 Chapter 5 Inventories and Cost of Sales

d. WA—Periodic.

Ending Cost of WA Inventory Goods Sold

10 × $4 .65 (computed from $167 .40/36) . . . . . . . . . . . . . . . . . . . . . . . . . . . . . . . . . . . . . $46 .50 26 × $4 .65 (computed from $167 .40/36) or $167 .40 [Total Goods Available] − $46 .50 [Ending Inventory] . . . . . . . . . . . . . . . . . $120 .90

Do More: QS 5-7, QS 5-8, QS 5-9, QS 5-14, QS 5-15, QS 5-16, QS 5-17, E 5-5

APPENDIX

Inventory Estimation Methods5B Inventory sometimes is estimated for two reasons. First, companies often report interim financial state- ments (financial statements prepared for periods of less than one year), but they only annually take a physi- cal count of inventory. Second, companies may require an inventory estimate if some casualty such as fire or flood makes taking a physical count impossible. Estimates are usually only required for companies that use the periodic system. Companies using a perpetual system would presumably have updated inventory data. This appendix describes two methods to estimate inventory.

Retail Inventory Method To avoid the time-consuming process of taking a physical inven- tory, some companies use the retail inventory method to estimate cost of goods sold and ending inventory. The retail inventory method uses a three-step process to estimate ending inventory. We need to know the amount of inventory a company had at the beginning of the period in both cost and retail amounts. We

already explained how to com- pute the cost of inventory. The retail amount of inventory is measured using selling prices of inventory items. We also need to know the net amount of goods purchased (minus returns, allow- ances, and discounts) in the pe- riod, both at cost and at retail. The amount of net sales at retail also is needed. The process is shown in Exhibit 5B.1.

The reasoning behind the re- tail inventory method is that if we can get a good estimate of the cost-to-retail ratio, we can multiply ending inventory at re- tail by this ratio to estimate end-

ing inventory at cost. Exhibit 5B.2 shows how these steps are applied to estimate ending inventory. First, we find that $100,000 of goods (at retail selling prices) were available for sale. A total of $70,000 of these

P4 Apply both the retail inventory and gross profit methods to estimate inventory.

Step 2

Step 1

Step 3

Goods available for sale at retail

Net sales at retail

Ending inventory at retail

Goods available for sale at cost

Goods available for sale at retail

Cost-to- retail ratio

Ending inventory at retail

Cost-to- retail ratio

Estimated ending inventory

at cost ×

÷

=

=

=

EXHIBIT 5B.1 Retail Inventory Method of Inventory Estimation

EXHIBIT 5B.2 Estimated Inventory Using the Retail Inventory Method

At Cost At Retail

Goods available for sale

Beginning inventory . . . . . . . . . . . . . . . . . . . . . . . . . . . . . . . . . . . . . . . . . . . . . $  20,500 $  34,500

Cost of goods purchased . . . . . . . . . . . . . . . . . . . . . . . . . . . . . . . . . . . . . . . . . 39,500 65,500

Goods available for sale . . . . . . . . . . . . . . . . . . . . . . . . . . . . . . . . . . . . . . . . . . 60,000 100,000

Step 1: Deduct net sales at retail . . . . . . . . . . . . . . . . . . . . . . . . . . . . . . . . . . . . . . . . . . 70,000 Ending inventory at retail . . . . . . . . . . . . . . . . . . . . . . . . . . . . . . . . . . . . . . . . . . $ 30,000

Step 2: Cost-to-retail ratio: ($60,000 ÷ $100,000) = 60%

Step 3: Estimated ending inventory at cost ($30,000 × 60%) . . . . . . . . . . . . . . . . . . $18,000

Chapter 5 Inventories and Cost of Sales 215

goods were sold, leaving $30,000 (retail value) of merchandise in ending inventory. Second, the cost of these goods is 60% of the $100,000 retail value. Third, because cost for these goods is 60% of retail, the estimated cost of ending inventory is $18,000.

Gross Profit Method The gross profit method estimates the cost of ending inventory by applying the gross profit ratio to net sales (at retail). This type of estimate often is used when inventory is destroyed, lost, or stolen. This method uses the historical rela- tion between cost of goods sold and net sales to estimate the pro- portion of cost of goods sold making up current sales. This cost of goods sold estimate is then subtracted from cost of goods available for sale to esti- mate the ending inventory at cost. These two steps are shown in Exhibit 5B.3. To demonstrate, assume that a company’s inventory is destroyed by fire in March. When the fire oc- curs, the company’s accounts show the following balances for January through March: Net Sales, $30,000; Beginning Inventory, $12,000 (at January 1); and Cost of Goods Purchased, $20,500. If this company’s gross profit ratio is 30%, then 30% of each net sales dollar is gross profit and 70% is cost of goods sold. We show in Exhibit 5B.4 how this 70% is used to estimate lost inventory of $11,500.

Step 2

Step 1 Net sales at retail

1.0 – Gross profit ratio

Estimated cost of

goods sold

Goods available for sale at cost

Estimated cost of

goods sold

Estimated ending

inventory at cost

×

=

=

EXHIBIT 5B.3 Gross Profit Method of Inventory Estimation

EXHIBIT 5B.4 Estimated Inventory Using the Gross Profit Method

Goods available for sale

Beginning inventory, January 1 . . . . . . . . . . . . . . . . . . . . . . . . . . . . . . $ 12,000

Cost of goods purchased . . . . . . . . . . . . . . . . . . . . . . . . . . . . . . . . . . . 20,500

Goods available for sale (at cost) . . . . . . . . . . . . . . . . . . . . . . . . . . . . . 32,500

Net sales at retail . . . . . . . . . . . . . . . . . . . . . . . . . . . . . . . . . . . . . . . . . . . . $30,000

Step 1: Estimated cost of goods sold ($30,000 × 70%) . . . . . . . . . . . . . . . . . (21,000) × 0 .70 Step 2: Estimated March inventory at cost . . . . . . . . . . . . . . . . . . . . . . . . . . . . $11,500

Using the retail method and the following data, estimate the cost of ending inventory.

Retail Inventory Estimation

NEED-TO-KNOW 5-8

Solution

Estimated ending inventory (at cost) is $327,000. It is computed as follows.

Step 1: ($530,000 + $335,000) − $320,000 = $545,000

Step 2: $324,000 + $195,000 $530,000 + $335,000

= 60%

Step 3: $545,000 × 60% = $327,000

P4

Cost Retail

Beginning inventory . . . . . . . . . . . . . . . . . $324,000 $530,000

Cost of goods purchased . . . . . . . . . . . . . 195,000 335,000

Net sales . . . . . . . . . . . . . . . . . . . . . . . . . . 320,000

Do More: QS 5-22, E 5-16, E 5-17, P 5-9

INVENTORY BASICS FOB shipping point: Goods are included in buyer’s inventory once they are shipped. FOB destination: Goods are included in buyer’s inventory after arrival at their destination.

Summary: Cheat Sheet

Consignee: Never reports consigned goods in inventory; stays in consign- or’s inventory until sold. Merchandise inventory: Includes any necessary costs to make an item ready for sale. Examples—shipping, storage, import fees, and insurance.

216 Chapter 5 Inventories and Cost of Sales

INVENTORY COSTING FIFO: Earliest units purchased are the first to be reported as cost of goods sold. LIFO: Latest units purchased are the first to be reported as cost of goods sold. Weighted average: The weighted average cost per unit (formula below) of inventory at the time of each sale is reported as cost of goods sold.

Cost of goods available for sale (at each sale) Number of units available for sale (at each sale)

Specific identification: Each unit is assigned a cost, and when that unit is sold, its cost is reported as cost of goods sold.

Financial Statement Effects Rising Costs—FIFO reports lowest cost of goods sold and highest net income. LIFO reports highest cost of goods sold and lowest income. Weighted average reports results in between LIFO and FIFO. Falling Costs—FIFO reports highest cost of goods sold and lowest net income. LIFO reports lowest cost of goods sold and highest income.

INVENTORY VALUATION, ERRORS, & ANALYSIS Lower of cost or market (LCM): When market value of inventory is lower than its cost, a loss is recorded. When market value is higher than cost of inventory, no adjustment is made. LCM Example (applied to individual items separately)

LCM Journal Entry: To get from $220,000 reported inventory to the $190,000 LCM inventory, make the following entry.

Cost of Goods Sold . . . . . . . . . . . . . . . . . . . . . . . . . 30,000 Merchandise Inventory . . . . . . . . . . . . . . . . . 30,000

Effects of Overstated or Understated Inventory for Income Statement

Year 1 Year 2

Ending Inventory Cost of Goods Sold Net Income Cost of Goods Sold Net Income

Understated  Overstated  Understated  Understated  Overstated 

Overstated  Understated  Overstated  Overstated  Understated 

Effects of Overstated or Understated Inventory for Balance Sheet

Ending Inventory Assets Equity

Understated . . . . Understated Understated

Overstated . . . . . Overstated Overstated

Average cost (196, 211) Consignee (191) Consignor (191) Days’ sales in inventory (203) First-in, first-out (FIFO) (195, 210)

Gross profit method (215) Interim financial statements (214) Inventory turnover (203) Last-in, first-out (LIFO) (195, 211) Lower of cost or market (LCM) (200)

Net realizable value (192) Retail inventory method (214) Specific identification (SI) (194, 210) Weighted average (WA) (196, 211)

Key Terms

Multiple Choice Quiz

Use the following information from Marvel Company for the month of July to answer questions 1 through 4.

1. Perpetual: Assume that Marvel uses a perpetual FIFO inven- tory system. What is the dollar value of its ending inventory? a. $2,940 c. $2,625 e. $2,705 b. $2,685 d. $2,852

2. Perpetual: Assume that Marvel uses a perpetual LIFO inven- tory system. What is the dollar value of its ending inventory? a. $2,940 c. $2,625 e. $2,705 b. $2,685 d. $2,852

July 1 Beginning inventory . . . . . . . . . . . . 75 units @ $25 each

July 3 Purchase . . . . . . . . . . . . . . . . . . . . . 348 units @ $27 each

July 8 Sale . . . . . . . . . . . . . . . . . . . . . . . . . 300 units

July 15 Purchase . . . . . . . . . . . . . . . . . . . . . 257 units @ $28 each

July 23 Sale . . . . . . . . . . . . . . . . . . . . . . . . . 275 units

$180 3

= $60 each

2. Last-in, first-out (LIFO) Costs flow in the reverse

order incurred.

3. Weighted average Costs flow at an average

of costs available.

1. First-in, first-out (FIFO) Costs flow in the order

incurred.

× 2× 1

Income Statement Net sales.................... $100 Cost of goods sold.. 45 Gross profit................ $ 55

Balance Sheet Inventory.................... $135

Income Statement Net sales.................... $100 Cost of goods sold.. 70 Gross profit................ $ 30

Balance Sheet Inventory.................... $1 10

Income Statement Net sales.................... $100 Cost of goods sold.. 60 Gross profit................ $ 40

Balance Sheet Inventory..................... $120

$65 May 3

$45 May 1 G

o o d s

s o l d

G o o d s

s o l d

G o o d s

s o l d

G o o d s

l e f t

G o o d s

l e f t

G o o d s

l e f t

$70 May 6

$65 May 3

$45 May 1

$70 May 6

$70 May 6

$65 May 3

$45 May 1

Cost Flow Assumptions Example

Per Unit Inventory Total Total LCM Applied Items Units Cost Market Cost Market to Items

Roadster 20 $8,500 $7,000 $170,000 $140,000 $  140,000

Sprint 10 5,000 6,000 50,000 60,000 50,000

Totals $220,000 $190,000

Roadster: $140,000 is the lower of the $170,000 cost and $140,000 market. Sprint: $50,000 is the lower of the $50,000 cost and $60,000 market. LCM: Results in a $190,000 reported inventory.

Chapter 5 Inventories and Cost of Sales 217

3. Perpetual and Periodic: Assume that Marvel uses a spe- cific identification inventory system. Its ending inventory consists of 20 units from beginning inventory, 40 units from the July 3 purchase, and 45 units from the July 15 purchase. What is the dollar value of its ending inventory? a. $2,940 c. $2,625 e. $2,840 b. $2,685 d. $2,852

4.A Periodic: Assume that Marvel uses a periodic FIFO inven- tory system. What is the dollar value of its ending inventory? a. $2,940 c. $2,625 e. $2,705 b. $2,685 d. $2,852

5.A Periodic: A company reports the following beginning in- ventory and purchases, and it ends the period with 30 units in inventory.

i) Compute ending inventory using the FIFO periodic system.

a. $400 b. $1,460 c. $1,360 d. $300 ii) Compute cost of goods sold using the LIFO periodic

system. a. $400 b. $1,460 c. $1,360 d. $300 6. A company has cost of goods sold of $85,000 and ending

inventory of $18,000. Its days’ sales in inventory equals a. 49.32 days. c. 4.72 days. e. 1,723.61 days. b. 0.21 day. d. 77.29 days.

Beginning inventory . . . . . . . . . 100 units at $10 cost per unit Purchase 1 . . . . . . . . . . . . . . . . 40 units at $12 cost per unit Purchase 2 . . . . . . . . . . . . . . . . 20 units at $14 cost per unit

ANSWERS TO MULTIPLE CHOICE QUIZ

1. a; FIFO perpetual

2. b; LIFO perpetual

3. e; Specific identification (perpetual and periodic are identical for specific identification)—Ending inventory computation follows.

4. a; FIFO periodic. Ending inventory computation: 105 units @ $28 each = $2,940. (Hint: FIFO periodic inventory computation is identical to the FIFO perpetual inventory computation.)

5. i) a; FIFO periodic inventory = (20 × $14) + (10 × $12) = $400 ii) b; LIFO periodic cost of goods sold = (20 × $14) + (40 × $12)

+ (70 × $10) = $1,460 6. d; Days’ sales in inventory = (Ending inventory/Cost of goods sold)

× 365 = ($18,000∕$85,000) × 365 = 77.29 days

Date Goods Purchased Cost of Goods Sold Inventory Balance

July 1 75 units @ $25 = $ 1,875 July 3 348 units @ $27 = $9,396 75 units @ $25 348 units @ $27

= $11,271

July 8 75 units @ $25 123 units @ $27 = $ 3,321 225 units @ $27

= $ 7,950

July 15 257 units @ $28 = $7,196 123 units @ $27 257 units @ $28

= $10,517

July 23 123 units @ $27 105 units @ $28 = $ 2,940 152 units @ $28

= $ 7,577

$15,527

⎧ ⎨ ⎩

⎧ ⎨ ⎩

⎧ ⎨ ⎩

⎧ ⎨ ⎩

⎧ ⎨ ⎩

⎧ ⎪ ⎨ ⎪ ⎩

Date Goods Purchased Cost of Goods Sold Inventory Balance

July 1 75 units @ $25 = $ 1,875 July 3 348 units @ $27 = $9,396 75 units @ $25 348 units @ $27

= $ 11,271

July 8 300 units @ $27 = $ 8,100 75 units @ $25 48 units @ $27

= $ 3,171

July 15 257 units @ $28 = $7,196 75 units @ $25 48 units @ $27

257 units @ $28 = $10,367

July 23 257 units @ $28 75 units @ $25

18 units @ $27 = $ 7,682

30 units @ $27 = $ 2,685

$15,782

⎧ ⎨ ⎩

⎧ ⎨ ⎩

⎧ ⎨ ⎩

20 units @ $25 $ 500

40 units @ $27 1,080

45 units @ $28 1,260

105 units $2,840

218 Chapter 5 Inventories and Cost of Sales

A(B) Superscript letter A or B denotes assignments based on Appendix 5A or 5B.

Icon denotes assignments that involve decision making.

1. Describe how costs flow from inventory to cost of goods sold for the following methods: (a) FIFO and (b) LIFO.

2. Where is the amount of merchandise inventory disclosed in the financial statements?

3. If costs are declining, will the LIFO or FIFO method of inventory valuation yield the lower cost of goods sold? Why?

4. If inventory errors are said to correct themselves, why are accounting users concerned when such errors are made?

5. Explain the following statement: “Inventory errors correct themselves.”

6. What is the meaning of market as it is used in determining the lower of cost or market for inventory?

7. What factors contribute to (or cause) inventory shrinkage?

8.B When preparing interim financial statements, what two methods can companies utilize to estimate cost of goods sold and ending inventory?

9. Refer to Apple’s financial statements in Appendix A. On September 30, 2017, what percent of current assets is represented by inventory?

10. Refer to Apple’s financial statements in Appendix A and compute its cost of goods available for sale for the year ended September 30, 2017.

11. Refer to Samsung’s financial statements in Appendix A. Compute its cost of goods available for sale for the year ended December 31, 2017.

12. Refer to Samsung’s financial statements in Appendix A. What percent of its cur- rent assets is inventory as of December 31, 2017 and 2016?

Discussion Questions

APPLE

APPLE

Samsung

Samsung

QUICK STUDY

QS 5-1 Inventory ownership

C1

Homestead Crafts, a distributor of handmade gifts, operates out of owner Emma Finn’s house. At the end of the current period, Emma looks over her inventory and finds that she has

∙ 1,300 units (products) in her basement, 20 of which were damaged by water and cannot be sold. ∙ 350 units in her van, ready to deliver per a customer order, terms FOB destination. ∙ 80 units out on consignment to a friend who owns a retail store.

How many units should Emma include in her company’s period-end inventory?

A car dealer acquires a used car for $14,000, with terms FOB shipping point. Compute total inventory costs assigned to the used car if additional costs include

∙ $250 for transportation-in. ∙ $150 for advertising. ∙ $300 for shipping insurance. ∙ $1,250 for sales staff salaries. ∙ $900 for car import duties. ∙ $180 for trimming shrubs.

QS 5-2 Inventory costs

C2

Wattan Company reports beginning inventory of 10 units at $60 each. Every week for four weeks it pur- chases an additional 10 units at respective costs of $61, $62, $65, and $70 per unit for weeks 1 through 4. Compute the cost of goods available for sale and the units available for sale for this four-week period. Assume that no sales occur during those four weeks.

QS 5-3 Computing goods available for sale P1

A company reports the following beginning inventory and two purchases for the month of January. On January 26, the company sells 350 units. Ending inventory at January 31 totals 150 units.

QS 5-4 Perpetual: Inventory costing with FIFO

P1 Units Unit Cost

Beginning inventory on January 1 . . . . . . . . . . . . . 320 $3 .00

Purchase on January 9 . . . . . . . . . . . . . . . . . . . . . . 80 3 .20

Purchase on January 25 . . . . . . . . . . . . . . . . . . . . . 100 3 .34

Required

Assume the perpetual inventory system is used. Determine the costs assigned to ending inventory when costs are assigned based on the FIFO method. (Round per unit costs and inventory amounts to cents.)

Chapter 5 Inventories and Cost of Sales 219

Refer to the information in QS 5-4 and assume the perpetual inventory system is used. Determine the costs assigned to ending inventory when costs are assigned based on LIFO. (Round per unit costs and inventory amounts to cents.)

QS 5-5 Perpetual: Inventory costing with LIFO P1

Refer to the information in QS 5-10 and assume the perpetual inventory system is used. Determine the costs assigned to ending inventory when costs are assigned based on the LIFO method. (Round per unit costs and inventory amounts to cents.)

QS 5-11 Perpetual: Inventory costing with LIFO P1

Refer to the information in QS 5-4 and assume the perpetual inventory system is used. Determine the costs assigned to ending inventory when costs are assigned based on the weighted average method. (Round per unit costs and inventory amounts to cents.)

QS 5-6 Perpetual: Inventory costing with weighted average P1

Refer to the information in QS 5-4 and assume the periodic inventory system is used. Determine the costs assigned to ending inventory when costs are assigned based on the weighted average method. (Round per unit costs and inventory amounts to cents.)

QS 5-9A Periodic: Inventory costing with weighted average P3

Refer to the information in QS 5-4 and assume the periodic inventory system is used. Determine the costs assigned to ending inventory when costs are assigned based on the FIFO method. (Round per unit costs and inventory amounts to cents.)

QS 5-7A Periodic: Inventory costing with FIFO P3

Refer to the information in QS 5-4 and assume the periodic inventory system is used. Determine the costs assigned to ending inventory when costs are assigned based on the LIFO method. (Round per unit costs and inventory amounts to cents.)

QS 5-8A Periodic: Inventory costing with LIFO P3

Trey Monson starts a merchandising business on December 1 and enters into the following three inventory purchases. Also, on December 15, Monson sells 15 units for $20 each.

QS 5-10 Perpetual: Assigning costs with FIFO

P1Purchases on December 7 . . . . . . 10 units @ $ 6 .00 cost Purchases on December 14 . . . . . 20 units @ $12 .00 cost

Purchases on December 21 . . . . . 15 units @ $14 .00 cost

Required

Monson uses a perpetual inventory system. Determine the costs assigned to the December 31 ending in- ventory based on the FIFO method. (Round per unit costs and inventory amounts to cents.)

Refer to the information in QS 5-10 and assume the perpetual inventory system is used. Determine the costs assigned to ending inventory when costs are assigned based on the weighted average method. (Round per unit costs and inventory amounts to cents.)

QS 5-12 Perpetual: Inventory costing with weighted average P1

Refer to the information in QS 5-10 and assume the perpetual inventory system is used. Determine the costs assigned to ending inventory when costs are assigned based on specific identification. Of the units sold, eight are from the December 7 purchase and seven are from the December 14 purchase. (Round per unit costs and inventory amounts to cents.)

QS 5-13 Perpetual: Inventory costing with specific identification P1

Refer to the information in QS 5-10 and assume the periodic inventory system is used. Determine the costs assigned to ending inventory when costs are assigned based on the FIFO method. (Round per unit costs and inventory amounts to cents.)

QS 5-14A Periodic: Inventory costing with FIFO P3

Refer to the information in QS 5-10 and assume the periodic inventory system is used. Determine the costs assigned to ending inventory when costs are assigned based on the LIFO method. (Round per unit costs and inventory amounts to cents.)

QS 5-15A Periodic: Inventory costing with LIFO P3

220 Chapter 5 Inventories and Cost of Sales

Refer to the information in QS 5-10 and assume the periodic inventory system is used. Determine the costs assigned to ending inventory when costs are assigned based on the weighted average method. (Round per unit costs and inventory amounts to cents.)

QS 5-16A Periodic: Inventory costing with weighted average P3

Refer to the information in QS 5-10 and assume the periodic inventory system is used. Determine the costs assigned to ending inventory when costs are assigned based on specific identification. Of the units sold, eight are from the December 7 purchase and seven are from the December 14 purchase. (Round per unit costs and inventory amounts to cents.)

QS 5-17A Periodic: Inventory costing with specific identification P3

Endor Company begins the year with $140,000 of goods in inventory. At year-end, the amount in inven- tory has increased to $180,000. Cost of goods sold for the year is $1,200,000. Compute Endor’s inventory turnover and days’ sales in inventory. Assume there are 365 days in the year.

QS 5-21 Analyzing inventory A3

Ames Trading Co. has the following products in its ending inventory. Compute lower of cost or market for inventory applied separately to each product.

QS 5-19 Applying LCM to inventories

P2 Product Quantity Cost per Unit Market per Unit

Mountain bikes . . . . . . . . . . . 11 $600 $550

Skateboards . . . . . . . . . . . . . 13 350 425

Gliders . . . . . . . . . . . . . . . . . . 26 800 700

Identify the inventory costing method (SI, FIFO, LIFO, or WA) best described by each of the following separate statements. Assume a period of increasing costs.

1. Results in the highest cost of goods sold. 2. Yields the highest net income. 3. Has the lowest tax expense because of reporting the lowest net income. 4. Better matches current costs with revenues. 5. Precisely matches the costs of items with the revenues they generate.

QS 5-18 Contrasting inventory costing methods

A1

In taking a physical inventory at the end of Year 1, Grant Company forgot to count certain units and un- derstated ending inventory by $10,000. Determine how this error affects each of the following. a. Year 1 cost of goods sold c. Year 2 cost of goods sold b. Year 1 net income d. Year 2 net income

QS 5-20 Inventory errors

A2

Confucius Bookstore’s inventory is destroyed by a fire on September 5. The following data for the current year are available from the accounting records. Estimate the cost of the inventory destroyed.

QS 5-22B Estimating inventories— gross profit method

P4 Beginning inventory, Jan . 1 . . . . . . . . . . . . . . . . . . . . . . $190,000 Jan . 1 through Sept . 5 purchases (net) . . . . . . . . . . . . . $352,000

Jan . 1 through Sept . 5 sales (net) . . . . . . . . . . . . . . . . . $685,000

Current year’s estimated gross profit rate . . . . . . . . . . 44%

A solar panel dealer acquires a used panel for $9,000, with terms FOB shipping point. Compute total in- ventory costs assigned to the used panel if additional costs include

∙ $1,500 for sales staff salaries. ∙ $135 for shipping insurance. ∙ $280 for transportation-in by train. ∙ $550 for used panel restoration. ∙ $110 for online advertising. ∙ $300 for lawn care.

QS 5-23 Inventory costs

C2

EXERCISES

Exercise 5-1 Inventory ownership C1

1. At year-end, Barr Co. had shipped $12,500 of merchandise FOB destination to Lee Co. Which com- pany should include the $12,500 of merchandise in transit as part of its year-end inventory?

2. Parris Company has shipped $20,000 of goods to Harlow Co., and Harlow Co. has arranged to sell the goods for Parris. Identify the consignor and the consignee. Which company should include any unsold goods as part of its inventory?

Chapter 5 Inventories and Cost of Sales 221

Walberg Associates, antique dealers, purchased goods for $75,000. Terms of the purchase were FOB ship- ping point, and the cost of transporting the goods to Walberg Associates’s warehouse was $2,400. Walberg Associates insured the shipment at a cost of $300. Prior to putting the goods up for sale, they cleaned and refurbished them at a cost of $980. Determine the cost of inventory.

Exercise 5-2 Inventory costs

C2

Laker Company reported the following January purchases and sales data for its only product. Exercise 5-3 Perpetual: Inventory costing methods

P1 Date Activities Units Acquired at Cost Units Sold at Retail

Jan . 1 Beginning inventory . . . . . . . . . . . . 140 units @ $6 .00 = $ 840 Jan . 10 Sales . . . . . . . . . . . . . . . . . . . . . . . . . 100 units @ $15

Jan . 20 Purchase . . . . . . . . . . . . . . . . . . . . . 60 units @ $5 .00 = 300 Jan . 25 Sales . . . . . . . . . . . . . . . . . . . . . . . . . 80 units @ $15

Jan . 30 Purchase . . . . . . . . . . . . . . . . . . . . . 180 units @ $4 .50 = 810 Totals . . . . . . . . . . . . . . . . . . . . . . . . 380 units $1,950 180 units

Required

The company uses a perpetual inventory system. Determine the cost assigned to ending inventory and to cost of goods sold using (a) specific identification, (b) weighted average, (c) FIFO, and (d) LIFO. (Round per unit costs and inventory amounts to cents.) For specific identification, ending inventory consists of 200 units, where 180 are from the January 30 purchase, 5 are from the January 20 purchase, and 15 are from beginning inventory.

Check Ending inventory: LIFO, $930; WA, $918

Use the data in Exercise 5-3 to prepare comparative income statements for the month of January for Laker Company similar to those shown in Exhibit 5.8 for the four inventory methods. Assume expenses are $1,250 and the applicable income tax rate is 40%. (Round amounts to cents.) 1. Which method yields the highest net income? 2. Does net income using weighted average fall above, between, or below that using FIFO and LIFO? 3. If costs were rising instead of falling, which method would yield the highest net income?

Exercise 5-4 Perpetual: Income effects of inventory methods

A1

Refer to the information in Exercise 5-3 and assume the periodic inventory system is used. Determine the costs assigned to ending inventory and to cost of goods sold using (a) specific identification, (b) weighted average, (c) FIFO, and (d) LIFO. (Round per unit costs and inventory amounts to cents.) For specific identification, ending inventory consists of 200 units, where 180 are from the January 30 purchase, 5 are from the January 20 purchase, and 15 are from beginning inventory.

Exercise 5-5A Periodic: Inventory costing

P3

Use the data and results from Exercise 5-5 to prepare comparative income statements for the month of January for the company similar to those shown in Exhibit 5.8 for the four inventory methods. Assume expenses are $1,250 and the applicable income tax rate is 40%. (Round amounts to cents.)

Required

1. Which method yields the highest net income? 2. Does net income using weighted average fall above, between, or below that using FIFO and LIFO? 3. If costs were rising instead of falling, which method would yield the highest net income?

Exercise 5-6A Periodic: Income effects of inventory methods

P3 A1

Hemming Co. reported the following current-year purchases and sales for its only product. Exercise 5-7 Perpetual: Inventory costing methods—FIFO and LIFO

P1

Date Activities Units Acquired at Cost Units Sold at Retail

Jan . 1 Beginning inventory . . . . . . . . . . . . 200 units @ $10 = $ 2,000 Jan . 10 Sales . . . . . . . . . . . . . . . . . . . . . . . . 150 units @ $40

Mar . 14 Purchase . . . . . . . . . . . . . . . . . . . . 350 units @ $15 = 5,250 Mar . 15 Sales . . . . . . . . . . . . . . . . . . . . . . . . 300 units @ $40

July 30 Purchase . . . . . . . . . . . . . . . . . . . . 450 units @ $20 = 9,000 Oct . 5 Sales . . . . . . . . . . . . . . . . . . . . . . . . 430 units @ $40

Oct . 26 Purchase . . . . . . . . . . . . . . . . . . . . 100 units @ $25 = 2,500 Totals . . . . . . . . . . . . . . . . . . . . . . . 1,100 units $18,750 880 units

222 Chapter 5 Inventories and Cost of Sales

Required

Hemming uses a perpetual inventory system. Determine the costs assigned to ending inventory and to cost of goods sold using (a) FIFO and (b) LIFO. (c) Compute the gross margin for each method. (Round amounts to cents.)

Check Ending inventory: LIFO, $4,150

Refer to the information in Exercise 5-7. Ending inventory consists of 45 units from the March 14 purchase, 75 units from the July 30 purchase, and all 100 units from the October 26 purchase. Using the specific iden- tification method, compute (a) the cost of goods sold and (b) the gross profit. (Round amounts to cents.)

Exercise 5-8 Specific identification P1

Refer to the information in Exercise 5-7 and assume the periodic inventory system is used. Determine the costs assigned to ending inventory and to cost of goods sold using (a) FIFO and (b) LIFO. (c) Compute the gross margin for each method.

Exercise 5-9A Periodic: Inventory costing P3

Martinez Company’s ending inventory includes the following items. Compute the lower of cost or market for ending inventory applied separately to each product.

Exercise 5-10 Lower of cost or market

P2 Product Units Cost per Unit Market per Unit

Helmets . . . . . . . . . . 24 $50 $54

Bats . . . . . . . . . . . . . 17 78 72

Shoes . . . . . . . . . . . . 38 95 91

Uniforms . . . . . . . . . 42 36 36 Check LCM = $7,394

Vibrant Company had $850,000 of sales in each of Year 1, Year 2, and Year 3, and it purchased merchan- dise costing $500,000 in each of those years. It also maintained a $250,000 physical inventory from the beginning to the end of that three-year period. In accounting for inventory, it made an error at the end of Year 1 that caused its Year 1 ending inventory to appear on its statements as $230,000 rather than the correct $250,000. 1. Determine the correct amount of the company’s gross profit in each of Year 1, Year 2, and Year 3. 2. Prepare comparative income statements as in Exhibit 5.11 to show the effect of this error on the com-

pany’s cost of goods sold and gross profit for each of Year 1, Year 2, and Year 3.

Exercise 5-12 Analyzing inventory errors

A2

Cruz Company uses LIFO for inventory costing and reports the following financial data. It also recom- puted inventory and cost of goods sold using FIFO for comparison purposes.

Exercise 5-11 Comparing LIFO numbers to FIFO numbers; ratio analysis

A1 A3 Year 2 Year 1

LIFO inventory . . . . . . . . . . . . . . . . . . . . . . . . $160 $110

LIFO cost of goods sold . . . . . . . . . . . . . . . . 740 680

FIFO inventory . . . . . . . . . . . . . . . . . . . . . . . . 240 110

FIFO cost of goods sold . . . . . . . . . . . . . . . . 660 645

Current assets (using LIFO) . . . . . . . . . . . . . 220 180

Current assets (using FIFO) . . . . . . . . . . . . . . 300 180

Current liabilities . . . . . . . . . . . . . . . . . . . . . . 200 170

1. Compute its current ratio, inventory turnover, and days’ sales in inventory for Year 2 using (a) LIFO numbers and (b) FIFO numbers. (Round answers to one decimal.)

2. Comment on and interpret the results of part 1.

Check (1) FIFO: Current ratio, 1.5; Inventory turnover, 3.8 times

Use the following information for Palmer Co. to compute inventory turnover for Year 3 and Year 2, and its days’ sales in inventory at December 31, Year 3 and Year 2. (Round answers to one decimal.) Comment on Palmer’s efficiency in using its assets to increase sales from Year 2 to Year 3.

Exercise 5-13 Inventory turnover and days’ sales in inventory

A3 Year 3 Year 2 Year 1

Cost of goods sold . . . . . . . . . . . . $643,825 $426,650 $391,300

Ending inventory . . . . . . . . . . . . . . 97,400 87,750 92,500

Chapter 5 Inventories and Cost of Sales 223

Lopez Company reported the following current-year data for its only product. The company uses a peri- odic inventory system, and its ending inventory consists of 150 units—50 from each of the last three pur- chases. Determine the cost assigned to ending inventory and to cost of goods sold using (a) specific identification, (b) weighted average, (c) FIFO, and (d) LIFO. (Round per unit costs and inventory amounts to cents.) (e) Which method yields the highest net income?

Exercise 5-14A Periodic: Cost flow assumptions

P3

Jan . 1 Beginning inventory . . . . . . . . . . . 96 units @ $2 .00 = $ 192 Mar . 7 Purchase . . . . . . . . . . . . . . . . . . . . 220 units @ $2 .25 = 495 July 28 Purchase . . . . . . . . . . . . . . . . . . . . 544 units @ $2 .50 = 1,360 Oct . 3 Purchase . . . . . . . . . . . . . . . . . . . . 480 units @ $2 .80 = 1,344 Dec . 19 Purchase . . . . . . . . . . . . . . . . . . . . 160 units @ $2 .90 = 464 Totals . . . . . . . . . . . . . . . . . . . . . . . 1,500 units $3,855 Check Inventory; LIFO,

$313.50; FIFO, $435.00

Flora’s Gifts reported the following current-month data for its only product. The company uses a periodic inventory system, and its ending inventory consists of 60 units—50 units from the January 6 purchase and 10 units from the January 25 purchase. Determine the cost assigned to ending inventory and to cost of goods sold using (a) specific identification, (b) weighted average, (c) FIFO, and (d) LIFO. (Round per unit costs and inventory amounts to cents.) (e) Which method yields the lowest net income?

Exercise 5-15A Periodic: Cost flow assumptions

P3

Jan . 1 Beginning inventory . . . . . . . . . . . 138 units @ $3 .00 = $ 414 Jan . 6 Purchase . . . . . . . . . . . . . . . . . . . . 300 units @ $2 .80 = 840 Jan . 17 Purchase . . . . . . . . . . . . . . . . . . . . 540 units @ $2 .30 = 1,242 Jan . 25 Purchase . . . . . . . . . . . . . . . . . . . . 22 units @ $2 .00 = 44 Totals . . . . . . . . . . . . . . . . . . . . . . . 1,000 units $2,540 Check Inventory: LIFO,

$180.00; FIFO, $131.40

Dakota Company had net sales (at retail) of $260,000. The following additional information is available from its records. Use the retail inventory method to estimate Dakota’s year-end inventory at cost.

Exercise 5-16B Estimating ending inventory—retail method

P4 At Cost At Retail

Beginning inventory . . . . . . . . . . . . . . . . $ 63,800 $128,400

Cost of goods purchased . . . . . . . . . . . . 115,060 196,800 Check End. inventory at cost, $35,860

On January 1, JKR Shop had $225,000 of beginning inventory at cost. In the first quarter of the year, it purchased $795,000 of merchandise, returned $11,550, and paid freight charges of $18,800 on purchased merchandise, terms FOB shipping point. The company’s gross profit averages 30%, and the store had $1,000,000 of net sales (at retail) in the first quarter of the year.

Use the gross profit method to estimate its cost of inventory at the end of the first quarter.

Exercise 5-17B Estimating ending inventory—gross profit method P4

Required

The company uses a perpetual inventory system. Determine the costs assigned to ending inventory and to cost of goods sold using (a) FIFO and (b) LIFO. (c) Compute the gross margin for each method.

Tree Seedlings has the following current-year purchases and sales for its only product. Exercise 5-18 Perpetual inventory costing

P1Date Activities Units Acquired at Cost Units Sold at Retail

Jan . 1 Beginning inventory . . . . . . . . . . . 40 units @ $2 = $ 80 Jan . 3 Sales . . . . . . . . . . . . . . . . . . . . . . . 30 units @ $8

Feb . 14 Purchase . . . . . . . . . . . . . . . . . . . . 70 units @ $3 = $210 Feb . 15 Sales . . . . . . . . . . . . . . . . . . . . . . . 60 units @ $8

June 30 Purchase . . . . . . . . . . . . . . . . . . . . 90 units @ $4 = $360 Nov . 6 Sales . . . . . . . . . . . . . . . . . . . . . . . 86 units @ $8

Nov . 19 Purchase . . . . . . . . . . . . . . . . . . . . 20 units @ $5 = $100 Totals . . . . . . . . . . . . . . . . . . . . . . 220 units $750 176 units

224 Chapter 5 Inventories and Cost of Sales

Refer to the information in Exercise 5-18 and assume the periodic inventory system is used. Determine the costs assigned to ending inventory and to cost of goods sold using (a) FIFO and (b) LIFO. (c) Compute the gross margin for each method.

Exercise 5-19A Periodic inventory costing

P3

PROBLEM SET A

Problem 5-1A Perpetual: Alternative cost flows

P1

Warnerwoods Company uses a perpetual inventory system. It entered into the following purchases and sales transactions for March. (For specific identification, the March 9 sale consisted of 80 units from beginning inventory and 340 units from the March 5 purchase; the March 29 sale consisted of 40 units from the March 18 purchase and 120 units from the March 25 purchase.)

Date Activities Units Acquired at Cost Units Sold at Retail

Mar . 1 Beginning inventory . . . . . . . . . . . . . 100 units @ $50 .00 per unit

Mar . 5 Purchase . . . . . . . . . . . . . . . . . . . . . . 400 units @ $55 .00 per unit

Mar . 9 Sales . . . . . . . . . . . . . . . . . . . . . . . . . 420 units @ $85 .00 per unit

Mar . 18 Purchase . . . . . . . . . . . . . . . . . . . . . . 120 units @ $60 .00 per unit

Mar . 25 Purchase . . . . . . . . . . . . . . . . . . . . . . 200 units @ $62 .00 per unit

Mar . 29 Sales . . . . . . . . . . . . . . . . . . . . . . . . . 160 units @ $95 .00 per unit

Totals . . . . . . . . . . . . . . . . . . . . . . . . . 820 units 580 units

Required

1. Compute cost of goods available for sale and the number of units available for sale. 2. Compute the number of units in ending inventory. 3. Compute the cost assigned to ending inventory using (a) FIFO, (b) LIFO, (c) weighted average, and

(d) specific identification. (Round all amounts to cents.) 4. Compute gross profit earned by the company for each of the four costing methods in part 3.

Check (3) Ending inventory: FIFO, $14,800; LIFO, $13,680; WA, $14,352 (4) LIFO gross profit, $17,980

Refer to the information in Problem 5-1A and assume the periodic inventory system is used.

Required

1. Compute cost of goods available for sale and the number of units available for sale. 2. Compute the number of units in ending inventory. 3. Compute the cost assigned to ending inventory using (a) FIFO, (b) LIFO, (c) weighted average, and

(d) specific identification. (Round all amounts to cents.) 4. Compute gross profit earned by the company for each of the four costing methods in part 3.

Problem 5-2AA Periodic: Alternative cost flows

P3

Montoure Company uses a perpetual inventory system. It entered into the following calendar-year pur- chases and sales transactions. (For specific identification, units sold consist of 600 units from beginning inventory, 300 from the February 10 purchase, 200 from the March 13 purchase, 50 from the August 21 purchase, and 250 from the September 5 purchase.)

Problem 5-3A Perpetual: Alternative cost flows

P1

Date Activities Units Acquired at Cost Units Sold at Retail

Jan . 1 Beginning inventory . . . . . . . . . . . . . 600 units @ $45 .00 per unit

Feb . 10 Purchase . . . . . . . . . . . . . . . . . . . . . . 400 units @ $42 .00 per unit

Mar . 13 Purchase . . . . . . . . . . . . . . . . . . . . . . 200 units @ $27 .00 per unit

Mar . 15 Sales . . . . . . . . . . . . . . . . . . . . . . . . . 800 units @ $75 .00 per unit

Aug . 21 Purchase . . . . . . . . . . . . . . . . . . . . . . 100 units @ $50 .00 per unit

Sep . 5 Purchase . . . . . . . . . . . . . . . . . . . . . . 500 units @ $46 .00 per unit

Sep . 10 Sales . . . . . . . . . . . . . . . . . . . . . . . . . 600 units @ $75 .00 per unit

Totals . . . . . . . . . . . . . . . . . . . . . . . . . 1,800 units 1,400 units

Chapter 5 Inventories and Cost of Sales 225

Required

1. Compute cost of goods available for sale and the number of units available for sale. 2. Compute the number of units in ending inventory. 3. Compute the cost assigned to ending inventory using (a) FIFO, (b) LIFO, (c) weighted average, and

(d) specific identification. (Round all amounts to cents.) 4. Compute gross profit earned by the company for each of the four costing methods in part 3.

Analysis Component

5. The company’s manager earns a bonus based on a percent of gross profit. Which method of inventory costing produces the highest bonus for the manager?

Check (3) Ending inventory: FIFO, $18,400; LIFO, $18,000; WA, $17,760 (4) LIFO gross profit, $45,800

Refer to the information in Problem 5-3A and assume the periodic inventory system is used.

Required

1. Compute cost of goods available for sale and the number of units available for sale. 2. Compute the number of units in ending inventory. 3. Compute the cost assigned to ending inventory using (a) FIFO, (b) LIFO, (c) weighted average, and

(d) specific identification. (Round all amounts to cents.) 4. Compute gross profit earned by the company for each of the four costing methods in part 3.

Analysis Component

5. The company’s manager earns a bonus based on a percentage of gross profit. Which method of inven- tory costing produces the highest bonus for the manager?

Problem 5-4AA Periodic: Alternative cost flows

P3

A physical inventory of Liverpool Company taken at December 31 reveals the following. Problem 5-5A Lower of cost or market

P2 Car audio equipment

Security equipment

Binocular equipment

Speakers Stereos Amplifiers Subwoofers

Alarms

Locks Cameras

Stabilizers Tripods

Units Cost per UnitItem Market per Unit

345 260 326 204

480

291 212

185 170

$ 90 1 1 1 86 52

150

93 310

70 97

$ 98 100 95 41

125

84 322

84 105

Required

1. Compute the lower of cost or market for the inventory applied separately to each item. 2. If the market amount is less than the recorded cost of the inventory, then record the LCM adjustment

to the Merchandise Inventory account.

Check (1) $273,054

Navajo Company’s financial statements show the following. The company recently discovered that in making physical counts of inventory, it had made the following errors: Year 1 ending inventory is under- stated by $56,000 and Year 2 ending inventory is overstated by $20,000.

Problem 5-6A Analysis of inventory errors

A2

For Year Ended December 31 Year 1 Year 2 Year 3

(a) Cost of goods sold . . . . . . . . . . . . . . . . . . . $ 615,000 $ 957,000 $ 780,000

(b) Net income . . . . . . . . . . . . . . . . . . . . . . . . . 230,000 285,000 241,000

(c) Total current assets . . . . . . . . . . . . . . . . . . 1,255,000 1,365,000 1,200,000

(d) Total equity . . . . . . . . . . . . . . . . . . . . . . . . . 1,387,000 1,530,000 1,242,000

226 Chapter 5 Inventories and Cost of Sales

2. What is the total error in combined net income for the three-year period resulting from the inventory errors? Explain.

Required

1. For each key financial statement figure—(a), (b), (c), and (d) above—prepare a table similar to the following to show the adjustments necessary to correct the reported amounts.

Figure: Year 1 Year 2 Year 3

Reported amount . . . . . . . . . . . . . . . . . . . . . . . . .

Adjustments for: Year 1 error . . . . . . . . . . . . . . . . . .

Year 2 error . . . . . . . . . . . . . . . . . .

Corrected amount . . . . . . . . . . . . . . . . . . . . . . . . .

Check (1) Corrected net income: Year 1, $286,000; Year 2, $209,000; Year 3, $261,000

Seminole Co. began the year with 23,000 units of product in its January 1 inventory costing $15 each. It made four purchases of its product during the year as follows. The company uses a periodic inventory system. On December 31, a physical count reveals that 40,000 units of its product remain in inventory.

Problem 5-7AA Periodic: Alternative cost flows P3

Mar . 7 . . . . . . . . . . . 30,000 units @ $18 .00 each

May 25 . . . . . . . . . . . 39,000 units @ $20 .00 each

Aug . 1 . . . . . . . . . . . 23,000 units @ $25 .00 each

Nov . 10 . . . . . . . . . . . 35,000 units @ $26 .00 each

Required

1. Compute the number and total cost of the units available for sale during the year. 2. Compute the amounts assigned to ending inventory and the cost of goods sold using (a) FIFO,

(b) LIFO, and (c) weighted average. (Round all amounts to cents.)

Check (2) Cost of goods sold: FIFO, $2,115,000; LIFO, $2,499,000; WA, $2,310,000

QP Corp. sold 4,000 units of its product at $50 per unit during the year and incurred operating expenses of $5 per unit in selling the units. It began the year with 700 units in inventory and made successive pur- chases of its product as follows.

Problem 5-8AA Periodic: Income comparisons and cost flows

A1 P3 Jan . 1 Beginning inventory . . . . . . . . . . . . 700 units @ $18 .00 per unit

Feb . 20 Purchase . . . . . . . . . . . . . . . . . . . . 1,700 units @ $19 .00 per unit

May 16 Purchase . . . . . . . . . . . . . . . . . . . . 800 units @ $20 .00 per unit

Oct . 3 Purchase . . . . . . . . . . . . . . . . . . . . 500 units @ $21 .00 per unit

Dec . 11 Purchase . . . . . . . . . . . . . . . . . . . . 2,300 units @ $22 .00 per unit

Total . . . . . . . . . . . . . . . . . . . . . . . . 6,000 units

Required

1. Prepare comparative income statements similar to Exhibit 5.8 for the three inventory costing methods of FIFO, LIFO, and weighted average. (Round all amounts to cents.) Include a detailed cost of goods sold section as part of each statement. The company uses a periodic inventory system, and its income tax rate is 40%.

2. How would the financial results from using the three alternative inventory costing methods change if the company had been experiencing declining costs in its purchases of inventory?

3. What advantages and disadvantages are offered by using (a) LIFO and (b) FIFO? Assume the continu- ing trend of increasing costs.

Check (1) Net income: FIFO, $61,200; LIFO, $57,180; WA, $59,196

The records of Alaska Company provide the following information for the year ended December 31.Problem 5-9AB Retail inventory method

P4 At Cost At Retail

Beginning inventory, January 1 . . . . . . . . . . . $ 469,010 $ 928,950

Cost of goods purchased . . . . . . . . . . . . . . . . 3,376,050 6,381,050

Sales . . . . . . . . . . . . . . . . . . . . . . . . . . . . . . . . 5,595,800

Sales returns . . . . . . . . . . . . . . . . . . . . . . . . . . 42,800

Required

1. Use the retail inventory method to estimate the company’s year-end inventory at cost. 2. A year-end physical inventory at retail prices yields a total inventory of $1,686,900. Prepare a calcula-

tion showing the company’s loss from shrinkage at cost and at retail.

Check (1) Inventory, $924,182 cost (2) Inventory shortage at cost, $36,873

Chapter 5 Inventories and Cost of Sales 227

Wayward Company wants to prepare interim financial statements for the first quarter. The company wishes to avoid making a physical count of inventory. Wayward’s gross profit rate averages 34%. The fol- lowing information for the first quarter is available from its records.

Problem 5-10AB Gross profit method P4

Beginning inventory, January 1 . . . . . . . . . . . $ 302,580

Cost of goods purchased . . . . . . . . . . . . . . . . 941,040

Sales . . . . . . . . . . . . . . . . . . . . . . . . . . . . . . . . 1,211,160

Sales returns . . . . . . . . . . . . . . . . . . . . . . . . . . 8,410

Required

Use the gross profit method to estimate the company’s first-quarter ending inventory. Check Estimated ending inventory, $449,805

PROBLEM SET B

Problem 5-1B Perpetual: Alternative cost flows

P1

Ming Company uses a perpetual inventory system. It entered into the following purchases and sales trans- actions for April. (For specific identification, the April 9 sale consisted of 8 units from beginning inven- tory and 27 units from the April 6 purchase; the April 30 sale consisted of 12 units from beginning inventory, 3 units from the April 6 purchase, and 10 units from the April 25 purchase.)

Date Activities Units Acquired at Cost Units Sold at Retail

Apr . 1 Beginning inventory . . . . . . . . . . . . . 20 units @ $3,000 .00 per unit

Apr . 6 Purchase . . . . . . . . . . . . . . . . . . . . . . 30 units @ $3,500 .00 per unit

Apr . 9 Sales . . . . . . . . . . . . . . . . . . . . . . . . . 35 units @ $12,000 .00 per unit

Apr . 17 Purchase . . . . . . . . . . . . . . . . . . . . . . 5 units @ $4,500 .00 per unit

Apr . 25 Purchase . . . . . . . . . . . . . . . . . . . . . . 10 units @ $4,800 .00 per unit

Apr . 30 Sales . . . . . . . . . . . . . . . . . . . . . . . . . 25 units @ $14,000 .00 per unit

Total . . . . . . . . . . . . . . . . . . . . . . . . . . 65 units 60 units

Required

1. Compute cost of goods available for sale and the number of units available for sale. 2. Compute the number of units in ending inventory. 3. Compute the cost assigned to ending inventory using (a) FIFO, (b) LIFO, (c) weighted average, and

(d) specific identification. (Round all amounts to cents.) 4. Compute gross profit earned by the company for each of the four costing methods in part 3.

Check (3) Ending inventory: FIFO, $24,000; LIFO, $15,000; WA, $20,000 (4) LIFO gross profit, $549,500

Refer to the information in Problem 5-1B and assume the periodic inventory system is used.

Required

1. Compute cost of goods available for sale and the number of units available for sale. 2. Compute the number of units in ending inventory. 3. Compute the cost assigned to ending inventory using (a) FIFO, (b) LIFO, (c) weighted average, and

(d) specific identification. (Round all amounts to cents.) 4. Compute gross profit earned by the company for each of the four costing methods in part 3.

Problem 5-2BA Periodic: Alternative cost flows

P3

Aloha Company uses a perpetual inventory system. It entered into the following calendar-year purchases and sales transactions. (For specific identification, the May 9 sale consisted of 80 units from beginning inventory and 100 units from the May 6 purchase; the May 30 sale consisted of 200 units from the May 6 purchase and 100 units from the May 25 purchase.)

Problem 5-3B Perpetual: Alternative cost flows

P1

Date Activities Units Acquired at Cost Units Sold at Retail

May 1 Beginning inventory . . . . . . . . . . . . . 150 units @ $300 .00 per unit

May 6 Purchase . . . . . . . . . . . . . . . . . . . . . . 350 units @ $350 .00 per unit

May 9 Sales . . . . . . . . . . . . . . . . . . . . . . . . . 180 units @ $1,200 .00 per unit

May 17 Purchase . . . . . . . . . . . . . . . . . . . . . . 80 units @ $450 .00 per unit

May 25 Purchase . . . . . . . . . . . . . . . . . . . . . . 100 units @ $458 .00 per unit

May 30 Sales . . . . . . . . . . . . . . . . . . . . . . . . . 300 units @ $1,400 .00 per unit

Total . . . . . . . . . . . . . . . . . . . . . . . . . . 680 units 480 units

228 Chapter 5 Inventories and Cost of Sales

Required

1. Compute cost of goods available for sale and the number of units available for sale. 2. Compute the number of units in ending inventory. 3. Compute the cost assigned to ending inventory using (a) FIFO, (b) LIFO, (c) weighted average, and

(d) specific identification. (Round all amounts to cents.) 4. Compute gross profit earned by the company for each of the four costing methods in part 3.

Analysis Component

5. If the company’s manager earns a bonus based on a percent of gross profit, which method of inventory costing will the manager likely prefer?

Check (3) Ending inventory: FIFO, $88,800; LIFO, $62,500; WA, $75,600 (4) LIFO gross profit, $449,200

Refer to the information in Problem 5-3B and assume the periodic inventory system is used.

Required

1. Compute cost of goods available for sale and the number of units available for sale. 2. Compute the number of units in ending inventory. 3. Compute the cost assigned to ending inventory using (a) FIFO, (b) LIFO, (c) weighted average, and

(d) specific identification. (Round all amounts to cents.) 4. Compute gross profit earned by the company for each of the four costing methods in part 3.

Analysis Component

5. If the company’s manager earns a bonus based on a percentage of gross profit, which method of inven- tory costing will the manager likely prefer?

Problem 5-4BA Periodic: Alternative cost flows

P3

A physical inventory of Office Necessities Company taken at December 31 reveals the following.Problem 5-5B Lower of cost or market

P2

Required

1. Compute the lower of cost or market for the inventory applied separately to each item. 2. If the market amount is less than the recorded cost of the inventory, then record the LCM adjustment

to the Merchandise Inventory account.

Check (1) $580,054

O�ce furniture

Filing cabinets

O�ce equipment

Desks Chairs Mats Bookshelves

Two-drawer Four-drawer Lateral

Copiers Projectors

Phones

536 395 687 421

1 14 298

75

370 475

$261 227

49 93

81 135 104

168 317

$305 256

43 82

70 122 118

200 288

302 125 1 17

Units Cost per UnitItem Market per Unit

For Year Ended December 31 Year 1 Year 2 Year 3

(a) Cost of goods sold . . . . . . . . . . . . . . . . . . . . $207,200 $213,800 $197,030

(b) Net income . . . . . . . . . . . . . . . . . . . . . . . . . . 175,800 212,270 184,910

(c) Total current assets . . . . . . . . . . . . . . . . . . . . 276,000 277,500 272,950

(d) Total equity . . . . . . . . . . . . . . . . . . . . . . . . . . 314,000 315,000 346,000

Hallam Company’s financial statements show the following. The company recently discovered that in making physical counts of inventory, it had made the following errors: Year 1 ending inventory is over- stated by $18,000 and Year 2 ending inventory is understated by $26,000.

Problem 5-6B Analysis of inventory errors

A2

Chapter 5 Inventories and Cost of Sales 229

2. What is the total error in combined net income for the three-year period resulting from the inventory errors? Explain.

Required

1. For each key financial statement figure—(a), (b), (c), and (d) above—prepare a table similar to the following to show the adjustments necessary to correct the reported amounts.

Figure: Year 1 Year 2 Year 3

Reported amount . . . . . . . . . . . . . . . . . . . . . . . . .

Adjustments for: Year 1 error . . . . . . . . . . . . . . . . . .

Year 2 error . . . . . . . . . . . . . . . . . .

Corrected amount . . . . . . . . . . . . . . . . . . . . . . . . .

Check (1) Corrected net income: Year 1, $157,800; Year 2, $256,270; Year 3, $158,910

Seneca Co. began the year with 6,500 units of product in its January 1 inventory costing $35 each. It made four purchases of its product during the year as follows. The company uses a periodic inventory system. On December 31, a physical count reveals that 8,500 units of its product remain in inventory.

Problem 5-7BA Periodic: Alternative cost flows

P3 July 9 . . . . . . . . . . . . 11,000 units @ $29 each

Nov . 21 . . . . . . . . . . . . 7,600 units @ $27 each

Jan . 4 . . . . . . . . . . . . 11,500 units @ $33 each

May 18 . . . . . . . . . . . . 13,400 units @ $32 each

Required

1. Compute the number and total cost of the units available for sale during the year. 2. Compute the amounts assigned to ending inventory and the cost of goods sold using (a) FIFO,

(b) LIFO, and (c) weighted average. (Round all amounts to cents.)

Check (2) Cost of goods sold: FIFO, $1,328,700; LIFO, $1,266,500; WA, $1,294,800

Shepard Company sold 4,000 units of its product at $100 per unit during the year and incurred operating expenses of $15 per unit in selling the units. It began the year with 840 units in inventory and made suc- cessive purchases of its product as follows.

Problem 5-8BA Periodic: Income comparisons and cost flows

A1 P3 Jan . 1 Beginning inventory . . . . . . . . . . . . . 840 units @ $58 per unit

Apr . 2 Purchase . . . . . . . . . . . . . . . . . . . . . . 600 units @ $59 per unit

June 14 Purchase . . . . . . . . . . . . . . . . . . . . . . 1,205 units @ $61 per unit

Aug . 29 Purchase . . . . . . . . . . . . . . . . . . . . . . 700 units @ $64 per unit

Nov . 18 Purchase . . . . . . . . . . . . . . . . . . . . . . 1,655 units @ $65 per unit

Total . . . . . . . . . . . . . . . . . . . . . . . . . . 5,000 units

Required

1. Prepare comparative income statements similar to Exhibit 5.8 for the three inventory costing methods of FIFO, LIFO, and weighted average. (Round all amounts to cents.) Include a detailed cost of goods sold sec- tion as part of each statement. The company uses a periodic inventory system, and its income tax rate is 40%.

2. How would the financial results from using the three alternative inventory costing methods change if the company had been experiencing decreasing prices in its purchases of inventory?

3. What advantages and disadvantages are offered by using (a) LIFO and (b) FIFO? Assume the continu- ing trend of increasing costs.

Check (1) Net income: LIFO, $52,896; FIFO, $57,000; WA, $55,200

Problem 5-9BB Retail inventory method

P4

The records of Macklin Co. provide the following information for the year ended December 31.

At Cost At Retail

Beginning inventory, January 1 . . . . . . . . . . . $ 90,022 $115,610

Cost of goods purchased . . . . . . . . . . . . . . . . 502,250 761,830

Sales . . . . . . . . . . . . . . . . . . . . . . . . . . . . . . . . 782,300

Sales returns . . . . . . . . . . . . . . . . . . . . . . . . . . 3,460

Required

1. Use the retail inventory method to estimate the company’s year-end inventory. 2. A year-end physical inventory at retail prices yields a total inventory of $80,450. Prepare a calculation

showing the company’s loss from shrinkage at cost and at retail.

Check (1) Inventory, $66,555 cost (2) Inventory shortage at cost, $12,251.25

230 Chapter 5 Inventories and Cost of Sales

Otingo Equipment Co. wants to prepare interim financial statements for the first quarter. The company wishes to avoid making a physical count of inventory. Otingo’s gross profit rate averages 35%. The follow- ing information for the first quarter is available from its records.

Problem 5-10BB Gross profit method

P4

Beginning inventory, January 1 . . . . . . . . . . . $ 802,880

Cost of goods purchased . . . . . . . . . . . . . . . . 2,209,636

Sales . . . . . . . . . . . . . . . . . . . . . . . . . . . . . . . . 3,760,260

Sales returns . . . . . . . . . . . . . . . . . . . . . . . . . . 79,300

Required

Use the gross profit method to estimate the company’s first-quarter ending inventory. Check Est. ending inventory, $619,892

SERIAL PROBLEM Business Solutions

A3 P2

This serial problem began in Chapter 1 and continues through most of the book. If previous chapter seg- ments were not completed, the serial problem can begin at this point.

SP 5 Part A

Santana Rey of Business Solutions is evaluating her inventory to determine whether it must be adjusted based on lower of cost or market rules. Business Solutions has three different types of software in its inventory, and the following information is available for each.

Inventory Items Units Cost per Unit Market per Unit

Office productivity . . . . . . . . . . . 3 $ 76 $ 74

Desktop publishing . . . . . . . . . . . 2 103 100

Accounting . . . . . . . . . . . . . . . . . 3 90 96

Required

Compute the lower of cost or market for ending inventory assuming Rey applies the lower of cost or mar- ket rule to each product in inventory. Must Rey adjust the reported inventory value? Explain.

Part B

Selected accounts and balances for the three months ended March 31, 2020, for Business Solutions follow.

Beginning inventory, January 1 . . . . . . . . . . . $ 0

Cost of goods sold . . . . . . . . . . . . . . . . . . . . . 14,052

Ending inventory, March 31 . . . . . . . . . . . . . . 704

Required

1. Compute inventory turnover and days’ sales in inventory for the three months ended March 31, 2020. 2. Assess the company’s performance if competitors average 15 times for inventory turnover and 25 days

for days’ sales in inventory.

©Alexander Image/Shutterstock

COMPANY ANALYSIS C2 A3

Accounting Analysis

AA 5-1 Use Apple’s financial statements in Appendix A to answer the following.

Required

1. What amount of inventories did Apple report as a current asset (a) on September 30, 2017? (b) On September 24, 2016?

2. Inventories make up what percent of total assets (a) on September 30, 2017? (b) On September 24, 2016? 3. Assuming Apple has enough inventory to meet demand, does Apple prefer inventory to be a lower or

higher percentage of total assets? 4. Compute (a) inventory turnover for fiscal year ended September 30, 2017, and (b) days’ sales in inven-

tory as of September 30, 2017.

APPLE

Chapter 5 Inventories and Cost of Sales 231

AA 5-2 Comparative figures for Apple and Google follow.

Required

1. Compute inventory turnover for each company for the most recent two years shown. 2. Compute days’ sales in inventory for each company for the three years shown. 3. In the current year, does (a) Apple’s and (b) Google’s inventory turnover underperform or outperform

the industry (assumed) average of 15?

Apple Google

Current One Year Two Years Current One Year Two Years $ millions Year Prior Prior Year Prior Prior

Inventory . . . . . . . . . . . . . . $ 4,855 $ 2,132 $ 2,349 $ 749 $ 268 $ 491

Cost of sales . . . . . . . . . . . 141,048 131,376 140,089 45,583 35,138 28,164

COMPARATIVE ANALYSIS A3

APPLE

Required

1. Compute Samsung’s (a) inventory turnover and (b) days’ sales in inventory for the most recent two years.

2. Is Samsung’s inventory turnover on a favorable or unfavorable trend? 3. In the current year, does Samsung’s inventory turnover underperform or outperform the industry

(assumed) average of 15?

AA 5-3 Key figures for Samsung follow.

₩ millions Current Year One Year Prior Two Years Prior

Inventory . . . . . . . . . . . . . . ₩ 24,983,355 ₩ 18,353,503 ₩ 18,811,794

Cost of sales . . . . . . . . . . . 129,290,661 120,277,715 123,482,118

GLOBAL ANALYSIS A3

Samsung

ETHICS CHALLENGE A1

BTN 5-1 Golf Challenge Corp. is a retail sports store carrying golf apparel and equipment. The store is at the end of its second year of operation and is struggling. A major problem is that its cost of inventory has continually increased in the past two years. In the first year of operations, the store assigned inventory costs using LIFO. A loan agreement the store has with its bank, its prime source of financing, requires the store to maintain a certain profit margin and current ratio. The store’s owner is currently looking over Golf Challenge’s preliminary financial statements for its second year. The numbers are not favorable. The only way the store can meet the financial ratios agreed on with the bank is to change from LIFO to FIFO. The store originally decided on LIFO because of its tax advantages. The owner recalculates ending inventory using FIFO and submits those numbers and statements to the loan officer for the required bank review. The owner thankfully reflects on the available latitude in choosing the inventory costing method.

Required

1. How does Golf Challenge’s use of FIFO improve its net profit margin and current ratio? 2. Is the action by Golf Challenge’s owner ethical? Explain.

Beyond the Numbers

BTN 5-2 You are a financial adviser with a client in the wholesale produce business that just completed its first year of operations. Due to weather conditions, the cost of acquiring produce to resell has escalated during the latter part of this period. Your client, Javonte Gish, mentions that because her business sells perishable goods, she has striven to maintain a FIFO flow of goods. Although sales are good, the increas- ing cost of inventory has put the business in a tight cash position. Gish has expressed concern regarding the ability of the business to meet income tax obligations.

Required

Prepare a memorandum that identifies, explains, and justifies the inventory method you recommend that Ms. Gish adopt.

COMMUNICATING IN PRACTICE A1

GOOGLE

232 Chapter 5 Inventories and Cost of Sales

BTN 5-3 Access the September 30, 2017, 10-K report for Apple, Inc. (ticker: AAPL), filed on November 3, 2017, from the EDGAR filings at SEC.gov.

Required

1. What products are manufactured by Apple? 2. What inventory method does Apple use? Hint: See Note 1 to its financial statements. 3. Compute its gross margin and gross margin ratio for the 2017 fiscal year. Comment on your

computations—assume an industry average of 40% for the gross margin ratio. 4. Compute its inventory turnover and days’ sales in inventory for the year ended September 30, 2017.

Comment on your computations—assume an industry average of 15 for inventory turnover and 9 for days’ sales in inventory.

TAKING IT TO THE NET A3

BTN 5-4 Each team member has the responsibility to become an expert on an inventory method. This expertise will be used to facilitate teammates’ understanding of the concepts relevant to that method. 1. Each learning team member should select an area for expertise by choosing one of the following

inventory methods: specific identification, LIFO, FIFO, or weighted average. 2. Form expert teams made up of students who have selected the same area of expertise. The instructor

will identify where each expert team will meet. 3. Using the following data, each expert team must collaborate to develop a presentation that illustrates

the relevant concepts and procedures for its inventory method. Each team member must write the pre- sentation in a format that can be shown to the learning team.

Data

The company uses a perpetual inventory system. It had the following beginning inventory and current- year purchases of its product.

TEAMWORK IN ACTION A1 P1

Point: Step 1 allows four choices or areas for expertise. Larger teams will have some duplication of choice, but the specific identification method should not be duplicated.

Jan . 1 Beginning inventory . . . . . . . . . . . 50 units @ $100 = $ 5,000 Jan . 14 Purchase . . . . . . . . . . . . . . . . . . . . 150 units @ $120 = 18,000 Apr . 30 Purchase . . . . . . . . . . . . . . . . . . . . 200 units @ $150 = 30,000 Sep . 26 Purchase . . . . . . . . . . . . . . . . . . . . 300 units @ $200 = 60,000

Jan . 10 30 units . . . . . . . . . . specific cost: 30 @ $100

Feb . 15 100 units . . . . . . . . . . specific cost: 100 @ $120

Oct . 5 350 units . . . . . . . . . . specific cost: 100 @ $150 and 250 @ $200

The company transacted sales on the following dates at a $350 per unit sales price.

Concepts and Procedures to Illustrate in Expert Presentation

a. Identify and compute the costs to assign to the units sold. (Round per unit costs to three decimals.) b. Identify and compute the costs to assign to the units in ending inventory. (Round inventory bal-

ances to the dollar.) c. How likely is it that this inventory costing method will reflect the actual physical flow of goods?

How relevant is that factor in determining whether this is an acceptable method to use? d. What is the impact of this method versus others in determining net income and income taxes? e. How closely does the ending inventory amount reflect replacement cost? 4. Re-form learning teams. In rotation, each expert is to present to the team the presentation developed in

part 3. Experts are to encourage and respond to questions.

BTN 5-5 Review the chapter’s opening feature highlighting Danny Meyer and Shake Shack. Assume that the business consistently maintains an inventory level of $30,000, meaning that its average and ending inventory levels are the same. Also assume its annual cost of sales is $120,000. To cut costs, the business proposes to slash inventory to a constant level of $15,000 with no impact on cost of sales. The business plans to work with suppliers to get quicker deliveries and to order smaller quantities more often.

ENTREPRENEURIAL DECISION A3

APPLE

Chapter 5 Inventories and Cost of Sales 233

Required

1. Compute the company’s inventory turnover and its days’ sales in inventory under (a) current condi- tions and (b) proposed conditions.

2. Evaluate and comment on the merits of the proposal given your analysis for part 1. Identify any con- cerns you might have about the proposal.

BTN 5-6 Visit four retail stores with another classmate. In each store, identify whether the store uses a bar coding system to help manage its inventory. Try to find at least one store that does not use bar coding. If a store does not use bar coding, ask the store’s manager or clerk whether he or she knows which type of inventory method the store employs. Create a table that shows columns for the name of store visited, type of merchandise sold, use or nonuse of bar coding, and the inventory method used if bar coding is not employed. You also might inquire as to what the store’s inventory turnover is and how often physical inventory is taken.

HITTING THE ROAD C1 C2

Design elements: Lightbulb: ©Chuhail/Getty Images; Blue globe: ©nidwlw/Getty Images and ©Dizzle52/Getty Images; Chess piece: ©Andrei Simonenko/Getty Images and ©Dizzle52/Getty Images; Mouse: ©Siede Preis/Getty Images; Global View globe: ©McGraw-Hill Education and ©Dizzle52/Getty Images; Sustainability: ©McGraw-Hill Education and ©Dizzle52/Getty Images

Learning Objectives

CONCEPTUAL C1 Define internal control and identify its

purpose and principles.

C2 Define cash and cash equivalents and explain how to report them.

ANALYTICAL A1 Compute the days’ sales uncollected

ratio and use it to assess liquidity.

P3 Prepare a bank reconciliation.

P4 Appendix 6A—Describe use of documentation and verification to control cash payments.

PROCEDURAL P1 Apply internal control to cash receipts

and payments.

P2 Explain and record petty cash fund transactions.

Chapter Preview

6 Cash, Fraud, and Internal Control

NTK 6-3, 6-4

TOOLS OF CONTROL AND ANALYSIS

P2 Control of petty cash P3 Bank reconciliation as a

control tool

A1 Assessing liquidity

NTK 6-1

FRAUD AND INTERNAL CONTROL

C1 Purpose and principles of controls

Technology and controls

Limitations of controls

NTK 6-2

CONTROL OF CASH

C2 Definition and reporting of cash

P1 Control of cash receipts and cash payments

235

“Take the risks”—Sheila Marcelo

Taking Care of Business

WALTHAM, MA—Sheila Marcelo was in college when her first child was born. “We had to scramble for child care throughout our college years,” recalls Sheila. “It was harder than it should have been.”

The struggle to find child care led Sheila to start Care.com (Care.com). Care.com matches caregivers with families online.

A key part of Care.com’s business is its internal control sys- tems. Sheila explains that controls are important to Care.com’s future, to the integrity of its systems, and to the trust of its mem- bers. Her controls extend to monitoring transactions and safe- guarding its assets and members.

Sheila insists that controls raise productivity, cut expenses, reduce fraud, and enhance the member experience. “People fear finance [and accounting] courses,” admits Sheila. “[But] if you want to be an entrepreneur,” declares Sheila, “don’t under- estimate the value of skills learned in those classes.”

Sheila offers two suggestions for pursuing a business. First, “don’t worry about how you’re being perceived . . . about fitting into the mold.” Second, “to grow in leadership, you have to be a

narcissist.” Adds Sheila, “Focus on yourself, understand your- self, take time for yourself. It will make you a better leader.”

Sources: Care.com website, January 2019; EAK, October 2016; Business Insider, March 2014; Boston Globe, August 2014; Bloomberg, September 2012

©Jin Lee/Bloomberg/Getty Images

Purpose of Internal Control Managers or owners of small businesses often control the entire operation. They know if the business is actually receiving the assets and services it paid for. Most companies, however, can- not maintain personal supervision and must rely on internal controls.

Internal Control System Managers use an internal control system to monitor and control business activities. An internal control system is policies and procedures used to Protect assets. Promote efficient operations. Ensure reliable accounting. Uphold company policies.

Managers use internal control systems to prevent avoidable losses, plan operations, and monitor company and employee performance. For example, internal controls for UnitedHealth Group protect patient records and privacy.

Sarbanes-Oxley Act (SOX) Sarbanes-Oxley Act (SOX) requires managers and audi- tors of companies whose stock is traded on an exchange (called public companies) to document and verify internal controls. Following are some of the requirements. The company must have effective internal controls. Auditors must evaluate internal controls. Violators receive harsh penalties—up to 25 years in prison with fines. Auditors’ work is overseen by the Public Company Accounting Oversight Board (PCAOB).

Committee of Sponsoring Organizations (COSO) Committee of Sponsoring Organizations (COSO) lists five ingredients of internal control that add to the quality of accounting information. Control environment—company structure, ethics, and integrity for internal control. Risk assessment—identify, analyze, and manage risk factors. Control activities—policies and procedures to reduce risk of loss. Information & communication—reports to internal and external parties. Monitoring—regular review of internal control effectiveness.

FRAUD AND INTERNAL CONTROL C1 Define internal control and identify its purpose and principles.

©Wright Studio/Shutterstock

Information &CommunicationControl Activities

Monitoring

Control Environment

Risk Assessment

236 Chapter 6 Cash, Fraud, and Internal Control

Principles of Internal Control Internal control varies from company to company, but internal control principles apply to all companies. The principles of internal control are to

1. Establish responsibilities. 2. Maintain adequate records. 3. Insure assets and bond key employees. 4. Separate recordkeeping from custody of assets. 5. Divide responsibility for related transactions. 6. Apply technological controls. 7. Perform regular and independent reviews.

Establish Responsibilities Responsibility for a task should be clearly established and assigned to one person. When a problem occurs in a company where responsibility is not estab- lished, determining who is at fault is difficult. For example, if two salesclerks share the same cash register and cash is missing, neither clerk can be held accountable. To prevent this prob- lem, a company can use separate cash drawers for each clerk.

Maintain Adequate Records Good recordkeeping helps protect assets and helps managers monitor company activities. When there are detailed records of equipment, for ex- ample, items are unlikely to be lost or stolen without detection. Similarly, transactions are less likely to be entered in wrong accounts if a chart of accounts is used. Preprinted forms are also part of good internal control. When sales slips are properly designed, employees can record information efficiently with fewer errors. When sales slips are prenumbered, each slip is the responsibility of one salesperson, preventing the salesperson from stealing cash by making a sale and destroying the sales slip. Computerized point-of-sale systems achieve the same control results.

Insure Assets and Bond Key Employees Assets should be insured against losses, and employees handling lots of cash and easily transferable as- sets should be bonded. An employee is bonded when a company purchases an insurance policy, or a bond, against theft by that employee. Bonding discour- ages theft because bonded employees know the bonding company will pursue reported theft.

Separate Recordkeeping from Custody of Assets A person who controls or has access to an asset must not have access to that asset’s account- ing records. This principle reduces the risk of theft or waste of an asset because the person with control over it knows that another person keeps its records. Also, a recordkeeper who does not have access to the asset has no reason to falsify re- cords. This means that to steal an asset and hide the theft from the records, two or

more people must collude—or agree in secret to commit the fraud.

Divide Responsibility for Related Transactions Responsibility for a transac- tion should be divided between two or more individuals or departments. This ensures the work of one person acts as a check on the other to prevent fraud and errors. This principle, called separation of duties, does not mean duplication of work. For example, when a company orders inventory, the task should be split among several employees. One employee submits a request to purchase inventory, a second employee approves the request, a third employee makes the payment, and a fourth employee records the transaction.

Apply Technological Controls Cash registers, time clocks, and ID scanners are ex- amples of devices that can improve internal control. A cash register with a locked-in tape or electronic file makes a record of each cash sale. A time clock records the exact hours worked by an employee. ID scanners limit access to authorized individuals.

Point: Many companies have a mandatory vacation policy for em- ployees who handle cash. When another employee must cover for the one on vacation, it is more difficult to hide cash frauds.

A control system is only as strong as its weakest link

Courtesy of Commercial Collection Agency Association of the Commercial Law League of America

Point: ACFE estimates that employee fraud costs more than $150,000 per incident.

Chapter 6 Cash, Fraud, and Internal Control 237

Perform Regular and Independent Reviews Regular reviews of internal con- trols help ensure that procedures are followed. These reviews are preferably done by auditors not directly involved in the activities. Auditors evaluate the efficiency and effectiveness of internal controls. Many companies pay for audits by independent auditors. These auditors test the com- pany’s financial records and evaluate the effectiveness of internal controls.

Entrepreneur As owner of a start-up surfboard company, you hire a systems analyst. The analyst sees that your company employs only two workers. She says that as owner you must serve as a compensating control. What does the analyst mean? ■ Answer: Transaction authorization, recording, and asset custody are ideally handled by three employees. Many small busi- nesses do not employ three workers. In such cases, an owner must make sure that the lack of separation of duties does not result in fraud.

Decision Maker

Technology, Fraud, and Internal Control Principles of internal control are relevant no matter what the technological state of the account- ing system, from manual to fully automated. Technology allows us quicker access to informa- tion and improves managers’ abilities to monitor and control business activities. This section describes technological impacts we must be alert to.

Reduced Processing Errors Technology reduces, but does not eliminate, errors in processing information. Less human involvement can cause data entry errors to go undiscov- ered. Also, errors in software can produce consistent but inaccurate processing of transactions.

More Extensive Testing of Records When accounting records are kept manually, only small samples of data are usually checked for accuracy. When data are accessible using technology, large samples or even the entire database can be tested quickly.

New Evidence of Processing Technology makes it possible to record additional transaction details not possible with manual systems. For example, a system can record who made the entry, the date and time, the source of the entry, and so on. This means that internal control depends more on the design and operation of the information system and less on the analysis of its resulting documents.

Separation of Duties A company with few employees risks losing separation of duties. For example, the person who designs the information system should not operate it. The company also must separate control over programs and files from the activities related to cash receipts and payments. For example, a computer operator should not control check-writing activities.

Increased E-Commerce Amazon and eBay are examples of successful e-commerce companies. All e-commerce transactions involve at least three risks: (1) credit card number theft, (2) computer viruses, and (3) impersonation or identity theft. Companies use technologi- cal internal controls to combat these risks.

Point: Internal control failure reduces confidence in financial statements.

Point: To assess a company’s in- ternal controls, review the audi- tor’s report, management report on controls (if available), manage- ment discussion and analysis, and financial press.

Butterfingers Internal control failures can cost a company and its customers millions. Amazon learned the hard way when its web services failed. This failure led hundreds of websites to slow down. Reports say this failure cost compa- nies in the S&P 500 index $150 million. The culprit? A typo in Amazon’s code. ■

Decision Insight

Limitations of Internal Control Internal controls have limitations from (1) human error or fraud and (2) the cost-benefit principle.

Human error occurs from carelessness, misjudgment, or confusion. Human fraud is inten- tionally defeating internal controls, such as management override, for personal gain. Human fraud is driven by the triple threat of fraud. Opportunity—internal control weaknesses in a business. Pressure—financial, family, and societal stresses to succeed. Rationalization—employees justifying fraudulent behavior.

O pp

or tu

nit y

Rationalization

Financial Pressure

©EpicStockMedia/iStockphoto/ Getty Images

238 Chapter 6 Cash, Fraud, and Internal Control

The cost-benefit principle says that the costs of internal controls must not exceed their ben- efits. Analysis of costs and benefits considers all factors, including morale. For example, most companies have a legal right to read employees’ e-mails but rarely do unless there is evidence of potential harm.

Hacker’s Guide to Cyberspace Pharming Viruses attached to e-mails and websites monitor keystrokes; when you sign on to financial websites, it steals your passwords.

Phishing Hackers send e-mails to you posing as banks; you are asked for infor- mation using fake websites where they steal your passwords and personal data.

Wi-Phishing Cybercrooks set up wireless networks hoping you will use them to connect to the web; passwords and data are stolen when you connect.

Bot-Networking Hackers send out spam and viruses from your PC.

Typo-Squatting Hackers set up websites with addresses similar to legit businesses; when you make a typo and hit their sites, they infect your PC.

Identify each of the following as a (a) purpose of an internal control system, (b) principle of internal control, or (c) limitation of internal control.

1. Protect assets 2. Establish responsibilities 3. Human error 4. Maintain adequate records 5. Apply technological controls 6. Ensure reliable accounting 7. Insure assets and bond key employees

Internal Controls

NEED-TO-KNOW 6-1

C1 8. Human fraud 9. Separate recordkeeping from custody of assets 10. Divide responsibility for related transactions 11. Cost-benefit principle 12. Promote efficient operations 13. Perform regular and independent reviews 14. Uphold company policies

Solution

1. a 2. b 3. c 4. b 5. b 6. a 7. b 8. c 9. b 10. b 11. c 12. a 13. b 14. a Do More: QS 6-1, E 6-1, E 6-2,

E 6-3, P 6-1

Cash is easily hidden and moved. Internal controls protect cash and meet three guidelines.

1. Handling cash is separate from recordkeeping of cash. 2. Cash receipts are promptly deposited in a bank. 3. Cash payments are made by check or electronic funds transfer (EFT).

The first guideline applies separation of duties to minimize errors and fraud. When duties are separated, two or more people must collude to steal cash and hide this action. The second guide- line uses immediate deposits of all cash receipts to produce an independent record of the cash received. It also reduces the chance of cash theft (or loss). The third guideline uses payments by check to develop an independent record of cash payments. It also reduces the risk of cash theft (or loss).

Cash, Cash Equivalents, and Liquidity Liquidity refers to a company’s ability to pay for its current liabilities. Cash and similar assets are called liquid assets because they can be readily used to pay for liabilities.

CONTROL OF CASH C2 Define cash and cash equivalents and explain how to report them.

Chapter 6 Cash, Fraud, and Internal Control 239

Cash includes currency, coins, and deposits in bank accounts. Cash also includes items that can be deposited in these accounts such as customer checks, cashier’s checks, certified checks, and money orders. Cash equivalents are short-term, highly liquid investment assets meeting two criteria: (1) readily convertible to a known cash amount and (2) close enough to their due date so that their market value will not greatly change. Only investments within three months of their due date usually meet these criteria. Cash equivalents are short-term investments such as U.S. Treasury bills. Most companies combine cash equivalents with cash on the balance sheet.

Cash Management A common reason companies fail is inability to manage cash. Companies must plan both cash receipts and cash payments. Goals of cash management are to

1. Plan cash receipts to meet cash payments when due. 2. Keep a minimum level of cash necessary to operate.

The treasurer is responsible for cash management. Effective cash management involves apply- ing the following cash management strategies.

Encourage collection of receivables. The quicker customers and others pay the company, the quicker it can use the money. Some companies offer discounts for quicker payments.

Delay payment of liabilities. The more delayed a company is in paying others, the more time it has to use the money. Companies regularly wait to pay bills until the last day allowed.

Keep only necessary assets. Acquiring expensive and rarely used assets can cause cash short- ages. Some companies lease warehouses or rent equipment to avoid large up-front payments.

Plan expenditures. Companies must look at seasonal and business cycles to plan expendi- tures when money is available.

Invest excess cash. Excess cash earns no return and should be invested in productive assets like factories. Excess cash from seasonal cycles can be placed in a short-term investment for interest.

Control of Cash Receipts Internal control of cash receipts ensures that cash received is properly recorded and deposited. Cash receipts arise from transactions such as cash sales, collections of customer accounts, receipts of interest, bank loans, sales of assets, and owner investments. This section explains internal control over two types of cash receipts: over-the-counter and by mail.

Over-the-Counter Cash Receipts Over-the-counter cash sales should be recorded on a cash register after each sale, and customers should get a receipt. Cash registers should hold a permanent, locked-in record of each transaction. The register is often linked with the account- ing system. Less advanced registers record each transaction on a paper tape or electronic file locked inside the register.

Custody over cash should be separate from recordkeeping. The clerk who has access to cash in the register should not have access to its record. At the end of the clerk’s work period, the clerk should count the cash in the register, record the amount, and turn over the cash and record to the company cashier. The cashier, like the clerk, has access to the cash but should not have access to accounting records (or the register tape or file). A third employee, often a supervisor, compares the record of total register transactions with the cash receipts reported by the cashier. This record is used for a journal entry recording over- the-counter cash receipts. The third employee has access to the records for cash but not to the actual cash. The clerk and the cashier have access to cash but not to the accounting

P1 Apply internal control to cash receipts and payments.

Point: The most liquid assets are usually reported first on a balance sheet; the least liquid assets are reported last.

Point: Companies invest idle cash in cash equivalents to increase income.

Point: Many businesses have signs that read: If you receive no receipt, your purchase is free! This helps ensure that clerks ring up all transactions on registers.

240 Chapter 6 Cash, Fraud, and Internal Control

records. None of them can make a mistake or steal cash without the difference being noticed (see the following diagram).

CashCash

Register Sheet

NameDateSer.No Rank Signature

FastForward Los Angeles

Deposit

Paid in by _______________ ________________________ Credit account of C & L Computer LTD ________________________

Date _______ Notes _______ Coin _______

$

9821 0058 478211006

Paid in by ____________ _____________________ Credit accoount of C & L Commputer LTD _____________________

9821 00058 478211000

$

$

Supervisor reads register data, prepares register sheet (and keeps copy), and sends

both to company cashier.

Cashier prepares cash records, deposit slip, and

journal entry.

Sales Department Cashier Department

Cash

Cash sheets Received from..................................

Amount dollar ....................................

Authorized Signature

Cash sheets Received from..................................

Amount dollar ....................................

Authorised Signature

DepositFastForwward Los Angelees

$$$

$

Cash

Caash ssheetss Receiveed from..................................

AAAAAAAAmAmAmAmmountAmountAmmountmountmountA d lld lld lldolladollarr ....................................

Authorised Signature

Register Sheet

NameDateSer.No Rank Signature

Clerk rings up cash sales on register; clerk prepares cash count sheet (and keeps copy) and sends to

company cashier along with the cash.

Cash Over and Short One or more customers can be given too much or too little change. This means that at the end of a work period, the cash in a cash register might not equal the record of cash receipts. This difference is reported in the Cash Over and Short account, also called Cash Short and Over, which is an income statement account recording the income effects of cash overages and cash shortages. If a cash register’s record shows $550 but the count of cash in the register is $555, the entry to record cash sales and its overage is

Alternatively, if a cash register’s record shows $625 but the count of cash in the register is $621, the entry to record cash sales and its shortage is

Cash . . . . . . . . . . . . . . . . . . . . . . . . . . . . . . . . . . . . . . . . . . . . . . . . . 555

Cash Over and Short . . . . . . . . . . . . . . . . . . . . . . . . . . . . . . 5 Sales . . . . . . . . . . . . . . . . . . . . . . . . . . . . . . . . . . . . . . . . . . . . 550

Record cash sales and a cash overage.

Assets = Liabilities + Equity +555 + 5 +550

Cash . . . . . . . . . . . . . . . . . . . . . . . . . . . . . . . . . . . . . . . . . . . . . . . . . 621

Cash Over and Short . . . . . . . . . . . . . . . . . . . . . . . . . . . . . . . . . . . 4 Sales . . . . . . . . . . . . . . . . . . . . . . . . . . . . . . . . . . . . . . . . . . . . 625

Record cash sales and a cash shortage.

Assets = Liabilities + Equity +621 − 4 +625

Because customers are more likely to dispute being shortchanged than being given too much change, the Cash Over and Short account usually has a debit balance. A debit balance reflects an expense. It is reported on the income statement as part of selling, general, and administrative expenses. (Because the amount is usually small, it is often reported as part of miscellaneous expenses—or as part of miscellaneous revenues if it has a credit balance.)

Cash Receipts by Mail Two people are assigned the task of opening the mail. In this case, theft of cash receipts by mail requires collusion between these two employees. The person(s) opening the mail enters a list (in triplicate) of money received. This list has each sender’s name, the amount, and an explanation of why the money was sent. The first copy is sent with the money to the cashier. A second copy is sent to the recordkeeper. A third copy is kept by the person(s) who opened the mail. The cashier deposits the money in a bank, and the recordkeeper records the amounts received.

Chapter 6 Cash, Fraud, and Internal Control 241

This process is good internal control because the bank’s record of cash deposited must agree with the records from each of the three. If the mail person(s) does not report all receipts cor- rectly, customers will question their account balances. If the cashier does not deposit all the cash, the bank balance does not agree with the recordkeeper’s cash balance. The recordkeeper does not have access to cash and has no opportunity to steal cash. This system makes errors and fraud highly unlikely. The exception is employee collusion.

Cash Register Insight Walmart uses a network of information links with its point-of-sale cash registers to coordi- nate sales, purchases, and distribution. Its stores ring up tens of thousands of separate sales on heavy days. By using cash register information, the company can fix pricing mistakes quickly and capitalize on sales trends. ■

Decision Insight

Control of Cash Payments Control of cash payments is important as most large thefts occur from payment of fictitious invoices. One key to controlling cash payments is to require all payments to be made by check. The only exception is small payments made from petty cash. Another key is to deny access to accounting records to anyone other than the owner who has the authority to sign checks. A small-business owner often signs checks and knows that the items being paid for are actually received. Large businesses cannot maintain personal supervision and must rely on internal con- trols described here, including the voucher system and petty cash system.

Cash Budget Projected cash receipts and cash payments are summarized in a cash bud- get. If there is enough cash for operations, companies wish to minimize the cash they hold be- cause of its risk of theft and its low return versus other assets.

Voucher System of Control A voucher system is a set of procedures and approvals designed to control cash payments and the acceptance of liabilities that consist of Verifying, approving, and recording liabilities for cash payment. Issuing checks for payment of verified, approved, and recorded liabilities.

A voucher system’s control over cash payments begins when a company incurs a liability that will result in cash payment. The system only allows authorized departments and individuals to incur liabilities and limits the type of liabilities. In a large retail store, for example, only a pur- chasing department is authorized to incur liabilities for inventory. Purchasing, receiving, and paying for merchandise are divided among several departments (or individuals). These depart- ments include the one requesting the purchase, the purchasing department, the receiving department, and the accounting department.

To coordinate and control responsibilities of these departments, a company uses several dif- ferent business documents. Exhibit 6.1 shows how documents are accumulated in a voucher, which is an internal document (or file) used to collect information to control cash payments and to ensure that a transaction is properly recorded. This specific example begins with a purchase requisition and ends with issuing a check.

A voucher system should be applied to all payments (except those using petty cash). When a company receives a monthly telephone bill, it should review the charges, prepare a voucher (file), and insert the bill. This transaction is then recorded. If the amount is due, a check is issued. If not, the voucher is filed for payment on its due date. Without records, an employee could collude with a supplier to get more than one payment, payment for excessive amounts, or payment for goods and services not received. A voucher system helps prevent such frauds.

Point: A purchase requisition is a request to purchase merchandise.

Cash Fraud The Association of Certified Fraud Examiners (ACFE) reports that 87% of fraud is from asset theft. Of those asset thefts, a few stand out—in both frequency and median loss. Namely, cash is most frequently stolen through billing (22%) and theft (20%), followed by expense reimbursements (14%), skimming (12%), check tampering (11%), and payroll (9%). Interestingly, the average loss per incident is greatest for check tampering ($158,000) and billing ($100,000). Source: “Report to the Nations,” ACFE. ■

Ethical Risk

©Amble Design/Shutterstock

242 Chapter 6 Cash, Fraud, and Internal Control

Petty Cash System of Control To avoid writing checks for small amounts, a com- pany sets up a petty cash system. Petty cash payments are small payments for items such as shipping fees, minor repairs, and low-cost supplies.

Operating a Petty Cash Fund A petty cash fund requires estimating the amount of small payments to be made during a short period such as a week or month. A check is then drawn by the company cashier for an amount slightly in excess of this estimate. The check is cashed and given to an employee called the petty cashier or petty cash custodian. The petty cashier keeps this cash safe, makes payments from the fund, and keeps records of it in a secure petty cashbox.

When a cash payment is made, the person receiving payment signs a prenumbered petty cash receipt, also called petty cash ticket—see Exhibit 6.2. The petty cash receipt is then placed in the petty cashbox with the remaining money. Under this system, the total of all receipts plus the remaining cash equals the total fund amount. A $100 petty cash fund, for example, contains any combination of cash and petty cash receipts that totals $100 (examples are $80 cash plus $20 in receipts, or $10 cash plus $90 in receipts).

The petty cash fund is reimbursed when it is nearing zero and at the end of an accounting period. The petty cashier sorts the paid receipts by the type of expense or account and then totals the receipts. The petty cashier gives all paid receipts to the company cashier, who stamps all receipts paid so they cannot be reused, files them for recordkeeping, and gives the petty cashier a check. When this check is cashed and the money placed in the cashbox, the total money in the cashbox is restored to its original amount. The fund is now ready for a new cycle of petty cash payments.

Point: Companies use surprise petty cash counts for verification.

10 Michigan Street Chicago, IL 60521

Pay to the order of $

Date........................... 20 .......

Dollars

Memo ...............................................

99-DT/101

No. 119CheckZ-Mart November 12

1,200Trex NO INVOICE APPROVALDocument

By Date

Receiving Report

Date:...............

Request purchase of the follo wing item(s):

Model No. Descr iption

Total

Quantity Price Amount

Invoice

Purchase Order

Purchase Requisition

Cashier

Accounting

Receiving

Supplier (vendor)

Purchasing

Sender

Requesting

Accounting

Supplier (vendor)

Accounting; Requesting; and Purchasing

Cashier

Supplier; Requesting; Receiving; and Accounting

Purchasing; and Accounting

Receiver(s)

Voucher

19

9

9

9

8

EXHIBIT 6.1 Document Flow in a Voucher System

Which of the following statements are true regarding the control of cash receipts and cash payments? 1. Over-the-counter cash sales should be recorded on a cash register after each sale. 2. Custody over cash should be separate from the recordkeeping of cash. 3. For control of cash receipts that arrive through the mail, two people should be present for opening

that mail. 4. One key to controlling cash payments is to require that no expenditures be made by check;

instead, all expenditures should be made from petty cash. 5. A voucher system of control should be applied only to purchases of inventory and never to other

expenditures.

Solution

1. True 2. True 3. True 4. False 5. False

Control of Cash Receipts and Payments

NEED-TO-KNOW 6-2

P1 C2

Do More: QS 6-2, QS 6-4, QS 6-5, E 6-4, E 6-5, E 6-6,

E 6-7

P2 Explain and record petty cash fund transactions.

For

Date

Charge to Amount Approved by

Received by

EXHIBIT 6.2 Petty Cash Receipt

Chapter 6 Cash, Fraud, and Internal Control 243

Illustrating a Petty Cash Fund Assume Z-Mart sets up a petty cash fund on November 1. A $75 check is drawn, cashed, and the proceeds given to the petty cashier. The entry to record the setup of this petty cash fund is

After the petty cash fund is established, the Petty Cash account is not debited or credited again unless the amount of the fund is changed.

Next, assume that Z-Mart’s petty cashier makes several November payments from petty cash. On November 27, after making a $46.50 cash payment for tile cleaning, only $3.70 cash remains in the fund. The petty cashier then summarizes and totals the petty cash receipts as shown in Exhibit 6.3.

The petty cash payments report and all receipts are given to the company cashier in exchange for a $71.30 check to reimburse the fund. The petty cashier cashes the check and puts the $71.30 cash in the petty cashbox. The company records this reimbursement as follows. A petty cash fund is usually reimbursed at the end of an accounting period so that expenses are recorded in the proper period, even if the fund is not low on money.

EXHIBIT 6.3 Petty Cash Payments Report

Petty Cash Payments Report

Miscellaneous Expense Nov . 27 Tile cleaning . . . . . . . . . . . . . . . . . . . . . . . . . . . . . . . . . . . . . . $ 46 .50

Merchandise Inventory (transportation-in) Nov . 5 Transport of merchandise purchased . . . . . . . . . . . . . . . . . . 15 .05

Delivery Expense Nov . 18 Customer’s package delivered . . . . . . . . . . . . . . . . . . . . . . . 5 .00

Office Supplies Expense Nov . 15 Purchase of office supplies immediately used . . . . . . . . . . . 4 .75

Total . . . . . . . . . . . . . . . . . . . . . . . . . . . . . . . . . . . . . . . . . . . . . . . . . . . . . . . . . $71.30

Point: This report also can include receipt number and names of those who approved and received cash payment (see Need-to-Know 6-3).

Increasing or Decreasing a Petty Cash Fund A decision to increase or decrease a petty cash fund is often made when reimbursing it. Assume Z-Mart decides to increase its petty cash fund from $75 to $100 on November 27 when it reimburses the fund. The entries required are to (1) reimburse the fund as usual (see the preceding November 27 entry) and (2) increase the fund amount as follows.

Nov . 1 Petty Cash . . . . . . . . . . . . . . . . . . . . . . . . . . . . . . . . . . . . . . . . . 75

Cash . . . . . . . . . . . . . . . . . . . . . . . . . . . . . . . . . . . . . . . . . 75

Establish a petty cash fund.

Assets = Liabilities + Equity +75 −75

Nov . 27 Miscellaneous Expenses . . . . . . . . . . . . . . . . . . . . . . . . . . . . . . . . . . . . . 46 .50

Merchandise Inventory . . . . . . . . . . . . . . . . . . . . . . . . . . . . . . . . . . . . . . 15 .05

Delivery Expense . . . . . . . . . . . . . . . . . . . . . . . . . . . . . . . . . . . . . . . . . . . 5 .00

Office Supplies Expense . . . . . . . . . . . . . . . . . . . . . . . . . . . . . . . . . . . . . 4 .75

Cash* . . . . . . . . . . . . . . . . . . . . . . . . . . . . . . . . . . . . . . . . . . . . . . . . 71 .30

Reimburse petty cash. *$75 fund bal. − $3.70 cash remaining.

Assets = Liabilities + Equity −71.30 −46.50 +15.05 − 5.00 − 4.75

Instead, if it decreases the petty cash fund from $75 to $55 on November 27, the entry is

Nov . 27 Petty Cash . . . . . . . . . . . . . . . . . . . . . . . . . . . . . . . . . . . . . . . . . . . . . . . . 25

Cash . . . . . . . . . . . . . . . . . . . . . . . . . . . . . . . . . . . . . . . . . . . . . . . . 25

Increase petty cash fund from $75 to $100.

Nov . 27 Cash . . . . . . . . . . . . . . . . . . . . . . . . . . . . . . . . . . . . . . . . . . . . . . . . . . . . . 20

Petty Cash . . . . . . . . . . . . . . . . . . . . . . . . . . . . . . . . . . . . . . . . . . . . 20

Decrease petty cash fund from $75 to $55.

244 Chapter 6 Cash, Fraud, and Internal Control

Event Petty Cash Cash Expenses

Set up fund . . . . . . . . Dr . Cr . —

Reimburse fund . . . . . — Cr . Dr .

Increase fund . . . . . . Dr . Cr . —

Decrease fund . . . . . . Cr . Dr . —

Summary of Petty Cash Accounting Cash Over and Short Sometimes a petty cashier fails to get a receipt for payment or overpays for the amount due. When this occurs and the fund is later reimbursed, the petty cash payments report plus the cash remaining will not equal the fund bal- ance. This mistake causes the fund to be short. This shortage is recorded as an ex- pense in the reimbursing entry with a debit to the Cash Over and Short account. (An overage in the petty cash fund is recorded with a credit to Cash Over and Short in the reimbursing entry.)

Following is the June 1 entry to reimburse a $200 petty cash fund when its payments report shows $178 in miscellaneous expenses and only $15 cash remains.

$200 Petty Cash Fund

$15 Cash $7 Short $178 Receipts

For

Date

Charge to Amount

PETTY CASH RECEIPT Z-Mart No. 9

Approved by

Received by

June 1 Miscellaneous Expenses . . . . . . . . . . . . . . . . . . . . . . . . . . . . . . . . . . . . . 178

Cash Over and Short . . . . . . . . . . . . . . . . . . . . . . . . . . . . . . . . . . . . . . . 7 Cash* . . . . . . . . . . . . . . . . . . . . . . . . . . . . . . . . . . . . . . . . . . . . . . . . 185

Reimburse petty cash. *$200 fund bal. − $15 cash remaining.

Get Clued In There are clues to fraudulent activities. Clues from accounting include (1) an increase in customer refunds—could be fake, (2) missing documents—could be used for fraud, (3) differences between bank deposits and cash receipts—could be cash embezzled, and (4) delayed recording—could reflect fraudulent records. Clues from employees include (1) lifestyle change—could be embezzlement, (2) too close with suppliers—could signal fraudulent transactions, and (3) refusal to leave job, even for vacations—could conceal fraudulent activities. ■

Ethical Risk

Bacardi Company established a $150 petty cash fund with Eminem as the petty cashier. When the fund balance reached $19 cash, Eminem prepared a petty cash payments report, which follows.

Petty Cash System

NEED-TO-KNOW 6-3

P2 Petty Cash Payments Report Receipt No. Account Charged Approved by Received by

12 Delivery Expense . . . . . . . . . . . . . . . . $ 29 Eminem A . Smirnoff

13 Merchandise Inventory . . . . . . . . . . . 18 Eminem J . Daniels

15 (Omitted) . . . . . . . . . . . . . . . . . . . . . . . 32 Eminem C . Carlsberg

16 Miscellaneous Expense . . . . . . . . . . . 41 (Omitted) J . Walker

Total . . . . . . . . . . . . . . . . . . . . . . . . . . $120

Required

1. Identify four internal control weaknesses from the petty cash payments report. 2. Prepare general journal entries to record a. Establishment of the petty cash fund. b. Reimbursement of the fund. (Assume for this part only that petty cash Receipt No. 15 was issued

for miscellaneous expenses.) 3. What is the Petty Cash account balance immediately before reimbursement? After reimbursement?

Solution

1. Four internal control weaknesses that are apparent from the payments report include a. Petty cash Receipt No. 14 is missing. This raises questions about the petty cashier’s management of

the fund. b. The $19 cash balance means that $131 has been withdrawn ($150 − $19 = $131). However, the

total amount of the petty cash receipts is only $120 ($29 + $18 + $32 + $41). The fund is $11 short of cash ($131 − $120 = $11). Management should investigate.

c. The petty cashier (Eminem) did not sign petty cash Receipt No. 16. This could have been a mistake on his part or he might not have authorized the payment.

d. Petty cash Receipt No. 15 does not say which account to charge. Management should check with C. Carlsberg and the petty cashier (Eminem) about the transaction. Without further information, debit Miscellaneous Expense.

Chapter 6 Cash, Fraud, and Internal Control 245

2. Petty cash general journal entries. a. Entry to establish the petty cash fund. b. Entry to reimburse the fund.

Petty Cash . . . . . . . . . . . . . . . . . . . . . . . . . . 150 Delivery Expense . . . . . . . . . . . . . . . . . . . . . . . . . . . 29

Cash . . . . . . . . . . . . . . . . . . . . . . . . . . 150 Merchandise Inventory . . . . . . . . . . . . . . . . . . . . . . 18

Miscellaneous Expense ($41 + $32) . . . . . . . . . . . 73 Cash Over and Short . . . . . . . . . . . . . . . . . . . . . . . . 11

Cash ($150 fund bal . − $19 cash rem .) . . . . . 131

3. The Petty Cash account balance always equals its fund balance, in this case $150. This account balance does not change unless the fund is increased or decreased.

Do More: QS 6-6, E 6-8, E 6-9, E 6-10, P 6-2, P 6-3

Basic Bank Services Banks safeguard cash and provide detailed records of cash transactions. They provide services and documents that help control cash, which is the focus of this section.

Bank Account, Deposit, and Check A bank account is used to deposit money for safekeeping and helps control withdrawals. Persons authorized to write checks on the account must sign a signature card, which the bank uses to verify signatures.

Each bank deposit has a deposit ticket, which lists items such as currency, coins, and checks deposited along with amounts. The bank gives the customer a receipt as proof of the deposit. Exhibit 6.4 shows a deposit ticket.

Point: Firms often have multiple bank accounts for different needs and for specific transactions such as payroll.

BANKING ACTIVITIES AS CONTROLS

C H

EC KS

L IS

T S

IN G

LY D

O LL

A RS

C EN

TS

1 14

-2 87

/ 9 39

90 50

2 82

-7 59

/ 3 39

82 80

3 4 5 6 7 8 9 10 11 12 13 14

TO TA

L

EN TE

R TO

TA L

O N

T H

E FR

O N

T O

F TH

IS T

IC KE

T

76 -9

07 / 9

19 30

20

20 3

50

DEPOSIT TICKET

.......................................... (Memo)

USE OTHER SIDE FOR ADDITIONAL LISTINGS. BE SURE EACH ITEM IS PROPERLY ENDORSED.

Date ...................... 20 ......... CURRENCY

TOTAL

NET DEPOSIT

36 October 2

Deposit checks

19

203 240

240

50

50 00

00

CASH COIN

LIST CHECKS SINGLY

TOTAL FROM OTHER SIDE

Front Back

901 Main Street Hillcrest, NY 11749

EXHIBIT 6.4 Deposit Ticket

To withdraw money, the depositor can use a check, which is a document telling the bank to pay a specified amount to a designated recipient. A check involves three parties: a maker who signs the check, a payee who is the recipient, and a bank (or payer) on which the check is drawn. The bank provides the depositor the checks. Exhibit 6.5 shows one type of check. It has an optional remittance advice explaining the payment. The memo line is used for an explanation.

Electronic Funds Transfer Electronic funds transfer (EFT) is the electronic trans- fer of cash from one party to another. Companies are increasingly using EFT because of its convenience and low cost. Payroll, rent, utilities, insurance, and interest payments are usually done by EFT. The bank statement lists cash withdrawals by EFT with the checks and other deductions. Cash receipts by EFT are listed with deposits and other additions.

246 Chapter 6 Cash, Fraud, and Internal Control

Bank Statement Usually once a month, the bank sends a bank statement showing the account activity. Different banks use different formats for their bank statements, but all of them include the following.

1. Beginning-of-period account balance. 2. Checks and other debits decreasing the account during the period. 3. Deposits and other credits increasing the account during the period. 4. End-of-period account balance.

Exhibit 6.6 shows one type of bank statement. Part A of Exhibit 6.6 summarizes changes in the account. B lists paid checks along with other debits. C lists deposits and credits to the account.

Canceled checks are checks the bank has paid and deducted from the customer’s account. We say such checks cleared the bank. Other usual deductions on a bank statement include (1) bank

Point: Good control is to send a copy of the bank statement directly to a party without access to cash or recordkeeping.

Pay to the order of

Date Description Gross Amount Deductions Net Amount

VideoBuster Company, Hillcrest, NY

Detach this portion before cashing

$

........................ 20 .......

Dollars

Memo

99-DT/101

No. 438CheckMaker

Payee

Payer

Remittance Advice

901 Main Street Hillcrest, NY 11749

Hillcrest Lighting

Lighting design, Invoice No. 4658

October 3

375.

$375.00 $375.0010-3-19

19

Three Hundred and Seventy-Five Dollars and

Store Lighting Design

EXHIBIT 6.5 Check with Remittance Advice

VideoBuster Company 901 Main Street Hillcrest, NY 11749

October 31, 2019 Statement Date

494 504 2 Account Number

Previous Balance

Symbols: CM–Credit Memo DM–Debit Memo

EC–Error Correction IN–Interest Earned

NSF–Non-Su�cient Funds EFT–Electronic Funds Transfer

SC–Service Charge OD–Overdraft

Checks and Debits Deposits and Credits

Total Checks and Debits Total Deposits and Credits Current Balance

1,610

Date Oct. 3 Oct. 12 Oct. 14 Oct. 16 Oct. 23 Oct. 25

Oct. 2 Oct. 15 Oct. 16 Oct. 23 Oct. 31

240 No. Amount Date Amount

723 1,163 2,050

121 375 100 EFT

122 70 485 CM 330

123 25 EFT 8 IN125 15

127 185

20 NSF 10 DM

23 DM

CB

A

Bank Statement Member FDIC

Oct. 26

EXHIBIT 6.6 Bank Statement

Bank’s Liability to VideoBuster

Sep. 30 bal. 1,610 CRs 1,163 DRs 723

Oct. 31 bal. 2,050

Point: Debit memos (DM) from the bank produce credits on the depositor’s books. Credit memos (CM) from the bank produce debits on the depositor’s books.

Forms of Check Fraud (CkFraud.org)

•   Forged signatures—legitimate  checks with fake payer signature

•   Forged endorsements—stolen check  that is endorsed and cashed by someone other than the payee

•   Counterfeit checks—fraudulent  checks with fake payer signature

•   Altered checks—legitimate check  altered (such as changed payee or amount) to benefit perpetrator

•   Check kiting—deposit check from  one bank account (without sufficient funds) into a second bank account

Chapter 6 Cash, Fraud, and Internal Control 247

service fees, (2) checks deposited that are uncollectible, (3) corrections of previous errors, (4) withdrawals through automated teller machines (ATMs), and (5) payments arranged in advance by a depositor. A debit memorandum notifies a depositor of a deduction.

Increases to the depositor’s account include amounts the bank collects on behalf of the depositor and the corrections of previous errors. A credit memorandum notifies the depositor of all increases. Banks that pay interest on checking accounts credit interest earned to the depos- itor’s account each period. In Exhibit 6.6, the bank credits $8 of interest to the account.

Bank Reconciliation The balance of a checking account on the bank statement rarely equals the depositor’s book bal- ance (from its records). This is due to information that one party has that the other does not. We must therefore verify the accuracy of both the depositor’s records and the bank’s records. To do this, we prepare a bank reconciliation to explain differences between the checking account bal- ance in the depositor’s records and the balance on the bank statement. The following explains bank and book adjustments.

Bank Balance Adjustments + Deposits in transit (or outstanding deposits). Deposits in transit are deposits made and

recorded in the depositor’s books but not yet listed on the bank statement. For example, companies can make deposits (in the night depository) after the bank is closed. If such a deposit occurred on a bank statement date, it would not appear on this period’s statement. The bank would record such a deposit on the next business day, and it would appear on the next period’s bank statement. Deposits mailed to the bank near the end of a period also can be in transit and not listed on the bank statement.

− Outstanding checks. Outstanding checks are checks written by the depositor, subtracted on the depositor’s books, and sent to the payees but not yet turned in for payment at the bank statement date.

± Bank errors. Any errors made by the bank are accounted for in the reconciliation. To find errors, we (a) compare deposits on the bank statement with deposits in the accounting records and (b) compare canceled checks on the bank statement with checks recorded in the accounting records.

Book Balance Adjustments + Interest earned and unrecorded cash receipts. Banks sometimes collect notes for

depositors. Banks also receive electronic funds transfers to the depositor’s account. When a bank collects an item, it is added to the depositor’s account, less any service fee. The bank statement also includes any interest earned.

− Bank fees and NSF checks. A company sometimes deposits another party’s check that is uncollectible. This check is called a nonsufficient funds (NSF) check. The bank initially credits (increases) the depositor’s account for the check. When the check is uncollectible, the bank debits (reduces) the depositor’s account for that check. The bank may charge the depositor a fee for processing an uncollectible check. Other bank charges include printing new checks and service fees.

± Book errors. Any errors made by the depositor in the company books are accounted for in the reconciliation. To find errors, we use the same procedures described in the “Bank errors” section above.

Adjustments Summary Following is a summary of bank and book adjustments. Each of these items has already been recorded by either the bank or the company, but not both.

Point: Books refer to accounting records.

Point: The person preparing the bank reconciliation should not be responsible for processing cash receipts, managing checks, or maintaining cash records.

Point: Your checking account is a liability from the bank’s perspec- tive (but an asset from yours). When you make a deposit, they “credit your account.” Credits in- crease the bank’s liability to you. When you write a check or use your debit card, the bank de- creases its liability to you; they “debit your account.” Debits de- crease the bank’s liability to you.

P3 Prepare a bank reconciliation.

Point: Businesses with few employees often allow record- keepers to both write checks and keep the general ledger. If this is done, the owner must do the bank reconciliation.

Bank Balance Adjustments

Add deposits in transit .

Subtract outstanding checks .

Add or subtract corrections of bank errors .

Book Balance Adjustments

Add interest earned and unrecorded cash receipts .

Subtract bank fees and NSF checks .

Add or subtract corrections of book errors .

248 Chapter 6 Cash, Fraud, and Internal Control

Bank Reconciliation Demonstration In preparing the bank reconciliation, refer to Exhibit 6.7 and steps 1 through 8 .

1 Enter VideoBuster’s bank balance of $2,050 taken from the bank statement. 2 Add any unrecorded deposits and bank errors that understate the bank balance to the bank

balance. VideoBuster’s $145 deposit in the bank’s night depository on October 31 is not listed on its bank statement.

3 Subtract any outstanding checks and bank errors that overstate the bank balance from the bank balance. VideoBuster’s comparison of canceled checks with its books shows two checks outstanding: No. 124 for $150 and No. 126 for $200.

4 Compute the adjusted bank balance. 5 Enter VideoBuster’s cash account book balance of $1,405 taken from its accounting records. 6 Add any unrecorded cash receipts, interest earned, and errors understating the book balance

to the book balance. VideoBuster’s bank statement shows the bank collected a note receiv- able and increased VideoBuster’s account for $485. The bank statement also shows $8 for interest earned that was not yet recorded on the books.

7 Subtract any unrecorded bank fees, NSF checks, and errors overstating the book balance from the book balance. Deductions on VideoBuster’s bank statement that are not yet recorded include (a) a $23 charge for check printing and (b) an NSF check for $30. (The NSF check is dated October 16 and was in the book balance.)

8 Compute the adjusted book balance.

Verify that the two adjusted balances from steps 4 and 8 are equal (reconciled).

Point: Outstanding checks are identified by comparing canceled checks on the bank statement with checks recorded. This in- cludes identifying any outstand- ing checks listed on the previous period’s bank reconciliation that are not included in the canceled checks on this period’s bank statement.

VIDEOBUSTER Bank Reconciliation October 31, 2019

1 Bank statement balance . . . . . . . . . . . $ 2,050 5 Book balance . . . . . . . . . . . . . . . . . . . . . $ 1,405

2 Add 6 Add

Deposit of Oct . 31 in transit . . . . . . 145 Collected note . . . . . . . . . . . . . . . . . $485

2,195 Interest earned . . . . . . . . . . . . . . . . . 8 493

3 Deduct 1,898

Outstanding checks 7 Deduct

No . 124 . . . . . . . . . . . . . . . . . . . . $150 Check printing charge . . . . . . . . . . . 23

No . 126 . . . . . . . . . . . . . . . . . . . . 200 350 NSF check . . . . . . . . . . . . . . . . . . . . . 30 53

4 Adjusted bank balance . . . . . . . . . . . $1,845 8 Adjusted book balance . . . . . . . . . . . . $1,845

Balances are equal (reconciled)

EXHIBIT 6.7 Bank Reconciliation

Adjusting Entries from a Bank Reconciliation A bank reconciliation often finds unrecorded items that need recording by the company. In VideoBuster’s reconciliation, the adjusted balance of $1,845 is the correct balance as of October 31. But the company’s account- ing records show a $1,405 balance. We make adjusting entries so that the book balance equals the adjusted balance. Only items impacting the book balance need entries. Exhibit 6.7 shows that four entries are required.

Collection of Note The first entry is to record collection of a note receivable by the bank.

Interest Earned The second entry records interest earned.

Oct . 31 Cash . . . . . . . . . . . . . . . . . . . . . . . . . . . . . . . . . . . . . . . . . . . . . . 485

Notes Receivable . . . . . . . . . . . . . . . . . . . . . . . . . . . . . . . 485

Record note collected by bank.

Assets = Liabilities + Equity +485 −485

Oct . 31 Cash . . . . . . . . . . . . . . . . . . . . . . . . . . . . . . . . . . . . . . . . . . . . . . 8

Interest Revenue . . . . . . . . . . . . . . . . . . . . . . . . . . . . . . . . 8

Record interest earned in checking account.

Assets = Liabilities + Equity +8 +8

Chapter 6 Cash, Fraud, and Internal Control 249

Check Printing The third entry records expenses for the check printing charge.

NSF Check The fourth entry records the NSF check that is returned as uncollectible. The check was from T. Woods in payment of his account. The bank deducted $30 total from VideoBuster’s account. This means the entry must reverse the effects of the original entry when the check was received.

Point: The company will try to col- lect the $30 from the customer.

Assets = Liabilities + Equity −23 −23

Oct . 31 Miscellaneous Expenses . . . . . . . . . . . . . . . . . . . . . . . . . . . . . . 23

Cash . . . . . . . . . . . . . . . . . . . . . . . . . . . . . . . . . . . . . . . . . . 23

Check printing charge.

Assets = Liabilities + Equity +30 −30

Oct . 31 Accounts Receivable—T . Woods . . . . . . . . . . . . . . . . . . . . . . . . 30

Cash . . . . . . . . . . . . . . . . . . . . . . . . . . . . . . . . . . . . . . . . . . 30

Charge Woods’s account for $30 NSF check.

After these four entries are recorded, the book balance of cash is adjusted to the correct amount of $1,845 (the adjusted book balance). The Cash T-account to the side shows the computation, where entries match the steps in Exhibit 6.7.

Point: Need-to-Know 6-4 shows an adjusting entry for an error correction.

Cash

Unadj . bal . 1,405 6 485 7 23 6 8 7 30

Adj . bal . 1,845

Cause for Alarm The Association of Certified Fraud Examiners (ACFE) reports that the primary factor contributing to fraud is the lack of internal controls (30%), followed by the override of existing controls (19%), lack of management review (18%), poor tone at the top (10%), and lack of competent oversight (8%). These findings highlight the impor- tance of internal controls over cash. Source: “Report to the Nations,” ACFE. ■

Ethical Risk

©Redpixel.pl/Shutterstock

The following information is available to reconcile Gucci’s book balance of cash with its bank statement cash balance as of December 31. a. The December 31 cash balance according to the accounting records is $1,610, and the bank statement

cash balance for that date is $1,900. b. Gucci’s December 31 daily cash receipts of $800 were placed in the bank’s night depository on

December 31 but do not appear on the December 31 bank statement. c. Gucci’s comparison of canceled checks with its books shows three checks outstanding: No. 6242 for

$200, No. 6273 for $400, and No. 6282 for $100. d. When the December checks are compared with entries in the accounting records, it is found that Check

No. 6267 had been correctly drawn (taken from the bank) for $340 to pay for office supplies but was erroneously entered in the accounting records as $430.

e. The bank statement shows the bank collected a note receivable and increased Gucci’s account for $470. Gucci had not recorded this transaction before receiving the statement.

f. The bank statement included an NSF check for $150 received from Prada Inc. in payment of its account. It also included a $20 charge for check printing. Gucci had not recorded these transactions before receiving the statement.

Required

1. Prepare the bank reconciliation for this company as of December 31. 2. Prepare the journal entries to make Gucci’s book balance of cash equal to the reconciled cash balance

as of December 31.

Bank Reconciliation

NEED-TO-KNOW 6-4

P3

250 Chapter 6 Cash, Fraud, and Internal Control

Solutions

Part 1

GUCCI Bank Reconciliation

December 31

Bank statement balance . . . . . . . . . . . . $1,900 Book balance . . . . . . . . . . . . . . $1,610

Add Add

Deposit of Dec . 31 . . . . . . . . . . . . . . 800 Error (Ck . 6267) . . . . . . . . . . $ 90

2,700 Collected note . . . . . . . . . . . 470 560

2,170

Deduct Deduct

Outstanding Checks No . 6242 . . . . $200 NSF check . . . . . . . . . . . . . . 150

6273 . . . . 400 Printing fee . . . . . . . . . . . . . . 20

6282 . . . . 100 700 170

Adjusted bank balance . . . . . . . . . . . . . $2,000 Adjusted book balance . . . . . . $2,000

Part 2

Dec . 31 Cash . . . . . . . . . . . . . . . . . . . . . . . . . . . . . 90

Office Supplies . . . . . . . . . . . . . . . . 90

Correct an entry error.

Dec . 31 Cash . . . . . . . . . . . . . . . . . . . . . . . . . . . . . 470

Notes Receivable . . . . . . . . . . . . . . 470

Record note collection.

Dec . 31 Accounts Receivable—Prada Inc . . . . . . . 150

Cash . . . . . . . . . . . . . . . . . . . . . . . . . 150

Charge account for NSF check.

Dec . 31 Miscellaneous Expenses . . . . . . . . . . . . . 20

Cash . . . . . . . . . . . . . . . . . . . . . . . . . 20

Record check printing charge.

Do More: QS 6-7, QS 6-8, QS 6-9, E 6-11, E 6-12, E 6-13,

E 6-14, P 6-4, P 6-5

Days’ Sales UncollectedDecision Analysis

One measure of how quickly a company can convert its accounts receivable into cash is the days’ sales uncollected, also called days’ sales in receivables, which is defined in Exhibit 6.8.A1

Compute the days’ sales uncollected ratio and use it to assess liquidity.

Days’ sales uncollected = Accounts receivable

Net sales × 365EXHIBIT 6.8

Days’ Sales Uncollected

We use days’ sales uncollected to estimate how much time is likely to pass before the current amount of accounts receivable is received in cash. It is used to determine if cash is being collected quickly enough to pay upcoming obligations. Days’ sales uncollected are shown for Starbucks and Jack in the Box in Exhibit 6.9.

Company Figure ($ millions) Current Year 1 Year Ago 2 Years Ago

Starbucks Accounts receivable . . . . . . . . . . . . . . . . . . . . $ 870 $ 769 $ 719 Net sales . . . . . . . . . . . . . . . . . . . . . . . . . . . . . $22,387 $21,316 $19,163

Days’ sales uncollected . . . . . . . . . . . . . . . . 14.2 days 13.2 days 13.7 days Jack in the Box Accounts receivable . . . . . . . . . . . . . . . . . . . . $ 69 $ 73 $ 48 Net sales . . . . . . . . . . . . . . . . . . . . . . . . . . . . . $ 1,554 $ 1,599 $ 1,540

Days’ sales uncollected . . . . . . . . . . . . . . . . 16.2 days 16.7 days 11.4 days

EXHIBIT 6.9 Analysis Using Days’ Sales Uncollected

Days’ sales uncollected for Starbucks is 14.2 days for the current year, computed as ($870/$22,387) × 365 days. This means it takes 14.2 days to collect cash from ending accounts receivable. This number reflects one or more of the following factors: a company’s ability to collect receivables, customer financial health, customer payment strategies, and discount terms. To further assess Starbucks, we compare it to Jack in the Box. We see that Starbucks’s 14.2 days’ sales uncollected is better than Jack in the Box’s 16.2 days’ sales

Chapter 6 Cash, Fraud, and Internal Control 251

uncollected for the current year. Starbucks took less time to collect its receivables. The less time money is tied up in receivables, the better.

Sales Representative The sales staff are told to help reduce days’ sales uncollected for cash management purposes. What can you, a salesperson, do to reduce days’ sales uncollected? ■ Answer: A salesperson can (1) push cash sales over credit, (2) identify customers most delayed in their payments and require earlier payments or cash sales, and (3) eliminate credit sales to customers that never pay.

Decision Maker

Prepare a bank reconciliation for Jamboree Enterprises for the month ended November 30. The following information is available as of November 30. a. On November 30, the company’s book balance of cash is $16,380, but its bank statement shows a

$38,520 balance. b. Checks No. 2024 for $4,810 and No. 2026 for $5,000 are outstanding. c. In comparing the canceled checks on the bank statement with the entries in the accounting records, it

is found that Check No. 2025 in payment of rent is correctly drawn (taken from the bank) for $1,000 but is erroneously entered in the accounting records as $880.

d. The November 30 deposit of $17,150 was placed in the night depository after banking hours on that date, and this amount does not appear on the bank statement.

e. In reviewing the bank statement, a check written by Jumbo Enterprises in the amount of $160 was erroneously drawn against Jamboree’s account.

f. The bank statement says that the bank collected a $30,000 note and $900 of interest was earned. These transactions were not recorded by Jamboree prior to receiving the statement.

g. The bank statement lists a $1,100 NSF check received from a customer, Marilyn Welch. Jamboree had not recorded the return of this check before receiving the statement.

h. Bank service charges for November total $40. These charges were not recorded by Jamboree before receiving the statement.

PLANNING THE SOLUTION Set up a bank reconciliation (as in Exhibit 6.7). Examine each item a through h to determine whether it affects the book or the bank balance and

whether it should be added or subtracted. After all items are analyzed, complete the reconciliation and arrive at a reconciled balance between the

bank side and the book side. For each reconciling item on the book side, prepare an adjusting entry. Additions to the book side

require an adjusting entry that debits Cash. Deductions on the book side require an adjusting entry that credits Cash.

SOLUTION

COMPREHENSIVE

Preparing Bank Reconciliation and Adjusting Entries

NEED-TO-KNOW 6-5

JAMBOREE ENTERPRISES Bank Reconciliation

November 30

Bank statement balance . . . . . . . . $ 38,520 Book balance . . . . . . . . . . . . . . . . . $ 16,380

Add Add

Deposit of Nov . 30 . . . . . . . . . . $17,150 Collection of note . . . . . . . . . . . $30,000

Bank error (Jumbo) . . . . . . . . . 160 17,310 Interest earned . . . . . . . . . . . . . 900 30,900

55,830 47,280

Deduct Deduct

Outstanding checks NSF check (M . Welch) . . . . . . . . 1,100

No . 2024 . . . . . . . . . . . . . . . . 4,810 Recording error (No . 2025) . . . 120

No . 2026 . . . . . . . . . . . . . . . . 5,000 9,810 Service charge . . . . . . . . . . . . . 40 1,260

Adjusted bank balance . . . . . . . . $46,020 Adjusted book balance . . . . . . . . $46,020

PURCHASE REQUISITION

For Purchasing Department use only: Order Date P.O. No.

Z-Mart

Request purchase of the following item(s):

No. 917

10-30-19 P98

Date Preferred Vendor

Reason for Request Replenish inventory Approval for Request

DESCRIPTION

Boys/Girls –Speed Demon 1

QUANTITY

Toddler –Challenger X7 1

MODEL NO.

CH 015

SD 099

Date Preferred Vendor Trex

October 28, 2019From Sporting Goods Department

To Purchasing Department

EXHIBIT 6A.1 Purchase Requisition

252 Chapter 6 Cash, Fraud, and Internal Control

Nov . 30 Cash . . . . . . . . . . . . . . . . . . . . . . . . . . . . . . . 30,000

Notes Receivable . . . . . . . . . . . . . . . . 30,000

Record collection of note.

Nov . 30 Cash . . . . . . . . . . . . . . . . . . . . . . . . . . . . . . . 900

Interest Revenue . . . . . . . . . . . . . . . . . 900

Record collection of revenue.

Nov . 30 Accounts Receivable—M . Welch . . . . . . . . . 1,100

Cash . . . . . . . . . . . . . . . . . . . . . . . . . . . 1,100

Reinstate account due from an NSF check.

Nov . 30 Rent Expense . . . . . . . . . . . . . . . . . . . . . . . . 120

Cash . . . . . . . . . . . . . . . . . . . . . . . . . . . 120

Correct recording error on Check No. 2025.

Nov . 30 Miscellaneous Expenses . . . . . . . . . . . . . . . 40

Cash . . . . . . . . . . . . . . . . . . . . . . . . . . . 40

Record bank service charges.

Required Adjusting Entries for Jamboree

APPENDIX

Documentation and Verification6A This appendix covers the documents of a voucher system of control.

Purchase Requisition Department managers are usually not allowed to place orders directly with suppliers for control purposes. Instead, a department manager must inform the purchasing depart- ment of its needs by preparing and signing a purchase requisition, which lists the merchandise requested to be purchased—see Exhibit 6A.1. Two copies of the purchase requisition are sent to the purchasing department, which then sends one copy to the accounting department. When the accounting department receives a purchase requisition, it creates and maintains a voucher for this transaction. The requesting department keeps a third copy.

P4 Describe use of documentation and verification to control cash payments.

Purchase Order A purchase order is a document the purchasing department uses to place an order with a vendor (seller or supplier). A purchase order authorizes a vendor to ship merchandise at the stated price and terms—see Exhibit 6A.2. When the purchasing department receives a purchase requisi- tion, it prepares at least five copies of a purchase order. The copies are distributed as follows: copy 1 to the vendor as a purchase request to ship merchandise; copy 2, along with a copy of the purchase requisition, to the accounting department, where it is entered in the voucher and used in approving payment of the invoice; copy 3 to the requesting department to inform its manager of the purchase; copy 4 to the receiving department without order quantity so it can compare with goods received and provide an independent count of goods received; and copy 5 kept on file by the purchasing department.

Invoice An invoice is an itemized statement of goods prepared by the vendor listing the customer’s name, items sold, sales prices, and terms of sale. An invoice is also a bill sent to the buyer from the supplier. From the vendor’s point of view, it is a sales invoice. The buyer, or vendee, treats it as a purchase

Point: This appendix shows one example of a common voucher system design, but not the only design.

Chapter 6 Cash, Fraud, and Internal Control 253

invoice. The invoice is sent to the buyer’s accounting department, where it is placed in the voucher. (Refer back to Exhibit 4.6, which shows Z-Mart’s purchase invoice.)

Receiving Report Many companies have a receiving department to receive all merchandise and purchased assets. When each shipment arrives, this receiving department counts the goods and checks them for damage and agreement with the purchase order. It then prepares four or more copies of a receiv- ing report, which is used within the company to notify that ordered goods have been received and to describe the quantities and condition of the goods. One copy is sent to accounting and placed in the voucher. Copies also are sent to the requesting department and the purchasing department to notify them that the goods have arrived. The receiving department keeps a copy in its files.

Invoice Approval When a receiving report arrives, the accounting department should have cop- ies of the following documents in the voucher: purchase requisition, purchase order, and invoice. With the information in these documents, the accounting department can record the purchase and approve its pay- ment. In approving an invoice for payment, it checks and compares information across all documents. To verify this information and to ensure that no step is missing, it often uses an invoice approval, also called check authorization—see Exhibit 6A.3. An invoice approval is a checklist of steps necessary for approving an invoice for recording and payment. It is a separate document either filed in the voucher or preprinted (or stamped) on the voucher.

PURCHASE ORDERZ-Mart 10 Michigan Street

Chicago, Illinois 60521 No. P98

To: Trex W9797 Cherry Road Antigo, Wisconsin 54409

Date 10-30-19

2/15, n/30

FOB Destination As soon as possibleShip by

Terms Request shipment of the following item(s):

All shipments and invoices must include purchase order number.

ORDERED BY

Model No. Description

Boys/Girls –Speed Demon 1

Quantity Price Amount

350 350

CH 015

SD 099

Toddler –Challenger X7 1 150 150

Point: Shipping terms and credit terms are shown on the purchase order.

EXHIBIT 6A.2 Purchase Order

INVOICE APPROVAL

Purchase requisition

Purchase order

Receiving report

Invoice:

Price

Calculations

Terms

Approved for payment

917

P98

R85

4657

10-28-19TZ

JW

SK

JK

JK

JK

BC

10-30-19

11-03-19

11-12-19

11-12-19

11-12-19

11-12-19

BY DATEDOCUMENT

EXHIBIT 6A.3 Invoice Approval

As each step in the checklist is approved, the person initials the invoice approval and records the cur- rent date. Final approval means the following steps have occurred.

1. Requisition check: Items on invoice are requested per purchase requisition. 2. Purchase order check: Items on invoice are ordered per purchase order. 3. Receiving report check: Items on invoice are received per receiving report. 4. Invoice check: Price: Invoice prices are as agreed with the vendor. Calculations: Invoice has no mathematical errors. Terms: Terms are as agreed with the vendor.

Voucher Once an invoice has been checked and approved, the voucher is complete. A complete voucher is a record summarizing a transaction. Once the voucher certifies a transaction, it authorizes record- ing an obligation. A voucher also contains approval for paying the obligation on an appropriate date.

Point: Recording a purchase is initiated by an invoice approval, not an invoice. An invoice ap- proval verifies that the amount is consistent with that requested, ordered, and received. This con- trols and verifies purchases and related liabilities.

Point: Auditors, when auditing inventory, check a sampling of purchases by reviewing the purchase order, receiving report, and invoice.

254 Chapter 6 Cash, Fraud, and Internal Control

Completion of a voucher usually requires a person to enter certain information on both the inside and outside of the voucher. Typical information required on the inside of a voucher is on the left-hand side of Exhibit 6A.4, and that for the outside is on the right-hand side. This information is taken from the invoice and the supporting documents filed in the voucher. A complete voucher is sent to an authorized individual (often called an auditor). This person performs a final review, approves the accounts and amounts for debiting (called the accounting distribution), and authorizes recording of the voucher.

Chicago, Illinois

Date

For the following: (attach all invoices and supporting documents)

Oct. 28, 2019 Trex

Voucher No. 4657

Pay to AntigoCity WisconsinState

TERMS TERMS

Nov. 2, 2019 2/15, n/30 Invoice No. 4657 Less discount

Net amount payable

500 10 490

Payment approved

Auditor

Z-Mart

DATE OF INVOICE INVOICE NUMBER AND OTHER DETAILS

Inside Voucher No. 4657

November 12, 2019Due Date TrexPay to

AntigoCity WisconsinState

500 Summary of charges: Total charges Discount Net payment

10 490

Record of payment: Paid Check No.

Accounting Distribution ACCOUNT DEBITED

Merch. Inventory Store Supplies O�ce Supplies Sales Salaries Other

Total Vouch. Pay. Cr.

500

500

AMOUNT

Outside

EXHIBIT 6A.4 A Voucher

After a voucher is approved and recorded (in a journal called a voucher register), it is filed by its due date. A check is then sent on the payment date from the cashier, the voucher is marked “paid,” and the voucher is sent to the accounting department and recorded (in a journal called the check register). The person issuing checks relies on the approved voucher and its signed supporting documents as proof that an obligation has been incurred and must be paid. The purchase requisition and purchase order confirm the purchase was authorized. The receiving report shows that items have been received, and the invoice ap- proval form verifies that the invoice has been checked for errors. There is little chance for error and even less chance for fraud without collusion unless all the documents and signatures are forged.

FRAUD AND INTERNAL CONTROL Principles of Internal Control Establish responsibilities: Responsibility for a task should be assigned to one person. If responsibility is not established, determining who is at fault is difficult. Maintain adequate records: Good recordkeeping helps protect assets and helps managers monitor company activities. Insure assets and bond key employees: Assets should be insured, and employees handling cash and easily transferable assets should be bonded. Separate recordkeeping from custody of assets: An employee who has access to an asset must not have access to that asset’s accounting records. Divide responsibility for related transactions: Responsibility for a trans- action should be divided between two or more individuals or departments. One person’s work is a check on the others to prevent errors. This is not duplication of work. Apply technological controls: Use technology such as ID scanners to pro- tect assets and improve control. Perform regular and independent reviews: Regular reviews of internal controls should be performed by outside reviewers, preferably auditors.

CONTROL OF CASH Cash account: Includes currency, coins, checks, and deposits in bank accounts. Cash equivalents: Short-term, liquid investment assets meeting two crite- ria: (1) convertible to a known cash amount and (2) close to their due date, usually within 3 months. An example is a U.S. Treasury bill. Cash management strategies: (a) Encourage early collection of receiv- ables, (b) delay payment of liabilities, (c) keep only necessary assets, (d) plan expenditures, and (e) invest excess cash.

Summary: Cheat Sheet

Over-the-Counter Cash Receipt Control Procedures ∙ Sales are recorded on a cash register after each sale, and customers are

given a receipt. ∙ Cash registers hold a locked-in record of each transaction and often are

linked with the accounting system. ∙ Custody over cash is separate from recordkeeping. The clerk who has

access to cash in the register cannot access accounting records. The recordkeeper cannot access the cash.

Cash Over and Short Journal Entries

Cash Receipts by Mail Control Procedures ∙ Two people are tasked with opening mail. Theft of cash would require

collusion between these two employees. ∙ A list (in triplicate) is kept of each sender’s name, the amount, and an

explanation of why money was sent. The first copy is sent with the money to the cashier. A second copy is sent to the recordkeeper. The employees who opened the mail keep the third copy. The cashier depos- its the money in a bank, and the recordkeeper records amounts received.

∙ No employee has access to both accounting records and cash.

Cash . . . . . . . . . . . . . . . . . . . . . . . . . . . . . . . . . . . . . . . . . 555

Cash Over and Short . . . . . . . . . . . . . . . . . . . . . . . 5 Sales . . . . . . . . . . . . . . . . . . . . . . . . . . . . . . . . . . . . . 550

If cash received is more than recorded cash sales:

Cash . . . . . . . . . . . . . . . . . . . . . . . . . . . . . . . . . . . . . . . . . 621

Cash Over and Short . . . . . . . . . . . . . . . . . . . . . . . . . . . 4 Sales . . . . . . . . . . . . . . . . . . . . . . . . . . . . . . . . . . . . . 625

If cash received is less than recorded cash sales:

Chapter 6 Cash, Fraud, and Internal Control 255

Cash Payment Control Procedures ∙ Require all payments to be made by check or EFT. The only exception is

small payments made from petty cash. ∙ Deny access to records to employees who can sign checks (other than the

owner). Voucher system: Set of procedures to control cash payments. Applied to all payments.

TOOLS OF CONTROL AND ANALYSIS Petty cash: System of control used for small payments.

Canceled checks: Checks the bank has paid and deducted from the cus- tomer’s account. Bank reconciliation adjustments:

Entry to set up a petty cash fund:

Petty Cash . . . . . . . . . . . . . . . . . . . . . . . . . . . . . . . . . . . . . 75

Cash . . . . . . . . . . . . . . . . . . . . . . . . . . . . . . . . . . . . . 75

Increasing a petty cash fund (after reimbursement):

Petty Cash . . . . . . . . . . . . . . . . . . . . . . . . . . . . . . . . . . . . . 25

Cash . . . . . . . . . . . . . . . . . . . . . . . . . . . . . . . . . . . . . 25

Decreasing a petty cash fund (after reimbursement):

Cash . . . . . . . . . . . . . . . . . . . . . . . . . . . . . . . . . . . . . . . . . 20

Petty Cash . . . . . . . . . . . . . . . . . . . . . . . . . . . . . . . . 20

Adjusting Entries from Bank Reconciliation—Examples Collection of note:

Cash . . . . . . . . . . . . . . . . . . . . . . . . . . . . . . . . . . . . . . . . . 485

Notes Receivable . . . . . . . . . . . . . . . . . . . . . . . . . . . 485

Interest earned:

Cash . . . . . . . . . . . . . . . . . . . . . . . . . . . . . . . . . . . . . . . . . 8

Interest Revenue . . . . . . . . . . . . . . . . . . . . . . . . . . . 8

Bank fees:

Miscellaneous Expenses . . . . . . . . . . . . . . . . . . . . . . . . . 23

Cash . . . . . . . . . . . . . . . . . . . . . . . . . . . . . . . . . . . . . 23

NSF checks:

Accounts receivable–Name . . . . . . . . . . . . . . . . . . . . . . . 30

Cash . . . . . . . . . . . . . . . . . . . . . . . . . . . . . . . . . . . . . 30

Petty cash fund has unexplained shortage:

Miscellaneous Expenses . . . . . . . . . . . . . . . . . . . . . . . . . 178

Cash Over and Short . . . . . . . . . . . . . . . . . . . . . . . . . . . 7 Cash . . . . . . . . . . . . . . . . . . . . . . . . . . . . . . . . . . . . . 185

Reimburse and record expenses for petty cash:

Miscellaneous Expenses . . . . . . . . . . . . . . . . . . . . . . . . . 46 .50

Merchandise Inventory . . . . . . . . . . . . . . . . . . . . . . . . . . . 15 .05

Delivery Expense . . . . . . . . . . . . . . . . . . . . . . . . . . . . . . . 5 .00

Office Supplies Expense . . . . . . . . . . . . . . . . . . . . . . . . . . 4 .75

Cash . . . . . . . . . . . . . . . . . . . . . . . . . . . . . . . . . . . . . 71 .30

Bank Balance Adjustments

Add deposits in transit . Subtract outstanding checks . Add or subtract corrections of bank errors .

Book Balance Adjustments

Add interest earned and unrecorded cash receipts . Subtract bank fees and NSF checks . Add or subtract corrections of book errors .

Bank reconciliation (247) Bank statement (246) Canceled checks (246) Cash (239) Cash equivalents (239) Cash Over and Short (240) Check (245) Check register (254) Committee of Sponsoring

Organizations (COSO) (235) Credit memorandum (247) Days’ sales uncollected (250)

Debit memorandum (247) Deposit ticket (245) Deposits in transit (247) Electronic funds transfer (EFT) (245) Internal control system (235) Invoice (252) Invoice approval (253) Liquid assets (238) Liquidity (238) Outstanding checks (247) Petty cash (242) Principles of internal control (236)

Purchase order (252) Purchase requisition (252) Receiving report (253) Sarbanes-Oxley Act (SOX) (235) Signature card (245) Vendee (252) Vendor (252) Voucher (241) Voucher register (254) Voucher system (241)

Key Terms

Multiple Choice Quiz

1. The following information is available for Hapley Co. ∙ November 30 bank statement shows a $1,895 balance. ∙ The general ledger shows a $1,742 balance at November 30. ∙ A $795 deposit placed in the bank’s night depository on

November 30 does not appear on the November 30 bank statement.

∙ Outstanding checks amount to $638 at November 30. ∙ A customer’s $320 note was collected by the bank and

deposited in Hapley’s account in November.

∙ A bank service charge of $10 is deducted by the bank and appears on the November 30 bank statement.

How will the customer’s note appear on Hapley’s November 30 bank reconciliation? a. $320 appears as an addition to the book balance of cash. b. $320 appears as a deduction from the book balance of cash. c. $320 appears as an addition to the bank balance of cash. d. $320 appears as a deduction from the bank balance of cash. e. $335 appears as an addition to the bank balance of cash.

256 Chapter 6 Cash, Fraud, and Internal Control

2. Using the information from question 1, what is the reconciled balance on Hapley’s November 30 bank reconciliation? a. $2,052 c. $1,742 e. $1,184 b. $1,895 d. $2,201

3. A company needs to replenish its $500 petty cash fund. Its petty cashbox has $75 cash and petty cash receipts of $420. The journal entry to replenish the fund includes a. A debit to Cash for $75. b. A credit to Cash for $75.

c. A credit to Petty Cash for $420. d. A credit to Cash Over and Short for $5. e. A debit to Cash Over and Short for $5.

4. A company had net sales of $84,000 and accounts receiv- able of $6,720. Its days’ sales uncollected is a. 3.2 days. c. 230.0 days. e. 12.5 days. b. 18.4 days. d. 29.2 days.

ANSWERS TO MULTIPLE CHOICE QUIZ

1. a; recognizes cash collection of note by bank. 2. a; the bank reconciliation follows.

3. e; The entry follows.

Bank Reconciliation November 30

Balance per bank statement . . . . $1,895

Add: Deposit in transit . . . . . . . . . 795

Deduct: Outstanding checks . . . . (638)

Reconciled balance . . . . . . . . . . . $2,052

Balance per books . . . . . . . . $1,742

Add: Note collected . . . . . . . 320

Deduct: Service charge . . . . (10)

Reconciled balance . . . . . . . $2,052

Debits to expenses (or assets) . . . . . . . . . . 420

Cash Over and Short . . . . . . . . . . . . . . . . . . 5

Cash . . . . . . . . . . . . . . . . . . . . . . . . . . 425

4. d; ($6,720∕$84,000) × 365 = 29.2 days

A Superscript letter A denotes assignments based on Appendix 6A.

Icon denotes assignments that involve decision making.

1. List the seven broad principles of internal control. 2. Internal control procedures are important in every busi-

ness, but at what stage in the development of a business do they become especially critical?

3. Why should responsibility for related transactions be divided among different departments or individuals?

4. Why should the person who keeps the records of an as- set not be the person responsible for its custody?

5. When a store purchases merchandise, why are individ- ual departments not allowed to directly deal with suppliers?

6. What are the limitations of internal controls? 7. Which of the following assets—inventory, building, accounts

receivable, or cash—is most liquid? Which is least liquid? 8. What is a petty cash receipt? Who should sign it? 9. Why should cash receipts be deposited on the day of receipt? 10. Apple’s statement of cash flows in Appendix A

describes changes in cash and cash equivalents for the year ended September 30, 2017. What total amount

is provided (used) by investing activities? What amount is provided (used) by financing activities?

11. Refer to Google’s financial statements in Appendix A. Identify Google’s net earn- ings (income) for the year ended December 31, 2017. Is its net earnings equal to the change in cash and cash equiva- lents for the year? Explain the difference between net earn- ings and the change in cash and cash equivalents.

12. Refer to Samsung’s balance sheet in Appendix A. How does its cash (titled “Cash and cash equivalents”) compare with its other cur- rent assets (in both amount and percent) as of December 31, 2017? Compare and assess its cash at December 31, 2017, with its cash at December 31, 2016.

13. Samsung’s statement of cash flows in Appendix A reports the change in cash and equivalents for the year ended December 31, 2017. Identify the cash generated (or used) by operating activities, by investing activities, and by financing activities.

Discussion Questions

APPLE

Samsung

Samsung

GOOGLE

QUICK STUDY

QS 6-1 Internal control objectives

C1

Indicate which statements are true and which are false. 1. Separation of recordkeeping for assets from the custody over assets helps reduce fraud. 2. The primary objective of internal control procedures is to safeguard the business against theft

from government agencies. 3. Internal control procedures should be designed to protect assets from waste and theft. 4. Separating the responsibility for a transaction between two or more individuals or departments

will not help prevent someone from creating a fictitious invoice and paying the money to himself.

Chapter 6 Cash, Fraud, and Internal Control 257

Choose from the following list of terms and phrases to best complete the following statements. a. Cash c. Outstanding check e. Cash over and short b. Cash equivalents d. Liquidity f. Voucher system

1. The category includes currency, coins, and deposits in bank accounts. 2. The term refers to a company’s ability to pay for its current liabilities. 3. The category includes short-term, highly liquid investment assets that are readily con-

vertible to a known cash amount and sufficiently close to their due dates so that their market value will not greatly change.

QS 6-3 Cash and equivalents

C2

1. Brooks Agency set up a petty cash fund for $150. At the end of the current period, the fund contained $28 and had the following receipts: entertainment, $70; postage, $30; and printing, $22. Prepare jour- nal entries to record (a) establishment of the fund and (b) reimbursement of the fund at the end of the current period.

2. Identify the two events from the following that cause a Petty Cash account to be credited in a journal entry.

a. Fund amount is being reduced. c. Fund is being eliminated. b. Fund amount is being increased. d. Fund is being established.

QS 6-6 Petty cash accounting

P2

Nolan Company’s Cash account shows a $22,352 debit balance and its bank statement shows $21,332 on deposit at the close of business on June 30. Prepare a bank reconciliation using the following information. a. Outstanding checks as of June 30 total $3,713. b. The June 30 bank statement lists $41 in bank service charges; the company has not yet recorded the

cost of these services.

QS 6-8 Bank reconciliation

P3

Record the journal entry for Sales and for Cash Over and Short for each of the following separate situations. a. The cash register’s record shows $420 of cash sales, but the count of cash in the register is $430. b. The cash register’s record shows $980 of cash sales, but the count of cash in the register is $972.

QS 6-5 Cash Over and Short

P1

Identify each of the following statements as either true or false. a. A guideline for safeguarding cash is that all cash receipts be deposited monthly or yearly. b. A voucher system of control is a control system exclusively for cash receipts. c. A guideline for safeguarding cash is to separate the duties of those who have custody of cash from

those who keep cash records. d. Separation of duties eliminates the possibility of collusion to steal an asset and hide the theft from

the records.

QS 6-4 Internal control for cash

P1

COSO lists five components of internal control: control environment, risk assessment, control activities, information and communication, and monitoring. Indicate the COSO component that matches with each of the following internal control activities.

a. Independent review of controls c. Reporting of control effectiveness b. Executives’ strong ethics d. Analyses of fraud risk factors

QS 6-2 COSO internal control components

C1

For a through g, indicate whether its amount (1) affects the bank or book side of a bank reconciliation, (2) is an addition or a subtraction in a bank reconciliation, and (3) requires an adjusting journal entry.

QS 6-7 Bank reconciliation

P3 Bank or Book Side Add or Subtract Adj. Entry or Not

a . Interest on cash balance . . . . . . . . . . . . . . . . .

b . Bank service charges . . . . . . . . . . . . . . . . . . .

c . Minimum balance bank fee . . . . . . . . . . . . . .

d . Outstanding checks . . . . . . . . . . . . . . . . . . . .

e . Collection of note by bank . . . . . . . . . . . . . . .

f . NSF checks . . . . . . . . . . . . . . . . . . . . . . . . . . .

g . Outstanding deposits . . . . . . . . . . . . . . . . . . .

[continued on next page]

258 Chapter 6 Cash, Fraud, and Internal Control

c. In reviewing the bank statement, a $90 check written by the company was mistakenly recorded in the company’s books as $99.

d. June 30 cash receipts of $4,724 were placed in the bank’s night depository after banking hours and were not recorded on the June 30 bank statement.

e. The bank statement included a $23 credit for interest earned on the company’s cash in the bank. The company has not yet recorded interest earned.

Organic Food Co.’s Cash account shows a $5,500 debit balance and its bank statement shows $5,160 on de- posit at the close of business on August 31. Prepare a bank reconciliation using the following information. a. August 31 cash receipts of $1,240 were placed in the bank’s night depository after banking hours and

were not recorded on the August 31 bank statement. b. The bank statement shows a $120 NSF check from a customer; the company has not yet recorded this

NSF check. c. Outstanding checks as of August 31 total $1,120. d. In reviewing the bank statement, an $80 check written by Organic Fruits was mistakenly drawn against

Organic Food’s account. e. The August 31 bank statement lists $20 in bank service charges; the company has not yet recorded the

cost of these services.

QS 6-9 Bank reconciliation

P3

Management uses a voucher system to help control and monitor cash payments. Which one or more of the four documents listed below are prepared as part of a voucher system of control?

a. Purchase order b. Outstanding check c. Invoice d. Voucher

QS 6-11A Documents in a voucher system P4

EXERCISES

Exercise 6-1 Analyzing internal control

C1

Identify the internal control principle that was violated in each of the following separate situations. a. The recordkeeper left town after the owner discovered a large sum of money had disappeared. An au-

dit found that the recordkeeper had written and signed several checks made payable to his fiancée and recorded the checks as salaries expense.

b. An employee was put in charge of handling cash. That employee later stole cash from the business. The company incurred an uninsured loss of $184,000.

c. There is $500 in cash missing from a cash register drawer. Three salesclerks shared the cash register drawer, so the owner cannot determine who is at fault.

Whole Fruits Market took the following actions to improve internal controls. For each of the following actions, identify the internal control principle the company followed. a. Prohibit the recordkeeper from having control over cash. b. Purchased an insurance (bonding) policy against losses from theft by a cashier. c. Each cashier is designated a specific cash drawer and is solely responsible for cash in that drawer. d. Detailed records of inventory are kept to ensure items lost or stolen do not go unnoticed. e. Digital time clocks are used to register which employees are at work at what times. f. External auditors are regularly hired to evaluate internal controls.

Exercise 6-2 Applying internal control principles

C1

Year 2 Year 1

Accounts receivable . . . . . . . . . . . . $  100,000 $   85,000

Net sales . . . . . . . . . . . . . . . . . . . . . 2,500,000 2,000,000

The following annual account balances are from Armour Sports at December 31.QS 6-10 Days’ sales uncollected

A1

a. What is the change in the number of days’ sales uncollected between Year 1 and Year 2? (Round the number of days to one decimal.)

b. From the analysis in part a, is the company’s collection of receivables improving?

Chapter 6 Cash, Fraud, and Internal Control 259

Determine whether each procedure described below is an internal control strength or weakness; then iden- tify the internal control principle violated or followed for each procedure. 1. The same employee requests, records, and makes payment for purchases of inventory. 2. The company saves money by having employees involved in operations perform the only review of

internal controls. 3. Time is saved by not updating records for use of supplies. 4. The recordkeeper is not allowed to write checks or initiate EFTs. 5. Each salesclerk is in charge of her own cash drawer.

Exercise 6-3 Internal control strengths and weaknesses

C1

Determine whether each policy below is good or bad cash management; then identify the cash manage- ment strategy violated or followed for each policy. 1. Bills are paid as soon as they are received. 2. Cash receipts and cash payments are regularly planned and reviewed. 3. Excess cash is put in checking accounts, earning no interest income. 4. Customers are regularly allowed to pay after due dates without concern. 5. Rarely used equipment is rented rather than purchased.

Exercise 6-4 Cash management strategies

C2

Determine whether each cash receipts procedure is an internal control strength or weakness. 1. If a salesclerk makes an error in recording a cash sale, she can access the register’s electronic record to

correct the transaction. 2. All sales transactions, even those for less than $1, are recorded on a cash register. 3. Two employees are tasked with opening mail that contains cash receipts. 4. One of the two employees tasked with opening mail is also the recordkeeper for the business. 5. The supervisor has access to both cash and the accounting records. 6. Receipts are given to customers only for sales that are above $20.

Exercise 6-6 Control of cash receipts

P1

Determine whether each cash payment procedure is an internal control strength or weakness. 1. A voucher system is used for all payments of liabilities. 2. The owner of a small business has authority to write and sign checks. 3. When the owner is out of town, the recordkeeper is in charge of signing checks. 4. To save time, all departments are allowed to incur liabilities. 5. Payments over $100 are made by check. 6. Requesting and receiving merchandise are handled by the same department.

Exercise 6-7 Voucher system and control of cash payments

P1

Specter Co. combines cash and cash equivalents on the balance sheet. Using the following information, determine the amount reported on the year-end balance sheet for cash and cash equivalents. ∙ $3,000 cash deposit in checking account. ∙ $200, 3-year loan to an employee. ∙ $20,000 bond investment due in 20 years. ∙ $1,000 of currency and coins. ∙ $5,000 U.S. Treasury bill due in 1 month. ∙ $500 of accounts receivable.

Exercise 6-5 Cash and cash equivalents

C2

Waupaca Company establishes a $350 petty cash fund on September 9. On September 30, the fund shows $104 in cash along with receipts for the following expenditures: transportation-in, $40; postage expenses, $123; and miscellaneous expenses, $80. The petty cashier could not account for a $3 shortage in the fund. The company uses the perpetual system in accounting for merchandise inventory. Prepare (1) the September 9 entry to establish the fund, (2) the September 30 entry to reimburse the fund, and (3) an October 1 entry to increase the fund to $400.

Exercise 6-8 Petty cash fund with a shortage P2

Check (2) Cr. Cash, $246 and (3) Cr. Cash, $50

EcoMart establishes a $1,050 petty cash fund on May 2. On May 30, the fund shows $326 in cash along with receipts for the following expenditures: transportation-in, $120; postage expenses, $369; and miscel- laneous expenses, $240. The petty cashier could not account for a $5 overage in the fund. The company uses the perpetual system in accounting for merchandise inventory. Prepare the (1) May 2 entry to establish the fund, (2) May 30 entry to reimburse the fund [Hint: Credit Cash Over and Short for $5 and credit Cash for $724], and (3) June 1 entry to increase the fund to $1,200.

Exercise 6-9 Petty cash fund with an overage

P2

260 Chapter 6 Cash, Fraud, and Internal Control

Palmona Co. establishes a $200 petty cash fund on January 1. On January 8, the fund shows $38 in cash along with receipts for the following expenditures: postage, $74; transportation-in, $29; delivery expenses, $16; and miscellaneous expenses, $43. Palmona uses the perpetual system in accounting for merchandise inventory. Prepare journal entries to (1) establish the fund on January 1, (2) reimburse it on January 8, and (3) both reimburse the fund and increase it to $450 on January 8, assuming no entry in part 2. Hint: Make two separate entries for part 3.

Exercise 6-10 Petty cash fund accounting

P2

Check (3) Cr. Cash, $162 & $250

Indicate whether each item should be added to or deducted from the book or bank balance and whether it should or should not appear on the September 30 reconciliation. For items that add or deduct from the book balance column, place a Dr. or Cr. after the “Add” or “Deduct” to show the accounting impact on Cash. 1. NSF check from a customer is shown on the bank statement but not yet recorded by the company. 2. Interest earned on the September cash balance in the bank is not yet recorded by the company. 3. Deposit made on September 5 and processed by the bank on September 6. 4. Checks written by another depositor but mistakenly charged against this company’s account. 5. Bank service charge for September is not yet recorded by the company. 6. Checks outstanding on August 31 that cleared the bank in September. 7. Check written against the company’s account and cleared by the bank; erroneously not recorded by

the company’s recordkeeper. 8. A note receivable is collected by the bank for the company, but it is not yet recorded by the company. 9. Checks written and mailed to payees on October 2. 10. Checks written by the company and mailed to payees on September 30. 11. Night deposit made on September 30 after the bank closed. 12. Bank fees for check printing are not yet recorded by the company.

Prepare a table with the following headings for a monthly bank reconciliation dated September 30.Exercise 6-11 Bank reconciliation and adjusting entries

P3 Shown or Not Shown Item Bank Balance Book Balance on Reconciliation

Add or Subtract Add or Subtract Dr . or Cr . Shown or Not Shown

Del Gato Clinic’s Cash account shows an $11,589 debit balance and its bank statement shows $10,555 on deposit at the close of business on June 30. Prepare its bank reconciliation using the following information. a. Outstanding checks as of June 30 total $1,829. b. The June 30 bank statement lists a $16 bank service charge. c. Check No. 919, listed with the canceled checks, was correctly drawn for $467 in payment of a utility

bill on June 15. Del Gato Clinic mistakenly recorded it with a debit to Utilities Expense and a credit to Cash in the amount of $476.

d. The June 30 cash receipts of $2,856 were placed in the bank’s night depository after banking hours and were not recorded on the June 30 bank statement.

Exercise 6-12 Bank reconciliation

P3

Check Reconciled bal., $11,582

Prepare the adjusting journal entries that Del Gato Clinic must record as a result of preparing the bank reconciliation in Exercise 6-12.

Exercise 6-13 Adjusting entries from bank reconciliation P3

Wright Company’s Cash account shows a $27,500 debit balance and its bank statement shows $25,800 on deposit at the close of business on May 31. Prepare its bank reconciliation using the following information. a. The May 31 bank statement lists $100 in bank service charges; the company has not yet recorded the

cost of these services. b. Outstanding checks as of May 31 total $5,600. c. May 31 cash receipts of $6,200 were placed in the bank’s night depository after banking hours and

were not recorded on the May 31 bank statement. d. In reviewing the bank statement, a $400 check written by Smith Company was mistakenly drawn

against Wright’s account. e. The bank statement shows a $600 NSF check from a customer; the company has not yet recorded this

NSF check.

Exercise 6-14 Bank reconciliation

P3

Check Reconciled bal., $26,800

Chapter 6 Cash, Fraud, and Internal Control 261

Barga Co.’s net sales for Year 1 and Year 2 are $730,000 and $1,095,000, respectively. Its year-end bal- ances of accounts receivable follow: Year 1, $65,000; and Year 2, $123,000. a. Compute its days’ sales uncollected at the end of each year. Round the number of days to one decimal. b. Did days’ sales uncollected improve or worsen in Year 2 versus Year 1?

Exercise 6-15 Liquid assets and accounts receivable A1

Match each document in a voucher system with its description. Document 1. Purchase requisition 2. Purchase order 3. Invoice 4. Receiving report 5. Invoice approval 6. Voucher

Exercise 6-16A Documents in a voucher system

P4

Description A. An itemized statement of goods prepared by the vendor listing the cus-

tomer’s name, items sold, sales prices, and terms of sale. B. An internal file used to store documents and information to control

cash payments and to ensure that a transaction is properly authorized and recorded.

C. A document used to place an order with a vendor that authorizes the vendor to ship ordered merchandise at the stated price and terms.

D. A checklist of steps necessary for the approval of an invoice for record- ing and payment; also known as a check authorization.

E. A document used by department managers to inform the purchasing department to place an order with a vendor.

F. A document used to notify the appropriate persons that ordered goods have arrived, including a description of the quantities and condition of goods.

PROBLEM SET A

Problem 6-1A Analyzing internal control

C1

Following are five separate cases involving internal control issues. a. Chi Han receives all incoming customer cash receipts for her employer and posts the customer pay-

ments to their respective accounts. b. At Tico Company, Julia and Trevor alternate lunch hours. Julia is the petty cash custodian, but if some-

one needs petty cash when she is at lunch, Trevor fills in as custodian. c. Nori Nozumi posts all patient charges and payments at the Hopeville Medical Clinic. Each night Nori

backs up the computerized accounting system but does not password lock her computer. d. Ben Shales prides himself on hiring quality workers who require little supervision. As office manager,

Ben gives his employees full discretion over their tasks and for years has seen no reason to perform independent reviews of their work.

e. Carla Farah’s manager has told her to reduce costs. Carla decides to raise the deductible on the plant’s property insurance from $5,000 to $10,000. This cuts the property insurance premium in half. In a related move, she decides that bonding the plant’s employees is a waste of money because the company has not experienced any losses due to employee theft. Carla saves the entire amount of the bonding insurance premium by dropping the bonding insurance.

Required

1. For each case, identify the principle(s) of internal control that is violated. 2. Recommend what should be done to adhere to principles of internal control in each case.

Kiona Co. set up a petty cash fund for payments of small amounts. The following transactions involving the petty cash fund occurred in May (the last month of the company’s fiscal year).

May 1 Prepared a company check for $300 to establish the petty cash fund. 15 Prepared a company check to replenish the fund for the following expenditures made since May 1. a. Paid $88 for janitorial expenses. b. Paid $53.68 for miscellaneous expenses. c. Paid postage expenses of $53.50. d. Paid $47.15 to Facebook for advertising expense. e. Counted $62.15 remaining in the petty cashbox. 16 Prepared a company check for $200 to increase the fund to $500.

Problem 6-2A Establishing, reimbursing, and adjusting petty cash

P2

[continued on next page]

262 Chapter 6 Cash, Fraud, and Internal Control

31 The petty cashier reports that $288.20 cash remains in the fund. A company check is drawn to replenish the fund for the following expenditures made since May 15.

f. Paid postage expenses of $147.36. g. Reimbursed the office manager for mileage expense, $23.50. h. Paid $34.75 in delivery expense for products to a customer, terms FOB destination. 31 The company decides that the May 16 increase in the fund was too large. It reduces the fund by

$100, leaving a total of $400.

Required

Prepare journal entries to establish the fund on May 1, to replenish it on May 15 and on May 31, and to reflect any increase or decrease in the fund balance on May 16 and May 31.

Check Cr. to Cash: May 15, $237.85; May 16, $200.00

Problem 6-3A Establishing, reimbursing, and increasing petty cash

P2

Nakashima Gallery had the following petty cash transactions in February of the current year. Nakashima uses the perpetual system to account for merchandise inventory.

Feb. 2 Wrote a $400 check to establish a petty cash fund. 5 Purchased paper for the copier for $14.15 that is immediately used. 9 Paid $32.50 shipping charges (transportation-in) on merchandise purchased for resale, terms

FOB shipping point. These costs are added to merchandise inventory. 12 Paid $7.95 postage to deliver a contract to a client. 14 Reimbursed Adina Sharon, the manager, $68 for mileage on her car. 20 Purchased office paper for $67.77 that is immediately used. 23 Paid a courier $20 to deliver merchandise sold to a customer, terms FOB destination. 25 Paid $13.10 shipping charges (transportation-in) on merchandise purchased for resale, terms

FOB shipping point. These costs are added to merchandise inventory. 27 Paid $54 for postage expenses. 28 The fund had $120.42 remaining in the petty cashbox. Sorted the petty cash receipts by ac-

counts affected and exchanged them for a check to reimburse the fund for expenditures. 28 The petty cash fund amount is increased by $100 to a total of $500.

Required

1. Prepare the journal entry to establish the petty cash fund. 2. Prepare a petty cash payments report for February with these categories: delivery expense, mileage

expense, postage expense, merchandise inventory (for transportation-in), and office supplies expense. 3. Prepare the journal entries for part 2 to both (a) reimburse and (b) increase the fund amount.

Check Cash credit: (3a) $279.58; (3b) $100.00

Check (1) Reconciled balance, $34,602; (2) Cr. Notes Receivable, $8,000

The following information is available to reconcile Branch Company’s book balance of cash with its bank statement cash balance as of July 31. a. On July 31, the company’s Cash account has a $27,497 debit balance, but its July bank statement

shows a $27,233 cash balance. b. Check No. 3031 for $1,482, Check No. 3065 for $382, and Check No. 3069 for $2,281 are outstanding

checks as of July 31. c. Check No. 3056 for July rent expense was correctly written and drawn for $1,270 but was erroneously

entered in the accounting records as $1,250. d. The July bank statement shows the bank collected $7,955 cash on a note for Branch. Branch had not

recorded this event before receiving the statement. e. The bank statement shows an $805 NSF check. The check had been received from a customer, Evan

Shaw. Branch has not yet recorded this check as NSF. f. The July statement shows a $25 bank service charge. It has not yet been recorded in miscellaneous

expenses because no previous notification had been received. g. Branch’s July 31 daily cash receipts of $11,514 were placed in the bank’s night depository on that date

but do not appear on the July 31 bank statement.

Required

1. Prepare the bank reconciliation for this company as of July 31. 2. Prepare the journal entries necessary to make the company’s book balance of cash equal to the recon-

ciled cash balance as of July 31.

Problem 6-4A Preparing a bank reconciliation and recording adjustments

P3

Chapter 6 Cash, Fraud, and Internal Control 263

Chavez Company most recently reconciled its bank statement and book balances of cash on August 31 and it reported two checks outstanding, No. 5888 for $1,028 and No. 5893 for $494. Check No. 5893 was still outstanding as of September 30. The following information is available for its September 30 reconciliation.

Problem 6-5A Preparing a bank reconciliation and recording adjustments

P3From the September 30 Bank Statement

16,800 9,617 11,270 18,453

PREVIOUS BALANCE TOTAL CHECKS AND DEBITS TOTAL DEPOSITS AND CREDITS CURRENT BALANCE

Date Sep. 3 Sep. 4 Sep. 7 Sep. 17 Sep. 20 Sep. 22 Sep. 22 Sep. 28 Sep. 29

CHECKS AND DEBITS DEPOSITS AND CREDITS

Sep. 5 Sep. 12 Sep. 21 Sep. 25 Sep. 30 Sep. 30

5888 1,103 No. Amount Date Amount

5902 2,226 5901 4,093

5905 12 IN 2,351

5903 1,485 CM 5904 5907 5909

1,028 719

1,824

937 399

2,090 213

1,807

600 NSF

From Chavez Company’s Accounting Records

Cash Receipts Deposited

Date Cash Debit

Sep . 5 1,103

12 2,226

21 4,093

25 2,351

30 1,682

11,455 Cash Acct. No. 101

Date Explanation PR Debit Credit Balance

Aug . 31 Balance 15,278

Sep . 30 Total receipts R12 11,455 26,733

30 Total payments D23 9,329 17,404

Cash Payments

Check No. Cash Credit

5901 1,824

5902 719

5903 399

5904 2,060

5905 937

5906 982

5907 213

5908 388

5909 1,807

9,329

Additional Information (a) Check No. 5904 is correctly drawn for $2,090 to pay for computer equip- ment; however, the recordkeeper misread the amount and entered it in the accounting records with a debit to Computer Equipment and a credit to Cash of $2,060. (b) The NSF check shown in the statement was originally received from a customer, S. Nilson, in payment of her account. Its return has not yet been recorded by the company. (c) The credit memorandum (CM) is from the collection of a $1,485 note for Chavez Company by the bank. The collection is not yet recorded.

Required

1. Prepare the September 30 bank reconciliation for this company. 2. Prepare journal entries to adjust the book balance of cash to the reconciled balance.

Check (1) Reconciled balance, $18,271; (2) Cr. Notes Receivable, $1,485

PROBLEM SET B

Problem 6-1B Analyzing internal control

C1

Following are five separate cases involving internal control issues. a. Tywin Company keeps very poor records of its equipment. Instead, the company asserts its employees

are honest and would never steal from the company. b. Marker Theater has a computerized order-taking system for its tickets. The system is backed up once a year. c. Sutton Company has two employees handling acquisitions of inventory. One employee places pur-

chase orders and pays vendors. The second employee receives the merchandise.

264 Chapter 6 Cash, Fraud, and Internal Control

d. The owner of Super Pharmacy uses a check software/printer to prepare checks, making it difficult for anyone to alter the amount of a check. The check software/printer, which is not password protected, is on the owner’s desk in an office that contains company checks and is normally unlocked.

e. To ensure the company retreat would not be cut, the manager of Lavina Company decided to save money by canceling the external audit of internal controls.

Required

1. For each case, identify the principle(s) of internal control that is violated. 2. Recommend what should be done to adhere to principles of internal control in each case.

Moya Co. establishes a petty cash fund for payments of small amounts. The following transactions involv- ing the petty cash fund occurred in January (the last month of the company’s fiscal year).

Jan. 3 A company check for $150 is written and made payable to the petty cashier to establish the petty cash fund.

14 A company check is written to replenish the fund for the following expenditures made since January 3.

a. Purchased office supplies for $14.29 that are immediately used. b. Paid $19.60 COD shipping charges on merchandise purchased for resale, terms FOB

shipping point. Moya uses the perpetual system to account for inventory. c. Paid $38.57 to All-Tech for repairs expense to a computer. d. Paid $12.82 for items classified as miscellaneous expenses. e. Counted $62.28 remaining in the petty cashbox. 15 Prepared a company check for $50 to increase the fund to $200. 31 The petty cashier reports that $17.35 remains in the fund. A company check is written to replen-

ish the fund for the following expenditures made since January 14. f. Paid $50 to The Smart Shopper in advertising expense for January’s newsletter. g. Paid $48.19 for postage expenses. h. Paid $78 to Smooth Delivery for delivery expense of merchandise, terms FOB destination. 31 The company decides that the January 15 increase in the fund was too little. It increases the

fund by another $50.

Required

Prepare journal entries (in dollars and cents) to establish the fund on January 3, to replenish it on January 14 and January 31, and to reflect any increase or decrease in the fund balance on January 15 and 31.

Problem 6-2B Establishing, reimbursing, and adjusting petty cash

P2

Check Cr. to Cash: Jan. 14, $87.72; Jan. 31 (total), $232.65

Blues Music Center had the following petty cash transactions in March of the current year. Blues uses the perpetual system to account for merchandise inventory.

Mar. 5 Wrote a $250 check to establish a petty cash fund. 6 Paid $12.50 shipping charges (transportation-in) on merchandise purchased for resale, terms

FOB shipping point. These costs are added to merchandise inventory. 11 Paid $10.75 in delivery expense on merchandise sold to a customer, terms FOB destination. 12 Purchased office file folders for $14.13 that are immediately used. 14 Reimbursed Bob Geldof, the manager, $11.65 for office supplies purchased and used. 18 Purchased office printer paper for $20.54 that is immediately used. 27 Paid $45.10 shipping charges (transportation-in) on merchandise purchased for resale, terms

FOB shipping point. These costs are added to merchandise inventory. 28 Paid postage expense of $18. 30 Reimbursed Geldof $56.80 for mileage expense. 31 Cash of $61.53 remained in the fund. Sorted the petty cash receipts by accounts affected and

exchanged them for a check to reimburse the fund for expenditures. 31 The petty cash fund amount is increased by $50 to a total of $300.

Required

1. Prepare the journal entry to establish the petty cash fund. 2. Prepare a petty cash payments report for March with these categories: delivery expense, mileage

expense, postage expense, merchandise inventory (for transportation-in), and office supplies expense. 3. Prepare the journal entries for part 2 to both (a) reimburse and (b) increase the fund amount.

Problem 6-3B Establishing, reimbursing, and increasing petty cash

P2

Check (2) Total expenses, $189.47 (3a & 3b) Total Cr. to Cash, $238.47

Chapter 6 Cash, Fraud, and Internal Control 265

The following information is available to reconcile Severino Co.’s book balance of cash with its bank statement cash balance as of December 31. a. The December 31 cash balance according to the accounting records is $32,878.30, and the bank state-

ment cash balance for that date is $46,822.40. b. Check No. 1242 for $410.40, Check No. 1273 for $4,589.30, and Check No. 1282 for $400 are out-

standing checks as of December 31. c. Check No. 1267 had been correctly drawn for $3,456 to pay for office supplies but was erroneously

entered in the accounting records as $3,465. d. The bank statement shows a $762.50 NSF check received from a customer, Titus Industries, in pay-

ment of its account. The statement also shows a $99 bank fee in miscellaneous expenses for check printing. Severino had not yet recorded these transactions.

e. The bank statement shows that the bank collected $18,980 cash on a note receivable for the company. Severino did not record this transaction before receiving the statement.

f. Severino’s December 31 daily cash receipts of $9,583.10 were placed in the bank’s night depository on that date but do not appear on the December 31 bank statement.

Required

1. Prepare the bank reconciliation for this company as of December 31. 2. Prepare the journal entries necessary to make the company’s book balance of cash equal to the recon-

ciled cash balance as of December 31.

Check (1) Reconciled balance, $51,005.80; (2) Cr. Notes Receivable, $18,980.00

Problem 6-4B Preparing a bank reconciliation and recording adjustments

P3

Shamara Systems most recently reconciled its bank balance on April 30 and reported two checks outstand- ing at that time, No. 1771 for $781 and No. 1780 for $1,425.90. Check No. 1780 was still outstanding as of May 31. The following information is available for its May 31 reconciliation.

Problem 6-5B Preparing a bank reconciliation and recording adjustments

P3 From the May 31 Bank Statement

PREVIOUS BALANCE TOTAL CHECKS AND DEBITS TOTAL DEPOSITS AND CREDITS CURRENT BALANCE

CHECKS AND DEBITS DEPOSITS AND CREDITS

18,290.70

Date May 1 May 2 May 4 May 11 May 18 May 25 May 26 May 29 May 31

May 4 May 14 May 22 May 25 May 26

1771 No. Amount Date Amount

13,094.80 16,566.80 21,762.70

1783 1782 1784

1787 1785 1788

2,438.00 2,898.00 1,801.80

2,079.00 7,350.00 CM

781.00 382.50

1,285.50

431.80 NSF 8,032.50

63.90 654.00

14.00 SC

1,449.60

From Shamara Systems’s Accounting Records

Cash Acct. No. 101

Date Explanation PR Debit Credit Balance

Apr . 30 Balance 16,083 .80

May 31 Total receipts R7 11,944 .10 28,027 .90

31 Total payments D8 12,850 .60 15,177 .30

Cash Receipts Deposited

Date Cash Debit

May 4 2,438 .00

14 2,898 .00

22 1,801 .80

26 2,079 .00

31 2,727 .30

11,944 .10

Cash Payments

Check No. Cash Credit

1782 1,285 .50

1783 382 .50

1784 1,449 .60

1785 63 .90

1786 353 .10

1787 8,032 .50

1788 644 .00

1789 639 .50

12,850 .60

[continued on next page]

266 Chapter 6 Cash, Fraud, and Internal Control

Additional Information (a) Check No. 1788 is correctly drawn for $654 to pay for May utilities; how- ever, the recordkeeper misread the amount and entered it in the accounting records with a debit to Utilities Expense and a credit to Cash for $644. The bank paid and deducted the correct amount. (b) The NSF check shown in the statement was originally received from a customer, W. Sox, in payment of her account. The company has not yet recorded its return. (c) The credit memorandum (CM) is from a $7,350 note that the bank collected for the company. The collection has not yet been recorded.

Required

1. Prepare the May 31 bank reconciliation for Shamara Systems. 2. Prepare journal entries to adjust the book balance of cash to the reconciled balance.

Check (1) Reconciled balance, $22,071.50; (2) Cr. Notes Receivable, $7,350.00

SERIAL PROBLEM Business Solutions

P3

This serial problem began in Chapter 1 and continues through most of the book. If previous chapter seg- ments were not completed, the serial problem can begin at this point.

SP 6 Santana Rey receives the March bank statement for Business Solutions on April 11, 2020. The March 31 bank statement shows an ending cash balance of $67,566. The general ledger Cash account, No. 101, shows an ending cash balance per books of $68,057 as of March 31 (prior to any reconciliation). A comparison of the bank statement with the general ledger Cash account, No. 101, reveals the following. a. The bank erroneously cleared a $500 check against the company account in March that S. Rey did not

issue. The check was actually issued by Business Systems. b. On March 25, the bank statement lists a $50 charge for a safety deposit box. Santana has not yet re-

corded this expense. c. On March 26, the bank statement lists a $102 charge for printed checks that Business Solutions or-

dered from the bank. Santana has not yet recorded this expense. d. On March 31, the bank statement lists $33 interest earned on Business Solutions’s checking account

for the month of March. Santana has not yet recorded this revenue. e. S. Rey notices that the check she issued for $128 on March 31, 2020, has not yet cleared the bank. f. S. Rey verifies that all deposits made in March do appear on the March bank statement.

Required

1. Prepare a bank reconciliation for Business Solutions for the month ended March 31, 2020. 2. Prepare any necessary adjusting entries. Use Miscellaneous Expenses, No. 677, for any bank charges.

Use Interest Revenue, No. 404, for any interest earned on the checking account for March.

Check (1) Adj. bank bal., $67,938

©Alexander Image/Shutterstock

The General Ledger tool in Connect automates several of the procedural steps in the accounting cycle so that the financial professional can focus on the impacts of each transaction on the various financial reports.

GL 6-1 General Ledger assignment GL 6-1, based on Problem 6-2A, focuses on transactions related to the petty cash fund and highlights the impact each transaction has on net income, if any. Prepare the jour- nal entries related to the petty cash fund and assess the impact of each transaction on the company’s net income, if any.

GENERAL LEDGER PROBLEM

GL

COMPANY ANALYSIS C2 A1

Accounting Analysis

AA 6-1 Use Apple’s financial statements in Appendix A to answer the following. 1. Identify the total amount of cash and cash equivalents for fiscal years ended (a) September 30, 2017,

and (b) September 24, 2016. 2. Compute cash and cash equivalents as a percent (rounded to one decimal) of total current assets, total

current liabilities, total shareholders’ equity, and total assets at fiscal year-end for both 2017 and 2016.APPLE

Chapter 6 Cash, Fraud, and Internal Control 267

ETHICS CHALLENGE C1

BTN 6-1 Harriet Knox, Ralph Patton, and Marcia Diamond work for a family physician, Dr. Gwen Conrad, who is in private practice. Dr. Conrad is knowledgeable about office management practices and has segregated the cash receipt duties as follows. Knox opens the mail and prepares a triplicate list of money received. She sends one copy of the list to Patton, the cashier, who deposits the receipts daily in the bank. Diamond, the recordkeeper, receives a copy of the list and posts payments to patients’ accounts. About once a month the office clerks have an expensive lunch they pay for as follows. First, Patton

Beyond the Numbers

3. Compute the percent change (rounded to one decimal) between the beginning and ending year amounts of cash and cash equivalents for fiscal years ended (a) September 30, 2017, and (b) September 24, 2016.

4. Compute the days’ sales uncollected (rounded to one decimal) as of (a) September 30, 2017, and (b) September 24, 2016.

5. Does Apple’s collection of receivables show a favorable or unfavorable change?

AA 6-3 Key figures for Samsung follow. GLOBAL ANALYSIS C2 A1

W millions Current Year Prior Year

Cash . . . . . . . . . . . . . . . . . . . . . . . . . W 30,545,130 W 32,111,442

Accounts receivable . . . . . . . . . . . . 31,804,956 27,800,408

Current assets . . . . . . . . . . . . . . . . . 146,982,464 141,429,704

Total assets . . . . . . . . . . . . . . . . . . . 301,752,090 262,174,324

Current liabilities . . . . . . . . . . . . . . . 67,175,114 54,704,095

Shareholders’ equity . . . . . . . . . . . 214,491,428 192,963,033

Net sales . . . . . . . . . . . . . . . . . . . . . 239,575,376 201,866,745

Required

1. Compute cash and cash equivalents as a percent (rounded to one decimal) of total current assets, total assets, total current liabilities, and total shareholders’ equity for both years.

2. Compute the percentage change (rounded to one decimal) between the current year and prior year cash balances.

3. Compute the days’ sales uncollected (rounded to one decimal) at the end of both the (a) current year and (b) prior year.

4. Does Samsung’s collection of receivables show a favorable or unfavorable change?

Samsung

Required

1. Compute days’ sales uncollected (rounded to one decimal) for (a) Apple and (b) Google for the current and prior years.

2. Which company had more success collecting receivables?

AA 6-2 Key comparative figures for Apple and Google follow. COMPARATIVE ANALYSIS A1

APPLE GOOGLE

Apple Google

$ millions Current Year Prior Year Current Year Prior Year

Accounts receivable . . . . . . . . . . . . $ 17,874 $ 15,754 $ 18,336 $14,137

Net sales . . . . . . . . . . . . . . . . . . . . . 229,234 215,639 110,855 90,272

268 Chapter 6 Cash, Fraud, and Internal Control

endorses a patient’s check in Dr. Conrad’s name and cashes it at the bank. Knox then destroys the remit- tance advice accompanying the check. Finally, Diamond posts payment to the customer’s account as a miscellaneous credit. The three justify their actions by their relatively low pay and knowledge that Dr. Conrad will likely never miss the money.

Required

1. Who is the best person in Dr. Conrad’s office to reconcile the bank statement? 2. Would a bank reconciliation uncover this office fraud? 3. What are some procedures to detect this type of fraud? 4. Suggest additional internal controls that Dr. Conrad could implement.

BTN 6-2 Assume you are a business consultant. The owner of a company sends you an e-mail expressing concern that the company is not taking advantage of its discounts offered by vendors. The company cur- rently uses the gross method of recording purchases. The owner is considering a review of all invoices and payments from the previous period. Due to the volume of purchases, however, the owner recognizes that this is time-consuming and costly. The owner seeks your advice about monitoring purchase discounts in the future. Provide a response in memorandum form. Hint: It will help to review the recording of purchase discounts in Appendix 4C.

COMMUNICATING IN PRACTICE P4

BTN 6-3 Visit the Association of Certified Fraud Examiners website and open the “2016 Report to the Nations” (s3-us-west-2.amazonaws.com/acfepublic/2016-report-to-the-nations.pdf). Read the two- page Executive Summary and fill in the following blanks. 1. The median loss for all cases in our study was , with of cases causing losses of

$1 million or more. 2. The typical organization loses of revenues in a given year as a result of fraud. 3. The median duration—the amount of time from when the fraud commenced until it was detected—for

the fraud cases reported to us was . 4. Asset misappropriation was by far the most common form of occupational fraud, occurring in more

than of cases, but causing the smallest median loss of . 5. Financial statement fraud was on the other end of the spectrum, occurring in less than 10% of cases

but causing a median loss of . Corruption cases fell in the middle, with of cases and a median loss of .

6. The most common detection method in our study was (39.1% of cases). 7. Approximately of the cases reported to us targeted privately held or publicly owned compa-

nies. These for-profit organizations suffered the largest median losses among the types of organiza- tions analyzed, at and , respectively.

TAKING IT TO THE NET C1 P1

BTN 6-4 Organize the class into teams. Each team must prepare a list of 10 internal controls a con- sumer could observe in a typical retail department store. When called upon, the team’s spokesperson must be prepared to share controls identified by the team that have not been shared by another team’s spokesperson.

TEAMWORK IN ACTION C1

Chapter 6 Cash, Fraud, and Internal Control 269

BTN 6-5 Review the opening feature of this chapter that highlights Sheila Marcelo and her company Care.com. Her company plans to open a kiosk in the Ferry Building in San Francisco to sell Care.com shirts, hats, and other merchandise. Other retail outlets and expansion plans may be in the works.

Required

1. List the seven principles of internal control and explain how a retail outlet might implement each of the principles in its store.

2. Do you believe that a retail outlet will need to add controls to the business as it expands? Explain.

ENTREPRENEURIAL DECISION C1 P1

BTN 6-6 Visit an area of your college that serves the student community with either products or services. Some examples are food services, libraries, and bookstores. Identify and describe between four and eight internal controls being implemented.

HITTING THE ROAD C1

Design elements: Lightbulb: ©Chuhail/Getty Images; Blue globe: ©nidwlw/Getty Images and ©Dizzle52/Getty Images; Chess piece: ©Andrei Simonenko/Getty Images and ©Dizzle52/Getty Images; Mouse: ©Siede Preis/Getty Images; Global View globe: ©McGraw-Hill Education and ©Dizzle52/Getty Images; Sustainability: ©McGraw-Hill Education and ©Dizzle52/Getty Images

Learning Objectives

CONCEPTUAL C1 Describe accounts receivable and how

they occur and are recorded.

C2 Describe a note receivable, the computation of its maturity date, and the recording of its existence.

C3 Explain how receivables can be converted to cash before maturity.

P2 Apply the allowance method to accounts receivable.

P3 Estimate uncollectibles based on sales and accounts receivable.

P4 Record the honoring and dishonoring of a note and adjustments for interest.

ANALYTICAL A1 Compute accounts receivable turnover

and use it to help assess financial condition.

PROCEDURAL P1 Apply the direct write-off method to

accounts receivable.

Chapter Preview

7 Accounting for Receivables

NTK 7-4

ESTIMATING BAD DEBTS

P3 Percent of sales Percent of receivables

Aging of receivables

NTK 7-5

NOTES RECEIVABLE

C2 Maturity and interest

P4 Accounting for notes

C3 Selling and pledging

A1 Receivable turnover

NTK 7-3

ALLOWANCE METHOD

P2 Recording bad debts

Writing off bad debts

Recovery of bad debts

VALUING RECEIVABLES

C1 Sales on credit Sales on store card

Sales on bank card

Sales on installment

NTK 7-2

DIRECT WRITE- OFF METHOD

P1 Recording bad debts

Recovery of bad debts

When to use direct write-off

NTK 7-1

271

“Taking initiative pays off”—Sheryl Sandberg

At Face Value

MENLO PARK, CA—Many know the story of how Mark Zuckerberg started Facebook (Facebook.com) in his college dorm room. How Facebook went from a “cool website” to a profitable company is less well known.

It began at a Christmas party when Sheryl Sandberg met Mark. “We talked for probably an hour by the door,” recalls Mark. After much convincing, Sheryl joined Facebook as its chief op- erating officer.

Sheryl began by reviewing Facebook’s financial statements and was alarmed by the lack of revenue and receivables. “There was this open question,” explains Sheryl. “Could we make money . . . ever?” She organized a meeting where ideas such as charging a subscription fee and inserting ads were pro- posed.

As we now know, Facebook committed to an ad-focused model. The strategy was a huge success, and revenues and receivables soared. Sheryl then moved to her next challenge: managing accounts receivable.

Sheryl and Mark saw that decisions on credit sales and ex- tending credit were impacting income. To combat risk of loss, credit is extended to customers who make timely payments.

Sheryl and Mark also look at cash inflow patterns to estimate uncollectibles and minimize bad debts.

Sheryl enjoys Facebook’s success, but her passion is “mission-based.” She explains: “I believe strongly in what Facebook’s doing. That’s why I get up and go to work every day.”

Sources: Facebook website, January 2019; BSR.org, April 2016; McKinsey, April 2013; New Yorker, July 2011

©Kim White/Bloomberg/Getty Images

A receivable is an amount due from another party. The two most common receivables are accounts receivable and notes receivable. Other receivables include interest receivable, rent receivable, tax refund receivable, and receivables from employees.

Accounts receivable are amounts due from customers for credit sales. Exhibit 7.1 shows amounts of receivables and their percent of total assets for some well-known companies.

VALUING ACCOUNTS RECEIVABLE C1 Describe accounts receivable and how they occur and are recorded.

Percent of Total Assets 0% 2% 4% 6% 8% 10% 12% 14% 16%

Abercrombie & Fitch 4.1%

$3,011 mil.John Deere

Callaway Golf 16.0%

Pfizer 4.8%

5.2%

$93 mil.

$8,225 mil.

$128 mil.

EXHIBIT 7.1 Accounts Receivable for Selected Companies

Sales on Credit Credit sales are recorded by increasing (debiting) Accounts Receivable. The general ledger has a single Accounts Receivable account (called a control account). A com- pany uses a separate account for each customer to track how much that customer purchases, has already paid, and still owes. A supplementary record has a separate account for each customer and is called the accounts receivable ledger (or accounts receivable subsidiary ledger).

Exhibit 7.2 shows the relation between the Accounts Receivable account in the general led- ger and its customer accounts in the accounts receivable ledger for TechCom, a small whole- saler. TechCom’s accounts receivable reports a $3,000 ending balance for June 30. TechCom has two credit customers: CompStore and RDA Electronics. Its schedule of accounts receivable shows that the $3,000 balance of the Accounts Receivable account in the general ledger equals the total of its two customers’ balances in the accounts receivable ledger.

272 Chapter 7 Accounting for Receivables

To see how to record accounts receivable from credit sales, we look at two transactions between TechCom and its credit customers—see Exhibit 7.3. The first is a credit sale of $950 to CompStore. The second is a collection of $720 from RDA Electronics from a prior credit sale.

General Ledger

Date June 30

Debit

3,0003,0003,000

PR Accounts Receivable

Accounts Receivable Ledger

Date June 30

Debit Balance

1,0001,0001,000

PR RDA Electronics

Date June 30

Debit Credit Balance

2,0002,0002,000

PR CompStore

RDA Electronics……………… $1,000 CompStore……………………… 2,000

TechCom Schedule of Accounts Receivable

Credit Balance Credit

Total………………………………… $3,000

EXHIBIT 7.2 General Ledger and the Accounts Receivable Ledger (before July 1 transactions)

Assets = Liabilities + Equity +950 +950

Assets = Liabilities + Equity +720 −720

EXHIBIT 7.3 Accounts Receivable Transactions

July 1 Accounts Receivable—CompStore . . . . . . . . . . . . . . . . . . . . . . 950

Sales . . . . . . . . . . . . . . . . . . . . . . . . . . . . . . . . . . . . . . . . . 950

Record credit sales.*

July 1 Cash . . . . . . . . . . . . . . . . . . . . . . . . . . . . . . . . . . . . . . . . . . . . . . 720

Accounts Receivable—RDA Electronics . . . . . . . . . . . . . . 720

Record collection of credit sales.

* We omit the entry to Dr. Cost of Sales and Cr. Inventory to focus on sales and receivables; no sales returns and allowances are expected.

Exhibit 7.4 shows the general ledger and the accounts receivable ledger after recording the two July 1 transactions. The general ledger shows the effects of the sale, the collection, and the resulting balance of $3,230. These transactions are also shown in the individual customer accounts: RDA Electronics’s ending balance is $280 and CompStore’s ending balance is $2,950. The $3,230 total of customer accounts equals the balance of the Accounts Receivable account in the general ledger.

General Ledger

Date

June 30 July 1 July 1

Debit Credit Balance

3,000 3,950 3,230

3,000 3,950 3,230

3,000 950

720

PR Accounts Receivable

Accounts Receivable Ledger

Date

June 30 July 1

Debit Credit Balance

1,0001,0001,000 720 280

PR RDA Electronics

Date

June 30 July 1

Debit Credit Balance

2,0002,000 2,9502,950

2,000 950

PR CompStore

TechCom Schedule of Accounts Receivable

RDA Electronics……………… $ 280 CompStore……………………… 2,950

Total………………………………… $3,230

EXHIBIT 7.4 General Ledger and the Accounts Receivable Ledger (after July 1 transactions)

Sales on Store Credit Cards Like TechCom, many large retailers such as Home Depot sell on credit. Many also have their own credit cards to grant credit to approved custom- ers and to earn interest on any balance past due. The entries in this case are the same as those for TechCom except for added interest revenue as follows.

Assets = Liabilities + Equity +1,000 +1,000

Assets = Liabilities + Equity +15 +15

Nov . 1 Accounts Receivable . . . . . . . . . . . . . . . . . . . . . . . . . . . . . . . . . . . . . . . . . . 1,000

Sales . . . . . . . . . . . . . . . . . . . . . . . . . . . . . . . . . . . . . . . . . . . . . . . . . . 1,000

Record sales on store credit card.

Dec . 31 Accounts Receivable . . . . . . . . . . . . . . . . . . . . . . . . . . . . . . . . . . . . . . . . . . 15

Interest Revenue . . . . . . . . . . . . . . . . . . . . . . . . . . . . . . . . . . . . . . . . . 15

Interest of $15 earned on store card sales past due.

Chapter 7 Accounting for Receivables 273

Sales on Bank Credit Cards Most companies allow customers to pay using bank (or third-party) credit cards, such as Visa, Mastercard, or American Express, and debit cards. Sellers allow customers to use credit cards and debit cards for several reasons. First, the seller does not have to decide who gets credit and how much. Second, the seller avoids the risk of customers not paying (this risk is transferred to the card company). Third, the seller typically receives cash from the card company sooner than had it granted credit directly to customers. Fourth, more credit options for customers can lead to more sales.

The seller pays a fee when a card is used by the customer, often ranging from 1% to 5% of card sales. This fee reduces the cash received by the seller. If TechCom has $100 of credit card sales with a 4% fee, the entry follows. Some sellers report Credit Card Expense in the income statement as a discount subtracted from sales to get net sales. Other sellers report it as a selling expense or an administrative expense. In this text, we report credit card expense as a selling expense.

Point: JCPenney reported third- party credit card costs exceeding $10 million.

©Science Photo Library/Image Source

Assets = Liabilities + Equity +96 +100 −4

July 15 Cash . . . . . . . . . . . . . . . . . . . . . . . . . . . . . . . . . . . . . . . . . . . . . . . . . . . . . . . 96

Credit Card Expense . . . . . . . . . . . . . . . . . . . . . . . . . . . . . . . . . . . . . . . . . . 4

Sales . . . . . . . . . . . . . . . . . . . . . . . . . . . . . . . . . . . . . . . . . . . . . . . . . . 100

Record credit card sales less a 4% credit card expense.*

*We omit the entry to Dr. Cost of Sales and Cr. Inventory to focus on credit card expense.

Sales on Installment Many companies allow their credit customers to make periodic payments over several months. For example, Harley-Davidson reports more than $2 billion in installment receivables. The seller reports such assets as installment accounts (or finance) re- ceivable, which are amounts owed by customers from credit sales for which payment is required in periodic amounts. Most installment receivables require interest payments, and they can be either current or noncurrent assets depending on the time of repayment.

Credit or Debit? A credit card is authorization by the card company of a line of credit for the buyer—hence, the term credit card. A buyer’s debit card purchase reduces the buyer’s Cash account balance at the card company, which is often a bank. Because the buyer’s Cash account balance is a liability (with a credit balance) for the card com- pany to the buyer, the card company would debit that account for a buyer’s purchase—hence, the term debit card. ■

Decision Insight

Entrepreneur As a small retailer, you are considering allowing customers to use credit cards. Until now, your store accepted only cash. What analysis do you use to decide? ■ Answer: This analysis must weigh benefits versus costs. The main benefit is the potential to increase sales by attracting customers who prefer credit cards. The main cost is the fee charged by the credit card company. We must estimate the expected increase in sales from allowing credit cards and then subtract (1) normal costs and expenses and (2) card fees from the expected sales increase. If analysis shows an increase in profit, the store should probably accept credit cards.

Decision Maker

©PhotoAlto

A small retailer accepts credit cards and has its own store credit card. Prepare journal entries to record the following transactions for the retailer. (The retailer uses the perpetual inventory system.)

Jan. 2 Sold merchandise for $1,000 (that had cost $600) and accepted the customer’s AA Bank Card. AA charges a 5% fee.

6 Sold merchandise for $400 (that had cost $300) and accepted the customer’s VIZA Card. VIZA charges a 3% fee.

31 Recognized the $75 interest revenue earned on its store credit card for January.

C1 Credit Card Sales

NEED-TO-KNOW 7-1

Solution

Jan . 2 Cash . . . . . . . . . . . . . . . . . . . . . . . . . . . . . . . . . . . . . . . . . . . . . . 950

Credit Card Expense* . . . . . . . . . . . . . . . . . . . . . . . . . . . . . . . . . 50

Sales . . . . . . . . . . . . . . . . . . . . . . . . . . . . . . . . . . . . . . . . . 1,000

Record credit card sales less 5% fee. *($1,000 × 0.05) Jan . 2 Cost of Goods Sold . . . . . . . . . . . . . . . . . . . . . . . . . . . . . . . . . . 600

Merchandise Inventory . . . . . . . . . . . . . . . . . . . . . . . . . . . 600

Record cost of sales.

Jan . 6 Cash . . . . . . . . . . . . . . . . . . . . . . . . . . . . . . . . . . . . . . . . . . . . . . 388

Credit Card Expense† . . . . . . . . . . . . . . . . . . . . . . . . . . . . . . . . . 12

Sales . . . . . . . . . . . . . . . . . . . . . . . . . . . . . . . . . . . . . . . . . 400

Record credit card sales less 3% fee. †($400 × 0.03) Jan . 6 Cost of Goods Sold . . . . . . . . . . . . . . . . . . . . . . . . . . . . . . . . . . 300

Merchandise Inventory . . . . . . . . . . . . . . . . . . . . . . . . . . . 300

Record cost of sales.

Jan . 31 Accounts Receivable . . . . . . . . . . . . . . . . . . . . . . . . . . . . . . . . . 75

Interest Revenue . . . . . . . . . . . . . . . . . . . . . . . . . . . . . . . . 75

Record interest earned from store credit card.Do More: QS 7-1, E 7-2, E 7-3

274 Chapter 7 Accounting for Receivables

When a company directly grants credit to customers, it expects some customers will not pay what they promised. The accounts of these customers are uncollectible accounts, or bad debts. Uncollectible accounts are an expense of selling on credit. Why do companies sell on credit if they expect uncollectible accounts? The answer is that companies believe that granting credit will increase total sales enough to offset bad debts. Companies use two methods for uncollect- ible accounts: (1) direct write-off method and (2) allowance method.

Recording and Writing Off Bad Debts The direct write-off method records the loss from an uncollectible account receivable when it is determined to be uncollectible. No at- tempt is made to predict bad debts expense. If TechCom determines on January 23 that it cannot collect $520 owed by its customer J. Kent, it records the loss as follows. The debit in this entry charges the uncollectible amount directly to the current period’s Bad Debts Expense account. The credit removes its balance from the Accounts Receivable account.

Point: Managers realize that some credit sales will be uncollectible, but which credit sales is unknown.

DIRECT WRITE-OFF METHOD P1 Apply the direct write-off method to accounts receivable.

Jan . 23 Bad Debts Expense . . . . . . . . . . . . . . . . . . . . . . . . . . . . . . . . . . 520

Accounts Receivable—J . Kent . . . . . . . . . . . . . . . . . . . . . 520

Write off an uncollectible account.

Assets = Liabilities + Equity −520 −520

Mar . 11 Accounts Receivable—J . Kent . . . . . . . . . . . . . . . . . . . . . . . . . . 520

Bad Debts Expense . . . . . . . . . . . . . . . . . . . . . . . . . . . . . . 520

Reinstate account previously written off.

Mar . 11 Cash . . . . . . . . . . . . . . . . . . . . . . . . . . . . . . . . . . . . . . . . . . . . . . 520

Accounts Receivable—J . Kent . . . . . . . . . . . . . . . . . . . . . 520

Record full payment of account.

Assets = Liabilities + Equity +520 +520

Assets = Liabilities + Equity +520 −520

Recovering a Bad Debt Sometimes an account written off is later collected. If the ac- count of J. Kent that was written off directly to Bad Debts Expense is later collected in full, then we record two entries.

Point: Recovery of a bad debt always requires two journal entries.

Assessing the Direct Write-Off Method Many publicly traded companies and thousands of privately held companies use the direct write-off method; they include

Chapter 7 Accounting for Receivables 275

Rand Medical Billing, Gateway Distributors, First Industrial Realty, New Frontier Energy, Globalink, Solar3D, and Sub Surface Waste Management. The following disclosure by Pharma-Bio Serv is the usual justification: Bad debts are mainly accounted for using the direct write-off method . . . this method approximates that of the allowance method.

Companies weigh at least two concepts when considering use of the direct write-off method. (1) Expense recognition requires expenses be reported in the same period as the sales they helped produce. The direct write-off method usually does not best match sales and expenses because bad debts expense is not recorded until an account becomes uncollectible, which often occurs in a period after the credit sale. (2) The materiality constraint permits use of the direct write-off method when its results are similar to using the allowance method. Otherwise, compa- nies must use the allowance method.

Direct write-off method

Advantages: •  Simple •  No estimates needed

Disadvantages: •   Receivables and income

temporarily overstated •   Bad debts expense often not

matched with sales

A retailer uses the direct write-off method. Record the following transactions.

Feb. 14 The retailer determines that it cannot collect $400 of its accounts receivable from a customer named ZZZ Company.

Apr. 1 ZZZ Company unexpectedly pays its account in full to the retailer, which then records its recov- ery of this bad debt.

Solution

P1

Entries under Direct Write-Off Method

NEED-TO-KNOW 7-2

Do More: QS 7-2, QS 7-3, E 7-4

Feb . 14 Bad Debts Expense . . . . . . . . . . . . . . . . . . . . . . . . . . . . . . . . . . 400

Accounts Receivable—ZZZ Co . . . . . . . . . . . . . . . . . . . . . 400

Write off an account.

Apr . 1 Accounts Receivable—ZZZ Co . . . . . . . . . . . . . . . . . . . . . . . . . . 400

Bad Debts Expense . . . . . . . . . . . . . . . . . . . . . . . . . . . . . . 400

Reinstate an account previously written off.

Apr . 1 Cash . . . . . . . . . . . . . . . . . . . . . . . . . . . . . . . . . . . . . . . . . . . . . . 400

Accounts Receivable—ZZZ Co . . . . . . . . . . . . . . . . . . . . . 400

Record cash received on account.

The allowance method for bad debts matches the estimated loss from uncollectible accounts receivable against the sales they helped produce. We use estimated losses because when sales occur, sellers do not know which customers will not pay. This means that at the end of each period, the allowance method requires an estimate of the total bad debts expected from that period’s sales. This method has two advantages over the direct write-off method: (1) It records estimated bad debts expense in the period when the related sales are recorded and (2) it reports accounts receivable on the balance sheet at the estimated amount to be collected.

Recording Bad Debts Expense The allowance method estimates bad debts expense at the end of each accounting period and records it with an adjusting entry. TechCom had credit sales of $300,000 in its first year of operations. At the end of the first year, $20,000 of credit sales were uncollected. Based on the experience of similar businesses, TechCom estimates that $1,500 of its accounts receiv- able is uncollectible and makes the following adjusting entry.

ALLOWANCE METHOD P2 Apply the allowance method to accounts receivable.

Method Bad Debts Expense Recorded . . .

Direct write-off. . . In future, when accounts are uncollectible. Allowance . . . . . . . Currently, using estimated uncollectibles.

Assets = Liabilities + Equity −1,500 −1,500

Dec . 31 Bad Debts Expense . . . . . . . . . . . . . . . . . . . . . . . . . . . . . . . . . . 1,500

Allowance for Doubtful Accounts . . . . . . . . . . . . . . . . . . . 1,500

Record estimated bad debts.

276 Chapter 7 Accounting for Receivables

The estimated Bad Debts Expense of $1,500 is reported on the income statement (as either a selling expense or an administrative expense). The Allowance for Doubtful Accounts is a con- tra asset account. TechCom’s account balances for Accounts Receivable and the Allowance for Doubtful Accounts follow.

Allowance method

Advantages: •   Receivables fairly stated •   Bad debts expense matched 

with sales •   Writing off bad debt does not affect 

net receivables or income

Disadvantages: •   Estimates needed

Dec . 31 20,000

Accounts Receivable

Dec . 31 1,500

Allowance for Doubtful Accounts

Current assets

Accounts receivable . . . . . . . . . . . . . . . . . . . . . . . . . . . . . . . . . . . . . . $20,000 Less allowance for doubtful accounts . . . . . . . . . . . . . . . . . . . . . . . 1,500 $18,500

Current assets

Accounts receivable (net of $1,500 doubtful accounts) . . . . . . . . . . $18,500

The Allowance for Doubtful Accounts credit balance of $1,500 reduces accounts receivable to its realizable value, which is the amount expected to be received. Although credit customers owe $20,000 to TechCom, only $18,500 is expected from customers. (TechCom still bills its customers for $20,000.) In the balance sheet, the Allowance for Doubtful Accounts is subtracted from Accounts Receivable and is often reported as follows.

Sometimes the Allowance for Doubtful Accounts is not reported separately as follows.

Writing Off a Bad Debt When specific accounts become uncollectible, they are writ- ten off against the Allowance for Doubtful Accounts. TechCom decides that J. Kent’s $520 ac- count is uncollectible and makes the following entry to write it off.

Jan . 23 Allowance for Doubtful Accounts . . . . . . . . . . . . . . . . . . . . . . . 520

Accounts Receivable—J . Kent . . . . . . . . . . . . . . . . . . . . . 520

Write off an uncollectible account.

Assets = Liabilities + Equity +520 −520

This entry removes $520 from the Accounts Receivable account (and the subsidiary ledger). The general ledger accounts appear as follows.

Point: Bad Debts Expense is not debited in the write-off because it was recorded in the period when sales occurred.

The write-off does not affect the realizable value of accounts receivable; see Exhibit 7.5. Nei- ther total assets nor net income is affected by the write-off of a specific account. Instead, both assets and net income are affected in the period when bad debts expense is predicted and recorded with an adjusting entry.

Point: In posting a write-off, the Explanation column shows the reason for this credit so it is not misinterpreted as payment in full.

EXHIBIT 7.5 Realizable Value before and after Write-Off of a Bad Debt

Before Write-Off After Write-Off

Accounts receivable . . . . . . . . . . . . . . . . . . . . . . . . $ 20,000 $ 19,480

Less allowance for doubtful accounts . . . . . . . . . . 1,500 980

Realizable value of accounts receivable . . . . . . $18,500 $18,500

Dec . 31 20,000 Jan . 23 520

Accounts Receivable

Jan . 23 520 Dec . 31 1,500

Allowance for Doubtful Accounts

Chapter 7 Accounting for Receivables 277

Recovering a Bad Debt If an account that was written off is later collected, two en- tries are made. The first is to reverse the write-off and reinstate the customer’s account. The second is to record the collection of the reinstated account. If on March 11 Kent pays in full his account previously written off, the entries are

Exhibit 7.6 portrays the allowance method. It shows the creation of the allowance for future write-offs—adding to a cookie jar. It also shows the decrease of the allowance through write- offs—taking cookies from the jar.

A dj

us tin

g en

tr ie

s Adjusting entries add to allowance

for doubtful accounts. Allowance for

doubtful accounts

Write-o�s

Allowance for doubtful accounts

Bad debt write-o�s subtract from allowance for doubtful accounts.

Increase Allowance Decrease Allowance

Bad Debts Expense… # Allow. for Doubtful Accts… #

Allow. for Doubtful Accts… # Accts Receivable—J.Kent… #

EXHIBIT 7.6 Increases and Decreases to the Allowance for Doubtful Accounts

Assets = Liabilities + Equity +520 −520

Assets = Liabilities + Equity +520 −520

Mar . 11 Accounts Receivable—J . Kent . . . . . . . . . . . . . . . . . . . . . . . . . . 520

Allowance for Doubtful Accounts . . . . . . . . . . . . . . . . . . . 520

Reinstate account previously written off.

Mar . 11 Cash . . . . . . . . . . . . . . . . . . . . . . . . . . . . . . . . . . . . . . . . . . . . . . 520

Accounts Receivable—J . Kent . . . . . . . . . . . . . . . . . . . . . 520

Record full payment of account.

Kent paid the entire amount previously written off, but sometimes a customer pays only a por- tion. If we believe this customer will later pay in full, we return the entire amount owed to accounts receivable (in the first entry only). If we expect no further collection, we return only the amount paid.

A retailer uses the allowance method. Record the following transactions.

Dec. 31 The retailer estimates $3,000 of its accounts receivable are uncollectible at its year-end. Feb. 14 The retailer determines that it cannot collect $400 of its accounts receivable from a customer

named ZZZ Company. Apr. 1 ZZZ Company unexpectedly pays its account in full to the retailer, which then records its

recovery of this bad debt.

Solution

P2

Entries under Allowance Method

NEED-TO-KNOW 7-3

Dec . 31 Bad Debts Expense . . . . . . . . . . . . . . . . . . . . . . . . . . . . . . . . . . 3,000

Allowance for Doubtful Accounts . . . . . . . . . . . . . . . . . . . 3,000

Record estimated bad debts.

Feb . 14 Allowance for Doubtful Accounts . . . . . . . . . . . . . . . . . . . . . . . 400

Accounts Receivable—ZZZ Co . . . . . . . . . . . . . . . . . . . . . 400

Write off an account.

Apr . 1 Accounts Receivable—ZZZ Co . . . . . . . . . . . . . . . . . . . . . . . . . . 400

Allowance for Doubtful Accounts . . . . . . . . . . . . . . . . . . . 400

Reinstate an account previously written off.

Apr . 1 Cash . . . . . . . . . . . . . . . . . . . . . . . . . . . . . . . . . . . . . . . . . . . . . . 400

Accounts Receivable—ZZZ Co . . . . . . . . . . . . . . . . . . . . . 400

Record cash received on account. Do More: QS 7-4, QS 7-5, E 7-5

278 Chapter 7 Accounting for Receivables

Bad debts expense is estimated under the allowance method. This section covers methods for estimating bad debts expense.

Percent of Sales Method The percent of sales method, or income statement method, assumes that a percent of credit sales for the period is uncollectible. For example, Musicland has credit sales of $400,000 in 2019. Music land estimates 0.6% of credit sales to be uncollectible. This means Musicland expects $2,400 of bad debts expense from its sales ($400,000 × 0.006) and makes the following adjusting entry.

Point: Focus on credit sales because cash sales do not produce bad debts.

ESTIMATING BAD DEBTS P3 Estimate uncollectibles based on sales and accounts receivable.

Allowance for Doubtful Accounts, a balance sheet account, is not closed at period end. Unless a company is in its first period of operations, its Allowance for Doubtful Accounts balance rarely equals the Bad Debts Expense balance. (When computing bad debts expense as a percent of sales, managers monitor and adjust the percent so it is not too high or too low.)

Percent of Receivables Method The percent of accounts receivable method, also called a balance sheet method, assumes that a percent of a company’s receivables is uncollectible. This percent is based on experience and economic trends. Total receivables is multiplied by this percent to get the estimated uncollect- ible amount as reported in the balance sheet as Allowance for Doubtful Accounts.

Assume Musicland has $50,000 of accounts receivable on December 31, 2019. It estimates 5% of its receivables is uncollectible. This means that after the adjusting entry is posted, we want the Allowance for Doubtful Accounts to show a $2,500 credit balance (5% of $50,000). Musicland’s beginning balance is $2,200 on December 31, 2018—see Exhibit 7.7.

Point: When using the percent of sales method for estimating uncollectibles, and because the “Unadj. bal.” in Bad Debts Expense is always $0, the adjusting entry amount always equals the % of sales.

Dec . 31* Bad Debts Expense . . . . . . . . . . . . . . . . . . . . . . . . . . . . . . . . . . 2,400

Allowance for Doubtful Accounts . . . . . . . . . . . . . . . . . . . 2,400

Record estimated bad debts.

*The adjusting entry applies our three-step adjusting entry process: Step 1: Current balance for Bad Debts Expense is $0 debit (as the expense account was closed in prior period). Step 2: Current balance for Bad Debts Expense should be $2,400 debit. Step 3: Record entry to get from step 1 to step 2.

Assets = Liabilities + Equity −2,400 −2,400

Bad Debts Expense

Unadj. bal. 0 Adj. (% sales) 2,400

Est. bal. 2,400

During 2019, accounts of customers are written off on July 10 and November 20. The account has a $200 credit balance before the December 31, 2019, adjustment. The adjusting entry to give the allowance account the estimated $2,500 balance is

Assets = Liabilities + Equity −2,300 −2,300

Dec . 31* Bad Debts Expense . . . . . . . . . . . . . . . . . . . . . . . . . . . . . . . . . . 2,300 Allowance for Doubtful Accounts . . . . . . . . . . . . . . . . . . . 2,300 Record estimated bad debts.

*The adjusting entry applies our three-step adjusting entry process: Step 1: Current balance for Allowance account is $200 credit. Step 2: Current balance for Allowance account should be $2,500 credit. Step 3: Record entry to get from step 1 to step 2.

EXHIBIT 7.7 Allowance for Doubtful Accounts after Bad Debts Adjusting Entry

Prior-year estimate of allowance for

doubtful accounts

Adjusting entry

Current-year estimate of allowance for doubtful accounts

Current-year write-o�s

Dec. 31, 2018, bal. 2,200

Dec. 31, 2019, bal.

Dec. 31 adjustment

2,500

July 10

Nov. 20

Unadjusted bal.

Allowance for Doubtful Accounts

200 2,300

1,500 500

Chapter 7 Accounting for Receivables 279

Aging of Receivables Method The aging of accounts receivable method, also called a balance sheet method, is applied like the percent of receivables method except that several percentages are used (versus one) to esti- mate the allowance. Each receivable is classified by how long it is past its due date. Then estimates of uncollectible amounts are made assuming that the longer an amount is past due, the more likely it is uncollectible. After the amounts are classified (or aged), experience is used to estimate the percent of each uncollectible class. These percents are multiplied by the amounts in each class to get the estimated balance of the Allowance for Doubtful Accounts. An example schedule is shown in Exhibit 7.8.

Exhibit 7.8 lists each customer’s balance assigned to one of five classes based on its days past due. The amounts in each class are totaled and multiplied by the estimated percent of uncollect- ible accounts for each class.

For the Ages Unlike wine, accounts receivable do not improve with age. The longer a receivable is past due, the less likely it is to be collected. An aging schedule uses this knowledge to estimate bad debts. The chart here is from a survey that reported estimates of bad debts for receivables grouped by how long they were past their due dates. Each company sets its own estimates based on its customers and its customers’ payment patterns. ■

Decision Insight

9–11

6–8

3–5

2

1

≥12

0% 100%

M on

th s

Pa st

D ue

82%

58%

43%

15%

27%

6% Bad Debts Percentage

Each receivable is grouped by how long it

is past its due date.

MUSICLAND Schedule of Accounts Receivable by Age

December 31, 2019

Customer

Carlie Abbott…………………… 5,890$ $

$ $ $ $ $

5,890

Jamie Allen…………………….. 710 710$

Chavez Andres……………… 10,500

Balicia Company…………… 2,800

Zem Services……………......

Total receivables………..

Percent uncollectible…….

Estimated uncollectible…

× 2% × 5% × 10% × 25% × 40%

21,000

$50,000

$ 2,270$ 2,270

$37,000

740 325 370 475 360

$6,500 $3,700

Texas Rawhide……………… 9,100 6,110 2,990

$1,900 900$

20,810 190

1,900$ $ 900

200$ 10,300

Totals Not Yet

Due

1 to 30 Days

Past Due

31 to 60 Days

Past Due

61 to 90 Days

Past Due

Over 90 Days Past Due

Each age group is multiplied by its estimated

bad debts percent.

Estimated bad debts for each group are totaled.

To explain, Musicland has $3,700 in accounts receivable that are 31 to 60 days past due. Management estimates 10% of the amounts in this class are uncollectible, or a total of $370 ($3,700 × 10%). Similar analysis is done for each class. The final total of $2,270 ($740 + $325 + $370 + $475 + $360) shown in the first column is the estimated balance for the Allowance for Doubtful Accounts. Exhibit 7.9 shows that because the allowance account has an unadjusted

EXHIBIT 7.9 Computation of the Required Adjustment for the Accounts Receivable Method

Unadjusted balance . . . . . . . . . . . . . . . $ 200 credit

Estimated balance . . . . . . . . . . . . . . . . . 2,270 credit

Required adjustment . . . . . . . . . . . . . $2,070 credit

Step 1: Current account balance equals

Step 2: Determine what account balance should be

Step 3: Make adjustment to get from step 1 to step 2

credit balance of $200, the required adjustment to the Allowance for Doubtful Accounts is $2,070. (We can use a T-account for this analysis as shown to the side.) This analysis yields the following end-of-period adjusting entry.

Allowance for Doubtful Accounts

Unadj. bal. 200 Req. adj. 2,070

Est. bal. 2,270

Dec . 31 Bad Debts Expense . . . . . . . . . . . . . . . . . . . . . . . . . . . . . . . . . . 2,070 Allowance for Doubtful Accounts . . . . . . . . . . . . . . . . . . . 2,070 Record estimated bad debts.

Assets = Liabilities + Equity −2,070 −2,070

EXHIBIT 7.8 Aging of Accounts Receivable

280 Chapter 7 Accounting for Receivables

Unadjusted Debit Balance in the Allowance Account If the allowance account had an unad- justed debit balance of $500 (instead of the $200 credit balance), its required adjustment is computed as follows. (A T-account can be used for this analysis as shown to the side.)

Point: A debit balance implies that write-offs for that period exceed the total allowance.

Estimating Bad Debts—Summary of Methods Exhibit 7.10 summarizes the three estimation methods. The aging of accounts receivable method focuses on specific ac- counts and is usually the most reliable of the estimation methods.

Point: Credit approval is usually not assigned to the selling dept. because its goal is to increase sales, and it may approve custom- ers at the cost of increased bad debts.

The entry to record the end-of-period adjustment is

Allowance for Doubtful Accounts

Unadj. bal. 500 Req. adj. 2,770

Est. bal. 2,270

Unadjusted balance . . . . . . . . . . . . . . . $ 500 debit

Estimated balance . . . . . . . . . . . . . . . . . 2,270 credit

Required adjustment . . . . . . . . . . . . . $2,770 credit

Step 1: Current account balance equals

Step 2: Determine what account balance should be

Step 3: Make adjustment to get from step 1 to step 2

Current-year estimate of allowance for doubtful accounts Adjusting entry amount

Assets = Liabilities + Equity −2,770 −2,770

Dec . 31 Bad Debts Expense . . . . . . . . . . . . . . . . . . . . . . . . . . . . . . . . . . 2,770 Allowance for Doubtful Accounts . . . . . . . . . . . . . . . . . . . 2,770 Record estimated bad debts.

Income Statement Focus [Emphasis on Matching]

Percent of Sales

Sales × Rate = Bad Debts Expense

Percent of Receivables Aging of Receivables

Balance Sheet Focus [Emphasis on Realizable Value]

Bad Debts Estimation

Allowance for Doubtful

Accounts × Rate = Accounts

Receivable

Allowance for Doubtful

Accounts

Accounts Receivable

(by Age)

Rates (by Age)

Adj. Entry Amt. = Percent of Sales Adj. Entry Amt. = Percent (or Aging) of Receivables − Unadj. bal. Cr. or + Unadj. bal. Dr.

or

or

EXHIBIT 7.10 Methods to Estimate Bad Debts under the Allowance Method

Labor Union One week prior to labor contract negotiations, financial statements are released showing no income growth. A 10% growth was predicted. Your analysis finds that the company increased its allowance for uncollectibles from 1.5% to 4.5% of receivables. Without this change, income would show a 9% growth. Does this analysis impact negotiations? ■ Answer: Yes, this information is likely to impact negotiations. The obvious question is why the company greatly increased this allow- ance. The large increase means a substantial increase in bad debts expense and a decrease in earnings. This change (coming prior to labor negotiations) also raises concerns because it reduces labor’s bargaining power. We want to ask management for documentation justifying this increase.

Decision Maker

©kali9/Getty Images

At its December 31 year-end, a company estimates uncollectible accounts using the allowance method. 1. It prepared the following aging of receivables analysis. (a) Estimate the balance of the Allowance for

Doubtful Accounts using the aging of accounts receivable method. (b) Prepare the adjusting entry to record bad debts expense using the estimate from part a. Assume the unadjusted balance in the Allowance for Doubtful Accounts is a $10 debit.

Estimating Bad Debts

NEED-TO-KNOW 7-4

P3

Days Past Due

Total 0 1 to 30 31 to 60 61 to 90 Over 90

Accounts receivable . . . . . . . . . . . . $2,600 $2,000 $300 $80 $100 $120

Percent uncollectible . . . . . . . . . . . 1% 2% 5% 7% 10%

Chapter 7 Accounting for Receivables 281

2. Refer to the data in part 1. (a) Estimate the balance of the Allowance for Doubtful Accounts assuming the company uses 2% of total accounts receivable to estimate uncollectibles instead of the aging of receiv- ables method in part 1. (b) Prepare the adjusting entry to record bad debts expense using the estimate from part 2a. Assume the unadjusted balance in the Allowance for Doubtful Accounts is a $4 credit.

3. Refer to the data in part 1. (a) Estimate the balance of the uncollectibles assuming the company uses 0.5% of annual credit sales (annual credit sales were $10,000). (b) Prepare the adjusting entry to record bad debts expense using the estimate from part 3a. Assume the unadjusted balance in the Allowance for Doubtful Accounts is a $4 credit.

Solutions

1a. Computation of the estimated balance of the allowance for uncollectibles.

Do More: QS 7-7, QS 7-8, QS 7-9, E 7-6, E 7-7, E 7-8,

E 7-9, E 7-10, E 7-11

2a. Computation of the estimated balance of the allowance for uncollectibles.

3a. Computation of the estimated balance of the bad debts expense.

$2,600 × 0.02 = $52 credit

3b. Dec . 31 Bad Debts Expense . . . . . . . . . . . . . . . . . . . . . . . . . . . . . . . 50 Allowance for Doubtful Accounts . . . . . . . . . . . . . . . . 50

Record estimated bad debts.

$10,000 × 0.005 = $50 credit

Bad Debts Expense

Unadj. Dec. 31 0 Adj. Dec. 31 50

Est. bal. Dec. 31 50

Allowance for Doubtful Accounts

Unadj. Dec. 31 10 Adj. Dec. 31 59

Est. bal. Dec. 31 49

1b. Dec . 31 Bad Debts Expense . . . . . . . . . . . . . . . . . . . . . . . . . . . . . . . 59 Allowance for Doubtful Accounts . . . . . . . . . . . . . . . . 59

Record estimated bad debts.*

*Unadjusted balance . . . . . . . . . . . . $10 debit Estimated balance . . . . . . . . . . . . . 49 credit Required adjustment . . . . . . . . . . . $59 credit

Step 1: Current account balance equals Step 2: Determine what account balance should be Step 3: Make adjustment to get from step 1 to step 2

Allowance for Doubtful Accounts

Unadj. Dec. 31 4 Adj. Dec. 31 48

Est. bal. Dec. 31 52

2b. Dec . 31 Bad Debts Expense . . . . . . . . . . . . . . . . . . . . . . . . . . . . . . . 48 Allowance for Doubtful Accounts . . . . . . . . . . . . . . . . 48

Record estimated bad debts.*

Step 1: Current account balance equals Step 2: Determine what account balance should be Step 3: Make adjustment to get from step 1 to step 2

*Unadjusted balance . . . . . . . . . . . . $ 4 credit Estimated balance . . . . . . . . . . . . . 52 credit Required adjustment . . . . . . . . . . . $48 credit

NOTES RECEIVABLE C2 Describe a note receivable, the computation of its maturity date, and the recording of its existence.

A promissory note is a written promise to pay a specified amount, usually with interest, either on demand or at a stated future date. Promissory notes are used in many transactions, including paying for products and services and lending and borrowing money. Sellers sometimes ask for a note to replace an account receivable when a customer requests more time to pay a past-due account. Sellers prefer notes when the credit period is long and when the receivable is for a large amount. If a lawsuit is needed to collect from a customer, a note is the customer’s written prom- ise to pay the debt, its amount, and its terms.

Not due . . . . . . . . . . $2,000 × 0 .01 = $20 1 to 30 . . . . . . . . . . 300 × 0 .02 = 6 31 to 60 . . . . . . . . . . 80 × 0 .05 = 4 61 to 90 . . . . . . . . . . 100 × 0 .07 = 7 Over 90 . . . . . . . . . . 120 × 0 .10 = 12 $49 credit

282 Chapter 7 Accounting for Receivables

Exhibit 7.11 shows a promissory note dated July 10, 2019. For this note, Julia Browne prom- ises to pay TechCom or to its order a specified amount ($1,000), called the principal of a note, at a stated future date (October 8, 2019). As the one who signed the note and promised to pay it, Browne is the maker of the note. As the person to whom the note is payable, TechCom is the payee of the note. To Browne, the note is a liability called a note payable. To TechCom, the same note is an asset called a note receivable. This note’s interest rate is 12%, as written on the note. Interest is the charge for using the money until its due date. To a borrower, interest is an expense. To a lender, it is revenue.

Principal

Date of note Due date

Payee

Interest rate

Maker

Promissory Note Amount: ............

...................... after date ........................... promise to pay to the order of

$1,000 Date: ....................July 10, 2019

Ninety days I

12% First National Bank of Los Angeles, CA

TechCom Company Los Angeles, CA

One thousand and no/100 ---------------------------------------------------- Dollars for value received with interest at the annual rate of .......... payable at ............................................................

EXHIBIT 7.11 Promissory Note

Computing Maturity and Interest This section covers a note’s maturity date, period covered, and interest computation.

Maturity Date and Period The maturity date of a note is the day the note (principal and interest) must be repaid. The period of a note is the time from the note’s (contract) date to its maturity date. Many notes mature in less than a full year, and the period they cover is often ex- pressed in days. As an example, a five-day note dated June 15 matures and is due on June 20. A 90-day note dated July 10 matures on October 8. This count is shown in Exhibit 7.12. The period of a note is sometimes expressed in months or years. When months are used, the note is payable in the month of its maturity on the same day of the month as its original date. A nine-month note dated July 10, for example, is payable on April 10. The same rule applies when years are used.

Point: When counting days, omit the day a note is issued, but count the due date.

EXHIBIT 7.12 Maturity Date Computation

Days in July . . . . . . . . . . . . . . . . . . . . . . . . . . . . . . . . . . . . . . . . . . 31

Minus the date of the note . . . . . . . . . . . . . . . . . . . . . . . . . . . . . . 10

Days remaining in July . . . . . . . . . . . . . . . . . . . . . . . . . . . . . . . . . 21

Add days in August . . . . . . . . . . . . . . . . . . . . . . . . . . . . . . . . . . . . 31

Add days in September . . . . . . . . . . . . . . . . . . . . . . . . . . . . . . . . 30

Days to equal 90 days, or maturity date of October 8 . . . . . . . 8 Period of the note in days . . . . . . . . . . . . . . . . . . . . . . . . . . . . . . 90

July 11–31

Aug . 1–31

Sep . 1–30

Oct . 1–8

July 10

21 days +31 days +30 days +8 days

90-day note Oct. 8

July 11–31

Aug. 1–31

Sep. 1–30

Oct. 1–8

Interest Computation Interest is the cost of borrowing money for the borrower and the profit from lending money for the lender. Unless otherwise stated, the rate of interest on a note is the rate charged for the use of principal for one year (annual rate). The formula for com- puting interest on a note is in Exhibit 7.13.

Point: Excel for maturity date.

A B

1 Note date 10-Jul

2 # of days 90

3 Maturity

=B1+B2 = 8-Oct

EXHIBIT 7.13 Computation of Interest Formula

Principal Annual Time expressed of the note × interest rate × in fraction of year = Interest

To simplify interest computations, a year is commonly treated as having 360 days (called the banker’s rule and widely used in business transactions). We treat a year as having 360 days

Chapter 7 Accounting for Receivables 283

for interest computations in examples and assignments. Using the promissory note in Exhibit 7.11, where we have a 90-day, 12%, $1,000 note, the total interest follows.

$1,000 × 12% × 90 360

= $1,000 × 0.12 × 0.25 = $30

Recording Notes Receivable Notes receivable are usually recorded in a single Notes Receivable account to simplify record- keeping. To show how we record receipt of a note, we use the $1,000, 90-day, 12% promissory note in Exhibit 7.11. TechCom received this note at the time of a product sale to Julia Browne. This is recorded as

Point: If the banker’s rule is not used, interest is $29.589041. The banker’s rule yields $30, which is easier to account for than $29.589041.

Point: Maturity value of a note equals principal plus interest earned.

When a seller accepts a note from an overdue customer to grant a time extension on a past-due account receivable, it often will collect part of the past-due balance in cash. Assume that Tech- Com agreed to accept $232 in cash along with a $600, 60-day, 15% note from Jo Cook to settle her $832 past-due account. TechCom makes the following entry.

July 10* Notes Receivable . . . . . . . . . . . . . . . . . . . . . . . . . . . . . . . . . . . . 1,000

Sales . . . . . . . . . . . . . . . . . . . . . . . . . . . . . . . . . . . . . . . . . 1,000

Sold goods in exchange for a 90-day, 12% note.

*We omit the entry to Dr. Cost of Sales and Cr. Inventory to focus on sales and receivables.

Assets = Liabilities + Equity +1,000 +1,000

Oct . 5 Cash . . . . . . . . . . . . . . . . . . . . . . . . . . . . . . . . . . . . . . . . . . . . . . 232

Notes Receivable . . . . . . . . . . . . . . . . . . . . . . . . . . . . . . . . . . . . 600

Accounts Receivable—J . Cook . . . . . . . . . . . . . . . . . . . . . 832

Received cash and note to settle account.

Assets = Liabilities + Equity +232 +600 −832

Valuing and Settling Notes Recording an Honored Note The principal and interest of a note are due on its ma- turity date. The maker of the note usually honors the note and pays it in full. When J. Cook pays the note above on its due date, TechCom records it as follows. Interest revenue, or interest earned, is reported on the income statement.

Recording a Dishonored Note When a note’s maker does not pay at maturity, the note is dishonored. Dishonoring a note does not mean the maker no longer has to pay. The payee still tries to collect. How do companies report this? The balance of the Notes Receivable ac- count should only include notes that have not matured. When a note is dishonored, we remove the amount of this note from Notes Receivable and charge it back to an account receivable from its maker. Assume that J. Cook dishonors the note at maturity. The following records the dis- honoring of the note.

P4 Record the honoring and dishonoring of a note and adjustments for interest.

Charging a dishonored note to accounts receivable does two things. First, it removes the note from the Notes Receivable account and records the dishonored note in the maker’s account. Second, if the maker of the dishonored note asks for credit in the future, his or her account will show the dishonored note.

Dec . 4 Cash . . . . . . . . . . . . . . . . . . . . . . . . . . . . . . . . . . . . . . . . . . . . . . 615

Notes Receivable . . . . . . . . . . . . . . . . . . . . . . . . . . . . . . . 600

Interest Revenue . . . . . . . . . . . . . . . . . . . . . . . . . . . . . . . . 15

Collect note with interest of $600 × 15% × 60∕360.

Assets = Liabilities + Equity +615 +15 −600

Dec . 4 Accounts Receivable—J . Cook . . . . . . . . . . . . . . . . . . . . . . . . . 615

Interest Revenue . . . . . . . . . . . . . . . . . . . . . . . . . . . . . . . . 15

Notes Receivable . . . . . . . . . . . . . . . . . . . . . . . . . . . . . . . 600

Charge account of J. Cook for a dishonored note and interest of $600 × 15% × 60∕360.

Assets = Liabilities + Equity +615 +15 −600

284 Chapter 7 Accounting for Receivables

Recording End-of-Period Interest Adjustment When notes receivable are out- standing at period-end, any accrued interest is recorded. Assume on December 16 TechCom accepts a $3,000, 60-day, 12% note from a customer. When TechCom’s accounting period ends on December 31, $15 of interest has accrued on this note ($3,000 × 12% × 15/360). The follow- ing adjusting entry records this revenue.

Total interest on the 60-day note is $60 ($3,000 × 12% × 60/360). The $15 credit to Interest Receivable is the collection of interest accrued from the December 31 entry. The $45 interest revenue is from holding the note from January 1 to February 14.

Interest revenue is on the income statement, and interest receivable is on the balance sheet as a current asset. When the December 16 note is collected on February 14, TechCom’s entry to record the cash receipt is

Dec . 31 Interest Receivable . . . . . . . . . . . . . . . . . . . . . . . . . . . . . . . . . . 15

Interest Revenue . . . . . . . . . . . . . . . . . . . . . . . . . . . . . . . . 15

Record accrued interest earned.

Assets = Liabilities + Equity +15 +15

Assets = Liabilities + Equity +3,060 +45 −15 −3,000

Feb . 14 Cash . . . . . . . . . . . . . . . . . . . . . . . . . . . . . . . . . . . . . . . . . . . . . . 3,060

Interest Revenue . . . . . . . . . . . . . . . . . . . . . . . . . . . . . . . . 45

Interest Receivable . . . . . . . . . . . . . . . . . . . . . . . . . . . . . . 15

Notes Receivable . . . . . . . . . . . . . . . . . . . . . . . . . . . . . . . 3,000

Received payment of note and its interest.

Ace Company purchases $1,400 of merchandise from Zitco on December 16. Zitco accepts Ace’s $1,400, 90-day, 12% note as payment. Zitco’s accounting period ends on December 31. a. Prepare entries for Zitco on December 16 and December 31. b. Prepare Zitco’s March 16 entry if Ace dishonors the note. c. Instead of the facts in part b, prepare Zitco’s March 16 entry if Ace honors the note. d. Assume the facts in part b (Ace dishonors the note). Then, on March 31, Zitco writes off the receivable

from Ace Company. Prepare that write-off entry assuming that Zitco uses the allowance method.

Solution

a.

C2 P4

Honoring and Dishonoring Notes

NEED-TO-KNOW 7-5

Mar . 16 Accounts Receivable—Ace . . . . . . . . . . . . . . . . . . . . . . . . . 1,442

Interest Revenue ($1,400 × 12% × 75∕360) . . . . . . 35 Interest Receivable . . . . . . . . . . . . . . . . . . . . . . . . . . . 7

Notes Receivable—Ace . . . . . . . . . . . . . . . . . . . . . . . 1,400

b.

Dec . 16 Note Receivable—Ace . . . . . . . . . . . . . . . . . . . . . . . . . . . . . 1,400

Sales . . . . . . . . . . . . . . . . . . . . . . . . . . . . . . . . . . . . . . 1,400

Dec . 31 Interest Receivable . . . . . . . . . . . . . . . . . . . . . . . . . . . . . . . 7

Interest Revenue ($1,400 × 12% × 15∕360) . . . . . . 7

Mar . 16 Cash . . . . . . . . . . . . . . . . . . . . . . . . . . . . . . . . . . . . . . . . . . . 1,442

Interest Revenue . . . . . . . . . . . . . . . . . . . . . . . . . . . . . 35

Interest Receivable . . . . . . . . . . . . . . . . . . . . . . . . . . . 7

Notes Receivable—Ace . . . . . . . . . . . . . . . . . . . . . . . 1,400

c.

Mar . 31 Allowance for Doubtful Accounts . . . . . . . . . . . . . . . . . . . . 1,442

Accounts Receivable—Ace . . . . . . . . . . . . . . . . . . . . . 1,442

d.Do More: QS 7-10, QS 7-11, QS 7-12, QS 7-13, E 7-12,

E 7-13, E 7-14, E 7-15

Chapter 7 Accounting for Receivables 285

Disposal of Receivables Companies convert receivables to cash before they are due if they need cash or do not want to deal with collecting receivables. This is usually done by (1) selling them or (2) using them as security for a loan.

Selling Receivables A company can sell its receivables to a finance company or bank. The buyer, called a factor, acquires ownership of the receivables and receives cash when they come due. The seller is charged a factoring fee. By incurring a factoring fee, the seller gets cash earlier and can pass the risk of bad debts to the factor. The seller also avoids costs of billing and accounting for receivables. If TechCom sells $20,000 of its accounts receivable and is charged a 4% factoring fee, it records this sale as follows.

C3 Explain how receivables can be converted to cash before maturity.

Point: A seller of receivables al- ways receives less cash than the amount of receivables sold due to factoring fees.

Pledging Receivables A company can borrow money by pledging its receivables as se- curity for the loan. If the borrower defaults on (does not pay) the loan, the lender is paid from the cash receipts of the receivables. The borrower discloses pledging receivables in financial state- ment footnotes. If TechCom borrows $35,000 and pledges its receivables as security, it records

Assets = Liabilities + Equity +19,200 −800 −20,000

Aug . 15 Cash . . . . . . . . . . . . . . . . . . . . . . . . . . . . . . . . . . . . . . . . . . . . . . 19,200

Factoring Fee Expense . . . . . . . . . . . . . . . . . . . . . . . . . . . . . . . 800

Accounts Receivable . . . . . . . . . . . . . . . . . . . . . . . . . . . . . 20,000

Sold accounts receivable for cash less 4% fee.

Aug . 20 Cash . . . . . . . . . . . . . . . . . . . . . . . . . . . . . . . . . . . . . . . . . . . . . . 35,000

Notes Payable . . . . . . . . . . . . . . . . . . . . . . . . . . . . . . . . . . 35,000

Borrow with a note secured by pledging receivables.

Assets = Liabilities + Equity +35,000 +35,000

©Rawpixel.com/Shutterstock

Analyst/Auditor You are reviewing accounts receivable. Over the past five years, the allowance account as a per- centage of gross accounts receivable shows a steady downward trend. What does this finding suggest? ■ Answer: The downward trend means the company is reducing the relative amount charged to bad debts expense each year. This could be to increase net income. Alternatively, collections may have improved and fewer bad debts are justified.

Decision Maker

Accounts Receivable Turnover Decision Analysis

Accounts receivable turnover helps assess the quality and liquidity of receivables. Quality of receivables is the likelihood of collection without loss. Liquidity of receivables is the speed of collection. Accounts receivable turnover measures how often, on average, receivables are collected during the period and is defined in Exhibit 7.14.

A1 Compute accounts receivable turnover and use it to help assess financial condition.

EXHIBIT 7.14 Accounts Receivable Turnover

Accounts receivable turnover = Net sales

Average accounts receivable, net

The denominator is the average accounts receivable, net balance, computed as (Beginning balance + Ending balance) ÷ 2. TechCom has an accounts receivable turnover of 5.1. This means its average accounts receivable balance is converted into cash 5.1 times during the period, which is pictured here.

Jan. Feb. Mar. Apr. May June July Aug. Sep. Oct. Nov. Dec.

5.1 times per year

54321

Accounts receivable turnover shows how well management is doing in granting credit to customers. A high turnover suggests that management should consider using less strict credit terms to increase sales. A low turnover suggests management should consider more strict credit terms and more aggressive collec- tion efforts to avoid having assets tied up in accounts receivable.

Clayco Company completes the following transactions during the year.

July 14 Writes off a $750 account receivable arising from a sale to Briggs Company that dates to 10 months ago. (Clayco Company uses the allowance method.)

30 Clayco Company receives a $1,000, 90-day, 10% note in exchange for merchandise sold to Sumrell Company (the merchandise cost $600).

Aug. 15 Receives $2,000 cash plus a $10,000 note from JT Co. in exchange for merchandise that sells for $12,000 (its cost is $8,000). The note is dated August 15, bears 12% interest, and matures in 120 days.

Nov. 1 Completes a $200 credit card sale with a 4% fee (the cost of sales is $150). The cash is trans- ferred immediately from the credit card company.

3 Sumrell Company refuses to pay the note that was due to Clayco Company on October 28. Pre- pare the journal entry to charge the dishonored note plus accrued interest to Sumrell Company’s accounts receivable.

5 Completes a $500 credit card sale with a 5% fee (the cost of sales is $300). The cash is trans- ferred immediately from the credit card company.

15 Receives the full amount of $750 from Briggs Company that was previously written off on July 14. Record the bad debts recovery.

Dec. 13 Receives payment of principal plus interest from JT for the August 15 note.

Required

1. Prepare Clayco Company’s journal entries to record these transactions. 2. Prepare a year-end adjusting journal entry as of December 31 for each separate situation. a. Bad debts are estimated to be $20,400 by aging accounts receivable. The unadjusted balance of the

Allowance for Doubtful Accounts is a $1,000 debit. b. Alternatively, assume that bad debts are estimated using the percent of sales method. The Allowance

for Doubtful Accounts had a $1,000 debit balance before adjustment, and the company estimates bad debts to be 1% of its credit sales of $2,000,000.

PLANNING THE SOLUTION Examine each transaction to determine the accounts affected, and then record the entries. For the year-end adjustment, record the bad debts expense for the two approaches.

COMPREHENSIVE

Recording Accounts and Notes Receivable Transactions; Estimating Bad Debts

NEED-TO-KNOW 7-6

286 Chapter 7 Accounting for Receivables

Exhibit 7.15 shows accounts receivable turnover for Visa and Mastercard.

EXHIBIT 7.15 Analysis Using Accounts Receivable Turnover

Company Figure ($ millions) Current Year 1 Year Ago 2 Years Ago

Visa Net sales . . . . . . . . . . . . . . . . . . . . . . . . . . . . . . . $18,358 $15,082 $13,880 Average accounts receivable, net . . . . . . . . . . . $ 1,087 $ 944 $ 835

Accounts receivable turnover . . . . . . . . . . . . . 16.9 16.0 16.6 Mastercard Net sales . . . . . . . . . . . . . . . . . . . . . . . . . . . . . . . $12,497 $10,776 $ 9,667 Average accounts receivable, net . . . . . . . . . . . $ 1,693 $ 1,248 $ 1,094

Accounts receivable turnover . . . . . . . . . . . . . 7.4 8.6 8.8

Visa’s current year turnover is 16.9, computed as $18,358/$1,087 ($ millions). This means that Visa’s average accounts receivable balance was converted into cash 16.9 times in the current year. Its turnover slightly increased in the current year (16.9) compared with one year ago (16.0). Visa’s turnover also exceeds that for Mastercard in each of these three years. Both Visa and Mastercard seem to be doing an adequate job of managing receivables.

Family Physician Your medical practice is barely profitable, so you hire an analyst. The analyst says, “Accounts receivable turnover is too low. Tighter credit policies are recommended along with discontinuing service to those most delayed in payments.” What actions do you take? ■ Answer: Both suggestions are probably financially wise recommendations, but we may be troubled by eliminating services to those less able to pay. One alternative is to follow the recommendations but start a care program directed at patients less able to pay for services. This allows you to continue services to patients less able to pay and to discontinue services to patients able but unwilling to pay.

Decision Maker

Chapter 7 Accounting for Receivables 287

SOLUTION 1.

July 14 Allowance for Doubtful Accounts . . . . . . . . 750

Accounts Receivable—Briggs Co . . . . . 750

Wrote off an uncollectible account.

July 30 Notes Receivable—Sumrell Co . . . . . . . . . . . 1,000

Sales . . . . . . . . . . . . . . . . . . . . . . . . . . 1,000

Sold merchandise for a 90-day, 10% note.

July 30 Cost of Goods Sold . . . . . . . . . . . . . . . . . . . 600

Merchandise Inventory . . . . . . . . . . . . 600

Record the cost of July 30 sale.

Aug . 15 Cash . . . . . . . . . . . . . . . . . . . . . . . . . . . . . . . 2,000

Notes Receivable—JT Co . . . . . . . . . . . . . . . 10,000

Sales . . . . . . . . . . . . . . . . . . . . . . . . . . 12,000

Sold merchandise for $2,000 cash and $10,000 note.

Aug . 15 Cost of Goods Sold . . . . . . . . . . . . . . . . . . . 8,000

Merchandise Inventory . . . . . . . . . . . . 8,000

Record the cost of Aug. 15 sale.

Nov . 1 Cash . . . . . . . . . . . . . . . . . . . . . . . . . . . . . . . 192

Credit Card Expense . . . . . . . . . . . . . . . . . . 8

Sales . . . . . . . . . . . . . . . . . . . . . . . . . . 200

Record credit card sale less a 4% credit card expense.

Nov . 1 Cost of Goods Sold . . . . . . . . . . . . . . . . . . . 150

Merchandise Inventory . . . . . . . . . . . . 150

Record the cost of Nov. 1 sale.

Nov . 3 Accounts Receivable—Sumrell Co . . . . . . . . 1,025

Interest Revenue . . . . . . . . . . . . . . . . . 25

Notes Receivable—Sumrell Co . . . . . . 1,000

Charge account of Sumrell Co. for a $1,000 dishonored note and interest of $1,000 × 10% × 90/360.

Nov . 5 Cash . . . . . . . . . . . . . . . . . . . . . . . . . . . . . . . 475

Credit Card Expense . . . . . . . . . . . . . . . . . . 25

Sales . . . . . . . . . . . . . . . . . . . . . . . . . . 500

Record credit card sale less a 5% credit card expense.

Nov . 5 Cost of Goods Sold . . . . . . . . . . . . . . . . . . . 300

Merchandise Inventory . . . . . . . . . . . . 300

Record the cost of Nov. 5 sale.

Nov . 15 Accounts Receivable—Briggs Co . . . . . . . . . 750

Allowance for Doubtful Accounts . . . . 750

Reinstate account of Briggs Co. previously written off.

Nov . 15 Cash . . . . . . . . . . . . . . . . . . . . . . . . . . . . . . . 750

Accounts Receivable—Briggs Co . . . . 750

Cash received in full payment of account.

Dec . 13 Cash . . . . . . . . . . . . . . . . . . . . . . . . . . . . . . . 10,400

Interest Revenue . . . . . . . . . . . . . . . . . 400

Note Receivable—JT Co . . . . . . . . . . . 10,000

Collect note with interest of $10,000 × 12% × 120/360.

2a. Aging of accounts receivable method.

Dec . 31 Bad Debts Expense . . . . . . . . . . . . . . . . . . . . . . . . . . . . . . . . . . 21,400

Allowance for Doubtful Accounts . . . . . . . . . . . . . . . . . . . 21,400

Adjust allowance account from a $1,000 debit balance to a $20,400 credit balance.

Dec . 31 Bad Debts Expense . . . . . . . . . . . . . . . . . . . . . . . . . . . . . . . . . . 20,000

Allowance for Doubtful Accounts . . . . . . . . . . . . . . . . . . . 20,000

Record bad debts expense as 1% × $2,000,000 of credit sales.

2b. Percent of sales method. (For the income statement approach, which requires estimating bad debts as a percent of sales or credit sales, the Allowance for Doubtful Accounts balance is not considered when making the adjusting entry.)

VALUING RECEIVABLES Accounts Receivable: Amounts due from customers for credit sales.

Store credit card interest revenue:

Sales using bank credit card: Accounts Receivable—CompStore . . . . . . . . . . . . . . . . . 950

Sales . . . . . . . . . . . . . . . . . . . . . . . . . . . . . . . . . . . . . 950

Cash . . . . . . . . . . . . . . . . . . . . . . . . . . . . . . . . . . . . . . . . . 720

Accounts Receivable—RDA Electronics . . . . . . . . . 720

Accounts Receivable . . . . . . . . . . . . . . . . . . . . . . . . . . . . 15

Interest Revenue . . . . . . . . . . . . . . . . . . . . . . . . . . . 15

Cash . . . . . . . . . . . . . . . . . . . . . . . . . . . . . . . . . . . . . . . . . 96

Credit Card Expense . . . . . . . . . . . . . . . . . . . . . . . . . . . . . 4

Sales . . . . . . . . . . . . . . . . . . . . . . . . . . . . . . . . . . . . . 100

Summary: Cheat Sheet

Credit sales and later collection:

288 Chapter 7 Accounting for Receivables

DIRECT WRITE-OFF METHOD Direct write-off method: Record bad debt expense when an account is determined to be uncollectible.

ESTIMATING BAD DEBTS When using the allowance method, we often use one of the following methods to estimate bad debts. ∙ Percent of sales: Uses a percent of credit sales for the period to estimate

bad debts. ∙ Percent of accounts receivable: Uses a percent of accounts receivable

to estimate bad debts. ∙ Aging of accounts receivable: Applies several percentages to accounts

receivable to estimate bad debts.

NOTES RECEIVABLE Note receivable: A promise to pay a specified amount of money at a future date. Principal of a note: Amount promised to be repaid. Maturity date: Day the note must be repaid.

Pledging of receivables: Borrowing money by pledging receivables as security for a loan. Borrower discloses pledging in notes to financial statement.

ALLOWANCE METHOD Allowance method: Matches estimated loss from uncollectible accounts receivable against the sales they helped produce.

Allowance for Doubtful Accounts: A contra asset account that reduces accounts receivable.

Bad debt is later recovered under allowance method:

Accounts Receivable—J . Kent . . . . . . . . . . . . . . . . . . . . . 520

Allowance for Doubtful Accounts . . . . . . . . . . . . . . 520

Cash . . . . . . . . . . . . . . . . . . . . . . . . . . . . . . . . . . . . . . . . . 520

Accounts Receivable—J . Kent . . . . . . . . . . . . . . . . . 520

Accounts Receivable—J . Kent . . . . . . . . . . . . . . . . . . . . . 520

Bad Debts Expense . . . . . . . . . . . . . . . . . . . . . . . . . 520

Cash . . . . . . . . . . . . . . . . . . . . . . . . . . . . . . . . . . . . . . . . . 520

Accounts Receivable—J . Kent . . . . . . . . . . . . . . . . . 520

Bad debt later recovered under direct method:

Income Statement Focus [Emphasis on Matching]

Percent of Sales Percent of Receivables Aging of Receivables

Balance Sheet Focus [Emphasis on Realizable Value]

Bad Debts Estimation

Allowance for Doubtful

Accounts × Rate = AccountsReceivable

Allowance for Doubtful

Accounts

Accounts Receivable

(by Age)

Rates (by Age)

=× Sales × Rate = Bad Debts Expense

Adj. Entry Amt. = Percent of Sales Adj. Entry Amt. = Percent (or Aging) of Receivables − Unadj. bal. Cr. or + Unadj. bal. Dr.

or

or

Note receivable from sales:

Notes Receivable . . . . . . . . . . . . . . . . . . . . . . . . . . . . . . . 1,000

Sales . . . . . . . . . . . . . . . . . . . . . . . . . . . . . . . . . . . . . 1,000

Accrue interest on note receivable:

Interest Receivable . . . . . . . . . . . . . . . . . . . . . . . . . . . . . . 15

Interest Revenue . . . . . . . . . . . . . . . . . . . . . . . . . . . 15

Note receivable and cash in exchange for accounts receivable:

Cash . . . . . . . . . . . . . . . . . . . . . . . . . . . . . . . . . . . . . . . . . 232

Notes Receivable . . . . . . . . . . . . . . . . . . . . . . . . . . . . . . . 600

Accounts Receivable—J . Cook . . . . . . . . . . . . . . . . 832

Note is honored; cash received in full (with interest):

Cash . . . . . . . . . . . . . . . . . . . . . . . . . . . . . . . . . . . . . . . . . 615

Notes Receivable . . . . . . . . . . . . . . . . . . . . . . . . . . . 600

Interest Revenue . . . . . . . . . . . . . . . . . . . . . . . . . . . 15

Note is dishonored; receivable and interest recorded:

Accounts Receivable—J . Cook . . . . . . . . . . . . . . . . . . . . . 615

Interest Revenue . . . . . . . . . . . . . . . . . . . . . . . . . . . 15

Notes Receivable . . . . . . . . . . . . . . . . . . . . . . . . . . . 600

Factoring (selling) receivables: Accounts receivable are sold to a bank and the seller is charged a factoring fee.

Note is honored; when note term runs over two periods:

Cash . . . . . . . . . . . . . . . . . . . . . . . . . . . . . . . . . . . . . . . . . 3,060

Interest Revenue . . . . . . . . . . . . . . . . . . . . . . . . . . . 45

Interest Receivable . . . . . . . . . . . . . . . . . . . . . . . . . 15

Notes Receivable . . . . . . . . . . . . . . . . . . . . . . . . . . . 3,000

Bad Debts Expense . . . . . . . . . . . . . . . . . . . . . . . . . . . . . 520

Accounts Receivable—J . Kent . . . . . . . . . . . . . . . . . 520

Writing off a bad debt under direct method:

Bad Debts Expense . . . . . . . . . . . . . . . . . . . . . . . . . . . . . 1,500

Allowance for Doubtful Accounts . . . . . . . . . . . . . . 1,500

Estimating bad debts:

Allowance for Doubtful Accounts . . . . . . . . . . . . . . . . . . 520

Accounts Receivable—J . Kent . . . . . . . . . . . . . . . . . 520

Writing off a bad debt under allowance method:

Cash . . . . . . . . . . . . . . . . . . . . . . . . . . . . . . . . . . . . . . . . . 19,200

Factoring Fee Expense . . . . . . . . . . . . . . . . . . . . . . . . . . . 800

Accounts Receivable . . . . . . . . . . . . . . . . . . . . . . . . 20,000

Sale of receivables for cash with a charged factor fee:

Principal Annual Time expressed of the note × interest rate × in fraction of year = Interest

Interest formula (year assumed to have 360 days):

Chapter 7 Accounting for Receivables 289

Icon denotes assignments that involve decision making.

1. How do sellers benefit from allowing their customers to use credit cards?

2. Why does the direct write-off method of accounting for bad debts usually fail to match revenues and expenses?

3. Explain the accounting constraint of materiality. 4. Why might a business prefer a note receivable to an account

receivable? 5. Explain why writing off a bad debt against the Allowance

for Doubtful Accounts does not reduce the estimated realiz- able value of a company’s accounts receivable.

6. Why does the Bad Debts Expense account usually not have the same adjusted balance as the Allowance for Doubtful Accounts?

7. Refer to the financial statements and notes of Apple in Appendix A. In its presentation of accounts receivable on the balance sheet, how does it title accounts receivable? What does it report for its allowance as of September 30, 2017?

8. Refer to the balance sheet of Google in Appendix A. Does it use the direct write-off

Discussion Questions

APPLE

GOOGLE

Accounts receivable (271) Accounts receivable turnover (285) Aging of accounts receivable (279) Allowance for Doubtful Accounts (276) Allowance method (275)

Bad debts (274) Direct write-off method (274) Interest (282) Maker of the note (282) Maturity date of a note (282)

Payee of the note (282) Principal of a note (282) Promissory note (or note) (281) Realizable value (276)

Key Terms

Multiple Choice Quiz

1. A company’s Accounts Receivable balance at its December 31 year-end is $125,650, and its Allowance for Doubtful Accounts has a credit balance of $328 before year-end ad- justment. Its net sales are $572,300. It estimates that 4% of outstanding accounts receivable are uncollectible. What amount of bad debts expense is recorded at December 31? a. $5,354 c. $5,026 e. $34,338 b. $328 d. $4,698

2. A company’s Accounts Receivable balance at its December 31 year-end is $489,300, and its Allowance for Doubtful Accounts has a debit balance of $554 before year-end ad- justment. Its net sales are $1,300,000. It estimates that 6% of outstanding accounts receivable are uncollectible. What amount of bad debts expense is recorded at December 31? a. $29,912 c. $78,000 e. $554 b. $28,804 d. $29,358

3. Total interest to be earned on a $7,500, 5%, 90-day note is a. $93.75. c. $1,125.00. e. $125.00. b. $375.00. d. $31.25.

4. A company receives a $9,000, 8%, 60-day note. The matu- rity value of the note is a. $120. c. $9,120. e. $9,720. b. $9,000. d. $720.

5. A company has net sales of $489,600 and average accounts receivable of $40,800. What is its accounts receivable turn- over? a. 0.08 c. 1,341.00 e. 111.78 b. 30.41 d. 12.00

ANSWERS TO MULTIPLE CHOICE QUIZ

1. d; Desired balance in Allowance for Doubtful Accounts = $ 5,026 cr. ($125,650 × 0.04)

Current balance in Allowance for Doubtful Accounts = (328) cr. Bad debts expense to be recorded = $ 4,698 2. a; Desired balance in Allowance for Doubtful Accounts = $ 29,358 cr.

($489,300 × 0.06) Current balance in Allowance for Doubtful Accounts = 554 dr. Bad debts expense to be recorded = $29,912

3. a; $7,500 × 0.05 × 90∕360 = $93.75 4. c; Principal amount . . . . . . . . $9,000 Interest accrued . . . . . . . . . 120 ($9,000 × 0.08 × 60∕360) Maturity value . . . . . . . . . . $9,120 5. d; $489,600∕$40,800 = 12

290 Chapter 7 Accounting for Receivables

method or allowance method in accounting for its accounts receivable? What is the realizable value of its receivables balance as of December 31, 2017?

9. Refer to the financial statements of Samsung in Appendix A. What is the amount of Samsung’s accounts receivable, titled as

“Trade receivables,” on its December 31, 2017, balance sheet?

10. Refer to the December 31, 2017, financial statements of Samsung in Appendix A. Does Samsung report its accounts receivable, titled as “Trade receivables,” as a current or noncurrent asset?

Samsung Samsung

QUICK STUDY

QS 7-1 Credit card sales

C1

Prepare journal entries for the following credit card sales transactions (the company uses the perpetual inventory system). 1. Sold $20,000 of merchandise, which cost $15,000, on Mastercard credit cards. Mastercard charges a

5% fee. 2. Sold $5,000 of merchandise, which cost $3,000, on an assortment of bank credit cards. These cards

charge a 4% fee.

QS 7-2 Direct write-off method

P1

Solstice Company determines on October 1 that it cannot collect $50,000 of its accounts receivable from its customer, P. Moore. Apply the direct write-off method to record this loss as of October 1.

QS 7-3 Recovering a bad debt

P1

Solstice Company determines on October 1 that it cannot collect $50,000 of its accounts receivable from its customer, P. Moore. It uses the direct write-off method to record this loss as of October 1. On October 30, P. Moore unexpectedly pays his account in full to Solstice Company. Record Solstice’s entries for recovery of this bad debt.

QS 7-4 Distinguishing between allowance method and direct write-off method

P1 P2

Indicate whether each statement best describes the allowance (A) method or the direct write-off (DW) method.

1. Does not predict bad debts expense. 2. Accounts receivable on the balance sheet is reported at net realizable value. 3. The write-off of a specific account does not affect net income. 4. When an account is written off, the debit is to Bad Debts Expense. 5. Usually does not best match sales and expenses because bad debts expense is not recorded

until an account becomes uncollectible, which usually occurs in a period after the credit sale. 6. Estimates bad debts expense related to the sales recorded in that period.

QS 7-5 Allowance method for bad debts

P2

Gomez Corp. uses the allowance method to account for uncollectibles. On January 31, it wrote off an $800 account of a customer, C. Green. On March 9, it receives a $300 payment from Green. 1. Prepare the journal entry for January 31. 2. Prepare the journal entries for March 9; assume no additional money is expected from Green.

QS 7-6 Reporting allowance for doubtful accounts

P2

On December 31 of Swift Co.’s first year, $50,000 of accounts receivable is not yet collected. Swift esti- mates that $2,000 of its accounts receivable is uncollectible and recorded the year-end adjusting entry. 1. Compute the realizable value of accounts receivable reported on Swift’s year-end balance sheet. 2. On January 1 of Swift’s second year, it writes off a customer’s account for $300. Compute the realiz-

able value of accounts receivable on January 1 after the write-off.

QS 7-7 Percent of accounts receivable method

P3

Warner Company’s year-end unadjusted trial balance shows accounts receivable of $99,000, allowance for doubtful accounts of $600 (credit), and sales of $280,000. Uncollectibles are estimated to be 1.5% of ac- counts receivable. 1. Prepare the December 31 year-end adjusting entry for uncollectibles. 2. What amount would have been used in the year-end adjusting entry if the allowance account had a

year-end unadjusted debit balance of $300?

Chapter 7 Accounting for Receivables 291

Warner Company’s year-end unadjusted trial balance shows accounts receivable of $99,000, allowance for doubtful accounts of $600 (credit), and sales of $140,000. Uncollectibles are estimated to be 1% of sales. Prepare the December 31 year-end adjusting entry for uncollectibles.

QS 7-8 Percent of sales method

P3

On August 2, Jun Co. receives a $6,000, 90-day, 12% note from customer Ryan Albany as payment on his $6,000 account receivable. (1) Compute the maturity date for this note. (2) Prepare Jun’s journal entry for August 2.

QS 7-11 Note receivable

C2

On August 2, Jun Co. receives a $6,000, 90-day, 12% note from customer Ryan Albany as payment on his $6,000 account receivable. Prepare Jun’s journal entry assuming the note is honored by the customer on October 31 of that same year.

QS 7-12 Note receivable honored

P4

On December 1, Daw Co. accepts a $10,000, 45-day, 6% note from a customer. (1) Prepare the year-end adjusting entry to record accrued interest revenue on December 31. (2) Prepare the entry required on the note’s maturity date assuming it is honored.

QS 7-13 Note receivable interest and maturity P4

Record the sale by Balus Company of $125,000 in accounts receivable on May 1. Balus is charged a 2.5% factoring fee.

QS 7-14 Factoring receivables C3

Net Zero Products, a wholesaler of sustainable raw materials, prepares the following aging of receivables analysis. (1) Estimate the balance of the Allowance for Doubtful Accounts using the aging of accounts receivable method. (2) Prepare the adjusting entry to record bad debts expense assuming the unadjusted balance in the Allowance for Doubtful Accounts is a $1,000 credit.

QS 7-9 Aging of receivables method

P3

Days Past Due

Total 0 1 to 30 31 to 60 61 to 90 Over 90

Accounts receivable . . . . . . . . . . . . $115,200 $80,000 $18,000 $7,200 $4,000 $6,000

Percent uncollectible . . . . . . . . . . . 1% 3% 5% 8% 11%

Determine the maturity date and compute interest for each note. QS 7-10 Computing note interest and maturity date

C2 Note Contract Date Principal Interest Rate Period of Note (Term)

1 . . . . . . . . . . . . . March 1 $10,000 6% 60 days

2 . . . . . . . . . . . . . May 15 15,000 8 90 days

3 . . . . . . . . . . . . . October 20 8,000 4 45 days

Selected accounts from Fair Trader Co.’s adjusted trial balance for the year ended December 31 follow. Prepare its income statement.

QS 7-15 Preparing an income statement

P2 P4 C3Factoring fees . . . . . . . . . . . . . . . . . . . . . . . . . . $ 300 Interest revenue . . . . . . . . . . . . . . . . . . . . . . . $ 3,000 Insurance expense . . . . . . . . . . . . . . . . . . . . . . . 4,000 Salaries expense . . . . . . . . . . . . . . . . . . . . . . 22,000

Sales . . . . . . . . . . . . . . . . . . . . . . . . . . . . . . . . . . 50,000 Supplies expense . . . . . . . . . . . . . . . . . . . . . . 200

Rent expense . . . . . . . . . . . . . . . . . . . . . . . . . . . 15,000 Bad debt expense . . . . . . . . . . . . . . . . . . . . . 1,000

Selected accounts from Bennett Co.’s adjusted trial balance for the year ended December 31 follow. Prepare a classified balance sheet. Note: Allowance for doubtful accounts is subtracted from accounts re- ceivable on the company’s balance sheet.

QS 7-16 Preparing a balance sheet

P2 P4 C3

Prepaid rent . . . . . . . . . . . . . . . . . . . . . . . . . . . . $ 1,000 Accounts payable . . . . . . . . . . . . . . . . . . . . . . $2,500

Accounts receivable . . . . . . . . . . . . . . . . . . . . . . 10,000 Allowance for doubtful accounts . . . . . . . . . . 500

Cash . . . . . . . . . . . . . . . . . . . . . . . . . . . . . . . . . . 12,000 Notes payable (due in 10 years) . . . . . . . . . . 6,000

Total equity . . . . . . . . . . . . . . . . . . . . . . . . . . . . . 18,000 Notes receivable (due in 4 years) . . . . . . . . . 4,000

292 Chapter 7 Accounting for Receivables

QS 7-17 Accounts receivable turnover

A1

The following data are for Ruggers Company. Compute and interpret its accounts receivable turnover for the current year (competitors average a turnover of 7.5).

Current Year 1 Year Ago

Accounts receivable, net . . . . . . . . . . . . $153,400 $138,500

Net sales . . . . . . . . . . . . . . . . . . . . . . . . . 861,105 910,600

Exercise 7-2 Accounting for credit card sales

C1

Levine Company uses the perpetual inventory system. Prepare journal entries to record the following credit card transactions of Levine Company.

Apr. 8 Sold merchandise for $8,400 (that had cost $6,000) and accepted the customer’s Suntrust Bank Card. Suntrust charges a 4% fee.

12 Sold merchandise for $5,600 (that had cost $3,500) and accepted the customer’s Continental Card. Continental charges a 2.5% fee.

Exercise 7-3 Sales on store credit card

C1

Z-Mart uses the perpetual inventory system and has its own credit card. Z-Mart charges a per-month inter- est fee for any unpaid balance on its store credit card at each month-end.

Apr. 30 Z-Mart sold merchandise for $1,000 (that had cost $650) and accepted the customer’s Z-Mart store credit card.

May 31 Z-Mart recorded $4 of interest earned from its store credit card as of this month-end.

Exercise 7-4 Direct write-off method

P1

Dexter Company uses the direct write-off method. Prepare journal entries to record the following transactions.

Mar. 11 Dexter determines that it cannot collect $45,000 of its accounts receivable from Leer Co. 29 Leer Co. unexpectedly pays its account in full to Dexter Company. Dexter records its recovery

of this bad debt.

Exercise 7-5 Writing off receivables

P2

On January 1, Wei Company begins the accounting period with a $30,000 credit balance in Allowance for Doubtful Accounts. a. On February 1, the company determined that $6,800 in customer accounts was uncollectible; specifically,

$900 for Oakley Co. and $5,900 for Brookes Co. Prepare the journal entry to write off those two accounts. b. On June 5, the company unexpectedly received a $900 payment on a customer account, Oakley

Company, that had previously been written off in part a. Prepare the entries to reinstate the account and record the cash received.

EXERCISES

Exercise 7-1 Accounts receivable subsidiary ledger; schedule of accounts receivable

C1

Vail Company recorded the following transactions during November.

1. Open a general ledger having T-accounts for Accounts Receivable, Sales, and Sales Returns and Allowances. Also open an accounts receivable subsidiary ledger having a T-account for each of its three customers. Post these entries to both the general ledger and the accounts receivable ledger.

2. Prepare a schedule of accounts receivable (see Exhibit 7.4) and compare its total with the balance of the Accounts Receivable controlling account as of November 30.

Check Accounts Receivable ending balance, $9,301

Nov . 5 Accounts Receivable—Ski Shop . . . . . . . . . . . . . . . . . . . . . . . . 4,615

Sales . . . . . . . . . . . . . . . . . . . . . . . . . . . . . . . . . . . . . . . . . 4,615

10  Accounts Receivable—Welcome Enterprises . . . . . . . . . . . . . . 1,350

Sales . . . . . . . . . . . . . . . . . . . . . . . . . . . . . . . . . . . . . . . . . 1,350

13 Accounts Receivable—Zia Natara . . . . . . . . . . . . . . . . . . . . . . . 832

Sales . . . . . . . . . . . . . . . . . . . . . . . . . . . . . . . . . . . . . . . . . 832

21 Sales Returns and Allowances . . . . . . . . . . . . . . . . . . . . . . . . . 209

Accounts Receivable—Zia Natara . . . . . . . . . . . . . . . . . . 209

30 Accounts Receivable—Ski Shop . . . . . . . . . . . . . . . . . . . . . . . . 2,713

Sales . . . . . . . . . . . . . . . . . . . . . . . . . . . . . . . . . . . . . . . . . 2,713

Chapter 7 Accounting for Receivables 293

Exercise 7-6 Percent of sales method; write-off

P3

At year-end (December 31), Chan Company estimates its bad debts as 1% of its annual credit sales of $487,500. Chan records its bad debts expense for that estimate. On the following February 1, Chan de- cides that the $580 account of P. Park is uncollectible and writes it off as a bad debt. On June 5, Park un- expectedly pays the amount previously written off. Prepare Chan’s journal entries to record the transactions of December 31, February 1, and June 5.

Exercise 7-7 Percent of accounts receivable method

P3

Mazie Supply Co. uses the percent of accounts receivable method. On December 31, it has outstanding accounts receivable of $55,000, and it estimates that 2% will be uncollectible. Prepare the year-end adjusting entry to record bad debts expense under the assumption that the Allowance for Doubtful Accounts has (a) a $415 credit balance before the adjustment and (b) a $291 debit balance before the adjustment.

Exercise 7-8 Aging of receivables method

P3

Daley Company prepared the following aging of receivables analysis at December 31.

Days Past Due

Total 0 1 to 30 31 to 60 61 to 90 Over 90

Accounts receivable . . . . . . . . . . . . $570,000 $396,000 $90,000 $36,000 $18,000 $30,000

Percent uncollectible . . . . . . . . . . . 1% 2% 5% 7% 10%

a. Estimate the balance of the Allowance for Doubtful Accounts using aging of accounts receivable. b. Prepare the adjusting entry to record bad debts expense using the estimate from part a. Assume the

unadjusted balance in the Allowance for Doubtful Accounts is a $3,600 credit. c. Prepare the adjusting entry to record bad debts expense using the estimate from part a. Assume the

unadjusted balance in the Allowance for Doubtful Accounts is a $100 debit.

Exercise 7-11 Estimating bad debts

P3

At December 31, Folgeys Coffee Company reports the following results for its calendar year.

Cash sales . . . . . . . . . . . . . . . . . . . . . . . . . . $900,000 Credit sales . . . . . . . . . . . . . . . . . . . . . . . . $300,000

Exercise 7-9 Percent of receivables method

P3

Refer to the information in Exercise 7-8 to complete the following requirements. a. Estimate the balance of the Allowance for Doubtful Accounts assuming the company uses 4.5% of

total accounts receivable to estimate uncollectibles, instead of the aging of receivables method. b. Prepare the adjusting entry to record bad debts expense using the estimate from part a. Assume the

unadjusted balance in the Allowance for Doubtful Accounts is a $12,000 credit. c. Prepare the adjusting entry to record bad debts expense using the estimate from part a. Assume the

unadjusted balance in the Allowance for Doubtful Accounts is a $1,000 debit.

Exercise 7-10 Aging of receivables schedule

P3

Following is a list of credit customers along with their amounts owed and the days past due at December 31. Following that list are five classifications of accounts receivable and estimated bad debts percent for each class. 1. Create an aging of accounts receivable schedule similar to Exhibit 7.8 and calculate the estimated bal-

ance for the Allowance for Doubtful Accounts. 2. Assuming an unadjusted credit balance of $100, record the required adjustment to the Allowance for

Doubtful Accounts.

Customer Accounts Receivable Days Past Due

BCC Company

Lannister Co.

Mike Properties

Ted Reeves

Jen Ste�ens

$4,000

1,000

5,000

500

2,000

12

0

107

72

35

Days Past Due 0 1 to 30 31 to 60 61 to 90 Over 90

Percent uncollectible . . . . . . . . . 1% 3% 5% 8% 12%

294 Chapter 7 Accounting for Receivables

Its year-end unadjusted trial balance includes the following items.

Accounts receivable . . . . . . . . . . . $125,000 debit Allowance for doubtful accounts . . . . . $5,000 debit

1. Prepare the adjusting entry to record bad debts expense assuming uncollectibles are estimated to be 3% of credit sales.

2. Prepare the adjusting entry to record bad debts expense assuming uncollectibles are estimated to be 1% of total sales.

3. Prepare the adjusting entry to record bad debts expense assuming uncollectibles are estimated to be 6% of year-end accounts receivable.

Check Dr. Bad Debts Expense: (1) $9,000

(3) $12,500

Exercise 7-12 Notes receivable transactions C2

Prepare journal entries for the following transactions of Danica Company.

Dec. 13 Accepted a $9,500, 45-day, 8% note in granting Miranda Lee a time extension on her past-due account receivable.

31 Prepared an adjusting entry to record the accrued interest on the Lee note.Check Dec. 31, Cr. Interest Revenue, $38

Prepare journal entries to record transactions for Vitalo Company.

Nov. 1 Accepted a $6,000, 180-day, 8% note from Kelly White in granting a time extension on her past-due account receivable.

Dec. 31 Adjusted the year-end accounts for the accrued interest earned on the White note. Apr. 30 White honored her note when presented for payment.

Exercise 7-14 Honoring a note

P4

Prepare journal entries to record the following transactions of Ridge Company.

Mar. 21 Accepted a $9,500, 180-day, 8% note from Tamara Jackson in granting a time extension on her past-due account receivable.

Sep. 17 Jackson dishonored her note. Dec. 31 After trying several times to collect, Ridge Company wrote off Jackson’s account against the

Allowance for Doubtful Accounts.

Exercise 7-15 Dishonoring a note

P4

On November 30, Petrov Co. has $128,700 of accounts receivable and uses the perpetual inventory sys- tem. (1) Prepare journal entries to record the following transactions. (2) Which transaction would most likely require a note to the financial statements?

Dec. 4 Sold $7,245 of merchandise (that had cost $5,000) to customers on credit, terms n/30. 9 Sold $20,000 of accounts receivable to Main Bank. Main charges a 4% factoring fee. 17 Received $5,859 cash from customers in payment on their accounts. 27 Borrowed $10,000 cash from Main Bank, pledging $12,500 of accounts receivable as security

for the loan.

Exercise 7-16 Selling and pledging accounts receivable

C3

Refer to the information in Exercise 7-12 and prepare the journal entries for the following year for Danica Company.

Jan. 27 Received Lee’s payment for principal and interest on the note dated December 13. Mar. 3 Accepted a $5,000, 10%, 90-day note in granting a time extension on the past-due account

receivable of Tomas Company. 17 Accepted a $2,000, 30-day, 9% note in granting H. Cheng a time extension on his past-due

account receivable. Apr. 16 Cheng dishonored his note. May 1 Wrote off the Cheng account against the Allowance for Doubtful Accounts. June 1 Received the Tomas payment for principal and interest on the note dated March 3.

Check Jan. 27, Dr. Cash, $9,595

June 1, Dr. Cash, $5,125

Exercise 7-13 Notes receivable transactions P4

The following information is from the annual financial statements of Raheem Company. (1) Compute its accounts receivable turnover for Year 2 and Year 3. (2) Assuming its competitor has a turnover of 11, is Raheem performing better or worse at collecting receivables than its competitor?

Exercise 7-17 Accounts receivable turnover

A1 Year 3 Year 2 Year 1

Net sales . . . . . . . . . . . . . . . . . . . . . . . . . . . . . . . . . . . $405,140 $335,280 $388,000

Accounts receivable, net (year-end) . . . . . . . . . . . . . 44,800 41,400 34,800

Chapter 7 Accounting for Receivables 295

PROBLEM SET A

Problem 7-1A Sales on account and credit card sales

C1

Mayfair Co. completed the following transactions and uses a perpetual inventory system.

June 4 Sold $650 of merchandise on credit (that had cost $400) to Natara Morris, terms n∕15. 5 Sold $6,900 of merchandise (that had cost $4,200) to customers who used their Zisa cards. Zisa

charges a 3% fee. 6 Sold $5,850 of merchandise (that had cost $3,800) to customers who used their Access cards.

Access charges a 2% fee. 8 Sold $4,350 of merchandise (that had cost $2,900) to customers who used their Access cards.

Access charges a 2% fee. 13 Wrote off the account of Abigail McKee against the Allowance for Doubtful Accounts. The

$429 balance in McKee’s account was from a credit sale last year. 18 Received Morris’s check in full payment for the June 4 purchase.

Required

Prepare journal entries to record the preceding transactions and events.

Check June 18, Dr. Cash, $650

Problem 7-2A Estimating and reporting bad debts

P2 P3

Required

1. Prepare the adjusting entry to record bad debts under each separate assumption. a. Bad debts are estimated to be 1.5% of credit sales. b. Bad debts are estimated to be 1% of total sales. c. An aging analysis estimates that 5% of year-end accounts receivable are uncollectible. 2. Show how Accounts Receivable and the Allowance for Doubtful Accounts appear on its December 31

balance sheet given the facts in part 1a. 3. Show how Accounts Receivable and the Allowance for Doubtful Accounts appear on its December 31

balance sheet given the facts in part 1c.

Check Bad Debts Expense: (1a) $85,230, (1c) $80,085

At December 31, Hawke Company reports the following results for its calendar year.

In addition, its unadjusted trial balance includes the following items.

Cash sales . . . . . . . . . . . . . . . . . . . $1,905,000 Credit sales . . . . . . . . . . . . . . . . . . . . . . $5,682,000

Accounts receivable . . . . . . . . . . . $1,270,100 debit Allowance for doubtful accounts . . . . . $16,580 debit

On December 31, Jarden Co.’s Allowance for Doubtful Accounts has an unadjusted credit balance of $14,500. Jarden prepares a schedule of its December 31 accounts receivable by age.

Problem 7-3A Aging accounts receivable and accounting for bad debts

P2 P31 2

3

4

5

6

7

A B C Age of

Accounts Receivable Expected Percent

Uncollectible Accounts

Receivable

Not yet due 1 to 30 days past due

31 to 60 days past due

61 to 90 days past due

Over 90 days past due

1.25% 2.00

6.50

32.75

68.00

$830,000 254,000

86,000

38,000

12,000

Required

1. Compute the required balance of the Allowance for Doubtful Accounts at December 31 using an aging of accounts receivable.

2. Prepare the adjusting entry to record bad debts expense at December 31.

Analysis Component

3. On June 30 of the next year, Jarden concludes that a customer’s $4,750 receivable is uncollectible and the account is written off. Does this write-off directly affect Jarden’s net income?

Check (2) Dr. Bad Debts Expense, $27,150

296 Chapter 7 Accounting for Receivables

Liang Company began operations in Year 1. During its first two years, the company completed a number of transactions involving sales on credit, accounts receivable collections, and bad debts. These transac- tions are summarized as follows.

Year 1

a. Sold $1,345,434 of merchandise (that had cost $975,000) on credit, terms n∕30. b. Wrote off $18,300 of uncollectible accounts receivable. c. Received $669,200 cash in payment of accounts receivable. d. In adjusting the accounts on December 31, the company estimated that 1.5% of accounts receivable

would be uncollectible.

Year 2

e. Sold $1,525,634 of merchandise on credit (that had cost $1,250,000), terms n∕30. f. Wrote off $27,800 of uncollectible accounts receivable. g. Received $1,204,600 cash in payment of accounts receivable. h. In adjusting the accounts on December 31, the company estimated that 1.5% of accounts receivable

would be uncollectible.

Required

Prepare journal entries to record Liang’s summarized transactions and its year-end adjustments to record bad debts expense. (The company uses the perpetual inventory system, and it applies the allowance method for its accounts receivable. Round to the nearest dollar.)

Check (d) Dr. Bad Debts Expense, $28,169

(h) Dr. Bad Debts Expense, $32,199

Problem 7-4A Accounts receivable transactions and bad debts adjustments

C1 P2 P3

The following transactions are from Ohlm Company.

Year 1

Dec. 16 Accepted a $10,800, 60-day, 8% note in granting Danny Todd a time extension on his past-due account receivable.

31 Made an adjusting entry to record the accrued interest on the Todd note.

Year 2

Feb. 14 Received Todd’s payment of principal and interest on the note dated December 16. Mar. 2 Accepted a $6,100, 8%, 90-day note in granting a time extension on the past-due account re-

ceivable from Midnight Co. 17 Accepted a $2,400, 30-day, 7% note in granting Ava Privet a time extension on her past-due

account receivable. Apr. 16 Privet dishonored her note. May 31 Midnight Co. dishonored its note. Aug. 7 Accepted a $7,440, 90-day, 10% note in granting a time extension on the past-due account re-

ceivable of Mulan Co. Sep. 3 Accepted a $2,100, 60-day, 10% note in granting Noah Carson a time extension on his past-due

account receivable. Nov. 2 Received payment of principal plus interest from Carson for the September 3 note. Nov. 5 Received payment of principal plus interest from Mulan for the August 7 note. Dec. 1 Wrote off the Privet account against the Allowance for Doubtful Accounts.

Required

1. Prepare journal entries to record these transactions and events.

Analysis Component

2. If Ohlm pledged its receivables as security for a loan from the bank, where on the financial statements does it disclose this pledge of receivables?

Check Feb. 14, Cr. Interest Revenue, $108

May 31, Cr. Interest Revenue, $122

Nov. 2, Cr. Interest Revenue, $35

Problem 7-5A Analyzing and journalizing notes receivable transactions

C2 C3 P4

Chapter 7 Accounting for Receivables 297

PROBLEM SET B

Problem 7-1B Sales on account and credit card sales C1

Archer Co. completed the following transactions and uses a perpetual inventory system.

Aug. 4 Sold $3,700 of merchandise on credit (that had cost $2,000) to McKenzie Carpenter, terms n∕10.

10 Sold $5,200 of merchandise (that had cost $2,800) to customers who used their Commerce Bank credit cards. Commerce charges a 3% fee.

11 Sold $1,250 of merchandise (that had cost $900) to customers who used their Goldman cards. Goldman charges a 2% fee.

14 Received Carpenter’s check in full payment for the August 4 purchase. 15 Sold $3,250 of merchandise (that had cost $1,758) to customers who used their Goldman cards.

Goldman charges a 2% fee. 22 Wrote off the account of Craw Co. against the Allowance for Doubtful Accounts. The $498 bal-

ance in Craw Co.’s account was from a credit sale last year.

Required

Prepare journal entries to record the preceding transactions and events.

Check Aug. 14, Dr. Cash, $3,700

Problem 7-2B Estimating and reporting bad debts

P2 P3

Required

1. Prepare the adjusting entry to record bad debts under each separate assumption. a. Bad debts are estimated to be 2.5% of credit sales. b. Bad debts are estimated to be 1.5% of total sales. c. An aging analysis estimates that 6% of year-end accounts receivable are uncollectible. 2. Show how Accounts Receivable and the Allowance for Doubtful Accounts appear on its December 31

balance sheet given the facts in part 1a. 3. Show how Accounts Receivable and the Allowance for Doubtful Accounts appear on its December 31

balance sheet given the facts in part 1c.

Check Dr. Bad Debts Expense: (1b) $35,505, (1c) $27,000

At December 31, Ingleton Company reports the following results for the year.

In addition, its unadjusted trial balance includes the following items.

Cash sales . . . . . . . . . . . . . . . . . . . $1,025,000 Credit sales . . . . . . . . . . . . . . . . . . . . . . $1,342,000

Accounts receivable . . . . . . . . . . . $575,000 debit Allowance for doubtful accounts . . . . . $7,500 credit

At December 31, Hovak Co.’s Allowance for Doubtful Accounts has an unadjusted debit balance of $3,400. Hovak prepares a schedule of its December 31 accounts receivable by age.

Problem 7-3B Aging accounts receivable and accounting for bad debts

P2 P3

Required

1. Compute the required balance of the Allowance for Doubtful Accounts at December 31 using an aging of accounts receivable.

2. Prepare the adjusting entry to record bad debts expense at December 31.

Analysis Component

3. On July 31 of the following year, Hovak concludes that a customer’s $3,455 receivable is uncollectible and the account is written off. Does this write-off directly affect Hovak’s net income?

Check (2) Dr. Bad Debts Expense, $31,390

Not yet due

1 to 30 days past due

31 to 60 days past due

61 to 90 days past due

Over 90 days past due

Age of Accounts Receivable

Expected Percent Uncollectible

Accounts Receivable

$396,400

277,800

48,000

6,600

2,800

2.0%

4.0

8.5

39.0

82.0

1

2

3

4

5

6

7

A B C

298 Chapter 7 Accounting for Receivables

Sherman Co. began operations in Year 1. During its first two years, the company completed several trans- actions involving sales on credit, accounts receivable collections, and bad debts. These transactions are summarized as follows.

Year 1

a. Sold $685,350 of merchandise on credit (that had cost $500,000), terms n∕30. b. Received $482,300 cash in payment of accounts receivable. c. Wrote off $9,350 of uncollectible accounts receivable. d. In adjusting the accounts on December 31, the company estimated that 1% of accounts receivable

would be uncollectible.

Year 2

e. Sold $870,220 of merchandise on credit (that had cost $650,000), terms n∕30. f. Received $990,800 cash in payment of accounts receivable. g. Wrote off $11,090 of uncollectible accounts receivable. h. In adjusting the accounts on December 31, the company estimated that 1% of accounts receivable

would be uncollectible.

Required

Prepare journal entries to record Sherman’s summarized transactions and its year-end adjusting entries to record bad debts expense. (The company uses the perpetual inventory system, and it applies the allowance method for its accounts receivable.)

Check (d) Dr. Bad Debts Expense, $11,287

(h) Dr. Bad Debts Expense, $9,773

Problem 7-4B Accounts receivable transactions and bad debts adjustments

C1 P2 P3

The following transactions are from Springer Company.

Year 1

Nov. 1 Accepted a $4,800, 90-day, 8% note in granting Steve Julian a time extension on his past-due account receivable.

Dec. 31 Made an adjusting entry to record the accrued interest on the Julian note.

Year 2

Jan. 30 Received Julian’s payment for principal and interest on the note dated November 1. Feb. 28 Accepted a $12,600, 30-day, 8% note in granting a time extension on the past-due account

receivable from King Co. Mar. 1 Accepted a $6,200, 60-day, 12% note in granting Myron Shelley a time extension on his

past-due account receivable. 30 The King Co. dishonored its note. Apr. 30 Received payment of principal plus interest from M. Shelley for the March 1 note. June 15 Accepted a $2,000, 72-day, 8% note in granting a time extension on the past-due account

receivable of Ryder Solon. 21 Accepted a $9,500, 90-day, 8% note in granting J. Felton a time extension on his past-due

account receivable. Aug. 26 Received payment of principal plus interest from R. Solon for the June 15 note. Sep. 19 Received payment of principal plus interest from J. Felton for the June 21 note. Nov. 30 Wrote off King’s account against the Allowance for Doubtful Accounts.

Required

1. Prepare journal entries to record these transactions and events.

Analysis Component

2. If Springer pledged its receivables as security for a loan from the bank, where on the financial state- ments does it disclose this pledge of receivables?

Check Jan. 30, Cr. Interest Revenue, $32

Apr. 30, Cr. Interest Revenue, $124

Sep. 19, Cr. Interest Revenue, $190

Problem 7-5B Analyzing and journalizing notes receivable transactions

C2 C3 P4

SERIAL PROBLEM Business Solutions

P1 P2

This serial problem began in Chapter 1 and continues through most of the book. If previous chapter seg- ments were not completed, the serial problem can begin at this point.

SP 7 Santana Rey, owner of Business Solutions, realizes that she needs to begin accounting for bad debts expense. Assume that Business Solutions has total revenues of $44,000 during the first three months of 2020 and that the Accounts Receivable balance on March 31, 2020, is $22,867.

Chapter 7 Accounting for Receivables 299

Required

1. Prepare the adjusting entry to record bad debts expense on March 31, 2020, under each separate assumption. There is a zero unadjusted balance in the Allowance for Doubtful Accounts at March 31.

a. Bad debts are estimated to be 1% of total revenues. b. Bad debts are estimated to be 2% of accounts receivable. (Round to the dollar.) 2. Assume that Business Solutions’s Accounts Receivable balance at June 30, 2020, is $20,250 and that

one account of $100 has been written off against the Allowance for Doubtful Accounts since March 31, 2020. If Rey uses the method in part 1b, what adjusting journal entry is made to recognize bad debts expense on June 30, 2020?

3. Should Rey consider adopting the direct write-off method of accounting for bad debts expense rather than one of the allowance methods considered in part 1? Explain. ©Alexander Image/Shutterstock

Check (2) Dr. Bad Debts Expense, $48

GENERAL LEDGER PROBLEM

The General Ledger tool in Connect automates several of the procedural steps in accounting so that the financial professional can focus on the impacts of each transaction on various financial reports and perfor- mance measures.

GL 7-1 General Ledger assignment GL 7-1, based on Problem 7-5A, focuses on transactions related to accounts and notes receivable and highlights the impact each transaction has on interest revenue.

GL

COMPANY ANALYSIS A1

Accounting Analysis

AA 7-1 Use Apple’s financial statements in Appendix A to answer the following. 1. What is the amount of Apple’s accounts receivable as of September 30, 2017? 2. Compute Apple’s accounts receivable turnover as of September 30, 2017. 3. How long does it take, on average, for the company to collect receivables for the fiscal year ended

September 30, 2017? 4. Apple’s most liquid assets include (a) cash and cash equivalents, (b) short-term marketable securities,

(c) accounts receivable, and (d ) inventory. Compute the percentage that these liquid assets (in total) make up of current liabilities as of September 30, 2017, and as of September 24, 2016.

5. Did Apple’s liquid assets as a percentage of current liabilities improve or worsen as of its fiscal 2017 year-end compared to its fiscal 2016 year-end?

APPLE

AA 7-2 Comparative figures for Apple and Google follow.

Apple Google

Current One Year Two Years Current One Year Two Years $ millions Year Prior Prior Year Prior Prior

Accounts receivable, net . . $ 17,874 $ 15,754 $ 16,849 $ 18,336 $14,137 $11,556

Net sales . . . . . . . . . . . . . . . 229,234 215,639 233,715 110,855 90,272 74,989

COMPARATIVE ANALYSIS A1 P2

APPLE GOOGLE

Required

1. Compute the accounts receivable turnover for (a) Apple and (b) Google for each of the two most recent years using the data shown.

2. Compute how many days, on average, it takes to collect receivables for the two most recent years for (a) Apple and (b) Google.

3. Which company more quickly collects its accounts receivable in the current year?

Hint: Average collection period equals 365 divided by the accounts receivable turnover.

300 Chapter 7 Accounting for Receivables

1. Compute its accounts receivable turnover for the current year. 2. How long does it take on average for Samsung to collect receivables in the current year? 3. In the current year, does Samsung’s accounts receivable turnover underperform or outperform the in-

dustry (assumed) average of 7?

AA 7-3 Key figures for Samsung follow.

₩ millions Current Year One Year Prior Two Years Prior

Accounts receivable, net . . . . . . . . . . . . ₩ 27,695,995 ₩ 24,279,211 ₩ 25,168,026

Sales . . . . . . . . . . . . . . . . . . . . . . . . . . . . 239,575,376 201,866,745 200,653,482

GLOBAL ANALYSIS C1 A1

Samsung

ETHICS CHALLENGE P2 P3

BTN 7-1 Anton Blair is the manager of a medium-size company. A few years ago, Blair persuaded the owner to base a part of his compensation on the net income the company earns each year. Each December he estimates year-end financial figures in anticipation of the bonus he will receive. If the bonus is not as high as he would like, he offers several recommendations to the accountant for year-end adjustments. One of his favorite recommendations is for the controller to reduce the estimate of doubtful accounts.

Required

1. What effect does lowering the estimate for doubtful accounts have on the income statement and bal- ance sheet?

2. Do you believe Blair’s recommendation to adjust the allowance for doubtful accounts is within his rights as manager, or do you believe this action is an ethics violation? Justify your response.

3. What type of internal control(s) might be useful for this company in overseeing the manager’s recom- mendations for accounting changes?

Beyond the Numbers

BTN 7-2 As the accountant for Pure-Air Distributing, you attend a sales managers’ meeting devoted to a discussion of credit policies. At the meeting, you report that bad debts expense is estimated to be $59,000 and accounts receivable at year-end amount to $1,750,000 less a $43,000 allowance for doubtful accounts. Sid Omar, a sales manager, expresses confusion over why bad debts expense and the allowance for doubt- ful accounts are different amounts. Write a one-page memorandum to him explaining why a difference in bad debts expense and the allowance for doubtful accounts is not unusual. The company estimates bad debts expense as 2% of sales.

COMMUNICATING IN PRACTICE P2 P3

BTN 7-3 Access eBay’s February 6, 2017, filing of its 10-K report for the year ended December 31, 2016, at SEC.gov.

Required

1. What is the amount of eBay’s net accounts receivable at December 31, 2016, and at December 31, 2015?

2. “Financial Statement Schedule II” of its 10-K report lists eBay’s allowance for doubtful accounts (including authorized credits). For the two years ended December 31, 2016 and 2015, identify its allowance for doubtful accounts (including authorized credits), and then compute it as a percent of gross accounts receivable.

3. Do you believe that these percentages are reasonable based on what you know about eBay? Explain.

TAKING IT TO THE NET C1 P3

BTN 7-4 Each member of a team is to participate in estimating uncollectibles using the aging schedule and percents shown in Problem 7-3A. The division of labor is up to the team. Your goal is to accurately complete this task as soon as possible. After estimating uncollectibles, check your estimate with the instructor. If the estimate is correct, the team then should prepare the adjusting entry and the presentation of accounts receivable (net) for the December 31 year-end balance sheet.

TEAMWORK IN ACTION P2 P3

Chapter 7 Accounting for Receivables 301

BTN 7-5 Sheryl Sandberg and Mark Zuckerberg of Facebook are introduced in the chapter’s opening feature. Assume that they are considering two options.

Plan A. Facebook would begin selling access to a premium version of its website. The new online cus- tomers would use their credit cards. The company has the capability of selling the premium service with no additional investment in hardware or software. Annual credit sales are expected to increase by $250,000.

Costs associated with Plan A: Additional wages related to these new sales are $135,500; credit card fees will be 4.75% of sales; and additional recordkeeping costs will be 6% of sales. Premium service sales will reduce advertising revenues for Facebook by $8,750 annually because some customers will now only use the premium service.

Plan B. The company would begin selling Facebook merchandise. It would make additional annual credit sales of $500,000.

Costs associated with Plan B: Cost of these new sales is $375,000; additional recordkeeping and ship- ping costs will be 4% of sales; and uncollectible accounts will be 6.2% of sales.

Required

1. Compute the additional annual net income or loss expected under (a) Plan A and (b) Plan B. 2. Should the company pursue either plan? Discuss both the financial and nonfinancial factors relevant

to this decision.

Check (1b) Additional net income, $74,000

ENTREPRENEURIAL DECISION C1

BTN 7-6 Many commercials include comments similar to the following: “We accept VISA” or “We do not accept American Express.” Conduct your own research by contacting at least five companies via interviews, phone calls, or the Internet to determine the reason(s) companies discriminate in their use of credit cards. Collect information on the fees charged by the different cards for the companies contacted. (The instructor can assign this as a team activity.)

HITTING THE ROAD C1

Design elements: Lightbulb: ©Chuhail/Getty Images; Blue globe: ©nidwlw/Getty Images and ©Dizzle52/Getty Images; Chess piece: ©Andrei Simonenko/Getty Images and ©Dizzle52/Getty Images; Mouse: ©Siede Preis/Getty Images; Global View globe: ©McGraw-Hill Education and ©Dizzle52/Getty Images; Sustainability: ©McGraw-Hill Education and ©Dizzle52/Getty Images

Learning Objectives

CONCEPTUAL C1 Compute the cost of plant assets.

C2 Explain depreciation for partial years and changes in estimates.

C3 Distinguish between revenue and capital expenditures, and account for them.

P2 Account for asset disposal through discarding or selling an asset.

P3 Account for natural resource assets and their depletion.

P4 Account for intangible assets.

P5 Appendix 8A—Account for asset exchanges.

ANALYTICAL A1 Compute total asset turnover and apply

it to analyze a company’s use of assets.

PROCEDURAL P1 Compute and record depreciation using

the straight-line, units-of-production, and declining-balance methods.

Chapter Preview

8 Accounting for Long-Term Assets

NTK 8-5

INTANGIBLE ASSETS

P4 Cost determination Amortization

Types of intangibles

A1 Analyze asset usage

NTK 8-1, 8-2, 8-3

PLANT ASSETS

C1 Cost determination P1 Depreciation C2 Partial years and changes

in estimates

C3 Additional expenditures P2 Disposal

NTK 8-4

NATURAL RESOURCES

P3 Cost determination Depletion

Presentation

Plant assets tied into extracting resources

303

“Strive to surpass yourself”—Deb Carey

Crafting the Dream

NEW GLARUS, WI—Deb Carey told her husband Dan, “I could start a brewery and you could work for me.” A few days later, she recalls, “We were bidding on equipment from a brew pub.” Dan reminded her, “But we don’t have any money.” Deb declared, “I’m going to sell the house!” Soon, she says, New Glarus Brewing (NewGlarusBrewing.com) was up and running.

“In that first year,” explains Deb, “we had no money, and we were working from 5 a.m. to midnight.” Deb focused on the busi- ness. She stresses that long-term assets in the brewery such as brew houses, packaging lines, and fermentation cellars are ex- pensive but key to success. Financing that equipment, buildings, and other assets, she says, is not easy.

A constant challenge for Deb and Dan is maintaining the right kind and amount of assets to meet business demands and be profitable. “Machinery cannot be divorced from the process,” insists Dan. “You have to work with the strengths and weak- nesses of your machinery.”

Deb explains that success depends on monitoring and control- ling the types and costs of long-term assets. Each of her tangible and intangible assets commands Deb’s attention. She accounts for, manages, and focuses on recovering all costs of those acquisitions.

Their company is on a roll—employing nearly 150 workers, offering unique products such as Spotted Cow, and generating over 250,000 barrels. Adds Deb, running a company “is like having a big family.”

Sources: New Glarus Brewing website, January 2019; Wisconsin State Journal, July 2011; NBC 26 Green Bay, February 2018; Daily Dose, October 2017

©Casper Hedberg/Bloomberg/Getty Images

Plant assets are tangible assets used in a company’s operations that have a useful life of more than one accounting period. Plant assets are also called plant and equipment; property, plant and equipment (PP&E); or fixed assets. Exhibit 8.1 shows plant assets as a percentage of total assets for several companies.

Plant assets are set apart from other assets by two important features. First, plant assets are used in opera- tions. A computer purchased to resell is reported on the balance sheet as inventory. If the same computer is used in operations, it is a plant asset. Another example is land held for expansion, which is reported as a long-term investment. Instead, if this land holds a factory used in operations, the land is a plant asset.

The second important feature is that plant assets have useful lives extending over more than one accounting period. This makes plant assets dif- ferent from current assets such as supplies that are normally used up within one period.

Exhibit 8.2 shows four issues in accounting for plant assets: (1) computing the costs of plant assets, (2) allocating the costs of plant assets, (3) accounting for subsequent expenditures to plant assets, and (4) recording the disposal of plant assets. The following sections discuss these issues.

Section 1—Plant Assets

As a Percentage of Total Assets 0% 20% 40% 60% 80%

McDonald's

eBay 6%$1,516 mil.

Boston Beer 66%$408 mil.

Walmart 57%$114,178 mil.

$21,258 mil. 69%

EXHIBIT 8.1 Plant Assets of Selected Companies

Point: Capital-intensive refers to companies with large amounts of plant assets.

Acquisition 1. Compute cost.

Disposal 4. Record disposal.

2. Allocate cost to periods benefited. 3. Account for subsequent expenditures.

Use

Decline in asset book value over its useful life

For Sale

EXHIBIT 8.2 Issues in Accounting for Plant Assets

304 Chapter 8 Accounting for Long-Term Assets

C1 Compute the cost of plant assets.

Plant assets are recorded at cost when acquired. Cost includes all expenditures necessary to get an asset in place and ready for use. The cost of a machine, for example, includes its invoice cost minus any discount, plus necessary shipping, assembling, installing, and testing costs. Exam- ples are the costs of building a base for a machine, installing electrical hookups, and testing the asset before using it in operations.

To be recorded as part of the cost of a plant asset, an expenditure must be normal, reasonable, and necessary in preparing it for its intended use. If an asset is damaged during unpacking, the repairs are not added to its cost. Instead, they are charged to an expense account. Costs to mod- ify or customize a new plant asset are added to the asset’s cost. This section explains how to determine the cost of plant assets for its four major classes.

Machinery and Equipment The costs of machinery and equipment consist of all costs normal and necessary to purchase them and prepare them for their intended use. These include the purchase price, taxes, transpor- tation charges, insurance while in transit, and the installing, assembling, and testing of the machinery and equipment.

Buildings A Building account consists of the costs of purchasing or constructing a building that is used in operations. A purchased building’s costs include its purchase price, taxes, title fees, and lawyer fees. Its costs also include all expenditures to ready it for its intended use, including necessary repairs or renovations. When a company constructs a building or any plant asset for its own use, its costs include materials and labor plus indirect overhead cost. Overhead includes heat, lighting, power, and depreciation on machinery used to construct the asset. Costs of construction also include design fees, building permits, and insurance during construction. However, costs such as insurance to cover the asset after it is being used are operating expenses.

Land Improvements Land improvements are additions to land and have limited useful lives. Examples are parking lots, driveways, walkways, fences, and lighting systems. Land improvements include costs nec- essary to make those improvements ready for their intended use.

Land Land is the earth’s surface and has an indefinite (unlimited) life. Land includes costs necessary to make it ready for its intended use. When land is purchased for a building site, its cost includes the total amount paid for the land, including real estate commissions, title insurance fees, legal fees, and any accrued property taxes paid by the purchaser. Payments for surveying, clearing, grading, and draining also are included in the cost of land. Other costs include government assessments, whether incurred at the time of purchase or later, for items such as public roads, sewers, and sidewalks. These assessments are included because they permanently add to the land’s value (and are not depreciated as they are not the company’s responsibility). Land pur- chased as a building site can include unwanted structures. The cost of removing those struc- tures, less amounts recovered through sale of salvaged materials, is charged to the Land account.

Assume Starbucks paid $167,000 cash to acquire land for a coffee shop. This land had an old service garage that was removed at a net cost of $13,000 ($15,000 in costs less $2,000 pro- ceeds from salvaged materials). Additional closing costs total $10,000, consisting of brokerage fees ($8,000), legal fees ($1,500), and title costs ($500). The cost of this land to Starbucks is $190,000 and is computed as shown in Exhibit 8.3.

Point: Entry for cash purchase of land improvements: Land Improvements . . . . . . . . . . # Cash . . . . . . . . . . . . . . . . . . . . #

COST DETERMINATION

©Syda Productions/Shutterstock

Chapter 8 Accounting for Long-Term Assets 305

Lump-Sum Purchase Plant assets sometimes are purchased as a group in a single transaction for a lump-sum price. This transaction is called a lump-sum purchase, or group, bulk, or basket purchase. When this occurs, we allocate the cost to the assets acquired based on their relative market (or appraised) values. Assume CarMax paid $90,000 cash to acquire a group of items consisting of a building appraised at $60,000 and land appraised at $40,000. The $90,000 cost is allocated based on appraised values as shown in Exhibit 8.4. The entry to record the lump-sum purchase also is shown in Exhibit 8.4.

EXHIBIT 8.3 Computing and Recording Cost of Land

Cash price of land . . . . . . . . . . . . . . . . . . . . . $  167,000

Net cost of garage removal . . . . . . . . . . . . . 13,000

Closing costs . . . . . . . . . . . . . . . . . . . . . . . . . 10,000

Cost of land . . . . . . . . . . . . . . . . . . . . . . . . . $190,000

Land . . . . . . . . . . . . . . . . . . . . . . 190,000

Cash . . . . . . . . . . . . . . . . . 190,000

Record purchase of land.

Entry for cash purchase of land:

Appraised Value Percent of Total Apportioned Cost

Building . . . $ 60,000 60% ($60,000/$100,000) $54,000 ($90,000 × 60%) Land . . . . . 40,000 40 ($40,000/$100,000) 36,000 ($90,000 × 40%) Totals . . . . $100,000 100% $ 90,000

EXHIBIT 8.4 Computing and Recording Costs in a Lump-Sum Purchase

Building . . . . . . . . . . . . . . 54,000

Land . . . . . . . . . . . . . . . . 36,000

Cash . . . . . . . . . . . . 90,000

Record costs of plant assets.

Entry for lump-sum cash purchase:

Depreciation is the process of allocating the cost of a plant asset to expense while it is in use. Depreciation does not measure the decline in the asset’s market value or its physical deteriora- tion. This section covers computing depreciation.

Factors in Computing Depreciation Factors that determine depreciation are (1) cost, (2) salvage value, and (3) useful life.

Cost The cost of a plant asset consists of all necessary and reasonable expenditures to ac- quire it and to prepare it for its intended use.

Salvage Value The salvage value, also called residual value or scrap value, is an esti- mate of the asset’s value at the end of its useful life. This is the amount the owner expects to receive from disposing of the asset at the end of its useful life. If the asset is expected to be traded in on a new asset, its salvage value is the expected trade-in value.

Useful Life The useful life of a plant asset is the length of time it is used in a company’s operations. Useful life, or service life, might not be as long as the asset’s total productive life. For example, the productive life of a computer can be eight years or more. Some companies, however, trade in old computers for new ones every two years. In this case, these computers

Point: If we expect disposal costs, the salvage value equals the ex- pected amount from disposal less any disposal costs.

DEPRECIATION

P1 Compute and record depreciation using the straight-line, units-of- production, and declining- balance methods.

Point: Useful life and salvage value are estimates.

Compute the recorded cost of a new machine given the following payments related to its purchase: gross purchase price, $700,000; sales tax, $49,000; purchase discount taken, $21,000; freight cost—terms FOB shipping point, $3,500; normal assembly costs, $3,000; cost of necessary machine platform, $2,500; and cost of parts used in maintaining machine, $4,200.

Solution

$737,000 = $700,000 + $49,000 − $21,000 + $3,500 + $3,000 + $2,500

Cost Determination

NEED-TO-KNOW 8-1

C1

Do More: QS 8-1, QS 8-2, E 8-1, E 8-2, E 8-3

306 Chapter 8 Accounting for Long-Term Assets

have a two-year useful life. The useful life of a plant asset is impacted by inadequacy and obso- lescence. Inadequacy is the inability of a plant asset to meet its demands. Obsolescence is the process of becoming outdated and no longer used.

Sweet Life The useful life of plant assets is different for each company. Hershey Foods and Tootsie Roll are com- petitors and apply similar manufacturing processes, but their equipment’s life expectancies are different. Hershey depreciates equipment over 3 to 15 years, but Tootsie Roll depreciates them over 5 to 20 years. Such differences impact financial statements. ■

Decision Insight

Depreciation Methods Depreciation methods are used to allocate a plant asset’s cost over its useful life. The most fre- quently used method is the straight-line method. The units-of-production and double-declining methods are also commonly used. We explain all three methods. Computations in this section use information about a machine used by Reebok and Adidas to inspect athletic shoes before packaging. Data for this machine are in Exhibit 8.5.

©Fuse/Getty Images

EXHIBIT 8.5 Data for Inspection Machine

Cost . . . . . . . . . . . . . . . . . . $10,000

Salvage value . . . . . . . . . . 1,000

Depreciable cost . . . . . . . . $ 9,000

Useful life:

Accounting periods . . . . . . . . 5 years

Units inspected . . . . . . . . . . . 36,000 shoes

Straight-Line Method Straight-line depreciation charges the same amount to each period of the asset’s useful life. A two-step process is used. We first compute the depreciable cost of the asset, also called cost to be depreciated. It is computed as asset total cost minus salvage value. Second, depreciable cost is divided by the number of accounting periods in the asset’s useful life. The computation for the inspection machine is in Exhibit 8.6.

EXHIBIT 8.6 Straight-Line Depreciation Formula and Example

Cost – Salvage value Useful life in periods

= =$10,000 – $1,000 $1,800 per year 5 years

If this machine is purchased on December 31, 2018, and used during its predicted useful life of five years, the straight-line method allocates equal depreciation to each of the years 2019 through 2023. We make the following adjusting entry at the end of each of the five years to record straight-line depreciation.

Point: Excel for SLN.

A B

1 Cost $10,000

2 Salvage $1,000

3 Life 5

4 SLN depr.

=SLN(B1,B2,B3) = $1,800

The $1,800 Depreciation Expense is reported on the income statement. The $1,800 Accumu- lated Depreciation is a contra asset account to the Machinery account on the balance sheet. The left graph in Exhibit 8.7 shows the $1,800 per year expense reported in each of the five years. The right graph shows the Machinery account balance (net) on each of the six December 31 balance sheets.

Dec . 31 Depreciation Expense . . . . . . . . . . . . . . . . . . . . . . . . . . . . . . . . 1,800

Accumulated Depreciation—Machinery . . . . . . . . . . . . . 1,800

Record annual depreciation.

Assets = Liabilities + Equity −1,800 −1,800

Chapter 8 Accounting for Long-Term Assets 307

Depreciation Expense (on Income Statement)

2018 20232019 2020 2021 2022

$1,800

600

1,200

Year-End Book Value (on Balance Sheet)

20232018 2019 2020 2021 2022

$10,000

2,000

6,000$ 1,8

00

$1 ,80

0

$1 ,80

0

$1 ,80

0

$1 ,80

0

$1 0,0

00

$8 ,20

0

$6 ,40

0

$4 ,60

0

$2 ,80

0

$1 ,00

0

EXHIBIT 8.7 Financial Statement Effects of Straight-Line Depreciation

The net balance sheet amount is the asset book value, or book value, and is computed as the asset’s total cost minus accumulated depreciation. For example, at the end of Year 2 (December 31, 2020), its book value is $6,400, which is $10,000 minus $3,600 (2 years × $1,800), and is reported in the balance sheet as follows.

Book value = Cost − Accumulated depreciation

We also can compute the straight-line depreciation rate, which is 100% divided by the num- ber of periods in the asset’s useful life. For the inspection machine, this rate is 20% (100% ÷ 5 years, or 20% per period). We use this rate, along with other information, to compute the machine’s straight-line depreciation schedule shown in Exhibit 8.8. This exhibit shows (1) straight-line depreciation is the same each period, (2) accumulated depreciation is the total of current and prior periods’ depreciation expense, and (3) book value declines each period until it equals salvage value.

Machinery . . . . . . . . . . . . . . . . . . . . $10,000

Less accumulated depreciation . . . 3,600 $6,400 Machinery (net of $3,600 accumulated depreciation) . . . $6,400Book value

OR

Point: Once an asset’s book value equals its salvage value, depreciation stops.

Example: If salvage value of the machine is $2,500, what is the annual depreciation? Answer: ($10,000 − $2,500)/ 5 years = $1,500 per year

EXHIBIT 8.8 Straight-Line Depreciation Schedule

Depreciation for the Period End of Period

Annual Depreciable Depreciation Depreciation Accumulated Book Period Cost* Rate Expense Depreciation Value†

2018 — — — — $10,000

2019 $9,000 20% $1,800 $1,800 8,200 2020 9,000 20 1,800 3,600 6,400 2021 9,000 20 1,800 5,400 4,600 2022 9,000 20 1,800 7,200 2,800 2023 9,000 20 1,800 9,000 1,000 $9,000

*$10,000 − $1,000. †Book value is total cost minus accumulated depreciation.

$10,000 cost − $1,000 salvage

Salvage value is not depreciated .

Units-of-Production Method The use of some plant assets varies greatly from one period to the next. For example, a builder might use a piece of equipment for a month and then not use it again for several months. When equipment use varies from period to period, the units-of-production depreciation method can better match expenses with revenues. Units- of-production depreciation charges a varying amount for each period depending on an asset’s usage.

A two-step process is used. We first compute depreciation per unit as the asset’s total cost minus salvage value and then divide by the total units expected to be produced during its useful life. Units of production can be expressed in product or other units such as hours used or miles driven. The second step is to compute depreciation for the period by multiply- ing the units produced in the period by the depreciation per unit. The computation for the machine described in Exhibit 8.5 is in Exhibit 8.9. Note: 7,000 shoes are inspected and sold in its first year.

308 Chapter 8 Accounting for Long-Term Assets

EXHIBIT 8.10 Units-of-Production Depreciation Schedule

Depreciation for the Period End of Period

Annual Number of Depreciation per Depreciation Accumulated Book Period Units Unit Expense Depreciation Value

2018 — — — — $10,000

2019 7,000 $0 .25 $1,750 $1,750 8,250 2020 8,000 0 .25 2,000 3,750 6,250 2021 9,000 0 .25 2,250 6,000 4,000 2022 7,000 0 .25 1,750 7,750 2,250 2023 5,000 0 .25 1,250 9,000 1,000 36,000 units $9,000$10,000 cost − $1,000 salvage Salvage value is not depreciated .

EXHIBIT 8.11 Double-Declining-Balance Depreciation Formula*

Straight-line rate = 100% ÷ Useful life = 100% ÷ 5 years = 20%

Double-declining-balance rate = 2 × Straight-line rate = 2 × 20% = 40%

Depreciation expense = Double-declining-balance rate × Beginning-period book value 40% × $10,000 = $4,000 (for 2019)

Step 1

Step 2

Step 3

*In simple form: DDB depreciation = (2 × Beginning-period book value)∕Useful life.

$0.25 per shoe × 7,000 shoes = $1,750 Depreciation expense = Depreciation per unit × Units produced in period

Depreciation per unit = Cost – Salvage valueStep 1

Step 2

Total units of production = $10,000 – $1,000

36,000 shoes = $0.25 per shoe EXHIBIT 8.9 Units-of-Production Depreciation Formula and Example

Using data on the number of units inspected (shoes produced) by the machine, we compute the units-of-production depreciation schedule in Exhibit 8.10. For example, depreciation for the first year is $1,750 (7,000 shoes at $0.25 per shoe). Depreciation for the second year is $2,000 (8,000 shoes at $0.25 per shoe). Exhibit 8.10 shows (1) depreciation expense depends on unit output, (2) accumulated depreciation is the total of current and prior periods’ depreciation expense, and (3) book value declines each period until it equals salvage value.

Example: Refer to Exhibit 8.10. If the number of shoes inspected in 2023 is 5,500, what is depreciation for 2023? Answer: $1,250 (never depreci- ate below salvage value)

Declining-Balance Method An accelerated depreciation method has more depre- ciation in the early years and less depreciation in later years. The most common accelerated method is the declining-balance method, which uses a depreciation rate that is a multiple of the straight-line rate. A common depreciation rate is double the straight-line rate. This is called double-declining-balance (DDB). This is done in three steps.

1. Compute the asset’s straight-line depreciation rate. 2. Double the straight-line rate. 3. Compute depreciation by multiplying this rate by the asset’s beginning-period book value.

Let’s return to the machine in Exhibit 8.5 and use double-declining-balance to compute depreciation. Exhibit 8.11 shows the first-year depreciation computation. The three steps are (1) divide 100% by five years to get the straight-line rate of 20%, or 1/5, per year; (2) double this 20% rate to get the declining-balance rate of 40%, or 2/5, per year; and (3) compute depreciation as 40%, or 2/5, multiplied by the beginning-period book value.

SL rate = 100%

Useful life

DDB rate = 200%

Useful life

Point: Excel for DDB.

A B

1 Cost $10,000

2 Salvage $1,000

3 Life 5

4 DDB depr.

5 1

6 2

7 etc.

=DDB(B1,B2,B3,A5) = $4,000

=DDB(B1,B2,B3,A6) = $2,400

Chapter 8 Accounting for Long-Term Assets 309

The double-declining-balance depreciation schedule is in Exhibit 8.12. The schedule follows the formula except for year 2023, when depreciation is $296. This $296 is not equal to 40% × $1,296, or $518.40. If we had used the $518.40 for depreciation in 2023, the ending book value would equal $777.60, which is less than the $1,000 salvage value. Instead, the $296 is computed as $1,296 book value minus $1,000 salvage value (for the year when DDB depreciation cuts into salvage value).

Units CostItem Market Straight-LinePeriod Double-Declining-Balance

2020 2021 2022 2023

Totals

2019

1,800 1,800 1,800

$9,000

1,800 $1,800

Units-of-Production

2,250 1,750 1,250

$9,000

2,000 $1,750

1,440

2,400 $4,000

864 296

$9,000

EXHIBIT 8.13 Depreciation Expense for the Different Methods

Depreciation for Tax Reporting Many companies use accelerated depreciation in computing taxable income. Reporting higher depreciation expense in the early years of an as- set’s life reduces the company’s taxable income in those years and increases it in later years. The goal is to postpone its tax payments. The U.S. tax law has rules for depreciating assets. These rules include the Modified Accelerated Cost Recovery System (MACRS), which allows straight-line depreciation for some assets but requires accelerated depreciation for most kinds of assets. MACRS is not acceptable for financial reporting because it does not consider an asset’s useful life or salvage value.

Partial-Year Depreciation When an asset is purchased or sold at a time other than the beginning or end of an accounting period, depreciation is recorded for part of that period.

Mid-Period Asset Purchase Assume that the machine in Exhibit 8.5 is purchased and placed in service on October 1, 2018, and the annual accounting period ends on December 31. Because this machine is used for three months in 2018, the calendar-year income statement reports depreciation for those three months. Using straight-line depreciation, we compute three months’ depreciation of $450 as follows.

$10,000 − $1,000 5 years

× 3 12

= $450

Most Popular Methods

Declining-balance, 4%

Units-of-production, 5%

Straight-line, 85%

Accelerated and other, 6%

C2 Explain depreciation for partial years and changes in estimates.

EXHIBIT 8.12 Double-Declining-Balance Depreciation Schedule

Depreciation for the Period End of Period

Annual Beginning-of- Depreciation Depreciation Accumulated Book Period Period Book Value Rate Expense Depreciation Value

2018 — — — — $10,000

2019 $10,000 40% $4,000 $4,000 6,000 2020 6,000 40 2,400 6,400 3,600 2021 3,600 40 1,440 7,840 2,160 2022 2,160 40 864 8,704 1,296 2023 1,296 40 296* 9,000 1,000 $9,000

*Year 2023 depreciation is $1,296 − $1,000 = $296 (never depreciate book value below salvage value).

Salvage value is not depreciated .

$10,000 cost − $1,000 salvage

Comparing Depreciation Methods Exhibit 8.13 shows depreciation for each year under the three methods. While depreciation per period differs, total depreciation of $9,000 is the same over the useful life.

Example: What is the DDB depreciation in year 2022 if salvage value is $2,000? Answer: $2,160 − $2,000 = $160

310 Chapter 8 Accounting for Long-Term Assets

Mid-Period Asset Sale Assume that the machine above is sold on June 1, 2023. Depreciation is recorded in 2023 for the period January 1 through June 1 as follows.

$10,000 − $1,000 5 years

× 5 12

= $750

Change in Estimates Depreciation is based on estimates of salvage value and useful life. If our estimate of an asset’s useful life and/or salvage value changes, what should we do? The answer is to use the new esti- mate to compute depreciation for current and future periods. Revising an estimate of the useful life or salvage value of a plant asset is called a change in an accounting estimate and only affects current and future financial statements. We do not go back and restate (change) prior years’ statements. This applies to all depreciation methods.

Let’s return to the machine in Exhibit 8.8 using straight-line depreciation. At the beginning of this asset’s third year, its book value is $6,400. Assume that at the beginning of its third year, the estimated number of years remaining in its useful life changes from three to four years and its estimate of salvage value changes from $1,000 to $400. Depreciation for each of the four remaining years is computed as in Exhibit 8.14.

Impairment Loss . . . . . . . . . . . . . . . . . . . . . . . . . . . . . . . . . . . . . . . . . . . . . . 50

Accumulated Depreciation—Equipment . . . . . . . . . . . . . . . . . . . . . . 50

Record impairment of equipment.

Controller You are the controller for a struggling wingsuit company. Depreciation is its largest expense. Competitors depreciate equipment over three years. The company president tells you to revise useful lives of equipment from three to six years. What should you do? ■ Answer: The president’s instructions may be an honest and reasonable prediction of the future. However, you might confront the president if you believe the aim is only to increase income.

Decision Ethics

Part 1. A machine costing $22,000 with a five-year life and an estimated $2,000 salvage value is installed on January 1. The manager estimates the machine will produce 1,000 units of product during its life. It actually produces the following units: 200 in Year 1, 400 in Year 2, 300 in Year 3, 80 in Year 4, and 30 in Year 5. The total units produced by the end of Year 5 exceed the original estimate—this difference was not predicted. (The machine must not be depreciated below its estimated salvage value.) Compute depreciation expense for each year and total depreciation for all years combined under straight-line, units-of- production, and double-declining-balance.

Part 2. In early January, a company acquires equipment for $3,800. The company estimates this equip- ment has a useful life of three years and a salvage value of $200. On January 1 of the third year, the com- pany changes its estimates to a total four-year useful life and zero salvage value. Using the straight-line method, what is depreciation expense for the third year?

Depreciation Computations

NEED-TO-KNOW 8-2

C2 P1

EXHIBIT 8.14 Computing Revised Straight-Line Depreciation

Book value − Revised salvage value Revised remaining useful life

= $6,400 − $400

4 years = $1,500 per year

Reporting Depreciation Some companies, such as O’Reilly Auto, report both the cost and accumulated depreciation of plant assets on the balance sheet. Apple and many other companies show plant assets on one line with the net amount of cost minus accumulated depreciation. When this is done, accumu- lated depreciation is disclosed in a note—see Appendix A for Apple.

Impairment When there is a permanent decline in the fair value of an asset relative to its book value, the company writes down the asset to this fair value. This is called an asset impairment. Assume equipment has a book value of $800 and a fair (market) value of $750, and this $50 decline in value meets the impairment test (details are in advanced courses). The impairment entry is

Annual Original Revised Period Depreciation Depreciation

2018 — — 2019 $1,800 $1,800 2020 1,800 1,800 2021 1,800 1,500 2022 1,800 1,500 2023 1,800 1,500 2024 — 1,500

$9,000 $9,600

©Cultura Creative/Alamy Stock Photo

Point: Assets purchased on days 1 through 15 of a month are usually recorded as purchased on the 1st of that month. Assets purchased on days 16 to month- end are recorded as if purchased on the 1st of the next month. The same applies to asset sales.

Chapter 8 Accounting for Long-Term Assets 311

Solution—Part 1

Year Straight-Linea Units-of-Productionb Double-Declining-Balancec

1 . . . . . . . . . . . . . . . $ 4,000 $ 4,000 $ 8,800

2 . . . . . . . . . . . . . . . 4,000 8,000 5,280

3 . . . . . . . . . . . . . . . 4,000 6,000 3,168

4 . . . . . . . . . . . . . . . 4,000 1,600 1,901

5 . . . . . . . . . . . . . . . 4,000 400 851

Totals . . . . . . . . . . . $20,000 $20,000 $20,000

aStraight-line: Cost per year = ($22,000 − $2,000)∕5 years = $4,000 per year bUnits-of-production: Cost per unit = ($22,000 − $2,000)∕1,000 units = $20 per unit

Year Units Depreciation per Unit Depreciation Accum. Deprec. Book Value

1 . . . . . . . . . . . . . . . . . . 200 $20 $ 4,000 $ 4,000 $18,000

2 . . . . . . . . . . . . . . . . . . 400 20 8,000 12,000 10,000

3 . . . . . . . . . . . . . . . . . . 300 20 6,000 18,000 4,000

4 . . . . . . . . . . . . . . . . . . 80 20 1,600 19,600 2,400

5 . . . . . . . . . . . . . . . . . . 30 20 400* 20,000* 2,000

Total . . . . . . . . . . . . . . . $20,000

* 30 × $20 = $600; however, $600 would make accumulated depreciation exceed the $20,000 total depreciable cost. This means we take only enough depreciation in Year 5, or $400, to decrease book value to the asset’s $2,000 salvage value (never lower).

cDouble-declining-balance: (100%∕5) × 2 = 40% depreciation rate

Annual Ending Book Value Depreciation Accumulated ($22,000 cost less Beginning (40% of Depreciation accumulated Year Book Value book value) at Year-End depreciation)

1 . . . . . . . . $22,000 $ 8,800 $ 8,800 $13,200

2 . . . . . . . . 13,200 5,280 14,080 7,920

3 . . . . . . . . 7,920 3,168 17,248 4,752

4 . . . . . . . . 4,752 1,901* 19,149 2,851

5 . . . . . . . . 2,851 851† 20,000 2,000

Total . . . . . $20,000

*Rounded to the nearest dollar. † Set depreciation in Year 5 to reduce book value to the $2,000 salvage value; namely, instead of $1,140 ($2,851 × 40%), we use the maximum of $851 ($2,851 − $2,000).

Solution—Part 2

($3,800 − $200)∕3 years = $1,200 (original depreciation per year)

$1,200 × 2 years = $2,400 (accumulated depreciation at date of change in estimate)

($3,800 − $2,400)∕2 years = $700 (revised depreciation)

Do More: QS 8-3 through QS 8-8, E 8-4

through E 8-13

Plant assets require maintenance, repairs, and improvements. We must decide whether to expense or capitalize these expenditures (to capitalize is to increase the asset account).

Revenue expenditures, also called income statement expenditures, are costs that do not materially increase the plant asset’s life or capabilities. They are recorded as expenses on the current-period income statement.

Capital expenditures, also called balance sheet expenditures, are costs of plant assets that provide benefits for longer than the current period. They increase the asset on the balance sheet.

ADDITIONAL EXPENDITURES C3 Distinguish between revenue and capital expenditures, and account for them.

312 Chapter 8 Accounting for Long-Term Assets

Ordinary Repairs Ordinary repairs are expenditures to keep an asset in good operating condition. Ordinary repairs do not extend an asset’s useful life or increase its productivity beyond original expecta- tions. Examples are normal costs of cleaning, lubricating, changing oil, and replacing small parts of a machine. Ordinary repairs are revenue expenditures. This means their costs are reported as expenses on the current-period income statement. Following this rule, Brunswick reports that “maintenance and repair costs are expensed as incurred.” If Brunswick’s current- year repair costs are $9,500, it makes the following entry.

Example: Assume a firm owns a web server. Identify each cost as a revenue or capital expenditure: (1) purchase price, (2) necessary wiring, (3) platform for operation, (4) circuits to increase capacity, (5) monthly cleaning, (6) repair of a faulty switch, and (7) replace- ment of a worn fan. Answer: Capital expenditures: 1, 2, 3, 4; Revenue expenditures: 5, 6, 7.

Additional Expenditures Examples Expense Timing Entry

Ordinary repairs • Cleaning • Lubricating Expensed currently Repairs Expense. . . . . # • Adjusting • Repainting Cash . . . . . . . . . . . . . #

Betterments and • Replacing main parts Expensed in future Asset (such as Equip.) # extraordinary repairs • Major asset expansions Cash . . . . . . . . . . . . . #

Betterments and Extraordinary Repairs Betterments and extraordinary repairs are capital expenditures.

Betterments (Improvements) Betterments, or improvements, are expenditures that make a plant asset more efficient or productive. A betterment often involves adding a component to an asset or replacing an old com- ponent with a better one and does not always increase useful life. An example is replacing manual controls on a

machine with automatic controls. One special type of betterment is an addition, such as adding a new dock to a warehouse. Because a betterment benefits future periods, it is debited to the as- set account as a capital expenditure. The new book value (less salvage value) is then depreciated over the asset’s remaining useful life. Assume a company pays $8,000 for a machine with an eight-year useful life and no salvage value. After three years and $3,000 of depreciation, it adds an automated control system to the machine at a cost of $1,800. The cost of the betterment is added to the Machinery account with the following entry.

Assets = Liabilities + Equity −9,500 −9,500

Dec . 31 Repairs Expense . . . . . . . . . . . . . . . . . . . . . . . . . . . . . . . . . . . . 9,500

Cash . . . . . . . . . . . . . . . . . . . . . . . . . . . . . . . . . . . . . . . . . 9,500

Record ordinary repairs of equipment.

Assets = Liabilities + Equity +1,800 −1,800

Jan . 2 Machinery . . . . . . . . . . . . . . . . . . . . . . . . . . . . . . . . . . . . . . . . . 1,800

Cash . . . . . . . . . . . . . . . . . . . . . . . . . . . . . . . . . . . . . . . . . 1,800

Record installation of automated system.

After this entry, the remaining cost to be depreciated is $6,800, computed as $8,000 − $3,000 + $1,800. Depreciation for the remaining five years is $1,360 per year, computed as $6,800∕5 years.

Extraordinary Repairs (Replacements) Extraordinary repairs are expenditures that extend the asset’s useful life beyond its original estimate. Their costs are debited to the asset account.

Point: Both extraordinary repairs and betterments require revising future depreciation.

To the Moon and Back SpaceX made history when it relaunched a used Falcon 9 rocket. This was the first time an orbital rocket was launched into space a second time. SpaceX made extraordinary repairs to the rocket to make this relaunch possible. However, these repairs were considerably less costly than building a new rocket for tens of millions of dollars. ■

Decision Insight

Source: NASA/Tony Gray and Kevin O’Connell

Disposal of plant assets occurs in one of three ways: discarding, sale, or exchange. Discarding and selling are covered here; Appendix 8A covers exchanges. The steps for disposing plant assets are in Exhibit 8.15.

DISPOSALS OF PLANT ASSETS

Chapter 8 Accounting for Long-Term Assets 313

Discarding Plant Assets A plant asset is discarded when it is no longer useful to the company and it has no market value. Assume that a machine costing $9,000 with accumulated depreciation of $9,000 is discarded. When accumulated depreciation equals the asset’s cost, it is said to be fully depreciated (zero book value). The entry to record the discarding of this asset is

EXHIBIT 8.15 Accounting for Disposals of Plant Assets

1 . Record depreciation up to the date of disposal—this also updates Accumulated Depreciation .

2 . Record the removal of the disposed asset’s account balances—including its accumulated depreciation .

3 . Record any cash (and/or other assets) received or paid in the disposal .

4 . Record any gain or loss—equal to the value of any assets received minus the disposed asset’s book value .

This entry reflects all four steps of Exhibit 8.15. Step 1 is unnecessary because the machine is fully depreciated. Step 2 is reflected in the debit to Accumulated Depreciation and credit to Machinery. Because no other asset is involved, step 3 is irrelevant. Finally, because book value is zero and no other asset is involved, no gain or loss is recorded in step 4.

How do we account for discarding an asset that is not fully depreciated or one whose depre- ciation is not up-to-date? To answer this, consider equipment costing $8,000 with accumulated depreciation of $6,000 on December 31 of the prior fiscal year-end. This equipment is being depreciated by $1,000 per year using the straight-line method over eight years with zero salvage. On July 1 of the current year it is discarded. Step 1 is to bring depreciation up-to-date.

Point: Recording depreciation expense up-to-date gives an up-to-date book value for determining gain or loss.

Steps 2 through 4 of Exhibit 8.15 are reflected in the second (and final) entry.

This loss is computed by comparing the equipment’s $1,500 book value ($8,000 − $6,000 − $500) with the zero net cash proceeds. The loss is reported in the Other Expenses and Losses section of the income statement. Discarding an asset can sometimes require a cash payment that would increase the loss.

Selling Plant Assets To demonstrate selling plant assets, consider BTO’s March 31 sale of equipment that cost $16,000 and has accumulated depreciation of $12,000 at December 31 of the prior year- end. Annual depreciation on this equipment is $4,000 using straight-line. Step 1 of this sale is to record depreciation expense and update accumulated depreciation to March 31 of the current year.

June 5 Accumulated Depreciation—Machinery . . . . . . . . . . . . . . . . . 9,000

Machinery . . . . . . . . . . . . . . . . . . . . . . . . . . . . . . . . . . . . . 9,000

Discarding of fully depreciated machinery.

Assets = Liabilities + Equity +9,000 −9,000

July 1 Depreciation Expense . . . . . . . . . . . . . . . . . . . . . . . . . . . . . . . 500

Accumulated Depreciation—Equipment . . . . . . . . . . . . . 500

Record 6 months’ depreciation ($1,000 × 6∕12).

Assets = Liabilities + Equity −500 −500

July 1 Accumulated Depreciation—Equipment . . . . . . . . . . . . . . . . . . 6,500

Loss on Disposal of Equipment . . . . . . . . . . . . . . . . . . . . . . . . . 1,500

Equipment . . . . . . . . . . . . . . . . . . . . . . . . . . . . . . . . . . . . . 8,000

Discard equipment with a $1,500 book value.

Assets = Liabilities + Equity +6,500 −1,500 −8,000

Mar . 31 Depreciation Expense . . . . . . . . . . . . . . . . . . . . . . . . . . . . . . . 1,000

Accumulated Depreciation—Equipment . . . . . . . . . . . . . 1,000

Record 3 months’ depreciation ($4,000 × 3∕12).

Assets = Liabilities + Equity −1,000 −1,000

P2 Account for asset disposal through discarding or selling an asset.

314 Chapter 8 Accounting for Long-Term Assets

Steps 2 through 4 need one final entry that depends on the amount received from the sale. We cover three different possibilities.

Sale at Book Value If BTO receives $3,000 cash, an amount equal to the equipment’s book value as of March 31 (book value = $16,000 − $12,000 − $1,000), no gain or loss is re- corded. The entry is

Sale price = Book value → No gain or loss

Sale above Book Value If BTO receives $7,000, an amount that is $4,000 above the equipment’s $3,000 book value as of March 31, a gain is recorded. The entry is

Sale price > Book value → Gain

Sale below Book Value If BTO receives $2,500, an amount that is $500 below the equipment’s $3,000 book value as of March 31, a loss is recorded. The entry is

Sale price < Book value → Loss

Mar . 31 Cash . . . . . . . . . . . . . . . . . . . . . . . . . . . . . . . . . . . . . . . . . . . . . . 3,000

Accumulated Depreciation—Equipment . . . . . . . . . . . . . . . . . 13,000

Equipment . . . . . . . . . . . . . . . . . . . . . . . . . . . . . . . . . . . . 16,000

Record sale of equipment for no gain or loss.

Assets = Liabilities + Equity +3,000 +13,000 −16,000

Mar . 31 Cash . . . . . . . . . . . . . . . . . . . . . . . . . . . . . . . . . . . . . . . . . . . . . . 7,000

Accumulated Depreciation—Equipment . . . . . . . . . . . . . . . . . 13,000

Gain on Disposal of Equipment . . . . . . . . . . . . . . . . . . . . 4,000

Equipment . . . . . . . . . . . . . . . . . . . . . . . . . . . . . . . . . . . . 16,000

Record sale of equipment for a $4,000 gain.

Assets = Liabilities + Equity +7,000 +4,000 +13,000 −16,000

Mar . 31 Cash . . . . . . . . . . . . . . . . . . . . . . . . . . . . . . . . . . . . . . . . . . . . . . 2,500

Loss on Disposal of Equipment . . . . . . . . . . . . . . . . . . . . . . . . 500

Accumulated Depreciation—Equipment . . . . . . . . . . . . . . . . . 13,000

Equipment . . . . . . . . . . . . . . . . . . . . . . . . . . . . . . . . . . . . 16,000

Record sale of equipment for a $500 loss.

Assets = Liabilities + Equity +2,500 −500 +13,000 −16,000

Part 1. A company pays $1,000 for equipment expected to last four years and have a $200 salvage value. Prepare journal entries to record the following costs related to the equipment. a. During the second year of the equipment’s life, $400 cash is paid for a new component expected to

materially increase the equipment’s productivity. b. During the third year, $250 cash is paid for normal repairs necessary to keep the equipment in good

working order. c. During the fourth year, $500 is paid for repairs expected to increase the useful life of the equipment

from four to five years.

Part 2. A company owns a machine that cost $500 and has accumulated depreciation of $400. Prepare the entry to record the disposal of the machine on January 2 in each separate situation. a. The company disposed of the machine, receiving nothing in return. b. The company sold the machine for $80 cash. c. The company sold the machine for $100 cash. d. The company sold the machine for $110 cash.

Solution—Part 1

a.

Additional Expenditures and Asset Disposals

NEED-TO-KNOW 8-3

C3 P2

Year 2 Equipment . . . . . . . . . . . . . . . . . . . . . . . . . . . . . . . . . . . . . . 400 Cash . . . . . . . . . . . . . . . . . . . . . . . . . . . . . . . . . . . . . . . 400 Record betterment.

b. Year 3 Repairs Expense . . . . . . . . . . . . . . . . . . . . . . . . . . . . . . . . . 250 Cash . . . . . . . . . . . . . . . . . . . . . . . . . . . . . . . . . . . . . . . 250 Record ordinary repairs.

Chapter 8 Accounting for Long-Term Assets 315

Solution—Part 2 (Note: Book value of machine = $500 − $400 = $100)

a. Disposed of at no value.

c. Year 4 Equipment . . . . . . . . . . . . . . . . . . . . . . . . . . . . . . . . . . . . . . 500 Cash . . . . . . . . . . . . . . . . . . . . . . . . . . . . . . . . . . . . . . . 500 Record extraordinary repairs.

Jan . 2 Loss on Disposal of Machine . . . . . . . . . . . . 100

Accumulated Depreciation—Machine . . . . . 400

Machine . . . . . . . . . . . . . . . . . . . . . . . . 500

Record disposal of machine.

Jan . 2 Cash . . . . . . . . . . . . . . . . . . . . . . . . . . . . . . . 80

Loss on Sale of Machine . . . . . . . . . . . . . . . 20

Accumulated Depreciation—Machine . . . . . 400

Machine . . . . . . . . . . . . . . . . . . . . . . . . 500

Record sale of machine below book value.

b. Sold for $80 cash.

Jan . 2 Cash . . . . . . . . . . . . . . . . . . . . . . . . . . . . . . . 100

Accumulated Depreciation—Machine . . . . . 400

Machine . . . . . . . . . . . . . . . . . . . . . . . . 500

Record sale of machine at book value.

c. Sold for $100 cash.

Jan . 2 Cash . . . . . . . . . . . . . . . . . . . . . . . . . . . . . . . 110

Accumulated Depreciation—Machine . . . . . 400

Gain on Sale of Machine . . . . . . . . . . . 10

Machine . . . . . . . . . . . . . . . . . . . . . . . . 500

Record sale of machine above book value.

d. Sold for $110 cash.

Do More: QS 8-9, QS 8-10, E 8-14, E 8-15, E 8-16,

E 8-17

Natural resources are assets that are physically consumed when used. Examples are standing timber, mineral deposits, and oil and gas fields. These assets are soon-to-be inventories of raw materials after cutting, mining, or pumping. Until that conversion happens, they are reported as noncurrent assets under either plant assets or their own category using titles such as Timber- lands, Mineral deposits, or Oil reserves.

Cost Determination and Depletion Natural resources are recorded at cost, which includes all expenditures necessary to acquire the resource and prepare it for use. Depletion is the process of allocating the cost of a natural resource to the period when it is consumed. Natural resources are reported on the balance sheet at cost minus accumulated depletion. The depletion expense per period is usually based on units extracted from cutting, mining, or pumping. This is similar to units-of-production depreciation.

To demonstrate, consider a mineral deposit with an estimated 250,000 tons of available ore. It is purchased for $500,000, and we expect zero salvage value. The depletion charge per ton of ore mined is $2, computed as $500,000 ÷ 250,000 tons. If 85,000 tons are mined and sold in the first year, the depletion charge for that year is $170,000. These computations are in Exhibit 8.16.

Section 2—Natural Resources P3 Account for natural resource assets and their depletion.

Depletion expense for the first year is recorded as follows.

Depletion expense = Depletion per unit × Units extracted and sold in period = $2 × 85,000 = $170,000

Depletion per unit = Cost – Salvage value

Total units of capacity =

$500,000 – $0 250,000 tons

= $2 per tonStep 1

Step 2

EXHIBIT 8.16 Depletion Formula and Example

Dec . 31 Depletion Expense—Mineral Deposit . . . . . . . . . . . . . . . . . . . . 170,000

Accumulated Depletion—Mineral Deposit . . . . . . . . . . . . 170,000

Record depletion of the mineral deposit.

Assets = Liabilities + Equity −170,000 −170,000

316 Chapter 8 Accounting for Long-Term Assets

The period-end balance sheet reports the mineral deposit as shown in Exhibit 8.17.

Mineral deposit . . . . . . . . . . . . . . . . . . . . . . . . . . $500,000

Less accumulated depletion . . . . . . . . . . . . . . 170,000 $330,000

EXHIBIT 8.17 Balance Sheet Presentation of Natural Resources

Because all 85,000 tons of the mined ore are sold during the year, the entire $170,000 of depletion is reported on the income statement. If some of the ore remains unsold at year-end, the depletion related to the unsold ore is carried forward on the balance sheet and reported as Ore Inventory, a current asset. Altering our example, assume that of the 85,000 tons mined the first year, only 70,000 tons are sold. We record depletion of $140,000 (70,000 tons × $2 depletion per unit) and the remaining ore inventory of $30,000 (15,000 tons × $2 depletion per unit) as follows.

Dec . 31 Depletion Expense—Mineral Deposit . . . . . . . . . . . . . . . . . . . 140,000

Ore Inventory . . . . . . . . . . . . . . . . . . . . . . . . . . . . . . . . . . . . . . 30,000

Accumulated Depletion—Mineral Deposit . . . . . . . . . . . 170,000

Record depletion and inventory of mineral deposit.

Assets = Liabilities + Equity −170,000 −140,000 +30,000

Plant Assets Tied into Extracting Mining, cutting, or pumping natural resources requires machinery, equipment, and buildings. When the usefulness of these plant assets is directly related to the depletion of a natural resource, their costs are depreciated using the units-of-production method in proportion to the depletion of the natural resource. For example, if a machine is permanently installed in a mine and 10% of the ore is mined and sold in the period, then 10% of the machine’s cost (minus any salvage value) is depreciated. The same procedure is used when a machine is abandoned once resources are extracted. If the machine will be used at another site when extraction is complete, it is depre- ciated over its own useful life.

Lost Cause Long-term assets must be safeguarded against theft, misuse, and damage. Controls include use of secu- rity tags, monitoring of rights infringements, and approvals of asset disposals. A study reports that 43% of employees in operations and services witnessed the wasting, mismanaging, or abusing of assets in the past year (KPMG). ■

Ethical Risk

©GIRODJL/Shutterstock

A company acquires a zinc mine at a cost of $750,000 on January 1. At that same time, it incurs additional costs of $100,000 to access the mine, which is estimated to hold 200,000 tons of zinc. The estimated value of the land after the zinc is removed is $50,000. 1. Prepare the January 1 entry(ies) to record the cost of the zinc mine. 2. Prepare the December 31 year-end adjusting entry if 50,000 tons of zinc are mined, but only 40,000

tons are sold the first year.

Solution

1.

Depletion Accounting

NEED-TO-KNOW 8-4

P3

2. Depletion per unit = ($750,000 + $100,000 − $50,000)/200,000 tons = $4.00 per ton

Do More: QS 8-11, E 8-18, P 8-7

Jan . 1 Zinc Mine . . . . . . . . . . . . . . . . . . . . . . . . . . . . . . . . . . . . . . . 850,000 Cash . . . . . . . . . . . . . . . . . . . . . . . . . . . . . . . . . . . . . . . 850,000 Record cost of zinc mine.

Dec . 31 Depletion Expense—Zinc Mine . . . . . . . . . . . . . . . . . . . . . . 160,000 Zinc Inventory . . . . . . . . . . . . . . . . . . . . . . . . . . . . . . . . . . . 40,000 Accumulated Depletion—Zinc Mine . . . . . . . . . . . . . . 200,000 Record depletion of zinc mine (50,000 × $4.00).

Chapter 8 Accounting for Long-Term Assets 317

Intangible assets are nonphysical assets used in operations that give companies long-term rights or competitive advantages. Examples are patents, copyrights, licenses, leaseholds, franchises, and trademarks. Lack of physical substance does not always mean an intangible asset. For example, notes and accounts receivable lack physical substance but are not intangi- bles. This section covers common types of intangible assets.

Cost Determination and Amortization An intangible asset is recorded at cost when purchased. Intangibles can have limited lives or indefinite lives. If an intangible has a limited life, its cost is expensed over its estimated use- ful life using amortization. If an intangible asset has an indefinite life—meaning that no legal, competitive, economic, or other factors limit its useful life—it is not amortized. (If an intangible with an indefinite life is later judged to have a limited life, it is amortized over that limited life.)

Amortization of intangible assets is similar to depreciation. However, only the straight-line method is used for amortizing intangibles unless the company can show that another method is preferred. Amortization is recorded in a contra account, Accumulated Amortization. The acqui- sition cost of intangible assets is disclosed along with the accumulated amortization. The dis- posal of an intangible asset involves removing its book value, recording any other asset(s) received or given up, and recognizing any gain or loss for the difference.

Many intangibles have limited lives due to laws, contracts, or other reasons. Examples are patents, copyrights, and leaseholds. The cost of intangible assets is amortized over the periods expected to benefit from their use, but this period cannot be longer than the assets’ legal existence. Other intangibles such as trademarks and trade names have indefinite lives and are not amortized. An intangible asset that is not amortized is tested annually for impairment—if necessary, an impairment loss is recorded. (Details are in advanced courses.)

Intangible assets are often in a separate section of the balance sheet immediately after plant assets. For example, Nike follows this approach in reporting nearly $300 million of intangible assets in its balance sheet, plus $140 million in goodwill. Companies usually disclose their amortization periods for intangibles. The remainder of our discussion focuses on accounting for specific types of intangible assets.

Types of Intangibles Patents The federal government grants patents to encourage the invention of new technology and processes. A patent is an exclusive right granted to its owner to manufacture and sell a pat- ented item or to use a process for 20 years. When patent rights are purchased, the cost to acquire the rights is debited to an account called Patents. If the owner engages in lawsuits to successfully defend a patent, the cost of lawsuits is debited to the Patents account; if the defense is unsuccess- ful, the book value of the patent is expensed. However, the costs of research and development leading to a new patent are expensed when incurred.

A patent’s cost is amortized over its estimated useful life (not to exceed 20 years). If we purchase a patent costing $25,000 with a useful life of 10 years, we make the following adjusting entry at the end of each of the 10 years to amortize one-tenth of its cost. The $2,500 debit to Amortization Expense is on the income statement as a cost of the patented product or service. The Accumulated Amortization—Patents account is a contra asset account to Patents.

Section 3—Intangible Assets P4 Account for intangible assets.

©Michael DeYoung/Blend Images

Dec . 31 Amortization Expense—Patents . . . . . . . . . . . . . . . . . . . . . . . . 2,500

Accumulated Amortization—Patents . . . . . . . . . . . . . . . . 2,500

Amortize patent costs over its useful life.

Assets = Liabilities + Equity −2,500 −2,500

318 Chapter 8 Accounting for Long-Term Assets

Copyrights A copyright gives its owner the exclusive right to publish and sell a musical, literary, or artistic work during the life of the creator plus 70 years, although the useful life of most copyrights is much shorter. The costs of a copyright are amortized over its useful life. The only identifiable cost of many copyrights is the fee paid to the Copyright Office. Identifiable costs of a copyright are capitalized (recorded in an asset account) and amortized by debiting an account called Amortization Expense—Copyrights.

Franchises and Licenses Franchises and licenses are rights that a company or gov- ernment grants an entity to sell a product or service under specified conditions. Many organiza- tions grant franchise and license rights—Anytime Fitness, Firehouse Subs, and Major League Baseball are just a few examples. The costs of franchises and licenses are debited to a Franchises and Licenses asset account and are amortized over the life of the agreement. If an agreement is for an indefinite time, those costs are not amortized.

Trademarks and Trade Names A trademark or trade (brand) name is a sym- bol, name, phrase, or jingle identified with a company, product, or service. Examples are Nike Swoosh, Big Mac, Coca-Cola, and Corvette. Ownership and exclusive right to use a trademark or trade name often are granted to the company that used it first. Ownership is best established by registering a trademark or trade name with the government’s Patent Office. The cost of developing, maintaining, or enhancing the value of a trademark or trade name (such as advertising) is charged to expense when incurred. If a trademark or trade name is purchased, however, its cost is debited to an asset account and then amortized over its expected life. If the company plans to renew indefinitely its right to the trademark or trade name, the cost is not amortized.

Goodwill Goodwill is the amount by which a company’s value exceeds the value of its in- dividual assets and liabilities. This implies that the company as a whole has certain valuable attributes not measured in assets and liabilities. These can include superior management, skilled workforce, good supplier or customer relations, quality products or services, good location, or other competitive advantages.

Goodwill is only recorded when an entire company or business segment is purchased. Purchased goodwill is computed as purchase price of the company minus the market value of net assets (excluding goodwill). Google paid $1.19 billion to acquire YouTube; about $1.13 of the $1.19 billion was for goodwill. Goodwill is recorded as an asset, and it is not amortized. Instead, goodwill is annually tested for impairment. (Details are in advanced courses.)

Right-of-Use Asset (Lease) Property is rented under a contract called a lease. The property’s owner, called the lessor, grants the lease. The one who secures the right to possess and use the property is called the lessee. A leasehold is the rights the lessor grants to the lessee under the terms of the lease.

Lease or Buy Some advantages of leasing an asset versus buying it are that Little or no up-front payment is normally required (making it more affordable). Lease terms often allow exchanges to trade up on leased assets (reducing obsolescence).

Lease Accounting For noncurrent leases, the lessee records a “Right-of-Use Asset” and “Lease Liability” equal to the value of lease payments. At each period-end, the lessee records amortization with a debit to Amortization Expense and a credit to Accumulated Amortization— Right-of-Use Asset.

Leasehold Improvements A lessee sometimes pays for improvements to the leased property such as partitions, painting, and storefronts. These improvements are called leasehold improvements, and the lessee debits these costs to a Leasehold Improvements account. The lessee amortizes these costs over the life of the lease or the life of the improvements, whichever

Point: McDonald’s “golden arches” are one of the world’s most valuable trademarks, yet this asset is not on McDonald’s balance sheet.

Point: Amortization of goodwill is different for financial accounting and tax accounting. The IRS requires the amortization of goodwill over 15 years.

Example: Assume goodwill has a book value of $500, an implied fair value of $475, and this $25 decline in value meets the impair- ment test. The impairment entry is Impairment Loss . . . . . . . . . . 25 Goodwill . . . . . . . . . . . . . . 25

Point: At lease start: Right-of-Use Asset . . . . . . . # Lease Liability . . . . . . . . . #

At each period-end: Amortization Expense . . . . # Acc Amor—RoU Asset . . . #

Point: A Leasehold account im- plies existence of future benefits that the lessee controls because of a prepayment. It also meets the definition of an asset.

Chapter 8 Accounting for Long-Term Assets 319

is shorter. The amortization entry debits Amortization Expense—Leasehold Improvements and credits Accumulated Amortization—Leasehold Improvements.

Other Intangibles There are other types of intangible assets such as software, non- compete covenants, customer lists, and so forth. Accounting for them is the same as for other intangibles.

Research and Development Research and development costs are expenditures to discover new products, new processes, or knowledge. Creating patents, copyrights, and innova- tive products and services requires research and development costs. The costs of research and development are expensed when incurred because it is difficult to predict the future benefits from research and development. GAAP does not include them as intangible assets.

Free Mickey The Walt Disney Company successfully lobbied Congress to extend copyright protection from the life of the creator plus 50 years to the life of the creator plus 70 years. This extension allows the company to protect its characters for 20 additional years before the right to use them enters the public domain. Mickey Mouse is now pro- tected by copyright law until 2023. The law is officially termed the Copyright Term Extension Act (CTEA), but it is also known as the Mickey Mouse Protection Act. ■

Decision Insight

©Yoshikazu Tsuno/AFP/Getty Images

Part 1. A publisher purchases the copyright on a book for $1,000 on January 1 of this year. The copyright lasts five more years. The company plans to sell prints for seven years. Prepare entries to record the pur- chase of the copyright on January 1 and its annual amortization on December 31.

Part 2. On January 3 of this year, a retailer pays $9,000 to modernize its store. Improvements include lighting, partitions, and a sound system. These improvements are estimated to yield benefits for five years. The retailer leases its store and has three years remaining on its lease. Prepare the entry to record (a) the cost of modernization and (b) amortization at the end of this year.

Part 3. On January 6 of this year, a company pays $6,000 for a patent with a remaining 12-year legal life to produce a supplement expected to be marketable for 3 years. Prepare entries to record its acquisition and the December 31 amortization entry.

Solution—Part 1

P4

Accounting for Intangibles

NEED-TO-KNOW 8-5

Jan . 1 Copyright . . . . . . . . . . . . . . . . . . . . . . . . . . . . . . . . . . . . . . . . . . . . . . . . 1,000

Cash . . . . . . . . . . . . . . . . . . . . . . . . . . . . . . . . . . . . . . . . . . . . . . . 1,000

Record purchase of copyright.

Dec . 31 Amortization Expense—Copyright . . . . . . . . . . . . . . . . . . . . . . . . . . . . 200

Accumulated Amortization—Copyright . . . . . . . . . . . . . . . . . . . . 200

Record amortization of copyright ($1,000/5 years).

Solution—Part 2

a. Jan . 3 Leasehold Improvements . . . . . . . . . . . . . . . . . . . . . . . . . . . . . . . . . . . 9,000

Cash . . . . . . . . . . . . . . . . . . . . . . . . . . . . . . . . . . . . . . . . . . . . . . . 9,000

Record leasehold improvements.

b. Dec . 31 Amortization Expense—Leasehold Improvements . . . . . . . . . . . . . . . 3,000 Accumulated Amortization—Leasehold Improvements . . . . . . . 3,000

Record amortization of leasehold over remaining lease life.*

*Amortization = $9,000∕3-year lease term = $3,000 per year.

320 Chapter 8 Accounting for Long-Term Assets

Total Asset TurnoverDecision Analysis

One important measure of a company’s ability to use its assets efficiently and effectively is total asset turnover, defined in Exhibit 8.18.

A1 Compute total asset turn- over and apply it to analyze a company’s use of assets.

Net sales is net amounts earned from the sale of products and services. Average total assets is (Current period-end total assets + Prior period-end total assets)/2. A higher total asset turnover means a company is generating more net sales for each dollar of assets. Management is evaluated on efficient and effective use of total assets by looking at total asset turnover. Let’s look at total asset turnover in Exhibit 8.19 for two competing companies: Starbucks and Jack in the Box.

EXHIBIT 8.18 Total Asset Turnover Total asset turnover =

Net sales Average total assets

To show how we use total asset turnover, let’s look at Starbucks. We express Starbucks’s use of assets in generating net sales by saying “it turned its assets over 1.56 times during the current year.” This means that each $1.00 of assets produced $1.56 of net sales.

Is a total asset turnover of 1.56 good or bad? All companies want a high total asset turnover. Interpreting the total asset turnover requires an understanding of company operations. Some operations are capital- intensive, meaning that a relatively large amount is invested in plant assets to generate sales. This results in a lower total asset turnover. Other companies’ operations are labor-intensive, meaning that they gener- ate sales using people instead of assets. In that case, we expect a higher total asset turnover. Starbucks’s turnover is higher than that for Jack in the Box. However, Starbucks’s total asset turnover decreased over the last three years. To maintain a strong total asset turnover, Starbucks must grow sales at a rate equal to, or higher than, its total asset growth.

Environmentalist A paper manufacturer claims it cannot afford more environmental controls. It points to its low total asset turnover of 1.9 and argues that it cannot compete with companies whose total asset turnover is much higher. Examples cited are food stores (5.5) and auto dealers (3.8). How do you respond? ■ Answer: The paper manufacturer’s com- parison of its total asset turnover with food stores and auto dealers is misdirected. You need to collect data from competitors in the paper industry to show that a 1.9 total asset turnover is about the norm for this industry.

Decision Maker

Do More: QS 8-12, QS 8-13, E 8-19, E 8-20

Jan . 6 Patents . . . . . . . . . . . . . . . . . . . . . . . . . . . . . . . . . . . . . . . . . . . . . . . . . . 6,000

Cash . . . . . . . . . . . . . . . . . . . . . . . . . . . . . . . . . . . . . . . . . . . . . . . 6,000

Record purchase of patent.

Dec . 31 Amortization Expense* . . . . . . . . . . . . . . . . . . . . . . . . . . . . . . . . . . . . . 2,000

Accumulated Amortization—Patents . . . . . . . . . . . . . . . . . . . . . . 2,000

Record amortization of patent. *$6,000/3 years = $2,000

Solution—Part 3

EXHIBIT 8.19 Analysis Using Total Asset Turnover

Company Figure ($ millions) Current Year 1 Year Ago 2 Years Ago

Starbucks Net sales . . . . . . . . . . . . . . . . . . . . . . . . . . . . . $22,387 $21,316 $19,163 Average total assets . . . . . . . . . . . . . . . . . . . . $14,339 $13,364 $11,585 Total asset turnover . . . . . . . . . . . . . . . . . . . 1.56 1.60 1.65 Jack in the Box Net sales . . . . . . . . . . . . . . . . . . . . . . . . . . . . . $1,554 $1,599 $1,540 Average total assets . . . . . . . . . . . . . . . . . . . . $1,289 $1,326 $1,287 Total asset turnover . . . . . . . . . . . . . . . . . . . 1.21 1.21 1.20

Chapter 8 Accounting for Long-Term Assets 321

Required

1. Allocate the total $600,000 purchase cost among the separate assets. 2. Compute the 2018 (six months) and 2019 depreciation expense for each asset, and compute the company’s

total depreciation expense for both years. The machinery produced 700 units in 2018 and 1,800 units in 2019. 3. On the last day of calendar-year 2020, Tulsa discarded equipment that had been on its books for five

years. The equipment’s original cost was $12,000 (estimated life of five years) and its salvage value was $2,000. No depreciation had been recorded for the fifth year when the disposal occurred. Journalize the fifth year of depreciation (straight-line method) and the asset’s disposal.

4. At the beginning of year 2020, Tulsa purchased a patent for $100,000 cash. The company estimated the patent’s useful life to be 10 years. Journalize the patent acquisition and its amortization for the year 2020.

5. Late in the year 2020, Tulsa acquired an ore deposit for $600,000 cash. It added roads and built mine shafts for an additional cost of $80,000. Salvage value of the mine is estimated to be $20,000. The company estimated 330,000 tons of available ore. In year 2020, Tulsa mined and sold 10,000 tons of ore. Journalize the mine’s acquisition and its first year’s depletion.

6.A (This question applies to this chapter’s Appendix coverage.) On the first day of 2020, Tulsa exchanged the machinery that was acquired on July 1, 2018, along with $5,000 cash for machinery with a $210,000 market value. Journalize the exchange of these assets assuming the exchange has commercial substance. (Refer to background information in parts 1 and 2.)

PLANNING THE SOLUTION Complete a three-column table showing the following amounts for each asset: appraised value, percent

of total value, and apportioned cost. Using allocated costs, compute depreciation for 2018 (only one-half year) and 2019 (full year) for each

asset. Summarize those computations in a table showing total depreciation for each year. Depreciation must be recorded up-to-date before discarding an asset. Calculate and record depreciation

expense for the fifth year using the straight-line method. Record the loss on the disposal as well as the removal of the discarded asset and its accumulated depreciation.

Record the patent (an intangible asset) at its purchase price. Use straight-line amortization over its use- ful life to calculate amortization expense.

Record the ore deposit (a natural resource asset) at its cost, including any added costs to ready the mine for use. Calculate depletion per ton using the depletion formula. Multiply the depletion per ton by the amount of tons mined and sold to calculate depletion expense for the year.

Gains and losses on asset exchanges that have commercial substance are recognized. Make a journal entry to add the acquired machinery and remove the old machinery, along with its accumulated depre- ciation, and to record the cash given in the exchange.

SOLUTION 1. Allocation of the total cost of $600,000 among the separate assets.

Appraised Percent of Asset Value Total Value Apportioned Cost

Land . . . . . . . . . . . . . . . . . . . . . . . . $160,000 20% $120,000 ($600,000 × 20%) Land improvements . . . . . . . . . . . 80,000 10 60,000 ($600,000 × 10%) Building . . . . . . . . . . . . . . . . . . . . . 320,000 40 240,000 ($600,000 × 40%) Machinery . . . . . . . . . . . . . . . . . . . 240,000 30 180,000 ($600,000 × 30%) Total . . . . . . . . . . . . . . . . . . . . . . . . $800,000 100% $ 600,000

On July 1, 2018, Tulsa Company pays $600,000 to acquire a fully equipped factory. The purchase includes the following assets and information.

COMPREHENSIVE

Acquisition, Cost Allocation, and Disposal of Tangible and Intangible Assets

NEED-TO-KNOW 8-6

Asset Appraised Value Salvage Value Useful Life Depreciation Method

Land . . . . . . . . . . . . . . . . . . . . . . . . . . . . $160,000 Not depreciated Land improvements . . . . . . . . . . . . . . . 80,000 $ 0 10 years Straight-line Building . . . . . . . . . . . . . . . . . . . . . . . . . 320,000 100,000 10 years Double-declining-balance Machinery . . . . . . . . . . . . . . . . . . . . . . . 240,000 20,000 10,000 units Units-of-production Total . . . . . . . . . . . . . . . . . . . . . . . . . . . . $800,000

322 Chapter 8 Accounting for Long-Term Assets

2. Depreciation for each asset. (Land is not depreciated.)

Land Improvements Cost . . . . . . . . . . . . . . . . . . . . . . . . . . . . . . . . . . . . . . . . . . . . . . . . . . . . . . $ 60,000

Salvage value . . . . . . . . . . . . . . . . . . . . . . . . . . . . . . . . . . . . . . . . . . . . . . 0

Depreciable cost . . . . . . . . . . . . . . . . . . . . . . . . . . . . . . . . . . . . . . . . . . . $ 60,000

Useful life . . . . . . . . . . . . . . . . . . . . . . . . . . . . . . . . . . . . . . . . . . . . . . . . . 10 years

Annual depreciation expense ($60,000∕10 years) . . . . . . . . . . . . . . . . $ 6,000 2018 depreciation ($6,000 × 6∕12) . . . . . . . . . . . . . . . . . . . . . . . . . . . $ 3,000 2019 depreciation . . . . . . . . . . . . . . . . . . . . . . . . . . . . . . . . . . . . . . . . . $ 6,000

Building Straight-line rate = 100%∕10 years = 10% Double-declining-balance rate = 10% × 2 = 20% 2018 depreciation ($240,000 × 20% × 6∕12) . . . . . . . . . . . . . . . . . . $ 24,000 2019 depreciation [($240,000 − $24,000) × 20%] . . . . . . . . . . . . . . . $ 43,200

Machinery Cost . . . . . . . . . . . . . . . . . . . . . . . . . . . . . . . . . . . . . . . . . . . . . . . . . . . . . . $180,000

Salvage value . . . . . . . . . . . . . . . . . . . . . . . . . . . . . . . . . . . . . . . . . . . . . . 20,000

Depreciable cost . . . . . . . . . . . . . . . . . . . . . . . . . . . . . . . . . . . . . . . . . . . $160,000

Total expected units of production . . . . . . . . . . . . . . . . . . . . . . . . . . . . . 10,000 units

Depreciation per unit ($160,000∕10,000 units) . . . . . . . . . . . . . . . . . . $ 16 2018 depreciation ($16 × 700 units) . . . . . . . . . . . . . . . . . . . . . . . . . . $ 11,200 2019 depreciation ($16 × 1,800 units) . . . . . . . . . . . . . . . . . . . . . . . . $ 28,800

Total depreciation expense for each year.

3. Record the depreciation up-to-date on the discarded asset.

Record the removal of the discarded asset and its loss on disposal.

2018 2019

Land improvements . . . . . . . . . . . $ 3,000 $ 6,000

Building . . . . . . . . . . . . . . . . . . . . . 24,000 43,200

Machinery . . . . . . . . . . . . . . . . . . . 11,200 28,800

Total . . . . . . . . . . . . . . . . . . . . . . . . $38,200 $78,000

Depreciation Expense—Equipment . . . . . . . . . . . . . . . . . . . . . . . . . . . . . . . . . . . . . . . . 2,000

Accumulated Depreciation—Equipment . . . . . . . . . . . . . . . . . . . . . . . . . . . . . . . . 2,000

Record depreciation on date of disposal: ($12,000 − $2,000)∕5.

Accumulated Depreciation—Equipment . . . . . . . . . . . . . . . . . . . . . . . . . . . . . . . . . . . . 10,000

Loss on Disposal of Equipment . . . . . . . . . . . . . . . . . . . . . . . . . . . . . . . . . . . . . . . . . . . . 2,000

Equipment . . . . . . . . . . . . . . . . . . . . . . . . . . . . . . . . . . . . . . . . . . . . . . . . . . . . . . . . 12,000

Record the discarding of equipment with a $2,000 book value.

Amortization Expense—Patent . . . . . . . . . . . . . . . . . . . . . . . . . . . . . . . . . . . . . . . . . . . . 10,000

Accumulated Amortization—Patent . . . . . . . . . . . . . . . . . . . . . . . . . . . . . . . . . . . . 10,000

Record amortization expense: $100,000∕10 years = $10,000.

Patent . . . . . . . . . . . . . . . . . . . . . . . . . . . . . . . . . . . . . . . . . . . . . . . . . . . . . . . . . . . . . . . . 100,000

Cash . . . . . . . . . . . . . . . . . . . . . . . . . . . . . . . . . . . . . . . . . . . . . . . . . . . . . . . . . . . . 100,000

Record patent acquisition.

4.

Chapter 8 Accounting for Long-Term Assets 323

Depletion Expense—Ore Deposit . . . . . . . . . . . . . . . . . . . . . . . . . . . . . . . . . . . . . . . . . . 20,000

Accumulated Depletion—Ore Deposit . . . . . . . . . . . . . . . . . . . . . . . . . . . . . . . . . . 20,000

Record depletion expense: ($680,000 − $20,000)∕330,000 tons = $2 per ton. 10,000 tons mined and sold × $2 = $20,000 depletion.

Ore Deposit . . . . . . . . . . . . . . . . . . . . . . . . . . . . . . . . . . . . . . . . . . . . . . . . . . . . . . . . . . . 680,000

Cash . . . . . . . . . . . . . . . . . . . . . . . . . . . . . . . . . . . . . . . . . . . . . . . . . . . . . . . . . . . . 680,000

Record ore deposit acquisition and its related costs.

5.

6.A Record the asset exchange: The book value on the exchange date is $180,000 (cost) − $40,000 (accu- mulated depreciation). The book value of the machinery given up in the exchange ($140,000) plus the $5,000 cash paid is less than the $210,000 value of the machine acquired. The entry to record this ex- change of assets that has commercial substance and recognizes the $65,000 gain ($210,000 − $140,000 − $5,000) is

Machinery (new) . . . . . . . . . . . . . . . . . . . . . . . . . . . . . . . . . . . . . . . . . . . . . . . . . . . . . . . 210,000

Accumulated Depreciation—Machinery (old) . . . . . . . . . . . . . . . . . . . . . . . . . . . . . . . . 40,000

Machinery (old) . . . . . . . . . . . . . . . . . . . . . . . . . . . . . . . . . . . . . . . . . . . . . . . . . . . . 180,000

Cash . . . . . . . . . . . . . . . . . . . . . . . . . . . . . . . . . . . . . . . . . . . . . . . . . . . . . . . . . . . . 5,000

Gain on Exchange of Assets . . . . . . . . . . . . . . . . . . . . . . . . . . . . . . . . . . . . . . . . . . 65,000

Record exchange with commercial substance of old equipment plus cash for new equipment.

APPENDIX

Exchanging Plant Assets 8A Many plant assets such as machinery, automobiles, and equipment are exchanged for newer assets. In a typical exchange of plant assets, a trade-in allowance is received on the old asset and the balance is paid in cash. Accounting for the exchange of assets depends on whether the transaction has commercial sub- stance. An exchange has commercial substance if the company’s future cash flows change as a result of the exchange of one asset for another asset. If an asset exchange has commercial substance, a gain or loss is recorded based on the difference between the book value of the asset(s) given up and the market value of the asset(s) received. Because most exchanges have commercial substance, we cover gains and losses for only that situation. Advanced courses cover exchanges without commercial substance.

Exchange with Commercial Substance: A Loss A company acquires $42,000 in new equipment. In exchange, the company pays $33,000 cash and trades in old equipment. The old equipment originally cost $36,000 and has accumulated depreciation of $20,000, which implies a $16,000 book value at the time of exchange. This exchange has commercial substance and the old equipment has a trade-in allowance of $9,000. This exchange yields a loss as computed in the middle (Loss) columns of Exhibit 8A.1; the loss is computed as Asset received − Assets given = $42,000 − $49,000 = $(7,000). We also can compute the loss as Trade-in allowance − Book value of assets given = $9,000 − $16,000 = $(7,000).

P5 Account for asset exchanges.

EXHIBIT 8A.1 Computing Gain or Loss on Asset Exchange with Commercial Substance

Asset Exchange Has Commercial Substance Loss Gain

Market value of asset received . . . . . . . . . . . . . . . . . . . . . . . . . . . . $42,000 $42,000

Book value of assets given:

Equipment ($36,000 − $20,000) . . . . . . . . . . . . . . . . . . . . . . . . $16,000 $16,000 Cash . . . . . . . . . . . . . . . . . . . . . . . . . . . . . . . . . . . . . . . . . . . . . . . 33,000 49,000 23,000 39,000

Gain (loss) on exchange . . . . . . . . . . . . . . . . . . . . . . . . . . . . . . . . . $(7,000) $ 3,000

324 Chapter 8 Accounting for Long-Term Assets

A company acquires $45,000 in new web servers. In exchange, the company trades in old web servers along with a cash payment. The old servers originally cost $30,000 and had accumulated depreciation of $23,400 at the time of the trade. Prepare entries to record the trade under two different assumptions where (a) the exchange has commercial substance and the old servers have a trade-in allowance of $3,000 and (b) the exchange has commercial substance and the old servers have a trade-in allowance of $7,000.

Solution

Equipment (new) . . . . . . . . . . . . . . . . . . . . . . . . . . . . . . . . . . . . . . . . . . . . . . . . . . . . . . . 45,000

Loss on Exchange of Assets . . . . . . . . . . . . . . . . . . . . . . . . . . . . . . . . . . . . . . . . . . . . . . 3,600

Accumulated Depreciation—Equipment (old) . . . . . . . . . . . . . . . . . . . . . . . . . . . . . . . . 23,400

Equipment (old) . . . . . . . . . . . . . . . . . . . . . . . . . . . . . . . . . . . . . . . . . . . . . . . . . . . 30,000

Cash ($45,000 − $3,000) . . . . . . . . . . . . . . . . . . . . . . . . . . . . . . . . . . . . . . . . . . . 42,000

Equipment (new) . . . . . . . . . . . . . . . . . . . . . . . . . . . . . . . . . . . . . . . . . . . . . . . . . . . . . . . 45,000

Accumulated Depreciation—Equipment (old) . . . . . . . . . . . . . . . . . . . . . . . . . . . . . . . . 23,400

Equipment (old) . . . . . . . . . . . . . . . . . . . . . . . . . . . . . . . . . . . . . . . . . . . . . . . . . . . 30,000

Cash ($45,000 − $7,000) . . . . . . . . . . . . . . . . . . . . . . . . . . . . . . . . . . . . . . . . . . . 38,000 Gain on Exchange of Assets . . . . . . . . . . . . . . . . . . . . . . . . . . . . . . . . . . . . . . . . . . 400

Asset Exchange

NEED-TO-KNOW 8-7

P5

Do More: QS 8-16, E 8-23, E 8-24

a.

b.

The entry to record this asset exchange and the loss follows.

Exchange with Commercial Substance: A Gain Let’s assume the same facts as in the preceding asset exchange except that the company pays $23,000 cash, not $33,000, with the trade-in. This exchange has commercial substance and the old equipment has a trade-in allowance of $19,000. This exchange yields a gain as computed in the right-most (Gain) columns of Exhibit 8A.1; the gain is computed as Asset received − Assets given = $42,000 − $39,000 = $3,000. We also can compute the gain as Trade-in allow- ance − Book value of assets given = $19,000 − $16,000 = $3,000. The entry to record this asset exchange and the gain follows.

Jan . 3 Equipment (new) . . . . . . . . . . . . . . . . . . . . . . . . . . . . . . . . . . . . 42,000 Loss on Exchange of Assets . . . . . . . . . . . . . . . . . . . . . . . . . . . 7,000

Accumulated Depreciation—Equipment (old) . . . . . . . . . . . . . 20,000 Equipment (old) . . . . . . . . . . . . . . . . . . . . . . . . . . . . . . . . 36,000 Cash . . . . . . . . . . . . . . . . . . . . . . . . . . . . . . . . . . . . . . . . . . 33,000

Record exchange (with commercial substance) of old equipment and cash for new equipment.

Assets = Liabilities + Equity +42,000 −7,000 +20,000 −36,000 −33,000

Point: “New” and “old” equipment are for illustration only. Both the debit and credit are to the same Equipment account.

Jan . 3 Equipment (new) . . . . . . . . . . . . . . . . . . . . . . . . . . . . . . . . . . . . 42,000 Accumulated Depreciation—Equipment (old) . . . . . . . . . . . . . 20,000 Equipment (old) . . . . . . . . . . . . . . . . . . . . . . . . . . . . . . . . 36,000 Cash . . . . . . . . . . . . . . . . . . . . . . . . . . . . . . . . . . . . . . . . . . 23,000

Gain on Exchange of Assets . . . . . . . . . . . . . . . . . . . . . . . 3,000

Record exchange (with commercial substance) of old equipment and cash for new equipment.

Assets = Liabilities + Equity +42,000 +3,000 +20,000 −36,000 −23,000

PLANT ASSETS Cost of plant assets: Normal, reasonable, and necessary costs in prepar- ing an asset for its intended use. If an asset is damaged during unpacking, the repairs are not added to its cost. Instead, they are charged to an expense account.

Summary: Cheat Sheet

Machinery and equipment: Cost includes purchase price, taxes, trans- portation, insurance while in transit, installation, assembly, and testing. Building: A purchased building’s costs include its purchase price, real estate fees, taxes, title fees, and attorney fees. A constructed building’s costs include construction costs and insurance during construction, but not insurance after it is completed.

Chapter 8 Accounting for Long-Term Assets 325

Depreciation expense = Depreciation per unit × Units produced in period

Depreciation per unit = Cost – Salvage valueStep 1

Step 2

Total units of production

Depreciation expense = Cost – Salvage value Useful life in periods

Land improvements: Additions to land that have limited useful lives. Examples are parking lots, driveways, and lights. Land: Has an indefinite (unlimited) life and costs include real estate com- missions, clearing, grading, and draining. Lump-sum purchase: Plant assets purchased as a group for a single lump- sum price. We allocate the cost to the assets acquired based on their rela- tive market (or appraised) values.

Depreciation: Process of allocating the cost of a plant asset to expense while it is in use. Salvage value: Estimate of the asset’s value at the end of its useful life. Useful life: Length of time a plant asset is to be used in operations.

Straight-line depreciation: Charges the same amount of depreciation expense in each period of the asset’s useful life.

Betterments (capital expenditure): Expenditures to make a plant asset more efficient or productive. Include upgrading components and adding additions onto plant assets. Extraordinary repairs (capital expenditure): Expenditures that extend the asset’s useful life beyond its original estimate.

Appraised Value Percent of Total Apportioned Cost

Building . . . $ 60,000 60% ($60,000/$100,000) $54,000 ($90,000 × 60%) Land . . . . . 40,000 40 ($40,000/$100,000) 36,000 ($90,000 × 40%) Totals . . . . . $100,000 100% $ 90,000

Building . . . . . . . . . . . . . . 54,000

Land . . . . . . . . . . . . . . . . 36,000

Cash . . . . . . . . . . . . 90,000

Record costs of plant assets.

Entry for lump-sum cash purchase:

Record depreciation expense:

Depreciation Expense . . . . . . . . . . . . . . . . . . . . . . . . . . . 1,800

Accumulated Depreciation—“Asset Type” . . . . . . . 1,800

Straight-line depreciation formula:

Asset book value (or book value): Computed as the asset’s total cost minus accumulated depreciation. Units-of-production depreciation: Charges a varying amount for each period depending on an asset’s usage. Units-of-production formula:

Double-declining-balance depreciation: Charges more depreciation in early years and less depreciation in later years. Double-declining-balance formula:

Straight-line rate = 100% ÷ Useful life

Double-declining-balance rate = 2 × Straight-line rate

Depreciation expense = Double-declining-balance rate × Beginning-period book value

Step 1

Step 2

Step 3

Depletion expense = Depletion per unit × Units extracted and sold in period

Depletion per unit = Cost – Salvage value

Total units of capacityStep 1

Step 2

Change in an accounting estimate: For plant assets, it is changing the estimate of useful life or salvage value. It only affects current and future depreciation expense. Do not go back and change prior years’ depreciation.

Straight-line depreciation after change in accounting estimate:

Book value − Revised salvage value Revised remaining useful life

Impairment: Permanent decline in the fair value of an asset relative to its book value.

Impairment Loss . . . . . . . . . . . . . . . . . . . . . . . . . . . . . . . . 50

Accumulated Depreciation—Equipment . . . . . . . . 50

Ordinary repairs (revenue expenditure): Expenditures to keep an asset in good operating condition. They do not increase useful life or productiv- ity. Include cleaning, changing oil, and minor repairs.

Repairs Expense . . . . . . . . . . . . . . . . . . . . . . . . . . . . . . . . 9,500

Cash . . . . . . . . . . . . . . . . . . . . . . . . . . . . . . . . . . . . . 9,500

Betterments and extraordinary repairs: These expenditures are “capitalized” by adding their costs to the plant asset.

“Plant Asset” . . . . . . . . . . . . . . . . . . . . . . . . . . . . . . . . . . . 1,800

Cash . . . . . . . . . . . . . . . . . . . . . . . . . . . . . . . . . . . . . 1,800

Before discarding, selling, or exchanging a plant asset: Must record depreciation up to that date.

Depreciation Expense . . . . . . . . . . . . . . . . . . . . . . . . . . . 500

Accumulated Depreciation—Equipment . . . . . . . . 500

Discarding fully depreciated asset:

Accumulated Depreciation—Machinery . . . . . . . . . . . . . 9,000

Machinery . . . . . . . . . . . . . . . . . . . . . . . . . . . . . . . . 9,000

Discarding partially depreciated asset: Loss is the book value (Cost – Accumulated depreciation) of the asset when discarded.

Accumulated Depreciation—Equipment . . . . . . . . . . . . . 6,500

Loss on Disposal of Equipment . . . . . . . . . . . . . . . . . . . . 1,500

Equipment . . . . . . . . . . . . . . . . . . . . . . . . . . . . . . . . 8,000

Sale of asset at book value: If sale price = book value, no gain or loss.

Cash . . . . . . . . . . . . . . . . . . . . . . . . . . . . . . . . . . . . . . . . . 3,000

Accumulated Depreciation—Equipment . . . . . . . . . . . . . 13,000

Equipment . . . . . . . . . . . . . . . . . . . . . . . . . . . . . . . . 16,000

Sale of asset above book value: If sale price > book value → gain.

Cash . . . . . . . . . . . . . . . . . . . . . . . . . . . . . . . . . . . . . . . . . 7,000

Accumulated Depreciation—Equipment . . . . . . . . . . . . . 13,000

Gain on Disposal of Equipment . . . . . . . . . . . . . . . . 4,000

Equipment . . . . . . . . . . . . . . . . . . . . . . . . . . . . . . . . 16,000

Sale of asset below book value: If sale price < book value → loss.

Cash . . . . . . . . . . . . . . . . . . . . . . . . . . . . . . . . . . . . . . . . . 2,500

Loss on Disposal of Equipment . . . . . . . . . . . . . . . . . . . . 500

Accumulated Depreciation—Equipment . . . . . . . . . . . . . 13,000

Equipment . . . . . . . . . . . . . . . . . . . . . . . . . . . . . . . . 16,000

NATURAL RESOURCES Natural resources: Assets that are physically consumed when used. Examples are standing timber, mineral deposits, and oil and gas fields. Depletion: Process of allocating the cost of a natural resource. Depletion formula:

Depletion expense (when all units extracted are sold):

Depletion Expense—Mineral Deposit . . . . . . . . . . . . . . . 170,000

Accumulated Depletion—Mineral Deposit . . . . . . . 170,000

Depletion expense (when not all units extracted are sold):

Depletion Expense—Mineral Deposit . . . . . . . . . . . . . . . 140,000

Ore Inventory . . . . . . . . . . . . . . . . . . . . . . . . . . . . . . . . . . 30,000

Accumulated Depletion—Mineral Deposit . . . . . . . 170,000

326 Chapter 8 Accounting for Long-Term Assets

INTANGIBLE ASSETS Intangible assets: Nonphysical assets (used in operations) that give com- panies long-term rights, privileges, or competitive advantages.

Patent: Exclusive right to manufacture and sell a patented item or to use a process for 20 years. Copyright: Exclusive right to publish and sell a musical, literary, or artis- tic work during the life of the creator plus 70 years.

Franchises or licenses: Rights to sell a product or service under specified conditions. Trademark or trade (brand) name: A symbol, name, phrase, or jingle identified with a company, product, or service. Goodwill: Amount by which a company’s value exceeds the value of its individual assets and liabilities (net assets). Goodwill is only recorded when an entire company or business segment is purchased. Not amortized, but tested for impairment. Right-of-use asset (lease): Rights the lessor grants to the lessee under terms of the lease. Leasehold improvements: Improvements to a leased (rented) property such as partitions, painting, and storefronts. The lessee amortizes these costs over the life of the lease or the life of the improvements, whichever is shorter.

Amortization: Intangible assets with limited useful lives require amorti- zation. It is similar to depreciation and uses the shorter of the legal life or useful life of the intangible for straight-line amortization.

Amortization Expense—Patents . . . . . . . . . . . . . . . . . . . . 2,500

Accumulated Amortization—Patents . . . . . . . . . . . 2,500

Accelerated depreciation method (308) Amortization (317) Asset book value (307) Betterments (312) Capital expenditures (311) Change in an accounting estimate (310) Copyright (318) Cost (304) Declining-balance method (308) Depletion (315) Depreciation (305) Extraordinary repairs (312) Franchises (318) Goodwill (318)

Impairment (310, 317) Inadequacy (306) Indefinite life (317) Intangible assets (317) Land improvements (304) Lease (318) Leasehold (318) Leasehold improvements (318) Lessee (318) Lessor (318) Licenses (318) Limited life (317) Modified Accelerated Cost Recovery

System (MACRS) (309)

Natural resources (315) Obsolescence (306) Ordinary repairs (312) Patent (317) Plant assets (303) Research and development costs (319) Revenue expenditures (311) Salvage value (305) Straight-line depreciation (306) Total asset turnover (320) Trademark or trade (brand) name (318) Units-of-production depreciation (307) Useful life (305)

Key Terms

Multiple Choice Quiz

1. A company paid $326,000 for property that included land, land improvements, and a building. The land was appraised at $175,000, the land improvements were appraised at $70,000, and the building was appraised at $105,000. What is the allocation of costs to the three assets? a. Land, $150,000; Land Improvements, $60,000; Building,

$90,000 b. Land, $163,000; Land Improvements, $65,200; Building,

$97,800 c. Land, $150,000; Land Improvements, $61,600; Building,

$92,400 d. Land, $159,000; Land Improvements, $65,200; Building,

$95,400 e. Land, $175,000; Land Improvements, $70,000; Building,

$105,000 2. A company purchased a truck for $35,000 on January 1,

2019. The truck is estimated to have a useful life of four years and a salvage value of $1,000. Assuming that the company uses straight-line depreciation, what is deprecia- tion expense for the year ended December 31, 2020? a. $8,750 c. $8,500 e. $25,500 b. $17,500 d. $17,000

3. A company purchased machinery for $10,800,000 on January 1, 2019. The machinery has a useful life of 10 years and an estimated salvage value of $800,000. What is depre- ciation expense for the year ended December 31, 2020, as- suming that the double-declining-balance method is used? a. $2,160,000 c. $1,728,000 e. $1,600,000 b. $3,888,000 d. $2,000,000

4. A company sold a machine that originally cost $250,000 for $120,000 when accumulated depreciation on the machine was $100,000. The gain or loss recorded on the sale of this machine is a. $0 gain or loss. d. $30,000 gain. b. $120,000 gain. e. $150,000 loss. c. $30,000 loss.

5. A company had average total assets of $500,000, gross sales of $575,000, and net sales of $550,000. The compa- ny’s total asset turnover is a. 1.15. d. 0.87. b. 1.10. e. 1.05. c. 0.91.

Chapter 8 Accounting for Long-Term Assets 327

A Superscript letter A denotes assignments based on Appendix 8A.

Icon denotes assignments that involve decision making.

1. What characteristics of a plant asset make it different from other assets?

2. What is the general rule for cost inclusion for plant assets? 3. What is different between land and land improvements? 4. Why is the cost of a lump-sum purchase allocated to the

individual assets acquired? 5. Does the balance in the Accumulated Depreciation—

Machinery account represent funds to replace the machin- ery when it wears out? If not, what does it represent?

6. Why is the Modified Accelerated Cost Recovery System not generally accepted for financial accounting purposes?

7. What is the difference between ordinary repairs and ex- traordinary repairs? How should each be recorded?

8. Identify events that might lead to disposal of a plant asset.

9. What is the process of allocating the cost of natural re- sources to expense as they are used?

10. Is the declining-balance method an acceptable way to com- pute depletion of natural resources? Explain.

11. What are the characteristics of an intangible asset? 12. What general procedures are applied in accounting for the

acquisition and potential cost allocation of intangible assets? 13. When do we know that a company has goodwill? When

can goodwill appear in a company’s balance sheet?

14. Assume that a company buys another business and pays for its goodwill. If the company plans to incur costs each year to maintain the value of the goodwill, must it also amortize this goodwill?

15. How is total asset turnover computed? Why would a financial statement user be interested in total asset turn- over?

16. On its recent balance sheet in Appendix A, Apple lists its plant assets as “Property, plant and equipment, net.” What does “net” mean in this title?

17. Refer to Google’s recent balance sheet in Appendix A. What is the book value of its total net property, plant, and equipment assets at December 31, 2017?

18. Refer to Samsung’s balance sheet in Appendix A. What does it title its plant assets? What is the book value of its plant assets at December 31, 2017?

19. Refer to Samsung’s December 31, 2017, balance sheet in Appendix A. What long- term assets discussed in this chapter are reported by the company?

20. Identify the main difference between (a) plant assets and current assets, (b) plant assets and inventory, and (c) plant assets and long-term investments.

Discussion Questions

APPLE

Samsung

Samsung

GOOGLE

ANSWERS TO MULTIPLE CHOICE QUIZ

1. b; 4. c;

Appraisal Total Value % Cost Allocated

Land . . . . . . . . . . . . . . . $175,000 50% $326,000 $163,000

Land improvements . . 70,000 20 326,000 65,200

Building . . . . . . . . . . . . 105,000 30 326,000 97,800

Totals . . . . . . . . . . . . . . $350,000 $326,000

2. c; ($35,000 − $1,000)∕4 years = $8,500 per year 3. c; 2019: $10,800,000 × (2 × 10%) = $2,160,000

2020: ($10,800,000 − $2,160,000) × (2 × 10%) = $1,728,000

Cost of machine . . . . . . . . . . . . . . . . . . . . $250,000

Accumulated depreciation . . . . . . . . . . . . 100,000

Book value . . . . . . . . . . . . . . . . . . . . . . . . 150,000

Cash received . . . . . . . . . . . . . . . . . . . . . . 120,000

Loss on sale . . . . . . . . . . . . . . . . . . . . . . . $ 30,000

5. b; $550,000∕$500,000 = 1.10

QUICK STUDY

QS 8-1 Cost of plant assets

C1

Kegler Bowling buys scorekeeping equipment with an invoice cost of $190,000. The electrical work re- quired for the installation costs $20,000. Additional costs are $4,000 for delivery and $13,700 for sales tax. During the installation, the equipment was damaged and the cost of repair was $1,850.

What is the total recorded cost of the scorekeeping equipment?

328 Chapter 8 Accounting for Long-Term Assets

On January 1, the Matthews Band pays $65,800 for sound equipment. The band estimates it will use this equipment for four years and perform 200 concerts. It estimates that after four years it can sell the equip- ment for $2,000. During the first year, the band performs 45 concerts.

Compute the first-year depreciation using the straight-line method.

QS 8-3 Straight-line depreciation

P1

On January 1, the Matthews Band pays $65,800 for sound equipment. The band estimates it will use this equipment for four years and perform 200 concerts. It estimates that after four years it can sell the equip- ment for $2,000. During the first year, the band performs 45 concerts.

Compute the first-year depreciation using the units-of-production method.

QS 8-4 Units-of-production depreciation P1

A building is acquired on January 1 at a cost of $830,000 with an estimated useful life of eight years and salvage value of $75,000. Compute depreciation expense for the first three years using the double- declining-balance method.

QS 8-5 Double-declining-balance method P1

On January 1, the Matthews Band pays $65,800 for sound equipment. The band estimates it will use this equipment for four years and after four years it can sell the equipment for $2,000. Matthews Band uses straight-line depreciation but realizes at the start of the second year that this equipment will last only a total of three years. The salvage value is not changed.

Compute the revised depreciation for both the second and third years.

QS 8-7 Computing revised depreciation

C2

Equipment has a book value of $16,000 and a fair value of $14,750. The decline in value meets the im- pairment test. Prepare the entry to record this $1,250 impairment.

QS 8-8 Recording plant asset impairment C2

Garcia Co. owns equipment that cost $76,800, with accumulated depreciation of $40,800. Record the sale of the equipment under the following three separate cases assuming Garcia sells the equipment for (1) $47,000 cash, (2) $36,000 cash, and (3) $31,000 cash.

QS 8-10 Disposal of assets P2

Listed below are costs (or discounts) to purchase or construct new plant assets. (1) Indicate whether the costs should be expensed or capitalized (meaning they are included in the cost of the plant assets on the balance sheet). (2) For costs that should be capitalized, indicate in which category of plant assets (Equipment, Building, or Land) the related costs should be recorded on the balance sheet. Expensed or Asset Capitalized Category 1. Wages paid to train employees to use new equipment. 2. Invoice cost paid for new equipment. 3. Early payment discount taken on the purchase of new equipment. 4. Realtor commissions incurred on land purchased. 5. Property taxes on land incurred after it was purchased. 6. Costs of oil for the truck used to deliver new equipment. 7. Costs to lay foundation for a new building. 8. Insurance on a new building during its construction.

QS 8-2 Assigning costs to plant assets

C1

1. Classify the following as either a revenue expenditure (RE) or a capital expenditure (CE). a. Paid $40,000 cash to replace a motor on equipment that extends its useful life by four years. b. Paid $200 cash per truck for the cost of their annual tune-ups. c. Paid $175 for the monthly cost of replacement filters on an air-conditioning system. d. Completed an addition to a building for $225,000 cash.

2. Prepare the journal entries to record the four transactions from part 1.

QS 8-9 Revenue and capital expenditures

C3

On October 1, Organic Farming purchases wind turbines for $140,000. The wind turbines are expected to last six years, have a salvage value of $20,000, and be depreciated using the straight-line method. 1. Compute depreciation expense for the last three months of the first year. 2. Compute depreciation expense for the second year.

QS 8-6 Straight-line, partial-year depreciation C2

Chapter 8 Accounting for Long-Term Assets 329

QS 8-13 Intangible assets and amortization P4

On January 1 of this year, Diaz Boutique pays $105,000 to modernize its store. Improvements include new floors, ceilings, wiring, and wall coverings. These improvements are estimated to yield benefits for 10 years. Diaz leases (does not own) its store and has eight years remaining on the lease. Prepare the entry to record (1) the cost of modernization and (2) amortization at the end of this current year.

QS 8-11 Natural resources and depletion

P3

Perez Company acquires an ore mine at a cost of $1,400,000. It incurs additional costs of $400,000 to ac- cess the mine, which is estimated to hold 1,000,000 tons of ore. The estimated value of the land after the ore is removed is $200,000. 1. Prepare the entry(ies) to record the cost of the ore mine. 2. Prepare the year-end adjusting entry if 180,000 tons of ore are mined and sold the first year.

Exercise 8-2 Recording costs of assets

C1

Cala Manufacturing purchases land for $390,000 as part of its plans to build a new plant. The company pays $33,500 to tear down an old building on the lot and $47,000 to fill and level the lot. It also pays con- struction costs of $1,452,200 for the new building and $87,800 for lighting and paving a parking area. Prepare a single journal entry to record these costs incurred by Cala, all of which are paid in cash.

Exercise 8-3 Lump-sum purchase of plant assets C1

Rodriguez Company pays $395,380 for real estate with land, land improvements, and a building. Land is appraised at $157,040; land improvements are appraised at $58,890; and the building is appraised at $176,670. Allocate the total cost among the three assets and prepare the journal entry to record the purchase.

Exercise 8-4 Straight-line depreciation

P1

Ramirez Company installs a computerized manufacturing machine in its factory at the beginning of the year at a cost of $43,500. The machine’s useful life is estimated at 10 years, or 385,000 units of product, with a $5,000 salvage value. During its second year, the machine produces 32,500 units of product. Determine the machine’s second-year depreciation under the straight-line method.

Identify the following as intangible assets (IA), natural resources (NR), or some other asset (O). a. Oil well b. Trademark c. Leasehold

d. Gold mine e. Building f. Copyright

g. Franchise h. Coal mine i. Salt mine

QS 8-12 Classifying assets

P3 P4

QS 8-16A Asset exchange

P5

Caleb Co. owns a machine that had cost $42,400 with accumulated depreciation of $18,400. Caleb exchanges the machine for a newer model that has a market value of $52,000. 1. Record the exchange assuming Caleb paid $30,000 cash and the exchange has commercial substance. 2. Record the exchange assuming Caleb paid $22,000 cash and the exchange has commercial substance.

Aneko Company reports the following: net sales of $14,800 for Year 2 and $13,990 for Year 1; end-of-year total assets of $19,100 for Year 2 and $17,900 for Year 1. (1) Compute total asset turnover for Year 2. (2) Aneko’s competitor has a turnover of 2.0. Is Aneko performing better or worse than its competitor based on total asset turnover?

QS 8-15 Computing total asset turnover A1

EXERCISES

Exercise 8-1 Cost of plant assets

C1

Rizio Co. purchases a machine for $12,500, terms 2∕10, n∕60, FOB shipping point. Rizio paid within the discount period and took the $250 discount. Transportation costs of $360 were paid by Rizio. The machine required mounting and power connections costing $895. Another $475 is paid to assemble the machine, and $40 of materials are used to get it into operation. During installation, the machine was damaged and $180 worth of repairs were made. Compute the cost recorded for this machine.

QS 8-14 Preparing an income statement

P1 P3 P4

Selected accounts from Westeros Co.’s adjusted trial balance for the year ended December 31 follow. Prepare its income statement.

Sales . . . . . . . . . . . . . . . . . . . . . . . . . . . . . . . . $30,000 Depreciation expense . . . . . . . . . . . . . . . . . . $ 5,000

Repairs expense . . . . . . . . . . . . . . . . . . . . . . . 500 Salaries expense . . . . . . . . . . . . . . . . . . . . . . 10,000

Depletion expense . . . . . . . . . . . . . . . . . . . . . 4,000 Amortization expense . . . . . . . . . . . . . . . . . . 2,000

330 Chapter 8 Accounting for Long-Term Assets

Ramirez Company installs a computerized manufacturing machine in its factory at the beginning of the year at a cost of $43,500. The machine’s useful life is estimated at 10 years, or 385,000 units of product, with a $5,000 salvage value. During its second year, the machine produces 32,500 units of product. Determine the machine’s second-year depreciation using the units-of-production method.

Exercise 8-5 Units-of-production depreciation P1

Ramirez Company installs a computerized manufacturing machine in its factory at the beginning of the year at a cost of $43,500. The machine’s useful life is estimated at 10 years, or 385,000 units of product, with a $5,000 salvage value. During its second year, the machine produces 32,500 units of product. Determine the machine’s second-year depreciation using the double-declining-balance method.

Exercise 8-6 Double-declining-balance depreciation P1

NewTech purchases computer equipment for $154,000 to use in operating activities for the next four years. It estimates the equipment’s salvage value at $25,000. Prepare a table showing depreciation and book value for each of the four years assuming straight-line depreciation.

Exercise 8-7 Straight-line depreciation

P1

On April 1, Cyclone Co. purchases a trencher for $280,000. The machine is expected to last five years and have a salvage value of $40,000. Compute depreciation expense at December 31 for both the first year and second year assuming the company uses the straight-line method.

Exercise 8-11 Straight-line, partial-year depreciation C2

NewTech purchases computer equipment for $154,000 to use in operating activities for the next four years. It estimates the equipment’s salvage value at $25,000. Prepare a table showing depreciation and book value for each of the four years assuming double-declining-balance depreciation.

Exercise 8-8 Double-declining-balance depreciation P1

On April 1, Cyclone Co. purchases a trencher for $280,000. The machine is expected to last five years and have a salvage value of $40,000. Compute depreciation expense at December 31 for both the first year and second year assuming the company uses the double-declining-balance method.

Exercise 8-12 Double-declining- balance, partial-year depreciation C2

Apex Fitness Club uses straight-line depreciation for a machine costing $23,860, with an estimated four- year life and a $2,400 salvage value. At the beginning of the third year, Apex determines that the machine has three more years of remaining useful life, after which it will have an estimated $2,000 salvage value. Compute (1) the machine’s book value at the end of its second year and (2) the amount of depreciation for each of the final three years given the revised estimates.

Exercise 8-13 Revising depreciation

C2

Check (2) $3,710

Oki Company pays $264,000 for equipment expected to last four years and have a $29,000 salvage value. Prepare journal entries to record the following costs related to the equipment. 1. Paid $22,000 cash for a new component that increased the equipment’s productivity. 2. Paid $6,250 cash for minor repairs necessary to keep the equipment working well. 3. Paid $14,870 cash for significant repairs to increase the useful life of the equipment from four to

seven years.

Exercise 8-14 Ordinary repairs, extraordinary repairs, and betterments

C3

Tory Enterprises pays $238,400 for equipment that will last five years and have a $43,600 salvage value. By using the equipment in its operations for five years, the company expects to earn $88,500 annually, after deducting all expenses except depreciation. Prepare a table showing income before depreciation, depreciation expense, and net (pretax) income for each year and for the total five-year period, assuming straight-line depreciation is used.

Exercise 8-9 Straight-line depreciation and income effects

P1

Tory Enterprises pays $238,400 for equipment that will last five years and have a $43,600 salvage value. By using the equipment in its operations for five years, the company expects to earn $88,500 annually, after deducting all expenses except depreciation. Prepare a table showing income before depreciation, depreciation expense, and net (pretax) income for each year and for the total five-year period, assuming double-declining-balance depreciation is used.

Exercise 8-10 Double-declining-balance depreciation P1

Check Year 3 NI, $54,170

Chapter 8 Accounting for Long-Term Assets 331

Exercise 8-16 Disposal of assets

P2

Diaz Company owns a machine that cost $250,000 and has accumulated depreciation of $182,000. Prepare the entry to record the disposal of the machine on January 1 in each separate situation. 1. The machine needed extensive repairs and was not worth repairing. Diaz disposed of the machine,

receiving nothing in return. 2. Diaz sold the machine for $35,000 cash. 3. Diaz sold the machine for $68,000 cash. 4. Diaz sold the machine for $80,000 cash.

Exercise 8-18 Depletion of natural resources

P3

Montana Mining Co. pays $3,721,000 for an ore deposit containing 1,525,000 tons. The company installs machinery in the mine costing $213,500. Both the ore and machinery will have no salvage value after the ore is completely mined. Montana mines and sells 166,200 tons of ore during the year. Prepare the year- end entries to record both the ore deposit depletion and the mining machinery depreciation. Mining machinery depreciation should be in proportion to the mine’s depletion.

Exercise 8-19 Amortization of intangible assets P4

Milano Gallery purchases the copyright on a painting for $418,000 on January 1. The copyright is good for 10 more years, after which the copyright will expire and anyone can make prints. The company plans to sell prints for 11 years. Prepare entries to record the purchase of the copyright on January 1 and its an- nual amortization on December 31.

Exercise 8-20 Goodwill

P4

Robinson Company purchased Franklin Company at a price of $2,500,000. The fair market value of the net assets purchased equals $1,800,000. 1. What is the amount of goodwill that Robinson records at the purchase date? 2. Does Robinson amortize goodwill at year-end for financial reporting purposes? If so, over how many

years is it amortized? 3. Robinson believes that its employees provide superior customer service, and through their efforts,

Robinson believes it has created $900,000 of goodwill. Should Robinson Company record this goodwill?

Exercise 8-17 Partial-year depreciation; disposal of plant asset

P2

Rayya Co. purchases a machine for $105,000 on January 1, 2019. Straight-line depreciation is taken each year for four years assuming a seven-year life and no salvage value. The machine is sold on July 1, 2023, during its fifth year of service. Prepare entries to record the partial year’s depreciation on July 1, 2023, and to record the sale under each separate situation. 1. The machine is sold for $45,500 cash. 2. The machine is sold for $25,000 cash.

Martinez Company owns a building that appears on its prior year-end balance sheet at its original $572,000 cost less $429,000 accumulated depreciation. The building is depreciated on a straight-line basis assuming a 20-year life and no salvage value. During the first week in January of the current calendar year, major structural repairs are completed on the building at a $68,350 cost. The repairs extend its useful life for 5 years beyond the 20 years originally estimated. 1. Determine the building’s age (plant asset age) as of the prior year-end balance sheet date. 2. Prepare the entry to record the cost of the structural repairs that are paid in cash. 3. Determine the book value of the building immediately after the repairs are recorded. 4. Prepare the entry to record the current calendar year’s depreciation.

Check (3) $211,350

Exercise 8-15 Extraordinary repairs; plant asset age

C3

Lok Co. reports net sales of $5,856,480 for Year 2 and $8,679,690 for Year 3. End-of-year balances for total assets are Year 1, $1,686,000; Year 2, $1,800,000; and Year 3, $1,982,000. (a) Compute Lok’s total asset turnover for Year 2 and Year 3. (b) Lok’s competitor has a turnover of 3.0. Is Lok performing better or worse than its competitor on the basis of total asset turnover?

Exercise 8-22 Evaluating efficient use of assets A1

Exercise 8-21 Preparing a balance sheet

P1 P3 P4

Selected accounts from Gregor Co.’s adjusted trial balance for the year ended December 31 follow. Prepare a classified balance sheet.

Total equity . . . . . . . . . . . . . . . . . . . . . . . . . . $50,000 Accounts payable… . . . . . . . . . . . . . . . . . . . . . . . . . $ 2,000

Patents . . . . . . . . . . . . . . . . . . . . . . . . . . . . . 4,000 Accumulated depreciation—Equipment . . . . . . . . . 13,000

Cash . . . . . . . . . . . . . . . . . . . . . . . . . . . . . . . 6,000 Notes payable (due in 9 years) . . . . . . . . . . . . . . . . 11,000

Land . . . . . . . . . . . . . . . . . . . . . . . . . . . . . . . 30,000 Goodwill . . . . . . . . . . . . . . . . . . . . . . . . . . . . . . . . . . 5,000

Equipment . . . . . . . . . . . . . . . . . . . . . . . . . . . 20,000 Accumulated depletion—Silver mine . . . . . . . . . . . . 3,000

Silver mine . . . . . . . . . . . . . . . . . . . . . . . . . . 15,000 Accumulated amortization—Patents . . . . . . . . . . . . 1,000

332 Chapter 8 Accounting for Long-Term Assets

On January 2, Bering Co. disposes of a machine costing $44,000 with accumulated depreciation of $24,625. Prepare the entries to record the disposal under each separate situation. 1. The machine is sold for $18,250 cash. 2. The machine is traded in for a new machine having a $60,200 cash price. A $25,000 trade-in allowance

is received, and the balance is paid in cash. Assume the asset exchange has commercial substance. 3. The machine is traded in for a new machine having a $60,200 cash price. A $15,000 trade-in allowance

is received, and the balance is paid in cash. Assume the asset exchange has commercial substance.

Exercise 8-24A Recording plant asset disposals

P5

Check (3) Dr. Loss on Exchange, $4,375

Gilly Construction trades in an old tractor for a new tractor, receiving a $29,000 trade-in allowance and paying the remaining $83,000 in cash. The old tractor had cost $96,000 and had accumulated depreciation of $52,500. Answer the following questions assuming the exchange has commercial substance. 1. What is the book value of the old tractor at the time of exchange? 2. What is the loss on this asset exchange? 3. What amount should be recorded (debited) in the asset account for the new tractor?

Exercise 8-23A Exchanging assets

P5

Check (2) $14,500

PROBLEM SET A

Problem 8-1A Plant asset costs; depreciation methods

C1 P1

Timberly Construction makes a lump-sum purchase of several assets on January 1 at a total cash price of $900,000. The estimated market values of the purchased assets are building, $508,800; land, $297,600; land improvements, $28,800; and four vehicles, $124,800.

Required

1. Allocate the lump-sum purchase price to the separate assets purchased. Prepare the journal entry to record the purchase.

2. Compute the first-year depreciation expense on the building using the straight-line method, assuming a 15-year life and a $27,000 salvage value.

3. Compute the first-year depreciation expense on the land improvements assuming a five-year life and double-declining-balance depreciation.

Analysis Component

4. Compared to straight-line depreciation, does accelerated depreciation result in payment of less total taxes over the asset’s life?

Check (2) $30,000

(3) $10,800

Problem 8-2A Depreciation methods

P1

A machine costing $257,500 with a four-year life and an estimated $20,000 salvage value is installed in Luther Company’s factory on January 1. The factory manager estimates the machine will produce 475,000 units of product during its life. It actually produces the following units: 220,000 in Year 1, 124,600 in Year 2, 121,800 in Year 3, and 15,200 in Year 4. The total number of units produced by the end of Year 4 exceeds the original estimate—this difference was not predicted. Note: The machine cannot be depreci- ated below its estimated salvage value.

Required

Prepare a table with the following column headings and compute depreciation for each year (and total depreciation of all years combined) for the machine under each depreciation method.

Year Straight-Line Units-of-Production Double-Declining-Balance Check Year 4: units-of- production depreciation, $4,300; DDB depreciation, $12,187

Problem 8-3A Asset cost allocation; straight-line depreciation

C1 P1

On January 1, Mitzu Co. pays a lump-sum amount of $2,600,000 for land, Building 1, Building 2, and Land Improvements 1. Building 1 has no value and will be demolished. Building 2 will be an office and is appraised at $644,000, with a useful life of 20 years and a $60,000 salvage value. Land Improvements 1 is valued at $420,000 and is expected to last another 12 years with no salvage value. The land is valued at $1,736,000. The company also incurs the following additional costs.

Cost to demolish Building 1 . . . . . . . . . . . . . . . . . . $ 328,400 Cost of additional land grading . . . . . . . . . . . . . . . . . . . $175,400

Cost to construct Building 3, having a useful life Cost of new Land Improvements 2, having a 20-year of 25 years and a $392,000 salvage value . . . . 2,202,000 useful life and no salvage value . . . . . . . . . . . . . . . . 164,000

Chapter 8 Accounting for Long-Term Assets 333

Required

1. Prepare a table with the following column headings: Land, Building 2, Building 3, Land Improvements 1, and Land Improvements 2. Allocate the costs incurred by Mitzu to the appropriate columns and total each column.

2. Prepare a single journal entry to record all the incurred costs assuming they are paid in cash on January 1. 3. Using the straight-line method, prepare the December 31 adjusting entries to record depreciation for

the first year these assets were in use.

Check (1) Land costs, $2,115,800; Building 2 costs, $598,000

(3) Depr.—Land Improv. 1 and 2, $32,500 and $8,200

Problem 8-4A Computing and revising depreciation; revenue and capital expenditures

C1 C2 C3

Champion Contractors completed the following transactions involving equipment.

Year 1

Jan. 1 Paid $287,600 cash plus $11,500 in sales tax and $1,500 in transportation (FOB shipping point) for a new loader. The loader is estimated to have a four-year life and a $20,600 salvage value. Loader costs are recorded in the Equipment account.

3 Paid $4,800 to install air-conditioning in the loader to enable operations under harsher condi- tions. This increased the estimated salvage value of the loader by another $1,400.

Dec. 31 Recorded annual straight-line depreciation on the loader.

Year 2

Jan. 1 Paid $5,400 to overhaul the loader’s engine, which increased the loader’s estimated useful life by two years.

Feb. 17 Paid $820 for minor repairs to the loader after the operator backed it into a tree. Dec. 31 Recorded annual straight-line depreciation on the loader.

Required

Prepare journal entries to record these transactions and events.

Check Dec. 31, Year 1: Dr. Depr. Expense—Equip., $70,850

Dec. 31, Year 2: Dr. Depr. Expense—Equip., $43,590

Yoshi Company completed the following transactions and events involving its delivery trucks.

Year 1

Jan. 1 Paid $20,515 cash plus $1,485 in sales tax for a new delivery truck estimated to have a five-year life and a $2,000 salvage value. Delivery truck costs are recorded in the Trucks account.

Dec. 31 Recorded annual straight-line depreciation on the truck.

Year 2

Dec. 31 The truck’s estimated useful life was changed from five to four years, and the estimated salvage value was increased to $2,400. Recorded annual straight-line depreciation on the truck.

Year 3

Dec. 31 Recorded annual straight-line depreciation on the truck. 31 Sold the truck for $5,300 cash.

Required

Prepare journal entries to record these transactions and events.

Problem 8-5A Computing and revising depreciation; selling plant assets

C2 P1 P2

Check Dec. 31, Year 2: Dr. Depr. Expense—Trucks, $5,200

Dec. 31, Year 3: Dr. Loss on Disposal of Trucks, $2,300

Onslow Co. purchased a used machine for $178,000 cash on January 2. On January 3, Onslow paid $2,840 to wire electricity to the machine and an additional $1,160 to secure it in place. The machine will be used for six years and have a $14,000 salvage value. Straight-line depreciation is used. On December 31, at the end of its fifth year in operations, it is disposed of.

Required

1. Prepare journal entries to record the machine’s purchase and the costs to ready it for use. Cash is paid for all costs incurred.

2. Prepare journal entries to record depreciation of the machine at December 31 of (a) its first year of operations and (b) the year of its disposal.

3. Prepare journal entries to record the machine’s disposal under each separate situation: (a) it is sold for $15,000 cash; (b) it is sold for $50,000 cash; and (c) it is destroyed in a fire and the insurance company pays $30,000 cash to settle the loss claim.

Problem 8-6A Disposal of plant assets

C1 P1 P2

Check (2b) Depr. Exp., $28,000

(3c) Dr. Loss from Fire, $12,000

334 Chapter 8 Accounting for Long-Term Assets

Problem 8-7A Natural resources

P3

On July 23 of the current year, Dakota Mining Co. pays $4,715,000 for land estimated to contain 5,125,000 tons of recoverable ore. It installs and pays for machinery costing $410,000 on July 25. The company removes and sells 480,000 tons of ore during its first five months of operations ending on December 31. Depreciation of the machinery is in proportion to the mine’s depletion as the machinery will be abandoned after the ore is mined.

Required

Prepare entries to record (a) the purchase of the land, (b) the cost and installation of machinery, (c) the first five months’ depletion assuming the land has a net salvage value of zero after the ore is mined, and (d) the first five months’ depreciation on the machinery.

Analysis Component

(e) If the machine will be used at another site when extraction is complete, how would we depreciate this machine?

Check (c) Depletion, $441,600 (d) Depreciation, $38,400

Problem 8-8A Right-of-use lease asset

P4

On January 1, Falk Company signed a contract to lease space in a building for three years. The current value of the three lease payments is $270,000.

Required

Prepare entries for Falk to record (a) the lease asset and obligation at January 1 and (b) the $90,000 straight-line amortization at December 31 of the first year.

Nagy Company makes a lump-sum purchase of several assets on January 1 at a total cash price of $1,800,000. The estimated market values of the purchased assets are building, $890,000; land, $427,200; land improvements, $249,200; and five trucks, $213,600.

Required

1. Allocate the lump-sum purchase price to the separate assets purchased. Prepare the journal entry to record the purchase.

2. Compute the first-year depreciation expense on the building using the straight-line method, assuming a 12-year life and a $120,000 salvage value.

3. Compute the first-year depreciation expense on the land improvements assuming a 10-year life and double-declining-balance depreciation.

Analysis Component

4. Compared to straight-line depreciation, does accelerated depreciation result in payment of less total taxes over the asset’s life?

Check (2) $65,000

(3) $50,400

Problem 8-2B Depreciation methods

P1

On January 1, Manning Co. purchases and installs a new machine costing $324,000 with a five-year life and an estimated $30,000 salvage value. Management estimates the machine will produce 1,470,000 units of product during its life. Actual production of units is as follows: 355,600 in Year 1, 320,400 in Year 2, 317,000 in Year 3, 343,600 in Year 4, and 138,500 in Year 5. The total number of units produced by the end of Year 5 exceeds the original estimate—this difference was not predicted. Note: The machine cannot be depreciated below its estimated salvage value.

Required

Prepare a table with the following column headings and compute depreciation for each year (and total depreciation of all years combined) for the machine under each depreciation method.

Year Straight-Line Units-of-Production Double-Declining-Balance Check DDB Depreciation, Year 3, $46,656; U-of-P Depreciation, Year 4, $68,720

Problem 8-3B Asset cost allocation; straight-line depreciation

C1 P1

On January 1, ProTech Co. pays a lump-sum amount of $1,550,000 for land, Building A, Building B, and Land Improvements B. Building A has no value and will be demolished. Building B will be an office and is appraised at $482,800, with a useful life of 15 years and a $99,500 salvage value. Land Improvements B is valued at $142,000 and is expected to last another five years with no salvage value. The land is valued at $795,200. The company also incurs the following additional costs.

PROBLEM SET B

Problem 8-1B Plant asset costs; depreciation methods

C1 P1

Chapter 8 Accounting for Long-Term Assets 335

Cost to demolish Building A . . . . . . . . . . . . . . . . . . . $ 122,000 Cost of additional land grading . . . . . . . . . . . . . . . . . . . . . $174,500

Cost to construct Building C, having a useful life . . Cost of new Land Improvements C, having a 10-year of 20 years and a $258,000 salvage value . . . . . 1,458,000 useful life and no salvage value . . . . . . . . . . . . . . . . . . 103,500

Required

1. Prepare a table with the following column headings: Land, Building B, Building C, Land Improvements B, and Land Improvements C. Allocate the costs incurred by ProTech to the appropriate columns and total each column.

2. Prepare a single journal entry to record all incurred costs assuming they are paid in cash on January 1. 3. Using the straight-line method, prepare the December 31 adjusting entries to record depreciation for

the first year these assets were in use.

Check (1) Land costs, $1,164,500; Building B costs, $527,000

(3) Depr.—Land Improv. B and C, $31,000 and $10,350

Mercury Delivery Service completed the following transactions involving equipment.

Year 1

Jan. 1 Paid $25,860 cash plus $1,810 in sales tax for a new delivery van that was estimated to have a five-year life and a $3,670 salvage value. Van costs are recorded in the Equipment account.

3 Paid $1,850 to install sorting racks in the van for more accurate and quicker delivery of pack- ages. This increases the estimated salvage value of the van by another $230.

Dec. 31 Recorded annual straight-line depreciation on the van.

Year 2

Jan. 1 Paid $2,064 to overhaul the van’s engine, which increased the van’s useful life by two years. May 10 Paid $800 for minor repairs to the van after the driver backed it into a loading dock. Dec. 31 Recorded annual straight-line depreciation on the van.

Required

Prepare journal entries to record these transactions and events.

Problem 8-4B Computing and revising depreciation; revenue and capital expenditures

C1 C2 C3

Check Dec. 31, Year 1: Dr. Depr. Expense—Equip., $5,124

Dec. 31, Year 2: Dr. Depr. Expense—Equip., $3,760

York Instruments completed the following transactions and events involving its machinery.

Year 1

Jan. 1 Paid $107,800 cash plus $6,470 in sales tax for a new machine. The machine is estimated to have a six-year life and a $9,720 salvage value.

Dec. 31 Recorded annual straight-line depreciation on the machinery.

Year 2

Dec. 31 The machine’s estimated useful life was changed from six to four years, and the estimated salvage value was increased to $14,345. Recorded annual straight-line depreciation on the machinery.

Year 3

Dec. 31 Recorded annual straight-line depreciation on the machinery. 31 Sold the machine for $25,240 cash.

Required

Prepare journal entries to record these transactions and events.

Problem 8-5B Computing and revising depreciation; selling plant assets

C2 P1 P2

Check Dec. 31, Year 2: Dr. Depr. Expense— Machinery, $27,500

Dec. 31, Year 3: Dr. Loss on Disposal of Machinery, $16,605

On January 1, Walker purchased a used machine for $150,000. On January 4, Walker paid $3,510 to wire electricity to the machine and an additional $4,600 to secure it in place. The machine will be used for seven years and have an $18,110 salvage value. Straight-line depreciation is used. On December 31, at the end of its sixth year of use, the machine is disposed of.

Required

1. Prepare journal entries to record the machine’s purchase and the costs to ready it for use. Cash is paid for all costs incurred.

2. Prepare journal entries to record depreciation of the machine at December 31 of (a) its first year of operations and (b) the year of its disposal.

3. Prepare journal entries to record the machine’s disposal under each separate situation: (a) it is sold for $28,000 cash; (b) it is sold for $52,000 cash; and (c) it is destroyed in a fire and the insurance company pays $25,000 cash to settle the loss claim.

Check (2b) Depr. Exp., $20,000

(3c) Dr. Loss from Fire, $13,110

Problem 8-6B Disposal of plant assets

C1 P1 P2

336 Chapter 8 Accounting for Long-Term Assets

Problem 8-7B Natural resources

P3

On February 19 of the current year, Quartzite Co. pays $5,400,000 for land estimated to contain 4 million tons of recoverable ore. It installs and pays for machinery costing $400,000 on March 21. The company removes and sells 254,000 tons of ore during its first nine months of operations ending on December 31. Depreciation of the machinery is in proportion to the mine’s depletion as the machinery will be abandoned after the ore is mined.

Required

Prepare entries to record (a) the purchase of the land, (b) the cost and installation of the machinery, (c) the first nine months’ depletion assuming the land has a net salvage value of zero after the ore is mined, and (d) the first nine months’ depreciation on the machinery.

Analysis Component

(e) If the machine will be used at another site when extraction is complete, how would we depreciate this machine?

Check (c) Depletion, $342,900 (d) Depreciation, $25,400

Problem 8-8B Right-of-use lease asset

P4

On January 1, Mason Co. entered into a three-year lease on a building. The current value of the three lease payments is $60,000.

Required

Prepare entries for Mason to record (a) the lease asset and obligation at January 1 and (b) the $20,000 straight-line amortization at December 31 of the first year.

SERIAL PROBLEM Business Solutions

A1 P1

This serial problem began in Chapter 1 and continues through most of the book. If previous chapter seg- ments were not completed, the serial problem can begin at this point.

SP 8 Selected ledger account balances for Business Solutions follow.

©Alexander Image/Shutterstock

For Three Months For Three Months Ended December 31, 2019 Ended March 31, 2020

Office equipment . . . . . . . . . . . . . . . . . . . . . . . . . . . . . . . $ 8,000 $ 8,000

Accumulated depreciation—Office equipment . . . . . . . . 400 800

Computer equipment . . . . . . . . . . . . . . . . . . . . . . . . . . . . 20,000 20,000

Accumulated depreciation—Computer equipment . . . . . 1,250 2,500

Total revenue . . . . . . . . . . . . . . . . . . . . . . . . . . . . . . . . . . 31,284 44,000

Total assets . . . . . . . . . . . . . . . . . . . . . . . . . . . . . . . . . . . . 83,460 120,268

Required

1. Assume that Business Solutions does not acquire additional office equipment or computer equipment in 2020. Compute amounts for the year ended December 31, 2020, for Depreciation Expense—Office Equipment and for Depreciation Expense—Computer Equipment (assume use of the straight-line method).

2. Given the assumptions in part 1, what is the book value of both the office equipment and the computer equipment as of December 31, 2020?

3. Compute the three-month total asset turnover for Business Solutions as of March 31, 2020. Use total revenue for the numerator and average the December 31, 2019, total assets and the March 31, 2020, total assets for the denominator. Interpret its total asset turnover if competitors average 2.5 for annual periods. (Round turnover to two decimals.)

Check (3) Three-month (annual) turnover = 0.43 (1.73 annual)

COMPANY ANALYSIS A1

Accounting Analysis

AA 8-1 Refer to Apple’s financial statements in Appendix A to answer the following. 1. What percent of the original cost of Apple’s Property, Plant and Equipment account remains to be

depreciated as of (a) September 30, 2017, and (b) September 24, 2016? Assume these assets have no salvage value and the entire account is depreciable. Hint: Accumulated Depreciation is listed under “Property, Plant and Equipment” in the notes to Apple’s financial statements in Appendix A.APPLE

Chapter 8 Accounting for Long-Term Assets 337

2. Much research and development is needed to create the next iPhone. Does Apple capitalize and amor- tize research and development costs over the life of the product, or are research and development costs expensed as incurred?

3. Compute Apple’s total asset turnover for the year ended (a) September 30, 2017, and (b) September 24, 2016. Assume total assets at September 26, 2015, are $290,345 ($ millions).

4. Using the results in part 3, is the change in Apple’s asset turnover favorable or unfavorable?

COMPARATIVE ANALYSIS A1

Required

1. Compute total asset turnover for the most recent two years for Apple and Google using the data shown. 2. In the current year, which company is more efficient in generating net sales given total assets? 3. Does each company’s asset turnover underperform or outperform the industry (assumed) asset turn-

over of 0.5 for (a) Apple and (b) Google?

AA 8-2 Comparative figures for Apple and Google follow.

Apple Google

Current One Year Two Years Current One Year Two Years $ millions Year Prior Prior Year Prior Prior

Total assets . . . . . . . . . . . . $375,319 $321,686 $290,345 $197,295 $167,497 $147,461

Net sales . . . . . . . . . . . . . . 229,234 215,639 233,715 110,855 90,272 74,989

APPLE GOOGLE

Required

1. Compute total asset turnover for the most recent two years for Samsung using the data shown. 2. Is the change in Samsung’s asset turnover favorable or unfavorable? 3. For the current year, is Samsung’s asset turnover better or worse than the asset turnover for (a) Apple

and (b) Google?

Samsung Apple Google

Current Prior Two Years Current Prior Current Prior In millions Year Year Prior Year Year Year Year

Total assets . . . W301,752,090 W262,174,324 W242,179,521 $375,319 $321,686 $197,295 $167,497

Net sales . . . . . 239,575,376 201,866,745 200,653,482 229,234 215,639 110,855 90,272

AA 8-3 Comparative figures for Samsung, Apple, and Google follow. GLOBAL ANALYSIS A1

Samsung APPLE GOOGLE

ETHICS CHALLENGE C1

BTN 8-1 Flo Choi owns a small business and manages its accounting. Her company just finished a year in which a large amount of borrowed funds was invested in a new building addition as well as in equip- ment and fixture additions. Choi’s banker requires her to submit semiannual financial statements so he can monitor the financial health of her business. He has warned her that if profit margins erode, he might raise the interest rate on the borrowed funds to reflect the increased loan risk from the bank’s point of view. Choi knows profit margin is likely to decline this year. As she prepares year-end adjusting entries, she decides to apply the following depreciation rule: All asset additions are considered to be in use on the first day of the following month. (The previous rule assumed assets are in use on the first day of the month nearest to the purchase date.)

Beyond the Numbers

338 Chapter 8 Accounting for Long-Term Assets

Required

1. Identify decisions that managers like Choi must make in applying depreciation methods. 2. Is Choi’s rule an ethical violation, or is it a legitimate decision in computing depreciation? 3. How will Choi’s new depreciation rule affect the profit margin of her business?

BTN 8-2 Teams are to select an industry, and each team member is to select a different company in that industry. Each team member is to acquire the financial statements (Form 10-K) of the company selected— see the company’s website or the SEC’s EDGAR database (SEC.gov). Use the financial statements to compute total asset turnover. Communicate with teammates via a meeting, e-mail, or telephone to discuss the meaning of this ratio, how different companies compare to each other, and the industry norm. The team must prepare a one-page report that describes the ratios for each company and identifies the conclu- sions reached during the team’s discussion.

COMMUNICATING IN PRACTICE A1

BTN 8-3 Access the Yahoo! (renamed as Altaba, ticker: AABA) 10-K report for the year ended December 31, 2016, filed on March 1, 2017, at SEC.gov.

Required

1. What amount of goodwill is reported on Yahoo!’s balance sheet? What percentage of total assets does its goodwill represent? Is goodwill a major asset for Yahoo!? Explain.

2. Compute the change in goodwill from December 31, 2015, to December 31, 2016. Comment on the change in goodwill over this period.

3. Locate Note 6 to its financial statements. What are the three categories of intangible assets that Yahoo! reports at December 31, 2016? What proportion of total assets do the intangibles represent?

4. What does Yahoo! indicate is the life of “Tradenames, trademarks, and domain names” according to its Note 6?

TAKING IT TO THE NET P4

BTN 8-4 Each team member is to become an expert on one depreciation method to facilitate teammates’ understanding of that method. Follow these procedures: a. Each team member is to select an area of expertise from one of the following depreciation methods:

straight-line, units-of-production, or double-declining-balance. b. Expert teams are to be formed from those who have selected the same area of expertise. The instructor

will identify the location where each expert team meets. c. Using the following data, expert teams are to collaborate and develop a presentation answering the

requirements. Expert team members must write the presentation in a format they can show to their learning teams.

Data and Requirements On January 8, 2017, Whitewater Riders purchases a van to transport rafters back to the point of departure at the conclusion of the rafting adventures they operate. The cost of the van is $44,000. It has an estimated salvage value of $2,000 and is expected to be used for four years and driven 60,000 miles. The van is driven 12,000 miles in 2017; 18,000 miles in 2018; 21,000 in 2019; and 10,000 in 2020. 1. Compute the annual depreciation expense for each year of the van’s estimated useful life. 2. Explain when and how annual depreciation is recorded. 3. Explain the impact on income of this depreciation method versus others over the van’s life. 4. Identify the van’s book value for each year of its life and illustrate the reporting of this amount for

any one year. d. Re-form original learning teams. In rotation, experts are to present to their teams the results from

part c. Experts are to encourage and respond to questions.

TEAMWORK IN ACTION P1

Point: This activity can follow an overview of each method. Step 1 allows for three areas of exper- tise. Larger teams will have some duplication of areas, but the straight-line choice should not be duplicated. Expert teams can use the book and consult with the instructor.

Chapter 8 Accounting for Long-Term Assets 339

BTN 8-5 Review the chapter’s opening feature involving Deb and Dan Carey and their company, New Glarus Brewing Company. Assume that the company currently has net sales of $8,000,000 and that it is planning an expansion that will increase net sales by $4,000,000. To accomplish this expansion, the com- pany must increase its average total assets from $2,500,000 to $3,000,000.

Required

1. Compute the company’s total asset turnover under (a) current conditions and (b) proposed conditions. 2. Evaluate and comment on the merits of the proposal given the analysis in part 1. Identify any concerns

we would express about the proposal.

ENTREPRENEURIAL DECISION A1

BTN 8-6 Team up with one or more classmates for this activity. Identify companies in your community or area that must account for at least one of the following assets: natural resource, patent, lease, leasehold improvement, copyright, trademark, or goodwill. You might find a company that has more than one type of asset. Once you identify a company with a specific asset, describe the accounting this company uses to allocate the cost of that asset to the periods that benefit from its use.

HITTING THE ROAD P3 P4

Design elements: Lightbulb: ©Chuhail/Getty Images; Blue globe: ©nidwlw/Getty Images and ©Dizzle52/Getty Images; Chess piece: ©Andrei Simonenko/Getty Images and ©Dizzle52/Getty Images; Mouse: ©Siede Preis/Getty Images; Global View globe: ©McGraw-Hill Education and ©Dizzle52/Getty Images; Sustainability: ©McGraw-Hill Education and ©Dizzle52/Getty Images

Learning Objectives

CONCEPTUAL C1 Describe current and long-term liabilities

and their characteristics.

C2 Identify and describe known current liabilities.

C3 Explain how to account for contingent liabilities.

P3 Compute and record employer payroll expenses and liabilities.

P4 Account for estimated liabilities, including warranties and bonuses.

P5 Appendix 9A—Identify and describe the details of payroll reports, records, and procedures.

ANALYTICAL A1 Compute the times interest earned ratio

and use it to analyze liabilities.

PROCEDURAL P1 Prepare entries to account for short-term

notes payable.

P2 Compute and record employee payroll deductions and liabilities.

Chapter Preview

9 Accounting for Current Liabilities

PAYROLL LIABILITIES

P2 Employee payroll and deductions

P3 Employer payroll taxes

Multi-period liabilities

ESTIMATED LIABILITIES

P4 Reporting for: Health and pension

Vacation benefits

Bonus plans

Warranty liabilities

KNOWN LIABILITIES

C1 Reporting liabilities C2 Sales taxes payable

Unearned revenues

P1 Short-term notes

NTK 9-1 NTK 9-2 NTK 9-3

CONTINGENCIES AND ANALYSIS

C3 Accounting for contingencies:

Probable

Possible

Remote

A1 Times interest earned

NTK 9-4

341

“Good stuff doesn’t come easy”—Tim Westergren

Sounds Like a Winner!

OAKLAND, CA—“I was a senior in college,” recalls Tim Westergren, “when unbeknownst to me, I decided to become an entrepreneur.” Tim was playing in a band and considering ways to discover new music.

“I shared the idea with a former college classmate, Jon Kraft . . . and in a matter of weeks it went from ‘we have an idea’ to ‘we have a business plan and we’re pitching it.’” The business Tim and Jon built is now known as Pandora Media (Pandora.com), an Internet radio that plays music based on the listener’s preferences.

Tim and Jon started Pandora with financing help. However, within a year of starting the business, Tim and Jon ran out of money.

“We weren’t paying our employees,” admits Tim. “About 50 or 55 people worked without getting paid for over two years during that time.” To keep the business afloat, Tim and Jon learned about managing current liabilities for payroll, supplies, employee benefits, vacations, training, and taxes.

Tim and Jon insist that effective management of liabilities, especially payroll and employee benefits, is crucial for new businesses. Tim and Jon’s ability to juggle their current liabilities enabled them to “hang on” for those crucial first two years.

“Business is an execution game,” insists Tim, “not an inven- tion game.”

Tim encourages people to start a business doing some- thing they love. “If you’re doing it because you love it and because it has meaning for you,” proclaims Tim, “then you can’t really fail.”

Sources: Pandora website, January 2019; Billboard.com, March 2016; Fortune, June 2015; Washington Post, February 2015; GreenBiz, November 2012

©Jason Davis/Pandora Media/Getty Images

Characteristics of Liabilities This section discusses characteristics of liabilities and how liabilities are classified.

Defining Liabilities A liability is a probable future payment of assets or services that a company is presently obligated to make as a result of past transactions or events. This definition includes three elements that are shown in Exhibit 9.1. No liability is reported when one or more of those elements are missing. For example, companies expect to pay wages in future years, but these future payments are not liabilities because no past event such as employee work resulted in a present obligation. Instead, liabilities are recorded when employees perform work and earn wages.

C1 Describe current and long-term liabilities and their characteristics.

KNOWN LIABILITIES

EXHIBIT 9.1 Characteristics of a Liability

Due to a past

transaction or event . . .

Past Present Future

company has a present obligation

. . . for future

payment of assets or services.

Supplies

Date:...............

Request purchase of the follo wing item(s):

Model No. Descr iption

Total

Quantity Price Amount

Payable

Point: Most liability accounts use payable or unearned in their titles.

Classifying Liabilities Liabilities are classified as either current or long term.

Current Liabilities Current liabilities, or short-term liabilities, are liabilities due within one year (or the company’s operating cycle if longer). Most are paid using current assets or by creating other current liabilities. Common examples are accounts payable, short-term notes

Point: For simplicity we assume an operating cycle of one year.

342 Chapter 9 Accounting for Current Liabilities

payable, wages payable, warranty liabilities, and taxes payable. Some liabilities do not have a fixed due date but instead are payable on the creditor’s demand. These are reported as current liabilities because of the possibility of payment in the near term.

Current liabilities differ across companies because they depend on the type of company oper- ations. For example, MGM Resorts reports casino outstanding chip liability. Harley-Davidson reports different current liabilities such as warranty, recall, and dealer incentive liabilities. Exhibit 9.2 shows current liabilities as a percentage of total liabilities for selected companies.

0% 20% 60%40% 80%

Six Flags 14%

Bowl America 58%

Apple 41%

100%

$3.3 mil.

$79,006 mil.

$316 mil.

Columbia Sportswear 84%$363 mil. EXHIBIT 9.2 Current Liabilities as a Percentage of Total Liabilities

Long-Term Liabilities Long-term liabilities are obligations due after one year (or the company’s operating cycle if longer). They include long-term notes payable, warranty liabili- ties, lease liabilities, and bonds payable. For example, Domino’s Pizza reports long-term liabil- ities of $2,196 million. A single liability can be divided between the current and noncurrent sections if a company expects to make payments toward it in both the short and long term. Domino’s reports long-term debt of $2,149 million and current portion of long-term debt of $39 million. The current portion is reported in current liabilities.

Uncertainty in Liabilities Accounting for liabilities involves answering three impor- tant questions: Whom to pay? When to pay? How much to pay? Answers are usually decided when a liability is incurred. For example, if a company has a $100 account payable to a firm, payable on March 15, the answers are clear. However, answers to one or more of these three questions are uncertain for some liabilities.

Uncertainty in Whom to Pay Liabilities can involve uncertainty in whom to pay. For ex- ample, a company can create a liability with a known amount when issuing a note that is pay- able to its holder. In this case, a specific amount is payable to the note’s holder at a specified date, but the company does not know who the holder is until that date. Despite this uncertainty, the company reports this liability on its balance sheet.

Uncertainty in When to Pay A company can have an obligation of a specific amount to a known creditor but not know when it must be paid. For example, a law firm can accept fees in advance from a client who plans to use the firm’s services in the future. This means that the firm has a liability that it settles by providing services at an unknown future date. Although this un- certainty exists, the law firm’s balance sheet must report this liability. These types of obligations are reported as current liabilities because they are likely to be settled in the short term.

Uncertainty in How Much to Pay A company can be aware of an obligation but not know how much it will be required to pay. For example, a company using electrical power is billed only after the meter has been read. This cost is incurred and the liability created before a bill is received. A liability to the power company is reported as an estimated amount if the balance sheet is prepared before a bill arrives.

Examples of Known Liabilities Known liabilities are measurable obligations arising from agreements, contracts, or laws. Known liabilities include accounts payable, notes payable, payroll obligations, sales taxes, and unearned revenues.

S M T W T F S

JANUARY 1 2 3 4 5 6 7 8 9 10 11 12 13 14 15 16 17 18 19 20 21 22 23 24 25 26 27 28 29 30 31

C2 Identify and describe known current liabilities.

Chapter 9 Accounting for Current Liabilities 343

Accounts Payable Accounts payable, or trade accounts payable, are amounts owed to suppliers for products or services purchased on credit. Accounts payable are a focus of the merchandising chapter.

Sales Taxes Payable Nearly all states and many cities levy taxes on retail sales. Sales taxes are shown as a percent of selling prices. The seller collects sales taxes from customers when sales occur and sends these collections to the government. Because sellers currently owe these collections to the govern- ment, this amount is a current liability. If Home Depot sells materials on August 31 for $6,000 cash that are subject to a 5% sales tax, the revenue portion of this transaction is recorded as fol- lows. Later, when Home Depot sends the $300 collected to the government, it debits Sales Taxes Payable and credits Cash.

Assets = Liabilities + Equity +6,300 +300 +6,000

Aug. 31 Cash. . . . . . . . . . . . . . . . . . . . . . . . . . . . . . . . . . . . . . . . . . . . . . . . . . . . 6,300

Sales. . . . . . . . . . . . . . . . . . . . . . . . . . . . . . . . . . . . . . . . . . . . . . . 6,000

Sales Taxes Payable ($6,000 × 0.05) . . . . . . . . . . . . . . . . . . . . 300 Record cash sales and 5% sales tax.*

*We also Dr. Cost of Sales and Cr. Inventory for cost of sales.

June 30 Cash. . . . . . . . . . . . . . . . . . . . . . . . . . . . . . . . . . . . . . . . . . . . . . . . . . . . 5,000,000

Unearned Ticket Revenue. . . . . . . . . . . . . . . . . . . . . . . . . . . . . . 5,000,000

Record sale of tickets for eight concerts.

Assets = Liabilities + Equity +5,000,000 +5,000,000

Oct. 31 Unearned Ticket Revenue . . . . . . . . . . . . . . . . . . . . . . . . . . . . . . . . . . 625,000

Ticket Revenue . . . . . . . . . . . . . . . . . . . . . . . . . . . . . . . . . . . . . . 625,000

Record concert revenues earned ($5,000,000 × 1∕8).

Assets = Liabilities + Equity −625,000 +625,000

Unearned Revenues Unearned revenues, or deferred revenues, are amounts received in advance from customers for future products or services. Unearned revenues arise with airline ticket sales, magazine sub- scriptions, construction projects, hotel reservations, gift card sales, and custom orders. Advance ticket sales for sporting events or concerts are other examples. If Selena Gomez sells $5 million in tickets for eight concerts, the entry is ©Dwphotos/Shutterstock

Point: To defer a revenue means to postpone recording a revenue collected in advance.

Unearned Ticket Revenue is reported as a current liability. As each concert is played, 1/8 of the liability is satisfied and 1/8 of the revenue is earned—this entry follows.

Short-Term Notes Payable A short-term note payable is a written promise to pay a specified amount on a stated future date within one year. Notes can be sold or transferred. Most notes payable bear interest. The written documentation with notes is helpful in resolving legal disputes. We describe two trans- actions that create notes payable.

Note Given to Extend Credit Period A company can replace an account payable with a note payable. A common example is a creditor that requires an interest-bearing note for an overdue account payable. Assume that on August 23, Brady asks to extend its past-due $600 account payable to McGraw. After negotiations, McGraw agrees to accept $100 cash and a 60-day, 12%, $500 note payable to replace the account payable. Brady records the following.

P1 Prepare entries to account for short-term notes payable.

Point: Note requirements: (1) un- conditional promise, (2) in writing, (3) specific amount, and (4) stated due date.

Assets = Liabilities + Equity −100 −600 +500

Aug. 23 Accounts Payable—McGraw. . . . . . . . . . . . . . . . . . . . . . . . . . . . . . . . . 600

Cash . . . . . . . . . . . . . . . . . . . . . . . . . . . . . . . . . . . . . . . . . . . . . . . 100

Notes Payable—McGraw. . . . . . . . . . . . . . . . . . . . . . . . . . . . . . . 500

Sent cash and a note for payment on account.

344 Chapter 9 Accounting for Current Liabilities

Signing the note changes Brady’s debt from an account payable to a note payable. McGraw prefers the note payable over the account payable because it earns interest and it is written docu- mentation of the debt’s existence, term, and amount. When the note comes due, Brady pays the note and interest to McGraw and records this entry.

Interest expense is computed by multiplying the principal of the note ($500) by the annual interest rate (12%) for the fraction of the year the note is outstanding (60 days∕360 days).

Note Given to Borrow from Bank A bank requires a borrower to sign a note when making a loan. When the note comes due, the borrower repays the note with an amount larger than the amount borrowed. The difference between the amount borrowed and the amount repaid is in- terest. The amount borrowed is called principal or face value of the note. Assume that a company borrows $2,000 from a bank at 12% annual interest. The loan is made on September 30, 2019, and is due in 60 days. The note says: “I promise to pay $2,000 plus interest at 12% within 60 days after September 30.” The borrower records its receipt of cash and the new liability with this entry.

Point: Firms commonly compute interest using a 360-day year, called the banker’s rule.

Point: A loan is reported as an asset (receivable) on a bank’s balance sheet.

Point: Excel for accrued interest.

A B

1 Principal $500

2 Rate 12%

3 Issue date 8/23

4 Days 60

5 Accrued interest

=ACCRINTM(B3,B3+B4,B2,B1,2)=$10

Assets = Liabilities + Equity −510 −500 −10

Oct. 22 Notes Payable—McGraw . . . . . . . . . . . . . . . . . . . . . . . . . . . . . 500

Interest Expense . . . . . . . . . . . . . . . . . . . . . . . . . . . . . . . . . . . . 10

Cash . . . . . . . . . . . . . . . . . . . . . . . . . . . . . . . . . . . . . . . . . . 510

Paid note with interest ($500 × 12% × 60∕360).

When principal and interest are paid, the borrower records payment with this entry.

When Note Extends over Two Periods When a note is issued in one period but paid in the next, interest expense is recorded in each period based on the number of days the note extends over each period. Assume a company borrows $2,000 cash on December 16, 2019, at 12% annual interest. This 60-day note matures on February 14, 2020, and the company’s fis- cal year ends on December 31. This means 15 of the 60 days are in 2019 and 45 of the 60 days are in 2020. Interest for these two periods is: 12/16/2019 to 12/31/2019 = 15 days. Interest expense = $2,000 × 12% × 15/360 = $10. 01/01/2020 to 02/14/2020 = 45 days. Interest expense = $2,000 × 12% × 45/360 = $30.

The borrower records the 2019 expense with the following adjusting entry.

Point: Excel for accrued interest.

A B

1 Principal $2,000

2 Rate 12%

3 Issue date 9/30

4 Days 60

5 Accrued interest

=ACCRINTM(B3,B3+B4,B2,B1,2)=$40

Sep. 30 Cash . . . . . . . . . . . . . . . . . . . . . . . . . . . . . . . . . . . . . . . . . . . . . . 2,000

Notes Payable . . . . . . . . . . . . . . . . . . . . . . . . . . . . . . . . . 2,000

Borrowed $2,000 cash with a 60-day, 12%, $2,000 note.

Assets = Liabilities + Equity +2,000 +2,000

Dec. 31, 2019 Interest Expense . . . . . . . . . . . . . . . . . . . . . . . . . . . . . . . . . . . . 10

Interest Payable . . . . . . . . . . . . . . . . . . . . . . . . . . . . . . . . 10

Record accrued interest ($2,000 × 12% × 15∕360).

Assets = Liabilities + Equity +10 −10

Nov. 29 Notes Payable . . . . . . . . . . . . . . . . . . . . . . . . . . . . . . . . . . . . . . 2,000

Interest Expense . . . . . . . . . . . . . . . . . . . . . . . . . . . . . . . . . . . . 40

Cash . . . . . . . . . . . . . . . . . . . . . . . . . . . . . . . . . . . . . . . . . . 2,040

Paid note with interest ($2,000 × 12% × 60∕360). Assets = Liabilities + Equity −2,040 −2,000 −40

When this note is paid on February 14, the borrower records 45 days of interest expense in 2020 and removes the balances of the two liability accounts.

Feb. 14, 2020 Interest Expense* . . . . . . . . . . . . . . . . . . . . . . . . . . . . . . . . . . . 30

Interest Payable . . . . . . . . . . . . . . . . . . . . . . . . . . . . . . . . . . . . . 10

Notes Payable . . . . . . . . . . . . . . . . . . . . . . . . . . . . . . . . . . . . . . 2,000

Cash . . . . . . . . . . . . . . . . . . . . . . . . . . . . . . . . . . . . . . . . . . 2,040

Paid note with interest. *$2,000 × 12% × 45∕360

Assets = Liabilities + Equity −2,040 −10 −30 −2,000

Chapter 9 Accounting for Current Liabilities 345

Debt to Pay Franchisors such as Pizza Hut and Papa John’s use notes to help entrepreneurs acquire their own franchises, including notes to pay for the franchise fee and equipment. Payments on these notes are usually collected monthly and often are secured by the franchisees’ assets. For example, a McDonald’s franchise can cost from under $200,000 to over $2 million, depending on the type selected. ■

Decision Insight

Part 1. A retailer sells merchandise for $500 cash on June 30 (cost of merchandise is $300). The retailer collects 7% sales tax. Record the entry for the $500 sale and its applicable sales tax. Also record the entry that shows the taxes collected being sent to the government on July 15.

Part 2. A ticket agency receives $40,000 cash in advance ticket sales for Haim’s upcoming four-date tour. Record the advance ticket sales on April 30. Record the revenue earned for the first concert date of May 15, assuming it represents one-fourth of the advance ticket sales.

Part 3. On November 25 of the current year, a company borrows $8,000 cash by signing a 90-day, 5% note payable with a face value of $8,000. (a) Compute the accrued interest payable on December 31 of the current year, (b) prepare the journal entry to record the accrued interest expense at December 31 of the current year, and (c) prepare the journal entry to record payment of the note at maturity.

Solution—Part 1

Accounting for Known Liabilities

NEED-TO-KNOW 9-1

C2 P1

Solution—Part 2

June 30 Cash . . . . . . . . . . . . . . . . . . . . . . . . . . . . . . . . . . . . . . . . . . . . . . 535 Sales . . . . . . . . . . . . . . . . . . . . . . . . . . . . . . . . . . . . . . . . . 500 Sales Taxes Payable . . . . . . . . . . . . . . . . . . . . . . . . . . . . . 35 Record cash sales and 7% sales tax. June 30 Cost of Goods Sold . . . . . . . . . . . . . . . . . . . . . . . . . . . . . . . . . . 300 Merchandise Inventory . . . . . . . . . . . . . . . . . . . . . . . . . . . 300 Record cost of June 30 sales. July 15 Sales Taxes Payable . . . . . . . . . . . . . . . . . . . . . . . . . . . . . . . . . . 35 Cash . . . . . . . . . . . . . . . . . . . . . . . . . . . . . . . . . . . . . . . . . . 35 Record sales taxes sent to govt.

Apr. 30 Cash . . . . . . . . . . . . . . . . . . . . . . . . . . . . . . . . . . . . . . . . . . . . . . 40,000 Unearned Ticket Revenue . . . . . . . . . . . . . . . . . . . . . . . . 40,000 Record sales in advance of concerts. May 15 Unearned Ticket Revenue . . . . . . . . . . . . . . . . . . . . . . . . . . . . . 10,000 Earned Ticket Revenue . . . . . . . . . . . . . . . . . . . . . . . . . . 10,000 Record concert revenues earned ($40,000 × 1∕4).

Point: Maturity date is the day a note’s principal and interest are due. Maturity value is a note’s principal plus interest owed on its maturity date.

Dec. 31 Interest Expense . . . . . . . . . . . . . . . . . . . . . . . . . . . . . . . . . . . . 40 Interest Payable . . . . . . . . . . . . . . . . . . . . . . . . . . . . . . . . 40 Record accrued interest (5% × $8,000 × 36∕360).

b.

Solution—Part 3

a. Computation of interest payable at December 31: Days from November 25 to December 31 . . . . . . . . . . . . . . . . . . . . . . 36 days Accrued interest (5% × $8,000 × 36∕360) . . . . . . . . . . . . . . . . . . . . . $40

Feb. 23 Interest Expense . . . . . . . . . . . . . . . . . . . . . . . . . . . . . . . . . . . . 60 Interest Payable . . . . . . . . . . . . . . . . . . . . . . . . . . . . . . . . . . . . . 40 Notes Payable . . . . . . . . . . . . . . . . . . . . . . . . . . . . . . . . . . . . . . 8,000 Cash . . . . . . . . . . . . . . . . . . . . . . . . . . . . . . . . . . . . . . . . . . 8,100 Record payment of note plus interest (5% × $8,000 × 90∕360 = $100 total interest) (5% × $8,000 × 54∕360 = $60 interest expense).

c.

Do More: QS 9-2, QS 9-3, QS 9-4, E 9-2, E 9-3, E 9-4

Point: Feb. 23 entry assumes no reversing entry was made.

Point: Accrued interest, 11/25–12/31.

A B

1 Principal $8,000

2 Rate 5%

3 Issue date 11/25

4 Days 36

5 Accrued interest

=ACCRINTM(B3,B3+B4,B2,B1,2)=$40

Point: Accrued Interest, 1/1–2/23.

A B

1 Principal $8,000

2 Rate 5%

3 Issue date 11/25

4 Days 54

5 Accrued interest

=ACCRINTM(B3,B3+B4,B2,B1,2)=$60

346 Chapter 9 Accounting for Current Liabilities

Payroll liabilities are from salaries and wages, employee benefits, and payroll taxes levied on the employer. For example, Boston Beer reports current payroll liabilities of more than $14 mil- lion from accrued “employee wages, benefits and reimbursements.”

EMPLOYEE Payroll and Deductions Gross pay is the total compensation an employee earns including wages, salaries, commissions, bonuses, and any compensation earned before deductions such as taxes. (Wages usually refer to payments to employees at an hourly rate. Salaries usually refer to payments to employees at a monthly or yearly rate.) Net pay, or take-home pay, is gross pay minus all deductions. Payroll deductions, or withholdings, are amounts withheld from an employee’s gross pay, either required or voluntary. Required deductions result from laws and include income taxes and Social Security taxes. Voluntary deductions, at an employee’s option, include pension and health contributions, health and life insurance premiums, union dues, and donations.

Exhibit 9.3 shows typical employee payroll deductions. The employer withholds payroll deductions from employees’ pay and sends this money to the designated group or government. The employer records payroll deductions as current liabilities until these amounts are sent. This section covers major payroll deductions.

P2 Compute and record employee payroll deductions and liabilities.

Point: Deductions at some compa- nies, such as those for insurance coverage, are “required” under labor contracts.

PAYROLL LIABILITIES

FICA Taxes (Social Security)

Gross Pay

Voluntary Deductions

FICA Taxes (Medicare)

State and Local Income Taxes

minus deductions

Net pay = Gross pay – Deductions

Net PayNet PayNet Pay

Federal Income Tax

EXHIBIT 9.3 Payroll Deductions

Employee FICA Taxes Employers withhold Federal Insurance Contributions Act (FICA) taxes from employees’ pay. Employers separate FICA taxes into two groups.

1. Social Security taxes—withholdings to cover retirement, disability, and survivorship. 2. Medicare taxes—withholdings to cover medical benefits.

Taxes for Social Security and Medicare are computed separately. For 2018, the amount with- held from each employee’s pay for Social Security tax is 6.2% of the first $128,400 the employee earns in the calendar year. The Medicare tax is 1.45% of all amounts the employee earns; there is no maximum limit to Medicare tax. A 0.9% Additional Medicare Tax is imposed on the high- income employee for pay usually in excess of $200,000 (this additional tax is not imposed on the employer, whereas the others are). Until the taxes are sent to the Internal Revenue Service (IRS), they are included in employers’ current liabilities. For any changes in rates or earnings levels, check IRS.gov or SSA.gov.

Employee Income Tax Most employers withhold federal income tax from each em- ployee’s paycheck. The amount withheld is computed using IRS tables. The amount depends on the employee’s income and the number of withholding allowances the employee claims. Allowances reduce taxes owed to the government. Employees can claim allowances for

Point: Sources of U.S. tax receipts: 50% Personal income tax 35% FICA and FUTA taxes 10% Corporate income tax 5% Other taxes

Chapter 9 Accounting for Current Liabilities 347

themselves and their dependents. Until the government is paid, withholdings are reported as a current liability on the employer’s balance sheet.

Employee Voluntary Deductions Voluntary deduction withholdings come from employee requests, contracts, unions, or other agreements. They include charitable giving, med- ical and life insurance premiums, pension contributions, and union dues. Until they are paid, voluntary withholdings are reported as part of employers’ current liabilities.

Employee Payroll Recording Employers accrue payroll expenses and liabilities at the end of each pay period. Assume that an employee earns a salary of $2,000 per month. At the end of January, the employer’s entry to accrue payroll expenses and liabilities for this employee is

Salaries Expense (debit) shows that the employee earns a gross salary of $2,000. The first five payables (credits) show the liabilities the employer owes on behalf of this employee to cover FICA taxes, income taxes, medical insurance, and union dues. The Salaries Payable account (credit) records the $1,524 net pay the employee receives from the $2,000 gross pay earned. The February 1 entry to record cash payment to this employee is

Jan. 31 Salaries Expense . . . . . . . . . . . . . . . . . . . . . . . . . . . . . . . . . . . . 2,000

FICA—Social Security Taxes Payable (6.2%) . . . . . . . . . . 124

FICA—Medicare Taxes Payable (1.45%) . . . . . . . . . . . . . 29

Employee Federal Income Taxes Payable* . . . . . . . . . . . 213

Employee Medical Insurance Payable* . . . . . . . . . . . . . . 85

Employee Union Dues Payable* . . . . . . . . . . . . . . . . . . . 25

Salaries Payable . . . . . . . . . . . . . . . . . . . . . . . . . . . . . . . . 1,524

Record accrued payroll for January. *Amounts taken from employer’s accounting records.

Assets = Liabilities + Equity +124 −2,000 +29 +213 +85 +25 +1,524

Feb. 1 Salaries Payable . . . . . . . . . . . . . . . . . . . . . . . . . . . . . . . . . . . . . 1,524

Cash . . . . . . . . . . . . . . . . . . . . . . . . . . . . . . . . . . . . . . . . . 1,524

Record payment of payroll.

EMPLOYER Payroll Taxes Employers must pay payroll taxes in addition to those required of employees. Employer taxes include FICA and unemployment taxes.

Employer FICA Tax Employers must pay FICA taxes on their payroll. For 2018, the employer must pay Social Security tax of 6.2% on the first $128,400 earned by each employee and 1.45% Medicare tax on all earnings of each employee. An employer’s tax is credited to the same FICA Taxes Payable accounts used to record the Social Security and Medicare taxes with- held from employees.

Employer Unemployment Taxes The federal government works with states in a joint federal and state unemployment insurance program. Each state has its own program. These programs provide unemployment benefits to qualified workers.

Federal Unemployment Tax Act (FUTA) Employers must pay a federal unemployment tax on wages and salaries earned by their employees. For the recent year, employers were required to pay FUTA taxes of as much as 6.0% of the first $7,000 earned by each employee. This federal tax can be reduced by a credit of up to 5.4% for taxes paid to a state program. As a result, the net federal unemployment tax is often 0.6%.

State Unemployment Tax Act (SUTA) All states fund their unemployment insurance pro- grams by placing a payroll tax on employers. (A few states require employees to make a contri- bution. In the book’s assignments, we assume this tax is only levied on the employer.) In most states, the base rate for SUTA taxes is 5.4% of the first $7,000 earned by each employee (the dollar level varies by state). This base rate is adjusted according to an employer’s merit rating.

P3 Compute and record employer payroll expenses and liabilities.

Point: A self-employed person must pay both the employee and employer FICA taxes.

348 Chapter 9 Accounting for Current Liabilities

The state assigns a merit rating based on a company’s stability in employing workers. A good rating reflects stability in employment and means an employer can pay less than the 5.4% base rate. A low rating means high turnover or seasonal hirings and layoffs.

Recording Employer Payroll Taxes Employer payroll taxes are an added expense beyond the wages and salaries earned by employees. These taxes are often recorded in an entry separate from the one recording payroll expenses and deductions. Assume that the $2,000 re- corded salaries expense from the previous example is earned by an employee whose earnings have not yet reached $5,000 for the year. This means the entire salaries expense for this period is subject to tax because year-to-date pay is under $7,000. Consequently, the FICA portion of the employer’s tax is $153, computed by multiplying both the 6.2% and 1.45% by the $2,000 gross pay. Assume that the federal unemployment tax rate is 0.6% and the state unemployment tax rate is 5.4%. This means state unemployment (SUTA) taxes are $108 (5.4% of the $2,000 gross pay) and federal unemployment (FUTA) taxes are $12 (0.6% of $2,000). The entry to record the employer’s payroll tax expense and related liabilities is

Internal Control of Payroll Internal controls are crucial for payroll because of a high risk of fraud and error. Exhibit 9.4 identifies and explains four key areas of payroll activities that we aim to separate and monitor.

Assets = Liabilities + Equity +124 −273 +29 +108 +12

Jan. 31 Payroll Taxes Expense . . . . . . . . . . . . . . . . . . . . . . . . . . . . . . . 273

FICA—Social Security Taxes Payable (6.2%) . . . . . . . . . . 124

FICA—Medicare Taxes Payable (1.45%) . . . . . . . . . . . . . 29

State Unemployment Taxes Payable . . . . . . . . . . . . . . . . 108

Federal Unemployment Taxes Payable . . . . . . . . . . . . . . 12

Record employer payroll taxes.

NOW HIRING

Duty: Authorize, hire, and fire. Aim: Keep fake workers o payroll.

Duty: Verify tax rates and payroll amounts. Aim: Rates updated and amounts accurate.

Duty: Track and verify time worked. Aim: Paid for time worked only.

Duty: Sign and issue prenumbered checks. Aim: Checks valid, secured, and correct.

Employee Hiring Payroll Preparation Timekeeping Payroll PaymentEXHIBIT 9.4 Internal Controls in Four Key Areas of Payroll

Payroll Fraud Probably the greatest number of frauds involve payroll. Controls include proper approvals and pro- cesses for employee additions, deletions, and pay rate changes. A common fraud is a manager adding a fictitious employee to the payroll and then cashing the fictitious employee’s check. A study reports that 42% of employees in operations and service areas witnessed violations of employee wage, overtime, or benefit rules in the past year. Another 33% observed falsifying of time and expense reports (KPMG). ■

Ethical Risk

Ceridian Connection reports: 8.5% of fraud is tied to payroll; $72,000 is the median loss per payroll fraud; and 24 months is the median time to uncover pay- roll fraud.

Multi-Period Known Liabilities Many known liabilities extend over multiple periods. These often include unearned revenues and notes payable. For example, if Sports Illustrated sells a three-year digital magazine subscrip- tion, it records amounts received for this subscription in an Unearned Subscription Revenues account. Amounts in this account are liabilities, but are they current or long term? They are both. The portion of the Unearned Subscription Revenues account that will be fulfilled in the next year is reported as a current liability. The remaining portion is reported as a long-term liability.

Chapter 9 Accounting for Current Liabilities 349

The same analysis applies to notes payable. For example, a borrower reports a three-year note payable as a long-term liability in the first two years it is outstanding. In the third year, the bor- rower reclassifies this note as a current liability because it is due within one year. The current portion of long-term debt is that part of long-term debt due within one year. Long-term debt is reported under long-term liabilities, but the current portion due is reported under current liabil- ities. Assume that a $7,500 debt is paid in installments of $1,500 per year for five years. The $1,500 due within the year is reported as a current liability. No journal entry is necessary for this reclassification. Instead, we simply classify the amounts for debt as either current or long term when the balance sheet is prepared.

Point: Some accounting systems make an entry to transfer the cur- rent amount due out of Long-Term Debt and into the Current Portion of Long-Term Debt as follows:

Long-Term Debt . . . . . 1,500 Current Portion

of L-T Debt . . . . . 1,500

Summer Intern You take a summer job working as a windsurfing instructor. On your first payday, the owner slaps you on the back, gives you full payment in cash, winks, and adds: “No need to pay those high taxes, eh?” What action, if any, do you take? ■ Answer: You do not want to be an accomplice to unlawful payroll activities. Not paying federal and state taxes on wages is illegal and unethical. One action is to request payment by check. If this fails, you must consider quitting.

Decision Ethics

An estimated liability is a known obligation of an uncertain amount that can be reasonably estimated. Common examples are employee benefits such as pensions, health care, and vacation pay, and warranties offered by a seller.

Health and Pension Benefits Many companies provide employee benefits. An employer often pays all or part of medical, dental, life, and disability insurance. Many employers also contribute to pension plans, which are agreements by employers to provide benefits (payments) to employees after retirement. Many companies also provide medical care and insurance benefits to their retirees. Assume

ESTIMATED LIABILITIES P4 Account for estimated liabilities, including warranties and bonuses.

A company’s first weekly pay period of the year ends on January 8. Sales employees earned $30,000 and office employees earned $20,000 in salaries. The employees are to have withheld from their salaries FICA Social Security taxes at the rate of 6.2%, FICA Medicare taxes at the rate of 1.45%, $9,000 of federal in- come taxes, $2,000 of medical insurance deductions, and $1,000 of pension contributions. No employee earned more than $7,000 in the first pay period.

Part 1. Compute FICA Social Security taxes payable and FICA Medicare taxes payable. Prepare the jour- nal entry to record the company’s January 8 (employee) payroll expenses and liabilities.

Part 2. Prepare the journal entry to record the company’s (employer) payroll taxes resulting from the January 8 payroll. Its state unemployment tax rate is 5.4% on the first $7,000 paid to each employee. The federal unemployment tax rate is 0.6%.

Solution—Part 1 Solution—Part 2

Payroll Liabilities

NEED-TO-KNOW 9-2

P2 P3

Jan. 8 Sales Salaries Expense . . . . . . . . . . . . . . . . 30,000

Office Salaries Expense . . . . . . . . . . . . . . . . 20,000

FICA—Social Security Taxes Payable* 3,100

FICA—Medicare Taxes Payable† . . . . 725

Employee Fed. Income Taxes Payable 9,000

Employee Med. Insurance Payable . . 2,000

Employee Pensions Payable . . . . . . . 1,000

Salaries Payable . . . . . . . . . . . . . . . . . 34,175

Record payroll for period. *$50,000 × 6.2% = $3,100 †$50,000 × 1.45% = $725

Jan. 8 Payroll Taxes Expense . . . . . . . . . . . . . . . . . 6,825

FICA—Social Security Taxes Payable . 3,100

FICA—Medicare Taxes Payable . . . . . 725

State Unemployment Taxes Payable* 2,700

Federal Unemployment Taxes Payable† 300

Record employer payroll taxes. *$50,000 × 5.4% = $2,700 †$50,000 × 0.6% = $300

Do More: QS 9-5, QS 9-6, E 9-5, E 9-6, E 9-7, E 9-8, E 9-9

350 Chapter 9 Accounting for Current Liabilities

an employer agrees to (1) pay $8,000 for medical insurance and (2) contribute an additional 10% of the employees’ $120,000 gross salaries to a retirement program. The entry to record these accrued benefits is

Vacation Benefits Many employers offer paid vacation benefits, or paid absences. Vacation benefits are esti- mated and expensed in the period when employees earn them. Assume that salaried employ- ees earn 2 weeks’ paid vacation per year. The year-end adjusting entry to record $3,200 of accrued vacation benefits follows.

Point: An accrued expense is an unpaid expense and is also called an accrued liability.

Dec. 31 Employee Benefits Expense . . . . . . . . . . . . . . . . . . . . . . . . . . . 20,000

Employee Medical Insurance Payable . . . . . . . . . . . . . . 8,000

Employee Retirement Program Payable . . . . . . . . . . . . . 12,000

Record costs of employee benefits.

Assets = Liabilities + Equity +8,000 −20,000 +12,000

Dec. 31 Vacation Benefits Expense . . . . . . . . . . . . . . . . . . . . . . . . . . . . 3,200

Vacation Benefits Payable . . . . . . . . . . . . . . . . . . . . . . . . 3,200

Record vacation benefits accrued.

Assets = Liabilities + Equity +3,200 −3,200

Rest on One’s Laurels Major League Baseball was the first pro sport to set up a pension, originally up to $100 per month depending on years played. Many former players now take home six-figure pensions. Cal Ripken Jr.’s pension at age 62 is estimated at $180,000 per year (he played 21 seasons). The same applies to Ichiro Suzuki, who has played 17 seasons—see photo. The requirement is 43 games for a full pension and just one game for full medical benefits for life. ■

Decision Insight

©Imac/Alamy Stock Photo

Vacation Benefits Expense is an operating expense, and Vacation Benefits Payable is a current liability. When an employee takes a one-week vacation, the employer reduces (debits) Vacation Benefits Payable and credits Cash.

Bonus Plans Many companies offer bonuses to employees, and many of the bonuses depend on net income. Assume that an employer gives a bonus to its employees based on the company’s annual net income (to be equally shared by all). The year-end adjusting entry to record a $10,000 bonus is

Warranty Liabilities A warranty is a seller’s obligation to replace or fix a product (or service) that fails to perform as expected within a specified period. For example, new Ford cars are sold with a warranty covering parts for a specified period of time. The seller reports the expected warranty expense in the period

Jan. 20 Vacation Benefits Payable . . . . . . . . . . . . . . . . . . . . . . . . . . . . . 400

Cash . . . . . . . . . . . . . . . . . . . . . . . . . . . . . . . . . . . . . . . . . . 400

Record vacation benefits taken.

Assets = Liabilities + Equity −400 −400

Assets = Liabilities + Equity +10,000 −10,000

Dec. 31 Employee Bonus Expense . . . . . . . . . . . . . . . . . . . . . . . . . . . . . 10,000

Bonus Payable . . . . . . . . . . . . . . . . . . . . . . . . . . . . . . . . . 10,000

Record expected bonus costs.

Chapter 9 Accounting for Current Liabilities 351

when revenue from the sale of the product or service is reported. The seller reports this warranty liability, even though the existence, amount, payee, and date of future payments are uncertain. This is because warranty costs are probable and the amount can be estimated using past experience.

Assume a dealer sells a car for $16,000 on December 1, 2019, with a one-year or 12,000-mile warranty covering parts. Experience shows that warranty expense is 4% of a car’s selling price, or $640 in this case ($16,000 × 4%). The dealer records the estimated expense and liability related to this sale with this entry.

1 YEAR WARRANTY

President SEAL MOTOR

Box: 25515 River Heights PO Newyork

EX CLUSIV

E

W

A RRANTY

1 YEAR

“ALL PARTS”

This entry alternatively could be made as part of end-of-period adjustments. Either way, the estimated warranty expense is reported on the 2019 income statement and the warranty liability on the 2019 balance sheet. Continuing this example, assume the customer brings the car in for warranty repairs on January 9, 2020. The dealer fixes the car by replacing parts costing $200. The entry to record the repair is

This entry reduces the balance of the Estimated Warranty Liability account, but no expense is recorded in 2020 for the repair. Warranty expense was previously recorded in 2019, the year the car was sold with the warranty. Finally, what happens if total warranty expenses are more or less than the estimated 4%, or $640? The answer is that management should monitor actual warranty expenses to see if a 4% rate is accurate. If not, the rate is changed for future periods.

Multi-Period Estimated Liabilities Estimated liabilities can be both current and long term. For example, pension liabilities to employees are long term to workers who will not retire within the next year. For employees who are retired or will retire within the next year, a portion of pension liabilities is current. Other examples include employee health benefits and warranties.

Dec. 1 Warranty Expense . . . . . . . . . . . . . . . . . . . . . . . . . . . . . . . . . . . 640

Estimated Warranty Liability . . . . . . . . . . . . . . . . . . . . . . 640

Record estimated warranty expense.

Assets = Liabilities + Equity +640 −640

Jan. 9 Estimated Warranty Liability . . . . . . . . . . . . . . . . . . . . . . . . . . . 200

Auto Parts Inventory . . . . . . . . . . . . . . . . . . . . . . . . . . . . 200

Record costs of warranty repairs.

Assets = Liabilities + Equity −200 −200

Promises, Promises When we purchase a new laptop at Best Buy, a sales clerk commonly asks: “Do you want the Geek Squad Protection Plan?” Best Buy earns about a 60% profit mar- gin on such warranty contracts, and those contracts are a large part of its profit—see table (BusinessWeek). ■

Decision Insight

Warranties as a percent of sales . . . . . . . . . . . . 4%

Warranties as a percent of operating profit . . . . 45%

Part 1. A company’s salaried employees earn two weeks’ vacation per year. The company estimated and must expense $9,000 of accrued vacation benefits for the year. (a) Prepare the year-end adjusting entry to record accrued vacation benefits. (b) Prepare the entry on May 1 of the next year when an employee takes a one-week vacation and is paid $450 cash for that week.

Part 2. For the current year ended December 31, a company has implemented an employee bonus pro- gram based on its net income, which employees share equally. Its bonus expense is $40,000. (a) Prepare the journal entry at December 31 of the current year to record the bonus due. (b) Prepare the journal entry at January 20 of the following year to record payment of that bonus to employees.

Part 3. On June 11 of the current year, a retailer sells a trimmer for $400 with a one-year warranty that covers parts. Warranty expense is estimated at 5% of sales. On March 24 of the next year, the trimmer is brought in for repairs covered under the warranty requiring $15 in materials taken from the Repair Parts Inventory. Prepare the (a) June 11 entry to record the trimmer sale—ignore the cost of sales part of this sales entry—and (b) March 24 entry to record warranty repairs.

Estimated Liabilities

NEED-TO-KNOW 9-3

P4

352 Chapter 9 Accounting for Current Liabilities

Solution—Part 1

Solution—Part 2

Solution—Part 3

June 11 Cash . . . . . . . . . . . . . . . . . . . . . . . . . . . . . . . . . . . . . . . . . . . . . . 400

Sales . . . . . . . . . . . . . . . . . . . . . . . . . . . . . . . . . . . . . . . . . 400

Record trimmer sales.

June 11 Warranty Expense . . . . . . . . . . . . . . . . . . . . . . . . . . . . . . . . . . . 20

Estimated Warranty Liability . . . . . . . . . . . . . . . . . . . . . . 20

Record estimated warranty expense ($400 × 5%).

Mar. 24 Estimated Warranty Liability . . . . . . . . . . . . . . . . . . . . . . . . . . . 15

Repair Parts Inventory . . . . . . . . . . . . . . . . . . . . . . . . . . . 15

Record cost of warranty repairs.

b. Jan. 20 Bonus Payable . . . . . . . . . . . . . . . . 40,000 Cash . . . . . . . . . . . . . . . . . . . . 40,000

Record payment of bonus.

b. May 1 Vacation Benefits Payable . . . . . . . 450

Cash . . . . . . . . . . . . . . . . . . . . 450

Record vacation benefits taken.

Dec. 31 Employee Bonus Expense . . . . . . . . 40,000

Bonus Payable . . . . . . . . . . . . . 40,000

Record expected bonus costs.

a.

Dec. 31 Vacation Benefits Expense . . . . . . . . 9,000

Vacation Benefits Payable . . . . 9,000

Record vacation benefits accrued.

a.

Do More: QS 9-7, QS 9-8, QS 9-9, QS 9-10, E 9-10,

E 9-11, E 9-12, E 9-13

A contingent liability is a potential obligation that depends on a future event arising from a past transaction or event. An example is a pending lawsuit. Here, a past transaction or event leads to a lawsuit whose financial outcome depends on the result of the suit.

Accounting for Contingent Liabilities Accounting for contingent liabilities depends on the likelihood that a future event will occur and the ability to estimate the future amount owed if this event occurs. Three different possibilities are shown in Exhibit 9.5: record liability with a journal entry, disclose in notes to financial statements, or no disclosure.

CONTINGENT LIABILITIES C3 Explain how to account for contingent liabilities.

Nonestimable

Estimable

Possible

Remote

Probable

Contingent liability

Future event is Amount owed is Record liability

No disclosure

Disclose in notes

Est. Expense . . . . . # Est. Liability . . . #

EXHIBIT 9.5 Accounting for Contingent Liabilities

The conditions that determine each of these three possibilities follow.

1. Record liability. The future event is probable (likely) and the amount owed can be reasonably estimated. Examples are warranties, vacation pay, and income taxes.

2. Disclose in notes. The future event is reasonably possible (could occur). 3. No disclosure. The future event is remote (unlikely).

Point: A contingency is an if. Namely, if a future event occurs, then financial consequences are likely for the entity.

Chapter 9 Accounting for Current Liabilities 353

Applying Rules of Contingent Liabilities This section covers common contingent liabilities.

Potential Legal Claims Many companies are sued or at risk of being sued. The ac- counting issue is whether the defendant records a liability or discloses a contingent liability in its notes while a lawsuit is outstanding and not yet settled. The answer is that a potential claim is recorded only if payment for damages is probable and the amount can be reasonably esti- mated. If the potential claim cannot be reasonably estimated but is reasonably possible, it is disclosed. For example, Ford includes the following note in its annual report: “Various legal actions, proceedings, and claims are pending . . . arising out of alleged defects in our products.”

Debt Guarantees Sometimes a company guarantees the payment of debt owed by a sup- plier, customer, or another company. The guarantor usually discloses the guarantee in its finan- cial statement notes as a contingent liability. If it is probable that the debtor will default, the guarantor reports the guarantee as a liability. The Boston Celtics report a unique guarantee: “Contracts provide for guaranteed payments which must be paid even if the employee [player] is injured or terminated.”

Other Contingencies Other examples of contingencies include environmental dam- ages, possible tax assessments, insurance losses, and government investigations. Chevron, for example, reports that it “is subject to loss contingencies . . . related to environmental matters. . . . The amount of additional future costs are not fully determinable.” Many of Chevron’s contingen- cies are revealed only in notes.

Uncertainties That Are Not Contingencies All organizations face uncertainties from future events such as natural disasters and new tech- nologies. These uncertainties are not contingent liabilities because they are future events not arising from past transactions. Accordingly, they are not disclosed.

The following legal claims exist for a company. Identify the accounting treatment for each claim as either (a) a liability that is recorded or (b) an item described in notes to its financial statements. 1. The company (defendant) estimates that a pending lawsuit could result in damages of $500,000; it is

reasonably possible that the plaintiff will win the case. 2. The company faces a probable loss on a pending lawsuit; the amount is not reasonably estimable. 3. The company estimates environmental damages in a pending case at $900,000 with a high probability

of losing the case.

Solution

1. (b); reason—is reasonably estimated but not a probable loss. 2. (b); reason—probable loss but cannot be reasonably estimated. 3. (a); reason—can be reasonably estimated and loss is probable.

Contingent Liabilities

NEED-TO-KNOW 9-4

C3

Do More: QS 9-11, E 9-14

Times Interest Earned Ratio Decision Analysis

Interest expense is often called a fixed expense because it usually does not vary due to short-term changes in sales or other operating activities. While fixed expenses can be good when a company is growing, they create risk. The risk is that a company might be unable to pay fixed expenses if sales decline. Consider Diego Co.’s results for 2019 and two possible outcomes for year 2020 in Exhibit 9.6. Expenses excluding interest are expected to remain at 75% of sales. Expenses that change with sales volume are variable expenses. Interest expense is fixed at $60 per year.

A1 Compute the times interest earned ratio and use it to analyze liabilities.

The following transactions took place at Kern Co. during its recent calendar-year reporting period. a. In September, Kern sold $140,000 of merchandise covered by a 180-day warranty. Prior experience

shows that costs of the warranty equal 5% of sales. Compute September’s warranty expense and pre- pare the adjusting journal entry for the warranty liability as recorded at September 30. Also prepare the journal entry on October 8 to record a $300 cash payment to provide warranty service on an item sold in September.

b. On October 12, Kern replaced an overdue $10,000 account payable by paying $2,500 cash and signing a note for $7,500. The note matures in 90 days and has a 12% interest rate. Prepare the entries recorded on October 12, December 31, and January 10.

c. In late December, Kern is facing a product liability suit filed by an unhappy customer. Kern’s lawyer says it will probably suffer a loss from the lawsuit, but the amount is impossible to estimate.

d. Sally Bline works for Kern. For the pay period ended November 30, her gross earnings are $3,000. Bline has $800 deducted for federal income taxes and $200 for state income taxes from each paycheck. Additionally, a $35 premium for health insurance and a $10 donation to United Way are deducted. Bline pays FICA Social Security taxes at a rate of 6.2% and FICA Medicare taxes at a rate of 1.45%. She has not earned enough this year to be exempt from any FICA taxes. Journalize the accrual of sala- ries expense for Bline by Kern.

e. On November 1, Kern borrows $5,000 cash from a bank in return for a 60-day, 12%, $5,000 note. Record the note’s issuance on November 1 and its repayment with interest on December 31.

f.B (Part f covers Appendix 9B.) Kern has estimated and recorded its quarterly income tax payments. In reviewing its year-end tax adjustments, it identifies an additional $5,000 of income taxes expense that should be recorded. A portion of this additional expense, $1,000, is deferred to future years. Record this year-end income taxes expense adjusting entry.

g. For this calendar year, Kern’s net income is $1,000,000, its interest expense is $275,000, and its income taxes expense is $225,000. Compute Kern’s times interest earned ratio.

COMPREHENSIVE

Accounting for Current Liabilities Including Warranties, Notes, Contingencies, Payroll, and Income Taxes

NEED-TO-KNOW 9-5

354 Chapter 9 Accounting for Current Liabilities

The Sales Increase column of Exhibit 9.6 shows that Diego’s net income increases by 83% to $165 if sales increase by 50% to $900. The Sales Decrease column shows that net income decreases by 83% if sales decline by 50%. These results show that the amount of fixed interest expense affects a company’s risk of its ability to pay interest. One measure of “ability to pay” is the times interest earned ratio in Exhibit 9.7.

EXHIBIT 9.6 Actual and Projected Results

2020 Projections

$ millions 2019 Sales Increase Sales Decrease

Sales . . . . . . . . . . . . . . . . . . . . . . . . . . $600 $900 $300

Expenses (75% of sales) . . . . . . . . . . . 450 675 225

Income before interest . . . . . . . . . . . . 150 225 75

Interest expense (fixed) . . . . . . . . . . . 60 60 60

Net income. . . . . . . . . . . . . . . . . . . . . . $ 90 $165 $ 15

EXHIBIT 9.7 Times Interest Earned Times interest earned =

Income before interest expense and income taxes Interest expense

For 2019, Diego’s times interest earned is computed as $150/$60, or 2.5 times. This ratio means that Diego has low to moderate risk because its sales must decline sharply before it is unable to pay its interest expenses. If times interest earned falls below around 1.5, a company will likely be at risk of not being able to pay its liabilities.

$0 Sales Decrease Sales Flat Sales Increase

$100 1.0

0.0

5.0

4.0

7.0

6.0

Times Interest Earned

$200

$300

$400

$500

$600

$700

$800

$900

$ mil.

2.0

3.0

8.0

Net Income Times Interest EarnedSales

Entrepreneur You wish to invest in a franchise for either one of two national chains. Each franchise has an expected annual net income after interest and taxes of $100,000. Net income for the first franchise includes a regular fixed interest charge of $200,000. The fixed interest charge for the second franchise is $40,000. Which franchise is riskier to you if sales forecasts are not met? ■ Answer: Times interest earned for the first franchise is 1.5 [($100,000 + $200,000)/$200,000], whereas it is 3.5 for the second [($100,000 + $40,000)/$40,000]. This shows the first franchise is more at risk of incurring a loss if its sales decline.

Decision Maker

Chapter 9 Accounting for Current Liabilities 355

PLANNING THE SOLUTION For a, compute the warranty expense for September and record it with an estimated liability. Record

the October payment as a decrease in the liability. For b, eliminate the liability for the account payable and create the liability for the note payable.

Compute interest expense for the 80 days that the note is outstanding in the current year and record it as a liability. Record the payment of the note, being sure to include the interest for the 10 days in January.

For c, decide whether the company’s contingent liability needs to be disclosed or accrued (recorded) according to the two necessary criteria: probable loss and reasonably estimable.

For d, set up payable accounts for all items in Bline’s paycheck that require deductions. After all deduc- tions, credit the remaining amount to Salaries Payable.

For e, record the issuance of the note. Compute 60 days’ interest due. For f, determine how much of the income taxes expense is payable in the current year and how much

needs to be deferred (see Appendix 9B). For g, apply and compute times interest earned.

SOLUTION a. Warranty expense = 5% × $140,000 = $7,000

Sep. 30 Warranty Expense . . . . . . . . . . . . . . . . . . . . . . . . . . . . . . . . . . . 7,000

Estimated Warranty Liability . . . . . . . . . . . . . . . . . . . . . . . 7,000

Record warranty expense for month.

Oct. 8 Estimated Warranty Liability . . . . . . . . . . . . . . . . . . . . . . . . . . . 300

Cash . . . . . . . . . . . . . . . . . . . . . . . . . . . . . . . . . . . . . . . . . . 300

Record cost of warranty service.

Oct. 12 Accounts Payable . . . . . . . . . . . . . . . . . . . . . . . . . . . . . . . . . . . 10,000

Notes Payable . . . . . . . . . . . . . . . . . . . . . . . . . . . . . . . . . . 7,500

Cash . . . . . . . . . . . . . . . . . . . . . . . . . . . . . . . . . . . . . . . . . . 2,500

Paid $2,500 cash and gave a 90-day, 12% note to extend due date on the account.

Dec. 31 Interest Expense . . . . . . . . . . . . . . . . . . . . . . . . . . . . . . . . . . . . 200

Interest Payable . . . . . . . . . . . . . . . . . . . . . . . . . . . . . . . . 200

Accrue interest on note payable.

Jan. 10 Interest Expense . . . . . . . . . . . . . . . . . . . . . . . . . . . . . . . . . . . . 25

Interest Payable . . . . . . . . . . . . . . . . . . . . . . . . . . . . . . . . . . . . . 200

Notes Payable . . . . . . . . . . . . . . . . . . . . . . . . . . . . . . . . . . . . . . 7,500

Cash . . . . . . . . . . . . . . . . . . . . . . . . . . . . . . . . . . . . . . . . . . 7,725

Paid note with interest, including accrued interest payable.

b. Interest expense for current year = 12% × $7,500 × 80/360 = $200 Interest expense for following year = 12% × $7,500 × 10/360 = $25

c. Disclose the pending lawsuit in the financial statement notes. Although the loss is probable, no liability is accrued because the loss cannot be reasonably estimated.

d. Nov. 30 Salaries Expense . . . . . . . . . . . . . . . . . . . . . . . . . . . . . . . . . . . . 3,000.00

FICA—Social Security Taxes Payable (6.2%) . . . . . . . . . . 186.00

FICA—Medicare Taxes Payable (1.45%) . . . . . . . . . . . . . 43.50

Employee Federal Income Taxes Payable . . . . . . . . . . . 800.00

Employee State Income Taxes Payable . . . . . . . . . . . . . 200.00

Employee Medical Insurance Payable . . . . . . . . . . . . . . 35.00

Employee United Way Payable . . . . . . . . . . . . . . . . . . . . 10.00

Salaries Payable . . . . . . . . . . . . . . . . . . . . . . . . . . . . . . . . 1,725.50

Record Bline’s accrued payroll.

356 Chapter 9 Accounting for Current Liabilities

e. Nov. 1 Cash . . . . . . . . . . . . . . . . . . . . . . . . . . . . . . . . . . . . . . . . . . . . . . 5,000

Notes Payable . . . . . . . . . . . . . . . . . . . . . . . . . . . . . . . . . . 5,000

Borrowed cash with a 60-day, 12% note.

Dec. 31 Notes Payable . . . . . . . . . . . . . . . . . . . . . . . . . . . . . . . . . . . . . . 5,000

Interest Expense . . . . . . . . . . . . . . . . . . . . . . . . . . . . . . . . . . . . 100

Cash . . . . . . . . . . . . . . . . . . . . . . . . . . . . . . . . . . . . . . . . . 5,100

Paid note with interest ($5,000 × 12% × 60∕360).

When the note and interest are paid 60 days later, Kern Co. records this entry.

g. Times interest earned = $1,000,000 + $275,000 + $225,000

$275,000 = 5.45 times

Dec. 31 Income Taxes Expense . . . . . . . . . . . . . . . . . . . . . . . . . . . . . . . 5,000

Income Taxes Payable . . . . . . . . . . . . . . . . . . . . . . . . . . . 4,000

Deferred Income Tax Liability . . . . . . . . . . . . . . . . . . . . . 1,000

Record added income taxes expense and the deferred tax liability.

f.B

APPENDIX

Payroll Reports, Records, and Procedures9A

This appendix focuses on payroll accounting reports, records, and procedures.

Payroll Reports Most employees and employers are required to pay local, state, and federal payroll taxes. Payroll expenses are liabilities to individual employees, to federal and state governments, and to other organizations such as insurance companies. Employers are required to prepare and submit reports explaining how they computed these payments.

Reporting FICA Taxes and Income Taxes The Federal Insurance Contributions Act (FICA) requires each employer to file an Internal Revenue Service (IRS) Form 941, the Employer’s Quarterly Federal Tax Return, within one month after the end of each calendar quarter. A sample Form 941 is shown in Exhibit 9A.1 for Phoenix Sales & Service, a landscape design company. Accounting information and soft- ware are helpful in tracking payroll transactions and reporting the accumulated information on Form 941. Specifically, the employer reports total wages subject to income tax withholding on line 2 of Form 941. (For simplicity, this appendix uses wages to refer to both wages and salaries.) The income tax withheld is reported on line 3. The combined amount of employee and employer FICA (Social Security) taxes for Phoenix Sales & Service is reported on line 5a (taxable Social Security wages, $36,599 × 12.4% = $4,538.28). The 12.4% is the sum of the Social Security tax withheld, computed as 6.2% tax withheld from the employee wages for the quarter, plus the 6.2% tax levied on the employer. The combined amount of employee Medicare wages is reported on line 5c. The 2.9% is the sum of 1.45% withheld from employee wages for the quarter plus 1.45% tax levied on the employer. Total FICA taxes are reported on line 5e and are added to the total income taxes withheld of $3,056.47 to yield a total of $8,656.12. For this year, assume that income up to $128,400 is subject to Social Security tax. There is no income limit on amounts subject to Medicare tax. Congress sets rates owed for Social Security tax (and it typically changes each year). Federal depository banks are authorized to accept deposits of amounts payable to the federal govern- ment. Deposit requirements depend on the amount of tax owed. For example, when the sum of FICA taxes plus the employee income taxes is less than $2,500 for a quarter, the taxes can be paid when Form 941 is filed.

Reporting FUTA Taxes and SUTA Taxes An employer’s federal unemployment taxes (FUTA) are reported on an annual basis by filing an Annual Federal Unemployment Tax Return, IRS Form 940. It must be mailed on or before January 31 following the end of each tax year. Ten more days are allowed if all required tax deposits are filed on a timely basis and the full amount of tax is paid on or before January 31. FUTA pay- ments are made quarterly to a federal depository bank if the total amount due exceeds $500. If $500 or less

Point: Deposits for federal payroll taxes must be made by electronic funds transfer (EFT).

P5 Identify and describe the details of payroll reports, records, and procedures.

Chapter 9 Accounting for Current Liabilities 357

3,079 11

2,049 77

3,527 24

8,656 12

enter the final date you paid wages .

If your business has closed or you stopped paying wages

Part 2: Tell us about your deposit schedule and tax liability for this quarter.

.

.

.

If you are unsure about whether you are a monthly schedule depositor or a semiweekly schedule depositor, see section 11 of Pub. 15.

16

17

18

Check one: Line 12 on this return is less than $2,500 or line 12 (line 10 if the prior quarter was the fourth quarter of last year) on the return for the prior quarter was less than $2,500, and you didn’t incur a $100,000 next-day deposit obligation during the current quarter. If line 12 (line 10 if the prior quarter was the fourth quarter of last year) for the prior quarter was less than $2,500 but line 12 on this reutrn is $100,000 or more, you must provide a record of your federal tax liability. If you are a monthly schedule depositor, complete the deposit schedule below; if you are a semiweekly schedule depositor, attach Schedule B (Form 941). Go to Part 3. You were a monthly schedule depositor for the entire quarter. Enter your tax liability for each month and total liability for the quarter, then go to Part 3.

.

Tax liability: Month 1

Month 2

Month 3

Total liability for quarter Total must equal line 12.

You were a semiweekly schedule depositor for any part of this quarter. Fill out Schedule B (Form 941): Report of Tax Liability for Semiweekly Schedule Depositors, and attach it to Form 941.

Check here, and

If you are a seasonal employer and you do not have to file a return for every quarter of the year

Do you want to allow an employee, a paid tax preparer, or another person to discuss this return with the IRS? See the instructions for details.

Yes. Designee’s name and phone number

Select a 5-digit Personal Identification Number (PIN) to use when talking to the IRS.

Part 3: Tell us about your business. If a question does NOT apply to your business, leave it blank.

Check here.

Part 4: May we speak with your third-party designee?

No.

Part 5: Sign here. You MUST complete both pages of Form 941 and SIGN it.

Under penalties of perjury, I declare that I have examined this return, including accompanying schedules and statements, and to the best of my knowledge and belief, it is true, correct, and complete. Declaration of preparer (other than taxpayer) is based on all information of which preparer has any knowledge.

Sign your name here

Print your name here Print your title here

Date Best daytime phone/ /

/ /enter the

If your bu

ll u

are ctio

17

18

eck

If you are

Do you wa instruction

Yes.

Part 3: Tell u

Part 4: May w

No.

Part 5: Sign

Under pe to the bes based on

Sign your name here

Date

Part 2: Tel

If you a see sec

16 Che

Department of the Treasury — Internal Revenue Service

(EIN) Employer identification number

Name (not your trade name)

Trade name (if any)

Address Suite or room number

State ZIP code

Report for this Quarter ... (Check one.)

1: January, February, March

2: April, May, June

3: July, August, September

4: October, November, December

941Form

Part 1: Answer these questions for this quarter.

1

4 5

5a

5b

5c

5d

10 11

12 Total taxes after adjustments and credits. Subtract line 11 from line 10

13

Number of employees who received wages, tips, or other compensation for the pay period including: Mar. 12 (Quarter 1), June 12 (Quarter 2), Sept. 12 (Quarter 3), Dec. 12 (Quarter 4)

2 Wages, tips, and other compensation

3 Total income tax withheld from wages, tips, and other compensation

If no wages, tips, and other compensation are subject to social security or Medicare tax Check and go to line 6. Taxable social security and Medicare wages and tips:

Column 1 Column 2

Taxable social security wages × .124 =

Taxable social security tips

Taxable Medicare wages & tips × .029 =

Taxable wages & tips subject to Additional Medicare Tax withholding

5e Add Column 2 from lines 5a, 5b, 5c, and 5d

5f Section 3121(q) Notice and Demand–Tax due to unreported tips (see instructions)

× 0.009 =

Current quarter’s adjustment for fractions of cents

Current quarter’s adjustment for sick pay

Current quarter’s adjustments for tips and group-term like insurance

Total taxes after adjustments. Combine lines 6 through 9 Qualified small business payroll tax credit for increasing research activities. Attach Form 8974

Total deposits for this quarter, including overpayment applied from a prior quarter and overpayments applied from Form 941-X, 941-X (PR), 944-X, or 944-X (SP) filed in the current quarter

14 Balance due. If line 12 is more than line 13, enter the di�erence and see instructions

15 Overpayment. If line 13 is more than line 12, enter the di�erence Check one: Apply to next return.

Send a refund.

× .124 =

7

Total taxes before adjustments. Add lines 3, 5e, and 5f6

7

8

9

10

12

13

14

11

8

9

1

2

3

5e

5f

.

.

.

. . . .

8 6 3 2 1 4 5 8 7

2

36,599

36,599 00

.36,599 00

4,538 28

1,061 37

00

3,056 47

Employer’s QUARTERLY Federal Tax Return

Phoenix Sales Service

1214 Mill Road

85621AZPhoenix Number Street

City

.

.

.

.

.

.

. 5,599.65

.

8,656.12

6 8,656.12

8,656.12

8,656.12 0.00

EXHIBIT 9A.1 Form 941

Point: Line 5a shows the matching nature of FICA tax as 6.2% × 2, or 12.4%, which is shown as 0.124.

Point: Auditors rely on the four 941 Forms filed during a year when auditing a company’s annual wages and salaries expense account.

is due, the taxes are remitted annually. Requirements for paying and reporting state unemployment taxes (SUTA) vary depending on the laws of each state. Most states require quarterly payments and reports.

Reporting Wages and Salaries Employers are required to give each employee an annual report of his or her wages subject to FICA and federal income taxes along with the amounts of these taxes withheld. This report is called a Wage and Tax Statement, or Form W-2. It must be given to employees before January 31 following the year covered by the report. Exhibit 9A.2 shows Form W-2 for one of the

Department of Treasury—Internal Revenue ServiceForm Copy 1–For State, City, or Local Tax Department

Wage and Tax StatementW-2

a Control number

AR101

86-3214587

OMB No. 1545-0006

b Employer identification number (EIN)

4,910.00 1 Wages, tips, other compensation

4,910.00 3 Social security wages

4,910.00 5 Medicare wages and tips

7 Social security tips

9 Advance EIC payment

11 Nonqualified plans 12a Code

12b Code

12c Code

333.37 2 Federal income tax withheld

304.42 4 Social security tax withheld

71.20 6 Medicare tax withheld

8 Allocated tips

10 Dependent care benefits

333-22-9999 d Employee’s social security number

Robert J. e Employee’s first name and initial

AZ 15 State 16 State wages, tips, etc.

13-902319 4,910.00 17 State income tax

26.68 18 Local wages, tips, etc. 19 Local income taxEmployer’s state ID number

f Employee’s address and ZIP code

Austin Last name

c Employer’s name, address and ZIP code

Phoenix Sales & Service 1214 Mill Road

Phoenix, AZ 85621

18 Roosevelt Blvd., Apt. C Tempe, AZ 86322

13 Statutoryemployee Retirement

plan Third-party

sick pay

14 Other

20 Locality name

EXHIBIT 9A.2 Form W-2

358 Chapter 9 Accounting for Current Liabilities

employees at Phoenix Sales & Service. Copies of Form W-2 must be sent to the Social Security Administration, where the amount of the employee’s wages subject to FICA taxes and FICA taxes withheld are posted to each employee’s Social Security account. These posted amounts become the basis for determining an employee’s retirement and survivors’ benefits. The Social Security Administration also transmits to the IRS the amount of each employee’s wages subject to federal income taxes and the amount of taxes withheld.

Payroll Records Employers must keep payroll records in addition to reporting and paying taxes. These records usually include a payroll register and an individual earnings report for each employee.

Payroll Register A payroll register usually shows the pay period dates, hours worked, gross pay, deduc- tions, and net pay of each employee for each pay period. Exhibit 9A.3 shows a payroll register for Phoenix Sales & Service. It is organized into nine columns:

Col. A Employee Identification (ID); Employee name; Social Security number (SS No.); Reference (check number); and Date (date check issued)

Col. B Pay Type (regular and overtime) Col. C Pay Hours (number of hours worked as regular and overtime) Col. D Gross Pay (amount of gross pay) Col. E FIT (federal income taxes withheld); FUTA (federal unemployment taxes) Col. F SIT (state income taxes withheld); SUTA (state unemployment taxes) Col. G FICA-SS_EE (Social Security taxes withheld, employee); FICA-SS_ER (Social Security taxes,

employer) Col. H FICA-Med_EE (Medicare tax withheld, employee); FICA-Med_ER (Medicare tax, employer) Col. I Net Pay (gross pay less amounts withheld from employees)

Net pay for each employee is computed as gross pay minus the items on the first line of columns E through H. The employer’s payroll tax for each employee is computed as the sum of items on the third line of columns E through H. A payroll register includes all data necessary to record payroll. In some software programs, the entries to record payroll are made in a special payroll journal.

Point: Gross Pay column shows regular hours worked on the first line multiplied by regular pay rate. Overtime hours multiplied by the overtime premium rate equals overtime pay on the second line. For this company, workers earn 150% of their regular rate for hours in excess of 40 per week.

A B C D E F G H I

Pay Type

Pay Hours

Gross Pay

Gross Pay

Net Pay

AR101 Robert Austin 333-22-9999 9001, 1/8/19 CJ102 Judy Cross 299-11-9201 9002, 1/8/19 DJ103 John Diaz 444-11-9090 9003, 1/8/19 KK104 Kay Keife 909-11-3344 9004, 1/8/19 ML105 Lee Miller 444-56-3211 9005, 1/8/19 SD106 Dale Sears 909-33-1234 9006, 1/8/19

Employee ID Employee SS No. Refer., Date

FICA-Med_EE

FICA-Med_ER [blank]

FICA-SS_ER

FICA-SS_EE [blank]

SUTA

SIT [blank]

FUTA

FIT [blank]

Phoenix Sales & Service Payroll Register

For Week Ended Jan. 8, 2019

Regular Overtime

Regular Overtime

Regular Overtime

Regular Overtime

Regular Overtime

Regular Overtime

Regular Overtime

40.00

40.00 1.00

40.00

40.00

40.00

40.00 2.00

0.00

0.00

0.00

0.00

240.00 3.00

0.00

400.00

400.00

560.00 21.00

560.00

560.00

560.00

560.00

560.00

560.00

560.00

42.00

581.00

602.00

0.00

0.00

0.00

3,200.00 63.00

3,263.00

–28.99

–2.40

–52.97

–3.49

–48.33

–3.61

–3.36

–3.36

–3.36

–68.57

–34.24

–68.57

–301.67

–19.58 –88.10

–24.15

–15.12

–16.25

–15.12

–5.49

–5.49

–15.12

–2.74

–3.87

–15.69

–4.24

–10.80

–2.32

–34.72

–34.72

–34.72

–34.72

–34.72

–34.72

–37.32

–37.32

–36.02

–36.02

–24.80

–24.80

–202.30

–202.30

–8.12

–8.12

–8.12

–8.12

–8.12

–8.12

–8.73

–8.73

–8.42

–8.42

–5.80

–5.80

–47.31

–47.31

338.09

479.35

503.75

443.10

480.18

443.10

2,687.57Totals

EXHIBIT 9A.3 Payroll Register

Chapter 9 Accounting for Current Liabilities 359

Payroll Check Payment of payroll is usually done by check or electronic funds transfer. Exhibit 9A.4 shows a payroll check for a Phoenix employee. This check includes a detachable statement of earnings (at top) showing gross pay, deductions, and net pay.

AR101 Robert Austin 333-22-9999 1/8/19 1/8/19 EMPLOYEE NO. EMPLOYEE NAME SOCIAL SECURITY NO. PAY PERIOD END

YEAR TO DATETHIS CHECKITEMTOTALHOURSRATEITEM

CHECK DATE

40.00

HOURS WORKED 400.00

GROSS THIS PERIOD 400.00

GROSS YEAR TO DATE

$338.09

NET CHECK 9001

CHECK NO.

(Detach and retain for your records)

Regular Overtime

10.00 15.00

40.00

Three Hundred Thirty–Eight and 9/100 Dollars

Gross Fed. Income tax FICA-Soc. Sec. FICA-Medicare State Income tax

400.00 -28.99 -24.80 -5.80 -2.32

400.00 -28.99 -24.80 -5.80 -2.32

PHOENIX SALES & SERVICE 1214 Mill Road Phoenix, AZ 85621 602-555-8900

Phoenix Bank and Trust Phoenix, AZ 85621 3312-87044

No. 9001

AUTHORIZED SIGNATURE

400.00

.................... 20 .......DATE January 8 19 Check No. .............9001

Robert Austin 18 Roosevelt Blvd., Apt C Tempe, AZ 86322

Pay to the order of

$ **************$338.09Amount .........................................................................................................

EXHIBIT 9A.4 Check and Statement of Earnings

Employee Earnings Report An employee earnings report is a cumulative record of an employee’s hours worked, gross earnings, deductions, and net pay. Payroll information on this report is taken from the payroll register. The employee earnings report for R. Austin at Phoenix Sales & Service is shown in Exhibit 9A.5. An employee earnings report accumulates information that can show when an employee’s

[blank][blank]Date Reference

Gross Pay

FICA-SS_EE FICA-Med_EE [blank] FIT

FUTA [blank] SIT

SUTA FICA-SS_ER FICA-Med_ER Net Pay

2,483.88

338.09

338.09

338.09

338.09

338.09

1,690.45

4,174.33

Beginning balance for Robert Austin

AR101 Robert Austin 333-22-9999

AR101 Robert Austin 333-22-9999

AR101 Robert Austin 333-22-9999

AR101 Robert Austin 333-22-9999

AR101 Robert Austin 333-22-9999

Total 5-wk month thru 12/31/19

Employee ID Employee SS No.

Year-to-date total for Robert Austin

12/03/19 9049

11/26/19 (balance)

12/10/19 9055

12/17/19 9061

12/24/19 9067

12/31/19 9073

12/31/19 (balance)

4,910.00

2,000.00

400.00

400.00

400.00

400.00

400.00

2,910.00

–29.46

–333.37 –12.00

–144.95 –2.40

–2.40

–2.40

–28.99

–28.99

–2.40 –28.99

–28.99

–28.99 –17.46

–188.42

–2.40

–304.42

–304.42 –124.00

–124.00 –24.80

–24.80

–24.80

–24.80

–24.80

–24.80 –24.80

–24.80

–24.80 –180.42

–180.42

–24.80

–71.20

–71.20 –29.00

–29.00 –5.80

–5.80

–5.80

–5.80

–5.80

–5.80 –5.80

–5.80

–5.80 –42.20

–42.20

–5.80

–132.57

–26.68 –54.00

–11.60 –10.80

–10.80

–10.80

–2.32

–2.32

–10.80 –2.32

–2.32

–2.32 –78.57

–15.08

–10.80

Phoenix Sales & Service Employee Earnings Report

For Month Ended Dec. 31, 2019

EXHIBIT 9A.5 Employee Earnings Report

Point: Year-end balances agree with W-2.

360 Chapter 9 Accounting for Current Liabilities

Employers often use a wage bracket withholding table similar to the one shown in Exhibit 9A.6 to compute the federal income taxes withheld from each employee’s gross pay. The table in Exhibit 9A.6 is for a single employee paid weekly. Tables also are provided for married employees and for biweekly, semimonthly, and monthly pay periods (most payroll software includes these tables). When using a wage bracket withholding table to compute federal income tax withheld from an employee’s gross wages, we need to locate an employee’s wage bracket within the first two columns. We then find the amount withheld by looking in the withholding allowance column for that employee.

SINGLE Persons—WEEKLY Payroll Period If the wages are–

At least

$600 610 620 630 640 650 660 670 680 690 700 710 720 730 740

$610 620 630 640 650 660 670 680 690 700 710 720 730 740 750

$76 79 81 84 86 89 91 94 96 99 101 104 106 109

111

$67 69 70 72 73 75 76 78 81 83 86 88 91 93 96

$58 59 61 62 64 65 67 68 70 71 73 74 76 78 80

$49 50 52 53 55 56 58 59 61 62 64 65 67 68 70

$39 41 42 44 45 47 48 50 51 53 54 56 57 59 60

$30 32 33 35 36 38 39 41 42 44 45 47 48 50 51

$21 22 24 25 27 28 30 31 33 34 35 37 39 40 42

$12 13 15 16 18 19 21 22 24 25 27 28 30 31 33

$6 7 8 9

10 11 12 13 14 16 17 19

20 22 23

$0 1 2 3 4 5 6 7 8 9

10 11 12 13 14

$0 0 0 0 0 0 0 1 2 3 4 5 6 7 8

But less than

And the number of withholding allowances claimed is— 0 1 2 3 4 5 6 7 8 9 10

The amount of income tax to be withheld is—

EXHIBIT 9A.6 Wage Bracket Withholding Table

Payroll Bank Account Companies with few employees often pay them with checks drawn on the compa- ny’s regular bank account. Companies with many employees often use a special payroll bank account to pay employees. When this account is used, a company either (1) draws one check for total payroll on the regular bank account and deposits it in the payroll bank account or (2) executes an electronic funds transfer to the payroll bank account. Individual payroll checks are then drawn on this payroll bank ac- count. Because only one check for the total payroll is drawn on the regular bank account each payday, use of a special payroll bank account helps with internal control. It also helps in reconciling the regular bank account. When companies use a payroll bank account, they usually include check numbers in the payroll register. The payroll register in Exhibit 9A.3 shows check numbers in column A. For instance, Check No.

Form

1

5

6

7

8 10

Your first name and middle initial

Home address (number and street or rural route)

Total number of allowances you are claiming (from line H above or from the applicable worksheet on page 2)

Employer’s name and address (Employer: Complete lines 8 and 10 only if sending to the IRS.) 9 O…ce code (optional)

Cat. No. 10220Q Form W-4

Employer identification number (EIN)

Under penalties of perjury, I declare that I have examined this certificate and, to the best of my knowledge and belief, it is true, correct, and complete.

Employee’s signature

For Privacy Act and Paperwork Reduction Act Notice, see page 2.

(This form is not valid unless you sign it.) ƒ Date ƒ

Additional amount, if any, you want withheld from each paycheck . . . . . . . . . . . . . . . . . . . . . . . . . . . . . . . . . . . . . . . . . . . I claim exemption from withholding for 20 , and I certify that I meet both of the following conditions for exemption.

• Last year I had a right to a refund of all federal income tax withheld because I had no tax liability, and • This year I expect a refund of all federal income tax withheld because I expect to have no tax liability. If you meet both conditions, write “Exempt” here . . . . . . . . . . . . . . . . . . . . . . . . . . . . . . . . . . . . . . . . . . .

City or town, state, and ZIP code

2 Your social security numberLast name

3

4

5

6

7

$

If your last name di‹ers from that shown on your social security card, check here. You must call 1–800–772–1213 for a replacement card.

Single Married Married, but withhold at higher Single rate.

Note: If married, but legally separated, or spouse is a nonresident alien, check the “Single” box.

Department of the Treasury Internal Revenue Service

OMB No. 1545–0074

20ƒ Whether you are entitled to claim a certain number of allowances or exemption from withholding issubject to review by the IRS. Your employer may be required to send a copy of this form to the IRS. W-4 Employee’s Withholding Allowance Certificate

Robert J. Austin 333-22-9999

18 Roosevelt Blvd., Apt. C

Tempe, AZ 86322

Robert J. Austin January 1

86-3214587Phoenix Sales & Service, 1214 Mill Rd, Phoenix, AZ 85621

ƒ

ƒ

1

earnings reach the tax-exempt points for FICA, FUTA, and SUTA taxes. It also gives data an employer needs to prepare Form W-2.

Payroll Procedures Employers must be able to compute federal income tax for payroll pur- poses. This section explains how we compute this tax and how to use a payroll bank account.

Computing Federal Income Taxes To compute the amount of taxes withheld from each employee’s wages, we need to determine both the employee’s wages earned and the employee’s number of withholding allow- ances. Each employee records the number of withholding allowances claimed on a withholding allowance certificate, Form W-4, filed with the employer. When the number of withholding allowances increases, the amount of income taxes withheld decreases.

Chapter 9 Accounting for Current Liabilities 361

9001 is issued to Robert Austin. With this information, the payroll register serves as a supplementary record of wages earned by and paid to employees.

Who Pays What Payroll Taxes and Benefits We conclude this appendix with the following table identifying who pays which payroll taxes and which common employee benefits such as medical, disability, pension, charitable, and union costs. Who pays which employee benefits, and what portion, is subject to agreements between companies and their workers. Also, self-employed workers must pay both the employer and employee FICA taxes for Social Security and Medicare.

APPENDIX

Corporate Income Taxes 9B This appendix covers current liabilities for income taxes of C corporations. Income tax on sole proprietor- ships, partnerships, S corporations, and LLCs is computed on their owner’s tax filings and is not covered here.

Income Tax Liabilities Corporations are subject to income taxes and must estimate their income tax liabil- ity when preparing financial statements. Because income tax expense is created by earning income, a liability is incurred when income is earned. This tax must be paid quarterly. Consider a corporation that prepares monthly financial statements. Based on its income in January, this corporation estimates that it owes income taxes of $12,100. The following adjusting entry records this estimate.

The tax liability is recorded each month until the first quarterly payment is made. If the company’s esti- mated taxes for this first quarter total $30,000, the entry to record its payment is

This process of accruing and then paying estimated income taxes continues through the year. When an- nual financial statements are prepared at year-end, the corporation knows its actual total income and the actual amount of income taxes it must pay. This information allows it to accurately record income taxes expense for the fourth quarter so that the total of the four quarters’ expense amounts equals the actual taxes paid to the government.

Deferred Income Tax Liabilities An income tax liability for corporations can arise when the amount of income before taxes that the corporation reports on its income statement is not the same as the amount of

Jan. 31 Income Taxes Expense . . . . . . . . . . . . . . . . . . . . . . . . . . . . . . . 12,100

Income Taxes Payable . . . . . . . . . . . . . . . . . . . . . . . . . . . 12,100

Accrue January income taxes.

Assets = Liabilities + Equity +12,100 −12,100

Assets = Liabilities + Equity −30,000 −30,000

Apr. 10 Income Taxes Payable . . . . . . . . . . . . . . . . . . . . . . . . . . . . . . . 30,000

Cash . . . . . . . . . . . . . . . . . . . . . . . . . . . . . . . . . . . . . . . . . . 30,000

Paid estimated first-quarter income taxes.

Point: IRS reports average (effective) income tax rates for categories of income earners:

Top 1% . . . . . . . . . . . . . . . . . 24% Top 5% . . . . . . . . . . . . . . . . . 20% Top 10% . . . . . . . . . . . . . . . 18% Lower 50% . . . . . . . . . . . . . <2%

Employer Payroll Taxes and Costs Employee Payroll Deductions

• FICA—Social Security taxes • FICA—Social Security taxes

• FICA—Medicare taxes • FICA—Medicare taxes

• FUTA (federal unemployment taxes) • Federal income taxes

• SUTA (state unemployment taxes) • State and local income taxes

• Share of medical coverage, if any • Share of medical coverage, if any

• Share of pension coverage, if any • Share of pension coverage, if any

• Share of other benefits, if any • Share of other benefits, if any

Year-To-Date Pay Employer Taxes Employee Taxes $0 to $7,000 FICA—Medicare FICA—Medicare FICA—Social Security FICA—Social Security FUTA State & Federal Income Tax SUTA

$7,000 to $128,400 FICA—Medicare FICA—Medicare FICA—Social Security FICA—Social Security State & Federal Income Tax

Above $128,400 FICA—Medicare FICA—Medicare State & Federal Income Tax

362 Chapter 9 Accounting for Current Liabilities

income reported on its income tax return. This difference occurs because income tax laws and GAAP measure income differently. Differences between tax laws and GAAP arise because Congress uses tax laws to generate receipts, stimulate the economy, and influence behavior, whereas GAAP is intended to provide financial information useful for business decisions. Also, tax accounting often follows the cash basis, whereas GAAP follows the accrual basis. Some differences between tax laws and GAAP are temporary. Temporary differences arise when the tax return and the income statement report a revenue or expense in different years. As an example, com- panies are often able to deduct higher amounts of depreciation in the early years of an asset’s life and smaller amounts in later years for tax reporting in comparison to GAAP. This means that in the early years, depreciation for tax reporting is often more than depreciation on the income statement. In later years, depreciation for tax reporting is often less than depreciation on the income statement. When tempo- rary differences exist between taxable income on the tax return and the income before taxes on the income statement, corporations compute income taxes expense based on the income reported on the income state- ment. The result is that income taxes expense reported in the income statement is often different from the amount of income taxes payable to the government. This difference is the deferred income tax liability. Assume that in recording its usual quarterly income tax payments, a corporation computes $25,000 of income taxes expense. It also determines that only $21,000 is currently due and $4,000 is deferred to future years (a timing difference). The entry to record this end-of-period adjustment is

Point: For a temporary difference, if GAAP income exceeds taxable income, a deferred tax liability is created. If GAAP income is initially less than taxable income, a deferred tax asset is created.

The credit to Income Taxes Payable is the amount currently due to be paid. The credit to Deferred Income Tax Liability is tax payments deferred until future years when the temporary difference reverses.

Deferred Income Tax Assets Temporary differences also can cause a company to pay income taxes before they are reported on the income statement. If so, the company reports a Deferred Income Tax Asset on its balance sheet.

Dec. 31 Income Taxes Expense . . . . . . . . . . . . . . . . . . . . . . . . . . . . . . . 25,000

Income Taxes Payable . . . . . . . . . . . . . . . . . . . . . . . . . . . 21,000

Deferred Income Tax Liability . . . . . . . . . . . . . . . . . . . . . 4,000

Record tax expense and deferred tax liability.

Assets = Liabilities + Equity +21,000 −25,000 +4,000

KNOWN LIABILITIES Current liabilities (or short-term liabilities): Liabilities due within one year. Long-term liabilities: Liabilities due after one year.

Summary: Cheat Sheet

Note and interest paid:

Notes Payable . . . . . . . . . . . . . . . . . . . . . . . . . . . . . . . . . . 500

Interest Expense . . . . . . . . . . . . . . . . . . . . . . . . . . . . . . . 10

Cash . . . . . . . . . . . . . . . . . . . . . . . . . . . . . . . . . . . . . 510

Interest expense incurred but not yet paid:

Interest Expense . . . . . . . . . . . . . . . . . . . . . . . . . . . . . . . 10

Interest Payable . . . . . . . . . . . . . . . . . . . . . . . . . . . 10

Unearned revenue is earned: To record service or product delivered.

Unearned Revenue . . . . . . . . . . . . . . . . . . . . . . . . . . 625,000

Revenue . . . . . . . . . . . . . . . . . . . . . . . . . . . . . . 625,000

Note given to replace accounts payable (partial cash paid):

Accounts Payable . . . . . . . . . . . . . . . . . . . . . . . . . . . 600

Cash . . . . . . . . . . . . . . . . . . . . . . . . . . . . . . . . . 100

Notes Payable . . . . . . . . . . . . . . . . . . . . . . . . . 500

Cash . . . . . . . . . . . . . . . . . . . . . . . . . . . . . . . . . . . . . 5,000,000

Unearned Revenue . . . . . . . . . . . . . . . . . . . . . 5,000,000

Unearned revenues (or deferred revenues): Amount received in advance from customers for future products or services; to record cash received in advance.

Short-term note payable: A written promise to pay a specified amount on a stated future date within one year.

Interest formula (year assumed to have 360 days):

Principal of the note ×

Annual interest rate ×

Time expressed in fraction of year = Interest

PAYROLL LIABILITIES Gross pay: Total compensation an employee earns before deductions such as taxes. Payroll deductions (or withholdings): Amounts withheld from an employ- ee’s gross pay, either required or voluntary. FICA—Social Security taxes payable: Withholdings to cover retirement, disability, and survivorship. Social Security tax is 6.2% of the first $128,400 the employee earns for the year.

Cash . . . . . . . . . . . . . . . . . . . . . . . . . . . . . . . . . . . . . 6,300

Sales . . . . . . . . . . . . . . . . . . . . . . . . . . . . . . . . . 6,000

Sales Taxes Payable . . . . . . . . . . . . . . . . . . . . 300

Sales tax collection:

Note given to borrow cash:

Cash . . . . . . . . . . . . . . . . . . . . . . . . . . . . . . . . . . . . . . . . . 2,000

Notes Payable . . . . . . . . . . . . . . . . . . . . . . . . . . . . . 2,000

Chapter 9 Accounting for Current Liabilities 363

FICA—Medicare taxes payable: Withholdings to cover medical bene- fits. The Medicare tax is 1.45% of all amounts the employee earns; there is no maximum limit to Medicare tax. Employee federal income taxes payable: Federal income tax withheld from each employee’s paycheck. Employee voluntary deductions: Voluntary withholdings for things such as union dues, charitable giving, and health insurance.

Federal Unemployment Tax Act (FUTA): Employers pay a federal unem- ployment tax on wages and salaries earned by their employees. FUTA taxes are between 0.6% and 6.0% of the first $7,000 earned by each employee. State Unemployment Tax Act (SUTA): Employers pay a state unemploy- ment tax on wages and salaries earned by their employees. SUTA taxes are up to 5.4% of the first $7,000 earned by each employee.

ESTIMATED LIABILITIES

Employee payroll taxes:

Salaries Expense . . . . . . . . . . . . . . . . . . . . . . . . . . . . . . . 2,000

FICA—Social Security Taxes Payable (6.2%) . . . . . 124

FICA—Medicare Taxes Payable (1.45%) . . . . . . . . 29

Employee Federal Income Taxes Payable . . . . . . . 213

Employee Medical Insurance Payable . . . . . . . . . . 85

Employee Union Dues Payable . . . . . . . . . . . . . . . 25

Salaries Payable . . . . . . . . . . . . . . . . . . . . . . . . . . . 1,524

Employer payroll taxes expense:

Payroll Taxes Expense . . . . . . . . . . . . . . . . . . . . . . . . . . . 273

FICA—Social Security Taxes Payable (6.2%) . . . . . 124

FICA—Medicare Taxes Payable (1.45%) . . . . . . . . 29

State Unemployment Taxes Payable . . . . . . . . . . . 108

Federal Unemployment Taxes Payable . . . . . . . . . 12

Payment of salary to employees:

Salaries Payable . . . . . . . . . . . . . . . . . . . . . . . . . . . . . . . . 1,524

Cash . . . . . . . . . . . . . . . . . . . . . . . . . . . . . . . . . . . . 1,524

Accrual of vacation benefits (also called paid absences):

Vacation Benefits Expense . . . . . . . . . . . . . . . . . . . . . . . 3,200

Vacation Benefits Payable . . . . . . . . . . . . . . . . . . . 3,200

Warranty: A seller’s obligation to replace or fix a product (or service) that fails to perform as expected within a specified period. Warranty expense is recorded in the period when revenue from the sale of the prod- uct or service is reported.

CONTINGENCIES AND ANALYSIS Contingent liability: A potential liability that depends on a future event arising from a past transaction or event. An example is a pending lawsuit.

Warranty expense accrued:

Warranty Expense . . . . . . . . . . . . . . . . . . . . . . . . . . . . . . 640

Estimated Warranty Liability . . . . . . . . . . . . . . . . . 640

Warranty repairs and replacements:

Estimated Warranty Liability . . . . . . . . . . . . . . . . . . . . . . 200

Auto Parts Inventory . . . . . . . . . . . . . . . . . . . . . . . . 200

Vacation benefits are used:

Vacation Benefits Payable . . . . . . . . . . . . . . . . . . . . . . . . 400

Cash . . . . . . . . . . . . . . . . . . . . . . . . . . . . . . . . . . . . . 400

Bonus plan accrued:

Employee Bonus Expense . . . . . . . . . . . . . . . . . . . . . . . . 10,000

Bonus Payable . . . . . . . . . . . . . . . . . . . . . . . . . . . . 10,000

Health and pension benefits:

Employee Benefits Expense. . . . . . . . . . . . . . . . . . . . . . . 20,000

Employee Medical Insurance Payable . . . . . . . . . . 8,000

Employee Retirement Program Payable . . . . . . . . 12,000

Nonestimable

Estimable

Possible

Remote

Probable

Contingent liability

Future event is Amount owed is Record liability

No disclosure

Disclose in notes

Contingent liability (352) Current liabilities (341) Current portion of long-term debt (349) Deferred income tax liability (362) Employee benefits (349) Employee earnings report (359) Estimated liability (349) Federal depository bank (356) Federal income taxes withheld (360) Federal Insurance Contributions Act

(FICA) taxes (346)

Federal Unemployment Tax Act (FUTA) (347)

Form 940 (356) Form 941 (356) Form W-2 (357) Form W-4 (360) Gross pay (346) Known liabilities (342) Long-term liabilities (342) Merit rating (348) Net pay (346)

Payroll bank account (360) Payroll deductions (346) Payroll register (358) Short-term note payable (343) State Unemployment Tax Act

(SUTA) (347) Times interest earned (354) Wage bracket withholding table (360) Warranty (350)

Key Terms

364 Chapter 9 Accounting for Current Liabilities

Multiple Choice Quiz

1. On December 1, a company signed a $6,000, 90-day, 5% note payable, with principal plus interest due on March 1 of the following year. What amount of interest expense should be accrued at December 31 on the note? a. $300 c. $100 e. $0 b. $25 d. $75

2. An employee earned $50,000 during the year. FICA tax for Social Security is 6.2% and FICA tax for Medicare is 1.45%. The employer’s share of FICA taxes is a. $0; employee’s pay exceeds FICA limit. b. $0; FICA is not an employer tax. c. $3,100. d. $725. e. $3,825.

3. Assume the FUTA tax rate is 0.6% and the SUTA tax rate is 5.4%. Both taxes are applied to the first $7,000 of an employee’s pay. What is the total unemployment tax an employer must pay on an employee’s annual wages of $40,000?

a. $2,400 b. $420 c. $42 d. $378 e. $0; employee’s wages exceed the $7,000 maximum.

4. A company sold 10,000 TVs in July and estimates warranty expense for these TVs to be $25,000. During July, 80 TVs were serviced under warranty at a cost of $18,000. The credit balance in the Estimated Warranty Liability account at July 1 was $26,000. What is the company’s warranty ex- pense for the month of July? a. $51,000 c. $25,000 e. $18,000 b. $1,000 d. $33,000

5. AXE Co. is the defendant in a lawsuit. AXE reasonably esti- mates that this pending lawsuit will result in damages of $99,000. It is probable that AXE will lose the case. What should AXE do? a. Record a liability c. Have no disclosure b. Disclose in notes

ANSWERS TO MULTIPLE CHOICE QUIZ

1. b; $6,000 × 0.05 × 30∕360 = $25 2. e; $50,000 × (0.062 + 0.0145) = $3,825 3. b; $7,000 × (0.006 + 0.054) = $420

4. c; $25,000 5. a; Reason—it is reasonably estimated and is a probable loss. AXE

would record an estimated legal expense and liability.

A(B) Superscript letter A or B denotes assignments based on Appendix 9A or 9B.

Icon denotes assignments that involve decision making.

1. What is the difference between a current and a long- term liability?

2. What is an estimated liability? 3. What are the three important questions concerning the

uncertainty of liabilities? 4. What is the combined amount (in percent) of the employee

and employer Social Security tax rate? (Assume wages do not exceed $128,400 per year.)

5. What is the current Medicare tax rate? This rate is applied to what maximum level of salary and wages?

6. Which payroll taxes are the employee’s responsibility and which are the employer’s responsibility?

7. What determines the amount deducted from an employee’s wages for federal income taxes?

8. What is an employer’s unemployment merit rating? How are these ratings assigned to employers?

9. Why are warranty liabilities usually recognized on the balance sheet as liabilities even when they are uncertain?

10. Suppose a company has a facility located where disas- trous weather conditions often occur. Should it report a

probable loss from a future disaster as a liability on its bal- ance sheet? Explain.

11.A What is a wage bracket withholding table? 12.A What amount of income tax is withheld from the salary of

an employee who is single with two withholding allow- ances and earns $725 per week? What if the employee earns $625 and has no withholding allowances? (Use Exhibit 9A.6.)

13. Refer to Apple’s balance sheet in Appendix A. What is the amount of Apple’s accounts pay- able as of September 30, 2017?

14. Refer to Google’s balance sheet in Appendix A. What “accrued” expenses (liabilities) does Google report at December 31, 2017?

15. Refer to Samsung’s balance sheet in Appendix A. List Samsung’s current lia- bilities as of December 31, 2017.

16. Refer to Samsung’s recent balance sheet in Appendix A. What current liabil- ities related to income taxes are on its balance sheet? Explain the meaning of each income tax account identified.

Discussion Questions

APPLE

Samsung

Samsung

GOOGLE

Chapter 9 Accounting for Current Liabilities 365

Dextra Computing sells merchandise for $6,000 cash on September 30 (cost of merchandise is $3,900). Dextra collects 5% sales tax. (1) Record the entry for the $6,000 sale and its sales tax. (2) Record the entry that shows Dextra sending the sales tax on this sale to the government on October 15.

QS 9-2 Accounting for sales taxes

C2

Ticketsales, Inc., receives $5,000,000 cash in advance ticket sales for a four-date tour of Bon Jovi. Record the advance ticket sales on October 31. Record the revenue earned for the first concert date of November 5, assuming it represents one-fourth of the advance ticket sales.

QS 9-3 Unearned revenue C2

On November 7, Mura Company borrows $160,000 cash by signing a 90-day, 8%, $160,000 note payable. (1) Compute the accrued interest payable on December 31; (2) prepare the journal entry to record the ac- crued interest expense at December 31; and (3) prepare the journal entry to record payment of the note at maturity on February 5.

QS 9-4 Interest-bearing note transactions P1

On January 15, the end of the first pay period of the year, North Company’s employees earned $35,000 of sales salaries. Withholdings from the employees’ salaries include FICA Social Security taxes at the rate of 6.2%, FICA Medicare taxes at the rate of 1.45%, $6,500 of federal income taxes, $772.50 of medical in- surance deductions, and $120 of union dues. No employee earned more than $7,000 in this first period. Prepare the journal entry to record North Company’s January 15 salaries expense and related liabilities. (Round amounts to cents.)

QS 9-5 Recording employee payroll taxes

P2

Merger Co. has 10 employees, each of whom earns $2,000 per month and has been employed since January 1. FICA Social Security taxes are 6.2% of the first $128,400 paid to each employee, and FICA Medicare taxes are 1.45% of gross pay. FUTA taxes are 0.6% and SUTA taxes are 5.4% of the first $7,000 paid to each employee. Prepare the March 31 journal entry to record the March payroll taxes expense.

QS 9-6 Recording employer payroll taxes P3

Noura Company offers an annual bonus to employees (to be shared equally) if the company meets certain net income goals. Prepare the journal entry to record a $15,000 bonus owed (but not yet paid) to its work- ers at calendar year-end.

QS 9-7 Accounting for bonuses

P4

On September 1, Home Store sells a mower (that costs $200) for $500 cash with a one-year warranty that covers parts. Warranty expense is estimated at 8% of sales. On January 24 of the following year, the mower is brought in for repairs covered under the warranty requiring $35 in materials taken from the Repair Parts Inventory. Prepare the September 1 entry to record the mower sale (and cost of sale) and the January 24 entry to record the warranty repairs.

QS 9-9 Recording warranty repairs

P4

Chavez Co.’s salaried employees earn four weeks’ vacation per year. Chavez estimated and must expense $8,000 of accrued vacation benefits for the year. (a) Prepare the December 31 year-end adjusting entry for accrued vacation benefits. (b) Prepare the entry on April 1 of the next year when an employee takes a one- week vacation and is paid $500 cash for that week.

QS 9-8 Accounting for vacations

P4

QUICK STUDY

QS 9-1 Classifying liabilities

C1

Which of the following items are normally classified as current liabilities for a company that has a one- year operating cycle?

1. Portion of long-term note due in 10 months. 4. Accounts payable due in 11 months. 2. Note payable maturing in 2 years. 5. FICA taxes payable. 3. Note payable due in 18 months. 6. Salaries payable.

Riverrun Co. provides medical care and insurance benefits to its retirees. In the current year, Riverrun agrees to pay $5,500 for medical insurance and contribute an additional $9,000 to a retirement program. Record the entry for these accrued (but unpaid) benefits on December 31.

QS 9-10 Accounting for health and pension benefits P4

Huprey Co. is the defendant in the following legal claims. For each of the following claims, indicate whether Huprey should (a) record a liability, (b) disclose in notes, or (c) have no disclosure.

1. Huprey can reasonably estimate that a pending lawsuit will result in damages of $1,250,000. It is probable that Huprey will lose the case.

2. It is reasonably possible that Huprey will lose a pending lawsuit. The loss cannot be estimated. 3. Huprey is being sued for damages of $2,000,000. It is very unlikely (remote) that Huprey will

lose the case.

QS 9-11 Accounting for contingent liabilities

C3

366 Chapter 9 Accounting for Current Liabilities

Sera Corporation has made and recorded its quarterly income tax payments. After a final review of taxes for the year, the company identifies an additional $40,000 of income tax expense that should be recorded. A portion of this additional expense, $6,000, is deferred for payment in future years. Record Sera’s year- end adjusting entry for income tax expense.

QS 9-15B Recording deferred income tax liability P4

1. On July 15, Piper Co. sold $10,000 of merchandise (costing $5,000) for cash. The sales tax rate is 4%. On August 1, Piper sent the sales tax collected from the sale to the government. Record entries for the July 15 and August 1 transactions.

2. On November 3, the Milwaukee Bucks sold a six-game pack of advance tickets for $300 cash. On November 20, the Bucks played the first game of the six-game pack (this represented one-sixth of the advance ticket sales). Record the entries for the November 3 and November 20 transactions.

Exercise 9-2 Recording known current liabilities

C2

The payroll records of Speedy Software show the following information about Marsha Gottschalk, an employee, for the weekly pay period ending September 30. Gottschalk is single and claims one allowance. Compute her Social Security tax (6.2%), Medicare tax (1.45%), federal income tax withholding (use the withholding table in Exhibit 9A.6), state income tax (1.0%), and net pay for the current pay period. Round tax amounts to the nearest cent.

QS 9-14A Net pay and tax computations

P5

Check Net pay, $579.99

Total (gross) earnings for current pay period . . . . . . . . . . . . . . . . $ 740

Cumulative earnings of previous pay periods. . . . . . . . . . . . . . . . $9,700

The following items appear on the balance sheet of a company with a one-year operating cycle. Identify the proper classification of each item as follows: C if it is a current liability, L if it is a long-term liability, or N if it is not a liability.

1. Notes payable (due in 13 to 24 months). 2. Notes payable (due in 6 to 11 months). 3. Notes payable (mature in five years). 4. Current portion of long-term debt. 5. Notes payable (due in 120 days).

6. FUTA taxes payable. 7. Accounts receivable. 8. Sales taxes payable. 9. Salaries payable. 10. Wages payable.

EXERCISES

Exercise 9-1 Classifying liabilities

C1

Sylvestor Systems borrows $110,000 cash on May 15 by signing a 60-day, 12%, $110,000 note. 1. On what date does this note mature? 2. Prepare the entries to record (a) issuance of the note and (b) payment of the note at maturity.

Exercise 9-3 Accounting for note payable P1 Check (2b) Interest expense, $2,200

Keesha Co. borrows $200,000 cash on November 1 of the current year by signing a 90-day, 9%, $200,000 note. 1. On what date does this note mature? 2. How much interest expense is recorded in the current year? (Assume a 360-day year.) 3. How much interest expense is recorded in the following year? (Assume a 360-day year.) 4. Prepare journal entries to record (a) issuance of the note, (b) accrual of interest on December 31, and

(c) payment of the note at maturity.

Exercise 9-4 Interest-bearing notes payable with year-end adjustments P1 Check (2) $3,000 (3) $1,500

Park Company reports interest expense of $145,000 and income before interest expense and income taxes of $1,885,000. (1) Compute its times interest earned. (2) Park’s competitor’s times interest earned is 4.0. Is Park in a better or worse position than its competitor to make interest payments if the economy turns bad?

QS 9-12 Times interest earned

A1

Organic Farmers Co-Op has three employees and pays them weekly. Using the withholding bracket table in Exhibit 9A.6, determine each employee’s federal income tax withholding. 1. Maria earns $735 per week and claims three withholding allowances. 2. Jeff earns $607 per week and claims five withholding allowances. 3. Alicia earns $704 per week and does not claim any withholding allowances.

QS 9-13A Federal income tax withholdings

P5

Chapter 9 Accounting for Current Liabilities 367

Using the data in situation (a) of Exercise 9-5, prepare the employer’s September 30 journal entries to record salary expense and its related payroll liabilities for this employee. The employee’s federal income taxes withheld by the employer are $80 for this pay period. Round amounts to cents.

Exercise 9-6 Payroll-related journal entries P2

Using the data in situation (a) of Exercise 9-5, prepare the employer’s September 30 journal entries to record the employer’s payroll taxes expense and its related liabilities. Round amounts to cents.

Exercise 9-7 Payroll-related journal entries P3

The following monthly data are taken from Ramirez Company at July 31: sales salaries, $200,000; office salaries, $160,000; federal income taxes withheld, $90,000; state income taxes withheld, $20,000; Social Security taxes withheld, $22,320; Medicare taxes withheld, $5,220; medical insurance premiums, $7,000; life insurance premiums, $4,000; union dues deducted, $1,000; and salaries subject to unemployment taxes, $50,000. The employee pays 40% of medical and life insurance premiums. Prepare journal entries to record (1) accrued payroll, including employee deductions, for July; (2) cash payment of the net payroll (salaries payable) for July; (3) accrued employer payroll taxes, and other related employment expenses, for July—assume that FICA taxes are identical to those on employees and that SUTA taxes are 5.4% and FUTA taxes are 0.6%; and (4) cash payment of all liabilities related to the July payroll.

Exercise 9-8 Recording payroll

P2 P3

BMX Company has one employee. FICA Social Security taxes are 6.2% of the first $128,400 paid to its employee, and FICA Medicare taxes are 1.45% of gross pay. For BMX, its FUTA taxes are 0.6% and SUTA taxes are 5.4% of the first $7,000 paid to its employee. Compute BMX’s amounts for each of these four taxes as applied to the employee’s gross earnings for September under each of three separate situa- tions (a), (b), and (c). Round amounts to cents.

Exercise 9-5 Computing payroll taxes

P2 P3

Check (a) FUTA, $3.60; SUTA, $32.40

Gross Pay through August 31 Gross Pay for September

a. $ 6,400 $ 800 b. 2,000 2,100 c. 122,100 8,000

Mest Company has nine employees. FICA Social Security taxes are 6.2% of the first $128,400 paid to each employee, and FICA Medicare taxes are 1.45% of gross pay. FUTA taxes are 0.6% and SUTA taxes are 5.4% of the first $7,000 paid to each employee. Cumulative pay for the current year for each of its employees follows.

Exercise 9-9 Computing payroll taxes

P2 P3

Employee Cumulative Pay Employee Cumulative Pay Employee Cumulative Pay

Ken S. . . . . . . . . . . . $ 6,000 Michelle W. . . . . . . . $143,500 Lori K. . . . . . . . . . . . $130,900

Tim V.. . . . . . . . . . . . 40,400 Michael M. . . . . . . . 106,900 Kitty O.. . . . . . . . . . . 36,900

Steve S. . . . . . . . . . . 87,000 Zach R. . . . . . . . . . . 128,400 John W. . . . . . . . . . . 4,000

a. Prepare a table with the following six column headings. Compute the amounts in this table for each employee and then total the numerical columns.

Pay Subject to Pay Subject Pay Subject Pay Subject Cumulative FICA Social to FICA to FUTA to SUTA Employee Pay Security Medicare Taxes Taxes

b. For the company, compute each total for FICA Social Security taxes, FICA Medicare taxes, FUTA taxes, and SUTA taxes. Hint: Remember to include in those totals any employee share of taxes that the company must collect. Round amounts to cents.

Hitzu Co. sold a copier (that costs $4,800) for $6,000 cash with a two-year parts warranty to a customer on August 16 of Year 1. Hitzu expects warranty costs to be 4% of dollar sales. It records warranty expense with an adjusting entry on December 31. On January 5 of Year 2, the copier requires on-site repairs that are completed the same day. The repairs cost $209 for materials taken from the repair parts inventory. These are the only repairs required in Year 2 for this copier. 1. How much warranty expense does the company report for this copier in Year 1? 2. How much is the estimated warranty liability for this copier as of December 31 of Year 1? 3. How much is the estimated warranty liability for this copier as of December 31 of Year 2? 4. Prepare journal entries to record (a) the copier’s sale; (b) the adjustment to recognize the warranty

expense on December 31 of Year 1; and (c) the repairs that occur on January 5 of Year 2.

Exercise 9-10 Warranty expense and liability computations and entries

P4 Check (1) $240

(3) $31

368 Chapter 9 Accounting for Current Liabilities

Prepare adjusting entries at December 31 for Maxum Company’s year-end financial statements for each of the following separate transactions. 1. Employees earn vacation pay at a rate of one day per month. Maxum estimated and must expense

$13,000 of accrued vacation benefits for the year. 2. During December, Maxum Company sold 12,000 units of a product that carries a 60-day warranty.

December sales for this product total $460,000. The company expects 10% of the units to need war- ranty repairs, and it estimates the average repair cost per unit will be $15.

Exercise 9-12 Accounting for estimated liabilities

P4

Vander Co. provides medical care and insurance benefits to its retirees. In the current year, Vander agrees to pay $9,500 for medical insurance and contribute an additional 5% of the employees’ $200,000 gross salaries to a retirement program. (1) Record the entry for these accrued (but unpaid) benefits on December 31. (2) Assuming $5,000 of the retirement benefits are not to be paid for five years, how should this amount be reported on the current balance sheet?

Exercise 9-13 Accounting for health and pension benefits

P4

For each separate situation, indicate whether Cruz Company should (a) record a liability, (b) disclose in notes, or (c) have no disclosure. 1. Cruz Company guarantees the $100,000 debt of a supplier. It is not probable that the supplier will

default on the debt. 2. A disgruntled employee is suing Cruz Company. Legal advisers believe that the company will likely

need to pay damages, but the amount cannot be reasonably estimated.

Exercise 9-14 Accounting for contingent liabilities

C3

For the year ended December 31, Lopez Company implements an employee bonus program based on company net income, which the employees share equally. Lopez’s bonus expense is computed as $14,563. 1. Prepare the journal entry at December 31 to record the bonus due the employees. 2. Prepare the later journal entry at January 19 to record payment of the bonus to employees.

Exercise 9-11 Recording bonuses

P4

Use the following information from separate companies a through d to compute times interest earned. Which company indicates the strongest ability to pay interest expense as it comes due?

Net Income (Loss) Interest Expense Income Taxes

a. $119,000 $44,000 $35,000 b. 135,000 16,000 25,000 c. 138,000 12,000 30,000 d. 314,000 14,000 50,000

Exercise 9-16 Computing and interpreting times interest earned

A1

Check (b) 11.0

Selected accounts from Lue Co.’s adjusted trial balance for the year ended December 31 follow. Prepare a classified balance sheet.

Exercise 9-15 Preparing a balance sheet

C1 P2 P3 Total equity . . . . . . . . . . . . . . . . . . . . . . . . . . . $30,000 Employee federal income taxes payable . . . . . . . . $9,000

Equipment . . . . . . . . . . . . . . . . . . . . . . . . . . . 40,000 Federal unemployment taxes payable . . . . . . . . . . 200

Salaries payable . . . . . . . . . . . . . . . . . . . . . . 34,000 FICA—Medicare taxes payable . . . . . . . . . . . . . . . . 725

Accounts receivable . . . . . . . . . . . . . . . . . . . 5,100 FICA—Social Security taxes payable . . . . . . . . . . . . 3,100

Cash . . . . . . . . . . . . . . . . . . . . . . . . . . . . . . . . 50,000 Employee medical insurance payable . . . . . . . . . . 2,000

Current portion of long-term debt . . . . . . . . 4,000 State unemployment taxes payable . . . . . . . . . . . . 1,800

Notes payable (due in 6 years) . . . . . . . . . . . 10,000 Sales tax payable (due in 2 weeks) . . . . . . . . . . . . 275

Nishi Corporation prepares financial statements for each month-end. As part of its accounting process, es- timated income taxes are accrued each month for 30% of the current month’s net income. The income taxes are paid in the first month of each quarter for the amount accrued for the prior quarter. The following infor-

Exercise 9-17B Accounting for income taxes P4

Chapter 9 Accounting for Current Liabilities 369

Lenny Florita, an unmarried employee, works 48 hours in the week ended January 12. His pay rate is $14 per hour, and his wages have deductions for FICA Social Security, FICA Medicare, and federal income taxes. He claims two withholding allowances. Compute his regular pay, overtime pay (Lenny earns $21 per hour for each hour over 40 per week), and gross pay. Then compute his FICA tax deduction (6.2% for the Social Security portion and 1.45% for the Medicare portion), income tax deduction (use the wage bracket withholding table from Exhibit 9A.6), total deductions, and net pay. Round tax amounts to the nearest cent.

Exercise 9-18A Computing gross and net pay

P5

Check Net pay, $596.30

Stark Company has five employees. Employees paid by the hour earn $10 per hour for the regular 40-hour workweek and $15 per hour beyond the 40 hours per week. Hourly employees are paid every two weeks, but salaried employees are paid monthly on the last biweekly payday of each month. FICA Social Security taxes are 6.2% of the first $128,400 paid to each employee, and FICA Medicare taxes are 1.45% of gross pay. FUTA taxes are 0.6% and SUTA taxes are 5.4% of the first $7,000 paid to each employee. The com- pany has a benefits plan that includes medical insurance, life insurance, and retirement funding for em- ployees. Under this plan, employees must contribute 5% of their gross income as a payroll withholding, which the company matches with double the amount. Following is the partially completed payroll register for the biweekly period ending August 31, which is the last payday of August.

Exercise 9-19A Preparing payroll register and related entries

P5

Cumulative Pay (Excludes Current Period)

Current-Period Gross Pay FUTA

Pay Hours

Gross Pay

Pay Type SIT

FIT

SUTA

Kathleen

Anthony

Nichole

Zoey

Gracie

Totals

Employee

$126,600.00

6,800.00

15,100.00

6,500.00

5,000.00

$160,000.00

Regular Overtime

Regular Overtime

Regular Overtime

Salary

Salary

0

4 74

8

80

80

388.00

21.00

22.00

2,380.00

90.00

25.00

100.00

110.00

80.00 300.00

$2,000.00

20.00 25.00 50.00

0.00 740.00

500.00

$7,000.00

FICA-SS_EE FICA-Med_EE

FICA-SS_ER FICA-Med_ER

EE-Ben_Plan Withholding ER-Ben_Plan Expense

Employee Net Pay (Current Period)

Note: Table abbreviations follow those in Exhibit 9A.3; “Ben_Plan” refers to employee (EE) withholding or the employer (ER) expense for the benefits plan.

a. Complete this payroll register by filling in all cells for the pay period ended August 31. Hint: See Exhibit 9A.5 for guidance. Round amounts to cents.

b. Prepare the August 31 journal entry to record the accrued biweekly payroll and related liabilities for deductions.

c. Prepare the August 31 journal entry to record the employer’s cash payment of the net payroll of part b. d. Prepare the August 31 journal entry to record the employer’s payroll taxes including the contribution

to the benefits plan. e. Prepare the August 31 journal entry to pay all liabilities (except for the net payroll in part c) for this

biweekly period.

October net income . . . . . . $28,600 November net income. . . . . $19,100 December net income . . . . . $34,600

1. Determine the amount of the accounting adjustment (dated as of December 31) to get the correct end- ing balance in the Income Taxes Payable account.

2. Prepare journal entries to record (a) the December 31 adjustment to the Income Taxes Payable account and (b) the later January 20 payment of the fourth-quarter taxes.

Check (1) $3,610

mation is available for the fourth quarter of the year just ended. When tax computations are completed on January 20 of the following year, Nishi determines that the quarter’s Income Taxes Payable account balance should be $28,300 on December 31 of the year just ended (its unadjusted balance is $24,690).

370 Chapter 9 Accounting for Current Liabilities

On January 8, the end of the first weekly pay period of the year, Regis Company’s employees earned $22,760 of office salaries and $65,840 of sales salaries. Withholdings from the employees’ salaries include FICA Social Security taxes at the rate of 6.2%, FICA Medicare taxes at the rate of 1.45%, $12,860 of federal income taxes, $1,340 of medical insurance deductions, and $840 of union dues. No employee earned more than $7,000 in this first period.

Required

1. Calculate FICA Social Security taxes payable and FICA Medicare taxes payable. Prepare the journal entry to record Regis Company’s January 8 employee payroll expenses and liabilities. Round amounts to cents.

2. Prepare the journal entry to record Regis’s employer payroll taxes resulting from the January 8 payroll. Regis’s state unemployment tax rate is 5.4% of the first $7,000 paid to each employee. The federal unemployment tax rate is 0.6%. Round amounts to cents.

Check (1) Cr. Salaries Payable, $66,782.10

(2) Dr. Payroll Taxes Expense, $12,093.90

Problem 9-2A Entries for payroll transactions

P2 P3

Tyrell Co. entered into the following transactions involving short-term liabilities.

Year 1

Apr. 20 Purchased $40,250 of merchandise on credit from Locust, terms n∕30. May 19 Replaced the April 20 account payable to Locust with a 90-day, 10%, $35,000 note payable

along with paying $5,250 in cash. July 8 Borrowed $80,000 cash from NBR Bank by signing a 120-day, 9%, $80,000 note payable. ___?____ Paid the amount due on the note to Locust at the maturity date. ___?____ Paid the amount due on the note to NBR Bank at the maturity date. Nov. 28 Borrowed $42,000 cash from Fargo Bank by signing a 60-day, 8%, $42,000 note payable. Dec. 31 Recorded an adjusting entry for accrued interest on the note to Fargo Bank.

Year 2

___?____ Paid the amount due on the note to Fargo Bank at the maturity date.

Required

1. Determine the maturity date for each of the three notes described. 2. Determine the interest due at maturity for each of the three notes. Assume a 360-day year. 3. Determine the interest expense recorded in the adjusting entry at the end of Year 1. 4. Determine the interest expense recorded in Year 2. 5. Prepare journal entries for all the preceding transactions and events.

Check (2) Locust, $875

(3) $308

(4) $252

PROBLEM SET A

Problem 9-1A Short-term notes payable transactions and entries

P1

In addition to gross pay, the company must pay two-thirds of the $60 per employee weekly health insur- ance; each employee pays the remaining one-third. The company also contributes an extra 8% of each employee’s gross pay (at no cost to employees) to a pension fund.

Required

Compute the following for the week ended August 25 (round amounts to the nearest cent): 1. Each employee’s FICA withholdings for Social Security. 2. Each employee’s FICA withholdings for Medicare. 3. Employer’s FICA taxes for Social Security.

Paloma Co. has four employees. FICA Social Security taxes are 6.2% of the first $128,400 paid to each employee, and FICA Medicare taxes are 1.45% of gross pay. Also, for the first $7,000 paid to each employee, the company’s FUTA taxes are 0.6% and SUTA taxes are 5.4%. The company is preparing its payroll calculations for the week ended August 25. Payroll records show the following information for the company’s four employees.

Problem 9-3A Payroll expenses, withholdings, and taxes

P2 P3

1

2

3

4

5

6

Current Week

30

$284 145 39

D

Income Tax Withholding Dali Trey Kiesha Chee

A

Name $127,300 127,500

6,900 1,250

B

Gross Pay through Aug. 18

C

Gross Pay $2,000

900 450 400

Check (3) $176.70

Chapter 9 Accounting for Current Liabilities 371

4. Employer’s FICA taxes for Medicare. 5. Employer’s FUTA taxes. 6. Employer’s SUTA taxes. 7. Each employee’s net (take-home) pay. 8. Employer’s total payroll-related expense for each employee.

(4) $54.38

(5) $3.00

(7) Total net pay, $2,940.92

On October 29, Lobo Co. began operations by purchasing razors for resale. The razors have a 90-day war- ranty. When a razor is returned, the company discards it and mails a new one from merchandise inventory to the customer. The company’s cost per new razor is $20 and its retail selling price is $75. The company expects warranty costs to equal 8% of dollar sales. The following transactions occurred.

Nov. 11 Sold 105 razors for $7,875 cash. 30 Recognized warranty expense related to November sales with an adjusting entry. Dec. 9 Replaced 15 razors that were returned under the warranty. 16 Sold 220 razors for $16,500 cash. 29 Replaced 30 razors that were returned under the warranty. 31 Recognized warranty expense related to December sales with an adjusting entry.

Jan. 5 Sold 150 razors for $11,250 cash. 17 Replaced 50 razors that were returned under the warranty. 31 Recognized warranty expense related to January sales with an adjusting entry.

Required

1. Prepare journal entries to record these transactions and adjustments. 2. How much warranty expense is reported for November and for December? 3. How much warranty expense is reported for January? 4. What is the balance of the Estimated Warranty Liability account as of December 31? 5. What is the balance of the Estimated Warranty Liability account as of January 31?

Check (3) $900

(4) $1,050 Cr.

(5) $950 Cr.

Problem 9-4A Estimating warranty expense and liability

P4

Shown here are condensed income statements for two different companies (assume no income taxes).

Required

1. Compute times interest earned for Miller Company and for Weaver Company. 2. What happens to each company’s net income if sales increase by 30%? 3. What happens to each company’s net income if sales increase by 50%? 4. What happens to each company’s net income if sales decrease by 10%? 5. What happens to each company’s net income if sales decrease by 40%?

Analysis Component

6. Which company would have a greater ability to pay interest expense if sales were to decrease?

Check (2) Miller net income, $200,000 (43% increase)

(4) Weaver net income, $100,000 (29% decrease)

Miller Company

Sales . . . . . . . . . . . . . . . . . . . . . . . . . . . $1,000,000

Variable expenses (80%) . . . . . . . . . . . 800,000

Income before interest . . . . . . . . . . . . . 200,000

Interest expense (fixed) . . . . . . . . . . . . 60,000

Net income . . . . . . . . . . . . . . . . . . . . . . $ 140,000

Weaver Company

Sales . . . . . . . . . . . . . . . . . . . . . . . . . . . $1,000,000

Variable expenses (60%) . . . . . . . . . . . 600,000

Income before interest . . . . . . . . . . . . . 400,000

Interest expense (fixed) . . . . . . . . . . . . 260,000

Net income . . . . . . . . . . . . . . . . . . . . . . $ 140,000

Problem 9-5A Computing and analyzing times interest earned

A1

Francisco Company has 10 employees, each of whom earns $2,800 per month and is paid on the last day of each month. All 10 have been employed continuously at this amount since January 1. On March 1, the following accounts and balances exist in its general ledger. a. FICA—Social Security Taxes Payable, $3,472; FICA—Medicare Taxes Payable, $812. (The balances

of these accounts represent total liabilities for both the employer’s and employees’ FICA taxes for the February payroll only.)

b. Employees’ Federal Income Taxes Payable, $4,000 (liability for February only). c. Federal Unemployment Taxes Payable, $336 (liability for January and February together). d. State Unemployment Taxes Payable, $3,024 (liability for January and February together).

Problem 9-6AA Entries for payroll transactions

P5

[continued on next page]

372 Chapter 9 Accounting for Current Liabilities

The company had the following payroll transactions.

Mar. 15 Issued check payable to Swift Bank, a federal depository bank authorized to accept employers’ payments of FICA taxes and employee income tax withholdings. The $8,284 check is in pay- ment of the February FICA and employee income taxes.

31 Recorded the journal entry for the March salaries payable. Then recorded the cash payment of the March payroll (the company issued checks payable to each employee in payment of the March payroll). The payroll register shows the following summary totals for the March pay period.

Salaries Federal

Office Shop Gross FICA Income Net Salaries Salaries Pay Taxes* Taxes Pay

$11,200 $16,800 $28,000 $1,736 $4,000 $21,858

$ 406

*FICA taxes are Social Security and Medicare, respectively.

31 Recorded the employer’s payroll taxes resulting from the March payroll. The company has a state unemployment tax rate of 5.4% on the first $7,000 paid to each employee. The federal rate is 0.6%.

Apr. 15 Issued check to Swift Bank in payment of the March FICA and employee income taxes. 15 Issued check to the State Tax Commission for the January, February, and March state unem-

ployment taxes. Filed the check and the first-quarter tax return with the Commission. 30 Issued check payable to Swift Bank in payment of the employer’s FUTA taxes for the first quar-

ter of the year. 30 Filed Form 941 with the IRS, reporting the FICA taxes and the employees’ federal income tax

withholdings for the first quarter.

Required

Prepare journal entries to record these transactions and events.

March 31: Dr. Payroll Taxes Expense, $2,982

April 15: Cr. Cash, $8,284 (Swift Bank)

Check March 31: Salaries Payable, $21,858

PROBLEM SET B

Problem 9-1B Short-term notes payable transactions and entries

P1

Warner Co. entered into the following transactions involving short-term liabilities.

Year 1

Apr. 22 Purchased $5,000 of merchandise on credit from Fox-Pro, terms n∕30. May 23 Replaced the April 22 account payable to Fox-Pro with a 60-day, 15% $4,600 note payable

along with paying $400 in cash. July 15 Borrowed $12,000 cash from Spring Bank by signing a 120-day, 10%, $12,000 note payable. ___?____ Paid the amount due on the note to Fox-Pro at maturity. ___?____ Paid the amount due on the note to Spring Bank at maturity. Dec. 6 Borrowed $8,000 cash from City Bank by signing a 45-day, 9%, $8,000 note payable. 31 Recorded an adjusting entry for accrued interest on the note to City Bank.

Year 2

___?____ Paid the amount due on the note to City Bank at maturity.

Required

1. Determine the maturity date for each of the three notes described. 2. Determine the interest due at maturity for each of the three notes. Assume a 360-day year. 3. Determine the interest expense recorded in the adjusting entry at the end of Year 1. 4. Determine the interest expense recorded in Year 2. 5. Prepare journal entries for all the preceding transactions and events.

Check (2) Fox-Pro, $115

(3) $50

(4) $40

Tavella Company’s first weekly pay period of the year ends on January 8. On that date, Tavella’s sales employees earned $34,745, office employees earned $21,225, and delivery employees earned $1,030 in salaries. The employees are to have withheld from their salaries FICA Social Security taxes at the rate of 6.2%, FICA Medicare taxes at the rate of 1.45%, $8,625 of federal income taxes, $1,160 of medical insur- ance deductions, and $138 of union dues. No employee earned more than $7,000 in the first pay period.

Problem 9-2B Entries for payroll transactions

P2 P3

Chapter 9 Accounting for Current Liabilities 373

Required

1. Calculate FICA Social Security taxes payable and FICA Medicare taxes payable. Prepare the journal entry to record Tavella Company’s January 8 employee payroll expenses and liabilities. Round amounts to cents.

2. Prepare the journal entry to record Tavella’s employer payroll taxes resulting from the January 8 pay- roll. Tavella’s state unemployment tax rate is 5.4% of the first $7,000 paid to each employee. The federal unemployment tax rate is 0.6%. Round amounts to cents.

Check (1) Cr. Salaries Payable, $42,716.50

(2) Dr. Payroll Taxes Expense, $7,780.50

Fishing Guides Co. has four employees. FICA Social Security taxes are 6.2% of the first $128,400 paid to each employee, and FICA Medicare taxes are 1.45% of gross pay. Also, for the first $7,000 paid to each employee, the company’s FUTA taxes are 0.6% and SUTA taxes are 5.4%. The company is preparing its payroll calculations for the week ended September 30. Payroll records show the following information for the company’s four employees.

Problem 9-3B Payroll expenses, withholdings, and taxes

P2 P3

Ahmed Carlos Jun Marie

$126,800 126,885

6,650 23,700 68

$198 182 32

1

2

3

4

5

6

BA D

Current Week Income Tax WithholdingName

Gross Pay through Sep. 23

C

Gross Pay $2,500

1,515 475

1,000

In addition to gross pay, the company must pay 60% of the $50 per employee weekly health insurance; each employee pays the remaining 40%. The company also contributes an extra 5% of each employee’s gross pay (at no cost to employees) to a pension fund.

Required

Compute the following for the week ended September 30 (round amounts to the nearest cent): 1. Each employee’s FICA withholdings for Social Security. 2. Each employee’s FICA withholdings for Medicare. 3. Employer’s FICA taxes for Social Security. 4. Employer’s FICA taxes for Medicare. 5. Employer’s FUTA taxes. 6. Employer’s SUTA taxes. 7. Each employee’s net (take-home) pay. 8. Employer’s total payroll-related expense for each employee.

Check (3) $284.58

(4) $79.61

(5) $2.10

(7) Total net pay, $4,565.81

On November 10, Lee Co. began operations by purchasing coffee grinders for resale. The grinders have a 60-day warranty. When a grinder is returned, the company discards it and mails a new one from merchan- dise inventory to the customer. The company’s cost per new grinder is $24 and its retail selling price is $50. The company expects warranty costs to equal 10% of dollar sales. The following transactions occurred.

Nov. 16 Sold 50 grinders for $2,500 cash. 30 Recognized warranty expense related to November sales with an adjusting entry. Dec. 12 Replaced six grinders that were returned under the warranty. 18 Sold 200 grinders for $10,000 cash. 28 Replaced 17 grinders that were returned under the warranty. 31 Recognized warranty expense related to December sales with an adjusting entry.

Jan. 7 Sold 40 grinders for $2,000 cash. 21 Replaced 36 grinders that were returned under the warranty. 31 Recognized warranty expense related to January sales with an adjusting entry.

Required

1. Prepare journal entries to record these transactions and adjustments. 2. How much warranty expense is reported for November and for December? 3. How much warranty expense is reported for January? 4. What is the balance of the Estimated Warranty Liability account as of December 31? 5. What is the balance of the Estimated Warranty Liability account as of January 31?

Problem 9-4B Estimating warranty expense and liability

P4

Check (3) $200

(4) $698 Cr.

(5) $34 Cr.

374 Chapter 9 Accounting for Current Liabilities

Shown here are condensed income statements for two different companies (assume no income taxes).

Ellis Company

Sales . . . . . . . . . . . . . . . . . . . . . . . . . . . $240,000

Variable expenses (50%) . . . . . . . . . . . 120,000

Income before interest . . . . . . . . . . . . . 120,000

Interest expense (fixed) . . . . . . . . . . . . 90,000

Net income . . . . . . . . . . . . . . . . . . . . . . $ 30,000

Seidel Company

Sales . . . . . . . . . . . . . . . . . . . . . . . . . . . $240,000

Variable expenses (75%) . . . . . . . . . . . 180,000

Income before interest . . . . . . . . . . . . . 60,000

Interest expense (fixed) . . . . . . . . . . . . 30,000

Net income . . . . . . . . . . . . . . . . . . . . . . $ 30,000

Problem 9-5B Computing and analyzing times interest earned

A1

Required

1. Compute times interest earned for Ellis Company and for Seidel Company. 2. What happens to each company’s net income if sales increase by 10%? 3. What happens to each company’s net income if sales increase by 40%? 4. What happens to each company’s net income if sales decrease by 20%? 5. What happens to each company’s net income if sales decrease by 50%?

Analysis Component

6. Which company would have a greater ability to pay interest expense if sales were to decrease?

Check (3) Ellis net income, $78,000 (160% increase) (4) Seidel net income, $18,000 (40% decrease)

MLS Company has five employees, each of whom earns $1,600 per month and is paid on the last day of each month. All five have been employed continuously at this amount since January 1. On June 1, the fol- lowing accounts and balances exist in its general ledger. a. FICA—Social Security Taxes Payable, $992; FICA—Medicare Taxes Payable, $232. (The balances of

these accounts represent total liabilities for both the employer’s and employees’ FICA taxes for the May payroll only.)

b. Employees’ Federal Income Taxes Payable, $1,050 (liability for May only). c. Federal Unemployment Taxes Payable, $66 (liability for April and May together). d. State Unemployment Taxes Payable, $594 (liability for April and May together). The company had the following payroll transactions.

June 15 Issued check payable to Security Bank, a federal depository bank authorized to accept employ- ers’ payments of FICA taxes and employee income tax withholdings. The $2,274 check is in payment of the May FICA and employee income taxes.

30 Recorded the journal entry for the June salaries payable. Then recorded the cash payment of the June payroll (the company issued checks payable to each employee in payment of the June pay- roll). The payroll register shows the following summary totals for the June pay period.

Problem 9-6BA Entries for payroll transactions

P5

Check June 30: Cr. Salaries Payable, $6,338

Salaries Federal

Office Shop Gross FICA Income Net Salaries Salaries Pay Taxes* Taxes Pay

$3,800 $4,200 $8,000 $496 $1,050 $6,338

$116

*FICA taxes are Social Security and Medicare, respectively.

30 Recorded the employer’s payroll taxes resulting from the June payroll. The company has a state unemployment tax rate of 5.4% on the first $7,000 paid to each employee. The federal rate is 0.6%.

July 15 Issued check payable to Security Bank in payment of the June FICA and employee income taxes. 15 Issued check to the State Tax Commission for the April, May, and June state unemployment

taxes. Filed the check and the second-quarter tax return with the State Tax Commission. 31 Issued check payable to Security Bank in payment of the employer’s FUTA taxes for the first

quarter of the year. 31 Filed Form 941 with the IRS, reporting the FICA taxes and the employees’ federal income tax

withholdings for the second quarter.

Required

Prepare journal entries to record the transactions and events.

Check June 30: Dr. Payroll Taxes Expense, $612

July 15: Cr. Cash, $2,274 (Security Bank)

Chapter 9 Accounting for Current Liabilities 375

SERIAL PROBLEM Business Solutions

C2 P2 P3

This serial problem began in Chapter 1 and continues through most of the book. If previous chapter seg- ments were not completed, the serial problem can begin at this point.

SP 9 Review the February 26 and March 25 transactions for Business Solutions (SP 4) from Chapter 4.

Feb. 26 The company paid cash to Lyn Addie for eight days’ work at $125 per day. Mar. 25 The company sold merchandise with a $2,002 cost for $2,800 on credit to Wildcat Services,

invoice dated March 25.

Required

1. Assume that Lyn Addie is an unmarried employee. Her $1,000 of wages have deductions for FICA Social Security taxes, FICA Medicare taxes, and federal income taxes. Her federal income taxes for this pay period total $159. Compute her net pay for the eight days’ work paid on February 26. Round amounts to the nearest cent.

2. Record the journal entry to reflect the payroll payment to Lyn Addie as computed in part 1. 3. Record the journal entry to reflect the (employer) payroll tax expenses for the February 26 payroll

payment. Assume Lyn Addie has not met earnings limits for FUTA and SUTA (the FUTA rate is 0.6% and the SUTA rate is 5.4% for the company). Round amounts to the nearest cent.

4. Record the entry(ies) for the merchandise sold on March 25 if a 4% sales tax rate applies. ©Alexander Image/Shutterstock

COMPREHENSIVE PROBLEM

Bug-Off Exterminators (Review of Chapters 1–9)

CP 9 Bug-Off Exterminators provides pest control services and sells extermination products manufac- tured by other companies. The following six-column table contains the company’s unadjusted trial bal- ance as of December 31, 2019.

Unadjusted Adjusted December 31, 2019 Trial Balance Adjustments Trial Balance

Cash . . . . . . . . . . . . . . . . . . . . . . . . . . . . . . . . . . $ 17,000 Accounts receivable . . . . . . . . . . . . . . . . . . . . . 4,000 Allowance for doubtful accounts . . . . . . . . . . . $ 828 Merchandise inventory . . . . . . . . . . . . . . . . . . . 11,700 Trucks. . . . . . . . . . . . . . . . . . . . . . . . . . . . . . . . . 32,000 Accum. depreciation—Trucks . . . . . . . . . . . . . . 0 Equipment . . . . . . . . . . . . . . . . . . . . . . . . . . . . . 45,000 Accum. depreciation—Equipment . . . . . . . . . . 12,200 Accounts payable . . . . . . . . . . . . . . . . . . . . . . . 5,000 Estimated warranty liability. . . . . . . . . . . . . . . . 1,400 Unearned services revenue . . . . . . . . . . . . . . . 0 Interest payable. . . . . . . . . . . . . . . . . . . . . . . . . 0 Long-term notes payable . . . . . . . . . . . . . . . . . 15,000 Common stock . . . . . . . . . . . . . . . . . . . . . . . . . 10,000 Retained earnings . . . . . . . . . . . . . . . . . . . . . . . 49,700 Dividends . . . . . . . . . . . . . . . . . . . . . . . . . . . . . . 10,000 Extermination services revenue . . . . . . . . . . . . 60,000 Interest revenue . . . . . . . . . . . . . . . . . . . . . . . . 872 Sales (of merchandise) . . . . . . . . . . . . . . . . . . . 71,026 Cost of goods sold. . . . . . . . . . . . . . . . . . . . . . . 46,300 Depreciation expense—Trucks . . . . . . . . . . . . . 0 Depreciation expense—Equipment . . . . . . . . . 0 Wages expense . . . . . . . . . . . . . . . . . . . . . . . . . 35,000 Interest expense . . . . . . . . . . . . . . . . . . . . . . . . 0 Rent expense. . . . . . . . . . . . . . . . . . . . . . . . . . . 9,000 Bad debts expense . . . . . . . . . . . . . . . . . . . . . . 0 Miscellaneous expense. . . . . . . . . . . . . . . . . . . 1,226 Repairs expense . . . . . . . . . . . . . . . . . . . . . . . . 8,000 Utilities expense . . . . . . . . . . . . . . . . . . . . . . . . 6,800 Warranty expense . . . . . . . . . . . . . . . . . . . . . . . 0 Totals . . . . . . . . . . . . . . . . . . . . . . . . . . . . . . . . . $226,026 $226,026

376 Chapter 9 Accounting for Current Liabilities

The following information in a through h applies to the company at the end of the current year. a. The bank reconciliation as of December 31, 2019, includes the following facts.

Reported on the bank statement is a canceled check that the company failed to record. (Information from the bank reconciliation allows you to determine the amount of this check, which is a payment on an account payable.)

b. An examination of customers’ accounts shows that accounts totaling $679 should be written off as uncollectible. Using an aging of receivables, the company determines that the ending balance of the Allowance for Doubtful Accounts should be $700.

c. A truck is purchased and placed in service on January 1, 2019. Its cost is being depreciated with the straight-line method using the following facts and estimates.

d. Two items of equipment (a sprayer and an injector) were purchased and put into service in early January 2017. They are being depreciated with the straight-line method using these facts and estimates.

Sprayer Injector

Original cost . . . . . . . . . . . . . . . . . . . . . . $27,000 $18,000

Expected salvage value . . . . . . . . . . . . . $ 3,000 $ 2,500

Useful life (years) . . . . . . . . . . . . . . . . . . 8 5

e. On August 1, 2019, the company is paid $3,840 cash in advance to provide monthly service for an apartment complex for one year. The company began providing the services in August. When the cash was received, the full amount was credited to the Extermination Services Revenue account.

f. The company offers a warranty for the services it sells. The expected cost of providing warranty ser- vice is 2.5% of the extermination services revenue of $57,760 for 2019. No warranty expense has been recorded for 2019. All costs of servicing warranties in 2019 were properly debited to the Estimated Warranty Liability account.

g. The $15,000 long-term note is an 8%, five-year, interest-bearing note with interest payable annually on December 31. The note was signed with First National Bank on December 31, 2019.

h. The ending inventory of merchandise is counted and determined to have a cost of $11,700. Bug-Off uses a perpetual inventory system.

Required

1. Use the preceding information to determine amounts for the following items. a. Correct (reconciled) ending balance of Cash; and the amount of the omitted check. b. Adjustment needed to obtain the correct ending balance of the Allowance for Doubtful Accounts. c. Depreciation expense for the truck used during year 2019. d. Depreciation expense for the two items of equipment used during year 2019. e. The adjusted 2019 ending balances of the Extermination Services Revenue and Unearned Services

Revenue accounts. f. The adjusted 2019 ending balances of the Warranty Expense and the Estimated Warranty Liability

accounts. g. The adjusted 2019 ending balances of the Interest Expense and the Interest Payable accounts.

(Round amounts to nearest whole dollar.) 2. Use the results of part 1 to complete the six-column table by first entering the appropriate adjustments

for items a through g and then completing the Adjusted Trial Balance columns. Hint: Item b requires two adjustments.

3. Prepare journal entries to record the adjustments entered on the six-column table. Assume Bug-Off’s adjusted balance for Merchandise Inventory matches the year-end physical count.

4. Prepare a single-step income statement, a statement of retained earnings (cash dividends during 2019 were $10,000), and a classified balance sheet.

Check (1a) Reconciled cash bal. $15,750 (1b) $551 credit

Cash balance per bank . . . . . . . . . . . . . . $15,100 Deposit in transit . . . . . . . . . . . . . . . . . . . . . . . . . . . . . . . . $2,450

Cash balance per books. . . . . . . . . . . . . . 17,000 Interest earned (on bank account) . . . . . . . . . . . . . . . . . . 52

Outstanding checks . . . . . . . . . . . . . . . . . 1,800 Bank service charges (miscellaneous expense) . . . . . . . 15

Original cost. . . . . . . . $32,000 Expected salvage value . . . . . . . . . $8,000 Useful life (years) . . . . . . . . 4

(2) Adjusted trial balance totals, $238,207

(4) Net income, $9,274; Total assets, $82,771

(1f ) Estimated Warranty Liability, $2,844 Cr.

Chapter 9 Accounting for Current Liabilities 377

GENERAL LEDGER PROBLEM

GL 9-1 General Ledger assignment GL 9-1, based on Problem 9-1A, focuses on transactions related to accounts and notes payable and highlights the impact each transaction has on interest expense, if any. Prepare the journal entries related to accounts and notes payable; the schedules for accounts payable and notes payable are automatically completed using the General Ledger tool. Compute both the amount and timing of interest expense for each note. Prepare the subsequent-period journal entries related to accrued interest.

GL

COMPANY ANALYSIS A1 P4

Accounting Analysis

AA 9-1 Use the table below and Apple’s financial statements in Appendix A to answer the following.

$ millions 2017 2016 2015

Interest expense. . . . . . . . . . $2,323 $1,456 $733

1. Compute times interest earned for each of the three years shown. 2. Is Apple in a good or bad position to pay interest obligations? Assume an industry average of 10. 3. Identify Apple’s total accrued expenses in 2017.

APPLE

Required

1. Compute times interest earned for the three years’ data shown for each company. 2. In the current year, and using times interest earned, which company appears better able to pay interest

obligations? 3. In the current year, and using times interest earned, is the company in a good or bad position to pay

interest obligations for (a) Apple and (b) Google? Assume an industry average of 10.

AA 9-2 Key figures for Apple and Google follow. COMPARATIVE ANALYSIS A1 Apple Google

Current One Year Two Years Current One Year Two Years $ millions Year Prior Prior Year Prior Prior

Net income . . . . . . . . . . . . . $48,351 $45,687 $53,394 $12,662 $19,478 $16,348

Income taxes. . . . . . . . . . . . 15,738 15,685 19,121 14,531 4,672 3,303

Interest expense. . . . . . . . . 2,323 1,456 733 109 124 104

APPLE GOOGLE

AA 9-3 Comparative figures for Samsung, Apple, and Google follow.

Samsung Apple Google

Current Prior Current Prior Current Prior In millions Year Year Year Year Year Year

Net income. . . . . . . . . . . . . . . . . . . . . . W42,186,747 W22,726,092 $48,351 $45,687 $12,662 $19,478

Income taxes . . . . . . . . . . . . . . . . . . . . 14,009,220 7,987,560 15,738 15,685 14,531 4,672

Interest expense . . . . . . . . . . . . . . . . . 655,402 587,831 2,323 1,456 109 124

GLOBAL ANALYSIS A1

Required

1. Compute the times interest earned ratio for the most recent two years for Samsung using the data shown.

2. Is the change in Samsung’s times interest earned ratio favorable or unfavorable? 3. In the current year, is Samsung’s times interest earned ratio better or worse than the same ratio for

(a) Apple and (b) Google?

Samsung APPLE GOOGLE

378 Chapter 9 Accounting for Current Liabilities

ETHICS CHALLENGE P4

BTN 9-1 Cameron Bly is a sales manager for an automobile dealership. He earns a bonus each year based on revenue from the number of autos sold in the year less related warranty expenses. Actual warranty expenses have varied over the prior 10 years from a low of 3% of an automobile’s selling price to a high of 10%. In the past, Bly has tended to estimate warranty expenses on the high end to be conservative. He must work with the dealership’s accountant at year-end to arrive at the warranty expense accrual for cars sold each year. 1. Does the warranty accrual decision create any ethical dilemma for Bly? 2. Because warranty expenses vary, what percent do you think Bly should choose for the current year?

Justify your response.

Beyond the Numbers

BTN 9-2 Dusty Johnson is the accounting and finance manager for a manufacturer. At year-end, he must determine how to account for the company’s contingencies. His manager, Tom Pretti, objects to Johnson’s proposal to recognize an expense and a liability for warranty service on units of a new product introduced in the fourth quarter. Pretti comments, “There’s no way we can estimate this warranty cost. We don’t owe anyone anything until a product fails and it is returned. Let’s report an expense if and when we do any warranty work.”

Required

Prepare a one-page memorandum for Johnson to send to Pretti defending his proposal.

COMMUNICATING IN PRACTICE C3

BTN 9-3 Access the March 1, 2017, filing of the December 31, 2016, annual 10-K report of McDonald’s Corporation (ticker: MCD), which is available from SEC.gov.

Required

1. Identify the current liabilities on McDonald’s balance sheet as of December 31, 2016. 2. Use the consolidated statement of income for the year ended December 31, 2016, to compute

McDonald’s times interest earned ratio. Comment on the result. Assume an industry average of 5.0.

TAKING IT TO THE NET C1 A1

BTN 9-4 Assume that your team is in business and you must borrow $6,000 cash for short-term needs. You have been shopping banks for a loan, and you have the following two options. A. Sign a $6,000, 90-day, 10% interest-bearing note dated June 1. B. Sign a $6,000, 120-day, 8% interest-bearing note dated June 1.

Required

1. Discuss these two options and determine the better choice. Ensure that all teammates concur with the decision and understand the rationale.

2. Each member of the team is to prepare one of the following journal entries. a. Option A—at date of issuance. b. Option B—at date of issuance. c. Option A—at maturity date. d. Option B—at maturity date. 3. In rotation, each member is to explain to the team the entry he or she prepared in part 2. Ensure that all

team members concur with and understand the entries. 4. Assume that the funds are borrowed on December 1 (instead of June 1) and your business operates on

a calendar-year reporting period. Each member of the team is to prepare one of the following entries. a. Option A—the year-end adjustment. b. Option B—the year-end adjustment. c. Option A—at maturity date. d. Option B—at maturity date. 5. In rotation, each member is to explain to the team the entry he or she prepared in part 4. Ensure that all

team members concur with and understand the entries.

TEAMWORK IN ACTION C2 P1

Chapter 9 Accounting for Current Liabilities 379

BTN 9-5 Review the chapter’s opening feature about Tim Westergren and the business he founded, Pandora. Assume that he is considering expanding the business to Europe and that the current abbrevi- ated income statement appears as follows.

PANDORA Income Statement

For Year Ended December 31

Sales . . . . . . . . . . . . . . . . . . . . . . . . . . . . . $1,000,000

Operating expenses (55%) . . . . . . . . . . . 550,000

Net income . . . . . . . . . . . . . . . . . . . . . . . . $ 450,000

Assume also that the company currently has no interest-bearing debt. If it expands to Europe, it will re- quire a $300,000 loan. The company has found a bank that will loan it the money on a 7% note payable. The company believes that, at least for the first few years, sales in Europe will equal $250,000 and that all expenses at both locations will continue to equal 55% of sales.

Required

1. Prepare an income statement (showing three separate columns for current operations, European, and total) for the company assuming that it borrows the funds and expands to Europe. Annual revenues for current operations are expected to remain at $1,000,000.

2. Compute the company’s times interest earned under the expansion assumptions in part 1. 3. Assume sales in Europe are $400,000. Prepare an income statement (with columns for current opera-

tions, European, and total) for the company and compute times interest earned. 4. Assume sales in Europe are $100,000. Prepare an income statement (with columns for current opera-

tions, European, and total) for the company and compute times interest earned. 5. Comment on your results from parts 1 through 4.

ENTREPRENEURIAL DECISION A1

BTN 9-6 Check the Social Security Administration website (SSA.gov) to locate the Social Security office near you. Visit the office to request a personal earnings and estimate form. Fill out the form and mail according to the instructions. You will receive a statement from the Social Security Administration regard- ing your earnings history and future Social Security benefits you can receive. (Formerly the request could be made online. The online service has been discontinued and is now under review by the Social Security Administration due to security concerns; however, it might once again be available online.) It is good to request an earnings and benefit statement every 5 to 10 years to make sure you have received credit for all wages earned and for which you and your employer have paid taxes into the system.

HITTING THE ROAD P2

Design elements: Lightbulb: ©Chuhail/Getty Images; Blue globe: ©nidwlw/Getty Images and ©Dizzle52/Getty Images; Chess piece: ©Andrei Simonenko/Getty Images and ©Dizzle52/Getty Images; Mouse: ©Siede Preis/Getty Images; Global View globe: ©McGraw-Hill Education and ©Dizzle52/Getty Images; Sustainability: ©McGraw-Hill Education and ©Dizzle52/Getty Images

Learning Objectives

CONCEPTUAL C1 Explain the types of notes and prepare

entries to account for notes.

C2 Appendix 10A—Explain and compute bond pricing.

C3 Appendix 10C—Describe accounting for leases and pensions.

ANALYTICAL A1 Compare bond financing with stock

financing.

P3 Record issuance and amortization of premium bonds using the straight-line method.

P4 Record the retirement of bonds.

P5 Appendix 10B—Compute and record amortization of a bond discount using the effective interest method.

P6 Appendix 10B—Compute and record amortization of a bond premium using the effective interest method.

A2 Assess debt features and their implications.

A3 Compute the debt-to-equity ratio and explain its use.

PROCEDURAL P1 Record issuance and interest expense

for par bonds.

P2 Record issuance and amortization of discount bonds using the straight-line method.

Chapter Preview

10 Accounting for Long-Term Liabilities

NTK 10-4

LONG-TERM NOTES

C1 Recording notes

DEBT ANALYSIS

A2 Debt features A3 Debt-to-equity

NTK 10-3

PREMIUM BONDS

P3 Bond payments Amortize premium

Straight-line

P4 Bond retirement

NTK 10-1

BOND BASICS

A1 Bond financing Bond trading

P1 Par bonds

NTK 10-2

DISCOUNT BONDS

Discount or premium

P2 Bond payments Amortize discount

Straight-line

381

“Believe in your product”—Scott Borba

At Face Value

OAKLAND, CA—Joey Shamah, a college student, met Scott Borba at a party. The two men talked at length, but it was not your typical “party” talk. Instead, they discussed the women’s cosmetics market!

Scott explains that he saw “all these women with Louis Vuitton purses . . . buying truckloads of lip balms and nail polishes” from 99 cent stores. “There’s a major market here,” insists Scott.

Joey and Scott agreed to work together to fill this market void by forming e.l.f. Cosmetics (elfCosmetics.com). “We felt women shouldn’t have to skip lunch or not go out for dinner or have other cutbacks to afford makeup,” recalls Joey.

As e.l.f. grows, Joey and Scott make decisions on how to finance that growth. Up to now, they have used a mix of long- term debt and equity.

Financing a large part of their business with long-term debt requires that Joey and Scott carefully manage liabilities. This is especially true with long-term financing from sources such as notes and bonds. They also know that retaining more equity in the business helped them personally when e.l.f. issued stock.

Joey and Scott welcome the financial rewards, yet they insist e.l.f. is about making the consumer feel more confident. “The con- sumer feels better inside” from using e.l.f. products, claims Scott. “There’s more of a glimmer.”

Sources: e.l.f. Cosmetics website, January 2019; CNN, January 2006

©Clemens Bilan/Douglas/Getty Images

This section explains bonds and reasons for issuing them. Both for-profit and nonprofit compa- nies, as well as governmental units, such as nations, states, cities, and schools, issue bonds.

Bond Financing Projects that need a lot of money often are financed with bonds. A bond is its issuer’s written promise to pay the par value of the bond with interest. The par value of a bond, or face value, is paid at a stated future date called the maturity date. Most bonds require the issuer to make semiannual (twice a year) interest payments. Interest is computed by multiplying the par value by the bond’s contract rate.

Advantages of Bonds There are three main advantages of bond financing. 1. Bonds do not affect owner control. Equity affects ownership in a company, but bonds do

not. A person who contributes $1,000 of a company’s $10,000 equity financing typically controls one-tenth of the company. A person who owns a $1,000, 11%, 20-year bond has no ownership.

2. Interest on bonds is tax deductible. Bond interest payments are tax deductible, but distri- butions to owners are not. A corporation with no bond financing, $15,000 in pretax income, and a 40% tax rate pays $6,000 ($15,000 × 40%) in taxes. Instead, if it issues bonds and pays $10,000 in bond interest expense, then taxes paid are only $2,000 ([$15,000 − $10,000] × 40%).

3. Bonds can increase return on equity. A company that earns a higher return with borrowed funds than it pays in interest on those funds increases its return on equity. This process is called financial leverage, or trading on the equity.

To demonstrate the third point, consider Magnum Co., which has $1,000 in equity and is planning a $500 expansion ($ millions). Magnum predicts the expansion will increase income by $125 before paying interest. It currently earns $100 per year and has no interest expense. Magnum is considering three plans. Plan A is to not expand. Plan B is to expand and raise $500 from equity financing. Plan C is to expand and issue $500 of bonds that pay 10% annual interest ($50). Exhibit 10.1 shows how these plans affect net income, equity, and return on equity (Net

BASICS OF BONDS

A1 Compare bond financing with stock financing.

IN C ONTROL!

382 Chapter 10 Accounting for Long-Term Liabilities

EXHIBIT 10.1 Financing with Bonds versus Equity

Plan A: Plan B: Plan C: $ millions Do Not Expand Equity Financing Bond Financing

Income before interest expense . . . . . . . . . . . $ 100 $ 225 $ 225

Interest expense . . . . . . . . . . . . . . . . . . . . . . . . — — (50)

Net income . . . . . . . . . . . . . . . . . . . . . . . . . . . $ 100 $ 225 $ 175 Equity . . . . . . . . . . . . . . . . . . . . . . . . . . . . . . . . $1,000 $1,500 $1,000

Return on equity . . . . . . . . . . . . . . . . . . . . . . . 10.0% 15.0% 17.5%

income/Equity). Magnum earns a higher return on equity under Plan C to issue bonds. Income under Plan C ($175) is smaller than under Plan B ($225), but the return on equity is larger because of less equity investment.

Disadvantages of Bonds There are two main disadvantages of bond financing. 1. Bonds can decrease return on equity. When a company earns a lower return with the bor-

rowed funds than it pays in interest, it decreases return on equity. This is more likely when a company has low income or losses.

2. Bonds require payment of both periodic interest and the par value at maturity. Bond pay- ments are a burden when income and cash flow are low. Equity does not require payments because withdrawals (dividends) are optional.

Bond Issuing Bond issuances state the number of bonds authorized, their par value, and the contract interest rate. The legal contract between the issuer and the bond- holders is called the bond indenture. A bondholder may receive a bond certificate, which is evidence of the company’s debt—see Exhibit 10.2.

Bond Trading A bond issue is the sale of bonds, usually in denominations of $1,000 or $5,000. After bonds are issued, they often are bought and sold among inves- tors, meaning that a bond probably has had many owners before it matures. When bonds are bought and sold, they have a market value (price). Bond market values are shown as a percent of par (face) value. For example, a bond trading at 1031⁄2 is bought or sold for 103.5% of par value. A bond trading at 95 is bought or sold at 95% of par value.

Point: There are nearly 5 million individual U.S. bond issues, compared to about 12,000 individual U.S. stocks.

Courtesy of RBC Wealth Management

EXHIBIT 10.2 Bond Certificate

Point: A bond with a par value of $1,000 trading at 103½ sells for $1,035 ($1,000 × 1.035).

Quotes The IBM bond quote here is interpreted (left to right) as Bonds, issuer name; Rate, contract interest rate (4%); Mat, matures in year 2042 when principal is paid; Yld, yield rate (3.81%) of bond at current price; Vol, dollar worth ($110,000) of trades (in 1,000s); Close, closing price (103.08) for the day as percentage of par value; Chg, change (+0.73%) in closing price from prior day’s close. ■

Decision Insight

Bonds Rate Mat Yld Vol Close Chg

IBM 4 42 3.81 110 103.08 +0.73%

Bonds issued at par value are called par bonds. Assume Nike issues $100,000 of 8%, two-year bonds dated December 31, 2019, that mature on December 31, 2021, and pay interest semian- nually each June 30 and December 31. If all bonds are sold at par value, Nike records the sale as follows.

PAR BONDS P1 Record issuance and interest expense for par bonds.

Example: Compute return on equity for all three plans if Magnum is subject to a 40% income tax. Answer ($ mil.):

A = 6.0% ($100[1 − 0.4]∕$1,000)

B = 9.0% ($225[1 − 0.4]∕$1,500)

C = 10.5% ($175[1 − 0.4]∕$1,000)

Dec . 31, 2019 Cash . . . . . . . . . . . . . . . . . . . . . . . . . . . . . . . . . . . . . . . . . . . . . 100,000

Bonds Payable . . . . . . . . . . . . . . . . . . . . . . . . . . . . . . . . . 100,000

Sold bonds at par.

Assets = Liabilities + Equity +100,000 +100,000

Chapter 10 Accounting for Long-Term Liabilities 383

Nike records the first semiannual interest payment as follows. The same entry is made every six months, including at the maturity date.

When the bonds mature, Nike records its payment of principal as follows.

June 30, 2020 Bond Interest Expense . . . . . . . . . . . . . . . . . . . . . . . . . . . . . . . 4,000

Cash . . . . . . . . . . . . . . . . . . . . . . . . . . . . . . . . . . . . . . . . . 4,000

Paid semiannual interest (8% × $100,000 × 1∕2 year).

Assets = Liabilities + Equity −4,000 −4,000

Dec . 31, 2021 Bonds Payable . . . . . . . . . . . . . . . . . . . . . . . . . . . . . . . . . . . . . 100,000

Cash . . . . . . . . . . . . . . . . . . . . . . . . . . . . . . . . . . . . . . . . . 100,000

Paid bond principal at maturity.

Assets = Liabilities + Equity −100,000 −100,000

This section covers bond issuances below par, called discount bonds.

Bond Discount or Premium The bond issuer pays the bond interest rate, called the contract rate (also called coupon rate, stated rate, or nominal rate). The annual interest paid is computed by multiplying the bond par value by the contract rate. The contract rate is usually stated on an annual basis, even if interest is paid semiannually. For example, a $1,000, 8% bond paying interest semiannually pays annual interest of $80 (8% × $1,000) in two semiannual payments of $40 each.

The contract rate sets the interest paid in cash, which is not necessarily the bond interest expense for the issuer. Bond interest expense depends on the bond’s market value at issuance. The bond’s market rate of interest is the rate that borrowers are willing to pay and lenders are willing to accept for a bond and its risk level. As bond risk increases, the market rate increases to compensate bond purchasers.

DISCOUNT BONDS

A company issues 8%, two-year bonds on December 31, 2019, with a par value of $7,000 and semiannual interest payments. On the issue date, the annual market rate for these bonds is 8%, which implies a selling price of $7,000. Prepare journal entries to record (a) the issuance of bonds on December 31, 2019; (b) the first through fourth interest payments on each June 30 and December 31; and (c) the maturity of the bonds on December 31, 2021.

Solution

a.

2020–2021 Bond Interest Expense . . . . . . . . . . . . . . . . . . . . . . . . . . . . . . . 280

June 30 and Cash . . . . . . . . . . . . . . . . . . . . . . . . . . . . . . . . . . . . . . . . . 280

Dec . 31 Pay semiannual interest ($7,000 × 8% × 1∕2).

b. The following entry is made for each of the four interest payments of June 30 and December 31 for both 2020 and 2021.

Dec . 31, 2021 Bonds Payable . . . . . . . . . . . . . . . . . . . . . . . . . . . . . . . . . . . . . 7,000

Cash . . . . . . . . . . . . . . . . . . . . . . . . . . . . . . . . . . . . . . . . . 7,000

Record maturity and payment of bonds.

c.

Recording Par Value Bonds

NEED-TO-KNOW 10-1

P1

Do More: QS 10-2, QS 10-3, E 10-2, E 10-3

Dec . 31, 2019 Cash . . . . . . . . . . . . . . . . . . . . . . . . . . . . . . . . . . . . . . . . . . . . . 7,000

Bonds Payable . . . . . . . . . . . . . . . . . . . . . . . . . . . . . . . . . 7,000

Sold bonds at par.

384 Chapter 10 Accounting for Long-Term Liabilities

When the contract rate and market rate are equal, a bond sells at par value. If they are not equal, it is sold at a premium above par value or at a discount below par value. Exhibit 10.3 shows the relation between the contract rate, the market rate, and a bond’s issue price.

Issuing Bonds at a Discount A discount on bonds payable occurs when a company issues bonds with a contract rate less than the market rate. This means the issue price is less than par value—the issuer gets less money at issuance than what the issuer must pay back at maturity. Assume Fila issues bonds with a $100,000 par value, an 8% annual contract rate (paid semiannually), and a two-year life. These bonds sell at a discount price of 96.400 (meaning 96.400% of par value, or $96,400); we show how to compute bond prices in Appendix 10A.

Cash Payments with Discount Bonds These bonds require Fila to pay 1. Par value of $100,000 cash at the end of the bonds’ two-year life. 2. Semiannual cash interest payments of $4,000 ($100,000 × 8% × 1/2 year).

The pattern of cash receipts and payments for Fila bonds is shown in Exhibit 10.4.

P2 Record issuance and amortization of discount bonds using the straight-line method.

Bond Sets Market Sets Bond Price Determined

Contract rate > Market rate Bond sells at premium

Contract rate = Market rate Bond sells at par

Contract rate < Market rate Bond sells at discount

Contract rate Market rate

EXHIBIT 10.3 Relation between Bond Issue Price, Contract Rate, and Market Rate

EXHIBIT 10.4 Discount Bond Cash Receipts and Payments

$100,000 $4,000 $4,000 $4,000 $4,000 o o o o o

0 6 mo . 12 mo . 18 mo . 24 mo .

} $116,000 paid

$9 6,

40 0

re c’

d

Recording Issuance of Discount Bonds When Fila accepts $96,400 cash for its bonds on the issue date of December 31, 2019, it records the sale as follows.

Bonds payable are reported as a long-term liability on Fila’s December 31, 2019, balance sheet as in Exhibit 10.5. A discount is subtracted from par value to get the carrying (book) value of bonds. Discount on Bonds Payable is a contra liability account.

Point: Book value at issuance always equals the issuer’s cash borrowed.

Long-term liabilities

Bonds payable, 8%, due December 31, 2021 . . . . . . . . . . . . $100,000

Less discount on bonds payable . . . . . . . . . . . . . . . . . . . . . . 3,600 $96,400 Carrying (book) value

EXHIBIT 10.5 Balance Sheet Presentation of Bond Discount

Dec . 31, 2019 Cash . . . . . . . . . . . . . . . . . . . . . . . . . . . . . . . . . . . . . . . . . . . . . 96,400

Discount on Bonds Payable . . . . . . . . . . . . . . . . . . . . . . . . . . . 3,600

Bonds Payable . . . . . . . . . . . . . . . . . . . . . . . . . . . . . . . . . 100,000

Sold bonds at a discount on their issue date.

Assets = Liabilities + Equity +96,400 +100,000 −3,600

Amortizing Discount Bonds Fila receives $96,400 for its bonds; in return it must pay bondholders $100,000 when the bonds mature in two years (plus four interest payments). Panel A in Exhibit 10.6 shows that the four $4,000 interest payments plus the $3,600 bond dis- count equals total bond interest expense of $19,600.

The total $19,600 bond interest expense is allocated over the four semiannual periods in the bonds’ life, and the bonds’ carrying value is updated at each balance sheet date. This is done using

Chapter 10 Accounting for Long-Term Liabilities 385

EXHIBIT 10.6 Interest Computation and Entry for Discount Bonds

Bonds Payable

12/31/2019 100,000 6/30/2020 — 12/31/2020 — 6/30/2021 — 12/31/2021 100,000

12/31/2021 0

Discount on Bonds Payable

12/31/2019 3,600 6/30/2020 900 12/31/2020 900 6/30/2021 900 12/31/2021 900

12/31/2021 0

the straight-line method (or the effective interest method in Appendix 10B). Both methods reduce the bond discount to zero over the bond life. This process is called amortizing a bond discount.

Straight-Line Method Straight-line bond amortization allocates equal bond interest expense to each interest period. We divide the total bond interest expense of $19,600 by 4 (num- ber of semiannual periods in bonds’ life). This gives a bond interest expense of $4,900 per period. Panel B of Exhibit 10.6 shows how the issuer records bond interest ex- pense and updates the bond liability ac- count at the end of each of the four semiannual interest periods (June 30, 2020, through December 31, 2021).

Exhibit 10.7 shows the pattern of decreases in the Discount on Bonds Payable account and the pattern of increases in the bonds’ carrying value. Three points summarize the discount bonds’ straight-line amortization.

1. At issuance, the $96,400 carrying value equals the $100,000 par value minus the $3,600 unamortized discount.

2. During the bonds’ life, the (unamortized) discount decreases each period by the $900 amortization ($3,600/4), and carrying value (par value less unamortized discount) increases each period by $900.

3. At maturity, unamortized discount equals zero, and carrying value equals the $100,000 par value that the issuer pays the holder.

Point: Another way to compute bond interest expense: (1) Divide the $3,600 discount by 4 periods to get $900 amortized each pe- riod. (2) Add $900 to the $4,000 cash payment to get bond interest expense of $4,900 per period.

EXHIBIT 10.7 Straight-Line Amortization of Bond Discount

* Total bond discount of $3,600 less accumulated periodic amortization of $900 per semiannual interest period.

†Bond par value of $100,000 less unamortized discount.

Semiannual Unamortized Carrying Period-End Discount* Value†

(0) 12/31/2019 . . . . . . . . . . . $3,600 $ 96,400

(1) 6/30/2020 . . . . . . . . . . . 2,700 97,300

(2) 12/31/2020 . . . . . . . . . . . 1,800 98,200

(3) 6/30/2021 . . . . . . . . . . . 900 99,100

(4) 12/31/2021 . . . . . . . . . . . 0 100,000 The columns always sum to par value for discount bonds .

$96,400

$97,300

$98,200

$99,100

$100,000

12 /31

/2 01

9

6/ 30

/2 02

0

12 /31

/2 02

0

6/ 30

/2 02

1

12 /31

/2 02

1

Carrying value

Point: Amortization always gets the carrying value of a bond closer to its par value.

Ratings Game Many bond buyers rely on rating services such as Standard & Poor’s, Moody’s, and Fitch to assess bond risk. These services analyze financial statements and other factors in setting ratings. Standard & Poor’s ratings, from best quality to default, are AAA, AA, A, BBB, BB, B, CCC, CC, C, and D. Bonds rated in the A and B range are referred to as investment grade; lower-rated bonds are considered riskier. ■

Decision Insight

A company issues 8%, two-year bonds on December 31, 2019, with a par value of $7,000 and semiannual interest payments. On the issue date, the annual market rate for these bonds is 10%, which implies a sell- ing price of 96.46 or $6,752. (a) Prepare an amortization table like Exhibit 10.7 for these bonds; use the straight-line method to amortize the discount. Then prepare journal entries to record (b) the issuance of bonds on December 31, 2019; (c) the first through fourth interest payments on each June 30 and December 31; and (d) the maturity of the bonds on December 31, 2021.

Recording Discount Bonds

NEED-TO-KNOW 10-2

P2

Panel A: Interest Computations Four payments of $4,000 (4 pymts × [$100,000 × 0 .08 × 1∕2 yr]) . . . . . . . . . . . . . . . . . . . $ 16,000 Plus discount . . . . . . . . . . . . . . . . . . . . . . . . . . . . . . . . . . . . . . . . . . . . . . . . . . . . . . . . . . . . . . 3,600 Total bond interest expense . . . . . . . . . . . . . . . . . . . . . . . . . . . . . . . . . . . . . . . . . . . . . . . . . $19,600

Bond interest expense

(per interest period) = Total bond interest expense Number of interest periods =

$19,600 4

= $4,900

Panel B: Entry to Record Interest Payment and Amortization

2020–2021 Bond Interest Expense . . . . . . . . . . . . . . . . . 4,900 June 30 and Discount on Bonds Payable . . . . . . . . 900 Dec . 31 Cash . . . . . . . . . . . . . . . . . . . . . . . . . . . 4,000

Record semiannual interest and discount amortization (straight-line method).

Par value × ½ × Contract rate

Discount ÷ Periods

Solution

a. Semiannual Unamortized Carrying Period-End Discount Value

(0) 12/31/2019 . . . . $248 $6,752

(1) 6/30/2020 . . . . 186 6,814

(2) 12/31/2020 . . . . 124 6,876

(3) 6/30/2021 . . . . 62 6,938

(4) 12/31/2021 . . . 0 7,000

Interest computations for solutions a, b, and c Four interest payments of $280 (4 pymts × [$7,000 × 0 .08 × 1∕2 yr]) . . . . . $1,120 Plus discount . . . . . . . . . . . . . . . . . . . . . . . . . . . 248

Total bond interest expense . . . . . . . . . . . . . . . . $1,368

Divided by number of periods . . . . . . . . . . . . . . ÷ 4 Bond interest expense per period . . . . . . . . . . . $ 342

d. Dec . 31, 2021 Bonds Payable . . . . . . . . . . . . . . . . . . . . . . . . . . . . . . . . . . . . . . . . . . . 7,000 Cash . . . . . . . . . . . . . . . . . . . . . . . . . . . . . . . . . . . . . . . . . . . . . . . 7,000

Record maturity and payment of bonds.

Do More: QS 10-5, QS 10-7, QS 10-8, E 10-4, E 10-5,

E 10-6, P 10-1

Bonds Payable 12/31/2019 7,000 6/30/2020 — 12/31/2020 — 6/30/2021 — 12/31/2021 7,000

12/31/2021 0

Discount on Bonds Payable 12/31/2019 248 6/30/2020 62 12/31/2020 62 6/30/2021 62 12/31/2021 62

12/31/2021 0

c. The following entry is made for each of the four interest payments on June 30 and December 31 for both 2020 and 2021.

*$248∕4 †$7,000 × 8% × 1∕2

2020–2021 Bond Interest Expense . . . . . . . . . . . . . . . . . . . . . . . . . . . . . . . . . . . . . 342

June 30 and Discount on Bonds Payable* . . . . . . . . . . . . . . . . . . . . . . . . . . . . 62

Dec . 31 Cash† . . . . . . . . . . . . . . . . . . . . . . . . . . . . . . . . . . . . . . . . . . . . . . 280

Pay semiannual interest and record amortization.

386 Chapter 10 Accounting for Long-Term Liabilities

b. Dec . 31, 2019 Cash . . . . . . . . . . . . . . . . . . . . . . . . . . . . . . . . . . . . . . . . . . . . . . . . . . . . 6,752 Discount on Bonds Payable . . . . . . . . . . . . . . . . . . . . . . . . . . . . . . . . . . 248

Bonds Payable . . . . . . . . . . . . . . . . . . . . . . . . . . . . . . . . . . . . . . . 7,000

Sold bonds at discount.

Point: Straight-line amortization is GAAP when the effect of using it approximates effective interest amortization.

This section covers bond issuances above par, called premium bonds.

Issuing Bonds at a Premium When the contract rate is higher than the market rate, the bonds sell at a price higher than par value—the issuer gets more money at issuance than what the issuer must pay back at maturity. The amount by which the bond price exceeds par value is the premium on bonds. Assume Adidas issues bonds with a $100,000 par value, a 12% annual contract rate, semiannual interest payments, and a two-year life. The Adidas bonds sell at a premium price of 103.600 (meaning 103.600% of par value, or $103,600); we show how to compute bond prices in Appendix 10A.

Cash Payments with Premium Bonds These bonds require Adidas to pay 1. Par value of $100,000 cash at the end of the bonds’ two-year life. 2. Semiannual cash interest payments of $6,000 ($100,000 × 12% × 1∕2 year).

The pattern of cash receipts and payments for Adidas bonds is shown in Exhibit 10.8.

Point: Contract rate yields cash interest payment. Market rate yields interest expense.

PREMIUM BONDS

P3 Record issuance and amortization of premium bonds using the straight- line method.

EXHIBIT 10.8 Premium Bond Cash Receipts and Payments

$100,000 $6,000 $6,000 $6,000 $6,000 o o o o o

0 6 mo . 12 mo . 18 mo . 24 mo .

} $124,000 paid

$1 03

,6 00

re c’

d

Chapter 10 Accounting for Long-Term Liabilities 387

Recording Issuance of Premium Bonds When Adidas receives $103,600 cash for its bonds on the issue date of December 31, 2019, it records this as follows.

Bonds payable are reported as a long-term liability on Adidas’s December 31, 2019, balance sheet as in Exhibit 10.9. A premium is added to par value to get the carrying (book) value of bonds. Premium on Bonds Payable is an adjunct (“add-on”) liability account.

Long-term liabilities

Bonds payable, 12%, due December 31, 2021 . . . . . . . . . . . . . $100,000

Plus premium on bonds payable . . . . . . . . . . . . . . . . . . . . . . . 3,600 $103,600

EXHIBIT 10.9 Balance Sheet Presentation of Bond Premium

Amortizing Premium Bonds Adidas receives $103,600 for its bonds. In return, it pays bondholders $100,000 after two years (plus four interest payments). Panel A of Exhibit 10.10 shows that the four $6,000 interest payments minus the $3,600 bond premium equals total bond interest expense of $20,400. The premium is subtracted because it reduces the issuer’s cost. Total bond interest expense is allocated over the four semiannual periods using the straight- line method (or the effective interest method in Appendix 10B).

Bonds Payable

12/31/2019 100,000 6/30/2020 — 12/31/2020 — 6/30/2021 — 12/31/2021 100,000

12/31/2021 0

Premium on Bonds Payable

12/31/2019 3,600 6/30/2020 900 12/31/2020 900 6/30/2021 900 12/31/2021 900

12/31/2021 0

Dec . 31, 2019 Cash . . . . . . . . . . . . . . . . . . . . . . . . . . . . . . . . . . . . . . . . . . . . . 103,600

Premium on Bonds Payable . . . . . . . . . . . . . . . . . . . . . . 3,600

Bonds Payable . . . . . . . . . . . . . . . . . . . . . . . . . . . . . . . . . 100,000

Sold bonds at a premium on their issue date.

Assets = Liabilities + Equity +103,600 +100,000 +3,600

Straight-Line Method The straight-line method allocates equal bond interest ex- pense to each semiannual interest period. We divide the total bond interest expense of $20,400 by 4 (number of semiannual periods in bonds’ life). This gives bond interest expense of $5,100 per period. Panel B of Exhibit 10.10 shows how Adidas records bond inter- est expense and updates the balance of the bond liability account for each semiannual period (June 30, 2020, through December 31, 2021).

Exhibit 10.11 shows the pattern of decreases in the unamortized Pre- mium on Bonds Payable account and

Point: A premium decreases Bond Interest Expense; a discount increases it.

Semiannual Unamortized Carrying Period-End Premium* Value†

(0) 12/31/2019 . . . . . . . . . $3,600 $103,600

(1) 6/30/2020 . . . . . . . . . 2,700 102,700

(2) 12/31/2020 . . . . . . . . . 1,800 101,800

(3) 6/30/2021 . . . . . . . . . 900 100,900

(4) 12/31/2021 . . . . . . . . . 0 100,000

* Total bond premium of $3,600 less accumulated periodic amortization of $900 per semiannual interest period.

†Bond par value of $100,000 plus unamortized premium.

During the bond life, carrying value is adjusted to par and the amortized premium to zero .

EXHIBIT 10.11 Straight-Line Amortization of Bond Premium

EXHIBIT 10.10 Interest Computation and Entry for Premium Bonds

Panel A: Interest Computations Four payments of $6,000 (4 pymts × [$100,000 × 0 .12 × 1∕2 yr]) . . . . . . . . . . . . . . . . . . . $ 24,000 Less premium . . . . . . . . . . . . . . . . . . . . . . . . . . . . . . . . . . . . . . . . . . . . . . . . . . . . . . . . . . . . . (3,600) Total bond interest expense . . . . . . . . . . . . . . . . . . . . . . . . . . . . . . . . . . . . . . . . . . . . . . . . . $20,400

Bond interest expense (per interest period)

= Total bond interest expense Number of interest periods

= $20,400

4 = $5,100

Panel B: Entry to Record Interest Payment and Amortization

2020–2021 Bond Interest Expense . . . . . . . . . . . . . . . . . . . 5,100 June 30 and Premium on Bonds Payable . . . . . . . . . . . . . . . 900 Dec . 31 Cash . . . . . . . . . . . . . . . . . . . . . . . . . . . . . 6,000

Record semiannual interest and premium amortization (straight-line method).

Premium ÷ Periods

Par value × ½ × Contract rate

388 Chapter 10 Accounting for Long-Term Liabilities

in the bonds’ carrying value. Three points summarize straight-line amortization of premium bonds.

1. At issuance, the $103,600 carrying value equals the $100,000 par value plus the $3,600 unamortized premium.

2. During the bonds’ life, the (unamortized) premium decreases each period by the $900 amor- tization ($3,600/4), and carrying value decreases each period by the same $900.

3. At maturity, unamortized premium equals zero, and carrying value equals the $100,000 par value that the issuer pays the holder.

$103,600

$102,700

$101,800

$100,900

$100,000

12 /31

/2 01

9

6/ 30

/2 02

0

12 /31

/2 02

0

6/ 30

/2 02

1

12 /31

/2 02

1

Carrying value

A company issues 8%, two-year bonds on December 31, 2019, with a par value of $7,000 and semiannual interest payments. On the issue date, the annual market rate for these bonds is 6%, which implies a selling price of 103.71 or $7,260. (a) Prepare an amortization table like Exhibit 10.11 for these bonds; use the straight- line method to amortize the premium. Then prepare journal entries to record (b) the issuance of bonds on December 31, 2019; (c) the first through fourth interest payments on each June 30 and December 31; and (d) the maturity of the bonds on December 31, 2021.

Solution

a.

Recording Premium Bonds

NEED-TO-KNOW 10-3

P3

Semiannual Unamortized Carrying Period-End Premium Value

(0) 12/31/2019 . . . . $260 $7,260

(1) 6/30/2020 . . . . 195 7,195

(2) 12/31/2020 . . . . 130 7,130

(3) 6/30/2021 . . . . 65 7,065

(4) 12/31/2021 . . . . 0 7,000

Interest computations for solutions a, b, and c Four interest payments of $280 (4 pymts × [$7,000 × 0 .08 × 1∕2 yr]) . . . . $1,120 Less premium . . . . . . . . . . . . . . . . . . . . . . . . . 260

Total bond interest expense . . . . . . . . . . . . . . $ 860

Divided by number of periods . . . . . . . . . . . . ÷ 4 Bond interest expense per period . . . . . . . . . $ 215

b. Dec . 31, 2019 Cash . . . . . . . . . . . . . . . . . . . . . . . . . . . . . . . . . . . . . . . . . . . . . . . . . . . . 7,260

Premium on Bonds Payable . . . . . . . . . . . . . . . . . . . . . . . . . . . . 260

Bonds Payable . . . . . . . . . . . . . . . . . . . . . . . . . . . . . . . . . . . . . . . 7,000

Sold bonds at premium.

Do More: QS 10-9, E 10-8, E 10-9, P 10-2, P 10-3

d. Dec . 31, 2021 Bonds Payable . . . . . . . . . . . . . . . . . . . . . . . . . . . . . . . . . . . . . . . . . . . 7,000

Cash . . . . . . . . . . . . . . . . . . . . . . . . . . . . . . . . . . . . . . . . . . . . . . . 7,000

Record maturity and payment of bonds.

Bonds Payable

12/31/2019 7,000 6/30/2020 — 12/31/2020 — 6/30/2021 — 12/31/2021 7,000

12/31/2021 0

Premium on Bonds Payable

12/31/2019 260 6/30/2020 65 12/31/2020 65 6/30/2021 65 12/31/2021 65

12/31/2021 0

c. The following entry is made for each of the four interest payments on June 30 and December 31 for both 2020 and 2021.

2020–2021 Bond Interest Expense . . . . . . . . . . . . . . . . . . . . . . . . . . . . . . . . . . . . . 215

June 30 and Premium on Bonds Payable* . . . . . . . . . . . . . . . . . . . . . . . . . . . . . . . . 65

Dec . 31 Cash† . . . . . . . . . . . . . . . . . . . . . . . . . . . . . . . . . . . . . . . . . . . . . . 280

Pay semiannual interest and record amortization.

*$260∕4 †$7,000 × 8% × 1∕2

Bond Retirement This section covers the retirement of bonds.

Bond Retirement at Maturity The carrying value of bonds at maturity always equals par value. For example, both Exhibits 10.7 (a discount) and 10.11 (a premium) show that the

P4 Record the retirement of bonds.

Chapter 10 Accounting for Long-Term Liabilities 389

carrying value of bonds at maturity equals par value ($100,000). Retirement of these bonds at maturity, assuming interest is already paid and recorded, is as follows.

Dec . 31, 2021 Bonds Payable . . . . . . . . . . . . . . . . . . . . . . . . . . . . . . . . . . . . . . 100,000

Cash . . . . . . . . . . . . . . . . . . . . . . . . . . . . . . . . . . . . . . . . . . 100,000

Record retirement of bonds at maturity.

Assets = Liabilities + Equity −100,000 −100,000

Bond Retirement before Maturity Issuers sometimes retire some or all of their bonds before maturity. If interest rates decline, an issuer may want to replace high-interest- paying bonds with new low-interest bonds. There are two common ways to retire bonds before maturity. Exercise a call option. An issuer can reserve the right to retire bonds early by issuing call-

able bonds. This gives the issuer an option to call the bonds before they mature by paying the par value plus a call premium.

Open market purchase. The issuer can repurchase them from bondholders at current market price.

Whether bonds are called or purchased, the issuer is likely to pay a price different from their carrying value. The issuer records a difference between the bonds’ carrying value and the amount paid as a gain or loss. Assume that Puma issued callable bonds with a par value of $100,000. The call option requires Puma to pay a call premium of $3,000 to bondholders plus the par value. Next, assume that after the June 30 interest payment, the bonds have a carrying value of $104,500. Then on July 1, Puma calls these bonds and pays $103,000 to bondholders. Puma records a $1,500 gain from the difference between the bonds’ carrying value of $104,500 and the retirement price of $103,000 as follows.

Point: Bond retirement is also called bond redemption.

Bond Retirement by Conversion Holders of convertible bonds have the right to convert their bonds to stock. When conversion occurs, the bonds’ carrying value is transferred to equity accounts and no gain or loss is recorded. (Convertible bonds are described further in the Decision Analysis section of this chapter.) Assume that on January 1 the $100,000 par value bonds of Converse, with a carrying value of $100,000, are converted to 15,000 shares of $2 par value common stock. The entry to record this conversion follows (market prices of the bonds and stock are not relevant to this entry).

Convertible Bond

July 1 Bonds Payable . . . . . . . . . . . . . . . . . . . . . . . . . . . . . . . . . . . . . . 100,000

Premium on Bonds Payable . . . . . . . . . . . . . . . . . . . . . . . . . . . 4,500

Gain on Bond Retirement . . . . . . . . . . . . . . . . . . . . . . . . . 1,500

Cash . . . . . . . . . . . . . . . . . . . . . . . . . . . . . . . . . . . . . . . . . . 103,000

Record retirement of bonds before maturity.

Assets = Liabilities + Equity −103,000 −100,000 +1,500 −4,500

Jan . 1 Bonds Payable . . . . . . . . . . . . . . . . . . . . . . . . . . . . . . . . . . . . . . 100,000

Common Stock . . . . . . . . . . . . . . . . . . . . . . . . . . . . . . . . . 30,000

Paid-In Capital in Excess of Par Value . . . . . . . . . . . . . . . 70,000

Record retirement of bonds by conversion.

Assets = Liabilities + Equity −100,000 +30,000 +70,000

Junk Bonds Junk bonds are company bonds with low credit ratings due to a higher likelihood of nonpayment. On the upside, the high risk of junk bonds can yield high returns if the issuer repays its debt. Investors in junk bonds identify and buy bonds with low credit ratings when they believe those bonds will survive and pay their debts. Financial statements are used to identify junk bonds that are better than what their ratings would suggest. ■

Decision Insight

390 Chapter 10 Accounting for Long-Term Liabilities

Payments of Principal and Interest Payments on an installment note include ac- crued interest expense plus part of the amount borrowed (the principal). For this section, let’s consider an installment note with equal payments. The equal total payments pattern has chang- ing amounts of both interest and principal. Foghog borrows $60,000 by signing a $60,000 note that requires three equal payments of $23,282 at each year-end. Exhibit 10.12 shows the pattern of equal total payments and its two parts, interest and principal. Column A shows the note’s beginning balance. Column B shows accrued interest at 8% of the beginning note balance. Column C shows the portion that reduces the principal owed, which equals total payment in column D minus interest expense in column B. Column E shows the note’s year-end balance.

Like bonds, notes are issued in exchange for assets such as cash. Unlike bonds, notes are usually issued to a single lender such as a bank. An issuer initially records a note at its selling price—the note’s face value minus any discount or plus any premium. Over the note’s life, the amount of interest expense allocated to each period is computed by multiplying the market rate (at issuance of the note) by the beginning-of-period note balance. The note’s carrying (book) value at any time equals its face value minus any unamortized discount or plus any unamortized premium.

Installment Notes An installment note is a liability requiring a series of payments to the lender. Installment notes are common for franchises and other businesses when lenders and borrowers agree to spread payments over time.

Issuance of Notes Assume Foghog borrows $60,000 from a bank to purchase equip- ment. It signs an 8% installment note requiring three annual payments of principal plus interest. Foghog records the note’s issuance at January 1, 2019, as follows.

LONG-TERM NOTES PAYABLE C1 Explain the types of notes and prepare entries to account for notes.

Jan . 1 Cash . . . . . . . . . . . . . . . . . . . . . . . . . . . . . . . . . . . . . . . . . . . . . . 60,000

Notes Payable . . . . . . . . . . . . . . . . . . . . . . . . . . . . . . . . . . 60,000

Borrowed $60,000 by signing 8%, three-year note.

Assets = Liabilities + Equity +60,000 +60,000

Years 2019 2020 2021

$2 3,

28 2

$2 3,

28 2

$2 3,

28 2

EXHIBIT 10.12 Installment Note: Equal Total Payments Amortization Schedule

Payments

(A) (B) (C) (D) (E) Debit Debit Credit

Beginning Balance

Interest Notes Ending

Period Ending Date Expense + Payable = Cash Balance 8% × (A) (D) – (B) (computed) (A) – (C)

(1 ) 12/31/2019 . . . . . . . . $60,000 $41,518 (2) 12/31/2020 . . . . . . . . 41,518 21,557 (3) 12/31/2021 . . . . . . . . 21,557 0

2021

2019

2020

0 $5,000 $15,000 $25,000

Equal Cash Payments*

Decreasing Accrued Interest

Increasing Principal Component

En d

of Y

ea r $18,482$4,800

$19,961$3,321

$21,557$1,725

Interest Principal

$4,800 $ 18,482 $23,282 3,321 19,961 23,282 1,725 21,557 23,282

$9,846 $60,000 $69,846

* Table B.3 in Appendix B is used to compute the dollar amount of three payments that equal the initial note balance of $60,000 at 8% interest. We go to Table B.3, row 3, and across to the 8% column, where the present value factor is 2.5771. The dollar amount is then computed by solving the following equation. The amount is computed by dividing $60,000 by 2.5771, yielding $23,282.

Table Present Value Factor Dollar Amount Present Value B.3 2.5771 × ? = $60,000

Point: Principal portion of note payments.

A B

1 Rate per period 8%

2 Number of periods 3

3 Loan amount $60,000

4 Period Principal

5 1

6 2

7 3

=−PPMT(B1,A5,B2,B3)=$18,482

=−PPMT(B1,A6,B2,B3)=$19,961

=−PPMT(B1,A7,B2,B3)=$21,557

Point: Installment note payments.

A B

1 Rate per period 8%

2 Number of periods 3

3 Loan amount $60,000

4 Loan payments

=−PMT(B1,B2,B3)=$23,282

Chapter 10 Accounting for Long-Term Liabilities 391

The three $23,282 cash payments are equal, but accrued interest decreases each year because the principal balance of the note decreases. As the amount of interest decreases each year, the portion of each payment applied to principal increases. This pattern is shown in the lower part of Exhibit 10.12. Foghog uses the amounts in Exhibit 10.12 to record its first two payments (for years 2019 and 2020) as follows. Foghog records a similar entry but with different amounts for the last payment. After three years, the Notes Payable account balance is zero.

Assets = Liabilities + Equity −23,282 −18,482 −4,800

Dec . 31, 2019 Interest Expense . . . . . . . . . . . . . . . . . . . . . . . . . . . . . . . . . . . . 4,800

Notes Payable . . . . . . . . . . . . . . . . . . . . . . . . . . . . . . . . . . . . . . 18,482

Cash . . . . . . . . . . . . . . . . . . . . . . . . . . . . . . . . . . . . . . . . . 23,282

Record first installment payment.

Assets = Liabilities + Equity −23,282 −19,961 −3,321

Dec . 31, 2020 Interest Expense . . . . . . . . . . . . . . . . . . . . . . . . . . . . . . . . . . . . 3,321

Notes Payable . . . . . . . . . . . . . . . . . . . . . . . . . . . . . . . . . . . . . . 19,961

Cash . . . . . . . . . . . . . . . . . . . . . . . . . . . . . . . . . . . . . . . . . 23,282

Record second installment payment.

Mortgage Notes and Bonds A mortgage is a legal agreement that helps protect a lender if a borrower does not make required payments on notes or bonds. A mortgage gives the lender a right to be paid from the cash proceeds of the sale of a borrower’s assets identified in the mortgage. A mortgage contract describes the mortgage terms. Mortgage notes pledge title to specific assets as security for the note. Mortgage notes are popular in the purchase of homes and plant assets. Mortgage bonds are backed by the issuer’s assets. Accounting for mortgage notes and bonds is similar to that for unsecured notes and bonds, except that the mortgage agreement must be disclosed. For example, TIBCO Software reports that its “mortgage note payable . . . is collateralized by the commercial real property acquired.”

Lurking Debt A study reports that 29% of employees in finance and accounting witnessed the falsifying or manipu- lating of accounting information in the past year. This includes nondisclosure of some long-term liabilities. Another study reports that most people committing fraud (36%) work in the finance function of their firm (KPMG). ■

Ethical Risk

On January 1, 2019, a company borrows $1,000 cash by signing a four-year, 5% installment note. The note requires four equal payments of $282, consisting of accrued interest and principal on December 31 of each year from 2019 through 2022. 1. Prepare an amortization table for this installment note like the one in Exhibit 10.12. 2. Prepare journal entries to record the loan on January 1, 2019, and the four payments from December 31,

2019, through December 31, 2022.

Solution

1. Amortization table for loan.

C1

Recording Installment Note

NEED-TO-KNOW 10-4

Payments

(A) (B) (C) (D) (E) Debit Debit Credit Beginning Interest Notes Ending Balance Expense Payable Cash Balance Period Ending Date [Prior (E)] [5% × (A)] + [(D) − (B)] = [computed] [(A) − (C)]

2019 . . . . . . . . . . . . . . . . $1,000 $ 50 $ 232 $ 282† $768

2020 . . . . . . . . . . . . . . . . 768 38 244 282 524

2021 . . . . . . . . . . . . . . . . 524 26 256 282 268

2022 . . . . . . . . . . . . . . . . 268 14* 268 282 0

$128 $1,000 $1,128

*Adjusted for rounding. †Amount of each payment = Initial note balance∕PV of annuity for 4 periods at 5% (from Table B.3) = $1,000∕3.5460 = $282 (rounded)

Point: An annuity is a series of equal payments occurring at equal time intervals.

392 Chapter 10 Accounting for Long-Term Liabilities

Jan . 1, 2019 Cash . . . . . . . . . . . . . . . . . . . . . . . . . . . . . 1,000

Notes Payable . . . . . . . . . . . . . . . . . 1,000

Borrowed $1,000 by giving a note.

Dec . 31, 2019 Interest Expense . . . . . . . . . . . . . . . . . . . 50

Notes Payable . . . . . . . . . . . . . . . . . . . . . 232

Cash . . . . . . . . . . . . . . . . . . . . . . . . . 282

Record first installment payment.

Dec . 31, 2020 Interest Expense . . . . . . . . . . . . . . . . . . . 38

Notes Payable . . . . . . . . . . . . . . . . . . . . . 244

Cash . . . . . . . . . . . . . . . . . . . . . . . . . 282

Record second installment payment.

Dec . 31, 2021 Interest Expense . . . . . . . . . . . . . . . . . . . 26

Notes Payable . . . . . . . . . . . . . . . . . . . . . 256

Cash . . . . . . . . . . . . . . . . . . . . . . . . . 282

Record third installment payment.

Dec . 31, 2022 Interest Expense . . . . . . . . . . . . . . . . . . . 14

Notes Payable . . . . . . . . . . . . . . . . . . . . . 268

Cash . . . . . . . . . . . . . . . . . . . . . . . . . 282

Record fourth installment payment.

2.

Do More: QS 10-12, E 10-12, E 10-13, P 10-5

Debt Features and the Debt-to-Equity RatioDecision Analysis

Features of Bonds and Notes This section covers features of debt securities.

Secured or Unsecured Secured bonds (and notes) have specific assets of the issuer pledged (or mortgaged) as collateral. If the issuer does not pay its debt, the secured holders can demand that the col- lateral be sold and the proceeds used to pay the obligation. Unsecured bonds (and notes), also called debentures, are backed by the issuer’s general credit standing and are riskier than secured debt.

Term or Serial Term bonds (and notes) mature on one specified date. Serial bonds (and notes) mature at more than one date (often in series) and thus are usually repaid over a number of periods. For instance, $100,000 of serial bonds might mature at the rate of $10,000 each year from 6 to 15 years after they are issued. Sinking fund bonds reduce the holder’s risk by requiring the issuer to set aside assets to pay debt in a sinking fund.

Registered or Bearer Bonds issued in the names and addresses of their holders are registered bonds. The issuer makes bond payments by sending checks (or cash transfers) to registered holders. Bonds pay- able to whoever holds them (the bearer) are called bearer bonds or unregistered bonds. The holder of a bearer bond is presumed to be its rightful owner. Many bearer bonds are also coupon bonds. This term reflects interest coupons that are attached to the bonds. When each coupon matures, the holder presents it to a bank or broker for collection.

Convertible and/or Callable Convertible bonds (and notes) can be exchanged for a fixed number of shares of the issuing corporation’s stock. Convertible debt offers holders the potential to profit from increases in stock price. Holders still receive interest while the debt is held and the par value if they hold the debt to matu- rity. In most cases, the holders decide whether and when to convert debt to stock. Callable bonds (and notes) give the issuer the option to retire them at a stated dollar

amount before maturity.

Debt-to-Equity Ratio A company financed mainly with debt is more risky because liabilities must be repaid with interest, whereas equity financing does not. A measure to assess the risk of a company’s financing structure is the debt-to-equity ratio (see Exhibit 10.13).

Secured Debt

Unsecured Debt

A2 Assess debt features and their implications.

Convertible Debt Callable Debt

A3 Compute the debt-to-equity ratio and explain its use.

EXHIBIT 10.13 Debt-to-Equity Ratio Debt-to-equity =

Total liabilities Total equity

Chapter 10 Accounting for Long-Term Liabilities 393 Chapter 10 Accounting for Long-Term Liabilities 393

The debt-to-equity ratios for Nike and Under Armour are in Exhibit 10.14. Nike’s current-year debt-to- equity ratio is 0.87, meaning that debtholders contributed $0.87 for each $1 contributed by equity hold- ers. This implies a low-risk financing structure for Nike and is similar to its competitors. In comparison, Under Armour’s current-year ratio is 0.98. Analysis across the years shows that Nike’s debt-to-equity ratio has risen to a riskier level in recent years. In the case of Nike, the increase in debt-to-equity ratio is less concerning as it has historically earned higher returns with this financing than the interest rate it pays. Still, investors and debtholders will continue to monitor Nike’s debt-to-equity ratio to be sure it does not reach risky levels.

Bond Investor You plan to purchase bonds from one of two companies in the same industry that are similar in size and performance. The first company has $350,000 in total liabilities and $1,750,000 in equity. The second company has $1,200,000 in total liabilities and $1,000,000 in equity. Which company’s bonds are less risky based on the debt- to-equity ratio? ■ Answer: The debt-to-equity ratio for the first company is 0.2 ($350,000/$1,750,000) and for the second is 1.2 ($1,200,000/$1,000,000), suggesting that financing for the second company is riskier than for the first.

Decision Maker

Water Sports Company (WSC) patented and successfully test-marketed a new product. To produce and market the new product, WSC needs to raise $800,000 of financing. On January 1, 2019, the company obtained the money in two ways. a. WSC signed a $400,000, 10% installment note to be repaid with five equal annual installments of

$105,519 to be made on December 31 of 2019 through 2023. b. WSC issued five-year bonds with a par value of $400,000 for $430,881 cash on January 1, 2019. The

bonds have a 12% annual contract rate and pay interest on June 30 and December 31. The bonds’ annual market rate is 10%.

Required

1. For the installment note, (a) prepare an amortization table similar to Exhibit 10.12 and (b) prepare the journal entry for the first payment.

2. For the bonds, (a) prepare the January 1, 2019, journal entry to record their issuance; (b) prepare an amortization table using the straight-line method; (c) prepare the June 30, 2019, journal entry to record the first interest payment; and (d) prepare a journal entry to record retiring the bonds at a $416,000 call price on January 1, 2021.

3.B Using Appendix 10B, redo parts 2(b), 2(c), and 2(d) assuming the bonds are amortized using the effec- tive interest method.

PLANNING THE SOLUTION For the installment note, prepare a table similar to Exhibit 10.12 and use the numbers in the table’s first

line for the journal entry. Record the bonds’ issuance. Next, prepare an amortization table like Exhibit 10.11 (and Exhibit 10B.2)

and use it to get the numbers for the journal entry. Also use the table to find the carrying value as of the date of the bonds’ retirement needed for the journal entry.

COMPREHENSIVE

Accounting for Bonds and Notes— Amortization, Journal Entries, and Disposal

NEED-TO-KNOW 10-5

EXHIBIT 10.14 Analysis using Debt-to-Equity Ratio

Company $ millions Current Year 1 Year Ago 2 Years Ago

Nike Total liabilities . . . . . . . . . . . . . . . . . . . . . . . . . $10,852 $ 9,138 $ 8,890 Total equity . . . . . . . . . . . . . . . . . . . . . . . . . . . $12,407 $12,258 $12,707 Debt-to-equity . . . . . . . . . . . . . . . . . . . . . . . . 0.87 0.75 0.70 Under Armour Total liabilities . . . . . . . . . . . . . . . . . . . . . . . . . $ 1,988 $ 1,613 $ 1,198 Total equity . . . . . . . . . . . . . . . . . . . . . . . . . . . $ 2,019 $ 2,031 $ 1,668 Debt-to-equity . . . . . . . . . . . . . . . . . . . . . . . . 0.98 0.79 0.72

394 Chapter 10 Accounting for Long-Term Liabilities

SOLUTION Part 1: Installment Note

a. An amortization table for the long-term note payable follows.

(1) 12/31/2019 (2) 12/31/2020

(3) 12/31/2021 (4) 12/31/2022

(5) 12/31/2023

$ 40,000 33,448

26,241 18,313

9,593

$127,595

$ 105,519* 105,519

105,519 105,519

105,519

$ 527,595

$334,481 262,410

183,132 95,926

0

$400,000 334,481

262,410 183,132

95,926

$ 65,519 72,071

79,278 87,206

95,926

$400,000

Annual Period Ending

Payments ( )

Beginning Balance

a

( )a ( ) –a ( )c

( )b Debit Credit

( )d ( )e

Interest Expense

10% × d ( )( ) – b

Debit ( )c

Notes Payable Cash

(computed)

Ending Balance

* Annual payment = Note balance ∕ PV annuity factor = $400,000/3.7908 = $105,519 (The present value annuity factor is for five payments at a rate of 10%.)

b. Journal entry for December 31, 2019, payment.

Dec . 31 Interest Expense . . . . . . . . . . . . . . . . . . . . . . . . . . . . . . . . . . . . 40,000

Notes Payable . . . . . . . . . . . . . . . . . . . . . . . . . . . . . . . . . . . . . . 65,519

Cash . . . . . . . . . . . . . . . . . . . . . . . . . . . . . . . . . . . . . . . . . . 105,519

Record first installment payment.

Part 2: Bonds (Straight-Line Amortization)

a. Journal entry for January 1, 2019, issuance.

b. The straight-line amortization table for premium bonds follows. The semiannual discount amortization is $3,088, computed as $30,881/10 periods.

(0) 1/1/2019

(1) 6/30/2019

(2) 12/31/2019

(3) 6/30/2020

(4) 12/31/2020

(5) 6/30/2021

(6) 12/31/2021 (7) 6/30/2022

(8) 12/31/2022 (9) 6/30/2023

(10) 12/31/2023

$ 30,881

27,793

24,705

21,617 18,529

15,441

12,353

9,265 6,177

3,089

0*

$ 430,881

427,793

424,705

421,617 418,529

415,441

412,353 409,265

406,177

403,089

400,000

Unamortized Discount

Carrying Value

Semiannual Period-End

*Adjusted for rounding.

Jan . 1 Cash . . . . . . . . . . . . . . . . . . . . . . . . . . . . . . . . . . . . . . . . . . . . . . 430,881

Premium on Bonds Payable . . . . . . . . . . . . . . . . . . . . . . . 30,881

Bonds Payable . . . . . . . . . . . . . . . . . . . . . . . . . . . . . . . . . 400,000

Sold bonds at a premium.

Point: Bond issue price equals present value of its future cash payments discounted at bond’s market rate.

Present Value Present Cash Flow Table Factor* Amount Value

Par (maturity) value . . . . B.1 in App. B (PV of 1) 0.6139 × $400,000 = $245,560 Interest payments . . . . . . B.3 in App. B (PV of annuity) 7.7217 × 24,000 = 185,321 Price of bond . . . . . . . . . $430,881

*Present value factors are for 10 payments using a semiannual market rate of 5%.

Chapter 10 Accounting for Long-Term Liabilities 395

c. Journal entry for June 30, 2019, bond payment.

d. Journal entry for January 1, 2021, bond retirement (use carrying value as of 12/31/2020).

Jan . 1 Bonds Payable . . . . . . . . . . . . . . . . . . . . . . . . . . . . . . . . . . . . . . 400,000 Premium on Bonds Payable . . . . . . . . . . . . . . . . . . . . . . . . . . . 18,529 Cash . . . . . . . . . . . . . . . . . . . . . . . . . . . . . . . . . . . . . . . . . . 416,000 Gain on Retirement of Bonds . . . . . . . . . . . . . . . . . . . . . . 2,529 Record bond retirement for cash.

June 30 Bond Interest Expense . . . . . . . . . . . . . . . . . . . . . . . . . . . . . . . 20,912 Premium on Bonds Payable . . . . . . . . . . . . . . . . . . . . . . . . . . . 3,088 Cash . . . . . . . . . . . . . . . . . . . . . . . . . . . . . . . . . . . . . . . . . . 24,000 Paid semiannual interest on bonds.

Part 3: Bonds (Effective Interest Amortization)—Using Appendix 10B

b. The effective interest amortization table for premium bonds.

c. Journal entry for June 30, 2019, bond payment.

d. Journal entry for January 1, 2021, bond retirement (use carrying value as of 12/31/2020).

Jan . 1 Bonds Payable . . . . . . . . . . . . . . . . . . . . . . . . . . . . . . . . . . . . . . 400,000 Premium on Bonds Payable . . . . . . . . . . . . . . . . . . . . . . . . . . . 20,295 Cash . . . . . . . . . . . . . . . . . . . . . . . . . . . . . . . . . . . . . . . . . . 416,000 Gain on Retirement of Bonds . . . . . . . . . . . . . . . . . . . . . . 4,295 Record bond retirement for cash.

June 30 Bond Interest Expense . . . . . . . . . . . . . . . . . . . . . . . . . . . . . . . 21,544 Premium on Bonds Payable . . . . . . . . . . . . . . . . . . . . . . . . . . . 2,456 Cash . . . . . . . . . . . . . . . . . . . . . . . . . . . . . . . . . . . . . . . . . . 24,000 Paid semiannual interest on bonds.

$30,881

28,425

25,846

23,138 20,295

17,310

14,176

10,885

7,429

3,800

0

$430,881

428,425

425,846

423,138 420,295

417,310

414,176

410,885 407,429

403,800

400,000

$ 24,000

24,000

24,000

24,000 24,000

24,000

24,000

24,000 24,000

24,000

$240,000

$ 21,544

21,421

21,292

21,157 21,015

20,866

20,709 20,544

20,371

20,200*

$209,119

$ 2,456

2,579

2,708 2,843

2,985

3,134

3,291

3,456 3,629

3,800

$30,881

(A) Cash

Interest Paid 6% × $400,000

(B) Interest Expense

5% × Prior (E)

(C) Premium

Amortization (A) – (B)

(D) Unamortized

Premium Prior (D) – (C)

(E) Carrying

Value $400,000 + (D)

Semiannual Interest Period

1/1/2019(0)

(1)

(2)

(3)

(4)

(5)

(6)

(7)

(8)

(9)

(10)

6/30/2019

12/31/2019

6/30/2020

12/31/2020

6/30/2021

12/31/2021 6/30/2022

12/31/2022 6/30/2023

12/31/2023

*Adjusted for rounding.

Point: Using effective interest, carrying value is also computed as the present value of all remain- ing payments, discounted using the market rate at issuance.

APPENDIX

Bond Pricing 10A This section shows how to price the Fila discount bond and the Adidas premium bond described earlier.

Present Value of Discount Bonds The issue price of bonds is the present value of the bonds’ cash payments, discounted at the bonds’ market rate. The annual market rate is 10.031% for the Fila bonds. However, for simplicity, we assume a 10% annual rate in this appendix. When computing the

C2 Explain and compute bond pricing.

396 Chapter 10 Accounting for Long-Term Liabilities

Point: Excel for bond pricing.

A B

1 Annual contract rate 8%

2 Annual market rate 10%

3 Payments within yr 2

4 Years to maturity 2

5 Par (face) value $100,000

6 Issue price

=−PV(B2/B3,B3*B4,B5*B1/B3,B5) =$96,454

Present Value Present Cash Flow Table Factor Amount Value

$100,000 par (maturity) value . . . . . . . . . . . . B .1 (PV of 1) 0 .8227 × $100,000 = $ 82,270 $4,000 interest payments . . . . . . . . . . . . . . . B .3 (PV of ann .) 3 .5460 × 4,000 = 14,184 Price of bond . . . . . . . . . . . . . . . . . . . . . . . . . (using a 5% semiannual market rate) $96,454

Calculator N = 4 I/Yr = 5

PMT = 4,000 FV = 100,000

PV = 96,454

EXHIBIT 10A.1 Computing Issue Price for Fila Discount Bonds

Present Value of Premium Bonds We compute the issue price of the Adidas bonds by using the market rate to compute the present value of the bonds’ future cash flows. The annual mar- ket rate is 9.97% for the Adidas bonds. However, for simplicity, we assume a 10% annual rate in this appendix. When computing the present value of these bonds, we again use semiannual compounding periods because this is the time between interest payments. The annual 10% market rate is applied as a semiannual rate of 5%, and the two-year bond life is viewed as four semiannual periods. The computa- tion has two parts.

1 Find the present value of the $100,000 par value paid at maturity. 2 Find the present value of the four payments of $6,000 each; see Exhibit 10.8.

These present values are found using Excel or a calculator (see directions to the side). We also can find present value if the market rate is in present value tables. The annual market rate is 10%, or 5% semiannually. In this case, go to Table B.1, row 4, and across to the 5% column, where the present value factor is 0.8227 for the maturity payment. Second, go to Table B.3, row 4, and across to the 5% column, where the present value factor is 3.5460 for the series of interest payments. The bonds’ price is computed by multiplying the cash flow payments by their present value factors and adding them— see Exhibit 10A.2.

Point: Excel for bond pricing.

A B

1 Annual contract rate 12%

2 Annual market rate 10%

3 Payments within yr 2

4 Years to maturity 2

5 Par (face) value $100,000

6 Issue price

=−PV(B2/B3,B3*B4,B5*B1/B3,B5) =$103,546

Calculator N = 4 I/Yr = 5

PMT = 6,000 FV = 100,000

PV = 103,546

EXHIBIT 10A.2 Computing Issue Price for Adidas Premium Bonds

Present Value Present Cash Flow Table Factor Amount Value

$100,000 par (maturity) value . . . . . . . . . . . . B .1 (PV of 1) 0 .8227 × $100,000 = $ 82,270 $6,000 interest payments . . . . . . . . . . . . . . . B .3 (PV of ann .) 3 .5460 × 6,000 = 21,276 Price of bond . . . . . . . . . . . . . . . . . . . . . . . . . (using a 5% semiannual market rate) $103,546

Equivalent Payments Concept Business decisions involve the time value of money. To help in those decisions, the present value factors can be thought of as equivalent payments. For example, using the data in Exhibit 10A.1, one payment of $100,000 scheduled two years from today is the equivalent of a 0.8227 payment of $100,000 today (assuming a market with 10% return). Similarly, four semiannual payments of $4,000 over the next two years are the equivalent of 3.5460 payments of $4,000 today (again, assuming a 10% return). ■

Decision Insight

Point: Calculator inputs defined: N Number of semiannual periods I/Yr Market rate per semiannual

period FV Future (maturity) value PMT Payment (interest) per

semiannual period PV Price (present value)

present value of the Fila bonds, we use semiannual compounding periods because this is the time between interest payments; the annual market rate of 10% is considered a semiannual rate of 5%. Also, the two- year bond life is viewed as four semiannual periods. The price computation has two parts.

1 Find the present value of the $100,000 par value paid at maturity. 2 Find the present value of the four semiannual payments of $4,000 each; see Exhibit 10.4.

The present values are found using Excel or a calculator (see directions to the side). We also can find present values if the market rate is in present value tables. Appendix B at the end of this book shows present value tables and describes their use. Table B.1 in Appendix B is used for the single $100,000 maturity payment, and Table B.3 in Appendix B is used for the $4,000 series of interest payments. The annual market rate is 10%, or 5% semiannually. In this case, we go to Table B.1, row 4, and across to the 5% column to identify the present value factor of 0.8227 for the maturity payment. Next, we go to Table B.3, row 4, and across to the 5% column, where the present value factor is 3.5460 for the interest payments. We compute bond price by multiplying the cash flow payments by their present value factors and adding them—see Exhibit 10A.1.

Chapter 10 Accounting for Long-Term Liabilities 397

APPENDIX

Effective Interest Amortization 10B Effective Interest Amortization of Discount Bonds The effective interest method allocates total bond interest expense over the bonds’ life in a way that yields a constant rate of interest. This constant rate of interest is the market rate at the issue date. This means bond interest expense for a period equals the carrying value of the bond at the beginning of that period multiplied by the market rate when issued. Exhibit 10B.1 shows an effective interest amortization table for Fila bonds (as described in Exhibit 10.4). The key difference between the effective interest and straight-line methods is computing bond interest expense. Instead of assigning an equal amount of bond interest expense to each period, the effective interest method assigns a bond interest expense amount that increases over the life of a discount bond. Both methods allocate the same $19,600 of total bond interest expense over the bonds’ life, but in different patterns. Specifically, the amortization table in Exhibit 10B.1 shows that the balance of the discount (column D) is amortized until it reaches zero. Also, the bonds’ carrying value (column E) changes each period until it equals par value at maturity. Compare columns D and E to the columns in Exhibit 10.7 to see the amortization patterns. Total bond interest expense is $19,600, consisting of $16,000 of semiannual cash payments and $3,600 of the original bond discount, the same for both methods.

P5 Compute and record amor- tization of a bond discount using the effective interest method.

Point: Contract rate determines cash interest paid, but market rate determines the actual interest expense.

Effective Interest Amortization of Premium Bonds Exhibit 10B.2 shows the amortization table using the effective interest method for Adidas bonds (as described in Exhibit 10.8). Column A lists the semiannual cash payments. Column B shows the amount of bond interest expense, computed as the 4.9851% semiannual market rate at issuance multiplied by the beginning-of-period car- rying value. The amount of cash paid in column A is larger than the bond interest expense because the

EXHIBIT 10B.1 Effective Interest Amortization of Bond Discount

6/30/2020 12/31/2020

12/31/2021

6/30/2021

12/31/2019(0)

( 1 ) (2) (3)

(4)

$ 4,000 4,000

4,000

4,000

$16,000

$ 4,835$ 4,835 4,877 4,921

4,967

$19,600

$ 835 877 921

967 $3,600

2,765 $3,600

1,888

967 0

97,235

$ 96,400

98,112 99,033

100,000

Bonds: $100,000 Par Value, Semiannual Interest Payments, Two-Year Life, 4% Semiannual Contract Rate, 5.0155% Semiannual Market Rate

(A) Cash

Interest Paid

4% × $100,000

(B) Bond

Interest Expense

5.0155% × Prior (E)

(C)

Discount Amortization

(B) – (A)

(D)

Unamortized Discount

Prior (D) – (C)

Semiannual Interest

Period-End

(E)

Carrying Value

$100,000 – (D)

Column (A) is the par value ($100,000) multiplied by the semiannual contract rate (4%). Column (B) is the prior period’s carrying value multiplied by the semiannual market rate (5.0155%). Column (C) is the difference between interest paid and bond interest expense, or [(B) – (A)]. Column (D) is the prior period’s unamortized discount less the current period’s discount amortization. Column (E) is the par value less unamortized discount, or [$100,000 – (D)].

June 30, 2020 Bond Interest Expense . . . . . . . . . . . . . . . . . . . . . . . . . . . . . . 4,835 Discount on Bonds Payable . . . . . . . . . . . . . . . . . . . . . . 835 Cash . . . . . . . . . . . . . . . . . . . . . . . . . . . . . . . . . . . . . . . . . 4,000

Record semiannual interest and discount amortization (effective interest method).

12 /31

/2 01

9

6/ 30

/2 02

0

12 /31

/2 02

0

6/ 30

/2 02

1

12 /31

/2 02

1

Carrying value

$96,400 $97,235

$98,112

$99,033

$100,000

Bonds Payable

12/31/2019 100,000 6/30/2020 — 12/31/2020 — 6/30/2021 — 12/31/2021 100,000

12/31/2021 0

Discount on Bonds Payable

12/31/2019 3,600 6/30/2020 835 12/31/2020 877 6/30/2021 921 12/31/2021 967

12/31/2021 0

Except for differences in amounts, journal entries recording the expense and updating the liability bal- ance are the same under the effective interest method and the straight-line method. We use the numbers in Exhibit 10B.1 to record each semiannual entry during the bonds’ two-year life (June 30, 2020, through December 31, 2021). The interest payment entry at the end of the first semiannual period is

P6 Compute and record amortization of a bond premium using the effective interest method.

398 Chapter 10 Accounting for Long-Term Liabilities

cash payment is based on the higher 6% semiannual contract rate. The excess cash payment over the inter- est expense reduces the principal. These amounts are shown in column C. Column E shows the carrying value after deducting the amortized premium in column C from the prior period’s carrying value. Column D shows the premium’s reduction by periodic amortization.

EXHIBIT 10B.2 Effective Interest Amortization of Bond Premium

June 30, 2020 Bond Interest Expense . . . . . . . . . . . . . . . . . . . . . . . . . . . . . . 5,165 Premium on Bonds Payable . . . . . . . . . . . . . . . . . . . . . . . . . . . 835 Cash . . . . . . . . . . . . . . . . . . . . . . . . . . . . . . . . . . . . . . . . . 6,000

Record semiannual interest and premium amortization (effective interest method).

Column (A) is the par value ($100,000) multiplied by the semiannual contract rate (6%). Column (B) is the prior period’s carrying value multiplied by the semiannual market rate (4.9851%). Column (C) is the difference between interest paid and bond interest expense, or [(A) – (B)]. Column (D) is the prior period’s unamortized premium less the current period’s premium amortization. Column (E) is the par value plus unamortized premium, or [$100,000 + (D)].

Semiannual Interest

Period-End

(4)

12/31/2019 6/30/2020 12/31/2020 6/30/2021

12/31/2021

(0)

(2) (3)

$ 6,000 6,000 6,000

6,000 $ 24,000

(A) Cash

Interest Paid

6% × $100,000

$ 5,165 5,123

5,079 5,033

$20,400

(B) Bond

Interest Expense

4.9851% × Prior (E)

$ 835 877

921 967

$3,600

(C)

Premium Amortization

(A) – (B)

2,765 $3,600

1,888

967 0

(D)

Unamortized Premium

Prior (D) – (C)

102,765 $103,600

101,888

100,967

100,000

(E)

Carrying Value

$100,000 + (D)

Bonds: $100,000 Par Value, Semiannual Interest Payments, Two-Year Life, 6% Semiannual Contract Rate, 4.9851% Semiannual Market Rate

( 1 )

$103,600

$102,765

$100,967

$100,000

$101,888

12 /31

/2 01

9

6/ 30

/2 02

0

12 /31

/2 02

0

6/ 30

/2 02

1

12 /31

/2 02

1

Carrying value

Bonds Payable

12/31/2019 100,000 6/30/2020 — 12/31/2020 — 6/30/2021 — 12/31/2021 100,000

12/31/2021 0

Premium on Bonds Payable

12/31/2019 3,600 6/30/2020 835 12/31/2020 877 6/30/2021 921 12/31/2021 967

12/31/2021 0

When the issuer makes the first semiannual interest payment, it records the following. Similar entries with different amounts are recorded at each payment date until the bond matures at the end of 2021. The effec- tive interest method yields decreasing amounts of bond interest expense and increasing amounts of pre- mium amortization over the bonds’ life.

APPENDIX

Leases and Pensions10C Lease Liabilities A lease is an agreement between a lessor (owner) and a lessee (renter or ten- ant) that gives the lessee the right to use the asset for a period of time in return for cash (rent) payments. The financing of leases is a $1 trillion industry. The advantages of lease financing include no up-front, full cash payment and the potential to deduct rental payments from taxable income. Leases are classified as either finance leases or operating leases. In either case, for noncurrent leases the lessee records a “Right-of-Use Asset” and “Lease Liability” equal to the present value of lease pay- ments. At each period-end, the lessee records financing expense differently depending on whether it’s a finance lease or operating lease.

Finance Leases Finance leases are long-term leases where the lessee receives substantially all remaining benefits of the asset. A finance lease meets one or more of five criteria: (1) transfers ownership of lease asset to lessee, (2) has a purchase option that lessee is reasonably certain to exercise, (3) lease term is for major part of the lease asset’s remaining economic life, (4) present value of lease payments equals or exceeds substantially all of the lease asset’s fair value, or (5) the lease asset is specialized and expected to have no alternative use to lessor at lease-end. A finance lease is similar to the financing of an asset purchase. Examples include most leases of air- planes, delivery trucks, medical equipment, railcars, and department store buildings. The lessee records

C3 Describe accounting for leases and pensions.

Chapter 10 Accounting for Long-Term Liabilities 399

the leased item as its own asset along with a lease liability at the start of the lease term; the amount re- corded equals the present value of all lease payments.

Lease Start and First Payment Assume KDI Co. enters into a three-year lease of a building in which it sells sporting equipment. The lease is accounted for as a finance lease, it requires three $21,000 payments (the first at the beginning of the lease and the others at December 31 of 2019 and 2020), and the present value of its annual lease payments is $60,000 (implying a 5.086% discount rate). KDI records the asset and liability along with the first-period lease payment as follows. KDI reports the right-of-use lease asset as a long-term asset and the lease liability as a long-term liability. The portion of the lease liability expected to be paid in the next year is reported as a current liability.

Lease Asset Amortization At each year-end, KDI records amortization on the right-of-use asset (assume straight-line amortization, three-year lease term, and no salvage value) as follows.

Lease Payment for Liability and Interest KDI accrues interest expense on the lease liability at each year- end. Interest expense is computed by multiplying the lease liability by the interest rate on the lease. It records interest expense as part of its $21,000 annual lease payment as follows (for its first year).

Jan . 1, 2019 Right-of-Use Asset . . . . . . . . . . . . . . . . . . . . . . . . . . . . . . . . . . . . . . 60,000

Lease Liability . . . . . . . . . . . . . . . . . . . . . . . . . . . . . . . . . . . . . 60,000

Record right-of-use asset and lease liability.

Jan . 1, 2019 Lease Liability . . . . . . . . . . . . . . . . . . . . . . . . . . . . . . . . . . . . . . . . . 21,000

Cash . . . . . . . . . . . . . . . . . . . . . . . . . . . . . . . . . . . . . . . . . . . . . 21,000

Record beginning-year cash lease payment.

Dec . 31, 2019 Amortization Expense . . . . . . . . . . . . . . . . . . . . . . . . . . . . . . . . . . . 20,000

Accumulated Amortization—Right-of-Use Asset . . . . . . . . . . 20,000

Record amortization on right-of-use asset. ($60,000−$0)∕3 yrs

Dec . 31, 2019 Interest Expense . . . . . . . . . . . . . . . . . . . . . . . . . . . . . . . . . . . . . . . 1,984

Lease Liability . . . . . . . . . . . . . . . . . . . . . . . . . . . . . . . . . . . . . . . . . 19,016

Cash . . . . . . . . . . . . . . . . . . . . . . . . . . . . . . . . . . . . . . . . . . . . . 21,000

Record lease payment for interest and lease liability.*

*Numbers are from a lease payment schedule as follows.

KDI’s entries for the final two years of this lease follow.

Payments

(A) (B) (C) (D) (E) Debit Debit Credit

Interest on + = Ending Balance Beginning Balance Lease Liability Lease Liability Cash Lease of Lease Liability Date of Lease Liability 5.086% × (A) (D) – (B) Payment (A) – (C)

Jan. 1, 2019 $60,000 $21,000 $21,000 $39,000 Dec 31, 2019 39,000 $1,984 19,016 21,000 19,984 Dec 31, 2020 19,984 1,016 19,984 21,000 0

$3,000 $60,000 $63,000

Dec . 31, 2020 Amortization Expense . . . . . . . . . . . . . . . . . . . . . . . . . . . . . . . . . 20,000 Accumulated Amortization—Right-of-Use Asset . . . . . . . 20,000 Record amortization on right-of-use asset. Dec . 31, 2020 Interest Expense . . . . . . . . . . . . . . . . . . . . . . . . . . . . . . . . . . . . . 1,016 Lease Liability . . . . . . . . . . . . . . . . . . . . . . . . . . . . . . . . . . . . . . . 19,984 Cash . . . . . . . . . . . . . . . . . . . . . . . . . . . . . . . . . . . . . . . . . . 21,000 Record lease payment for interest and lease liability. Dec . 31, 2021 Amortization Expense . . . . . . . . . . . . . . . . . . . . . . . . . . . . . . . . . 20,000 Accumulated Amortization—Right-of-Use Asset . . . . . . . 20,000 Record amortization on right-of-use asset.

Operating Leases Operating leases are long-term leases that do not meet any of the five cri- teria for finance leases.

Lease Start and Payments We prepare journal entries using the same lease payment schedule shown for the finance lease above. Recall this is a three-year lease that requires three $21,000 payments (the first at

400 Chapter 10 Accounting for Long-Term Liabilities

the beginning of the lease and the others at December 31 of 2019 and 2020), with a present value of its annual lease payments of $60,000 (implying a 5.086% discount rate). All entries under the finance lease apply here, but amounts for amortization entries differ.

Lease Amortization Total amortization for the lease life is the same for finance and operating leases. The difference is the yearly asset amortization. Those entries follow using the amortization calculated below.

Short-Term Leases Short-term leases have lease terms of 12 months or less and do not have long-term purchase options. Examples include most car and apartment rental agreements. The lessee records such lease payments as expenses. The lessee does not report the leased item as an asset or a liability (it is the lessor’s asset). If Verizon leases a kiosk from the mall for $300 per month, its entry follows.

2019 2020 2021

Dec . 31 Amortization Expense . . . . . . . . . . . . . . . . . . . . . . . . . . . . . 19,016 19,984 21,000

Accumulated Amortization—Right-of-Use Asset . . . . 19,016 19,984 21,000

Record amortization on right-of-use asset.*

Amortization* = Lease Payment − Interest on Lease Liability

For 2019 . . . $19,016 = $21,000 − $1,984 For 2020 . . . $19,984 = $21,000 − $1,016 For 2021 . . . $21,000 = $21,000 − $ 0

Point: In the income statement for an operating lease, Amortization Exp. and Interest Exp. are com- bined as one line item, “Lease Expense.” The balance sheet and ledger keep them separate.

July 4 Rental Expense . . . . . . . . . . . . . . . . . . . . . . . . . . . . . . . . . . . . . . . . 300

Cash . . . . . . . . . . . . . . . . . . . . . . . . . . . . . . . . . . . . . . . . . . . . . 300

Record short-term lease rental payment.

Pension Liabilities A pension plan is an agreement for the employer to provide benefits (pay- ments) to employees after they retire. Some employers pay the full cost of the pension, and some pay part of the cost. An employer records its payment into a pension plan with a debit to Pension Expense and a credit to Cash. A plan administrator invests the payments in pension assets and makes benefit payments to pension recipients (retired employees).

Defined Benefit Plan Defined benefit plans give workers defined future benefits; the employer’s contributions vary, depending on assumptions about future pension assets and liabilities. A pension liability is reported when the accumulated benefit obligation is more than the plan assets, called an underfunded plan. The accu- mulated benefit obligation is the present value of promised future pension payments to retirees. Plan assets refer to the market value of pension assets. A pension asset is reported when the accumulated benefit obliga- tion is less than the plan assets, called an overfunded plan. An employer reports pension expense when employees earn wages, which is sometimes decades before it pays pension benefits to employees.

Other Postretirement Benefits Other postretirement benefits refer to nonpension benefits such as health care and life insurance benefits. Costs of these benefits are estimated and liabilities accrued when the employees earn them. Many of these benefits are not funded.

Point: Fringe benefits are often 40% or more of salaries and wages, and pension benefits make up nearly 15% of fringe benefits.

Point: Two types of pension plans are (1) defined benefit plan—the retirement benefit is defined and the employer estimates the contribution necessary to pay these benefits—and (2) defined contribution plan—the pension contribution is defined and the employer and/or employee contribute amounts specified in the pension agreement.

BOND BASICS AND PAR BONDS Bond advantages: Bonds do not affect owner control, interest on bonds is tax deductible, and bonds can potentially increase return on equity. Bond disadvantages: Bonds can potentially decrease return on equity and require payments of both periodic interest and the par value at maturity.

Summary: Cheat Sheet

Bonds issued at par value (called par bonds):

Cash . . . . . . . . . . . . . . . . . . . . . . . . . . . . . . . . . . . . . . . . . 100,000

Bonds Payable . . . . . . . . . . . . . . . . . . . . . . . . . . . . . 100,000

Par bonds semiannual interest payment:

Maturity of bonds (payment of par): When the bond issuer pays the par value back to the bondholder.

Bond Interest Expense . . . . . . . . . . . . . . . . . . . . . . . . . . . 4,000

Cash . . . . . . . . . . . . . . . . . . . . . . . . . . . . . . . . . . . . . 4,000

Bonds Payable . . . . . . . . . . . . . . . . . . . . . . . . . . . . . . . . . 100,000

Cash . . . . . . . . . . . . . . . . . . . . . . . . . . . . . . . . . . . . . 100,000

Chapter 10 Accounting for Long-Term Liabilities 401

DISCOUNT BONDS Contract rate: The interest the bond issuer pays in cash. Market rate: The interest rate that borrowers are willing to pay and lenders are willing to accept.

Bond prices: A $1,000 bond with a price of 96.400 is sold for $964. A $1,000 bond with a price of 103½ is sold for $1,035. Carrying (book) value of a bond: Equals bond par value plus any pre- mium or minus any discount. Discount bonds: Bonds issued with a contract rate that is less than the market rate.

Contract rate > Market rate Bond sells at premium

Contract rate = Market rate Bond sells at par

Contract rate < Market rate Bond sells at discount

Issuance of discount bonds:

Cash . . . . . . . . . . . . . . . . . . . . . . . . . . . . . . . . . . . . . . . . . 96,400

Discount on Bonds Payable . . . . . . . . . . . . . . . . . . . . . . . 3,600

Bonds Payable . . . . . . . . . . . . . . . . . . . . . . . . . . . . . 100,000

PREMIUM BONDS Premium bonds: Bonds issued with a contract rate higher than the market rate.

Issuance of premium bonds:

Cash . . . . . . . . . . . . . . . . . . . . . . . . . . . . . . . . . . . . . . . . . 103,600

Premium on Bonds Payable . . . . . . . . . . . . . . . . . . 3,600

Bonds Payable . . . . . . . . . . . . . . . . . . . . . . . . . . . . . 100,000

Amortizing discount bonds (straight-line method):

Panel A: Interest Computations

Four payments of $4,000 (4 pymts × [$100,000 × 0 .08 × 1∕2 yr]) . . . . . $ 16,000

Plus discount . . . . . . . . . . . . . . . . . . . . . . . . . . . . . . . . . . . . . . . . . . . . . . . . 3,600 Total bond interest expense . . . . . . . . . . . . . . . . . . . . . . . . . . . . . . . . . . . $19,600

Bond interest expense (per interest period) =

Total bond interest expense Number of interest periods =

$19,600 4

= $4,900

Panel B: Entry to Record Interest Payment and Amortization

2020–2021 Bond Interest Expense . . . . . . . . . . . . . . . . 4,900 June 30 and Discount on Bonds Payable . . . . . . . 900 Dec . 31 Cash . . . . . . . . . . . . . . . . . . . . . . . . . . 4,000

Discount ÷ Periods

Par value × ½ × Contract rate

Straight-line discount bond amortization table:

Semiannual Unamortized Carrying Period-End Discount Value

(0) 12/31/2019 . . . . . . . . . . . $3,600 $ 96,400

(1) 6/30/2020 . . . . . . . . . . . 2,700 97,300

(2) 12/31/2020 . . . . . . . . . . . 1,800 98,200

(3) 6/30/2021 . . . . . . . . . . . 900 99,100

(4) 12/31/2021 . . . . . . . . . . . 0 100,000

Straight-line premium bond amortization table:

Semiannual Unamortized Carrying Period-End Premium Value

(0) 12/31/2019 . . . . . . . . . $3,600 $103,600

(1) 6/30/2020 . . . . . . . . . 2,700 102,700

(2) 12/31/2020 . . . . . . . . . 1,800 101,800

(3) 6/30/2021 . . . . . . . . . 900 100,900

(4) 12/31/2021 . . . . . . . . . 0 100,000

BOND RETIREMENT Bond retirement by call option: Some bonds give issuers an option to call the bonds before they mature by paying par value plus a call premium. Record a gain if carrying value is greater than retirement price (shown here). Record a loss if carrying value is less than retirement price.

Bonds Payable . . . . . . . . . . . . . . . . . . . . . . . . . . . . . . . . . 100,000

Premium on Bonds Payable . . . . . . . . . . . . . . . . . . . . . . . 4,500

Gain on Bond Retirement . . . . . . . . . . . . . . . . . . . . 1,500

Cash . . . . . . . . . . . . . . . . . . . . . . . . . . . . . . . . . . . . . 103,000

Bond retirement by conversion: Holders of convertible bonds can con- vert their bonds to stock. No gain or loss is recorded. Bonds are converted to stock at the bonds’ carrying value.

Bonds Payable . . . . . . . . . . . . . . . . . . . . . . . . . . . . . . . . . 100,000

Common Stock . . . . . . . . . . . . . . . . . . . . . . . . . . . . 30,000

Paid-In Capital in Excess of Par Value . . . . . . . . . . 70,000

Reporting of discount bonds:

Long-term liabilities

Bonds payable, 8%, due December 31, 2021 . . . . . . $100,000

Less discount on bonds payable . . . . . . . . . . . . . . . . . 3,600 $96,400

Reporting of premium bonds:

Long-term liabilities

Bonds payable, 12%, due December 31, 2021 . . . . . $100,000

Plus premium on bonds payable . . . . . . . . . . . . . . . . . 3,600 $103,600

Amortizing premium bonds (straight-line method):

Panel A: Interest Computations

Four payments of $6,000 (4 pymts × [$100,000 × 0 .12 × 1∕2 yr]) . . . . . $ 24,000 Less premium . . . . . . . . . . . . . . . . . . . . . . . . . . . . . . . . . . . . . . . . . . . . . . . (3,600)

Total bond interest expense . . . . . . . . . . . . . . . . . . . . . . . . . . . . . . . . . . . $20,400

Bond interest expense (per interest period) =

Total bond interest expense Number of interest periods =

$20,400 4

= $5,100

Panel B: Entry to Record Interest Payment and Amortization

2020–2021 Bond Interest Expense . . . . . . . . . . . . . . . . 5,100 June 30 and Premium on Bonds Payable . . . . . . . . . . . . 900 Dec . 31 Cash . . . . . . . . . . . . . . . . . . . . . . . . . . 6,000

Premium ÷ Periods

Par value × ½ × Contract rate

402 Chapter 10 Accounting for Long-Term Liabilities

Payments of principal and interest payments for note: Payments on an installment note include accrued interest expense plus part of the amount borrowed (the principal).

Issuance of notes:

Note installment payments:

Dec . 31, 2019 Interest Expense . . . . . . . . . . . . . . . . 4,800

Notes Payable . . . . . . . . . . . . . . . . . . 18,482

Cash . . . . . . . . . . . . . . . . . . . . . 23,282

Dec . 31, 2020 Interest Expense . . . . . . . . . . . . . . . . 3,321

Notes Payable . . . . . . . . . . . . . . . . . . 19,961

Cash . . . . . . . . . . . . . . . . . . . . . 23,282

Bearer bonds (392) Bond (381) Bond certificate (382) Bond indenture (382) Callable bonds (392) Carrying (book) value of bonds (384) Contract rate (383) Convertible bonds (392) Coupon bonds (392) Debt-to-equity ratio (392)

Discount on bonds payable (384) Effective interest method (397) Finance lease (398) Installment note (390) Lease (398) Market rate (383) Mortgage (391) Operating lease (399) Par value of a bond (381) Pension plan (400)

Premium on bonds (386) Registered bonds (392) Secured bonds (392) Serial bonds (392) Short-term lease (400) Sinking fund bonds (392) Straight-line bond amortization (385) Term bonds (392) Unsecured bonds (392)

Key Terms

Multiple Choice Quiz

1. A bond traded at 971⁄2 means that a. The bond pays 971⁄2% interest. b. The bond trades at $975 per $1,000 bond. c. The market rate of interest is below the contract rate of

interest for the bond. d. The bonds can be retired at $975 each. e. The bond’s interest rate is 21⁄2%.

2. A bondholder that owns a $1,000, 6%, 15-year (term) bond has a. The right to receive $1,000 at maturity. b. Ownership rights in the bond-issuing entity. c. The right to receive $60 per month until maturity. d. The right to receive $1,900 at maturity. e. The right to receive $600 per year until maturity.

3. A company issues 8%, 20-year bonds with a par value of $500,000. The current market rate for the bonds is 8%. The

amount of interest owed to the bondholders for each semi- annual interest payment is a. $40,000. c. $20,000. e. $400,000. b. $0. d. $800,000.

4. A company issued five-year, 5% bonds with a par value of $100,000. The company received $95,735 for the bonds. Using the straight-line method, the company’s interest ex- pense for the first semiannual interest period is a. $2,926.50. c. $2,500.00. e. $9,573.50. b. $5,853.00. d. $5,000.00.

5. A company issued eight-year, 5% bonds with a par value of $350,000. The company received proceeds of $373,745. Interest is payable semiannually. The amount of premium amortized for the first semiannual interest period, assuming straight-line bond amortization, is a. $2,698. c. $8,750. e. $1,484. b. $23,745. d. $9,344.

ANSWERS TO MULTIPLE CHOICE QUIZ

1. b 2. a 3. c; $500,000 × 0.08 × 1⁄2 year = $20,000

4. a; Cash interest paid = $100,000 × 5% × 1⁄2 year = $2,500 Discount amortization = ($100,000 − $95,735)∕10 periods = $426.50

Interest expense = $2,500.00 + $426.50 = $2,926.50 5. e; ($373,745 − $350,000)∕16 periods = $1,484

Payments

(A) (B) (C) (D) (E) Debit Debit Credit

Beginning Balance

Interest Notes Ending

Period Ending Date Expense + Payable = Cash Balance 8% × (A) (D) – (B) (computed) (A) – (C)

(1 ) 12/31/2019 . . . . . . . . $60,000 $41,518

(2) 12/31/2020 . . . . . . . . . 41,518 21,557

(3) 12/31/2021 . . . . . . . . 21,557 0

$4,800 $ 18,482 $23,282

3,321 19,961 23,282

1,725 21,557 23,282

$9,846 $60,000 $69,846

LONG-TERM NOTES Installment note: A liability requiring a series of payments to the lender. Usually issued to a single lender, such as a bank. Cash . . . . . . . . . . . . . . . . . . . . . . . . . . 60,000

Notes Payable . . . . . . . . . . . . . . 60,000

Chapter 10 Accounting for Long-Term Liabilities 403

A(B,C) Superscript letter A, B, or C denotes assignments based on Appendix 10A, 10B, or 10C.

Icon denotes assignments that involve decision making.

1. What is the main difference between notes payable and bonds payable?

2. What is the main difference between a bond and a share of stock?

3. What is the advantage of issuing bonds instead of ob- taining financing from the company’s owners?

4. What is a bond indenture? What provisions are usually in- cluded in it?

5. What are the contract rate and the market rate for bonds? 6. What factors affect the market rates for bonds? 7.B Does the straight-line or effective interest method pro-

duce an interest expense allocation that yields a constant rate of interest over a bond’s life? Explain.

8. Explain the concept of accrued interest on bonds at the end of an accounting period.

9. If you know the par value of bonds, the contract rate, and the market rate, how do you compute the bonds’ price?

10. What is the issue price of a $2,000 bond sold at 981⁄4? What is the issue price of a $6,000 bond sold at 1011⁄2?

11. Describe the debt-to-equity ratio and explain how creditors and owners use this ratio to evaluate a company’s risk.

12. What obligation does an entrepreneur (owner) have to investors that purchase bonds to finance the business?

13. Refer to Apple’s annual report in Appendix A. Is there any indication that Apple has issued long-term debt?

14. Refer to the statements for Samsung in Appendix A. By what amount did Samsung’s long-term borrowings increase or decrease in 2017?

15. Refer to the statement of cash flows for Samsung in Appendix A. For the year ended December 31, 2017, what was the amount for repay- ment of long-term borrowings and debentures?

16. Refer to the statements for Google in Appendix A. For the year ended December 31, 2017, what was its debt-to-equity ratio? What does this ratio tell us?

17.C When can a lease create both an asset and a liability for the lessee?

18.C Compare and contrast a finance lease with an operating lease. 19.C Describe the two basic types of pension plans.

Discussion Questions

APPLE

Samsung

Samsung

GOOGLE

QUICK STUDY

QS 10-1 Advantages of bond financing

A1

Identify the following as either an advantage (A) or a disadvantage (D) of bond financing for a company. a. Bonds do not affect owner control. b. A company earns a lower return with borrowed funds than it pays in interest. c. A company earns a higher return with borrowed funds than it pays in interest. d. Bonds require payment of periodic interest. e. Interest on bonds is tax deductible. f. Bonds require payment of par value at maturity.

Round dollar amounts to the nearest whole dollar for all assignments in this chapter.

QS 10-2 Issuing bonds at par P1

Dunphy Company issued $10,000 of 6%, 10-year bonds at par value on January 1. Interest is paid semian- nually each June 30 and December 31. Prepare the entries for (a) the issuance of the bonds and (b) the first interest payment on June 30.

QS 10-3 Issuing bonds at par

P1

Madrid Company plans to issue 8% bonds with a par value of $4,000,000. The company sells $3,600,000 of the bonds at par on January 1. The remaining $400,000 sells at par on July 1. The bonds pay interest semiannually on June 30 and December 31. 1. Record the entry for the first interest payment on June 30. 2. Record the entry for the July 1 cash sale of bonds.

On January 1, Renewable Energy issues bonds that have a $20,000 par value, mature in eight years, and pay 12% interest semiannually on June 30 and December 31. 1. Prepare the journal entry for issuance assuming the bonds are issued at (a) 99 and (b) 1031⁄2. 2. How much interest does the company pay (in cash) to its bondholders every six months if the bonds

are sold at par?

QS 10-4 Recording bond issuance and interest

P1 P2 P3

QS 10-5 Journalizing discount bond issuance P2

Enviro Company issues 8%, 10-year bonds with a par value of $250,000 and semiannual interest pay- ments. On the issue date, the annual market rate for these bonds is 10%, which implies a selling price of 871⁄2. Prepare the journal entry for the issuance of the bonds for cash on January 1.

404 Chapter 10 Accounting for Long-Term Liabilities

QS 10-6 Journalizing premium bond issuance P3

Garcia Company issues 10%, 15-year bonds with a par value of $240,000 and semiannual interest pay- ments. On the issue date, the annual market rate for these bonds is 8%, which implies a selling price of 1171⁄4. Prepare the journal entry for the issuance of these bonds for cash on January 1.

QS 10-7 Straight-Line: Discount bond computations

P2

Enviro Company issues 8%, 10-year bonds with a par value of $250,000 and semiannual interest pay- ments. On the issue date, the annual market rate for these bonds is 10%, which implies a selling price of 871⁄2. The straight-line method is used to allocate interest expense. 1. What are the issuer’s cash proceeds from issuance of these bonds? 2. What total amount of bond interest expense will be recognized over the life of these bonds? 3. What is the amount of bond interest expense recorded on the first interest payment date?

QS 10-8 Recording bond issuance and discount amortization

P2

Snap Company issues 10%, five-year bonds, on January 1 of this year, with a par value of $100,000 and semiannual interest payments. Use the following bond amortization table and prepare journal entries to record (a) the issuance of bonds on January 1, (b) the first interest payment on June 30, and (c) the second interest payment on December 31.

Semiannual Period-End Unamortized Discount Carrying Value

(0) January 1, issuance . . . . . . . . . . . . . . $7,360 $92,640

(1) June 30, first payment . . . . . . . . . . . . 6,624 93,376

(2) December 31, second payment . . . . . 5,888 94,112

QS 10-9 Straight-Line: Premium bond computations

P3

Enviro Company issues 8%, 10-year bonds with a par value of $250,000 and semiannual interest pay- ments. On the issue date, the annual market rate for these bonds is 5%, which implies a selling price of 123.375. The straight-line method is used to allocate interest expense. 1. What are the issuer’s cash proceeds from issuance of these bonds? 2. What total amount of bond interest expense will be recognized over the life of these bonds? 3. What is the amount of bond interest expense recorded on the first interest payment date?

QS 10-10 Bond retirement by call option P4

On July 1, Aloha Co. exercises a call option that requires Aloha to pay $408,000 for its outstanding bonds that have a carrying value of $416,000 and a par value of $400,000. The company exercises the call option after the semiannual interest is paid the day before on June 30. Record the entry to retire the bonds.

QS 10-11 Bond retirement by stock conversion P4

On January 1, the $3,000,000 par value bonds of Spitz Company with a carrying value of $3,000,000 are converted to 1,000,000 shares of $1 par value common stock. Record the entry for the conversion of the bonds.

QS 10-12 Issuance and interest for installment note

C1

On January 1, MM Co. borrows $340,000 cash from a bank and in return signs an 8% installment note for five annual payments of $85,155 each. 1. Prepare the journal entry to record issuance of the note. 2. For the first $85,155 annual payment at December 31, what amount goes toward interest expense?

What amount goes toward principal reduction of the note?

QS 10-13 Bond features and terminology

A2

Select the description that best fits each term or phrase. A. Records and tracks the bondholders’ names. B. Is unsecured; backed only by the issuer’s credit standing. C. Has varying maturity dates for amounts owed. D. The legal contract between the issuer and the bondholders. E. Can be exchanged for shares of the issuer’s stock. F. Is unregistered; interest is paid to whoever possesses them. G. Maintains a separate asset account from which bondholders are paid at maturity. H. Pledges specific assets of the issuer as collateral.

1. Registered bond 5. Convertible bond 2. Serial bond 6. Bond indenture 3. Secured bond 7. Sinking fund bond 4. Bearer bond 8. Debenture

Chapter 10 Accounting for Long-Term Liabilities 405

QS 10-15A Computing bond price C2

Compute the selling price of 8%, 10-year bonds with a par value of $250,000 and semiannual interest pay- ments. The annual market rate for these bonds is 10%. Use present value tables B.1 and B.3 in Appendix B.

QS 10-16A Computing bond price C2

Compute the selling price of 10%, 15-year bonds with a par value of $240,000 and semiannual interest pay- ments. The annual market rate for these bonds is 8%. Use present value tables B.1 and B.3 in Appendix B.

QS 10-17B Effective Interest: Bond discount computations

P5

Garcia Company issues 10%, 15-year bonds with a par value of $240,000 and semiannual interest pay- ments. On the issue date, the annual market rate for these bonds is 14%, which implies a selling price of 751⁄4. The effective interest method is used to allocate interest expense. 1. What are the issuer’s cash proceeds from issuance of these bonds? 2. What total amount of bond interest expense will be recognized over the life of these bonds? 3. What amount of bond interest expense is recorded on the first interest payment date?

QS 10-18B Effective Interest: Bond premium computations

P6

Garcia Company issues 10%, 15-year bonds with a par value of $240,000 and semiannual interest pay- ments. On the issue date, the annual market rate for these bonds is 8%, which implies a selling price of 1171⁄4. The effective interest method is used to allocate interest expense. 1. What are the issuer’s cash proceeds from issuance of these bonds? 2. What total amount of bond interest expense will be recognized over the life of these bonds? 3. What amount of bond interest expense is recorded on the first interest payment date?

QS 10-19C Recording short-term leases C3

Jin Li, an employee of ETrain.com, leases a car at O’Hare Airport for a three-day business trip. The rental cost is $250. Prepare the entry by ETrain.com to record Jin Li’s short-term car lease cost.

Exercise 10-2 Recording bond issuance at par, interest payments, and bond maturity

P1

Brussels Enterprises issues bonds at par dated January 1, 2019, that have a $3,400,000 par value, mature in four years, and pay 9% interest semiannually on June 30 and December 31. 1. Record the entry for the issuance of bonds for cash on January 1. 2. Record the entry for the first semiannual interest payment and the second semiannual interest payment. 3. Record the entry for the maturity of the bonds on December 31, 2022 (assume semiannual interest is

already recorded).

QS 10-14 Debt-to-equity ratio

A3

Compute the debt-to-equity ratio for each of the following companies. Which company appears to have a riskier financing structure?

Atlanta Company Spokane Company

Total liabilities . . . . . . . . . . . $429,000 $ 549,000

Total equity . . . . . . . . . . . . . 572,000 1,830,000

QS 10-20C Recording leases C3

Algoma, Inc., signs a five-year lease for office equipment with Office Solutions. The present value of the lease payments is $15,499. Prepare the journal entry that Algoma records at the inception of this finance lease.

No-Toxic-Toys currently has $200,000 of equity and is planning an $80,000 expansion to meet increasing demand for its product. The company currently earns $50,000 in net income, and the expansion will yield $25,000 in additional income before any interest expense.

The company has three options: (1) do not expand, (2) expand and issue $80,000 in debt that requires payments of 8% annual interest, or (3) expand and raise $80,000 from equity financing. For each option, compute (a) net income and (b) return on equity (Net income ÷ Equity). Ignore any income tax effects.

EXERCISES Exercise 10-1 Debt versus equity financing

A1

406 Chapter 10 Accounting for Long-Term Liabilities

Exercise 10-3 Recording bond issuance and interest

P1

On January 1, Boston Enterprises issues bonds that have a $3,400,000 par value, mature in 20 years, and pay 9% interest semiannually on June 30 and December 31. The bonds are sold at par. 1. How much interest will Boston pay (in cash) to the bondholders every six months? 2. Prepare journal entries to record (a) the issuance of bonds on January 1, (b) the first interest payment

on June 30, and (c) the second interest payment on December 31. 3. Prepare the journal entry for issuance assuming the bonds are issued at (a) 98 and (b) 102.

Exercise 10-4 Straight-Line: Amortization of bond discount

P2

Tano Company issues bonds with a par value of $180,000 on January 1, 2019. The bonds’ annual contract rate is 8%, and interest is paid semiannually on June 30 and December 31. The bonds mature in three years. The annual market rate at the date of issuance is 10%, and the bonds are sold for $170,862. 1. What is the amount of the discount on these bonds at issuance? 2. How much total bond interest expense will be recognized over the life of these bonds? 3. Prepare a straight-line amortization table like Exhibit 10.7 for these bonds.

Exercise 10-7 Straight-Line: Amortization table and bond interest expense

P2

Duval Co. issues four-year bonds with a $100,000 par value on January 1, 2019, at a price of $95,952. The annual contract rate is 7%, and interest is paid semiannually on June 30 and December 31. 1. Prepare a straight-line amortization table like Exhibit 10.7 for these bonds. 2. Prepare journal entries to record the first two interest payments. 3. Prepare the journal entry for maturity of the bonds on December 31, 2022 (assume semiannual interest

is already recorded).

Exercise 10-5 Straight-Line: Recording bond issuance and discount amortization

P2

Paulson Company issues 6%, four-year bonds, on January 1 of this year, with a par value of $200,000 and semiannual interest payments. Use the following bond amortization table and prepare journal entries to record (a) the issuance of bonds on January 1, (b) the first interest payment on June 30, and (c) the second interest payment on December 31.

Semiannual Period-End Unamortized Discount Carrying Value

(0) January 1, issuance . . . . . . . . . . . . . $13,466 $186,534

(1) June 30, first payment . . . . . . . . . . . 11,782 188,218

(2) December 31, second payment . . . 10,098 189,902

Exercise 10-6 Straight-Line: Recording bond issuance and discount amortization

P2

Dobbs Company issues 5%, two-year bonds, on December 31, 2019, with a par value of $200,000 and semiannual interest payments. Use the following bond amortization table and prepare journal entries to record (a) the issuance of bonds on December 31, 2019; (b) the first through fourth interest payments on each June 30 and December 31; and (c) the maturity of the bonds on December 31, 2021.

Semiannual Period-End Unamortized Discount Carrying Value

(0) 12/31/2019 . . . . . . . . . . . . . . . . . . . $12,000 $188,000

(1) 6/30/2020 . . . . . . . . . . . . . . . . . . . 9,000 191,000

(2) 12/31/2020 . . . . . . . . . . . . . . . . . . . 6,000 194,000

(3) 6/30/2021 . . . . . . . . . . . . . . . . . . . 3,000 197,000

(4) 12/31/2021 . . . . . . . . . . . . . . . . . . . 0 200,000

Exercise 10-8 Straight-Line: Recording bond issuance and premium amortization

P3

Wookie Company issues 10%, five-year bonds, on January 1 of this year, with a par value of $200,000 and semiannual interest payments. Use the following bond amortization table and prepare journal entries to record (a) the issuance of bonds on January 1, (b) the first interest payment on June 30, and (c) the second interest payment on December 31.

Semiannual Period-End Unamortized Premium Carrying Value

(0) January 1, issuance . . . . . . . . . . . . . $16,222 $216,222

(1) June 30, first payment . . . . . . . . . . . 14,600 214,600

(2) December 31, second payment . . . 12,978 212,978

Chapter 10 Accounting for Long-Term Liabilities 407

Exercise 10-9 Straight-Line: Amortization of bond premium

P3

Quatro Co. issues bonds dated January 1, 2019, with a par value of $400,000. The bonds’ annual contract rate is 13%, and interest is paid semiannually on June 30 and December 31. The bonds mature in three years. The annual market rate at the date of issuance is 12%, and the bonds are sold for $409,850. 1. What is the amount of the premium on these bonds at issuance? 2. How much total bond interest expense will be recognized over the life of these bonds? 3. Prepare a straight-line amortization table like Exhibit 10.11 for these bonds.

Exercise 10-10 Bond retirement by call option

P4

Tyrell Company issued callable bonds with a par value of $10,000. The call option requires Tyrell to pay a call premium of $500 plus par (or a total of $10,500) to bondholders to retire the bonds. On July 1, Tyrell exercises the call option. The call option is exercised after the semiannual interest is paid the day before on June 30. Record the entry to retire the bonds under each separate situation. 1. The bonds have a carrying value of $9,000. 2. The bonds have a carrying value of $11,000.

Exercise 10-12 Installment note amortization table C1

On January 1, 2019, Eagle Company borrows $100,000 cash by signing a four-year, 7% installment note. The note requires four equal payments of $29,523, consisting of accrued interest and principal on December 31 of each year from 2019 through 2022. Prepare an amortization table for this installment note like the one in Exhibit 10.12.

Exercise 10-13 Installment note entries

C1

Use the information in Exercise 10-12 to prepare the journal entries for Eagle to record the note’s issuance and each of the four payments.

Exercise 10-11 Straight-Line: Bond computations, amortization, and bond retirement

P2 P4

On January 1, 2019, Shay Company issues $700,000 of 10%, 15-year bonds. The bonds sell for $684,250. Six years later, on January 1, 2025, Shay retires these bonds by buying them on the open market for $731,500. All interest is accounted for and paid through December 31, 2024, the day before the purchase. The straight-line method is used to amortize any bond discount. 1. What is the amount of the discount on the bonds at issuance? 2. How much amortization of the discount is recorded on the bonds for the entire period from January 1,

2019, through December 31, 2024? 3. What is the carrying (book) value of the bonds as of the close of business on December 31, 2024? 4. Prepare the journal entry to record the bond retirement.

Exercise 10-14 Reporting liabilities section of balance sheet

C1 P2

Selected accounts from WooHoo Co.’s adjusted trial balance for the year ended December 31 follow. Prepare the liabilities section of its classified balance sheet.

Notes payable (due in 5 years) . . . . . . . . . . . $ 3,000 Discount on bonds payable . . . . . . . . . . . . . . $400

Accounts payable . . . . . . . . . . . . . . . . . . . . . . 500 Wages payable . . . . . . . . . . . . . . . . . . . . . . . . 200

Bonds payable (due in 10 years) . . . . . . . . . . 10,000 Interest payable (due in 2 weeks) . . . . . . . . . 100

Machinery . . . . . . . . . . . . . . . . . . . . . . . . . . . . 4,500 Sales tax payable . . . . . . . . . . . . . . . . . . . . . . 50

Exercise 10-15 Applying debt-to- equity ratio

A3

Montclair Company is considering a project that will require a $500,000 loan. It presently has total liabil- ities of $220,000 and total assets of $620,000. 1. Compute Montclair’s (a) current debt-to-equity ratio and (b) the debt-to-equity ratio assuming it bor-

rows $500,000 to fund the project. 2. If Montclair borrows the funds, does its financing structure become more or less risky?

Exercise 10-16A Computing bond interest and price; recording bond issuance C2

Bringham Company issues bonds with a par value of $800,000. The bonds mature in 10 years and pay 6% annual interest in semiannual payments. The annual market rate for the bonds is 8%. 1. Compute the price of the bonds as of their issue date. 2. Prepare the journal entry to record the bonds’ issuance.

408 Chapter 10 Accounting for Long-Term Liabilities

Exercise 10-17A Computing bond interest and price; recording bond issuance C2

Citywide Company issues bonds with a par value of $150,000. The bonds mature in five years and pay 10% annual interest in semiannual payments. The annual market rate for the bonds is 8%. 1. Compute the price of the bonds as of their issue date. 2. Prepare the journal entry to record the bonds’ issuance.

Exercise 10-18B Effective Interest: Amortization of bond discount

P5

Stanford issues bonds dated January 1, 2019, with a par value of $500,000. The bonds’ annual contract rate is 9%, and interest is paid semiannually on June 30 and December 31. The bonds mature in three years. The annual market rate at the date of issuance is 12%, and the bonds are sold for $463,140. 1. What is the amount of the discount on these bonds at issuance? 2. How much total bond interest expense will be recognized over the life of these bonds? 3. Prepare an effective interest amortization table like Exhibit 10B.1 for these bonds.

Exercise 10-19B Effective Interest: Amortization of bond premium

P6

Quatro Co. issues bonds dated January 1, 2019, with a par value of $400,000. The bonds’ annual contract rate is 13%, and interest is paid semiannually on June 30 and December 31. The bonds mature in three years. The annual market rate at the date of issuance is 12%, and the bonds are sold for $409,850. 1. What is the amount of the premium on these bonds at issuance? 2. How much total bond interest expense will be recognized over the life of these bonds? 3. Prepare an effective interest amortization table like Exhibit 10B.2 for these bonds.

Exercise 10-20C Identifying finance and operating leases

C3

In each of the following separate cases, indicate whether the company has entered into a finance lease or an operating lease.

1. The lessor retains title to the asset, and the lease term is 3 years on an asset that has a 10-year useful life.

2. The title is transferred to the lessee. The lessee can purchase the asset for $1 at the end of the lease, and the lease term is five years. The leased asset has an expected useful life of six years.

3. The present value of the lease payments is 95% of the leased asset’s market value, and the lease term is 90% of the leased asset’s useful life.

Exercise 10-21C Accounting for finance lease

C3

On January 1, Harbor (lessee) signs a five-year lease for equipment that is accounted for as a finance lease. The lease requires five $10,000 lease payments (the first at the beginning of the lease and the remaining four at December 31 of years 1, 2, 3, and 4), and the present value of the five annual lease pay- ments is $41,000, based on an 11% interest rate. 1. Prepare the January 1 journal entry Harbor records at inception of the lease for any asset or liability. 2. Prepare the January 1 entry Harbor records for the first $10,000 cash lease payment. 3. If the leased asset has a five-year useful life with no salvage value, prepare the December 31 journal

entry Harbor records each year for amortization of the leased asset.

Exercise 10-22C Analyzing lease purchase options

C3

General Motors advertised three alternatives for a 25-month lease on a new Tahoe: (1) zero dollars down and a lease payment of $1,750 per month for 25 months, (2) $5,000 down and $1,500 per month for 25 months, or (3) $38,500 down and no payments for 25 months. Use the present value Table B.3 in Appendix B to determine which is the best alternative for the customer (assume you have enough cash to accept any alternative and the annual interest rate is 12% compounded monthly).

PROBLEM SET A

Problem 10-1A Straight-Line: Amortization of bond discount

P2

Hillside issues $4,000,000 of 6%, 15-year bonds dated January 1, 2019, that pay interest semiannually on June 30 and December 31. The bonds are issued at a price of $3,456,448.

Required

1. Prepare the January 1 journal entry to record the bonds’ issuance. 2. For each semiannual period, compute (a) the cash payment, (b) the straight-line discount amortization,

and (c) the bond interest expense. 3. Determine the total bond interest expense to be recognized over the bonds’ life. 4. Prepare the first two years of a straight-line amortization table like Exhibit 10.7. 5. Prepare the journal entries to record the first two interest payments.

Check (3) $4,143,552 (4) 12/31/2020 carrying value, $3,528,920

Chapter 10 Accounting for Long-Term Liabilities 409

Problem 10-2A Straight Line: Amortization of bond premium

P3

Refer to the bond details in Problem 10-1A, except assume that the bonds are issued at a price of $4,895,980.

Required

1. Prepare the January 1 journal entry to record the bonds’ issuance. 2. For each semiannual period, compute (a) the cash payment, (b) the straight-line premium amortiza-

tion, and (c) the bond interest expense. 3. Determine the total bond interest expense to be recognized over the bonds’ life. 4. Prepare the first two years of a straight-line amortization table like Exhibit 10.11. 5. Prepare the journal entries to record the first two interest payments.

Check (3) $2,704,020 (4) 12/31/2020 carrying value, $4,776,516

Ellis Company issues 6.5%, five-year bonds dated January 1, 2019, with a $250,000 par value. The bonds pay interest on June 30 and December 31 and are issued at a price of $255,333. The annual market rate is 6% on the issue date.

Required

1. Calculate the total bond interest expense over the bonds’ life. 2. Prepare a straight-line amortization table like Exhibit 10.11 for the bonds’ life. 3. Prepare the journal entries to record the first two interest payments.

Check (2) 6/30/2021 carrying value, $252,668

Problem 10-3A Straight-Line: Amortization of bond premium

P3

Legacy issues $325,000 of 5%, four-year bonds dated January 1, 2019, that pay interest semiannually on June 30 and December 31. They are issued at $292,181 when the market rate is 8%.

Required

1. Prepare the January 1 journal entry to record the bonds’ issuance. 2. Determine the total bond interest expense to be recognized over the bonds’ life. 3. Prepare a straight-line amortization table like the one in Exhibit 10.7 for the bonds’ first two years. 4. Prepare the journal entries to record the first two interest payments.

Problem 10-4A Straight-Line: Amortization of bond discount P2

Check (2) $97,819

(3) 12/31/2020 carrying value, $308,589

On November 1, 2019, Norwood borrows $200,000 cash from a bank by signing a five-year installment note bearing 8% interest. The note requires equal payments of $50,091 each year on October 31.

Required

1. Complete an amortization table for this installment note similar to the one in Exhibit 10.12. 2. Prepare the journal entries in which Norwood records (a) accrued interest as of December 31, 2019

(the end of its annual reporting period), and (b) the first annual payment on the note.

Problem 10-5A Installment notes

C1

Check (1) 10/31/2023 ending balance, $46,382

Hartford Research issues bonds dated January 1 that pay interest semiannually on June 30 and December 31. The bonds have a $40,000 par value and an annual contract rate of 10%, and they mature in 10 years.

Required

For each separate situation, (a) determine the bonds’ issue price on January 1 and (b) prepare the journal entry to record their issuance. 1. The market rate at the date of issuance is 8%. 2. The market rate at the date of issuance is 10%. 3. The market rate at the date of issuance is 12%.

Problem 10-7AA Computing bond price and recording issuance

C2

Check (1) Premium, $5,437

(3) Discount, $4,588

At the end of the current year, the following information is available for both Pulaski Company and Scott Company.

Required

1. Compute the debt-to-equity ratios for both companies. 2. Which company has the riskier financing structure?

Pulaski Company Scott Company

Total assets . . . . . . $860,000 $440,000

Total liabilities . . . . 360,000 240,000

Total equity . . . . . . 500,000 200,000

Problem 10-6A Applying the debt-to- equity ratio

A3

410 Chapter 10 Accounting for Long-Term Liabilities

Problem 10-8AB Effective Interest: Amortization of bond discount P5

Refer to the bond details in Problem 10-4A.

Required

1. Prepare the January 1 journal entry to record the bonds’ issuance. 2. Determine the total bond interest expense to be recognized over the bonds’ life. 3. Prepare an effective interest amortization table like the one in Exhibit 10B.1 for the bonds’ first two years. 4. Prepare the journal entries to record the first two interest payments.

Check (2) $97,819

(3) 12/31/2020 carrying value, $307,308

Problem 10-9AB Effective Interest: Amortization of bond premium P6

Refer to the bond details in Problem 10-3A.

Required

1. Compute the total bond interest expense over the bonds’ life. 2. Prepare an effective interest amortization table like the one in Exhibit 10B.2 for the bonds’ life. 3. Prepare the journal entries to record the first two interest payments.

Check (2) 6/30/2021 carrying value, $252,865

Problem 10-10AB Effective Interest: Amortization of bond

P6

Ike issues $180,000 of 11%, three-year bonds dated January 1, 2019, that pay interest semiannually on June 30 and December 31. They are issued at $184,566 when the market rate is 10%.

Required

1. Prepare the January 1 journal entry to record the bonds’ issuance. 2. Determine the total bond interest expense to be recognized over the bonds’ life. 3. Prepare an effective interest amortization table like Exhibit 10B.2 for the bonds’ first two years. 4. Prepare the journal entries to record the first two interest payments.

Check (3) 6/30/2020 carrying value, $182,448

Problem 10-12AC Accounting for operating lease

C3

Refer to the lease details in Problem 10-11A. Assume that this lease is classified as an operating lease instead of a finance lease.

Required

1. Prepare the January 1 journal entry at the start of the lease to record any asset or liability. 2. Prepare the January 1 journal entry to record the first $18,000 cash lease payment. 3. Prepare the December 31 journal entry to record amortization at the end of (a) Year 1, (b) Year 2, and

(c) Year 3. 4. Prepare the December 31 journal entry to record the $18,000 cash lease payment at the end of (a) Year 1

and (b) Year 2.

Problem 10-11AC Accounting for finance lease

C3

On January 1, Rogers (lessee) signs a three-year lease for machinery that is accounted for as a finance lease. The lease requires three $18,000 lease payments (the first at the beginning of the lease and the remaining two at December 31 of Year 1 and Year 2). The present value of the three annual lease payments is $51,000, using a 6.003% interest rate. The lease payment schedule follows.

Payments (A) (B) (C) (D) (E) Debit Debit Credit Interest on Ending Balance Beginning Balance Lease Liability + Lease Liability = Cash Lease of Lease Liability Date of Lease Liability 6.003% × (A) (D) − (B) Payment (A) − (C) Jan. 1, Year 1 . . . . . $51,000 $18,000 $18,000 $33,000 Dec. 31, Year 1. . . . 33,000 $1,981 16,019 18,000 16,981 Dec. 31, Year 2 . . . 16,981 1,019 16,981 18,000 0 $3,000 $51,000 $54,000

Required

1. Prepare the January 1 journal entry at the start of the lease to record any asset or liability. 2. Prepare the January 1 journal entry to record the first $18,000 cash lease payment. 3. Prepare the December 31 journal entry to record straight-line amortization with zero salvage value at

the end of (a) Year 1, (b) Year 2, and (c) Year 3. 4. Prepare the December 31 journal entry to record the $18,000 cash lease payment at the end of (a) Year 1

and (b) Year 2.

Chapter 10 Accounting for Long-Term Liabilities 411

PROBLEM SET B

Problem 10-1B Straight-Line: Amortization of bond discount

P2

Romero issues $3,400,000 of 10%, 10-year bonds dated January 1, 2019, that pay interest semiannually on June 30 and December 31. The bonds are issued at a price of $3,010,000.

Required

1. Prepare the January 1 journal entry to record the bonds’ issuance. 2. For each semiannual period, compute (a) the cash payment, (b) the straight-line discount amortization,

and (c) the bond interest expense. 3. Determine the total bond interest expense to be recognized over the bonds’ life. 4. Prepare the first two years of a straight-line amortization table like Exhibit 10.7. 5. Prepare the journal entries to record the first two interest payments.

Check (3) $3,790,000 (4) 6/30/2020 carrying value, $3,068,500

Problem 10-2B Straight-Line: Amortization of bond premium

P3

Refer to the bond details in Problem 10-1B, except assume that the bonds are issued at a price of $4,192,932.

Required

1. Prepare the January 1 journal entry to record the bonds’ issuance. 2. For each semiannual period, compute (a) the cash payment, (b) the straight-line premium amortiza-

tion, and (c) the bond interest expense. 3. Determine the total bond interest expense to be recognized over the bonds’ life. 4. Prepare the first two years of a straight-line amortization table like Exhibit 10.11. 5. Prepare the journal entries to record the first two interest payments.

Check (3) $2,607,068

(4) 6/30/2020 carrying value, $4,073,991

Problem 10-3B Straight-Line: Amortization of bond premium

P3

Ripkin Company issues 9%, five-year bonds dated January 1, 2019, with a $320,000 par value. The bonds pay interest on June 30 and December 31 and are issued at a price of $332,988. Their annual market rate is 8% on the issue date.

Required

1. Calculate the total bond interest expense over the bonds’ life. 2. Prepare a straight-line amortization table like Exhibit 10.11 for the bonds’ life. 3. Prepare the journal entries to record the first two interest payments.

Check (2) 6/30/2021 carrying value, $326,493

Problem 10-4B Straight-Line: Amortization of bond discount

P2

Gomez issues $240,000 of 6%, 15-year bonds dated January 1, 2019, that pay interest semiannually on June 30 and December 31. They are issued at $198,494 when the market rate is 8%.

Required

1. Prepare the January 1 journal entry to record the bonds’ issuance. 2. Determine the total bond interest expense to be recognized over the life of the bonds. 3. Prepare a straight-line amortization table like the one in Exhibit 10.7 for the bonds’ first two years. 4. Prepare the journal entries to record the first two interest payments.

Analysis Component

5. Assume the market rate at issuance is 4% instead of 8%. Without providing numbers, describe how this change affects the amounts reported on Gomez’s financial statements.

Check (2) $257,506

(3) 6/30/2020 carrying value, $202,646

Problem 10-5B Installment notes

C1

On October 1, 2019, Gordon borrows $150,000 cash from a bank by signing a three-year installment note bearing 10% interest. The note requires equal payments of $60,316 each year on September 30.

Required

1. Complete an amortization table for this installment note similar to the one in Exhibit 10.12. 2. Prepare the journal entries to record (a) accrued interest as of December 31, 2019 (the end of its

annual reporting period), and (b) the first annual payment on the note.

Check (1) 9/30/2021 ending balance, $54,836

412 Chapter 10 Accounting for Long-Term Liabilities

At the end of the current year, the following information is available for both Atlas Company and Bryan Company.

Problem 10-6B Applying the debt-to- equity ratio

A3

Flagstaff Systems issues bonds dated January 1 that pay interest semiannually on June 30 and December 31. The bonds have a $90,000 par value and an annual contract rate of 12%, and they mature in five years.

Required

For each separate situation, (a) determine the bonds’ issue price on January 1 and (b) prepare the journal entry to record their issuance. 1. The market rate at the date of issuance is 10%. 2. The market rate at the date of issuance is 12%. 3. The market rate at the date of issuance is 14%.

Check (1) Premium, $6,948

(3) Discount, $6,326

Problem 10-7BA Computing bond price and recording issuance

C2

Required

1. Compute the debt-to-equity ratios for both companies. 2. Which company has the riskier financing structure?

Refer to the bond details in Problem 10-4B.

Required

1. Prepare the January 1 journal entry to record the bonds’ issuance. 2. Determine the total bond interest expense to be recognized over the bonds’ life. 3. Prepare an effective interest amortization table like the one in Exhibit 10B.1 for the bonds’ first two

years. 4. Prepare the journal entries to record the first two interest payments.

Problem 10-8BB Effective Interest: Amortization of bond discount P5 Check (2) $257,506

(3) 6/30/2020 carrying value, $200,803

Refer to the bond details in Problem 10-3B.

Required

1. Compute the total bond interest expense over the bonds’ life. 2. Prepare an effective interest amortization table like the one in Exhibit 10B.2 for the bonds’ life. 3. Prepare the journal entries to record the first two interest payments.

Problem 10-9BB Effective Interest: Amortization of bond premium P6 Check (2) 6/30/2021 carrying value, $327,136

Valdez issues $450,000 of 13%, four-year bonds dated January 1, 2019, that pay interest semiannually on June 30 and December 31. They are issued at $493,608 when the market rate is 10%.

Required

1. Prepare the January 1 journal entry to record the bonds’ issuance. 2. Determine the total bond interest expense to be recognized over the bonds’ life. 3. Prepare an effective interest amortization table like the one in Exhibit 10B.2 for the bonds’ first two

years. 4. Prepare the journal entries to record the first two interest payments.

Analysis Component

5. Assume that the market rate at issuance is 14% instead of 10%. Without presenting numbers, describe how this change affects the amounts reported on Valdez’s financial statements.

Problem 10-10BB Effective Interest: Amortization of bond

P6

Check (3) 6/30/2020 carrying value, $479,202

Atlas Company Bryan Company

Total assets . . . . . . $180,000 $750,000

Total liabilities . . . . 80,000 562,500

Total equity . . . . . . 100,000 187,500

Chapter 10 Accounting for Long-Term Liabilities 413

Problem 10-12BC Accounting for operating lease

C3

Refer to the lease details in Problem 10-11B. Assume that this lease is classified as an operating lease instead of a finance lease.

Required

1. Prepare the January 1 journal entry at the start of the lease to record any asset or liability. 2. Prepare the January 1 journal entry to record the first $14,000 cash lease payment. 3. Prepare the December 31 journal entry to record amortization at the end of (a) Year 1, (b) Year 2, and

(c) Year 3. 4. Prepare the December 31 journal entry to record the $14,000 cash lease payment at the end of (a) Year 1

and (b) Year 2.

Problem 10-11BC Accounting for finance lease

C3

On January 1, Kwak (lessee) signs a three-year lease for equipment that is accounted for as a finance lease. The lease requires three $14,000 lease payments (the first at the beginning of the lease and the remaining two at December 31 of Year 1 and Year 2). The present value of the three annual lease pay- ments is $39,000, using a 7.9% interest rate. The lease payment schedule follows.

Required

1. Prepare the January 1 journal entry at the start of the lease to record any asset or liability. 2. Prepare the January 1 journal entry to record the first $14,000 cash lease payment. 3. Prepare the December 31 journal entry to record straight-line amortization with zero salvage value at

the end of (a) Year 1, (b) Year 2, and (c) Year 3. 4. Prepare the December 31 journal entry to record the $14,000 cash lease payment at the end of (a) Year 1

and (b) Year 2.

Payments (A) (B) (C) (D) (E) Debit Debit Credit Interest on Ending Balance Beginning Balance Lease Liability + Lease Liability = Cash Lease of Lease Liability Date of Lease Liability 7.9% × (A) (D) − (B) Payment (A) − (C) Jan. 1, Year 1 . . . . . $39,000 $14,000 $14,000 $25,000 Dec. 31, Year 1 . . . 25,000 $1,975 12,025 14,000 12,975 Dec. 31, Year 2 . . . 12,975 1,025 12,975 14,000 0 $3,000 $39,000 $42,000

SERIAL PROBLEM Business Solutions

A1 A3

This serial problem began in Chapter 1 and continues through most of the book. If previous chapter seg- ments were not completed, the serial problem can begin at this point.

SP 10 Santana Rey has consulted with her local banker and is considering financing an expansion of her business by obtaining a long-term bank loan. Selected account balances at March 31, 2020, for Business Solutions follow.

Total assets . . . . . . . . . . $120,268 Total liabilities . . . . . . . . . $875 Total equity . . . . . . . . . . $119,393

Required

1. The bank has offered a long-term secured note to Business Solutions. The bank’s loan procedures require that a client’s debt-to-equity ratio not exceed 0.8. As of March 31, 2020, what is the maximum amount that Business Solutions could borrow from this bank?

2. If Business Solutions borrows the maximum amount allowed from the bank, what percentage of assets would be financed (a) by debt and (b) by equity?

3. What are some factors Santana Rey should consider before borrowing the funds? Check (1) $94,639 ©Alexander Image/Shutterstock

414 Chapter 10 Accounting for Long-Term Liabilities

ETHICS CHALLENGE C3 A1

BTN 10-1 Traverse County needs a new county government building that would cost $10 million. The politicians feel that voters will not approve a municipal bond issue to fund the building because it would increase taxes. They opt to have a state bank issue $10 million of tax-exempt securities to pay for the building construction. The county then will make yearly lease payments (of principal and interest) to repay the obligation. Unlike conventional municipal bonds, the lease payments are not binding obligations on the county and, therefore, require no voter approval.

Required

1. Do you think the actions of the politicians and the bankers in this situation are ethical? 2. In terms of risk, how do the tax-exempt securities used to pay for the building compare to a conven-

tional municipal bond issued by Traverse County?

Beyond the Numbers

COMPANY ANALYSIS A1 A2

Accounting Analysis

AA 10-1 Use Apple’s financial statements in Appendix A to answer the following. 1. Identify Apple’s long-term debt as reported on its balance sheet at (a) September 30, 2017, and

(b) September 24, 2016. 2. Calculate the percentage change in long-term debt from September 24, 2016, to September 30, 2017. 3. If Apple’s reported long-term debt continues on the current trend, do we expect total interest expense

to increase or decrease? APPLE

AA 10-2 Key figures for Apple and Google follow.

Apple Google

$ millions Current Year Prior Year Current Year Prior Year

Total assets . . . . . . . . . . . . . $375,319 $321,686 $197,295 $167,497

Total liabilities . . . . . . . . . . . 241,272 193,437 44,793 28,461

Total equity . . . . . . . . . . . . . 134,047 128,249 152,502 139,036

Required

1. Compute the debt-to-equity ratios for Apple and Google for both the current year and the prior year. 2. Use the ratios from part 1 to determine which company’s financing structure is least risky. 3. Is its debt-to-equity ratio more risky or less risky compared to the industry (assumed) average of 0.5

for (a) Apple and (b) Google?

COMPARATIVE ANALYSIS A3

APPLE GOOGLE

AA 10-3 Selected results from Samsung, Apple, and Google follow.

Samsung Apple Google

In millions Current Year Prior Year Current Year Current Year

Total assets . . . . . . . . . . . . W301,752,090 W262,174,324 $375,319 $197,295

Total liabilities . . . . . . . . . 87,260,662 69,211,291 241,272 44,793

Total equity . . . . . . . . . . . . 214,491,428 192,963,033 134,047 152,502

GLOBAL ANALYSIS A3

APPLE GOOGLE

Samsung

Required

1. Compute Samsung’s debt-to-equity ratio for the current year and the prior year. 2. Is Samsung’s financing structure more risky or less risky in the current year versus the prior year? 3. In the current year, is Samsung’s financing structure more risky or less risky than (a) Apple’s and

(b) Google’s?

BTN 10-2 Your business associate mentions that she is considering investing in corporate bonds cur- rently selling at a premium. She says that because the bonds are selling at a premium, they are highly valued and her investment will yield more than the going rate of return for the risk involved. Reply with a memorandum to confirm or correct your associate’s interpretation of premium bonds.

COMMUNICATING IN PRACTICE P3

Chapter 10 Accounting for Long-Term Liabilities 415

BTN 10-4B Break into teams and complete the following requirements related to effective interest amor- tization for a premium bond. 1. Each team member is to independently prepare a blank table with proper headings for amortization of

a bond premium. When all have finished, compare tables and ensure that all are in agreement.

Parts 2 and 3 require use of these facts: On January 1, 2019, McElroy issues $100,000, 9%, five-year bonds at 104.1. The market rate at issuance is 8%. McElroy pays interest semiannually on June 30 and December 31. 2. In rotation, each team member must explain how to complete one line of the bond amortization table,

including all computations for his or her line. All members are to fill in their tables during this pro- cess. You need not finish the table; stop after all members have explained a line.

3. In rotation, each team member is to identify a separate column of the table and indicate what the final number in that column will be and explain the reasoning.

4. Reach a team consensus as to what the total bond interest expense on this bond issue will be if the bond is not retired before maturity.

5. As a team, prepare a list of similarities and differences between the amortization table just prepared and the amortization table if the bond had been issued at a discount.

TEAMWORK IN ACTION P5 P6

Hint: Rotate teams to report on parts 4 and 5. Consider requiring entries for issuance and interest payments.

BTN 10-3 Access the March 23, 2017, filing of the 10-K report of Home Depot for the year ended January 29, 2017, from SEC.gov (ticker: HD). Refer to Home Depot’s balance sheet, including its note 4 (on debt).

Required

1. Identify Home Depot’s long-term liabilities and the amounts for those liabilities from Home Depot’s balance sheet at January 29, 2017.

2. Review Home Depot’s note 4. The note reports that as of January 29, 2017, it had $2.947 billion of “5.875% Senior Notes; due December 16, 2036; interest payable semiannually on June 16 and December 16.” These notes have a face value of $3.0 billion and were originally issued at $2.958 billion.

a. Why would Home Depot issue $3.0 billion of its notes for only $2.958 billion? b. How much cash interest must Home Depot pay each June 16 and December 16 on these notes?

TAKING IT TO THE NET A2

BTN 10-5 Joey Shamah and Scott Borba are the founders of e.l.f. Cosmetics. Assume that the company currently has $250,000 in equity and is considering a $100,000 expansion to meet increased demand. The $100,000 expansion would yield $16,000 in additional annual income before interest expense. Assume that the business currently earns $40,000 annual income before interest expense of $10,000, yielding a return on equity of 12% ($30,000/$250,000). To fund the expansion, the company is considering the issu- ance of a 10-year, $100,000 note with annual interest payments (the principal due at the end of 10 years).

Required

1. Using return on equity as the decision criterion, show computations to support or reject the expansion if interest on the $100,000 note is (a) 10%, (b) 15%, (c) 16%, (d) 17%, and (e) 20%.

2. What general rule do the results in part 1 illustrate?

ENTREPRENEURIAL DECISION A1

BTN 10-6 Visit your city or county library. Ask the librarian to help you locate the most recent financial records of your city or county government. Examine those records.

Required

1. Determine the amount of long-term bonds and notes currently outstanding. 2. Read the supporting information to your municipality’s financial statements and record

a. The market interest rate(s) when the bonds and/or notes were issued. b. The date(s) when the bonds and/or notes will mature. c. Any rating(s) on the bonds and/or notes received from Moody’s Investors Service, Standard &

Poor’s Ratings Services, Fitch Ratings, or another rating agency.

HITTING THE ROAD A1

Design elements: Lightbulb: ©Chuhail/Getty Images; Blue globe: ©nidwlw/Getty Images and ©Dizzle52/Getty Images; Chess piece: ©Andrei Simonenko/Getty Images and ©Dizzle52/Getty Images; Mouse: ©Siede Preis/Getty Images; Global View globe: ©McGraw-Hill Education and ©Dizzle52/Getty Images; Sustainability: ©McGraw-Hill Education and ©Dizzle52/Getty Images

Learning Objectives

CONCEPTUAL C1 Identify characteristics of corporations

and their organization.

C2 Explain characteristics of, and distribute dividends between, common and preferred stock.

C3 Explain the items reported in retained earnings.

PROCEDURAL P1 Record the issuance of corporate stock.

P2 Record transactions involving cash dividends, stock dividends, and stock splits.

P3 Record purchases and sales of treasury stock.

ANALYTICAL A1 Compute earnings per share and

describe its use.

A2 Compute price-earnings ratio and describe its use in analysis.

A3 Compute dividend yield and explain its use in analysis.

A4 Compute book value and explain its use in analysis.

Chapter Preview

11 Corporate Reporting and Analysis

NTK 11-4

TREASURY STOCK

P3 Purchasing treasury stock

Reissuing treasury stock

REPORTING AND ANALYSIS

C3 Retained earnings and equity

A1 EPS A2 PE ratio A3 Dividend yield A4 Book value

NTK 11-3

PREFERRED STOCK

C2 Issuance Dividend preferences

Rationale

NTK 11-1

COMMON STOCK

C1 Stock basics P1 Stock issuance:

Par value

No-par value

Stated value

Noncash assets

NTK 11-2

DIVIDENDS

P2 Cash dividends Stock dividends

Stock splits

NTK 11-5

417

“Trust of the consumer is critical” —Jeremy Stoppelman

Point of View

SAN FRANCISCO—“When I was in business school, I was think- ing about doing something entrepreneurial,” recalls Jeremy Stoppelman. “I’d always read the little vignettes about how someone started a small business.”

“Word of mouth was the best way to find local busi- nesses,” explains Jeremy. “If we could find a way to capture that and bring it online, that would be powerful.” To turn his idea into a business, Jeremy and his co-founders built Yelp (Yelp.com). Yelp publishes crowdsourced reviews about local businesses.

In the first few years of business, Jeremy had to make crucial decisions regarding creditor versus equity financing. When Google offered to purchase his business, Jeremy had to learn about stock types and ways to finance Yelp.

“I felt like we built this company,” recalls Jeremy, “there’s no fundamental reason for us to sell.” Instead of selling to Google, and armed with knowledge of equity financing, Jeremy raised money from individual investors. Also, instead of paying divi- dends, he reinvested Yelp income into the company.

Jeremy has some advice: “Building a great company takes time. If it’s not something you’re passionate about . . . you’re not going to make it.”

Sources: Yelp website, January 2019; Yelp Foundation, January 2018; Time, December 2014

©Maria J. Avila/MCT/Newscom

A corporation is an entity that is separate from its owners and has many of the same rights as a person. Owners of corporations are called stockholders or shareholders. Corporations are sepa- rated into two types. A privately held (or closely held) corporation does not offer its stock for public sale and usually has few stockholders. A publicly held corporation offers its stock for public sale and can have thousands of stockholders. Public sale means selling and trading stock on an organized stock market.

Corporate Advantages Separate legal entity: A corporation has many of the same rights, duties, and responsibili-

ties as a person. It takes actions through its agents, who are its officers and managers. Limited liability: Stockholders are not liable for corporate actions or debt. Transferable ownership rights: Transfer of shares from one stockholder to another has no direct

effect on operations except when it causes a change in directors who oversee the corporation. Continuous life: A corporation’s life is indefinite because it is not tied to the physical lives

of its owners. No mutual agency for stockholders: Stockholders, who are not officers and managers, can-

not bind the corporation to contracts—called lack of mutual agency. Easier capital accumulation: Buying stock is attractive to investors because of the advan-

tages above, which helps corporations collect large sums of money.

Corporate Disadvantages Government regulation: A corporation must follow a state’s incorporation laws.

Proprietorships and partnerships avoid many of these. Corporate taxation: Corporations pay many of the same taxes as proprietorships and part-

nerships plus additional taxes. The most burdensome are federal and state corporate income taxes that together can take 21% or more of pretax income. Also, corporate income is usually taxed a second time as part of stockholders’ personal income when they receive cash divi- dends. This is called double taxation.

CORPORATE FORM OF ORGANIZATION C1 Identify characteristics of corporations and their organization.

418 Chapter 11 Corporate Reporting and Analysis

Corporate Stockholders Rights of Stockholders Stockholders have specific rights under the corporation’s charter and general rights under state law. Stockholders also have the right to receive timely fi- nancial reports. When a corporation has only one class of stock, it is called common stock. State laws vary, but common stockholders usually have the right to Vote at stockholders’ meetings (or register proxy votes). Sell or dispose of their stock. Purchase their proportional share of any common stock later issued. This preemptive right

protects stockholders’ proportionate interest. For example, a stockholder who owns 25% of a corporation’s stock has the first opportunity to buy 25% of any new stock issued.

Receive the same dividend, if any, on each common share. Share in any assets remaining after creditors and preferred stockholders are paid if the corpo-

ration is liquidated. Each common share receives the same amount.

Artificial Unintelligence Dow Jones newswire mistakenly published a bogus news story about Google acquiring Apple for $9 billion. Informed investors were not fooled, as Apple’s market value was over $700 billion. However, bots designed to purchase stock of any company rumored of being acquired instantaneously purchased millions of shares of Apple. This event revealed how bots are increasingly impacting our financial markets. ■

Decision Insight

Corporate Organization and Management Incorporation A corporation is created by getting a charter from a state government. A charter application is signed by the prospective stockholders called incorporators or promoters and then filed with the state. When the application process is complete and fees paid, the charter is issued and the corporation is formed. Investors then purchase the corporation’s stock, meet as stockholders, and elect a board of directors.

Organization Expenses Organization expenses (or organization costs) are the costs to start a corporation; they include legal fees, promoters’ fees, and payments for a charter. The corporation records (debits) these costs to Organization Expenses. Organization costs are expensed as incurred.

Management Stockholders control a corpora- tion by electing a board of directors, or directors. A stockholder usually has one vote for each share of stock owned. This control relation is shown in Exhibit 11.1. Directors are responsible for overseeing corporate activities. A board is in charge of hiring and firing key executives who manage day-to-day operations. A corporation’s chief executive officer (CEO) is often its president. Several vice presidents are commonly assigned to specific areas such as finance, production, and marketing.

A corporation usually holds a stockholder meeting at least once a year to elect directors. Stockholders who do not attend stockholders’ meetings can give their voting rights to an agent by signing a proxy, a document that gives a designated agent the right to vote the stock.

EXHIBIT 11.1 Corporate Structure

Point: Bylaws are guidelines that govern the corporation.

Keep the Faith Sources for start-up money include (1) “angel” investors such as family, friends, or anyone who be- lieves in a company; (2) employees, investors, and even suppliers; and (3) venture capitalists (investors) who have a record of entrepreneurial success. ■

Decision Insight

Point: While rare, not all common stock has voting rights; Google’s C Class shares are nonvoting.

Point: Green Bay Packers are the only nonprofit, community-owned major professional team.

Stockholders

Board of Directors

President, Vice President, and Other O cers

Employees of the Corporation

Corporate governance is the system by which companies are directed and controlled.

Chapter 11 Corporate Reporting and Analysis 419

Stock Certificates and Transfer A corporation sometimes gives a stock certificate as proof of share ownership. Exhibit 11.2 shows a stock certificate issued by the Green Bay Packers. A certificate shows the company name, stockholder name, number of shares, and other information. Issuance of paper certifi- cates is becoming less common.

Registrar and Transfer Agents If a corporation’s stock is traded on a stock exchange, the corporation has a registrar and a transfer agent. A registrar keeps a list of stockholders for stock- holder meetings and dividend payments. A transfer agent assists with purchases and sales of shares. Registrars and transfer agents are usually large banks or trust companies.

Corporate Stock Capital stock is shares issued to obtain capital (owner financing).

Authorized Stock Authorized stock is the number of shares that a corporation’s charter allows it to sell. The number of authorized shares usually exceeds the number of shares issued (and outstanding) by a large amount. Outstanding stock is stock held by stockholders. No jour- nal entry is required for stock authorization. A corporation discloses the number of shares authorized in the equity section of its balance sheet or notes. Apple’s balance sheet reports 12.6 billion common shares authorized.

Selling (Issuing) Stock A corporation can sell stock directly or indirectly. To sell di- rectly, it offers its stock to buyers. This type of sale is common with privately held corporations. To sell indirectly, a corporation pays a brokerage house (investment banker) to sell its stock. Some brokerage houses underwrite stock, meaning they buy the stock from the corporation and resell it to investors.

Market Value of Stock Market value per share is the price at which a stock is bought and sold. Expected future income, dividends, growth, and economic factors influence market value. The current market value of previously issued shares does not impact the issuing corpora- tion’s stockholders’ equity.

Classes of Stock When all authorized shares have the same rights and characteristics, the stock is called common stock. A corporation sometimes issues more than one class of stock, includ- ing preferred stock and different classes of common stock. American Greetings has two types of common stock: Class A stock has 1 vote per share and Class B stock has 10 votes per share.

Par Value Stock Par value stock is stock that has a par value, which is an amount as- signed per share by the corporation in its charter. Monster Worldwide’s common stock has a par value of $0.001. Other commonly assigned par values are $5, $1 and $0.01. There is no restriction on assigned par value. In many states, the par value of a stock establishes minimum legal capital, which is the least amount that the buyers of stock must contribute to the corporation or be at risk to pay creditors at a future date.

No-Par Value Stock No-par value stock, or no-par stock, is stock not assigned an amount per share by the corporate charter. There is no minimum legal capital with no-par stock.

Stated Value Stock Stated value stock is no-par stock that has an assigned “stated” value per share. Stated value per share is the mini- mum legal capital per share in this case.

Stockholders’ Equity A corporation’s equity is called stock- holders’ equity, or shareholders’ equity. Exhibit 11.3 shows stockhold- ers’ equity consists of (1) paid-in (or contributed) capital and (2) retained

Point: Managers set a low par value when minimum legal capital or state issuance taxes are based on par.

Point: Par, no-par, and stated value do not affect the stock’s market value.

EXHIBIT 11.2 Stock Certificate

Courtesy of JJW Images

Subcategories of Authorized Stock

Authorized

Authorized & Issued

Authorized, Issued, and Outstanding

Innermost box would show a decline in shares issued if a company buys back its issued stock.

Corporation

To ta

l P ai

d- In

C ap

ita lCommon Stock

Normal bal.

Retained Earnings

Normal bal.

Normal bal.

Paid-In Capital in Excess of Par

EXHIBIT 11.3 Equity Composition

420 Chapter 11 Corporate Reporting and Analysis

earnings. Paid-in capital is the total amount of cash and other assets the corporation receives from its stockholders in exchange for its stock. Retained earnings is the cumulative net income (and loss) not distributed as dividends to its stockholders.

Stock Quote The AT&T stock quote is in- terpreted as (left to right): Hi, highest price in past 52 weeks; Lo, lowest price in past 52 weeks; Sym, company exchange symbol; Div, dividends paid per share in past year; Yld %, dividend divided by closing price; PE, stock price per share divided by earnings per share; Hi, highest price for the day; Lo, lowest price for the day; Close, closing price for the day; Net Chg, change in closing price from prior day. ■

Decision Insight

52 Weeks Hi Lo Sym Div Yld % PE Hi Lo Close Net Chg 42.70 32.55 T 2.00 5.24 7.95 38.31 37.77 37.81 +0.53

Issuance of stock affects paid-in (contributed) capital accounts; retained earnings is unaffected.

Issuing Par Value Stock Par value stock can be issued at par, at a premium (above par), or at a discount (below par). Cash or other assets are received in exchange for stock.

Issuing Par Value Stock at Par When common stock is issued at par value, we re- cord both the asset(s) received and the par value stock issued. The entry to record Dillon’s issu- ance of 30,000 shares of $10 par value stock for $300,000 cash on June 5 follows.

COMMON STOCK P1 Record the issuance of corporate stock.

Issuing Par Value Stock at a Premium A premium on stock occurs when a cor- poration sells its stock for more than par (or stated) value. If Dillon issues its $10 par value com- mon stock at $12 per share, its stock is sold at a $2 per share premium. The premium, called paid-in capital in excess of par value, is reported as part of equity; it is not revenue and is not listed on the income statement. The entry to issue 30,000 shares of $10 par value stock for $12 per share follows.

Point: Paid-In Capital in Excess of Par Value is also called Additional Paid-In Capital.

The Paid-In Capital in Excess of Par Value account is added to the par value of the stock in the equity section of the balance sheet, as shown in Exhibit 11.4.

Point: The phrase paid-in capital is interchangeable with contributed capital.

Common stock—$10 par value; 50,000 shares authorized; 30,000 shares issued and outstanding . . . . . . . . . $300,000

Paid-in capital in excess of par value, common stock . . . . . . . . . . . . . . . . . . . . . . . . . . . . . . . . . . . . . . . . . . . . 60,000 Retained earnings* . . . . . . . . . . . . . . . . . . . . . . . . . . . . . . . . . . . . . . . . . . . . . . . . . . . . . . . . . . . . . . . . . . . . . . . . . . 65,000

Total stockholders’ equity . . . . . . . . . . . . . . . . . . . . . . . . . . . . . . . . . . . . . . . . . . . . . . . . . . . . . . . . . . . . . . . . . . . . . $425,000

EXHIBIT 11.4 Stockholders’ Equity for Stock Issued at a Premium

*This is the company’s first year of operations, with income of $65,000 and no dividends.

Assets = Liabilities + Equity +300,000 +300,000

June 5 Cash . . . . . . . . . . . . . . . . . . . . . . . . . . . . . . . . . . . . . . . . . . . . . . 300,000

Common Stock, $10 Par Value* . . . . . . . . . . . . . . . . . . . . 300,000

Issued 30,000 shares of $10 par value stock at par.

*$10 par value × 30,000 shares

Assets = Liabilities + Equity +360,000 +300,000 +60,000

June 5 Cash. . . . . . . . . . . . . . . . . . . . . . . . . . . . . . . . . . . . . . . . . . . . . . 360,000

Common Stock, $10 Par Value* . . . . . . . . . . . . . . . . . . . 300,000

Paid-In Capital in Excess of Par Value, Common Stock† 60,000 Sold and issued 30,000 shares of $10 par

value common stock at $12 per share.

*$10 par value × 30,000 shares †[$12 issue price − $10 par value] × 30,000 shares

Chapter 11 Corporate Reporting and Analysis 421

Issuing Par Value Stock at a Discount A discount on stock occurs when it is sold for less than par value. Most states prohibit this. If stock is issued at a discount, the amount by which issue price is less than par is debited to a Discount on Common Stock account, a contra to the Common Stock account, and its balance is subtracted from the par value of stock.

Issuing No-Par Value Stock When no-par stock is issued, the amount the corporation receives is credited to a no-par stock account. The entry to issue 1,000 shares of no-par common stock for $40 cash per share follows.

Issuing Stated Value Stock When stated value stock is issued, the stated value is credited to the stock account. Any amount above the stated value is credited to Paid-In Capital in Excess of Stated Value, which is reported in stockholders’ equity. The entry to issue 1,000 shares of no-par common stock having a stated value of $40 per share in return for $50 cash per share follows.

Frequency of Stock Types

Par 88%

Stated 3%

No-par 9%

Issuing Stock for Noncash Assets A corporation can receive assets other than cash in exchange for its stock. (It also can take lia- bilities such as a mortgage on property received.) The corporation records the assets received at their market values as of the transaction date. The stock given in exchange is recorded at its par (or stated) value with any excess recorded in the Paid-In Capital in Excess of Par (or Stated) Value account. (If no-par stock is issued, the stock is recorded at the assets’ market value.) The entry to record receipt of land valued at $105,000 in return for 4,000 shares of $20 par value common stock is

Point: Stock issued for noncash assets is recorded at the market value of either the stock or the noncash assets, whichever is more determinable.

A corporation sometimes gives shares of its stock to promoters in exchange for their work in organizing the corporation, which it records as organization expenses. The entry to issue 600 shares of $15 par value common stock for $12,000 of organizing work is

Assets = Liabilities + Equity +40,000 +40,000

Oct. 20 Cash . . . . . . . . . . . . . . . . . . . . . . . . . . . . . . . . . . . . . . . . . . . . . . . . 40,000 Common Stock, No-Par Value* . . . . . . . . . . . . . . . . . . . . . . . 40,000 Issued 1,000 shares of no-par stock at $40 per share.

*$40 issue price × 1,000 no-par shares

Oct. 20 Cash . . . . . . . . . . . . . . . . . . . . . . . . . . . . . . . . . . . . . . . . . . . . . . . . 50,000 Common Stock, $40 Stated Value* . . . . . . . . . . . . . . . . . . . . 40,000 Paid-In Capital in Excess of Stated Value, Common Stock† . . . 10,000 Issued 1,000 shares of $40 per share stated

value stock at $50 per share.

*$40 stated value × 1,000 shares †[$50 issue price − $40 stated value] × 1,000 shares

Assets = Liabilities + Equity +50,000 +40,000 +10,000

June 10 Land . . . . . . . . . . . . . . . . . . . . . . . . . . . . . . . . . . . . . . . . . . . . . . . . 105,000 Common Stock, $20 Par Value* . . . . . . . . . . . . . . . . . . . . . . 80,000 Paid-In Capital in Excess of Par Value, Common Stock† . . . . . . 25,000 Exchanged 4,000 shares of $20 par value stock for land.

*$20 par value × 4,000 shares †$105,000 asset value − $80,000 par value

Assets = Liabilities + Equity +105,000 +80,000 +25,000

June 5 Organization Expenses. . . . . . . . . . . . . . . . . . . . . . . . . . . . . . . . . . 12,000 Common Stock, $15 Par Value* . . . . . . . . . . . . . . . . . . . . . . 9,000 Paid-In Capital in Excess of Par Value, Common Stock† . . . . . . 3,000 Gave promoters 600 shares of $15 par value

common stock in exchange for their services.

*$15 par value × 600 shares †$12,000 services value − $9,000 par value

Assets = Liabilities + Equity −12,000 +9,000 +3,000

422 Chapter 11 Corporate Reporting and Analysis

Prepare journal entries to record the following four separate issuances of stock. 1. Issued 80 shares of $5 par value common stock for $700 cash. 2. Issued 40 shares of no-par common stock to promoters in exchange for their efforts, estimated to be

worth $800. The stock has a $1 per share stated value. 3. Issued 40 shares of no-par common stock in exchange for land estimated to be worth $800. The stock

has no stated value. 4. Issued 20 shares of no-par common stock with a stated value of $30 per share for $900 cash.

Solution

Recording Stock Issuance

NEED-TO-KNOW 11-1

P1

Cash . . . . . . . . . . . . . . . . . . . . . . . . . . . . . . . . . . . . . . . . . . . . . . . 700

Common Stock, $5 Par Value* . . . . . . . . . . . . . . . . . . . . . . 400

Paid-In Capital in Excess of Par Value, Common Stock† . . 300

Issued common stock for cash.

1.

Organization Expenses . . . . . . . . . . . . . . . . . . . . . . . . . . . . . . . . 800

Common Stock, $1 Stated Value . . . . . . . . . . . . . . . . . . . . 40

Paid-In Capital in Excess of Stated Value, Common Stock . . . 760

Issued stock to promoters.

2.

Land . . . . . . . . . . . . . . . . . . . . . . . . . . . . . . . . . . . . . . . . . . . . . . . 800

Common Stock, No-Par Value . . . . . . . . . . . . . . . . . . . . . . . 800

Issued stock in exchange for land.

3.

Cash . . . . . . . . . . . . . . . . . . . . . . . . . . . . . . . . . . . . . . . . . . . . . . . 900

Common Stock, $30 Stated Value*. . . . . . . . . . . . . . . . . . . 600

Paid-In Capital in Excess of Stated Value, Common Stock† . . 300

Issued stated value stock for cash.

4.

Do More: QS 11-2, QS 11-3, QS 11-4, QS 11-5, E 11-3, E 11-4, E 11-5

Cash Dividends The board of directors decides whether to pay cash dividends. The directors may decide to keep the cash to invest in the corporation’s growth, to meet emergencies, or to pay off debt. Alterna- tively, many corporations pay cash dividends to their stockholders at regular dates.

Accounting for Cash Dividends Dividend payment has three important dates: declara- tion, record, and payment. Date of declaration is the date the directors vote to declare and pay a dividend. This creates a legal liability of the corporation to its stockholders. Date of record is the date for identifying those stockholders to receive dividends. Persons who own stock on the date of record receive dividends. Date of payment is the date when the corporation makes payment.

Declaration Date of Record

Cash Dividend Dates

Payment

The entry for a January 9 declaration of a $1 per share cash dividend by Z-Tech with 5,000 outstanding shares follows. Common Dividend Payable is a current liability.

DIVIDENDS P2 Record transactions involving cash dividends, stock dividends, and stock splits.

Point: Amazon has never declared a cash dividend.

Cash dividend to preferred

Cash dividend to

common

22%

76%

Percent of Corporations Paying Dividends

Assets = Liabilities + Equity +5,000 −5,000

Date of Declaration—Cash Dividend

Jan. 9 Retained Earnings. . . . . . . . . . . . . . . . . . . . . . . . . . . . . . . . . . . 5,000

Common Dividend Payable* . . . . . . . . . . . . . . . . . . . . . . 5,000

Declared $1 per common share cash dividend.†

*$1 per share declared dividend × 5,000 outstanding shares † To aid learning and show how dividends impact retained earnings, we debit (reduce) Retained Earnings on the date of declaration in this chapter and all assignments. We normally debit Dividends; then, at period-end, Dividends is closed to Retained Earnings. The effect is the same: Retained earnings is decreased from dividends.

*80 shares × $5 per share = $400 †$700 − $400 = $300

*20 shares × $30 stated value = $600 †$900 − $600 = $300

Chapter 11 Corporate Reporting and Analysis 423

The date of record for this dividend is January 22. No journal entry is made on the date of record.

The February 1 date of payment entry removes the liability and reduces cash.

Deficits and Cash Dividends A corporation with a debit (abnormal) balance for Retained Earnings has a retained earnings deficit, which occurs when a company has cumula- tive losses and/or pays more dividends than total earnings from current and prior years. A defi- cit reduces equity, as shown in Exhibit 11.5. Most states prohibit a corporation with a deficit from paying a cash dividend to protect creditors. Another type of dividend is a liquidating cash dividend, or liquidating dividend, where a corporation returns a portion of the capital contrib- uted back to stockholders.

Point: The Retained Earnings Deficit account is also called Accumulated Deficit.

Common stock—$10 par value, 5,000 shares authorized, issued, and outstanding . . . . . . . . . . . . . . . . . $50,000

Retained earnings deficit . . . . . . . . . . . . . . . . . . . . . . . . . . . . . . . . . . . . . . . . . . . . . . . . . . . . . . . . . . . . . . .   (6,000) Total stockholders’ equity. . . . . . . . . . . . . . . . . . . . . . . . . . . . . . . . . . . . . . . . . . . . . . . . . . . . . . . . . . . . . . . . $44,000

EXHIBIT 11.5 Stockholders’ Equity with a Deficit

Stock Dividends A stock dividend, declared by a corporation’s directors, is a distribution of additional shares of its own stock to its stockholders without any payment in return. Stock dividends and cash divi- dends are different. A stock dividend does not reduce assets and equity but instead transfers a portion of equity from retained earnings to contributed capital.

Reasons for Stock Dividends Stock dividends are given for at least two reasons. First, stock dividends keep the market price of the stock affordable. When a corporation has a stock dividend, it increases the number of outstanding shares, which lowers the per share stock price. Second, a stock dividend shows management’s confidence that the company is doing well and will continue to do well.

Accounting for Stock Dividends A stock dividend transfers part of retained earn- ings to contributed capital accounts, called capitalizing retained earnings. Accounting for a stock dividend depends on whether it is a small or large stock dividend. A small stock dividend is a distribution of 25% or less of previously

outstanding shares. It is recorded by capitalizing retained earnings for an amount equal to the market value of the shares to be distributed.

A large stock dividend is a distribution of more than 25% of previously outstanding shares. It is recorded by capitalizing retained earnings for the par or stated value of the stock.

The equity section of Quest’s balance sheet just before its declaration of a stock dividend on December 31 follows.

Hint: Five Steps to Record Stock Dividends

Step 1: Identify number of shares outstanding. Step 2: Identify the stock dividend percentage. Step 3: Compute number of new shares (step 1 × step 2). Step 4: Value new shares at market (small stock dividend)

or par (large stock dividend). Step 5: Determine debit (reduction) to Retained Earnings

(step 3 × step 4).

Stockholders’ Equity Before Dividend

Common stock—$10 par value, 15,000 shares authorized, 10,000 shares issued and outstanding . . . . . . $100,000 Paid-in capital in excess of par value, common stock . . . . . . . . . . . . . . . . . . . . . . . . . . . . . . . . . . . . . . . . . . . 8,000 Retained earnings. . . . . . . . . . . . . . . . . . . . . . . . . . . . . . . . . . . . . . . . . . . . . . . . . . . . . . . . . . . . . . . . . . . . . . . . 35,000 Total stockholders’ equity . . . . . . . . . . . . . . . . . . . . . . . . . . . . . . . . . . . . . . . . . . . . . . . . . . . . . . . . . . . . . . . . . $143,000

Small Stock Dividend Assume that Quest declares a 10% stock dividend on December 31. This stock dividend of 1,000 shares, computed as 10% of its 10,000 outstanding shares, is to be

Assets = Liabilities + Equity −5,000 −5,000

Date of Payment—Cash Dividend

Feb. 1 Common Dividend Payable . . . . . . . . . . . . . . . . . . . . . . . . . . . 5,000

Cash . . . . . . . . . . . . . . . . . . . . . . . . . . . . . . . . . . . . . . . . . 5,000

Paid $1 per common share cash dividend.

424 Chapter 11 Corporate Reporting and Analysis

distributed on January 20 to the stockholders of record on January 15. Because the market price of Quest’s stock on December 31 is $15 per share, this small stock dividend declaration is re- corded as follows.

EXHIBIT 11.6 Stockholders’ Equity before, during, and after a Stock Dividend

Before Date of Date of After Stockholders’ Equity Dividend Declaration Payment Dividend Common stock—$10 par value, 15,000 shares authorized, 10,000 shares issued and outstanding. . . . . . . . . . . $100,000 $ $+10,000 $110,000 Common stock dividend distributable—1,000 shares . . . . . . . . . . . 0 +10,000 −10,000 0 Paid-in capital in excess of par value, common stock . . . . . . . . . . . 8,000 + 5,000 13,000 Retained earnings. . . . . . . . . . . . . . . . . . . . . . . . . . . . . . . . . . . . . . . . 35,000 −15,000 20,000 Total stockholders’ equity . . . . . . . . . . . . . . . . . . . . . . . . . . . . . . . . . $143,000 $ 0 $ 0 $143,000

No entry is made on the date of record for a stock dividend. However, on January 20, the date of payment, Quest distributes the new shares and records the entry below (numbers from the “Pay- ment” column of Exhibit 11.6). The combined effect of these entries is to transfer (or capitalize) $15,000 of retained earnings to paid-in capital accounts (see far right column of Exhibit 11.6). A stock dividend has no effect on the ownership percentage of stockholders.

Point: A stock dividend does not affect total assets or total equity.

The balance sheet changes in three ways when a small stock dividend is declared. Common Stock Dividend Distributable, an equity account that exists only until the shares are

distributed, increases by $10,000. Paid-in capital in excess of par increases by $5,000, which is the amount in excess of par (or

stated) value. Retained earnings decreases by $15,000, reflecting the increase in both common stock and

paid-in capital in excess of par.

The impacts on stockholders’ equity from the 10% stock dividend are in Exhibit 11.6.

Point: The term distributable (not payable) is used for stock dividends. A stock dividend is never a liability because it never reduces assets.

Point: The credit to Paid-In Capital in Excess of Par Value is recorded when the stock dividend is declared. This account is not affected when stock is later distributed.

Large Stock Dividend A corporation capitalizes retained earnings equal to the par or stated value of the newly issued shares for a large stock dividend. Suppose Quest declares a stock dividend of 30% instead of 10% on December 31. Because this dividend is more than 25%, it is a large stock dividend. This means the par value of the 3,000 (10,000 outstanding shares × 30%) dividend shares is capitalized at the date of declaration with the entry below. This transaction decreases retained earnings and increases contributed capital by $30,000.

Assets = Liabilities + Equity −15,000 +10,000 + 5,000

Date of Declaration—Small Stock Dividend

Dec. 31 Retained Earnings. . . . . . . . . . . . . . . . . . . . . . . . . . . . . . . . . . . 15,000

Common Stock Dividend Distributable* . . . . . . . . . . . . . 10,000

Paid-In Capital in Excess of Par Value, Common Stock† 5,000

Declared a 10% stock dividend of 1,000 shares.

*10% dividend × 10,000 outstanding shares × $10 par value †10% dividend × 10,000 outstanding shares × [$15 market price − $10 par value]

Assets = Liabilities + Equity −10,000 +10,000

Date of Payment—Small Stock Dividend

Jan. 20 Common Stock Dividend Distributable . . . . . . . . . . . . . . . . . . 10,000

Common Stock, $10 Par Value . . . . . . . . . . . . . . . . . . . . 10,000

Record issuance of common stock dividend.

Date of Declaration—Large Stock Dividend

Dec. 31 Retained Earnings. . . . . . . . . . . . . . . . . . . . . . . . . . . . . . . . . . . 30,000

Common Stock Dividend Distributable* . . . . . . . . . . . . . 30,000

Declared a 30% stock dividend of 3,000 shares.

*30% dividend × 10,000 outstanding shares × $10 par value

Assets = Liabilities + Equity −30,000 +30,000

Chapter 11 Corporate Reporting and Analysis 425

Stock Splits A stock split is the distribution of additional shares to stockholders according to their per- cent ownership. When a stock split occurs, the corporation “calls in” its outstanding shares and issues more than one new share in exchange for each old share. Splits can be done in any ratio. Apple did a 7-for-1 stock split. Stock splits reduce the par or stated value per share. The reasons for stock splits are similar to those for stock dividends, including afford- ability and management confidence.

Assume CTI has 100,000 outstanding shares of $20 par value common stock with a cur- rent market value of $88 per share. A 2-for-1 stock split cuts par value in half as it replaces 100,000 shares of $20 par value stock with 200,000 shares of $10 par value stock. The split does not affect any equity amounts reported on the balance sheet or any individual stock- holder’s percent ownership. No journal entry is made. The only effect on the accounts is a change in the stock account description. After the split, CTI changes its stock account title to Common Stock, $10 Par Value. The stock’s description on the balance sheet also changes to reflect the additional issued and outstanding shares and the new par value.

Financial Statement Effects of Dividends and Splits

STOCK

Before 5 : 1 Split: 1 share, $50 par

After 5 : 1 Split: 5 shares, $10 par

STOCK

Cash Dividend Small Stock Dividend Large Stock Dividend Stock Split

Total assets Decrease No change No change No change

Total liabilities No change No change No change No change

Total stockholders’ equity Decrease No change No change No change

Common stock No change Increase Increase No change

Paid-in capital in excess of par No change Increase No change No change

Retained earnings Decrease Decrease Decrease No change

Entrepreneur A company you co-founded and own stock in announces a 50% stock dividend. Has the value of your stock investment increased, decreased, or remained the same? Would it make a difference if it was a 3-for-2 stock split executed in the form of a dividend? ■ Answer: The stock dividend does not affect the value of your investment or give you income. However, a stock dividend can reveal positive expectations and also improve a stock’s marketability by making it more affordable. The same answer applies to the 3-for-2 stock split.

Decision Maker

Point: A reverse stock split is the opposite of a stock split and re- sults in fewer shares. It increases the par or stated value per share.

A company began the current year with the following balances in its stockholders’ equity accounts.

P2 Recording Dividends

NEED-TO-KNOW 11-2 Common stock—$10 par, 500 shares authorized, 200 shares issued and outstanding . . . . . . . . . $2,000

Paid-in capital in excess of par, common stock . . . . . . . . . . . . . . . . . . . . . . . . . . . . . . . . . . . . . . . . . 1,000

Retained earnings. . . . . . . . . . . . . . . . . . . . . . . . . . . . . . . . . . . . . . . . . . . . . . . . . . . . . . . . . . . . . . . . . 5,000

Total. . . . . . . . . . . . . . . . . . . . . . . . . . . . . . . . . . . . . . . . . . . . . . . . . . . . . . . . . . . . . . . . . . . . . . . . . . . . $8,000

All outstanding common stock was issued for $15 per share when the company was created. Prepare journal entries to account for the following transactions during the current year.

Jan. 10 The board declared a $0.10 cash dividend per share to shareholders of record on January 28. Feb. 15 Paid the cash dividend declared on January 10. Mar. 31 Declared a 20% stock dividend when the market value of the stock was $18 per share. May 1 Distributed the stock dividend declared on March 31. Dec. 1 Declared a 40% stock dividend when the market value of the stock was $25 per share. Dec. 31 Distributed the stock dividend declared on December 1.

On the date of payment, the company makes the following entry.

Jan. 15 Common Stock Dividend Distributable . . . . . . . . . . . . . . . . . . 30,000

Common Stock, $10 Par Value . . . . . . . . . . . . . . . . . . . . 30,000

426 Chapter 11 Corporate Reporting and Analysis

Jan. 10 Retained Earningsa . . . . . . . . . . . . . . . . . . . . . . . . . . . . . . . . . . . . . . . . . 20 Common Dividend Payable . . . . . . . . . . . . . . . . . . . . . . . . . . . . . . 20 Declared a $0.10 per share cash dividend. a200 outstanding shares × $0.10

Feb. 15 Common Dividend Payable . . . . . . . . . . . . . . . . . . . . . . . . . . . . . . . . . . . 20 Cash . . . . . . . . . . . . . . . . . . . . . . . . . . . . . . . . . . . . . . . . . . . . . . . . . 20 Paid $0.10 per share cash dividend.

Mar. 31 Retained Earningsb . . . . . . . . . . . . . . . . . . . . . . . . . . . . . . . . . . . . . . . . . 720 Common Stock Dividend Distributablec . . . . . . . . . . . . . . . . . . . . . 400 Paid-In Capital in Excess of Par Value, Common Stock . . . . . . . . 320 Declared a small stock dividend of 20%, or

40 shares; market value is $18 per share. b200 outstanding shares × 20% × $18 market c40 new shares × $10 par

May 1 Common Stock Dividend Distributable . . . . . . . . . . . . . . . . . . . . . . . . . . 400 Common Stock . . . . . . . . . . . . . . . . . . . . . . . . . . . . . . . . . . . . . . . . 400 Distributed 40 shares of common stock.

Dec. 1 Retained Earningsd . . . . . . . . . . . . . . . . . . . . . . . . . . . . . . . . . . . . . . . . . 960 Common Stock Dividend Distributable . . . . . . . . . . . . . . . . . . . . . 960 Declared a large stock dividend of 40%, or 96 shares

(40% × [200 + 40]); par value is $10 per share. d240 outstanding shares × 40% × $10 par

Dec. 31 Common Stock Dividend Distributable . . . . . . . . . . . . . . . . . . . . . . . . . . 960 Common Stock . . . . . . . . . . . . . . . . . . . . . . . . . . . . . . . . . . . . . . . . 960 Distributed 96 shares of common stock.

Do More: QS 11-6, QS 11-7, QS 11-8, QS 11-9, QS 11-10,

E 11-6, E 11-7, E 11-8

Preferred stock has special rights that give it priority (or senior status) over common stock in one or more areas. Special rights usually include a preference for receiving dividends and assets in liquidation. Preferred stock has the rights of common stock unless the corporate charter excludes them. A common exclusion is the right to vote.

Issuance of Preferred Stock Preferred stock is recorded in its own separate capital accounts. If Dillon issues 50 shares of $100 par value preferred stock for $6,000 cash, the entry is

PREFERRED STOCK C2 Explain characteristics of, and distribute dividends between, common and preferred stock.

The equity section of the year-end balance sheet for Dillon, including preferred stock, is in Exhibit 11.7. (The entry for issuing no-par preferred stock is similar to issuing no-par common stock. Also, the entry for issuing preferred stock for noncash assets is similar to that for common stock.)

EXHIBIT 11.7 Stockholders’ Equity with Common and Preferred Stock

Stockholders’ Equity Preferred stock—$100 par value; 1,000 shares authorized; 50 shares issued and outstanding. . . . . . . . . . . . $ 5,000 Paid-in capital in excess of par value, preferred stock . . . . . . . . . . . . . . . . . . . . . . . . . . . . . . . . . . . . . . . . . . . . . . . 1,000 Common stock—$10 par value; 50,000 shares authorized; 30,000 shares issued and outstanding . . . . . . . . . . . . . 300,000 Retained earnings. . . . . . . . . . . . . . . . . . . . . . . . . . . . . . . . . . . . . . . . . . . . . . . . . . . . . . . . . . . . . . . . . . . . . . . . . . . . . . . 65,000 Total stockholders’ equity . . . . . . . . . . . . . . . . . . . . . . . . . . . . . . . . . . . . . . . . . . . . . . . . . . . . . . . . . . . . . . . . . . . . . . . . $371,000

Assets = Liabilities + Equity +6,000 +5,000 +1,000

July 1 Cash. . . . . . . . . . . . . . . . . . . . . . . . . . . . . . . . . . . . . . . . . . . . . . 6,000

Preferred Stock, $100 Par Value* . . . . . . . . . . . . . . . . . . 5,000

Paid-In Capital in Excess of Par Value, Preferred Stock† . . 1,000

Issued preferred stock for cash.

*$100 par value × 50 shares †$6,000 cash − [$100 par value × 50 shares]

Solution

Chapter 11 Corporate Reporting and Analysis 427

Dividend Preference of Preferred Stock Preferred stock has preference for dividends, meaning that preferred stockholders are paid their dividends before any dividends are paid to common stockholders. A preference for dividends does not guarantee dividends. If the directors do not declare a dividend, neither the preferred nor the common stockholders get dividends.

Cumulative or Noncumulative Most preferred stock has a cumulative dividend right. Cumulative preferred stock gives its owners a right to be paid both the current and all prior

periods’ unpaid dividends before any dividend is paid to common stockholders. When pre- ferred stock is cumulative and the directors either do not declare a dividend to preferred stockholders or declare one that does not cover the total amount of cumulative dividend, the unpaid dividend amount is called dividend in arrears. Accumulation of dividends in arrears on cumulative preferred stock does not guarantee they will be paid. Dividend in arrears is not a liability and is usually reported in notes to financial statements.

Noncumulative preferred stock does not have rights to prior periods’ unpaid dividends if they were not declared in those prior periods. It does have rights to current-period dividends.

To show the difference between cumulative and noncumulative preferred stock, assume that a corporation’s outstanding stock includes 1,000 shares of $100 par, 9% preferred stock—with potential dividends of $9,000 per year

(1,000 shares × $100 par × 9%). 4,000 shares of $50 par value common stock.

During 2018, the first year of operations, the directors declare cash dividends of $5,000. In 2019, they declare cash dividends of $42,000. Exhibit 11.8 shows the allocation of dividends. If the preferred stock is cumulative, the $4,000 in arrears is paid in 2019 before any other divi- dends are paid—shown in green below. With noncumulative preferred, the preferred stockhold- ers never receive the $4,000 skipped in 2018.

Point: Dividend preference does not mean that preferred stock- holders get more dividends than common stockholders.

EXHIBIT 11.8 Allocation of Dividends: Cumulative vs. Noncumulative

Preferred Stock Is Cumulative Preferred Common

Year 2018 . . . . . . . . . . . . . . . . . . . . . . . . . . . . . . . . . . $ 5,000 $ 0

Year 2019

Step 1: Dividend in arrears . . . . . . . . . . . . . . . . . . $ 4,000 Step 2: Current year’s preferred dividend . . . . . . 9,000

Step 3: Remainder to common . . . . . . . . . . . . . . . $29,000

Totals for year 2019 . . . . . . . . . . . . . . . . . . . . . . . . . . $13,000 $29,000

Totals for 2018–2019 . . . . . . . . . . . . . . . . . . . . . . . . $18,000 $29,000

Preferred Stock Is Noncumulative Preferred Common

Year 2018 . . . . . . . . . . . . . . . . . . . . . . . . . . . . . . . . $ 5,000 $ 0

Year 2019

Step 1: Current year’s preferred dividend . . . . $ 9,000

Step 2: Remainder to common . . . . . . . . . . . . . $33,000

Totals for 2018–2019 . . . . . . . . . . . . . . . . . . . . . . $14,000 $33,000

Participating or Nonparticipating Most preferred stock is nonparticipating. Nonparticipating preferred stock limits dividends each year. Once preferred stockholders

receive a stated amount, the common stockholders get any and all additional dividends. Participating preferred stock allows preferred stockholders to share with common stock-

holders any dividends paid in excess of the amount stated on the preferred stock. This par- ticipation feature applies after common stockholders get dividends equal to the preferred stock’s dividend percent.

Reasons for Issuing Preferred Stock Preferred stock is issued for several reasons. One reason is to raise money without giving up control. We can, for example, raise money by issuing preferred stock with no voting rights.

A second reason is to boost the return earned by common stockholders. Suppose a corpora- tion’s organizers expect to earn an annual after-tax income of $22,000 on an investment of $200,000. If they sell $200,000 worth of common stock, the $22,000 income produces an 11%

428 Chapter 11 Corporate Reporting and Analysis

return ($22,000∕$200,000). If they issue $150,000 of 8% preferred stock to outsiders and $50,000 of common stock to themselves, their own return increases to 20% ([$22,000 − $12,000]∕$50,000).

Use of preferred stock to increase return to common stockholders is an example of financial leverage. As a general rule, when the dividend rate on preferred stock is less than the rate the corporation earns on its assets, issuing preferred stock increases the rate earned by common stockholders.

Other reasons for issuing preferred stock include its appeal to some investors who believe that the corporation’s common stock is too risky or that the expected return on common stock is too low.

Concert Organizer Assume that you alter your business strategy from organizing concerts targeted at under 1,000 people to those targeted at between 5,000 and 20,000 people. You also incorporate because of an increased risk of lawsuits and a desire to issue stock for financing. It is important that you control the company for decisions on whom to schedule. What types of stock do you offer? ■ Answer: You have two options: (1) different classes of common stock or (2) common and preferred stock. You want to own stock that has all or a majority of voting power. The other class of stock, whether common or preferred, would have limited or no voting rights. In this way, you keep control and are able to raise money.

Decision Maker

A company’s outstanding stock consists of 80 shares of noncumulative 5% preferred stock with a $5 par value and also 200 shares of common stock with a $1 par value. During its first three years of operation, the corporation declared and paid the following total cash dividends.Allocating Cash

Dividends

NEED-TO-KNOW 11-3

C2 Part 1. Determine the amount of dividends paid each year to each of the two classes of stockholders: preferred and common. Also compute the total dividends paid to each class for the three years combined.

Part 2. Determine the amount of dividends paid each year to each of the two classes of stockholders as- suming that the preferred stock is cumulative. Also determine the total dividends paid to each class for the three years combined.

Solution—Part 2

Noncumulative Preferred Common

2018 ($15 paid) Preferred* . . . . . . . . . . . . . . . . . . . . . $15 Common—remainder . . . . . . . . . . . . $ 0 Total for the year . . . . . . . . . . . . . . . $15 $ 0

2019 ($5 paid) Preferred* . . . . . . . . . . . . . . . . . . . . . $ 5 Common—remainder . . . . . . . . . . . . $ 0 Total for the year . . . . . . . . . . . . . . . $ 5 $ 0

2020 ($200 paid) Preferred* . . . . . . . . . . . . . . . . . . . . . $20 Common—remainder . . . . . . . . . . . . $180 Total for the year . . . . . . . . . . . . . . . $20 $180

2018–2020 (combined $220 paid) Total for three years . . . . . . . . . . . . . $40 $180

* Holders of noncumulative preferred stock are entitled to no more than $20 of dividends in any one year (5% × $5 × 80 shares).

Cumulative Preferred Common

2018 ($15 paid) Preferred* . . . . . . . . . . . . . . . . . . . . . . . . . . . . . . . . . . . . . . . $15 Common—remainder . . . . . . . . . . . . . . . . . . . . . . . . . . . . . . $ 0 Total for the year . . . . . . . . . . . . . . . . . . . . . . . . . . . . . . . . . $15 $ 0 (Note: $5 in preferred dividends in arrears; [$20 × 1 yr] − $15 paid.) 2019 ($5 paid) Preferred—arrears from 2018 . . . . . . . . . . . . . . . . . . . . . . . $ 5 Preferred* . . . . . . . . . . . . . . . . . . . . . . . . . . . . . . . . . . . . . . . 0 Common—remainder . . . . . . . . . . . . . . . . . . . . . . . . . . . . . . $ 0 Total for the year . . . . . . . . . . . . . . . . . . . . . . . . . . . . . . . . . $ 5 $ 0 (Note: $20 in preferred dividends in arrears; [$20 × 2 yrs] − $15 paid − $5 paid.) 2020 ($200 paid) Preferred—arrears from 2019 . . . . . . . . . . . . . . . . . . . . . . . $20 Preferred* . . . . . . . . . . . . . . . . . . . . . . . . . . . . . . . . . . . . . . . 20 Common—remainder . . . . . . . . . . . . . . . . . . . . . . . . . . . . . . $160 Total for the year . . . . . . . . . . . . . . . . . . . . . . . . . . . . . . . . . $40 $160 (Note: $0 in preferred dividends in arrears; [$20 × 3 yrs] − $15 paid − $5 paid − $40 paid.) 2018–2020 (combined $220 paid) Total for three years . . . . . . . . . . . . . . . . . . . . . . . . . . . . . . . $60 $160

* Holders of cumulative preferred stock are entitled to $20 of dividends declared in any year (5% × $5 × 80 shares) plus any dividends in arrears.

Do More: QS 11-11, QS 11-12, QS 11-13, QS 11-14, E 11-9,

E 11-10, E 11-11

2018 total cash dividends . . . . $15 2019 total cash dividends. . . . $5 2020 total cash dividends. . . . . $200

No preferred stock 73%

Issued preferred stock 27%

Frequency of Preferred Stock

Solution—Part 1

Chapter 11 Corporate Reporting and Analysis 429

Corporations buy back their own stock for several reasons: (1) to use their shares to acquire another corporation, (2) to avoid a takeover of the company, (3) to give them to employees as compensation, and (4) to maintain a strong market for their stock or to show confidence in the current price.

A corporation’s reacquired shares are called treasury stock, which is similar to unissued stock in several ways: (1) neither treasury stock nor unissued stock is an asset, (2) neither receives cash dividends or stock dividends, and (3) neither has voting rights.

Purchasing Treasury Stock Purchasing treasury stock reduces the corporation’s assets and equity by equal amounts. We describe the cost method of accounting for treasury stock, which is the most popular method. (The par value method is explained in advanced courses.) The simple balance sheet below shows Cyber Inc.’s account balances before any treasury stock purchase (Cyber has no liabilities).

TREASURY STOCK P3 Record purchases and sales of treasury stock.

Corporations and Treasury Stock

With treasury

stock 62%

No treasury

stock 38%

Assets Stockholders’ Equity

Cash. . . . . . . . . . . . . . . . . . $ 30,000 Common stock—$10 par; 10,000 shares authorized, issued, and outstanding . . . . . . . . . . . . . . . . $100,000

Other assets . . . . . . . . . . . 95,000 Retained earnings . . . . . . . . . . . . . . . . . . . . . . . . . . . . . . . . . 25,000

Total assets . . . . . . . . . . . . $125,000 Total stockholders’ equity. . . . . . . . . . . . . . . . . . . . . . . . . . . $125,000

Cyber then purchases 1,000 of its own shares for $11,500. The entry below reduces equity with a debit to the Treasury Stock account, which is a contra equity account.

Assets = Liabilities + Equity −11,500 −11,500

May 1 Treasury Stock, Common* . . . . . . . . . . . . . . . . . . . . . . . . . . . . 11,500

Cash . . . . . . . . . . . . . . . . . . . . . . . . . . . . . . . . . . . . . . . . . 11,500

Purchased 1,000 treasury shares at $11.50 per share.

*$11.50 cost per share × 1,000 shares

Assets Stockholders’ Equity

Cash. . . . . . . . . . . . . . . . . . $ 18,500 Common stock—$10 par; 10,000 shares authorized and issued; 1,000 shares in treasury . . . . . . . . . . . . . . . $100,000

Other assets . . . . . . . . . . . 95,000 Retained earnings, $11,500 restricted by treasury stock purchase. . . 25,000

Less cost of treasury stock . . . . . . . . . . . . . . . . . . . . . . . . . . . . . . . . . . (11,500) Total assets . . . . . . . . . . . . $113,500 Total stockholders’ equity . . . . . . . . . . . . . . . . . . . . . . . . . . . . . . . . . . . . $113,500

Reissuing Treasury Stock Treasury stock can be reissued by selling it at cost, above cost, or below cost.

Selling Treasury Stock at Cost If treasury stock is reissued at cost, the entry is the reverse of the one made to record the purchase. If on May 21 Cyber reissues 100 of the treasury shares purchased on May 1 at the same $11.50 per share cost, the entry is

The balance sheet below shows account balances after this transaction. The treasury stock pur- chase reduces Cyber’s cash, total assets, and total equity by $11,500 but does not reduce Com- mon Stock or Retained Earnings. The stock description says that 1,000 issued shares are in treasury, leaving only 9,000 shares still outstanding. The description for retained earnings says that it is partly restricted.

Point: A treasury stock purchase is also called a stock buyback.

430 Chapter 11 Corporate Reporting and Analysis

Selling Treasury Stock above Cost If treasury stock is sold for more than cost, the amount received in excess of cost is credited to the Paid-In Capital, Treasury Stock account. This account is reported as a separate item in the stockholders’ equity section. No “gain” is ever reported from the sale of treasury stock. If Cyber receives $12 cash per share on June 3 for 400 treasury shares costing $11.50 per share, the entry is

Selling Treasury Stock below Cost When treasury stock is sold below cost, the entry depends on whether the Paid-In Capital, Treasury Stock account has a credit balance. If it has a zero balance, the excess of cost over the sales price is debited to Retained Earnings. If the Paid-In Capital, Treasury Stock account has a credit balance, it is debited for the excess of the cost over the selling price but not to exceed the credit balance. When the credit balance is eliminated, any remaining difference between the cost and selling price is debited to Retained Earnings. If Cyber sells its remaining 500 shares of treasury stock at $10 per share on July 10, equity is reduced by $750 (500 shares × $1.50 per share excess of cost over selling price), as shown below. This entry eliminates the $200 credit balance in the Paid-In Capital account cre- ated on June 3 and then reduces the Retained Earnings balance by the remaining $550. A com- pany never reports a “loss” from the sale of treasury stock.

Point: Paid-In Capital, Treasury Stock account can have a zero or credit balance but never a debit balance.

A company began the current year with the following balances in its stockholders’ equity accounts.

Recording Treasury Stock

NEED-TO-KNOW 11-4

P3

Common stock—$10 par, 500 shares authorized, 200 shares issued and outstanding . . . . . . . . . . $2,000

Paid-in capital in excess of par, common stock . . . . . . . . . . . . . . . . . . . . . . . . . . . . . . . . . . . . . . . . . . 1,000

Retained earnings. . . . . . . . . . . . . . . . . . . . . . . . . . . . . . . . . . . . . . . . . . . . . . . . . . . . . . . . . . . . . . . . . . 5,000

Total. . . . . . . . . . . . . . . . . . . . . . . . . . . . . . . . . . . . . . . . . . . . . . . . . . . . . . . . . . . . . . . . . . . . . . . . . . . . . $8,000

All outstanding common stock was issued for $15 per share when the company was created. Prepare jour- nal entries to account for the following transactions during the current year.

July 1 Purchased 30 shares of treasury stock at $20 per share. Sep. 1 Sold 20 treasury shares at $26 cash per share. Dec. 1 Sold the remaining 10 shares of treasury stock at $7 cash per share.

Assets = Liabilities + Equity +1,150 +1,150

May 21 Cash. . . . . . . . . . . . . . . . . . . . . . . . . . . . . . . . . . . . . . . . . . . . . . 1,150

Treasury Stock, Common* . . . . . . . . . . . . . . . . . . . . . . . . 1,150

Received $11.50 per share for 100 treasury shares costing $11.50 per share.

*$11.50 cost per share × 100 shares

June 3 Cash. . . . . . . . . . . . . . . . . . . . . . . . . . . . . . . . . . . . . . . . . . . . . . 4,800

Treasury Stock, Common* . . . . . . . . . . . . . . . . . . . . . . . . 4,600

Paid-In Capital, Treasury Stock†. . . . . . . . . . . . . . . . . . 200 Received $12 per share for 400 treasury

shares costing $11.50 per share.

*$11.50 cost per share × 400 shares †[$12 issue price − $11.50 cost per share] × 400 shares

Assets = Liabilities + Equity +4,800 +4,600 +200

July 10 Cash. . . . . . . . . . . . . . . . . . . . . . . . . . . . . . . . . . . . . . . . . . . . . . 5,000

Paid-In Capital, Treasury Stock* . . . . . . . . . . . . . . . . . . . . . . 200 Retained Earnings† . . . . . . . . . . . . . . . . . . . . . . . . . . . . . . . . . 550 Treasury Stock, Common‡ . . . . . . . . . . . . . . . . . . . . . . . . 5,750

Received $10 per share for 500 treasury shares costing $11.50 per share.

*[$10 issue price − $11.50 cost per share] × 500 shares; not to exceed $200 †For any amount exceeding $200 in Paid-In Capital, Treasury Stock ‡$11.50 cost per share × 500 shares

Assets = Liabilities + Equity +5,000 −200 −550 +5,750

Chapter 11 Corporate Reporting and Analysis 431

July 1 Treasury Stock, Commona . . . . . . . . . . . . . . . . . . . . . . . . . . . . . 600

Cash . . . . . . . . . . . . . . . . . . . . . . . . . . . . . . . . . . . . . . . . . . 600

Purchased 30 common shares at $20 per share. a30 shares × $20 cost

Sep. 1 Cashb . . . . . . . . . . . . . . . . . . . . . . . . . . . . . . . . . . . . . . . . . . . . . 520

Treasury Stock, Commonc . . . . . . . . . . . . . . . . . . . . . . . . 400

Paid-In Capital, Treasury Stock . . . . . . . . . . . . . . . . . . . . 120

Sold 20 treasury shares at $26 per share. b20 shares × $26 reissue price c20 shares × $20 cost

Dec. 1 Cashd . . . . . . . . . . . . . . . . . . . . . . . . . . . . . . . . . . . . . . . . . . . . . 70

Paid-In Capital, Treasury Stocke . . . . . . . . . . . . . . . . . . . . . . . . 120

Retained Earnings . . . . . . . . . . . . . . . . . . . . . . . . . . . . . . . . . . . 10

Treasury Stock, Commonf . . . . . . . . . . . . . . . . . . . . . . . . . 200

Sold 10 treasury shares at $7 per share. d10 shares × $7 reissue price

eNot to exceed existing balance f10 shares × $20 cost Do More: QS 11-15, E 11-12

Treasury Stock, Common

July 1 600 Sep. 1 400 Dec. 1 200

End. bal. 0

Statement of Retained Earnings Retained earnings generally consists of cumulative net income minus any net losses and divi- dends declared. Retained earnings does not mean that a certain amount of cash or other assets is available to pay stockholders. For example, Abercrombie & Fitch has $2,474,703 thousand in retained earnings, but only $547,189 thousand in cash.

Restrictions and Appropriations Restricted retained earnings are statutory and contractual restrictions. A common statutory (or legal) restriction is to limit treasury stock pur- chases to the amount of retained earnings. A common contractual restriction is a loan agree- ment that restricts paying dividends beyond a specified amount of retained earnings. Restrictions are usually described in the notes. Appropriated retained earnings is a voluntary transfer of amounts from the Retained Earnings account to the Appropriated Retained Earnings account to inform users of special activities that require funds.

Prior Period Adjustments Prior period adjustments are corrections of material errors in past financial statements. These errors include math errors, improper accounting, and missed facts. Prior period adjustments are reported in the statement of retained earnings, net of any income tax effects. Prior period adjustments result in changing the beginning balance of retained earnings for events occurring prior to the earliest period reported in the current set of financial statements. Assume that ComUS made an error two years ago in a journal entry for the purchase of land by incorrectly debiting an expense account. When this is discovered in the current year, the statement of retained earnings includes a prior period adjustment, as shown in Exhibit 11.9.

REPORTING OF EQUITY C3 Explain the items reported in retained earnings.

EXHIBIT 11.9 Statement of Retained Earnings with a Prior Period Adjustment

Statement of Retained Earnings

Retained earnings, Dec. 31, 2018, as previously reported . . . . . . . . . . . . . . . . . $4,700

Prior period adjustment

Cost of land incorrectly expensed (net of $60 of income tax benefit) . . . 200 Retained earnings, Dec. 31, 2018, as adjusted . . . . . . . . . . . . . . . . . . . . . . . . . . 4,900

Plus net income . . . . . . . . . . . . . . . . . . . . . . . . . . . . . . . . . . . . . . . . . . . . . . . . . . . 800

Less cash dividends declared . . . . . . . . . . . . . . . . . . . . . . . . . . . . . . . . . . . . . . . . (300)

Retained earnings, Dec. 31, 2019. . . . . . . . . . . . . . . . . . . . . . . . . . . . . . . . . . . . . $5,400

Solution

432 Chapter 11 Corporate Reporting and Analysis

Many items reported in financial statements are based on estimates. Future events reveal that some estimates were inaccurate even when based on the best data available at the time. These inaccuracies are not considered errors and are not reported as prior period adjustments. Instead, they are changes in accounting estimates and are accounted for in current and future periods.

Statement of Stockholders’ Equity A statement of stockholders’ equity lists the beginning and ending balances of key equity accounts and describes the changes that occur during the period. Exhibit 11.10 shows a condensed statement for Apple.

Statement of Stockholders’ Equity Common Common Retained Total $ millions, shares in thousands Stock Shares Stock Amount Earnings Other Equity

Beginning balance . . . . . . . . . . . . . . . . . . . . . . . . . 5,336,166 $31,251 $96,364 $ 634 $128,249 Net income . . . . . . . . . . . . . . . . . . . . . . . . . . . . . . . . — — 48,351 — 48,351

Issuance of common stock. . . . . . . . . . . . . . . . . . . . 36,531 (913) (581) — (1,494)

Repurchase of common stock & other. . . . . . . . . . . (246,496) 5,529 (33,001) (784) (28,256)

Cash dividends . . . . . . . . . . . . . . . . . . . . . . . . . . . . . — — (12,803) — (12,803)

Ending balance . . . . . . . . . . . . . . . . . . . . . . . . . . . . 5,126,201 $35,867 $98,330 $(150) $134,047

EXHIBIT 11.10 Statement of Stockholders’ Equity

Fake News Fake information can be used to pump up stock price and cause uninformed investors to buy the stock and drive up its price. After that, those who released fake information dump the stock at an inflated price. When later information reveals that the stock is overvalued, its price declines and investors still holding the stock lose value. This scheme is called pump ’n dump. A 15-year-old allegedly made about $1 million in one of the most infamous cases of pump ’n dump. (SEC Release No. 7891) ■

Ethical Risk

Earnings per Share, Price-Earnings Ratio, Dividend Yield, and Book Value per ShareDecision Analysis

Earnings per Share Earnings per share, also called EPS or net income per share, is the income earned per share of outstand- ing common stock. The basic earnings per share formula is in Exhibit 11.11. When a company has no preferred stock, then preferred dividends are zero. The weighted-average common shares outstanding is measured over the income reporting period; its computation is explained in advanced courses.

A1 Compute earnings per share and describe its use.

A2 Compute price-earnings ratio and describe its use in analysis.

EXHIBIT 11.11 Basic Earnings per Share Basic earnings per share =

Net income − Preferred dividends Weighted-average common shares outstanding

Assume Quantum Co. earns $40,000 net income in 2019 and declares dividends of $7,500 on its noncu- mulative preferred stock. (If preferred stock is noncumulative, preferred dividends are only subtracted if dividends are declared in that same period. If preferred stock is cumulative, preferred dividends are sub- tracted whether declared or not.) Quantum has 5,000 weighted-average common shares outstanding dur- ing 2019. Its basic EPS is $6.50, computed as ($40,000 − $7,500) / 5,000 shares.

Price-Earnings Ratio A comparison of a company’s EPS and its market value per share reveals market expectations. This com- parison is made using a price-earnings (or PE) ratio, also called price earnings or price to earnings. Some analysts interpret this ratio as what price the market is willing to pay for a company’s current earn- ings stream. Price-earnings ratios differ across companies that have similar earnings because of either higher or lower expectations of future earnings. The price-earnings ratio is in Exhibit 11.12.

Point: Diluted EPS is another EPS measure covered in advanced courses.

Price-earnings ratio = Market value (price) per share

Earnings per share

EXHIBIT 11.12 Price-Earnings Ratio

Price-earnings ratios for Visa and Mastercard follow. Both companies have relatively high PE ratios, show- ing that investors have high expectations of future earnings for both. Based on Mastercard’s higher PE versus Visa, one interpretation is the market is willing to pay more for Mastercard’s current earnings stream.

Point: The average PE ratio of stocks in the 1950–2019 period is about 14.

APPLE

Chapter 11 Corporate Reporting and Analysis 433

Company Market Value per Share Earnings per Share P/E Ratio

Visa . . . . . . . . . . . . . . . $105.24 $2.80 37.6

Mastercard . . . . . . . . . $151.36 $3.67 41.2

Dividend Yield Investors buy company stock to get a return from either or both cash dividends and stock price increases. Stocks that pay large dividends on a regular basis, called income stocks, are attractive to investors who want recurring cash flows from their investments. In contrast, growth stocks pay little or no cash dividends but are attractive to investors because of expected stock price increases. One way to help identify whether a stock is an income stock or a growth stock is to analyze its dividend yield. Dividend yield is defined in Exhibit 11.13.

Dividend yield = Annual cash dividends per share

Market value per share

EXHIBIT 11.13 Dividend Yield

The table below shows recent dividend and stock price data for Amazon and Altria Group to compute dividend yield. Dividend yield is zero for Amazon, implying it is a growth stock. An investor in Amazon expects increases in stock prices (and eventual cash from the sale of stock). Altria has a dividend yield of 5.0%, implying it is an income stock for which dividends are important in assessing its value.

Point: The payout ratio equals cash dividends declared on com- mon stock divided by net income. A low payout ratio suggests that it is retaining earnings for growth.

Company Cash Dividends per Share Market Value per Share Dividend Yield

Amazon . . . . . . . . . . . $0.00 $1,603 0.0%

Altria Group . . . . . . . . $2.80 $ 56 5.0%

Money Manager You plan to invest in one of two companies identified as having identical future prospects. One has a PE of 19 and the other a PE of 25. Which do you invest in? ■ Answer: Because one company requires a payment of $19 for each $1 of earnings and the other requires $25, you prefer the stock with a PE of 19; it is a better deal given identical prospects.

Decision Maker

A3 Compute dividend yield and explain its use in analysis.

A4 Compute book value and explain its use in analysis.

Book Value per Share Book value per common share, defined in Exhibit 11.14, is the amount of equity applicable to common shares on a per share basis. Book value per share is the value per share if a company is liquidated at bal- ance sheet amounts. Book value is also the starting point in many stock valuation models, merger negotia- tions, price setting for public utilities, and loan contracts. The main limitation in using book value is that the difference between market value and recorded value of assets and liabilities can be large.

Book value per common share = Stockholders’ equity applicable to common shares

Number of common shares outstanding

EXHIBIT 11.14 Book Value per Common Share

Consider LTD’s equity in the table below. At the current date there are two years of preferred dividends in arrears.

Preferred stock—$100 par value, 7% cumulative, 2,000 shares authorized, 1,000 shares issued and outstanding . . . . . $100,000

Common stock—$25 par value, 12,000 shares authorized, 10,000 shares issued and outstanding . . . . . . . . . . . . . . . 250,000

Paid-in capital in excess of par value, common stock . . . . . . . . . . . . . . . . . . . . . . . . . . . . . . . . . . . . . . . . . . . . . . . . . . . . 15,000

Retained earnings. . . . . . . . . . . . . . . . . . . . . . . . . . . . . . . . . . . . . . . . . . . . . . . . . . . . . . . . . . . . . . . . . . . . . . . . . . . . . . . . . 82,000

Total stockholders’ equity . . . . . . . . . . . . . . . . . . . . . . . . . . . . . . . . . . . . . . . . . . . . . . . . . . . . . . . . . . . . . . . . . . . . . . . . . . $447,000

LTD’s book value computations follow. Equity allocated to any preferred shares is removed before the book value of common shares is computed.

Total stockholders’ equity . . . . . . . . . . . . . . . . . . . . . . . . . . . . . . . . . . . . . . . . . . . . . . . . . . . . . . . . . . $ 447,000

Less equity applicable to preferred shares: Par value (1,000 shares × $100) . . . . . . . . . . . . . . . . $100,000 Dividends in arrears ($100,000 × 7% × 2 years) . . . 14,000 (114,000) Equity applicable to common shares. . . . . . . . . . . . . . . . . . . . . . . . . . . . . . . . . . . . . . . . . . . . . . . . . . $333,000

Book value per common share ($333,000∕10,000 shares) . . . . . . . . . . . . . . . . . . . . . . . . . . . . $ 33.30

434 Chapter 11 Corporate Reporting and Analysis

Barton Corporation began operations on January 1, 2018. The following transactions relating to stock- holders’ equity occurred in the first two years of the company’s operations.

2018

Jan. 1 Authorized the issuance of 2 million shares of $5 par value common stock and 100,000 shares of $100 par value, 10% cumulative preferred stock.

2 Issued 200,000 shares of common stock for $12 cash per share. 3 Issued 100,000 shares of common stock in exchange for a building valued at $820,000 and

merchandise inventory valued at $380,000. 4 Paid $10,000 cash to the company’s founders for organization activities. 5 Issued 12,000 shares of preferred stock for $110 cash per share.

2019

June 4 Issued 100,000 shares of common stock for $15 cash per share.

Required

1. Prepare journal entries to record these transactions. 2. Prepare the stockholders’ equity section of the balance sheet as of December 31, 2018 and 2019. 3. Prepare a table showing dividend allocations for 2018 and 2019 assuming Barton declares the follow-

ing cash dividends: 2018, $50,000, and 2019, $300,000. 4. Prepare the January 2, 2018, entry for issuance of 200,000 shares of common stock for $12 cash per share if a. Common stock is no-par stock without a stated value. b. Common stock is no-par stock with a stated value of $10 per share.

PLANNING THE SOLUTION Record journal entries for the transactions for 2018 and 2019. Determine the balances for the 2018 and 2019 equity accounts for the balance sheet. Prepare the contributed capital portion of the 2018 and 2019 balance sheets. Prepare a table similar to Exhibit 11.8 showing dividend allocations for 2018 and 2019. Record the issuance of common stock under both specifications of no-par stock.

SOLUTION 1. Journal entries.

COMPREHENSIVE

Issuance of, and Dividends to, Common and Preferred Stock; Reporting of Stockholders’ Equity

NEED-TO-KNOW 11-5

Jan. 2, 2018 Cash. . . . . . . . . . . . . . . . . . . . . . . . . . . . . . . . . . . . . . . . . . . . . . 2,400,000 Common Stock, $5 Par Value . . . . . . . . . . . . . . . . . . . . . . 1,000,000 Paid-In Capital in Excess of Par Value, Common Stock . . 1,400,000 Issued 200,000 shares of common stock.

Jan. 3, 2018 Building . . . . . . . . . . . . . . . . . . . . . . . . . . . . . . . . . . . . . . . . . . . 820,000 Merchandise Inventory . . . . . . . . . . . . . . . . . . . . . . . . . . . . . . . 380,000 Common Stock, $5 Par Value . . . . . . . . . . . . . . . . . . . . . . 500,000 Paid-In Capital in Excess of Par Value, Common Stock . . . . 700,000 Issued 100,000 shares of common stock.

Jan. 4, 2018 Organization Expenses . . . . . . . . . . . . . . . . . . . . . . . . . . . . . . . 10,000 Cash . . . . . . . . . . . . . . . . . . . . . . . . . . . . . . . . . . . . . . . . . . 10,000 Paid founders for organization costs.

Jan. 5, 2018 Cash . . . . . . . . . . . . . . . . . . . . . . . . . . . . . . . . . . . . . . . . . . . . . . 1,320,000 Preferred Stock, $100 Par Value . . . . . . . . . . . . . . . . . . . 1,200,000 Paid-In Capital in Excess of Par Value, Preferred Stock . . . . 120,000 Issued 12,000 shares of preferred stock.

June 4, 2019 Cash . . . . . . . . . . . . . . . . . . . . . . . . . . . . . . . . . . . . . . . . . . . . . . 1,500,000 Common Stock, $5 Par Value . . . . . . . . . . . . . . . . . . . . . . 500,000 Paid-In Capital in Excess of Par Value, Common Stock . . . . 1,000,000 Issued 100,000 shares of common stock.

Chapter 11 Corporate Reporting and Analysis 435

2. Balance sheet presentations (at December 31 year-end).

Stockholders’ Equity 2019 2018

Preferred stock—$100 par value, 10% cumulative, 100,000 shares authorized, 12,000 shares issued and outstanding . . . . . . . . . . . . . . . . . . . . . $1,200,000 $1,200,000

Paid-in capital in excess of par value, preferred stock . . . . . . . . . . . . . . . . . . . . . . . . . . . 120,000 120,000

Total paid-in capital by preferred stockholders. . . . . . . . . . . . . . . . . . . . . . . . . . . . . . . . . 1,320,000 1,320,000

Common stock—$5 par value, 2,000,000 shares authorized, 300,000 shares issued and outstanding in 2018, and 400,000 shares issued and outstanding in 2019. . . . . . . . . . . . . . . . . . . . . . . . . . . . . 2,000,000 1,500,000

Paid-in capital in excess of par value, common stock. . . . . . . . . . . . . . . . . . . . . . . . . . . . 3,100,000 2,100,000

Total paid-in capital by common stockholders . . . . . . . . . . . . . . . . . . . . . . . . . . . . . . . . . 5,100,000 3,600,000

Total paid-in capital. . . . . . . . . . . . . . . . . . . . . . . . . . . . . . . . . . . . . . . . . . . . . . . . . . . . . . . $6,420,000 $4,920,000

3. Dividend allocation table.

Common Preferred

2018 ($50,000) Preferred—current year (12,000 shares × $10 = $120,000) . . . . . . . . . . . . . . . . . . . . $ 0 $ 50,000 Common—remainder (300,000 shares outstanding) . . . . . . . . . . . . . . . . . . . . . . . . . . . 0 0

Total for the year . . . . . . . . . . . . . . . . . . . . . . . . . . . . . . . . . . . . . . . . . . . . . . . . . . . . . . . . $ 0 $ 50,000

2019 ($300,000) Preferred—dividend in arrears from 2018 ($120,000 − $50,000) . . . . . . . . . . . . . . . . $ 0 $ 70,000 Preferred—current year . . . . . . . . . . . . . . . . . . . . . . . . . . . . . . . . . . . . . . . . . . . . . . . . . . 0 120,000

Common—remainder (400,000 shares outstanding) . . . . . . . . . . . . . . . . . . . . . . . . . . . 110,000 0

Total for the year . . . . . . . . . . . . . . . . . . . . . . . . . . . . . . . . . . . . . . . . . . . . . . . . . . . . . . . . $110,000 $190,000

4. Journal entries. a. For 2018 (no-par stock without a stated value).

b. For 2018 (no-par stock with a stated value).

Jan. 2 Cash. . . . . . . . . . . . . . . . . . . . . . . . . . . . . . . . . . . . . . . . . . . . . . 2,400,000

Common Stock, No-Par Value . . . . . . . . . . . . . . . . . . . . . 2,400,000

Issued 200,000 shares of no-par stock at $12 per share.

Jan. 2 Cash. . . . . . . . . . . . . . . . . . . . . . . . . . . . . . . . . . . . . . . . . . . . . . 2,400,000

Common Stock, $10 Stated Value . . . . . . . . . . . . . . . . . 2,000,000

Paid-In Capital in Excess of Stated Value, Common Stock . 400,000

Issued 200,000 shares of $10 stated value common stock at $12 per share.

COMMON STOCK Corporate advantages: Separate legal entity, limited liability, transferable ownership, continuous life, no mutual agency for shareholders, and easier capital accumulation. Corporate disadvantages: More government regulation and corporate income taxes (double taxation).

Summary: Cheat Sheet

Issuing common stock at par value:

Cash . . . . . . . . . . . . . . . . . . . . . . . . . . . . . . . . . . . . . . . . . 300,000

Common Stock, $10 Par Value . . . . . . . . . . . . . . . . . . 300,000

Issuing no-par common stock:

Cash . . . . . . . . . . . . . . . . . . . . . . . . . . . . . . . . . . . . . . . . . 40,000

Common Stock, No-Par Value . . . . . . . . . . . . . . . . . . . 40,000

Issuing common stock above par: When market value > par value.

Cash . . . . . . . . . . . . . . . . . . . . . . . . . . . . . . . . . . . . . . . . . 360,000

Common Stock, $10 Par Value . . . . . . . . . . . . . . . . . . 300,000

Paid-In Capital in Excess of Par Value, Common Stock . 60,000

436 Chapter 11 Corporate Reporting and Analysis

Stock split: Distribution of additional shares to stockholders according to percent ownership. It does not affect any equity balances. No journal entry is made. Only effect is a change in stock account description.

Cumulative preferred stock: Preferred stockholders are paid both current and all prior periods’ unpaid dividends before any dividend is paid to com- mon stockholders. Dividend in arrears: Unpaid dividends due to cumulative preferred stock. Noncumulative preferred stock: Does not have rights to prior periods’ unpaid dividends, only current-period dividends.

TREASURY STOCK Treasury stock: Shares reacquired by the company. It reduces equity and does not receive dividends.

Issuing stated value common stock: When market value > stated value.

Cash . . . . . . . . . . . . . . . . . . . . . . . . . . . . . . . . . . . . . . . . . 50,000

Common Stock, $40 Stated Value . . . . . . . . . . . . . . . 40,000

Paid-in Capital in Excess of Stated Value, Common Stock 10,000

Small stock dividend: Distribution of 25% or less of previously outstand- ing shares. Retained earnings is capitalized for an amount equal to market value of shares.

Large stock dividend: Distribution of more than 25% of previously out- standing shares. Retained earnings is capitalized for an amount equal to par or stated value of shares.

Issuing common stock for noncash assets:

Land . . . . . . . . . . . . . . . . . . . . . . . . . . . . . . . . . . . . . . . . . 105,000

Common Stock, $20 Par Value . . . . . . . . . . . . . . . . . . 80,000

Paid-In Capital in Excess of Par Value, Common Stock 25,000

Small stock dividend—Date of declaration:

Retained Earnings . . . . . . . . . . . . . . . . . . . . . . . . . . . . . . 15,000

Common Stock Dividend Distributable . . . . . . . . . . . . 10,000

Paid-In Capital in Excess of Par Value, Common Stock 5,000

Issuing common stock in exchange for services:

Organization Expenses . . . . . . . . . . . . . . . . . . . . . . . . . . 12,000

Common Stock, $15 Par Value . . . . . . . . . . . . . . . . . . 9,000

Paid-In Capital in Excess of Par Value, Common Stock 3,000

DIVIDENDS

Declaration Record

Cash Dividend Dates

Payment

Cash dividend—Date of declaration:

Retained Earnings . . . . . . . . . . . . . . . . . . . . . . . . . . . . . . 5,000

Common Dividend Payable . . . . . . . . . . . . . . . . . . . . . 5,000

Cash dividend—Date of payment:

Common Dividend Payable . . . . . . . . . . . . . . . . . . . . . . . 5,000

Cash . . . . . . . . . . . . . . . . . . . . . . . . . . . . . . . . . . . . . . . 5,000

Small stock dividend—Date of payment:

Common Stock Dividend Distributable . . . . . . . . . . . . . . 10,000

Common Stock, $10 Par Value . . . . . . . . . . . . . . . . . . 10,000

Large stock dividend—Date of declaration:

Retained Earnings . . . . . . . . . . . . . . . . . . . . . . . . . . . . . . 30,000

Common Stock Dividend Distributable . . . . . . . . . . . . 30,000

Large stock dividend—Date of payment:

Common Stock Dividend Distributable . . . . . . . . . . . . . . 30,000

Common Stock, $10 Par Value . . . . . . . . . . . . . . . . . . 30,000

Treasury Stock, Common . . . . . . . . . . . . . . . . . . . . . . . . . 11,500

Cash . . . . . . . . . . . . . . . . . . . . . . . . . . . . . . . . . . . . . . . 11,500

Treasury stock in stockholders’ equity:

Stockholders’ Equity

Common stock—$10 par; 10,000 shares authorized and issued; 1,000 shares in treasury . . . . . . . . . . . . . . $100,000 Retained earnings, $11,500 restricted by treasury stock purchase . . . . 25,000 Less cost of treasury stock . . . . . . . . . . . . . . . . . . . . . . . . . . . . . . . . . (11,500) Total stockholders’ equity . . . . . . . . . . . . . . . . . . . . . . . . . . . . . . . . . . . $113,500

Selling treasury stock above cost: When sale price > reacquisition price.

Selling treasury stock below cost: When sale price < reacquisition price.

Cash . . . . . . . . . . . . . . . . . . . . . . . . . . . . . . . . . . . . . . . . . 5,000

Paid-In Capital, Treasury Stock . . . . . . . . . . . . . . . . . . . . 200

Retained Earnings . . . . . . . . . . . . . . . . . . . . . . . . . . . . . . 550

Treasury Stock, Common . . . . . . . . . . . . . . . . . . . . . . . 5,750

REPORTING AND ANALYSIS Prior period adjustments: Corrections of material errors in past financial statements. Errors include math errors, improper accounting, and missed facts. Prior period adjustments are reported in statement of retained earn- ings, net of any income tax effects. Changes in accounting estimates: Revised estimates that were inaccurate even when based on the best data available at the time. These are not errors and are not reported as prior period adjustments. Instead, they are accounted for in current and future periods.

Selling treasury stock at cost:

Cash . . . . . . . . . . . . . . . . . . . . . . . . . . . . . . . . . . . . . . . . . 1,150

Treasury Stock, Common . . . . . . . . . . . . . . . . . . . . . . . 1,150

Issuing preferred stock: When market value > par value.

Cash . . . . . . . . . . . . . . . . . . . . . . . . . . . . . . . . . . . . . . . . . 6,000

Preferred Stock, $100 Par Value . . . . . . . . . . . . . . . . . 5,000

Paid-In Capital in Excess of Par Value, Preferred Stock 1,000

PREFERRED STOCK

Statement of Retained Earnings

Retained earnings, Dec. 31, 2018, as previously reported . . . . . . . . . . . . . . $4,700 Prior period adjustment Cost of land incorrectly expensed (net of $60 of income tax benefit) . . . 200 Retained earnings, Dec. 31, 2018, as adjusted . . . . . . . . . . . . . . . . . . . . . . 4,900 Plus net income. . . . . . . . . . . . . . . . . . . . . . . . . . . . . . . . . . . . . . . . . . . . . . . . 800 Less cash dividends declared . . . . . . . . . . . . . . . . . . . . . . . . . . . . . . . . . . . . (300) Retained earnings, Dec. 31, 2019 . . . . . . . . . . . . . . . . . . . . . . . . . . . . . . . . . $5,400

Cash dividend—Date of record: No entry is made.

Cash . . . . . . . . . . . . . . . . . . . . . . . . . . . . . . . . . . . . . . . . . 4,800

Treasury Stock, Common . . . . . . . . . . . . . . . . . . . . . . . 4,600

Paid-In Capital, Treasury Stock . . . . . . . . . . . . . . . . . . 200

Chapter 11 Corporate Reporting and Analysis 437

Multiple Choice Quiz

1. A corporation issues 6,000 shares of $5 par value common stock for $8 cash per share. The entry to record this transac- tion includes a. A debit to Paid-In Capital in Excess of Par Value for

$18,000. b. A credit to Common Stock for $48,000. c. A credit to Paid-In Capital in Excess of Par Value for

$30,000. d. A credit to Cash for $48,000. e. A credit to Common Stock for $30,000.

2. A company reports net income of $75,000. Its weighted- average common shares outstanding is 19,000. It has no other stock outstanding. Its earnings per share is a. $4.69. c. $3.75. e. $4.41. b. $3.95. d. $2.08.

3. A company has 5,000 shares of $100 par preferred stock and 50,000 shares of $10 par common stock outstanding. Its to- tal stockholders’ equity is $2,000,000. Its book value per common share is a. $100.00. c. $40.00. e. $36.36. b. $10.00. d. $30.00.

4. A company paid cash dividends of $0.81 per share. Its earn- ings per share is $6.95 and its market price per share is $45.00. Its dividend yield is a. 1.8%. c. 15.4%. e. 8.6%. b. 11.7%. d. 55.6%.

5. A company’s shares have a market value of $85 per share. Its net income is $3,500,000, and its weighted-average common shares outstanding is 700,000. Its price-earnings ratio is a. 5.9. c. 17.0. e. 41.2. b. 425.0. d. 10.4.

ANSWERS TO MULTIPLE CHOICE QUIZ

1. e; Entry to record this stock issuance follows. 3. d; Preferred stock = 5,000 × $100 = $500,000; Book value per share = ($2,000,000 − $500,000)∕50,000 shares = $30 per common share

4. a; $0.81∕$45.00 = 1.8% 5. c; Earnings per share = $3,500,000/700,000 shares = $5 per

share; PE ratio = $85∕$5 = 17.0

Cash (6,000 × $8) . . . . . . . . . . . . . . . . . . . . . . . . . 48,000 Common Stock (6,000 × $5) . . . . . . . . . . . . . . . 30,000 Paid-In Capital in Excess of Par Value, Common Stock. . . . . . . . . . . . . . . . . . . . . . . . 18,000

Appropriated retained earnings (431) Authorized stock (419) Basic earnings per share (432) Book value per common share (433) Capital stock (419) Change in an accounting estimate (432) Common stock (418) Corporation (417) Cumulative preferred stock (427) Date of declaration (422) Date of payment (422) Date of record (422) Diluted earnings per share (432) Discount on stock (421) Dividend in arrears (427) Dividend yield (433)

Earnings per share (EPS) (432) Financial leverage (428) Large stock dividend (423) Liquidating cash dividend (423) Market value per share (419) Minimum legal capital (419) Noncumulative preferred stock (427) Nonparticipating preferred stock (427) No-par value stock (419) Organization expenses (costs) (418) Paid-in capital (420) Paid-in capital in excess of par value (420) Par value (419) Par value stock (419) Participating preferred stock (427) Preemptive right (418)

Preferred stock (426) Premium on stock (420) Price-earnings (PE) ratio (432) Prior period adjustment (431) Proxy (418) Restricted retained earnings (431) Retained earnings (420) Retained earnings deficit (423) Reverse stock split (425) Small stock dividend (423) Stated value stock (419) Statement of stockholders’ equity (432) Stock dividend (423) Stock split (425) Stockholders’ equity (419) Treasury stock (429)

Key Terms

Icon denotes assignments that involve decision making.

1. What are organization expenses? Provide examples. 2. How are organization expenses reported?

3. Who is responsible for overseeing corporate activities? 4. What is the difference between authorized shares and out-

standing shares?

Discussion Questions

2. b; $75,000∕19,000 shares = $3.95 per share

438 Chapter 11 Corporate Reporting and Analysis

5. What is the preemptive right of common stockholders? 6. List the general rights of common stockholders. 7. What is the difference between the market value per share

and the par value per share? 8. Identify and explain the importance of the three dates rele-

vant to corporate dividends. 9. Why is the term liquidating dividend used to describe cash

dividends debited against paid-in capital accounts? 10. How does declaring a stock dividend affect the corpo-

ration’s assets, liabilities, and total equity? What are the ef- fects of the eventual distribution of that stock?

11. What is the difference between a stock dividend and a stock split?

12. How does the purchase of treasury stock affect the purchas- er’s assets and total equity?

13. How are EPS results computed for a corporation with a simple capital structure?

14. How is book value per share computed for a corporation with no preferred stock? What is the main limitation of us- ing book value per share to value a corporation?

15. Refer to Apple’s fiscal 2017 balance sheet in Appendix A. How many shares of common stock are authorized? How many shares of common stock are issued and outstanding?

16. Refer to the 2017 balance sheet for Google in Appendix A. What is the par value per share of its preferred stock? Suggest a rationale for the amount of par value it assigned.

17. Refer to the financial statements for Samsung in Appendix A. How much were its cash payments for treasury stock acquisitions for the year ended December 31, 2017?

APPLE

Samsung

GOOGLE

QUICK STUDY

QS 11-1 Characteristics of corporations

C1

Identify which of the following statements are true for the corporate form of organization. 1. Ownership rights cannot be easily transferred. 2. Owners have unlimited liability for corporate debts. 3. Capital is more easily accumulated than with most other forms of organization. 4. Corporate income that is distributed to shareholders is usually taxed twice. 5. It is a separate legal entity. 6. It has a limited life. 7. Owners are not agents of the corporation.

Prepare the journal entry to record Zende Company’s issuance of 75,000 shares of $5 par value common stock assuming the shares sell for a. $5 cash per share. b. $6 cash per share.

QS 11-2 Issuance of common stock

P1

Prepare the journal entry to record Jevonte Company’s issuance of 36,000 shares of its common stock as- suming the shares have a a. $2 par value and sell for $18 cash per share. b. $2 stated value and sell for $18 cash per share.

QS 11-3 Issuance of par and stated value common stock P1

Prepare the issuer’s journal entry for each of the following separate transactions. a. On March 1, Atlantic Co. issues 42,500 shares of $4 par value common stock for $297,500 cash. b. On April 1, OP Co. issues no-par value common stock for $70,000 cash. c. On April 6, MPG issues 2,000 shares of $25 par value common stock for $45,000 of inventory,

$145,000 of machinery, and acceptance of a $94,000 note payable.

QS 11-5 Issuance of common stock

P1

Prepare journal entries to record the following transactions for Emerson Corporation.

July 15 Declared a cash dividend payable to common stockholders of $165,000. Aug. 15 Date of record is August 15 for the cash dividend declared on July 15. Aug. 31 Paid the dividend declared on July 15.

QS 11-6 Accounting for cash dividends

P2

Prepare the journal entry to record Autumn Company’s issuance of 63,000 shares of no-par value com- mon stock assuming the shares a. Sell for $29 cash per share. b. Are exchanged for land valued at $1,827,000.

QS 11-4 Issuance of no-par common stock P1

Epic Inc. has 10,000 shares of $2 par value common stock outstanding. Epic declares a 5% stock dividend on July 1 when the stock’s market value is $8 per share. The stock dividend is distributed on July 20. Prepare journal entries for (a) declaration and (b) distribution of the stock dividend.

QS 11-7 Accounting for small stock dividends P2

Chapter 11 Corporate Reporting and Analysis 439

Belkin Inc. has 100,000 shares of $3 par value common stock outstanding. Belkin declares a 40% stock dividend on March 2 when the stock’s market value is $72 per share. Prepare the journal entry for declara- tion of the stock dividend.

QS 11-9 Accounting for large stock dividends P2

QS 11-8 Accounting for small stock dividend

P2

The stockholders’ equity section of Jun Co.’s balance sheet as of April 1 follows. On April 2, Jun declares and distributes a 10% stock dividend. The stock’s per share market value on April 2 is $20 (prior to the dividend). Prepare the stockholders’ equity section immediately after the stock dividend is distributed.

Common stock—$5 par value, 375,000 shares authorized, 200,000 shares issued and outstanding. . . . . . . . . . $1,000,000

Paid-in capital in excess of par value, common stock . . . . . . . . . . . . . . . . . . . . . . . . . . . . . . . . . . . . . . . . . . . . . . . . 600,000

Retained earnings. . . . . . . . . . . . . . . . . . . . . . . . . . . . . . . . . . . . . . . . . . . . . . . . . . . . . . . . . . . . . . . . . . . . . . . . . . . . . 833,000

Total stockholders’ equity . . . . . . . . . . . . . . . . . . . . . . . . . . . . . . . . . . . . . . . . . . . . . . . . . . . . . . . . . . . . . . . . . . . . . . $2,433,000

QS 11-10 Accounting for dividends

P2

Indicate whether each of the following statements regarding dividends is true or false. 1. Cash and stock dividends reduce retained earnings. 2. Dividends payable is recorded at the time a cash dividend is declared. 3. The date of record is the date a cash dividend is paid to stockholders. 4. Stock dividends help keep the market price of stock affordable.

QS 11-15 Purchase and sale of treasury stock P3

On May 3, Zirbal Corporation purchased 4,000 shares of its own stock for $36,000 cash. On November 4, Zirbal reissued 850 shares of this treasury stock for $8,500. Prepare the May 3 and November 4 journal entries to record Zirbal’s purchase and reissuance of treasury stock.

QS 11-16 Impacts of stock issuances, dividends, splits, and treasury transactions

P2 P3

Identify whether stockholders’ equity would increase (I), decrease (D), or have no effect (NE) as a result of each separate transaction listed below.

1. A stock dividend equal to 30% of the previously outstanding shares is declared. 2. New shares of common stock are issued for cash. 3. Treasury shares of common stock are purchased. 4. Cash dividends are paid to shareholders.

QS 11-11 Preferred stock issuance and dividends C2

1. Prepare the journal entry to record Tamas Company’s issuance of 5,000 shares of $100 par value, 7% cumulative preferred stock for $102 cash per share.

2. Assuming the facts in part 1, if Tamas declares a year-end cash dividend, what is the amount of divi- dend paid to preferred shareholders? (Assume no dividends in arrears.)

QS 11-12 Dividend allocation between classes of shareholders C2

Stockholders’ equity of Ernst Company consists of 80,000 shares of $5 par value, 8% cumulative preferred stock and 250,000 shares of $1 par value common stock. Both classes of stock have been outstanding since the company’s inception. Ernst did not declare any dividends in the prior year, but it now declares and pays a $110,000 cash dividend at the current year-end. Determine the amount distributed to each class of stockholders for this two-year-old company.

QS 11-13 Dividends on noncumulative preferred stock

C2

Green Planet Corp. has 5,000 shares of noncumulative 10% preferred stock with a $2 par value and 17,000 shares of common stock with a $0.01 par value. During its first two years of operation, Green Planet declared and paid the following total cash dividends. Compute the dividends paid each year to each of the two classes of stockholders: preferred and common.

Year 1 total cash dividends . . . . $800 Year 2 total cash dividends . . . . $1,700

QS 11-14 Dividends on cumulative preferred stock C2

Use the information in QS 11-13 to compute the dividends paid each year to each of the two classes of stockholders assuming that the preferred stock is cumulative.

On December 31, Westworld Inc. has the following equity accounts and balances: Retained Earnings, $45,000; Common Stock, $1,000; Treasury Stock, $2,000; Paid-In Capital in Excess of Par Value, Common Stock, $39,000; Preferred Stock, $7,000; and Paid-In Capital in Excess of Par Value, Preferred Stock, $3,000. Prepare the stockholders’ equity section of Westworld’s balance sheet.

QS 11-17 Preparing stockholders’ equity section

C2 P1 P3

440 Chapter 11 Corporate Reporting and Analysis

On January 1, Payson Inc. had a retained earnings balance of $20,000. During the year, Payson reported net income of $30,000 and paid cash dividends of $17,000. Calculate the retained earnings balance at its December 31 year-end.

QS 11-19 Determining retained earnings balance C3

Indicate which activities of Stockton Corporation violated the rights of a stockholder who owned one share of common stock. 1. Did not allow the stockholder to sell the stock to her brother. 2. Rejected the stockholder’s request to be put in charge of its retail store. 3. Paid the stockholder a smaller dividend per share than another common stockholder. 4. Rejected the stockholder’s request to vote via proxy because she was home sick. 5. In liquidation, paid the common shareholder after all creditors were already paid.

Exercise 11-2 Rights of stockholders

C1

Murray Company reports net income of $770,000 for the year. It has no preferred stock, and its weighted- average common shares outstanding is 280,000 shares. Compute its basic earnings per share.

QS 11-20 Basic earnings per share A1

Epic Company earned net income of $900,000 this year. There were 400,000 weighted-average common shares outstanding, and preferred shareholders received a $20,000 cash dividend. Compute Epic Company’s basic earnings per share.

QS 11-21 Basic earnings per share A1

For each situation, identify whether it is treated as a prior period adjustment or change in accounting estimate. 1. A review of notes payable discovers that three years ago the company reported the entire amount of a

payment (principal and interest) on an installment note payable as interest expense. This mistake had a material effect on net income in that year.

2. After using an expected useful life of seven years and no salvage value to depreciate its office equip- ment over the preceding three years, the company decided early this year that the equipment will last only two more years.

3. Upon reviewing customer contracts, the company realizes it mistakenly reported $150,000 in revenue instead of the actual amount earned of $15,000. This mistake occurred two years ago and had a mate- rial effect on financial statements.

QS 11-18 Accounting for changes in estimates; error adjustments

C3

Compute Topp Company’s price-earnings ratio if its common stock has a market value of $20.54 per share and its EPS is $3.95. Its key competitor, Lower Deck, has a PE ratio of 9.5. For which company does the market have higher expectations of future performance?

QS 11-22 Price-earnings ratio

A2

Foxburo Company expects to pay a $2.34 per share cash dividend this year on its common stock. The cur- rent market value of Foxburo stock is $32.50 per share. Compute the expected dividend yield. If a com- petitor with a dividend yield of 3% is considered an income stock, would we classify Foxburo as a growth or an income stock?

QS 11-23 Dividend yield A3

EXERCISES

Exercise 11-1 Characteristics of corporations

C1

Next to each corporate characteristic 1 through 8, enter the letter of the description that best relates to it. 1. Owner authority and control 2. Ease of formation 3. Transferability of ownership 4. Ability to raise large capital amounts 5. Duration of life 6. Owner liability 7. Legal status 8. Tax status of income

a. Requires government approval b. Corporate income is taxed c. Separate legal entity d. Readily transferred e. One vote per share f. High ability g. Unlimited h. Limited

The stockholders’ equity section of Montel Company’s balance sheet follows. No preferred dividends are in arrears at the current date. Determine the book value per share of the common stock.

QS 11-24 Book value per common share

A4 Preferred stock—5% cumulative, $10 par value, 20,000 shares authorized, issued, and outstanding . . . . . . . . . $ 200,000 Common stock—$5 par value, 200,000 shares authorized, 150,000 shares issued and outstanding. . . . . . . . . . 750,000 Retained earnings. . . . . . . . . . . . . . . . . . . . . . . . . . . . . . . . . . . . . . . . . . . . . . . . . . . . . . . . . . . . . . . . . . . . . . . . . . . . . 900,000 Total stockholders’ equity . . . . . . . . . . . . . . . . . . . . . . . . . . . . . . . . . . . . . . . . . . . . . . . . . . . . . . . . . . . . . . . . . . . . . . $1,850,000

Chapter 11 Corporate Reporting and Analysis 441

Rodriguez Corporation issues 19,000 shares of its common stock for $152,000 cash on February 20. Prepare journal entries to record this event under each of the following separate situations. 1. The stock has a $2 par value. 3. The stock has a $5 stated value. 2. The stock has neither par nor stated value.

Exercise 11-3 Accounting for par, stated, and no-par stock issuances

P1

Exercise 11-5 Stock issuance for noncash assets P1

Sudoku Company issues 7,000 shares of $7 par value common stock in exchange for land and a building. The land is valued at $45,000 and the building at $85,000. Prepare the journal entry to record issuance of the stock in exchange for the land and building.

Exercise 11-7 Stock split P2

Refer to the information in Exercise 11-6. Assume that instead of distributing a stock dividend, Sharper did a 3-for-1 stock split. After the split, (1) prepare the updated stockholders’ equity section and (2) compute the number of shares outstanding. Hint: A 3-for-1 split means that each old share is replaced with 3 new shares.

On June 30, Sharper Corporation’s stockholders’ equity section of its balance sheet appears as follows before any stock dividend or split. Sharper declares and immediately distributes a 50% stock dividend. After the distribution is made, (1) prepare the updated stockholders’ equity section and (2) compute the number of shares outstanding.

Common stock—$10 par value, 50,000 shares issued and outstanding . . . . . . . . . . . . . . . . . . . . . . . . . . . . . . . . . $ 500,000 Paid-in capital in excess of par value, common stock . . . . . . . . . . . . . . . . . . . . . . . . . . . . . . . . . . . . . . . . . . . . . . . . 200,000 Retained earnings. . . . . . . . . . . . . . . . . . . . . . . . . . . . . . . . . . . . . . . . . . . . . . . . . . . . . . . . . . . . . . . . . . . . . . . . . . . . . 660,000 Total stockholders’ equity . . . . . . . . . . . . . . . . . . . . . . . . . . . . . . . . . . . . . . . . . . . . . . . . . . . . . . . . . . . . . . . . . . . . . . $1,360,000

Exercise 11-6 Large stock dividend

P2

Prepare journal entries to record each of the following four separate issuances of stock. 1. A corporation issued 4,000 shares of $5 par value common stock for $35,000 cash. 2. A corporation issued 2,000 shares of no-par common stock to its promoters in exchange for their

efforts, estimated to be worth $40,000. The stock has a $1 per share stated value. 3. A corporation issued 2,000 shares of no-par common stock to its promoters in exchange for their

efforts, estimated to be worth $40,000. The stock has no stated value. 4. A corporation issued 1,000 shares of $50 par value preferred stock for $60,000 cash.

Exercise 11-4 Recording stock issuances

P1

The stockholders’ equity section of TVX Company on February 4 follows. Exercise 11-8 Small stock dividend

P2Common stock—$10 par value, 150,000 shares authorized, 60,000 shares issued and outstanding. . . . . . . . . . $ 600,000 Paid-in capital in excess of par value, common stock . . . . . . . . . . . . . . . . . . . . . . . . . . . . . . . . . . . . . . . . . . . . . . . . 425,000 Retained earnings. . . . . . . . . . . . . . . . . . . . . . . . . . . . . . . . . . . . . . . . . . . . . . . . . . . . . . . . . . . . . . . . . . . . . . . . . . . . . 550,000 Total stockholders’ equity . . . . . . . . . . . . . . . . . . . . . . . . . . . . . . . . . . . . . . . . . . . . . . . . . . . . . . . . . . . . . . . . . . . . . . $1,575,000

On February 5, the directors declare a 20% stock dividend distributable on February 28 to the February 15 stockholders of record. The stock’s market value is $40 per share on February 5 before the stock dividend. 1. Prepare entries to record both the dividend declaration and its distribution. 2. Prepare the stockholders’ equity section after the stock dividend is distributed. (Assume no other

changes to equity.)

Match each description with the characteristic of preferred stock that it best describes. A. Cumulative B. Noncumulative C. Nonparticipating D. Participating

1. Receives current and all past dividends before common stockholders receive any dividends. 2. Receives dividends exceeding the stated rate under certain conditions. 3. Not entitled to receive dividends in excess of the stated rate. 4. Loses any dividends that are not declared in the current year.

Exercise 11-9 Identifying characteristics of preferred stock

C2

York’s outstanding stock consists of 80,000 shares of noncumulative 7.5% preferred stock with a $5 par value and also 200,000 shares of common stock with a $1 par value. During its first four years of opera- tion, the corporation declared and paid the following total cash dividends. Determine the amount of divi- dends paid each year to each of the two classes of stockholders: preferred and common. Also compute the total dividends paid to each class for the four years combined.

Exercise 11-10 Dividends on common and noncumulative preferred stock

C2

Year 1 total cash dividends . . . . . . . $20,000 Year 3 total cash dividends . . . . . . . $200,000 Year 2 total cash dividends . . . . . . . 28,000 Year 4 total cash dividends . . . . . . . 350,000 Check 4-year total paid to

preferred, $108,000

442 Chapter 11 Corporate Reporting and Analysis

Use the data in Exercise 11-10 to determine the amount of dividends paid each year to each of the two classes of stockholders assuming that the preferred stock is cumulative. Also determine the total dividends paid to each class for the four years combined.

Exercise 11-11 Dividends on common and cumulative preferred stock

C2

In Draco Corporation’s first year of business, the following transactions affected its equity accounts. Prepare the stockholders’ equity section of Draco’s balance sheet as of December 31.

∙ Issued 4,000 shares of $2 par value common stock for $18. It authorized 20,000 shares. ∙ Issued 1,000 shares of 12%, $10 par value preferred stock for $23. It authorized 3,000 shares. ∙ Reacquired 200 shares of common stock for $30 each. ∙ Retained earnings is impacted by reported net income of $50,000 and cash dividends of $15,000.

Exercise 11-13 Preparing stockholders’ equity section

C2 C3 P1 P3

Tuscan Inc. had a retained earnings balance of $60,000 at December 31, 2018. During the year, Tuscan had the following selected transactions. Calculate the retained earnings balance at December 31, 2019.

∙ Reported net income of $100,000. ∙ Revised an estimate of a machine’s salvage value. Depreciation increased by $1,000 per year. ∙ An error was discovered. Three years ago, a purchase of a building was incorrectly expensed. The

effect is understated retained earnings of $12,000 (net of tax benefit). ∙ Paid cash dividends of $33,000.

Exercise 11-14 Determining retained earnings balance

C3

The following information is from Amos Company for the year ended December 31, 2019. Prepare a statement of retained earnings for Amos Company.

∙ Retained earnings at December 31, 2018 (before discovery of error), $1,375,000. ∙ Cash dividends declared and paid during the year, $43,000. ∙ Two years ago, it forgot to record depreciation expense of $55,500 (net of tax benefit). ∙ The company earned $126,000 in net income this year.

Exercise 11-15 Preparing a statement of retained earnings

C3

On October 10, the stockholders’ equity section of Sherman Systems appears as follows.Exercise 11-12 Recording and reporting treasury stock transactions

P3 Common stock—$10 par value, 72,000 shares authorized, issued, and outstanding . . . . . . $ 720,000

Paid-in capital in excess of par value, common stock . . . . . . . . . . . . . . . . . . . . . . . . . . . . . . . 216,000

Retained earnings. . . . . . . . . . . . . . . . . . . . . . . . . . . . . . . . . . . . . . . . . . . . . . . . . . . . . . . . . . . . 864,000

Total stockholders’ equity . . . . . . . . . . . . . . . . . . . . . . . . . . . . . . . . . . . . . . . . . . . . . . . . . . . . . $1,800,000

1. Prepare journal entries to record the following transactions for Sherman Systems. a. Purchased 5,000 shares of its own common stock at $25 per share on October 11. b. Sold 1,000 treasury shares on November 1 for $31 cash per share. c. Sold all remaining treasury shares on November 25 for $20 cash per share. 2. Prepare the stockholders’ equity section after the October 11 treasury stock purchase.

Check (1c) Dr. Retained Earnings, $14,000

Kelley Company reports $960,000 of net income and declares $120,000 of cash dividends on its preferred stock for the year. At year-end, the company had 400,000 weighted-average shares of common stock. 1. What amount of net income is available to common stockholders? 2. What is the company’s basic EPS? Round your answer to the nearest whole cent.

Exercise 11-17 Earnings per share

A1 Check (2) $2.10

Ecker Company reports $2,700,000 of net income and declares $388,020 of cash dividends on its pre- ferred stock for the year. At year-end, the company had 678,000 weighted-average shares of common stock. 1. What amount of net income is available to common stockholders? 2. What is the company’s basic EPS?

Exercise 11-16 Earnings per share

A1

Check (2) $3.41

Chapter 11 Corporate Reporting and Analysis 443

Exercise 11-18 Price-earnings ratio computation and interpretation

A2

Compute the price-earnings ratio for each of these four separate companies. For which of these four com- panies does the market have the lowest expectation of future performance?

1

2

3

4

5

A B C

$12.00

10.00

7.50

50.00

Earnings per Share Market Value per ShareCompany

Hilton

SPG

Hyatt

Accor

$176.40

96.00

93.75

250.00

Exercise 11-19 Dividend yield computation and interpretation

A3

Compute the dividend yield for each of these four separate companies. Which company’s stock would probably not be classified as an income stock?

1

2

3

4

5

A B C

$16.06

13.86

3.96

0.48

Annual Cash Dividend per Share Market Value per ShareCompany

Etihad

United

Lingus

Allied

$220.00

132.00

72.00

80.00

Exercise 11-20 Book value per share

A4

The equity section of Cyril Corporation’s balance sheet shows the following.

Preferred stock—6% cumulative, $25 par value, 10,000 shares issued and outstanding. . . . . . . . . . . . . . . . . . . . $ 250,000

Common stock—$8 par value, 100,000 shares issued and outstanding . . . . . . . . . . . . . . . . . . . . . . . . . . . . . . . . . 800,000

Retained earnings. . . . . . . . . . . . . . . . . . . . . . . . . . . . . . . . . . . . . . . . . . . . . . . . . . . . . . . . . . . . . . . . . . . . . . . . . . . . . 535,000

Total stockholders’ equity . . . . . . . . . . . . . . . . . . . . . . . . . . . . . . . . . . . . . . . . . . . . . . . . . . . . . . . . . . . . . . . . . . . . . . $1,585,000

Determine the book value per share of common stock under two separate situations. 1. No preferred dividends are in arrears at the current date. 2. Three years of preferred dividends are in arrears at the current date.

Check (1) Book value of common, $13.35 per share

Common stock—$25 par value, 50,000 shares authorized, 30,000 shares issued and outstanding . . . . . . . . . . . $ 750,000

Paid-in capital in excess of par value, common stock . . . . . . . . . . . . . . . . . . . . . . . . . . . . . . . . . . . . . . . . . . . . . . . . 50,000

Retained earnings. . . . . . . . . . . . . . . . . . . . . . . . . . . . . . . . . . . . . . . . . . . . . . . . . . . . . . . . . . . . . . . . . . . . . . . . . . . . . 340,000

Total stockholders’ equity . . . . . . . . . . . . . . . . . . . . . . . . . . . . . . . . . . . . . . . . . . . . . . . . . . . . . . . . . . . . . . . . . . . . . . $1,140,000

Alexander Corporation reports the following components of stockholders’ equity at December 31, 2018. Exercise 11-21 Cash dividends, treasury stock, and statement of retained earnings

C3 P2 P3

During 2019, the following transactions affected its stockholders’ equity accounts.

Jan. 2 Purchased 3,000 shares of its own stock at $25 cash per share. Jan. 7 Directors declared a $1.50 per share cash dividend payable on February 28 to the February 9

stockholders of record. Feb. 28 Paid the dividend declared on January 7. July 9 Sold 1,200 of its treasury shares at $30 cash per share. Aug. 27 Sold 1,500 of its treasury shares at $20 cash per share. Sep. 9 Directors declared a $2 per share cash dividend payable on October 22 to the September 23

stockholders of record. Oct. 22 Paid the dividend declared on September 9. Dec. 31 Closed the $52,000 credit balance (from net income) in the Income Summary account to Re-

tained Earnings.

Required

1. Prepare journal entries to record each of these transactions. 2. Prepare a statement of retained earnings for the year ended December 31, 2019. 3. Prepare the stockholders’ equity section of the company’s balance sheet as of December 31, 2019.

444 Chapter 11 Corporate Reporting and Analysis

PROBLEM SET A

Problem 11-1A Stockholders’ equity transactions and analysis

P1 A4

Kinkaid Co. was incorporated at the beginning of this year and had a number of transactions. The follow- ing journal entries impacted its stockholders’ equity during its first year of operations.

Required

1. Explain the transaction(s) underlying each journal entry (a) through (d). 2. How many shares of common stock are outstanding at year-end? 3. What is the total paid-in capital at year-end? 4. What is the book value per share of the common stock at year-end if total paid-in capital plus retained

earnings equals $695,000?

Check (2) 20,000 shares

(3) $650,000

a. Cash . . . . . . . . . . . . . . . . . . . . . . . . . . . . . . . . . . . . . . . . . . . . . . . . . . . . . . 300,000 Common Stock, $25 Par Value . . . . . . . . . . . . . . . . . . . . . . . . . . . . . . . 250,000

Paid-In Capital in Excess of Par Value, Common Stock . . . . . . . . . . . . 50,000

b. Organization Expenses . . . . . . . . . . . . . . . . . . . . . . . . . . . . . . . . . . . . . . . 150,000 Common Stock, $25 Par Value . . . . . . . . . . . . . . . . . . . . . . . . . . . . . . . 125,000

Paid-In Capital in Excess of Par Value, Common Stock . . . . . . . . . . . . 25,000

c. Cash . . . . . . . . . . . . . . . . . . . . . . . . . . . . . . . . . . . . . . . . . . . . . . . . . . . . . . 43,000 Accounts Receivable . . . . . . . . . . . . . . . . . . . . . . . . . . . . . . . . . . . . . . . . . 15,000

Building. . . . . . . . . . . . . . . . . . . . . . . . . . . . . . . . . . . . . . . . . . . . . . . . . . . . 81,500

Notes Payable . . . . . . . . . . . . . . . . . . . . . . . . . . . . . . . . . . . . . . . . . . . . 59,500

Common Stock, $25 Par Value . . . . . . . . . . . . . . . . . . . . . . . . . . . . . . . 50,000

Paid-In Capital in Excess of Par Value, Common Stock . . . . . . . . . . . . 30,000

d. Cash . . . . . . . . . . . . . . . . . . . . . . . . . . . . . . . . . . . . . . . . . . . . . . . . . . . . . . 120,000 Common Stock, $25 Par Value . . . . . . . . . . . . . . . . . . . . . . . . . . . . . . . 75,000

Paid-In Capital in Excess of Par Value, Common Stock . . . . . . . . . . . . 45,000

Problem 11-2A Cash dividends, treasury stock, and statement of retained earnings

C3 P2 P3

Kohler Corporation reports the following components of stockholders’ equity at December 31, 2018.

During 2019, the following transactions affected its stockholders’ equity accounts.

Jan. 2 Purchased 4,000 shares of its own stock at $20 cash per share. Jan. 5 Directors declared a $2 per share cash dividend payable on February 28 to the February 5

stockholders of record. Feb. 28 Paid the dividend declared on January 5. July 6 Sold 1,500 of its treasury shares at $24 cash per share. Aug. 22 Sold 2,500 of its treasury shares at $17 cash per share. Sep. 5 Directors declared a $2 per share cash dividend payable on October 28 to the September 25

stockholders of record. Oct. 28 Paid the dividend declared on September 5. Dec. 31 Closed the $388,000 credit balance (from net income) in the Income Summary account to Re-

tained Earnings.

Required

1. Prepare journal entries to record each of these transactions. 2. Prepare a statement of retained earnings for the year ended December 31, 2019. 3. Prepare the stockholders’ equity section of the company’s balance sheet as of December 31, 2019.

Check (2) Ending retained earnings, $504,500

Common stock—$10 par value, 100,000 shares authorized, 40,000 shares issued and outstanding. . . . . . . . . . $400,000

Paid-in capital in excess of par value, common stock . . . . . . . . . . . . . . . . . . . . . . . . . . . . . . . . . . . . . . . . . . . . . . . . 60,000

Retained earnings. . . . . . . . . . . . . . . . . . . . . . . . . . . . . . . . . . . . . . . . . . . . . . . . . . . . . . . . . . . . . . . . . . . . . . . . . . . . . 270,000

Total stockholders’ equity . . . . . . . . . . . . . . . . . . . . . . . . . . . . . . . . . . . . . . . . . . . . . . . . . . . . . . . . . . . . . . . . . . . . . . $730,000

Chapter 11 Corporate Reporting and Analysis 445

Problem 11-3A Equity analysis—journal entries and account balances

P2

Required

1. Explain the transaction(s) underlying each journal entry. 2. Complete the following table showing the equity account balances at each indicated date (take into

account the beginning balances from September 30).

At September 30, the end of Beijing Company’s third quarter, the following stockholders’ equity accounts are reported.

In the fourth quarter, the following entries related to its equity are recorded.

Common stock, $12 par value. . . . . . . . . . . . . . . . . . . . . . . . . . . . . . . . . $360,000

Paid-in capital in excess of par value, common stock . . . . . . . . . . . . . . 90,000

Retained earnings. . . . . . . . . . . . . . . . . . . . . . . . . . . . . . . . . . . . . . . . . . . 320,000

Check Total equity: Oct. 2, $710,000; Dec. 31, $920,000

Sep. 30 Oct. 2 Oct. 25 Oct. 31 Nov. 5 Dec. 1 Dec. 31

Common stock . . . . . . . . . . . . . . . . . . . . $ $ $ $ $ $ $

Common stock dividend distributable . . . . . . . . . . . . . . . . . . .

Paid-in capital in excess of par, common stock . . . . . . . . . . . .

Retained earnings . . . . . . . . . . . . . . . . .

Total equity . . . . . . . . . . . . . . . . . . . . . . . $ $ $ $ $ $ $

360,000

0

90,000

320,000

770,000

The equity sections for Atticus Group at the beginning of the year (January 1) and end of the year (December 31) follow.

Problem 11-4A Analyzing changes in stockholders’ equity accounts

C3 P2 P3 Stockholders’ Equity (January 1) Common stock—$4 par value, 100,000 shares authorized, 40,000 shares issued and outstanding . . . . . . . . . . . . . $160,000

Paid-in capital in excess of par value, common stock . . . . . . . . . . . . . . . . . . . . . . . . . . . . . . . . . . . . . . . . . . . . . . . . . . 120,000

Retained earnings. . . . . . . . . . . . . . . . . . . . . . . . . . . . . . . . . . . . . . . . . . . . . . . . . . . . . . . . . . . . . . . . . . . . . . . . . . . . . . . 320,000

Total stockholders’ equity . . . . . . . . . . . . . . . . . . . . . . . . . . . . . . . . . . . . . . . . . . . . . . . . . . . . . . . . . . . . . . . . . . . . . . . . $600,000

Stockholders’ Equity (December 31) Common stock—$4 par value, 100,000 shares authorized, 47,400 shares issued, 3,000 shares in treasury. . . . . . $189,600

Paid-in capital in excess of par value, common stock . . . . . . . . . . . . . . . . . . . . . . . . . . . . . . . . . . . . . . . . . . . . . . . . . . 179,200

Retained earnings ($30,000 restricted by treasury stock) . . . . . . . . . . . . . . . . . . . . . . . . . . . . . . . . . . . . . . . . . . . . . . . 400,000

768,800

Less cost of treasury stock. . . . . . . . . . . . . . . . . . . . . . . . . . . . . . . . . . . . . . . . . . . . . . . . . . . . . . . . . . . . . . . . . . . . . . . . (30,000)

Total stockholders’ equity . . . . . . . . . . . . . . . . . . . . . . . . . . . . . . . . . . . . . . . . . . . . . . . . . . . . . . . . . . . . . . . . . . . . . . . . $738,800

Oct. 2 Retained Earnings. . . . . . . . . . . . . . . . . . . . . . . . . . . . . . . . . . . . . . . . . . . 60,000

Common Dividend Payable . . . . . . . . . . . . . . . . . . . . . . . . . . . . . . . 60,000

Oct. 25 Common Dividend Payable . . . . . . . . . . . . . . . . . . . . . . . . . . . . . . . . . . . 60,000

Cash . . . . . . . . . . . . . . . . . . . . . . . . . . . . . . . . . . . . . . . . . . . . . . . . . 60,000

Oct. 31 Retained Earnings. . . . . . . . . . . . . . . . . . . . . . . . . . . . . . . . . . . . . . . . . . . 75,000

Common Stock Dividend Distributable. . . . . . . . . . . . . . . . . . . . . . 36,000

Paid-In Capital in Excess of Par Value, Common Stock . . . . . . . . . 39,000

Nov. 5 Common Stock Dividend Distributable . . . . . . . . . . . . . . . . . . . . . . . . . . 36,000

Common Stock, $12 Par Value . . . . . . . . . . . . . . . . . . . . . . . . . . . . 36,000

Dec. 1 Memo—Change the title of the Common Stock account to reflect the new par value of $4.

Dec. 31 Income Summary . . . . . . . . . . . . . . . . . . . . . . . . . . . . . . . . . . . . . . . . . . . 210,000

Retained Earnings . . . . . . . . . . . . . . . . . . . . . . . . . . . . . . . . . . . . . . 210,000

446 Chapter 11 Corporate Reporting and Analysis

The following transactions and events affected its equity during the year. Jan. 5 Declared a $0.50 per share cash dividend, date of record January 10. Mar. 20 Purchased treasury stock for cash. Apr. 5 Declared a $0.50 per share cash dividend, date of record April 10. July 5 Declared a $0.50 per share cash dividend, date of record July 10. July 31 Declared a 20% stock dividend when the stock’s market value was $12 per share. Aug. 14 Issued the stock dividend that was declared on July 31. Oct. 5 Declared a $0.50 per share cash dividend, date of record October 10.

Required

1. How many common shares are outstanding on each cash dividend date? 2. What is the total dollar amount for each of the four cash dividends? 3. What is the amount of retained earnings transferred to paid-in capital accounts (capitalized) for the

stock dividend? 4. What is the per share cost of the treasury stock purchased? 5. How much net income did the company earn this year?

Check (3) $88,800

(4) $10

(5) $248,000

Raphael Corporation’s balance sheet shows the following stockholders’ equity section.Problem 11-5A Computing book values and dividend allocations

C2 A4

Required

1. What are the par values of the corporation’s preferred stock and its common stock? 2. If no dividends are in arrears at the current date, what is the book value per share of common stock?

Round per share value to the nearest cent. 3. If two years’ preferred dividends are in arrears at the current date, what is the book value per share of

common stock? Round per share value to the nearest cent. 4. If two years’ preferred dividends are in arrears at the current date and the board of directors declares cash

dividends of $11,500, what total amount will be paid to the preferred and to the common shareholders?

Check (3) Book value of common, $56.25

Preferred stock—5% cumulative, $___ par value, 1,000 shares authorized, issued, and outstanding . . . . . . . . . . . . $ 50,000

Common stock—$___ par value, 4,000 shares authorized, issued, and outstanding . . . . . . . . . . . . . . . . . . . . . . . . . 80,000

Retained earnings. . . . . . . . . . . . . . . . . . . . . . . . . . . . . . . . . . . . . . . . . . . . . . . . . . . . . . . . . . . . . . . . . . . . . . . . . . . . . . . 150,000

Total stockholders’ equity . . . . . . . . . . . . . . . . . . . . . . . . . . . . . . . . . . . . . . . . . . . . . . . . . . . . . . . . . . . . . . . . . . . . . . . . $280,000

PROBLEM SET B

Problem 11-1B Stockholders’ equity transactions and analysis

P1 A4

Weiss Company was incorporated at the beginning of this year and had a number of transactions. The fol- lowing journal entries impacted its stockholders’ equity during its first year of operations.

a. Cash . . . . . . . . . . . . . . . . . . . . . . . . . . . . . . . . . . . . . . . . . . . . . . . . . . . . . . 120,000 Common Stock, $1 Par Value . . . . . . . . . . . . . . . . . . . . . . . . . . . . . . 3,000

Paid-In Capital in Excess of Par Value, Common Stock . . . . . . . . . . 117,000

b. Organization Expenses . . . . . . . . . . . . . . . . . . . . . . . . . . . . . . . . . . . . . . . 40,000 Common Stock, $1 Par Value . . . . . . . . . . . . . . . . . . . . . . . . . . . . . . 1,000

Paid-In Capital in Excess of Par Value, Common Stock . . . . . . . . . . 39,000

c. Cash . . . . . . . . . . . . . . . . . . . . . . . . . . . . . . . . . . . . . . . . . . . . . . . . . . . . . . 13,300 Accounts Receivable . . . . . . . . . . . . . . . . . . . . . . . . . . . . . . . . . . . . . . . . . 8,000

Building. . . . . . . . . . . . . . . . . . . . . . . . . . . . . . . . . . . . . . . . . . . . . . . . . . . . 37,000

Notes Payable . . . . . . . . . . . . . . . . . . . . . . . . . . . . . . . . . . . . . . . . . . 18,300

Common Stock, $1 Par Value . . . . . . . . . . . . . . . . . . . . . . . . . . . . . . 800

Paid-In Capital in Excess of Par Value, Common Stock . . . . . . . . . . 39,200

d. Cash . . . . . . . . . . . . . . . . . . . . . . . . . . . . . . . . . . . . . . . . . . . . . . . . . . . . . . 60,000 Common Stock, $1 Par Value . . . . . . . . . . . . . . . . . . . . . . . . . . . . . . 1,200

Paid-In Capital in Excess of Par Value, Common Stock . . . . . . . . . . 58,800

Required

1. Explain the transaction(s) underlying each journal entry (a) through (d). 2. How many shares of common stock are outstanding at year-end?Check (2) 6,000 shares

Chapter 11 Corporate Reporting and Analysis 447

3. What is the total paid-in capital at year-end? 4. What is the book value per share of the common stock at year-end if total paid-in capital plus retained

earnings equals $283,200?

(3) $260,000

Problem 11-2B Cash dividends, treasury stock, and statement of retained earnings

C3 P2 P3

Balthus Corp. reports the following components of stockholders’ equity at December 31, 2018.

It completed the following transactions related to stockholders’ equity during 2019.

Jan. 10 Purchased 40,000 shares of its own stock at $12 cash per share. Mar. 2 Directors declared a $1.50 per share cash dividend payable on March 31 to the March 15 stock-

holders of record. Mar. 31 Paid the dividend declared on March 2. Nov. 11 Sold 24,000 of its treasury shares at $13 cash per share. Nov. 25 Sold 16,000 of its treasury shares at $9.50 cash per share. Dec. 1 Directors declared a $2.50 per share cash dividend payable on January 2 to the December 10

stockholders of record. Dec. 31 Closed the $1,072,000 credit balance (from net income) in the Income Summary account to

Retained Earnings.

Required

1. Prepare journal entries to record each of these transactions. 2. Prepare a statement of retained earnings for the year ended December 31, 2019. 3. Prepare the stockholders’ equity section of the company’s balance sheet as of December 31, 2019.

Check (2) Ending retained earnings, $2,476,000

Common stock—$1 par value, 320,000 shares authorized, 200,000 shares issued and outstanding. . . . . . . . . . . . $ 200,000

Paid-in capital in excess of par value, common stock . . . . . . . . . . . . . . . . . . . . . . . . . . . . . . . . . . . . . . . . . . . . . . . . . . 1,400,000

Retained earnings. . . . . . . . . . . . . . . . . . . . . . . . . . . . . . . . . . . . . . . . . . . . . . . . . . . . . . . . . . . . . . . . . . . . . . . . . . . . . . . 2,160,000

Total stockholders’ equity . . . . . . . . . . . . . . . . . . . . . . . . . . . . . . . . . . . . . . . . . . . . . . . . . . . . . . . . . . . . . . . . . . . . . . . . $3,760,000

In the fourth quarter, the following entries related to its equity are recorded.

Problem 11-3B Equity analysis—journal entries and account balances

P2

At December 31, the end of Chilton Communication’s third quarter, the following stockholders’ equity accounts are reported.

Common stock, $10 par value. . . . . . . . . . . . . . . . . . . . . . . . . . . . . . . . . . $ 960,000

Paid-in capital in excess of par value, common stock . . . . . . . . . . . . . . . 384,000

Retained earnings. . . . . . . . . . . . . . . . . . . . . . . . . . . . . . . . . . . . . . . . . . . . 1,600,000

Jan. 17 Retained Earnings. . . . . . . . . . . . . . . . . . . . . . . . . . . . . . . . . . . . . . . . . . . 96,000

Common Dividend Payable . . . . . . . . . . . . . . . . . . . . . . . . . . . . . . . 96,000

Feb. 5 Common Dividend Payable . . . . . . . . . . . . . . . . . . . . . . . . . . . . . . . . . . . 96,000

Cash . . . . . . . . . . . . . . . . . . . . . . . . . . . . . . . . . . . . . . . . . . . . . . . . . 96,000

Feb. 28 Retained Earnings. . . . . . . . . . . . . . . . . . . . . . . . . . . . . . . . . . . . . . . . . . . 252,000

Common Stock Dividend Distributable. . . . . . . . . . . . . . . . . . . . . . 120,000

Paid-In Capital in Excess of Par Value, Common Stock . . . . . . . . . 132,000

Mar. 14 Common Stock Dividend Distributable . . . . . . . . . . . . . . . . . . . . . . . . . . 120,000

Common Stock, $10 Par Value . . . . . . . . . . . . . . . . . . . . . . . . . . . . 120,000

Mar. 25 Memo—Change the title of the Common Stock account to reflect the new par value of $5.

Mar. 31 Income Summary . . . . . . . . . . . . . . . . . . . . . . . . . . . . . . . . . . . . . . . . . . . 720,000

Retained Earnings . . . . . . . . . . . . . . . . . . . . . . . . . . . . . . . . . . . . . . 720,000

448 Chapter 11 Corporate Reporting and Analysis

Required

1. Explain the transaction(s) underlying each journal entry. 2. Complete the following table showing the equity account balances at each indicated date (take into

account the beginning balances from December 31).

Dec. 31 Jan. 17 Feb. 5 Feb. 28 Mar. 14 Mar. 25 Mar. 31

Common stock . . . . . . . . . . . . . . . . . . . $ $ $ $ $ $ $

Common stock dividend distributable . . . . . . . . . . . . . . . . . .

Paid-in capital in excess of par, common stock . . . . . . . . . . . .

Retained earnings . . . . . . . . . . . . . . . .

Total equity . . . . . . . . . . . . . . . . . . . . . . $ $ $ $ $ $ $

960,000

0

384,000

1,600,000

2,944,000

Check Total equity: Jan. 17, $2,848,000; Mar. 31, $3,568,000

Stockholders’ Equity (January 1) Common stock—$20 par value, 30,000 shares authorized, 17,000 shares issued and outstanding . . . . . . . . . . . . . $340,000

Paid-in capital in excess of par value, common stock . . . . . . . . . . . . . . . . . . . . . . . . . . . . . . . . . . . . . . . . . . . . . . . . . . 60,000

Retained earnings. . . . . . . . . . . . . . . . . . . . . . . . . . . . . . . . . . . . . . . . . . . . . . . . . . . . . . . . . . . . . . . . . . . . . . . . . . . . . . . 270,000

Total stockholders’ equity . . . . . . . . . . . . . . . . . . . . . . . . . . . . . . . . . . . . . . . . . . . . . . . . . . . . . . . . . . . . . . . . . . . . . . . . $670,000

Stockholders’ Equity (December 31) Common stock—$20 par value, 30,000 shares authorized, 19,000 shares issued, 1,000 shares in treasury. . . . . . $380,000

Paid-in capital in excess of par value, common stock . . . . . . . . . . . . . . . . . . . . . . . . . . . . . . . . . . . . . . . . . . . . . . . . . . 104,000

Retained earnings ($40,000 restricted by treasury stock) . . . . . . . . . . . . . . . . . . . . . . . . . . . . . . . . . . . . . . . . . . . . . . . 295,200

779,200

Less cost of treasury stock. . . . . . . . . . . . . . . . . . . . . . . . . . . . . . . . . . . . . . . . . . . . . . . . . . . . . . . . . . . . . . . . . . . . . . . . (40,000)

Total stockholders’ equity . . . . . . . . . . . . . . . . . . . . . . . . . . . . . . . . . . . . . . . . . . . . . . . . . . . . . . . . . . . . . . . . . . . . . . . . $739,200

Problem 11-4B Analyzing changes in stockholders’ equity accounts

C3 P2 P3

The equity sections for Hovo Corp. at the beginning of the year (January 1) and end of the year (December 31) follow.

The following transactions and events affected its equity during the year. Feb. 15 Declared a $0.40 per share cash dividend, date of record five days later. Mar. 2 Purchased treasury stock for cash. May 15 Declared a $0.40 per share cash dividend, date of record five days later. Aug. 15 Declared a $0.40 per share cash dividend, date of record five days later. Oct. 4 Declared a 12.5% stock dividend when the stock’s market value is $42 per share. Oct. 20 Issued the stock dividend that was declared on October 4. Nov. 15 Declared a $0.40 per share cash dividend, date of record five days later.

Required

1. How many common shares are outstanding on each cash dividend date? 2. What is the total dollar amount for each of the four cash dividends? 3. What is the amount of retained earnings transferred to paid-in capital accounts (capitalized) for the

stock dividend? 4. What is the per share cost of the treasury stock purchased? 5. How much net income did the company earn this year?

Check (3) $84,000

(4) $40

(5) $136,000

Problem 11-5B Computing book values and dividend allocations

C2 A4

Soltech Company’s balance sheet shows the following stockholders’ equity section.

Preferred stock—8% cumulative, $___ par value, 1,500 shares authorized, issued, and outstanding . . . . . . . . . . . . $ 375,000

Common stock—$___ par value, 18,000 shares authorized, issued, and outstanding . . . . . . . . . . . . . . . . . . . . . . . . 900,000

Retained earnings. . . . . . . . . . . . . . . . . . . . . . . . . . . . . . . . . . . . . . . . . . . . . . . . . . . . . . . . . . . . . . . . . . . . . . . . . . . . . . . 1,125,000

Total stockholders’ equity . . . . . . . . . . . . . . . . . . . . . . . . . . . . . . . . . . . . . . . . . . . . . . . . . . . . . . . . . . . . . . . . . . . . . . . . $2,400,000

Chapter 11 Corporate Reporting and Analysis 449

Required

1. What are the par values of the corporation’s preferred stock and its common stock? 2. If no dividends are in arrears at the current date, what is the book value per share of common stock?

Round per share value to the nearest cent. 3. If two years’ preferred dividends are in arrears at the current date, what is the book value per share of

common stock? Round per share value to the nearest cent. 4. If two years’ preferred dividends are in arrears at the current date and the board of directors declares

cash dividends of $100,000, what total amount will be paid to the preferred and to the common shareholders?

Check (3) Book value of common, $109.17

SERIAL PROBLEM Business Solutions

P1 C1 C2

This serial problem began in Chapter 1 and continues through most of the book. If previous chapter seg- ments were not completed, the serial problem can begin at this point.

SP 11 Santana Rey created Business Solutions on October 1, 2019. The company has been successful, and Santana plans to expand her business. She believes that an additional $86,000 is needed and is inves- tigating three funding sources. a. Santana’s sister Cicely is willing to invest $86,000 in the business as a common shareholder. Because

Santana currently has about $129,000 invested in the business, Cicely’s investment will mean that Santana will maintain about 60% ownership and Cicely will have 40% ownership of Business Solutions.

b. Santana’s uncle Marcello is willing to invest $86,000 in the business as a preferred shareholder. Marcello would purchase 860 shares of $100 par value, 7% preferred stock.

c. Santana’s banker is willing to lend her $86,000 on a 7%, 10-year note payable. She would make monthly payments of $1,000 per month for 10 years.

Required

1. Prepare the journal entry to reflect the initial $86,000 investment under each of the options (a), (b), and (c).

2. Evaluate the three proposals for expansion, providing the pros and cons of each option. 3. Which option do you recommend Santana adopt? Explain.

©Alexander Image/Shutterstock

GENERAL LEDGER PROBLEM

The following General Ledger assignments highlight the impact, or lack thereof, on financial statements from equity-based transactions.

GL 11-1 General Ledger assignment 11-1 is adapted from Problem 11-2A, including beginning equity balances. Prepare journal entries related to treasury stock, cash dividends, and net income. Then prepare the statement of retained earnings and the stockholders’ equity section of the balance sheet.

GL 11-2 General Ledger assignment 11-2 is adapted from Problem 11-4A, including beginning and ending equity balances. Prepare journal entries related to cash dividends and stock dividends. Calculate the number of shares outstanding, the amount of net income, and the amount of retained earnings to be capitalized as a result of the stock dividend, if any.

GL

COMPANY ANALYSIS C2 A1 A4

Accounting Analysis

AA 11-1 Use Apple’s financial statements in Appendix A to answer the following. 1. How many shares of Apple common stock are issued and outstanding at (a) September 30, 2017, and

(b) September 24, 2016? 2. What is the total amount of cash dividends paid to common stockholders for the years ended

(a) September 30, 2017, and (b) September 24, 2016? 3. Identify basic EPS amounts for fiscal years (a) 2017 and (b) 2016. 4. Is the change in Apple’s EPS from 2016 to 2017 favorable or unfavorable? 5. If Apple buys back outstanding shares from investors, would you expect EPS to increase or decrease

from the buyback?

APPLE

450 Chapter 11 Corporate Reporting and Analysis

Required

1. Compute the book value per common share for each company using these data. 2. Compute the basic EPS for each company using these data. 3. Compute the dividend yield for each company using these data. 4. Compute the price-earnings ratio for each company using these data. 5. Based on the PE ratio, for which company do investors have greater expectations about future

performance?

AA 11-2 Use the following comparative figures for Apple and Google.COMPARATIVE ANALYSIS A1 A2 A3 A4 Key Figures Apple Google

Net income (in millions) . . . . . . . . . . . . . . . . . . . . . . . . . . . . . . . . . . . . . . . . . . . . . $ 48,351 $ 12,662

Cash dividends declared per common share . . . . . . . . . . . . . . . . . . . . . . . . . . . . $ 2.40 $ 0.00

Common shares outstanding (in millions) . . . . . . . . . . . . . . . . . . . . . . . . . . . . . . . 5,126.201 694.783

Weighted-average common shares outstanding (in millions) . . . . . . . . . . . . . . . 5,217.242 693.049

Market value (price) per share . . . . . . . . . . . . . . . . . . . . . . . . . . . . . . . . . . . . . . . . $ 154.12 $1,046.40

Equity applicable to common shares (in millions) . . . . . . . . . . . . . . . . . . . . . . . . . $ 134,047 $ 152,502

Required

1. Compute book value per share for Samsung. 2. Compute earnings per share (EPS) for Samsung. 3. If Samsung buys back outstanding shares from investors, would we expect EPS to increase or decrease

from the buyback?

AA 11-3 Use the following financial information for Samsung.GLOBAL ANALYSIS C3 A1

Net income less dividends available to preferred shares (in millions). . . . . . . . . . . . . . . . W 36,323,611

Number of common shares outstanding (in millions) . . . . . . . . . . . . . . . . . . . . . . . . . . . . . 119.688

Weighted-average common shares outstanding (in millions) . . . . . . . . . . . . . . . . . . . . . . 121.132

Equity applicable to common shares (in millions) . . . . . . . . . . . . . . . . . . . . . . . . . . . . . . . . W214,371,961

ETHICS CHALLENGE C3

BTN 11-1 Harriet Moore is an accountant for New World Pharmaceuticals. Her duties include tracking research and development spending in the new product development division. Over the course of the past six months, Harriet has noticed that a great deal of funds have been spent on a particular project for a new drug. She hears “through the grapevine” that the company is about to patent the drug and expects it to be a major advance in antibiotics. Harriet believes that this new drug will greatly improve company perfor- mance and will cause the company’s stock to increase in value. Harriet decides to purchase shares of New World in order to benefit from this expected increase.

Required

What are Harriet’s ethical responsibilities, if any, with respect to the information she has learned through her duties as an accountant for New World Pharmaceuticals? What are the implications of her planned purchase of New World shares?

Beyond the Numbers

BTN 11-2 Teams are to select an industry, and each team member is to select a different company in that industry. Each team member then is to acquire the selected company’s financial statements (or Form 10-K) from the SEC site (SEC.gov). Use these data to identify basic EPS. Use the financial press (or finance.yahoo.com) to determine the market price of this stock, and then compute the price-earnings ratio. Communicate with teammates via a meeting, e-mail, or telephone to discuss the meaning of this ratio, how companies compare, and the industry norm. The team must prepare a single memorandum reporting the ratio for each company and identifying the team conclusions or consensus of opinion. The memorandum is to be duplicated and distributed to the instructor and teammates.

COMMUNICATING IN PRACTICE A1 A2

Hint: Make a slide of each team’s memo for a class discussion.

Samsung

APPLE GOOGLE

Chapter 11 Corporate Reporting and Analysis 451

BTN 11-3 Access the March 1, 2017, filing of the 2016 calendar-year 10-K report of McDonald’s (ticker: MCD) from SEC.gov.

Required

1. Review McDonald’s balance sheet and identify how many classes of stock it has issued. 2. What are the par values, number of authorized shares, and number of issued shares of the classes of

stock you identified in part 1? 3. Review its statement of cash flows and identify what total amount of cash it paid in 2016 to purchase

treasury stock. 4. What amount did McDonald’s pay out in common stock cash dividends for 2016?

TAKING IT TO THE NET C1 C3

BTN 11-4 This activity requires teamwork to reinforce understanding of accounting for treasury stock. 1. Write a brief team statement (a) generalizing what happens to a corporation’s financial position when

it engages in a stock buyback and (b) identifying reasons why a corporation would engage in this activity.

2. Assume that an entity acquires 100 shares of its $100 par value common stock at a cost of $134 cash per share. Discuss the entry to record this acquisition. Next, assign each team member to prepare one of the following entries (assume each entry applies to all shares).

a. Reissue treasury shares at cost. b. Reissue treasury shares at $150 per share. c. Reissue treasury shares at $120 per share; assume the paid-in capital account from treasury shares

has a $1,500 balance. d. Reissue treasury shares at $120 per share; assume the paid-in capital account from treasury shares

has a $1,000 balance. e. Reissue treasury shares at $120 per share; assume the paid-in capital account from treasury shares

has a zero balance. 3. In sequence, each member is to present his/her entry to the team and explain the similarities and

differences between that entry and the previous entry.

TEAMWORK IN ACTION P3

Hint: Instructor must be sure each team accurately completes part 1 before proceeding.

BTN 11-5 Assume that Yelp decides to launch a new website to market discount bookkeeping services to consumers. This chain, named Aladin, requires $500,000 of start-up capital. The founder contributes $375,000 of personal assets in return for 15,000 shares of common stock, but he must raise another $125,000 in cash. There are two alternative plans for raising the additional cash.

∙ Plan A is to sell 3,750 shares of common stock to one or more investors for $125,000 cash. ∙ Plan B is to sell 1,250 shares of cumulative preferred stock to one or more investors for $125,000 cash

(this preferred stock would have a $100 par value, have an annual 8% dividend rate, and be issued at par). 1. If the new business is expected to earn $72,000 of after-tax net income in the first year, what rate of

return on beginning equity will the founder earn under each alternative plan? Which plan will provide the higher expected return?

2. If the new business is expected to earn $16,800 of after-tax net income in the first year, what rate of return on beginning equity will the founder earn under each alternative plan? Which plan will provide the higher expected return?

3. Analyze and interpret the differences between the results for parts 1 and 2.

ENTREPRENEURIAL DECISION C2 P2

BTN 11-6 Review 30 to 60 minutes of financial news programming on television. Take notes on compa- nies that are catching analysts’ attention. You might hear reference to over- and undervaluation of firms and to reports about PE ratios, dividend yields, and earnings per share. Be prepared to give a brief descrip- tion to the class of your observations.

HITTING THE ROAD A1 A2 A3

Design elements: Lightbulb: ©Chuhail/Getty Images; Blue globe: ©nidwlw/Getty Images and ©Dizzle52/Getty Images; Chess piece: ©Andrei Simonenko/Getty Images and ©Dizzle52/Getty Images; Mouse: ©Siede Preis/Getty Images; Global View globe: ©McGraw-Hill Education and ©Dizzle52/Getty Images; Sustainability: ©McGraw-Hill Education and ©Dizzle52/Getty Images

Learning Objectives

CONCEPTUAL C1 Distinguish between operating,

investing, and financing activities, and describe how noncash investing and financing activities are disclosed.

ANALYTICAL A1 Analyze the statement of cash flows and

apply the cash flow on total assets ratio.

P4 Appendix 12A—Illustrate use of a spreadsheet to prepare a statement of cash flows.

P5 Appendix 12B—Compute cash flows from operating activities using the direct method.

PROCEDURAL P1 Prepare a statement of cash flows.

P2 Compute cash flows from operating activities using the indirect method.

P3 Determine cash flows from both investing and financing activities.

Chapter Preview

12 Reporting Cash Flows

CASH FLOWS FROM OPERATING

P2 Indirect method Illustration of indirect method

Summary of indirect method adjustments

CASH FLOWS FROM INVESTING

P3 Three-step process of analysis

Analyzing noncurrent assets

Analyzing other assets

BASICS OF CASH FLOW REPORTING

C1 Purpose, measurement, and classification

Noncash activities

P1 Format and preparation

NTK 12-1

CASH FLOWS FROM FINANCING

P3 Three-step process of analysis

Analyzing noncurrent liabilities

Analyzing equity

Summary using T-accounts

A1 Analyzing cash

NTK 12-4NTK 12-2 NTK 12-3

453

“Work with people who have faith in you” —Barbara Bradley

True Colors

FORT WAYNE, IN—“I never saw myself going into business,” re- calls Barbara Bradley. Until one day, “we were at the airport when we noticed no one was carrying anything colorful or fun. So we decided to start a company to make handbags and lug- gage for women,” exclaims Barbara.

Barbara and her co-founder had no cash, so they borrowed $250 and started “cutting fabric out on a Ping-Pong table,” ex- plains Barbara. “We decided to name the company Vera Bradley (VeraBradley.com) after [my mother].”

As the business grew, Barbara had to manage cash flows. “The first year, we did $10,000 in sales,” proclaims Barbara. “Then things got chaotic.” While cash flows from operations were good, the business had to expand to meet demand.

“We went to a bank, seeking a $5,000 loan,” says Barbara. The loan was a welcome cash inflow that allowed the company to “build its own building!”

Barbara admits that she’s “not a great finance [and account- ing] person,” but she insists that accounting and attention to cash flows are key to running a successful business.

Although cash may be king, Barbara insists that “business is all about forming relationships. My father always said, ‘In busi- ness, you sell yourself first, your company second, and the product third,’ and he was right.”

Sources: Vera Bradley website, January 2019; Vera Bradley Foundation, January 2019; Fortune, October 2015

©Robin Marchant/Vera Bradley/Getty Images

Purpose of the Statement of Cash Flows The statement of cash flows reports cash receipts (inflows) and cash payments (outflows) for a period. Cash flows are separated into operating, investing, and financing activities. The details of sources and uses of cash make this statement useful. The statement of cash flows helps answer What explains the change in the cash balance? How does a company receive its cash? Where does a company spend its cash? Why do income and cash flows differ?

Importance of Cash Flows Information about cash flows influences decisions. Cash flows help users decide whether a company has enough cash to pay its debts. They also help evaluate a company’s ability to pursue opportunities. Managers use cash flow information to plan day-to-day operations and make long-term investment decisions.

W. T. Grant Co. is a classic example of the importance of cash flows. Grant reported net income of more than $40 million per year for three consecutive years. At that same time, cash outflow was more than $90 million by the end of that three-year period. Grant soon went bank- rupt. Users who relied only on Grant’s income numbers were caught off guard.

Measurement of Cash Flows Cash flows include both cash and cash equivalents. The statement of cash flows explains the difference between the beginning and ending balances of cash and cash equivalents. We con- tinue to use the phrases cash flows and the statement of cash flows, but remember that both phrases refer to cash and cash equivalents. Because cash and cash equivalents are combined, the statement of cash flows does not report transactions between cash and cash equivalents, such as cash paid to purchase cash equivalents and cash received from selling cash equivalents.

A cash equivalent has two criteria: (1) be readily convertible to a known amount of cash and (2) be sufficiently close to its maturity so its market value is unaffected by interest rate changes. American Express defines its cash equivalents as including “highly liquid investments with orig- inal maturities of 90 days or less.”

BASICS OF CASH FLOW REPORTING

Cash Equivalents

454 Chapter 12 Reporting Cash Flows

Classification of Cash Flows Cash receipts and cash payments are classified in one of three categories: operating, investing, or financing activities. A net cash inflow (source) occurs when the receipts in a category exceed the payments. A net cash outflow (use) occurs when the payments in a category exceed the receipts.

Operating Activities Operating activities include transactions and events that affect net income. Examples are the production and purchase of inventory, the sale of goods and ser- vices to customers, and the expenditures to operate the business. Not all items in income, such as unusual gains and losses, are operating activities (we discuss these exceptions later). Exhibit 12.1 lists common cash inflows and outflows from operating activities.

C1 Distinguish between operating, investing, and financing activities, and describe how noncash investing and financing activities are disclosed.

Operating Activities

Cash Outflows Cash Inflows

From collections on credit sales

From receipt of dividend revenue

From cash sales to customers

From receipt of interest revenue

To pay operating expenses

To pay taxes and fines

To pay interest owed

To pay salaries and wages

To pay suppliers for goods and services

EXHIBIT 12.1 Cash Flows from Operating Activities

Investing Activities Investing activities include transactions and events that come from the purchase and sale of long-term assets. They also include (1) the purchase and sale of short-term investments and (2) lending and collecting money for notes receivable. Exhibit 12.2 lists examples of cash flows from investing activities. Cash from collecting the principal on notes is an investing activity. However, collecting interest on notes is an operating activity; also, if a note results from sales to customers, it is an operating activity.

Point: For simplicity, we assume purchases and sales of equity and debt securities are investing activities.

Cash Inflows Cash Outflows

From selling long-term investments

From selling short-term investments

From collecting principal on notes

receivable

From selling plant assetsFrom selling

intangible assets

From selling (discounting) notes

receivable

To buy intangible assets

To buy short-term investments

To loan money in return for notes

receivable

To buy long-term investments

To buy plant assets

Investing Activities

EXHIBIT 12.2 Cash Flows from Investing Activities

Financing Activities Financing activities include transactions and events that affect long-term liabilities and equity. Examples are (1) getting cash from issuing debt and repaying debt and (2) receiving cash from or distributing cash to owners. Borrowing and repaying princi- pal on both short- and long-term debt are financing activities. However, payments of interest are operating activities. Exhibit 12.3 lists examples of cash flows from financing activities.

Financing Activities

Cash Inflows Cash OutflowsFrom issuing its common

and preferred stock

From issuing its short- and long-term debt (notes

payable and bonds payable)

From reissuing its treasury stock

From contributions by owners

To pay dividends to shareholders

Withdrawals by owners

To pay o� its short- and long-term debt (notes payable

and bonds payable)

To purchase treasury stock

EXHIBIT 12.3 Cash Flows from Financing Activities

Chapter 12 Reporting Cash Flows 455

Link between Classification of Cash Flows and the Balance Sheet Operating, investing, and financing activities are loosely linked to different parts of the balance sheet. Operating activities are affected by changes in current assets and current liabilities (and the in- come statement). Investing activities are affected by changes in long-term assets. Financing ac- tivities are affected by changes in long-term liabilities and equity. These links are shown in Exhibit 12.4. Exceptions to these links include (1) current assets unrelated to operations—such as short-term notes receivable from noncustomers and from investment securities, which are investing activities, and (2) current liabilities unrelated to operations—such as short-term notes payable and dividends payable, which are financing activities.

Current assets

Assets

Long-term assets

Operating cash flows

Investing cash flows

Financing cash flows

Current liabilities

Liabilities & Equity

Long-term liabilities & Equity

EXHIBIT 12.4 Linkage of Cash Flow Classifications to the Balance Sheet

Noncash Investing and Financing Some investing and financing activities do not affect cash flows. One example is the purchase of long-term assets using a long-term note payable (loan). This transaction impacts both investing and financing activities but does not impact current-period cash. Such transactions are reported at the bottom of the statement of cash flows or in a note to the statement—Exhibit 12.5 has examples.

EXHIBIT 12.5 Examples of Noncash Investing and Financing Activities

Retirement of debt by issuing equity stock.

Conversion of preferred stock to common stock.

Lease of assets in a long-term lease transaction.

Purchase of long-term assets by issuing a note or bond.

Exchange of noncash assets for other noncash assets.

Purchase of noncash assets by issuing equity or debt.

Format of the Statement of Cash Flows A statement of cash flows reports cash flows from three activities: operating, investing, and financing. Exhibit 12.6 shows the usual format. The statement shows the net increase or decrease from those activities and ties it into the cash balance. Any noncash investing and financing transactions are disclosed in a note or separate schedule.

P1 Prepare a statement of cash flows.

COMPANY NAME Statement of Cash Flows

For period Ended date

Cash flows from operating activities [Compute operating cash flows using indirect or direct method]

Net cash provided (used) by operating activities . . . . . . . . . . . . . . . . . . . . . . . . . . . . $ #

Cash flows from investing activities [List of individual inflows and outflows]

Net cash provided (used) by investing activities . . . . . . . . . . . . . . . . . . . . . . . . . . . . . #

Cash flows from financing activities [List of individual inflows and outflows]

Net cash provided (used) by financing activities . . . . . . . . . . . . . . . . . . . . . . . . . . . . . #

Net increase (decrease) in cash. . . . . . . . . . . . . . . . . . . . . . . . . . . . . . . . . . . . . . . . . . . $ # Cash (and equivalents) balance at prior period-end. . . . . . . . . . . . . . . . . . . . . . . . . . # Cash (and equivalents) balance at current period-end . . . . . . . . . . . . . . . . . . . . . . . $ #

Separate schedule or note disclosure of any noncash investing and financing transactions is required.

EXHIBIT 12.6 Format of the Statement of Cash Flows

Point: Positive cash flows for a section are titled net cash “provided by” or “from.” Negative cash flows are labeled as net cash “used by” or “for.”

456 Chapter 12 Reporting Cash Flows

Preparing the Statement of Cash Flows Preparing a statement of cash flows has five steps, shown in Exhibit 12.7. Computing the net increase or net decrease in cash is a simple but crucial computation. It equals the current peri- od’s cash balance minus the prior period’s cash balance. This is the bottom-line figure for the statement of cash flows and is a check on accuracy.

4 Compute net cash from or for financing activities.

5 Compute net cash from all sources; then prove it by adding it to beginning cash to get ending cash.

1 Compute net increase or decrease in cash.

3 Compute net cash from or for investing activities.

2 Compute net cash from or for operating activities.

EXHIBIT 12.7 Five Steps in Preparing the Statement of Cash Flows

Analyzing the Cash Account A company’s cash receipts and cash payments are recorded in its Cash account. The Cash account is one place to look for information about cash flows. The summarized Cash T-account of Genesis, Inc., is in Exhibit 12.8. Preparing a statement of cash flows requires classifying each cash inflow or outflow as an operating, investing, or financing activity.

Payments for inventory ................. Payments for operating exp. ....... Payments for interest .................... Payments for taxes ......................... Payments for notes retirement ... Payments for dividends ................

Balance, Dec. 31, 2018 ............... Receipts from customers .......... Receipts from asset sales ......... Receipts from stock issuance ..

Balance, Dec. 31, 2019 ...............

12,000 570,000

2,000 15,000

17,000

319,000 218,000

8,000 5,000

18,000 14,000

CashEXHIBIT 12.8 Summarized Cash Account

Cash = + Equity Noncash assetsLiabilities – EXHIBIT 12.9 Relation between Cash and Noncash Accounts

Information to Prepare the Statement Information to prepare the statement of cash flows comes from three sources: (1) comparative balance sheets, (2) the current income statement, and (3) additional information. Comparative balance sheets are used to compute changes in noncash accounts from the beginning to the end of the period. The current income statement is used to help compute cash flows from operating activities. Additional information includes details that help explain cash flows and noncash activities.

Additional InformationAdditional Information

Income StatementIncome Statement

Balance Sheets

Analyzing Noncash Accounts A second approach to preparing the statement of cash flows analyzes noncash accounts and uses double-entry accounting. Exhibit 12.9 uses the accounting equation to show the relation between the Cash account and the noncash balance sheet accounts. We can explain changes in cash and prepare a statement of cash flows by ana- lyzing changes in liability accounts, equity accounts, and noncash asset accounts (along with income statement accounts).

Chapter 12 Reporting Cash Flows 457

Entrepreneur You are considering purchasing a start-up business that recently reported a $110,000 annual net loss and a $225,000 annual net cash inflow. How are these results possible? ■ Answer: Several factors can explain an increase in net cash flows when a net loss is reported, including (1) early recognition of expenses relative to revenues generated (such as research and development), (2) cash advances on long-term sales contracts not yet recognized in income, (3) issuances of debt or equity for cash to finance expansion, (4) cash sale of assets, (5) delay of cash payments, and (6) cash prepayment on sales.

Decision Maker

Indirect and Direct Methods of Reporting Cash flows provided (used) by operating activities are reported using the direct method or the indirect method. These two different meth- ods apply only to the operating activities section. The direct method separately lists operating cash receipts (such as

cash received from customers) and operating cash payments (such as cash paid for inventory). The cash payments are then subtracted from cash receipts.

The indirect method reports net income and then adjusts it for items that do not affect cash. It does not report individual items of cash inflows and cash outflows from operating activities.

The net cash amount provided by operating activities is identical under both the direct and indirect methods. The difference is with the computation and presentation. The indirect method is arguably easier. Nearly all companies report operating cash flows using the indirect method, including Apple, Google, and Samsung in Appendix A.

Demonstration Data Exhibit 12.10 shows Genesis’s income statement and balance sheets. We use this information to prepare a statement of cash flows that explains the $5,000 increase in cash.

Applying the Indirect Method Net income is computed using accrual accounting. Revenues and expenses rarely match the receipt and payment of cash. The indirect method adjusts net income to get the net cash pro- vided or used by operating activities. We begin with Genesis’s income of $38,000 and adjust it

CASH FLOWS FROM OPERATING

Operating

DirectIndirect

1%

99%

Firms Using Indirect vs. Direct

Classify each of the following cash flows as operating, investing, or financing activities. a. Purchase equipment for cash g. Cash paid for utilities b. Cash payment of wages h. Cash paid to acquire investments c. Issuance of stock for cash i. Cash paid to retire debt d. Receipt of cash dividends from investments j. Cash received as interest on investments e. Cash collections from customers k. Cash received from selling investments f. Note payable issued for cash l. Cash received from a bank loan

Solution

a. Investing c. Financing e. Operating g. Operating i. Financing k. Investing b. Operating d. Operating f. Financing h. Investing j. Operating l. Financing

Classifying Cash Flows

NEED-TO-KNOW 12-1

C1 P1

Do More: QS 12-1, QS 12-2, E 12-1

458 Chapter 12 Reporting Cash Flows

GENESIS Balance Sheets

December 31, 2019 and 2018

2019 2018 Change

Assets Current assets Cash . . . . . . . . . . . . . . . . . . . . . . . $ 17,000 $ 12,000 $ 5,000 Increase Accounts receivable . . . . . . . . . . 60,000 40,000 20,000 Increase Inventory . . . . . . . . . . . . . . . . . . . 84,000 70,000 14,000 Increase Prepaid expenses . . . . . . . . . . . . 6,000 4,000 2,000 Increase Total current assets . . . . . . . . . . . 167,000 126,000 Long-term assets Plant assets . . . . . . . . . . . . . . . . . 250,000 210,000 40,000 Increase Accumulated depreciation . . . . . (60,000) (48,000) 12,000 Increase Total assets . . . . . . . . . . . . . . . . . . . . $357,000 $288,000

Liabilities Current liabilities Accounts payable. . . . . . . . . . . . . $ 35,000 $ 40,000 $ 5,000 Decrease Interest payable . . . . . . . . . . . . . . 3,000 4,000 1,000 Decrease Income taxes payable . . . . . . . . . 22,000 12,000 10,000 Increase Total current liabilities . . . . . . . . . 60,000 56,000 Long-term notes payable. . . . . . . . . 90,000 64,000 26,000 Increase Total liabilities. . . . . . . . . . . . . . . . . . 150,000 120,000

Equity Common stock, $5 par. . . . . . . . . . . 95,000 80,000 15,000 Increase Retained earnings . . . . . . . . . . . . . . 112,000 88,000 24,000 Increase Total equity . . . . . . . . . . . . . . . . . . . . 207,000 168,000 Total liabilities and equity . . . . . . . . $357,000 $288,000

GENESIS Income Statement

For Year Ended December 31, 2019

Sales . . . . . . . . . . . . . . . . . . . . . . . . . . . $590,000 Cost of goods sold . . . . . . . . . . . . . . . . $300,000 Wages and other operating expenses . . . . . . . . . . . . . 216,000 Interest expense. . . . . . . . . . . . . . . . . . 7,000 Depreciation expense . . . . . . . . . . . . . 24,000 (547,000)

43,000 Other gains (losses) Loss on sale of plant assets . . . . . . (6,000) Gain on retirement of notes . . . . . . 16,000 10,000 Income before taxes. . . . . . . . . . . . . . . 53,000 Income taxes expense . . . . . . . . . . . . . (15,000) Net income . . . . . . . . . . . . . . . . . . . . . . $ 38,000

Additional information for 2019 a. The accounts payable balances result from inventory purchases. b. Purchased $60,000 in plant assets by issuing $60,000 of notes

payable.

c. Sold plant assets with a book value of $8,000 (original cost of $20,000 and accumulated depreciation of $12,000) for $2,000 cash, yielding a $6,000 loss.

d. Received $15,000 cash from issuing 3,000 shares of common stock.

e. Paid $18,000 cash to retire notes with a $34,000 book value, yielding a $16,000 gain.

f. Declared and paid cash dividends of $14,000.

EXHIBIT 12.10 Financial Statements

EXHIBIT 12.11 Operating Activities Section—Indirect Method

GENESIS Statement of Cash Flows—Operating Section under Indirect Method

For Year Ended December 31, 2019

Cash flows from operating activities

Net income . . . . . . . . . . . . . . . . . . . . . . . . . . . . . . . . . . . . . . . . . . . . . . . . . . . . . . . . . $ 38,000

Adjustments to reconcile net income to net cash provided by operating activities

Income statement items not affecting cash

Depreciation expense. . . . . . . . . . . . . . . . . . . . . . . . . . . . . . . . . . . . . . . . . . . . 24,000

Loss on sale of plant assets . . . . . . . . . . . . . . . . . . . . . . . . . . . . . . . . . . . . . . . 6,000

Gain on retirement of notes . . . . . . . . . . . . . . . . . . . . . . . . . . . . . . . . . . . . . . . (16,000)

Changes in current assets and liabilities

Increase in accounts receivable. . . . . . . . . . . . . . . . . . . . . . . . . . . . . . . . . . . . (20,000)

Increase in inventory. . . . . . . . . . . . . . . . . . . . . . . . . . . . . . . . . . . . . . . . . . . . . (14,000)

Increase in prepaid expenses . . . . . . . . . . . . . . . . . . . . . . . . . . . . . . . . . . . . . (2,000)

Decrease in accounts payable . . . . . . . . . . . . . . . . . . . . . . . . . . . . . . . . . . . . . (5,000)

Decrease in interest payable . . . . . . . . . . . . . . . . . . . . . . . . . . . . . . . . . . . . . . (1,000)

Increase in income taxes payable . . . . . . . . . . . . . . . . . . . . . . . . . . . . . . . . . . 10,000

Net cash provided by operating activities . . . . . . . . . . . . . . . . . . . . . . . . . . . . . . $20,000

⎫ ⎪ ⎪ ⎬ ⎪ ⎪ ⎭

⎫ ⎪ ⎪ ⎪ ⎪ ⎪ ⎬ ⎪ ⎪ ⎪ ⎪ ⎪ ⎭

1

2

to get cash provided by operating activities of $20,000—see Exhibit 12.11. There are two types of adjustments: 1 Adjustments to income statement items that do not impact cash and 2 Adjustments for changes in current assets and current liabilities (linked to operating activi- ties). Nearly all companies group adjustments into these two types, including Apple, Google, and Samsung in Appendix A.

Chapter 12 Reporting Cash Flows 459

1 Adjustments for Income Statement Items Not Affecting Cash Some expenses and losses subtracted from net income were not cash outflows. Examples are depre- ciation, amortization, depletion, bad debts expense, loss from an asset sale, and loss from retire- ment of notes payable. The indirect method requires that

Expenses and losses with no cash outflows are added back to net income.

These expenses and losses did not reduce cash, and adding them back cancels their deductions from net income. Any cash received or paid from a transaction that yields a loss, such as from an asset sale or payoff of a note, is reported under investing or financing activities.

When net income has revenues and gains that are not cash inflows, the indirect method requires that

Revenues and gains with no cash inflows are subtracted from net income.

Section 1 of Exhibit 12.11 shows three adjustments for items that did not impact cash for Genesis.

Depreciation Depreciation expense is Genesis’s only operating item in net income that had no effect on cash flows. We add back the $24,000 depreciation expense to net income because depreciation did not reduce cash.

Loss on Sale of Plant Assets Genesis reported a $6,000 loss on sale of plant assets that reduced net income but did not affect cash flows. This $6,000 loss is added back to net income because it is not a cash outflow.

Gain on Retirement of Debt A $16,000 gain on retirement of debt increased net income but did not affect cash flows. This $16,000 gain is subtracted from net income because it was not a cash inflow.

2 Adjustments for Changes in Current Assets and Current Liabilities This section covers adjustments for changes in current assets and current liabilities.

Adjustments for Changes in Current Assets

Decreases in current assets are added to net income.

Increases in current assets are subtracted from net income.

Adjustments for Changes in Current Liabilities

Increases in current liabilities are added to net income.

Decreases in current liabilities are subtracted from net income.

The lower section of Exhibit 12.11 shows adjustments to the three noncash current assets and three current liabilities for Genesis. We explain each adjustment next.

Accounts Receivable The $20,000 increase in the current asset of accounts receivable is subtracted from income (showing less cash available). This increase means Genesis collects less cash than is reported in sales. To help see this, we use account analysis. This involves setting up a T-account, entering in black the balances and entries we know, and computing in red the cash receipts or payments. We see cash receipts are $20,000 less than sales, which is why we subtract $20,000 from income in computing the cash flow.

P2 Compute cash flows from operating activities using the indirect method.

Point: An income statement re- ports revenues, gains, expenses, and losses on an accrual basis. The statement of cash flows re- ports cash received and cash paid for operating, financing, and investing activities.

Point: Section 2 adjustments.

Account Account Increases Decreases

Current Subtract from Add to assets . . . . . net income net income

Current Add to Subtract from liabilities . . . net income net income

Accounts Receivable

Bal., Dec. 31, 2018 40,000

Sales 590,000 Cash receipts = 570,000

Bal., Dec. 31, 2019 60,000

Black numbers are from Exhibit 12.10. Red number

is computed. 40,000 + 590,000 − 60,000

460 Chapter 12 Reporting Cash Flows

Inventory The $14,000 increase in inventory is subtracted from income. The T-account shows that purchases are $14,000 more than cost of goods sold. This means that cost of goods sold excludes $14,000 of inventory purchased this year, which is why we subtract $14,000 from income in computing cash flow.

Inventory Bal., Dec. 31, 2018 70,000 Purchases = 314,000 Cost of goods sold 300,000 Bal., Dec. 31, 2019 84,000

Prepaid Expenses The $2,000 increase in prepaid expenses is subtracted from income. The T-account shows that cash paid is $2,000 more than expenses recorded, which is why we subtract $2,000 from income in computing cash flow.

Prepaid Expenses Bal., Dec. 31, 2018 4,000 Cash payments = 218,000 Wages and other

operating exp. 216,000

Bal., Dec. 31, 2019 6,000

Accounts Payable Bal., Dec. 31, 2018 40,000 Cash payments = 319,000 Purchases 314,000 Bal., Dec. 31, 2019 35,000

Accounts Payable The $5,000 decrease in accounts payable is subtracted from income. The T-account shows that cash paid is $5,000 more than purchases recorded, which is why we subtract $5,000 from income in computing cash flow.

Interest Payable Bal., Dec. 31, 2018 4,000

Cash paid for interest = 8,000 Interest expense 7,000 Bal., Dec. 31, 2019 3,000

Interest Payable The $1,000 decrease in interest payable is sub- tracted from income. The T-account shows that cash paid is $1,000 more than interest expense recorded, which is why we subtract $1,000 from income in computing cash flow.

Income Taxes Payable The $10,000 increase in income taxes payable is added to income. The T-account shows that cash paid is $10,000 less than tax expense recorded, which is why we add $10,000 to income in computing cash flow.

Income Taxes Payable Bal., Dec. 31, 2018 12,000

Cash paid for taxes = 5,000 Income taxes expense 15,000 Bal., Dec. 31, 2019 22,000

Summary of Adjustments for Indirect Method Exhibit 12.12 summarizes the adjustments to net income under the indirect method.

Net Income (or Loss) Adjustments for operating items not providing or using cash

+ Noncash expenses and losses Examples: Expenses for depreciation, depletion, and amortization; losses from disposal

of long-term assets and from retirement of debt

− Noncash revenues and gains Examples: Gains from disposal of long-term assets and from retirement of debt

Adjustments for changes in current assets and current liabilities

+ Decrease in noncash current operating asset − Increase in noncash current operating asset + Increase in current operating liability − Decrease in current operating liability Net cash provided (used) by operating activities

1

2

EXHIBIT 12.12 Summary of Adjustments for Operating Activities— Indirect Method

One for the Road Even though Tesla reported net losses and large cash outflows, its market value tripled in five years. Tesla now rivals both GM and Ford as one of the most valued U.S. automakers. Investors are counting on Tesla’s Model 3 to create positive operating cash flows. So far, Tesla has funded its operations with cash inflows from stock and debt issuances. ■

Decision Insight

©Mark Lennihan/AP Images

Chapter 12 Reporting Cash Flows 461

A company’s current-year income statement and selected balance sheet data at December 31 of the cur- rent and prior years follow. Prepare the operating activities section of the statement of cash flows using the indirect method for the current year.

P2

Reporting Operating Cash Flows (Indirect)

NEED-TO-KNOW 12-2

Income Statement For Current Year Ended December 31

Sales revenue . . . . . . . . . . . . . . . . . . . . $120

Expenses: Cost of goods sold. . . . . . . . 50

Depreciation expense . . . . . 30

Salaries expense . . . . . . . . . 17

Interest expense . . . . . . . . . 3

Net income. . . . . . . . . . . . . . . . . . . . . . . $ 20

Selected Balance Sheet Accounts

At December 31 Current Yr Prior Yr Accounts receivable . . . . $12 $10

Inventory . . . . . . . . . . . . . 6 9

Accounts payable . . . . . . 7 11

Salaries payable . . . . . . . 8 3

Interest payable. . . . . . . . 1 0

Solution

Cash Flows from Operating Activities—Indirect Method For Current Year Ended December 31

Cash flows from operating activities Net income. . . . . . . . . . . . . . . . . . . . . . . . . . . . . . . . . . . . . . . . . . . . . . . . . . . . . . . . . . . . . . . . . $20

Adjustments to reconcile net income to net cash provided by operating activities

Income statement items not affecting cash

Depreciation expense . . . . . . . . . . . . . . . . . . . . . . . . . . . . . . . . . . . . . . . . . . . . . . . . . . . $30

Changes in current assets and current liabilities

Increase in accounts receivable . . . . . . . . . . . . . . . . . . . . . . . . . . . . . . . . . . . . . . . . . . . (2)

Decrease in inventory . . . . . . . . . . . . . . . . . . . . . . . . . . . . . . . . . . . . . . . . . . . . . . . . . . . 3

Decrease in accounts payable. . . . . . . . . . . . . . . . . . . . . . . . . . . . . . . . . . . . . . . . . . . . . (4)

Increase in salaries payable . . . . . . . . . . . . . . . . . . . . . . . . . . . . . . . . . . . . . . . . . . . . . . 5

Increase in interest payable. . . . . . . . . . . . . . . . . . . . . . . . . . . . . . . . . . . . . . . . . . . . . . . 1 33

Net cash provided by operating activities . . . . . . . . . . . . . . . . . . . . . . . . . . . . . . . . . . . . . . . . $53

Do More: QS 12-3, QS 12-4, QS 12-5, QS 12-6, QS 12-7,

E 12-2, E 12-3, E 12-4, E 12-5, E 12-6, E 12-7

To compute cash flows from investing activities, we analyze changes in (1) all long-term asset accounts and (2) any current accounts for notes receivable and investments in securities. Reporting of investing activities is identical under the direct method and indirect method.

Three-Step Analysis To determine cash provided or used by investing activities: (1) identify changes in investing- related accounts, (2) explain these changes using T-accounts and reconstructed entries, and (3) report the cash flow effects.

Analyzing Noncurrent Assets Genesis both purchased and sold long-term assets during the period. These transactions are investing activities and are analyzed in this section.

Plant Asset Transactions First Step Analyze Genesis’s Plant Assets account and its Accumulated Depreciation account to identify changes in those accounts. Comparative balance sheets in Exhibit 12.10 show a $40,000 increase in plant assets from $210,000 to $250,000 and a $12,000 increase in accumu- lated depreciation from $48,000 to $60,000.

CASH FLOWS FROM INVESTING

P3 Determine cash flows from both investing and financing activities.

Point: Investing activities include (1) purchasing and selling long- term assets, (2) lending and collecting on notes receivable, and (3) purchasing and selling short- term investments other than cash equivalents and trading securities.

462 Chapter 12 Reporting Cash Flows

Second Step Items b and c of the additional information in Exhibit 12.10 relate to plant assets. Recall that the Plant Assets account is impacted by both asset purchases and sales; its Accumulated Depreciation account is increased by depreciation and decreased by the removal of accumulated depreciation in asset sales. To explain changes in these accounts and to identify their cash flow effects, we prepare reconstructed entries, which is our attempt to re-create actual entries made by the preparer. Item b says Genesis purchased plant assets of $60,000 by issuing $60,000 in notes payable. The reconstructed entry is

Bal., Dec. 31, 2018 210,000

Purchase 60,000

Bal., Dec. 31, 2019 250,000

Sale 20,000

Plant Assets

Sale 12,000 Bal., Dec. 31, 2018 48,000

Depr. expense 24,000

Bal., Dec. 31, 2019 60,000

Accumulated Depreciation—Plant Assets

Reconstruction Plant Assets . . . . . . . . . . . . . . . . . . . . . . . . . . . . . . . . . . . . . . . . . . . . . . . . 60,000 Notes Payable . . . . . . . . . . . . . . . . . . . . . . . . . . . . . . . . . . . . . . . . . . 60,000

Reconstruction Depreciation Expense . . . . . . . . . . . . . . . . . . . . . . . . . . . . . . . . . . . . . . . . 24,000 Accumulated Depreciation . . . . . . . . . . . . . . . . . . . . . . . . . . . . . . . . 24,000

Reconstruction Cash . . . . . . . . . . . . . . . . . . . . . . . . . . . . . . . . . . . . . . . . . . . . . . . . . . . . . . 2,000 Accumulated Depreciation . . . . . . . . . . . . . . . . . . . . . . . . . . . . . . . . . . . . 12,000

Loss on Sale of Plant Assets . . . . . . . . . . . . . . . . . . . . . . . . . . . . . . . . . . . 6,000

Plant Assets . . . . . . . . . . . . . . . . . . . . . . . . . . . . . . . . . . . . . . . . . . . . 20,000

Cash flows from investing activities Cash received from sale of plant assets . . . . . . . . . . . . . . . . . . . $2,000

Item c says Genesis sold plant assets costing $20,000 (with $12,000 of accumulated deprecia- tion) for $2,000 cash, resulting in a $6,000 loss. The reconstructed entry is

We also reconstruct the entry for depreciation from the income statement, which does not impact cash.

The three reconstructed entries are shown in the following T-accounts. This reconstruction anal- ysis is complete in that changes in the long-term asset accounts are entirely explained.

Third Step Look at the reconstructed entries to identify cash flows. The identified cash flows are reported in the investing section of the statement.

The $60,000 purchase in item b, paid for by issuing notes, is a noncash investing and financing activity. It is reported in a note or in a separate schedule to the statement.

Example: If a plant asset costing $40,000 with $37,000 of accu- mulated depreciation is sold at a $3,000 gain, what is the cash flow? Answer: +$6,000

Noncash investing and financing activity Purchased plant assets with issuance of notes . . . . . . . . . . . . . . $60,000

Additional Long-Term Assets Genesis did not have any additional noncurrent assets (or nonoperating current assets). If such assets do exist, we analyze and report investing cash flows using the same three-step process.

Location, Location, Location Cash flows can be delayed or accelerated at period-end to improve or reduce current-period cash flows. Cash flows also can be misclassified. We know cash outflows under operating activities are viewed as expense payments. However, cash outflows under investing activities are viewed as a sign of growth potential. This requires investors to review where cash flows are reported. ■

Ethical Risk

Chapter 12 Reporting Cash Flows 463

Use the following information to determine this company’s cash flows from investing activities. a. A factory with a book value of $100 and an original cost of $800 was sold at a loss of $10. b. Paid $70 cash for new equipment. c. Long-term stock investments were sold for $20 cash, yielding a loss of $4. d. Sold land costing $175 for $160 cash, yielding a loss of $15.

Solution

Do More: QS 12-8, QS 12-9, QS 12-10, QS 12-11, QS 12-12,

QS 12-13, E 12-8

P3

Reporting Investing Cash Flows

NEED-TO-KNOW 12-3

Cash flows from investing activities Cash received from sale of factory (from a*—also see margin entry) . . . . $ 90

Cash paid for new equipment (from b) . . . . . . . . . . . . . . . . . . . . . . . . . . . . (70)

Cash received from sale of long-term investments (from c). . . . . . . . . . . . 20

Cash received from sale of land (from d) . . . . . . . . . . . . . . . . . . . . . . . . . . 160

Net cash provided by investing activities . . . . . . . . . . . . . . . . . . . . . . . . . . $200

*Cash received from sale of factory = Book value − Loss = $100 − $10 = $90.

Reconstruction for part a. Cash . . . . . . . . . . . . . . 90 Loss on asset sale . . . . 10 Factory (BV) . . . . . 100

To compute cash flows from financing activities, we analyze changes in all noncurrent liability accounts (including the current portion of any notes and bonds) and equity accounts. These accounts include long-term debt, notes payable, bonds payable, common stock, and retained earnings. Reporting of financing activities is identical under the direct method and indirect method.

Three-Step Analysis To determine cash provided or used by financing activities: (1) identify changes in financing- related accounts, (2) explain these changes using T-accounts and reconstructed entries, and (3) report the cash flow effects.

Analyzing Noncurrent Liabilities Genesis retired notes payable by paying cash. This is a change in noncurrent liabilities.

Notes Payable Transactions First Step Review comparative balance sheets in Exhibit 12.10, which shows an increase in notes payable from $64,000 to $90,000.

Second Step Item e of the additional information in Exhibit 12.10 reports that notes with a carrying value of $34,000 are retired for $18,000 cash, resulting in a $16,000 gain. The recon- structed entry is

CASH FLOWS FROM FINANCING

Point: Examples of financing activities are (1) receiving cash from issuing debt or repaying amounts borrowed and (2) receiv- ing cash from or distributing cash to owners.

Reconstruction Notes Payable . . . . . . . . . . . . . . . . . . . . . . . . . . . . . . . . . . . . . . 34,000 Gain on retirement of debt . . . . . . . . . . . . . . . . . . . . . . . . 16,000

Cash . . . . . . . . . . . . . . . . . . . . . . . . . . . . . . . . . . . . . . . . . 18,000

Item b of the additional information reports that Genesis purchased plant assets costing $60,000 by issuing $60,000 in notes payable. This $60,000 increase to notes payable is reported as a noncash investing and financing transaction. The Notes Payable account is explained by these reconstructed entries.

Retired notes 34,000 Bal., Dec. 31, 2018 64,000

Issued notes 60,000

Bal., Dec. 31, 2019 90,000

Notes Payable

464 Chapter 12 Reporting Cash Flows

Third Step Report cash paid for the notes retirement in the financing activities section.

Cash flows from financing activities Cash paid to retire notes . . . . . . . . . . . . . . . . . . . . . . . . $(18,000)

Analyzing Equity Genesis had two equity transactions. The first is the issuance of common stock for cash. The second is the declaration and payment of cash dividends.

Common Stock Transactions First Step Review the comparative balance sheets in Exhibit 12.10, which show an increase in common stock from $80,000 to $95,000.

Second Step Item d of the additional information in Exhibit 12.10 reports that 3,000 shares of common stock are issued at par for $5 per share. The reconstructed entry and the complete Common Stock T-account follow.

Reconstruction Cash . . . . . . . . . . . . . . . . . . . . . . . . . . . . . . . . . . . . . . . . . . . . . . 15,000 Common Stock . . . . . . . . . . . . . . . . . . . . . . . . . . . . . . . . . 15,000

Bal., Dec. 31, 2018 80,000

Issued stock 15,000 Bal., Dec. 31, 2019 95,000

Common Stock

Cash flows from financing activities Cash received from issuing stock . . . . . . . . . . . . . . . . . $15,000

Reconstruction Retained Earnings . . . . . . . . . . . . . . . . . . . . . . . . . . . . . . . . . . . 14,000 Cash . . . . . . . . . . . . . . . . . . . . . . . . . . . . . . . . . . . . . . . . . 14,000

Third Step Report cash received from stock issuance in the financing activities section.

Retained Earnings Transactions First Step Review the comparative balance sheets in Exhibit 12.10, which show an increase in retained earnings from $88,000 to $112,000.

Second Step Item f of the additional information in Exhibit 12.10 reports that cash dividends of $14,000 are paid. The reconstructed entry follows.

Retained Earnings also is impacted by net income of $38,000. (Net income is covered in operat- ing activities.) The reconstructed Retained Earnings account follows.

Third Step Report cash paid for dividends in the financing activities section.Point: Stock dividends and splits do not impact cash.

Cash flows from financing activities Cash paid for dividends . . . . . . . . . . . . . . . . . . . . . . . . . $(14,000)

Cash dividend 14,000 Bal., Dec. 31, 2018 88,000

Net income 38,000

Bal., Dec. 31, 2019 112,000

Retained Earnings

Proving Cash Balances The final stage in preparing the statement is to report the beginning and ending cash balances and prove that the net change in cash is explained by operating, investing, and financing cash flows. The last three rows of Exhibit 12.13 show that the $5,000 net increase in cash, from $12,000 at the beginning of the period to $17,000 at the end, is reconciled by net cash flows from operating ($20,000 inflow), investing ($2,000 inflow), and financing ($17,000 outflow) activities.

Chapter 12 Reporting Cash Flows 465

GENESIS Statement of Cash Flows (Indirect Method)

For Year Ended December 31, 2019

Cash flows from operating activities

Net income . . . . . . . . . . . . . . . . . . . . . . . . . . . . . . . . . . . . . . . . . . . . . . . . . . . . . . . . . . . . $ 38,000

Adjustments to reconcile net income to net cash provided by operating activities

Income statement items not affecting cash

Depreciation expense . . . . . . . . . . . . . . . . . . . . . . . . . . . . . . . . . . . . . . . . . . . . . . . 24,000

Loss on sale of plant assets. . . . . . . . . . . . . . . . . . . . . . . . . . . . . . . . . . . . . . . . . . . 6,000

Gain on retirement of notes . . . . . . . . . . . . . . . . . . . . . . . . . . . . . . . . . . . . . . . . . . (16,000)

Changes in current assets and liabilities

Increase in accounts receivable . . . . . . . . . . . . . . . . . . . . . . . . . . . . . . . . . . . . . . . (20,000)

Increase in inventory . . . . . . . . . . . . . . . . . . . . . . . . . . . . . . . . . . . . . . . . . . . . . . . . (14,000)

Increase in prepaid expenses . . . . . . . . . . . . . . . . . . . . . . . . . . . . . . . . . . . . . . . . . (2,000)

Decrease in accounts payable . . . . . . . . . . . . . . . . . . . . . . . . . . . . . . . . . . . . . . . . (5,000)

Decrease in interest payable. . . . . . . . . . . . . . . . . . . . . . . . . . . . . . . . . . . . . . . . . . (1,000)

Increase in income taxes payable. . . . . . . . . . . . . . . . . . . . . . . . . . . . . . . . . . . . . . 10,000

Net cash provided by operating activities . . . . . . . . . . . . . . . . . . . . . . . . . . . . . . . . . . . . $ 20,000

Cash flows from investing activities

Cash received from sale of plant assets . . . . . . . . . . . . . . . . . . . . . . . . . . . . . . . . . . . . . 2,000

Net cash provided by investing activities . . . . . . . . . . . . . . . . . . . . . . . . . . . . . . . . . . . . 2,000

Cash flows from financing activities

Cash received from issuing stock . . . . . . . . . . . . . . . . . . . . . . . . . . . . . . . . . . . . . . . . . . 15,000

Cash paid to retire notes . . . . . . . . . . . . . . . . . . . . . . . . . . . . . . . . . . . . . . . . . . . . . . . . . (18,000)

Cash paid for dividends . . . . . . . . . . . . . . . . . . . . . . . . . . . . . . . . . . . . . . . . . . . . . . . . . . (14,000)

Net cash used in financing activities . . . . . . . . . . . . . . . . . . . . . . . . . . . . . . . . . . . . . . . . (17,000)

Net increase in cash . . . . . . . . . . . . . . . . . . . . . . . . . . . . . . . . . . . . . . . . . . . . . . . . . . . . . . . $ 5,000

Cash balance at prior year-end . . . . . . . . . . . . . . . . . . . . . . . . . . . . . . . . . . . . . . . . . . . . . . 12,000

Cash balance at current year-end . . . . . . . . . . . . . . . . . . . . . . . . . . . . . . . . . . . . . . . . . . . . $ 17,000

EXHIBIT 12.13 Complete Statement of Cash Flows—Indirect Method

Reporter Management is in labor contract negotiations and grants you an interview. It highlights a total net cash outflow of $550,000 (which includes net cash outflows of $850,000 for investing activities and $350,000 for financing activities). What is your assessment of this company? ■ Answer: An initial reaction from the $550,000 decrease in net cash is not positive. However, closer scrutiny shows a more positive picture. Cash flow from operations is $650,000, computed as [?] − $850,000 − $350,000 = $(550,000).

Decision Maker

Use the following information to determine cash flows from financing activities. a. Issued common stock for $40 cash. b. Paid $70 cash to retire a note payable at its $70 maturity value. c. Paid cash dividend of $15. d. Paid $5 cash to acquire its treasury stock.

Solution

Do More: QS 12-14, QS 12-15, QS 12-16, QS 12-17, E 12-9

P3

Reporting Financing Cash Flows

NEED-TO-KNOW 12-4

Cash flows from financing activities Cash received from issuance of common stock (from a) . . . . . . . . . . $ 40

Cash paid to settle note payable (from b) . . . . . . . . . . . . . . . . . . . . . . (70)

Cash paid for dividend (from c) . . . . . . . . . . . . . . . . . . . . . . . . . . . . . . (15)

Cash paid to acquire treasury stock (from d ) . . . . . . . . . . . . . . . . . . . (5)

Net cash used by financing activities . . . . . . . . . . . . . . . . . . . . . . . . . $(50)

466 Chapter 12 Reporting Cash Flows

Exhibit 12.14 uses T-accounts to summarize how changes in Genesis’s noncash balance sheet accounts affect its cash inflows and outflows (dollar amounts in thousands). The top of the exhibit shows Genesis’s Cash T-account, and the lower part shows T-accounts for its remaining balance sheet accounts. We see that the $20,000 net cash provided by operating activities and the $5,000 net increase in cash shown in the Cash T-account agree with the same figures in the statement of cash flows in Exhibit 12.13. We explain Exhibit 12.14 in five parts.

a. Entry (1) records $38 net income on the credit side of the Retained Earnings account and the debit side of the Cash account. This $38 net income in the Cash T-account is adjusted until it reflects the $5 net increase in cash.

b. Entries (2) through (4) add the $24 depreciation and $6 loss on asset sale to net income and subtract the $16 gain on retirement of notes.

c. Entries (5) through (10) adjust net income for changes in current asset and current liability accounts.

d. Entry (11) records the noncash investing and financing transaction involving a $60 purchase of assets by issuing $60 of notes.

e. Entries (12) and (13) record the $15 stock issuance and the $14 dividend.

SUMMARY USING T-ACCOUNTS

Long-Term Notes Payable

Beg. 64

(4) 34

(11) 60

End. 90

Common Stock

Beg. 80

(12) 15

End. 95

Retained Earnings

Beg. 88

(1) 38

(13) 14

End. 112

Accounts Receivable

Beg. 40

(5) 20

End. 60

Inventory

Beg. 70

(6) 14

End. 84

Prepaid Expenses

Beg. 4

(7) 2

End. 6

Plant Assets

Beg. 210

(3) 20

(11) 60

End. 250

Accumulated Depreciation

Beg. 48

(3) 12 (2) 24

End. 60

Accounts Payable

Beg. 40

(8) 5

End. 35

Interest Payable

Beg. 4

(9) 1

End. 3

Income Taxes Payable

Beg. 12

(10) 10

End. 22

Cash

(1) Net income 38

(2) Depreciation 24 (4) Gain on retirement of notes 16

(3) Loss on sale of plant assets 6

(10) Increase in income taxes payable 10 (5) Increase in accounts receivable 20

(6) Increase in inventory 14

(7) Increase in prepaid expense 2

(8) Decrease in accounts payable 5

(9) Decrease in interest payable 1

Net cash provided by operating activities [O] 20

(3) Cash received from sale of plant assets [I] 2 (4) Cash paid to retire notes [F] 18 (12) Cash received from issuing stock [F] 15 (13) Cash paid for dividends [F] 14 Net increase in cash 5

⎫ ⎪ ⎪ ⎪ ⎪ ⎪ ⎪ ⎪ ⎪ ⎪ ⎬ ⎪ ⎪ ⎪ ⎪ ⎪ ⎪ ⎪ ⎪ ⎪ ⎭

Info to prepare statement of cash flows

EXHIBIT 12.14 Balance Sheet T-Accounts to Explain the Change in Cash ($ thousands)

Chapter 12 Reporting Cash Flows 467

Cash Flow on Total Assets Cash flow information can help measure a company’s ability to meet its obligations, pay dividends, expand operations, and obtain financing. The cash flow on total assets ratio is in Exhibit 12.16.

Cash Flow Analysis Decision Analysis

Analyzing Cash Sources and Uses Managers review cash flows for business decisions. Creditors evaluate a company’s ability to generate enough cash to pay debt. Investors assess cash flows before buying and selling stock. To effectively evaluate cash flows, we separately analyze investing, financing, and operating activities. Consider data from three different companies in Exhibit 12.15 that operate in the same industry and have been in business for several years. Each company has the same $15,000 net increase in cash, but its sources and uses of cash flows are different. BMX’s operating activities provide net cash flows of $90,000, allowing it to purchase plant assets of $48,000 and repay $27,000 of its debt. ATV’s operating activities provide $40,000 of cash flows, limiting its purchase of plant assets to $25,000. Trex’s $15,000 net cash increase is due to selling plant assets and incurring additional debt. Its operating activities yield a cash outflow of $24,000. Overall, analysis of cash flows reveals that BMX is more capable of generating future cash flows than is ATV or Trex.

A1 Analyze the statement of cash flows and apply the cash flow on total assets ratio.

EXHIBIT 12.15 Cash Flows of Competing Companies

$ thousands BMX ATV Trex

Cash provided (used) by operating activities . . . $90,000 $40,000 $(24,000)

Cash provided (used) by investing activities

Proceeds from sale of plant assets. . . . . . . . . 26,000

Purchase of plant assets . . . . . . . . . . . . . . . . . (48,000) (25,000)

Cash provided (used) by financing activities

Proceeds from issuance of debt . . . . . . . . . . . 13,000

Repayment of debt . . . . . . . . . . . . . . . . . . . . . (27,000)

Net increase (decrease) in cash . . . . . . . . . . . . . $15,000 $15,000 $ 15,000

Free Cash Flows Many investors use cash flows to value company stock. However, cash-based valuation models often yield different stock values due to differences in measurement of cash flows. Most models require cash flows that are “free” for distribution to shareholders. These free cash flows are defined as cash flows available to sharehold- ers after operating asset reinvestments and debt payments. A company’s growth and financial flexibility depend on adequate free cash flows. ■

Decision Insight

Point: Cash flow from operations − Capital expenditures − Debt repayments = Free cash flows

EXHIBIT 12.16 Cash Flow on Total AssetsCash flow on total assets =

Cash flow from operations Average total assets

This ratio measures actual cash flows and is not affected by accounting recognition and measurement. It can help estimate the amount and timing of cash flows from operating activities. The cash flow on total assets for competitors Nike and Under Armour are in Exhibit 12.17. In all years, Nike’s cash flow on total assets ratio exceeded Under Armour’s ratio. This means that Nike did a better job of generating operating cash flows given its assets. However, Nike’s cash flow on total assets declined from two years ago, which is not a positive result. At the same time, Under Armour’s lower and uneven cash flow on total assets make it difficult to predict the amount and timing of its cash flows.

EXHIBIT 12.17 Cash Flow on Total Assets for Two Competitors

Company Figure ($ millions) Current Year 1 Year Ago 2 Years Ago

Nike Operating cash flows . . . . . . . . . . . . . . . . . . . $ 3,640 $ 3,096 $ 4,680 Average total assets. . . . . . . . . . . . . . . . . . . . $22,328 $21,497 $20,096

Cash flow on total assets. . . . . . . . . . . . . . . 16.3% 14.4% 23.3% Under Armour Operating cash flows . . . . . . . . . . . . . . . . . . . $ 234 $ 364 $ 15 Average total assets. . . . . . . . . . . . . . . . . . . . $ 3,825 $ 3,255 $ 2,479

Cash flow on total assets. . . . . . . . . . . . . . . 6.1% 11.2% 0.6%

468 Chapter 12 Reporting Cash Flows

Comparative balance sheets, an income statement, and additional information follow.

COMPREHENSIVE

Preparing Statement of Cash Flows—Indirect and Direct Methods

NEED-TO-KNOW 12-5

UMA COMPANY Income Statement

For Year Ended December 31, 2019

Sales . . . . . . . . . . . . . . . . . . . . . . . . $446,100 Cost of goods sold . . . . . . . . . . . . . $222,300 Other operating expenses . . . . . . 120,300 Depreciation expense . . . . . . . . . .    25,500 (368,100)     78,000 Other gains (losses) Loss on sale of equipment . . . . 3,300 Loss on retirement of bonds . . . 825 (4,125) Income before taxes . . . . . . . . . . . 73,875 Income tax expense . . . . . . . . . . . (13,725) Net income . . . . . . . . . . . . . . . . . . $  60,150

UMA COMPANY Balance Sheets

December 31, 2019 and 2018

2019 2018 Assets Cash . . . . . . . . . . . . . . . . . . . . . . . . . .   $  43,050 $ 23,925 Accounts receivable . . . . . . . . . . . . . .       34,125     39,825 Inventory . . . . . . . . . . . . . . . . . . . . . . .     156,000   146,475 Prepaid expenses . . . . . . . . . . . . . . . .         3,600       1,650 Total current assets . . . . . . . . . . . . . . .     236,775   211,875 Equipment . . . . . . . . . . . . . . . . . . . . . .     135,825   146,700 Accum. depreciation—Equipment . . .      (61,950)     (47,550) Total assets . . . . . . . . . . . . . . . . . . . . .   $310,650 $311,025

Liabilities Accounts payable . . . . . . . . . . . . . . . .   $ 28,800 $ 33,750 Income taxes payable . . . . . . . . . . . .         5,100       4,425 Dividends payable . . . . . . . . . . . . . . .                0       4,500 Total current liabilities . . . . . . . . . . . . .       33,900     42,675 Bonds payable. . . . . . . . . . . . . . . . . . .               0    37,500 Total liabilities . . . . . . . . . . . . . . . . . . .       33,900     80,175

Equity Common stock, $10 par . . . . . . . . . . .     168,750   168,750 Retained earnings . . . . . . . . . . . . . . .     108,000     62,100 Total liabilities and equity . . . . . . . . .   $310,650 $311,025

Additional Information a. Equipment costing $21,375 with accumulated deprecia-

tion of $11,100 is sold for cash.

b. Equipment purchases are for cash. c. Accumulated Depreciation is affected by depreciation

expense and the sale of equipment.

d. The balance of Retained Earnings is affected by dividend declarations and net income.

e. All sales are made on credit. f. All inventory purchases are on credit. g. Accounts Payable balances result from inventory purchases. h. Prepaid expenses relate to “other operating expenses.”

Required

1. Prepare a statement of cash flows using the indirect method for year 2019. 2.B Prepare a statement of cash flows using the direct method for year 2019.

PLANNING THE SOLUTION Prepare two blank statements of cash flows with sections for operating, investing, and financing activ-

ities using the (1) indirect method format and (2) direct method format. Compute the cash paid for equipment and the cash received from the sale of equipment using the addi-

tional information provided along with the amount for depreciation expense and the change in the bal- ances of Equipment and Accumulated Depreciation. Use T-accounts to help chart the effects of the sale and purchase of equipment on the balances of the Equipment account and the Accumulated Depreciation account.

Compute the effect of net income on the change in the Retained Earnings account balance. Assign the difference between the change in retained earnings and the amount of net income to dividends declared. Adjust the dividends declared amount for the change in the Dividends Payable balance.

Compute cash received from customers, cash paid for inventory, cash paid for other operating expenses, and cash paid for taxes.

Enter the cash effects of reconstruction entries to the appropriate section(s) of the statement. Total each section of the statement, determine the total net change in cash, and add it to the beginning

balance to get the ending balance of cash.

Chapter 12 Reporting Cash Flows 469

(3) Net income . . . . . . . . . . . . . . . . . . . . . . . . . . . . . . . . $ 60,150 Less increase in retained earnings . . . . . . . . . . . . . 45,900 Dividends declared . . . . . . . . . . . . . . . . . . . . . . . . . 14,250 Plus decrease in dividends payable . . . . . . . . . . . . 4,500 Cash paid for dividends . . . . . . . . . . . . . . . . . . . . . . $ 18,750 (4)B Sales . . . . . . . . . . . . . . . . . . . . . . . . . . . . . . . . . . . . . $ 446,100 Add decrease in accounts receivable . . . . . . . . . . . 5,700 Cash received from customers . . . . . . . . . . . . . . . . $451,800 (5)B Cost of goods sold . . . . . . . . . . . . . . . . . . . . . . . . . . $ 222,300 Plus increase in inventory . . . . . . . . . . . . . . . . . . . . 9,525 Purchases . . . . . . . . . . . . . . . . . . . . . . . . . . . . . . . . . 231,825 Plus decrease in accounts payable . . . . . . . . . . . . . 4,950 Cash paid for inventory . . . . . . . . . . . . . . . . . . . . . . $236,775 (6)B Other operating expenses . . . . . . . . . . . . . . . . . . . . $ 120,300 Plus increase in prepaid expenses . . . . . . . . . . . . . 1,950 Cash paid for other operating expenses. . . . . . . . . $122,250 (7)B Income tax expense. . . . . . . . . . . . . . . . . . . . . . . . . $ 13,725 Less increase in income taxes payable. . . . . . . . . . (675) Cash paid for income taxes . . . . . . . . . . . . . . . . . . . $ 13,050

(1) Cost of equipment sold* . . . . . . . . . . . . . . . . . . . . . . . . . . . $ 21,375 Accumulated depreciation of equipment sold . . . . . . . . . . (11,100) Book value of equipment sold. . . . . . . . . . . . . . . . . . . . . . . 10,275 Loss on sale of equipment. . . . . . . . . . . . . . . . . . . . . . . . . . (3,300) Cash received from sale of equipment . . . . . . . . . . . . . . . . $ 6,975

Cost of equipment sold . . . . . . . . . . . . . . . . . . . . . . . . . . . . $ 21,375 Less decrease in the Equipment account balance . . . . . . . (10,875) Cash paid for new equipment . . . . . . . . . . . . . . . . . . . . . . . $10,500 (2) Loss on retirement of bonds . . . . . . . . . . . . . . . . . . . . . . . . $ 825 Carrying value of bonds retired. . . . . . . . . . . . . . . . . . . . . . 37,500 Cash paid to retire bonds . . . . . . . . . . . . . . . . . . . . . . . . . . $38,325

*Supporting T-account analysis for part 1 follows.

Sale 11,100

Bal., Dec. 31, 2018 47,550

Depr. expense 25,500

Bal., Dec. 31, 2019 61,950

Accumulated Depreciation—Equipment

Bal., Dec. 31, 2018 146,700 Cash purchase 10,500

Bal., Dec. 31, 2019 135,825

Sale 21,375

Equipment

SOLUTION Supporting computations for cash receipts and cash payments.

1. Indirect method.

UMA COMPANY Statement of Cash Flows (Indirect Method)

For Year Ended December 31, 2019

Cash flows from operating activities

Net income. . . . . . . . . . . . . . . . . . . . . . . . . . . . . . . . . . . $ 60,150

Adjustments to reconcile net income to net cash provided by operating activities

Income statement items not affecting cash

Depreciation expense . . . . . . . . . . . . . . . . . . . . . 25,500

Loss on sale of plant assets . . . . . . . . . . . . . . . . . 3,300

Loss on retirement of bonds . . . . . . . . . . . . . . . . 825

Changes in current assets and current liabilities

Decrease in accounts receivable. . . . . . . . . . . . . 5,700

Increase in inventory . . . . . . . . . . . . . . . . . . . . . . (9,525)

Increase in prepaid expenses . . . . . . . . . . . . . . . (1,950)

Decrease in accounts payable. . . . . . . . . . . . . . . (4,950)

Increase in income taxes payable . . . . . . . . . . . . 675

Net cash provided by operating activities . . . . . . . . . . $ 79,725

Cash flows from investing activities

Cash received from sale of equipment. . . . . . . . . . . . . 6,975

Cash paid for equipment. . . . . . . . . . . . . . . . . . . . . . . . (10,500)

Net cash used in investing activities . . . . . . . . . . . . . . (3,525)

Cash flows from financing activities

Cash paid to retire bonds payable . . . . . . . . . . . . . . . . (38,325)

Cash paid for dividends. . . . . . . . . . . . . . . . . . . . . . . . . (18,750)

Net cash used in financing activities . . . . . . . . . . . . . . (57,075)

Net increase in cash. . . . . . . . . . . . . . . . . . . . . . . . . . . . . . $ 19,125

Cash balance at prior year-end. . . . . . . . . . . . . . . . . . . . . 23,925

Cash balance at current year-end. . . . . . . . . . . . . . . . . . . $ 43,050

UMA COMPANY Statement of Cash Flows (Direct Method)

For Year Ended December 31, 2019

Cash flows from operating activities

Cash received from customers. . . . . . . . . . . . . $ 451,800

Cash paid for inventory . . . . . . . . . . . . . . . . . . (236,775)

Cash paid for other operating expenses . . . . . (122,250)

Cash paid for income taxes . . . . . . . . . . . . . . . (13,050)

Net cash provided by operating activities . . . . $ 79,725

Cash flows from investing activities

Cash received from sale of equipment . . . . . . 6,975

Cash paid for equipment . . . . . . . . . . . . . . . . . (10,500)

Net cash used in investing activities . . . . . . . . (3,525)

Cash flows from financing activities

Cash paid to retire bonds payable. . . . . . . . . . (38,325)

Cash paid for dividends . . . . . . . . . . . . . . . . . . (18,750)

Net cash used in financing activities . . . . . . . . (57,075)

Net increase in cash . . . . . . . . . . . . . . . . . . . . . . . $ 19,125

Cash balance at prior year-end . . . . . . . . . . . . . . 23,925

Cash balance at current year-end . . . . . . . . . . . . $ 43,050

2.B Direct method (Appendix 12B).

470 Chapter 12 Reporting Cash Flows

APPENDIX

Spreadsheet Preparation of the Statement of Cash Flows12A

This appendix explains how to use a spreadsheet (work sheet) to prepare the statement of cash flows under the indirect method.

Preparing the Indirect Method Spreadsheet A spreadsheet, also called work sheet, can help us prepare a statement of cash flows. To demonstrate, we return to the comparative balance sheets and income statement shown in Exhibit 12.10. We use letters a through g to code changes in accounts, and letters h through m for additional information, to prepare the statement of cash flows.

a. Net income is $38,000. b. Accounts receivable increase by $20,000. c. Inventory increases by $14,000. d. Prepaid expenses increase by $2,000. e. Accounts payable decrease by $5,000. f. Interest payable decreases by $1,000. g. Income taxes payable increase by $10,000. h. Depreciation expense is $24,000. i. Plant assets costing $20,000 with accumulated depreciation of $12,000 are sold for $2,000 cash. This

yields a loss on sale of assets of $6,000. j. Notes with a book value of $34,000 are retired with a cash payment of $18,000, yielding a $16,000

gain on retirement. k. Plant assets costing $60,000 are purchased with an issuance of notes payable for $60,000. l. Issued 3,000 shares of common stock for $15,000 cash. m. Paid cash dividends of $14,000.

Exhibit 12A.1 shows the indirect method spreadsheet for Genesis. We enter both beginning and ending balance sheet amounts on the spreadsheet. We also enter information in the Analysis of Changes columns (keyed to the additional information items a through m) to explain changes in the accounts and determine the cash flows for operating, investing, and financing activities. Information about noncash investing and financing activities is reported near the bottom.

Entering the Analysis of Changes on the Spreadsheet The following steps are used to complete the spreadsheet after the beginning and ending balances of the balance sheet accounts are entered.

1 Enter net income as the first item in the statement of cash flows section for computing operating cash inflow (debit) and as a credit to Retained Earnings. (Entry a)

2 In the statement of cash flows section, adjustments to net income are entered as debits if they increase cash flows and as credits if they decrease cash flows. Applying this rule, adjust net income for the change in each noncash current asset and current liability account related to operating activities. For each adjustment to net income, the offsetting debit or credit must help reconcile the beginning and ending balances of a current asset or current liability account. (Entries b through g)

3 Enter adjustments to net income for income statement items not providing or using cash in the period. For each adjustment, the offsetting debit or credit must help reconcile a noncash balance sheet account. (Entry h)

4 Adjust net income to eliminate any gains or losses from investing and financing activities. Because the cash from a gain must be excluded from operating activities, the gain is entered as a credit in the operating activities section. Losses are entered as debits. For each adjustment, the related debit and/ or credit must help reconcile balance sheet accounts and involve reconstructed entries to show the cash flow from investing or financing activities. (Entries i and j)

5 After reviewing any unreconciled balance sheet accounts and related information, enter the remain- ing reconciling entries for investing and financing activities. Examples are purchases of plant assets,

P4 Illustrate use of a spread- sheet to prepare a statement of cash flows.

Chapter 12 Reporting Cash Flows 471

GENESIS Spreadsheet for Statement of Cash Flows—Indirect Method

For Year Ended December 31, 2019

Debit Credit

Analysis of Changes Dec. 31, 2019

Dec. 31, 2018

Balance Sheet—Debit Bal. Accounts Cash Accounts receivable Inventory Prepaid expenses Plant assets

Balance Sheet—Credit Bal. Accounts Accumulated depreciation Accounts payable Interest payable Income taxes payable Notes payable Common stock, $5 par value Retained earnings

Statement of Cash Flows Operating activities Net income Increase in accounts receivable Increase in inventory Increase in prepaid expenses Decrease in accounts payable Decrease in interest payable Increase in income taxes payable Depreciation expense Loss on sale of plant assets Gain on retirement of notes Investing activities Receipts from sale of plant assets Financing activities Payment to retire notes Receipts from issuing stock Payment of cash dividends

Noncash Investing and Financing Activities Purchase of plant assets with notes

$ 12,000 40,000 70,000

4,000 210,000

$336,000

$ 48,000 40,000

4,000 12,000 64,000 80,000 88,000

$336,000

$ 17,000 60,000 84,000

6,000 250,000

$417,000

$ 60,000 35,000

3,000 22,000 90,000 95,000

112,000 $417,000

$ 20,000 14,000 2,000

60,000

12,000 5,000 1,000

34,000

14,000

38,000

10,000 24,000

6,000

2,000

15,000

60,000 $317,000

$ 20,000

24,000

10,000 60,000 15,000 38,000

20,000 14,000 2,000 5,000 1,000

16,000

18,000

14,000

60,000 $317,000

(b) (c) (d) (k1)

( i ) (e) (f )

( j )

(m)

(a)

(g) (h) ( i )

( i )

(l )

(k2)

( i )

(h)

(g) (k2) (l) (a)

(b) (c) (d) (e) (f)

( j)

( j)

(m)

(k1)

EXHIBIT 12A.1 Spreadsheet for Preparing Statement of Cash Flows— Indirect Method

issuances of long-term debt, stock issuances, and dividend payments. Some of these may require entries in the noncash investing and financing section of the spreadsheet. (Entries k through m)

6 Check accuracy by totaling the Analysis of Changes columns and by determining that the change in each balance sheet account has been explained (reconciled).

Because adjustments i, j, and k are more challenging, we show them in the following debit and credit for- mat. These entries are for purposes of our understanding; they are not the entries actually made in the journals. Changes in the Cash account are identified as sources or uses of cash.

i. Cash—Receipt from sale of plant assets (source of cash) . . . . . . . . . . . . . . . . 2,000 Loss from sale of plant assets . . . . . . . . . . . . . . . . . . . . . . . . . . . . . . . . . . . . . . . 6,000 Accumulated depreciation . . . . . . . . . . . . . . . . . . . . . . . . . . . . . . . . . . . . . . . . . . 12,000 Plant assets . . . . . . . . . . . . . . . . . . . . . . . . . . . . . . . . . . . . . . . . . . . . . . . . . 20,000 Describe sale of plant assets. j. Notes payable. . . . . . . . . . . . . . . . . . . . . . . . . . . . . . . . . . . . . . . . . . . . . . . . . . . . 34,000 Cash—Payments to retire notes (use of cash) . . . . . . . . . . . . . . . . . . . . . 18,000 Gain on retirement of notes . . . . . . . . . . . . . . . . . . . . . . . . . . . . . . . . . . . . 16,000 Describe retirement of notes. k1. Plant assets. . . . . . . . . . . . . . . . . . . . . . . . . . . . . . . . . . . . . . . . . . . . . . . . . . . . . . 60,000 Cash—Purchase of plant assets financed by notes . . . . . . . . . . . . . . . . . . 60,000 Describe purchase of plant assets. k2. Cash—Purchase of plant assets financed by notes . . . . . . . . . . . . . . . . . . . . . . 60,000 Notes payable . . . . . . . . . . . . . . . . . . . . . . . . . . . . . . . . . . . . . . . . . . . . . . . 60,000 Issue notes for purchase of assets.

472 Chapter 12 Reporting Cash Flows

APPENDIX

Direct Method of Reporting Operating Cash Flows12B

We compute operating cash flows under the direct method by adjusting accrual-based income statement items to the cash basis as follows.P5

Compute cash flows from operating activities using the direct method.

The framework for reporting cash receipts and cash payments for the operating section under the direct method is shown in Exhibit 12B.1.

Landlord

Renters Dividends

Operations Interest Taxes

Operating cash payments–

Operating cash receipts+

Net cash provided (used) by operating

activities =

Customers

Suppliers Employees

jhk f �kjh

e as sdh

af jk sad k

f sa� kljsd

sd � jshf

jhk f fdf h

kjhe as sd

h af j k sad

kf sa � klj

sd sd �jsh

f

kl f s df hs

akf h ewre

fdsf asfu

oasf adsit

sdfi e o a

sf k�

jhk f �kjh

e as sdh

af jk sad k

f sa� kljsd

sd � jshf

jhk f fdf h

kjhe as sd

h af j k sad

kf sa � klj

sd sd �jsh

f

kl f s df hs

akf h ewre

fdsf asfu

oasf adsit

sdfi e o a

sf k�

jhk f �kjh

e as sdh

af jk sad k

f sa� kljsd

sd � jshf

hk

f fdf hkjhe

as s dh af

jk sa d kf

sa� kljsd

sd � jshf

kl f s df hs

akf h ewre

fdsf asfu

oasf adsit

sdfi e o a

sf k�

jhk f �kjh

e as sdh

af jk sad k

f sa� kljsd

sd � jshf

jhk f fdf h

kjhe as sd

h af j k sad

kf sa � klj

sd sd �jsh

f

kl f s df hs

akf h ewre

fdsf asfu

oasf adsit

sdfi e o a

sf k�

jhk f �kjh

e as sdh

af jk sad k

f sa� kljsd

sd � jshf

jhk f fdf

hkjh e as

sdh a f jk sa

d kf sa�

kljsd sd �

jshf

kl f s df hs

akf h ewre

fdsf asfu

oasf adsit

sdfi e o a

sf k�

jhk f �kjh

e as sdh

af jk sad k

f sa� kljsd

sd � jshf

jhk f fdf h

kjhe as sd

h af j k sad

kf sa � klj

sd sd �jsh

f

kl f s df hs

akf h ewre

fdsf asfu

oasf adsit

sdfi e o a

sf k�

jhk f �kjh

e as sdh

af jk sad k

f sa� kljsd

sd � jshf

jhk f fdf h

kjhe as sd

h af j k sad

kf sa � klj

sd sd �jsh

f

kl f s df hs

akf h ewre

fdsf asfu

oasf adsit

sdfi e o a

sf k�

Rental Agreement

Interest

EXHIBIT 12B.1 Major Classes of Operating Cash Flows

Operating Cash Receipts The financial statements and additional information reported by Genesis in Exhibit 12.10 show one cash receipt: sales to customers. We start with sales to customers as reported on the income statement and then adjust it to get cash received from customers.

Cash Received from Customers If all sales are for cash, cash received from customers equals the sales reported on the income statement. When some or all sales are on credit, we must adjust the amount of sales for the change in Accounts Receivable. To help us compute cash receipts, we use a T-account that includes accounts receivable balances for Genesis on December 31, 2018 and 2019. The beginning bal- ance is $40,000 and the ending balance is $60,000. Next, the income statement shows sales of $590,000, which is put on the debit side. We now reconstruct the account to determine the cash receipts from cus- tomers are $570,000, computed as $40,000 + $590,000 − [?] = $60,000.

Point: An accounts receivable in- crease implies that cash received from customers is less than sales (the converse is also true).

Bal., Dec. 31, 2018 40,000

Sales 590,000

Bal., Dec. 31, 2019 60,000

Cash receipts = 570,000

Accounts Receivable Reconstructed Entry

Cash . . . . . . . . . . 570,000 Accts Recble. . . . 20,000 Sales . . . . . . . 590,000

Cash receipts also can be computed as sales of $590,000 minus a $20,000 increase in accounts receivable. This computation is in Exhibit 12B.2. Genesis reports the $570,000 cash received from customers as a cash inflow from operating activities.

Example: If the ending balance of Accounts Receivable is $20,000 (instead of $60,000), what is cash received from customers? Answer: $610,000

+ or – Cash receipts

or cash payments

Revenue or

expense

Adjustments for changes in related

balance sheet accounts =

Chapter 12 Reporting Cash Flows 473

Other Cash Receipts Other common cash receipts involve rent, interest, and dividends. We compute cash received from these items by subtracting an increase in their receivable or adding a decrease. For example, if rent receivable increases in the period, cash received from renters is less than rent revenue reported on the income statement. If rent receivable decreases, cash received is more than reported rent revenue. The same applies to interest and dividends.

Operating Cash Payments The financial statements and additional information for Genesis in Exhibit 12.10 show four operating expenses: cost of goods sold; wages and other operating expenses; interest expense; and taxes expense. We analyze each expense to compute its cash impact.

Cash Paid for Inventory We compute cash paid for inventory by analyzing both cost of goods sold and inventory. If all inventory purchases are for cash and the balance of Inventory is unchanged, the amount of cash paid for inventory equals cost of goods sold—an uncommon situation. Instead, there normally is some change in the Inventory balance. Also, some or all purchases are often made on credit, which changes the Accounts Payable balance. When the balances of both Inventory and Accounts Payable change, we must adjust the cost of goods sold for changes in both accounts to compute cash paid for inven- tory. This is a two-step adjustment. First, we use the change in the account balance of Inventory, along with the cost of goods sold amount, to compute cost of purchases for the period. An increase in inventory means that we bought more than we sold, and we add this inventory increase to cost of goods sold to compute cost of purchases. A decrease in inventory means that we bought less than we sold, and we subtract the inventory decrease from cost of goods sold to compute purchases. We show the first step by reconstructing the Inventory account. We determine purchases to be $314,000, computed as cost of goods sold of $300,000 plus the $14,000 in- crease in inventory.

Cash received from customers = Sales + Decrease in accounts receivable

or – Increase in accounts receivable

EXHIBIT 12B.2 Formula to Compute Cash Received from Customers— Direct Method

The second step uses the change in the balance of Accounts Payable, and the cost of purchases, to com- pute cash paid for inventory. A decrease in accounts payable means that we paid for more goods than we acquired this period, and we would add the accounts payable decrease to cost of purchases to compute cash paid for inventory. An increase in accounts payable means that we paid for less than the amount of goods acquired, and we would subtract the accounts payable increase from purchases to compute cash paid for inventory. The second step is applied to Genesis by reconstructing its Accounts Payable account to get cash paid of $319,000 (or $40,000 + $314,000 − [?] = $35,000).

Bal., Dec. 31, 2018 70,000

Purchases = 314,000

Bal., Dec. 31, 2019 84,000

Cost of goods sold 300,000

Inventory

Cash payments = 319,000

Bal., Dec. 31, 2018 40,000

Purchases 314,000

Bal., Dec. 31, 2019 35,000

Accounts Payable Reconstructed EntryCOGS . . . . . . . . . . . . 300,000 Inventory . . . . . . . . . 14,000 Accounts Payable . . 5,000 Cash. . . . . . . . . . . 319,000

Alternatively, cash paid for inventory is equal to purchases of $314,000 plus the $5,000 decrease in ac- counts payable. The $319,000 cash paid for inventory is reported as a cash outflow under operating activities. This two-step adjustment to cost of goods sold to compute cash paid for inventory is in Exhibit 12B.3.

Example: If the ending balances of Inventory and Accounts Payable are $60,000 and $50,000, respectively (instead of $84,000 and $35,000), what is cash paid for inventory? Answer: $280,000

Purchases = Cost of goods sold + Increase in inventory

or –

+ or

Decrease in inventory

Cash paid for inventory = Purchases Decrease in accounts payable

Increase in accounts payable

1

2

EXHIBIT 12B.3 Two Steps to Compute Cash Paid for Inventory— Direct Method

474 Chapter 12 Reporting Cash Flows

Cash Paid for Wages and Operating Expenses (Excluding Depreciation) The Genesis income statement shows wages and other operating expenses of $216,000 (see Exhibit 12.10). To compute cash paid for wages and other operating expenses, we adjust for any changes in related balance sheet accounts. We begin by looking for any prepaid expenses and accrued liabilities related to wages and other operating expenses in the balance sheets in Exhibit 12.10. The balance sheets show prepaid expenses but no accrued liabilities. Thus, the adjustment is only for the change in prepaid expenses. The adjustment is computed by assuming that all cash paid for wages and other operating expenses is initially debited to Prepaid Expenses. This assumption allows us to reconstruct the Prepaid Expenses account to get cash paid of $218,000.

Bal., Dec. 31, 2018 4,000

Cash payments = 218,000

Bal., Dec. 31, 2019 6,000

Wages and other operating exp. 216,000

Prepaid Expenses

Reconstructed Entry Wages & Other Exp. . 216,000 Prepaid Expenses . . 2,000 Cash. . . . . . . . . . . 218,000

Cash paid also can be calculated as reported expenses of $216,000 plus the $2,000 increase in prepaid expenses. Exhibit 12B.4 summarizes the adjustments to wages (including salaries) and other operating expenses.

Cash paid for wages and other

operating expenses =

Wages and other

operating expenses

+ Increase in prepaid expenses

Decrease in prepaid expenses

+

Decrease in accrued liabilities

Increase in accrued liabilities

or or

EXHIBIT 12B.4 Formula to Compute Cash Paid for Wages and Operating Expenses— Direct Method

Cash Paid for Accrued Liabilities The Genesis balance sheet did not report accrued liabilities, but we include them in the formula to explain the adjustment to cash when they do exist. A decrease in accrued liabilities means that we paid cash for more goods or services than received this period, so cash paid is higher than the recorded expense. Alternatively, an increase in accrued liabilities implies that we paid less cash than what was received, so cash paid is less than the recorded expense.

Cash Paid for Interest and Income Taxes Computing operating cash flows for interest and taxes requires adjustments for amounts reported on the income statement for changes in related balance sheet accounts. The Genesis income statement shows interest expense of $7,000 and income taxes expense of $15,000. To compute the cash paid, we adjust interest expense for the change in interest payable and adjust income taxes expense for the change in income taxes payable. These computations involve reconstructing both liability accounts and show cash paid for interest of $8,000 and cash paid for income taxes of $5,000.

Cash paid for interest = 8,000 Bal., Dec. 31, 2018 4,000

Interest expense 7,000

Bal., Dec. 31, 2019 3,000

Interest Payable

Cash paid for taxes = 5,000 Bal., Dec. 31, 2018 12,000

Income taxes expense 15,000

Bal., Dec. 31, 2019 22,000

Income Taxes PayableReconstructed Entry Interest Expense . . . . . 7,000 Interest Payable. . . . . . 1,000 Cash. . . . . . . . . . . . . 8,000

Reconstructed Entry Income Tax Exp. . . . . . 15,000 Income Tax Pay. . . . 10,000 Cash. . . . . . . . . . . . . 5,000

The formulas to compute these amounts are in Exhibit 12B.5. Both of these cash payments are reported as operating cash outflows.

+ Decrease in interest payable

+

Increase in interest payable

Decrease in income taxes payable

Increase in income taxes payable Cash paid for taxes = Income taxes expense

Cash paid for interest = Interest expense or

or

EXHIBIT 12B.5 Formulas to Compute Cash Paid for Both Interest and Taxes—Direct Method

Analyzing Additional Expenses, Gains, and Losses Genesis has three more items reported on its income statement: depreciation, loss on sale of assets, and gain on retirement of debt. We consider each for its potential cash effects.

Depreciation Expense Depreciation expense is $24,000. It is often called a noncash expense because depreciation has no cash flows. Depreciation expense is never reported on a statement of cash flows using the direct method; nor is depletion or amortization expense.

Loss on Sale of Assets Sales of assets frequently result in gains and losses reported as part of net income, but the amount of recorded gain or loss does not impact cash. Thus, the loss or gain on a sale of assets is never reported on a statement of cash flows using the direct method.

Point: A decrease in prepaid expenses implies that reported expenses include an amount(s) that did not require a cash outflow in the period.

Chapter 12 Reporting Cash Flows 475

Gain on Retirement of Debt Retirement of debt usually yields a gain or loss reported as part of net income, but that gain or loss does not impact cash. Thus, the loss or gain from retirement of debt is never reported on a statement of cash flows using the direct method.

Summary of Adjustments for Direct Method Exhibit 12B.6 summarizes common adjustments for net income to yield net cash provided (used) by operating activities under the direct method.

Item From Income Statement Adjustments to Obtain Cash Flow Numbers

Receipts

From sales Sales Revenue + Decrease in Accounts Receivable

− Increase in Accounts Receivable

From rent Rent Revenue + Decrease in Rent Receivable

− Increase in Rent Receivable

From interest Interest Revenue + Decrease in Interest Receivable

− Increase in Interest Receivable

From dividends Dividend Revenue + Decrease in Dividends Receivable − Increase in Dividends Receivable Payments

To suppliers Cost of Goods Sold + Increase in Inventory + Decrease in Accounts Payable

− Decrease in Inventory − Increase in Accounts Payable

For operations Operating Expense + Increase in Prepaids + Decrease in Accrued Liabilities

− Decrease in Prepaids − Increase in Accrued Liabilities

To employees Wages (Salaries) Expense + Decrease in Wages (Salaries) Payable

− Increase in Wages (Salaries) Payable

For interest Interest Expense + Decrease in Interest Payable

− Increase in Interest Payable

For taxes Income Tax Expense + Decrease in Income Tax Payable

− Increase in Income Tax Payable

⎫ ⎬ ⎭

⎫ ⎬ ⎭

⎫ ⎬ ⎭

⎫ ⎬ ⎭

⎫ ⎬ ⎭

⎫ ⎬ ⎭

⎫ ⎬ ⎭

⎫ ⎬ ⎭

⎫ ⎬ ⎭

⎫ ⎬ ⎭

⎫ ⎬ ⎭

EXHIBIT 12B.6 Summary of Selected Adjustments for Direct Method

Direct Method Format of Operating Activities Section Exhibit 12B.7 shows the Genesis statement of cash flows using the direct method. Operating cash outflows are subtracted from operating cash inflows to get net cash provided (used) by operating activities.

Point: The FASB requires a recon- ciliation of net income to net cash provided (used) by operating activities when the direct method is used. This reconciliation follows the operating activities section using the indirect method.

GENESIS Statement of Cash Flows (Direct Method)

For Year Ended December 31, 2019

Cash flows from operating activities

Cash received from customers . . . . . . . . . . . . . . . . . . . . . $ 570,000

Cash paid for inventory . . . . . . . . . . . . . . . . . . . . . . . . . . . (319,000)

Cash paid for wages and other operating expenses . . . . (218,000)

Cash paid for interest. . . . . . . . . . . . . . . . . . . . . . . . . . . . . (8,000)

Cash paid for taxes . . . . . . . . . . . . . . . . . . . . . . . . . . . . . . (5,000)

Net cash provided by operating activities . . . . . . . . . . . . $ 20,000

Cash flows from investing activities

Cash received from sale of plant assets . . . . . . . . . . . . . . 2,000

Net cash provided by investing activities . . . . . . . . . . . . . 2,000

Cash flows from financing activities

Cash received from issuing stock . . . . . . . . . . . . . . . . . . . 15,000

Cash paid to retire notes . . . . . . . . . . . . . . . . . . . . . . . . . . (18,000)

Cash paid for dividends . . . . . . . . . . . . . . . . . . . . . . . . . . . (14,000)

Net cash used in financing activities. . . . . . . . . . . . . . . . . (17,000)

Net increase in cash . . . . . . . . . . . . . . . . . . . . . . . . . . . . . . . . $ 5,000

Cash balance at prior year-end . . . . . . . . . . . . . . . . . . . . . . . 12,000

Cash balance at current year-end . . . . . . . . . . . . . . . . . . . . . $ 17,000

EXHIBIT 12B.7 Statement of Cash Flows— Direct Method

Cash from investing

Cash from financing

Cash proved

Cash from operating

476 Chapter 12 Reporting Cash Flows

A company’s current-year income statement and selected balance sheet data at December 31 of the cur- rent and prior years follow. Prepare the operating activities section of the statement of cash flows using the direct method for the current year.

P5

Reporting Operating Cash Flows (Direct)

NEED-TO-KNOW 12-6

Solution

Income Statement For Current Year Ended December 31

Sales revenue . . . . . . . . . . . . . . . . . . . . . . . . . $120

Expenses: Cost of goods sold. . . . . . . . . . . . . 50

Depreciation expense . . . . . . . . . . 30

Salaries expense . . . . . . . . . . . . . . 17

Interest expense . . . . . . . . . . . . . . 3

Net income. . . . . . . . . . . . . . . . . . . . . . . . . . . . $ 20

Selected Balance Sheet Accounts

At December 31 Current Yr Prior Yr

Accounts receivable . . . . $12 $10

Inventory . . . . . . . . . . . . . 6 9

Accounts payable . . . . . . 7 11

Salaries payable . . . . . . . 8 3

Interest payable. . . . . . . . 1 0

Do More: QS 12-21, QS 12-22, QS 12-23, QS 12-24, QS 12-25, QS 12-26, QS 12-27, E 12-15,

E 12-16, E 12-17, E 12-18, E 12-19

Cash Flows from Operating Activities—Direct Method For Current Year Ended December 31

Cash flows from operating activities* Cash received from customers . . . . . . . . . . . . . . . . . . . . . . $118

Cash paid for inventory . . . . . . . . . . . . . . . . . . . . . . . . . . . . (51)

Cash paid for salaries . . . . . . . . . . . . . . . . . . . . . . . . . . . . . (12)

Cash paid for interest. . . . . . . . . . . . . . . . . . . . . . . . . . . . . .    (2)

Net cash provided by operating activities. . . . . . . . . . . . . . . . $53

*Supporting computations: Cash received from customers = Sales of $120 − Accounts Receivable increase of $2. Cash paid for inventory = COGS of $50 − Inventory decrease of $3 + Accounts Payable decrease of $4. Cash paid for salaries = Salaries Expense of $17 − Salaries Payable increase of $5. Cash paid for interest = Interest Expense of $3 − Interest Payable increase of $1.

BASICS OF CASH FLOW REPORTING Format for statement of cash flows:

Noncash investing and financing activities: Some investing and financ- ing activities do not affect cash flows, such as the purchase of long-term assets using a long-term note payable (loan). Such transactions are reported at the bottom of the statement of cash flows or in a note to the statement.

CASH FLOWS FROM OPERATING—INDIRECT Operating activities: Generally include transactions and events that affect net income.

Summary: Cheat Sheet

Operating cash inflow examples: Cash sales to customers, collections on credit sales, receipt of dividend revenue, receipt of interest revenue. Operating cash outflow examples: Cash to pay salaries and wages, pay operating expenses, pay suppliers for goods and services, pay interest owed, pay taxes and fines. Indirect method: Reports net income and then adjusts it for items that do not affect cash. Indirect method only affects the presentation of operating cash flows, not investing or financing sections.

COMPANY NAME Statement of Cash Flows

For period Ended date

Cash flows from operating activities [Compute operating cash flows using indirect or direct method]

Net cash provided (used) by operating activities . . . . . . . . . . . . . . . . . . . . . . . . . . . . . . . . . . . . . . . . $ #

Cash flows from investing activities [List of individual inflows and outflows]

Net cash provided (used) by investing activities. . . . . . . . . . . . . . . . . . . . . . . . . . . . . . . . . . . . . . . . . #

Cash flows from financing activities [List of individual inflows and outflows]

Net cash provided (used) by financing activities . . . . . . . . . . . . . . . . . . . . . . . . . . . . . . . . . . . . . . . . #

Net increase (decrease) in cash . . . . . . . . . . . . . . . . . . . . . . . . . . . . . . . . . . . . . . . . . . . . . . . . . . . . . . $ # Cash (and equivalents) balance at prior period-end . . . . . . . . . . . . . . . . . . . . . . . . . . . . . . . . . . . . . # Cash (and equivalents) balance at current period-end . . . . . . . . . . . . . . . . . . . . . . . . . . . . . . . . . . . $ #

Separate schedule or note disclosure of any noncash investing and financing transactions is required.

Summary of adjustments for indirect method:

Net Income (or Loss) Adjustments for operating items not providing or using cash

+ Noncash expenses and losses Examples: Expenses for depreciation, depletion, and amortization;

losses from disposal of long-term assets and from retirement of debt

− Noncash revenues and gains Examples: Gains from disposal of long-term assets and from retirement of debt

Adjustments for changes in current assets and current liabilities

+ Decrease in noncash current operating asset − Increase in noncash current operating asset + Increase in current operating liability − Decrease in current operating liability Net cash provided (used) by operating activities

1

2

Chapter 12 Reporting Cash Flows 477

CASH FLOWS FROM FINANCING Financing activities: Generally include transactions and events that affect long-term liabilities and equity. Financing cash inflow examples: Cash from issuing common and pre- ferred stock, issuing short- and long-term debt (notes payable and bonds payable), reissuing treasury stock. Financing cash outflow examples: Cash to pay dividends to sharehold- ers, pay off short- and long-term debt (notes payable and bonds payable), purchase treasury stock.

CASH FLOWS FROM OPERATING—DIRECT Direct method: Separately lists operating cash receipts and operating cash payments. Cash payments are subtracted from cash receipts. Unlike the indirect method, it does not start with net income. This only affects the operating section of the statement of cash flows.

CASH FLOWS FROM INVESTING Investing activities: Generally include transactions and events that come from the purchase and sale of long-term assets. Investing cash inflow examples: Cash from selling plant assets, selling intangible assets, selling short-term and long-term investments, selling notes receivable, collecting principal (but not interest) on notes receivable. Investing cash outflow examples: Cash to buy plant assets, buy intangi- ble assets, buy short-term and long-term investments, loan money in return for notes receivable.

Example of financing section format:

Example of investing section format:

Cash flows from investing activities

Cash received from sale of plant assets . . . . . . . . . . . . . $2,000

Net cash provided by investing activities . . . . . . . . . . . . $2,000

Cash flows from financing activities

Cash received from issuing stock . . . . . . . . . . . . . . $ 15,000

Cash paid to retire notes . . . . . . . . . . . . . . . . . . . . . (18,000)

Cash paid for dividends . . . . . . . . . . . . . . . . . . . . . . (14,000)

Net cash used in financing activities. . . . . . . . . . . . $(17,000)

Summary of adjustments for direct method: Item From Income Statement Adjustments to Obtain Cash Flow Numbers

Receipts

From sales Sales Revenue + Decrease in Accounts Receivable

− Increase in Accounts Receivable

From rent Rent Revenue + Decrease in Rent Receivable

− Increase in Rent Receivable

From interest Interest Revenue + Decrease in Interest Receivable

− Increase in Interest Receivable

From dividends Dividend Revenue + Decrease in Dividends Receivable − Increase in Dividends Receivable Payments

To suppliers Cost of Goods Sold + Increase in Inventory + Decrease in Accounts Payable

− Decrease in Inventory − Increase in Accounts Payable

For operations Operating Expense + Increase in Prepaids + Decrease in Accrued Liabilities

− Decrease in Prepaids − Increase in Accrued Liabilities

To employees Wages (Salaries) Expense + Decrease in Wages (Salaries) Payable

− Increase in Wages (Salaries) Payable

For interest Interest Expense + Decrease in Interest Payable

− Increase in Interest Payable

For taxes Income Tax Expense + Decrease in Income Tax Payable

− Increase in Income Tax Payable

⎫ ⎬ ⎭

⎫ ⎬ ⎭

⎫ ⎬ ⎭

⎫ ⎬ ⎭

⎫ ⎬ ⎭

⎫ ⎬ ⎭

⎫ ⎬ ⎭

⎫ ⎬ ⎭

⎫ ⎬ ⎭

⎫ ⎬ ⎭

⎫ ⎬ ⎭

Cash flow on total assets (467) Direct method (457) Financing activities (457)

Indirect method (457) Investing activities (454)

Operating activities (454) Statement of cash flows (453)

Key Terms

Multiple Choice Quiz

1. A company uses the indirect method to determine its cash flows from operating activities. Use the following informa- tion to determine its net cash provided or used by operating activities.

a. $23,550 used by operating activities b. $23,550 provided by operating activities c. $15,550 provided by operating activities d. $42,400 provided by operating activities e. $20,850 provided by operating activities

2. A machine with a cost of $175,000 and accumulated deprecia- tion of $94,000 is sold for $87,000 cash. The amount reported as a source of cash under cash flows from investing activities is a. $81,000. b. $6,000. c. $87,000. d. $0; this is a financing activity. e. $0; this is an operating activity.

3. A company settles a long-term note payable plus interest by paying $68,000 cash toward the principal amount and $5,440 cash for interest. The amount reported as a use of cash under cash flows from financing activities is a. $0; this is an investing activity. b. $0; this is an operating activity. c. $73,440. d. $68,000. e. $5,440.

Net income . . . . . . . . . . . . . . . . . . . . . . . . . . . . . $15,200

Depreciation expense . . . . . . . . . . . . . . . . . . . . 10,000

Cash payment on note payable . . . . . . . . . . . . . 8,000

Gain on sale of land . . . . . . . . . . . . . . . . . . . . . . 3,000

Increase in inventory . . . . . . . . . . . . . . . . . . . . . 1,500

Increase in accounts payable. . . . . . . . . . . . . . . 2,850

478 Chapter 12 Reporting Cash Flows

A(B) Superscript letter A or B denotes assignments based on Appendix 12A or 12B.

Icon denotes assignments that involve decision making.

1. What is the reporting purpose of the statement of cash flows? Identify at least two questions that this statement can answer.

2. What are some investing activities reported on the state- ment of cash flows?

3. What are some financing activities reported on the state- ment of cash flows?

4.B Describe the direct method of reporting cash flows from operating activities.

5.B When a statement of cash flows is prepared using the direct method, what are some of the operating cash flows?

6. Describe the indirect method of reporting cash flows from operating activities.

7. Where on the statement of cash flows is the payment of cash dividends reported?

8. Assume that a company purchases land for $1,000,000, paying $400,000 cash and borrowing the remainder with a long-term note payable. How should this transaction be re- ported on a statement of cash flows?

9. On June 3, a company borrows $200,000 cash by giv- ing its bank a 90-day, interest-bearing note. On the state- ment of cash flows, where should this be reported?

10. If a company reports positive net income for the year, can it also show a net cash outflow from operating activi- ties? Explain.

11. Is depreciation a source of cash flow? 12. Refer to Apple’s statement of cash flows in

Appendix A. (a) Which method is used to com- pute its net cash provided by operating activities? (b) Its bal- ance sheet shows an increase in accounts receivable from September 24, 2016, to September 30, 2017; why is this in- crease in accounts receivable subtracted when computing net cash provided by operating activities for the fiscal year ended September 30, 2017?

13. Refer to Google’s statement of cash flows in Appendix A. What are its cash flows from financing activities for the year ended December 31, 2017? List the items and amounts.

14. Refer to Samsung’s 2017 statement of cash flows in Appendix A. List its cash flows from operating activities, investing activities, and fi- nancing activities.

15. Refer to Samsung’s statement of cash flows in Appendix A. What invest- ing activities result in cash outflows for the year ended December 31, 2017? List items and amounts.

Discussion Questions

APPLE

Samsung

Samsung

GOOGLE

4. The following information is available regarding a company’s annual salaries and wages. What amount of cash is paid for salaries and wages?

5. The following information is available for a company. What amount of cash is paid for inventory for the current year?

a. $252,300 c. $255,000 e. $235,900 b. $257,700 d. $274,100

a. $545,000 c. $540,800 e. $549,200 b. $554,800 d. $535,200

ANSWERS TO MULTIPLE CHOICE QUIZ

2. c; Cash received from sale of machine is reported as an investing activity.

3. d; FASB requires cash interest paid to be reported under operating. 4. a; Cash paid for salaries and wages = $255,000 + $8,200 −

$10,900 = $252,300 5. e; Increase in inventory = $112,000 − $105,000 = $7,000 Increase in accounts payable = $101,300 − $98,500 = $2,800 Cash paid for inventory = $545,000 + $7,000 − $2,800 = $549,200

Salaries and wages expense . . . . . . . . . . . . . . . . . . . . $255,000 Salaries and wages payable, prior year-end. . . . . . . . 8,200 Salaries and wages payable, current year-end. . . . . . 10,900

Cost of goods sold . . . . . . . . . . . . . . . . . . $545,000 Inventory, prior year-end . . . . . . . . . . . . . 105,000 Inventory, current year-end . . . . . . . . . . . 112,000 Accounts payable, prior year-end . . . . . . 98,500 Accounts payable, current year-end . . . . 101,300

1. b; Net income . . . . . . . . . . . . . . . . . . . . . . . . . . . . . . $15,200

Depreciation expense . . . . . . . . . . . . . . . . . . . . . 10,000

Gain on sale of land . . . . . . . . . . . . . . . . . . . . . . . (3,000)

Increase in inventory . . . . . . . . . . . . . . . . . . . . . . (1,500)

Increase in accounts payable. . . . . . . . . . . . . . . . 2,850

Net cash provided by operations. . . . . . . . . . . . . $23,550

Classify the following cash flows as either operating (O), investing (I), or financing (F) activities. 1. Sold stock investments for cash. 6. Issued common stock for cash. 2. Received cash payments from customers. 7. Received cash interest on a note. 3. Paid cash for wages and salaries. 8. Paid cash interest on outstanding notes. 4. Purchased inventories with cash. 9. Received cash from sale of land. 5. Paid cash dividends. 10. Paid cash for property taxes on building.

QUICK STUDY

QS 12-1 Classifying transactions by activity

C1

Chapter 12 Reporting Cash Flows 479

Label the following headings, line items, and notes with the numbers 1 through 13 according to their se- quential order (from top to bottom) for presentation on the statement of cash flows.

a. “Cash flows from investing activities” title b. “For period Ended date” heading c. “Cash flows from operating activities” title d. Company name e. Schedule or note disclosure of noncash investing and financing transactions f. “Statement of Cash Flows” heading g. Net increase (decrease) in cash. . . . . . . . . . . . . . . . . . . . . . . . . . $ h. Net cash provided (used) by operating activities . . . . . . . . . . . . $ i. Cash (and equivalents) balance at prior period-end . . . . . . . . . . $ j. Net cash provided (used) by financing activities . . . . . . . . . . . . $ k. “Cash flows from financing activities” title l. Net cash provided (used) by investing activities. . . . . . . . . . . . . $ m. Cash (and equivalents) balance at current period-end . . . . . . . . $

QS 12-2 Statement of cash flows

P1

Bryant Co. reports net income of $20,000. For the year, depreciation expense is $7,000 and the company reports a gain of $3,000 from sale of machinery. It also had a $2,000 loss from retirement of notes. Compute cash flows from operations using the indirect method.

QS 12-3 Indirect: Computing cash flows from operations P2

Cain Inc. reports net income of $15,000. Its comparative balance sheet shows the following changes: ac- counts receivable increased $6,000; inventory decreased $8,000; prepaid insurance decreased $1,000; accounts payable increased $3,000; and taxes payable decreased $2,000. Compute cash flows from opera- tions using the indirect method.

QS 12-4 Indirect: Computing cash flows from operations P2

For each separate company, compute cash flows from operations using the indirect method. QS 12-5 Indirect: Computing cash flows from operations

P2 Twix Dots Skor

Net income . . . . . . . . . . . . . . . . . . . . . . . . . . . . . . . . . . . . . $ 4,000 $100,000 $72,000

Depreciation expense . . . . . . . . . . . . . . . . . . . . . . . . . . . . 30,000 8,000 24,000

Accounts receivable increase (decrease) . . . . . . . . . . . . . 40,000 20,000 (4,000)

Inventory increase (decrease) . . . . . . . . . . . . . . . . . . . . . . (20,000) (10,000) 10,000

Accounts payable increase (decrease) . . . . . . . . . . . . . . . 24,000 (22,000) 14,000

Accrued liabilities increase (decrease) . . . . . . . . . . . . . . . (44,000) 12,000 (8,000)

QS 12-6 Indirect: Computing cash from operations P2

Use the following information to determine cash flows from operating activities using the indirect method.

MOSS COMPANY Selected Balance Sheet Information

December 31, 2019 and 2018

2019 2018 Current assets

Cash . . . . . . . . . . . . . . . . . $84,650 $26,800

Accounts receivable. . . . . 25,000 32,000

Inventory. . . . . . . . . . . . . . 60,000 54,100

Current liabilities

Accounts payable. . . . . . . 30,400 25,700

Income taxes payable . . . 2,050 2,200

MOSS COMPANY Income Statement

For Year Ended December 31, 2019

Sales . . . . . . . . . . . . . . . . . . . $515,000

Cost of goods sold . . . . . . . . 331,600

Gross profit . . . . . . . . . . . . . . 183,400

Operating expenses

Depreciation expense . . . $ 36,000

Other expenses . . . . . . . . 121,500 157,500

Income before taxes. . . . . . . 25,900

Income taxes expense . . . . . 7,700

Net income . . . . . . . . . . . . . . $ 18,200

480 Chapter 12 Reporting Cash Flows

CRUZ, INC. Comparative Balance Sheets

At December 31 2019 2018

Assets Cash. . . . . . . . . . . . . . . . . . . . . . . . . . . $ 94,800 $ 24,000

Accounts receivable, net . . . . . . . . . . 41,000 51,000

Inventory . . . . . . . . . . . . . . . . . . . . . . . 85,800 95,800

Prepaid expenses . . . . . . . . . . . . . . . . 5,400 4,200

Total current assets . . . . . . . . . . . . . . 227,000 175,000

Furniture . . . . . . . . . . . . . . . . . . . . . . . 109,000 119,000

Accum. depreciation—Furniture . . . . (17,000) (9,000)

Total assets . . . . . . . . . . . . . . . . . . . . . $319,000 $285,000

Liabilities and Equity Accounts payable . . . . . . . . . . . . . . . . $ 15,000 $ 21,000

Wages payable . . . . . . . . . . . . . . . . . . 9,000 5,000

Income taxes payable. . . . . . . . . . . . . 1,400 2,600

Total current liabilities . . . . . . . . . . . . 25,400 28,600

Notes payable (long-term) . . . . . . . . . 29,000 69,000

Total liabilities . . . . . . . . . . . . . . . . . . . 54,400 97,600

Equity Common stock, $5 par value . . . . . . . 229,000 179,000

Retained earnings. . . . . . . . . . . . . . . . 35,600 8,400

Total liabilities and equity. . . . . . . . . . $319,000 $285,000

CRUZ, INC. Income Statement

For Year Ended December 31, 2019

Sales . . . . . . . . . . . . . . . . . . . . $488,000

Cost of goods sold . . . . . . . . . 314,000

Gross profit . . . . . . . . . . . . . . . 174,000

Operating expenses

Depreciation expense . . . . $37,600

Other expenses . . . . . . . . . 89,100 126,700

Income before taxes. . . . . . . . 47,300

Income taxes expense . . . . . . 17,300

Net income . . . . . . . . . . . . . . . $ 30,000

QS 12-7 Indirect: Computing cash from operations P2

Required

Use the indirect method to prepare the operating activities section of Cruz’s statement of cash flows.

QS 12-9 Computing investing cash flows

P3

Indicate the effect each separate transaction has on investing cash flows. a. Sold a truck costing $40,000, with $22,000 of accumulated depreciation, for $8,000 cash. The sale

results in a $10,000 loss. b. Sold a machine costing $10,000, with $8,000 of accumulated depreciation, for $5,000 cash. The sale

results in a $3,000 gain. c. Purchased stock investments for $16,000 cash. The purchaser believes the stock is worth at least $30,000.

QS 12-8 Computing cash from asset sales

P3

The following information is from Ellerby Company’s comparative balance sheets. The current-year in- come statement reports depreciation expense on furniture of $18,000. During the year, furniture costing $52,500 was sold for its book value. Compute cash received from the sale of furniture.

At December 31 Current Year Prior Year Furniture . . . . . . . . . . . . . . . . . . . . . . . . . . . . . $132,000 $ 184,500

Accumulated depreciation—Furniture. . . . . . (88,700) (110,700)

QS 12-10 Computing investing cash flows

P3

The plant assets section of the comparative balance sheets of Anders Company is reported below.

ANDERS COMPANY Comparative Year-End Balance Sheets

Plant assets 2019 2018 Equipment . . . . . . . . . . . . . . . . . . . . . . . . . . . . . . . . $ 180,000 $ 270,000

Accumulated depreciation—Equipment . . . . . . . . . (100,000) (210,000)

Equipment, net . . . . . . . . . . . . . . . . . . . . . . . . . . . . . $ 80,000 $ 60,000

Buildings. . . . . . . . . . . . . . . . . . . . . . . . . . . . . . . . . . $ 380,000 $ 400,000

Accumulated depreciation—Buildings . . . . . . . . . . (100,000) (285,000)

Buildings, net . . . . . . . . . . . . . . . . . . . . . . . . . . . . . . $ 280,000 $ 115,000

Chapter 12 Reporting Cash Flows 481

Refer to the balance sheet data above from Anders Company. During 2019, equipment with a book value of $40,000 and an original cost of $210,000 was sold at a loss of $3,000. 1. How much cash did Anders receive from the sale of equipment? 2. How much depreciation expense was recorded on equipment during 2019? 3. What was the cost of new equipment purchased by Anders during 2019?

QS 12-11 Computing investing cash flows

P3

Refer to the balance sheet data in QS 12-10 from Anders Company. During 2019, a building with a book value of $70,000 and an original cost of $300,000 was sold at a gain of $60,000. 1. How much cash did Anders receive from the sale of the building? 2. How much depreciation expense was recorded on buildings during 2019? 3. What was the cost of buildings purchased by Anders during 2019?

QS 12-14 Computing financing cash flows

P3

Indicate the effect, if any, that each separate transaction has on financing cash flows. a. Notes payable with a carrying value of $15,000 are retired for $16,000 cash, resulting in a $1,000 gain. b. Paid cash dividends of $11,000 to common stockholders. c. Acquired $20,000 worth of machinery in exchange for common stock.

Compute cash flows from investing activities using the following company information. QS 12-12 Computing cash flows from investing

P3 Sale of short-term stock investments . . . . . . . . . $ 6,000 Cash purchase of used equipment . . . . . . $5,000

Cash collections from customers . . . . . . . . . . . . . 16,000 Depreciation expense . . . . . . . . . . . . . . . . 2,000

Compute cash flows from financing activities using the following company information. QS 12-16 Computing cash flows from financing

P3 Cash received from short-term note payable . . . . . . . $20,000 Cash dividends paid . . . . . . . . . . . . . . . . . . $16,000

Purchase of short-term stock investments . . . . . . . . 5,000 Interest paid . . . . . . . . . . . . . . . . . . . . . . . . 8,000

Use the following information for VPI Co. to prepare a statement of cash flows for the year ended December 31 using the indirect method.

QS 12-18 Indirect: Preparing statement of cash flows

P2 P3Cash balance at prior year-end . . . . . . . . . . . . . $40,000 Gain on sale of machinery . . . . . . . . . . . . . . . . . $ 2,000 Increase in inventory . . . . . . . . . . . . . . . . . . . . . 5,000 Cash received from sale of machinery . . . . . . . 9,500

Depreciation expense . . . . . . . . . . . . . . . . . . . . 4,000 Increase in accounts payable . . . . . . . . . . . . . . 1,500

Cash received from issuing stock . . . . . . . . . . . 8,000 Net income . . . . . . . . . . . . . . . . . . . . . . . . . . . . . 23,000

Cash paid for dividends . . . . . . . . . . . . . . . . . . . 1,000 Decrease in accounts receivable . . . . . . . . . . . 3,000

QS 12-13 Computing cash from asset sales P3

Refer to the data in QS 12-7. Furniture costing $55,000 is sold at its book value in 2019. Acquisitions of furniture total $45,000 cash, on which no depreciation is necessary because it is acquired at year-end. What is the cash inflow from the sale of furniture?

QS 12-17 Computing financing cash outflows P3

Refer to the data in QS 12-7. 1. Assume that all common stock is issued for cash. What amount of cash dividends is paid during 2019? 2. Assume that no additional notes payable are issued in 2019. What cash amount is paid to reduce the

notes payable balance in 2019?

The following information is from Princeton Company’s comparative balance sheets. QS 12-15 Computing financing cash flows

P3

The company’s net income for the current year ended December 31 was $48,000. 1. Compute the cash received from the sale of its common stock during the current year. 2. Compute the cash paid for dividends during the current year.

At December 31 Current Year Prior Year Common stock, $10 par value. . . . . . . . . . . . $105,000 $100,000

Paid-in capital in excess of par . . . . . . . . . . . 567,000 342,000

Retained earnings. . . . . . . . . . . . . . . . . . . . . . 313,500 287,500

482 Chapter 12 Reporting Cash Flows

Financial data from three competitors in the same industry follow. 1. Rank the three companies from high to low on cash from operating activities. 2. Which company has the largest cash outflow for investing activities? 3. Which company has the largest cash inflow from financing activities? 4. Which company has the highest cash flow on total assets ratio?

QS 12-19 Interpreting disclosures on sources and uses of cash

A1

$ 70,000 (28,000) (6,000) $ 36,000 $790,000

$ 60,000 (34,000) 0 $ 26,000

$625,000

$ (24,000) 26,000

23,000 $ 25,000

$300,000

Cash provided (used) by operating activities Cash provided (used) by investing activities Cash provided (used) by financing activities Net increase (decrease) in cash

Average total assets

CluedoYahtzeeMancala

QS 12-20A Recording entries in a spreadsheet

P4

A company uses a spreadsheet to prepare its statement of cash flows. Indicate whether each of the follow- ing items would be recorded in the Debit column or Credit column of the spreadsheet’s statement of cash flows section. a. Decrease in accounts payable d. Loss on sale of machinery b. Payment of cash dividends e. Net income c. Increase in accounts receivable f. Increase in interest payable

QS 12-21B Direct: Computing cash receipts from operations

P5

Russell Co. reports sales revenue of $30,000 and interest revenue of $5,000. Its comparative balance sheet shows that accounts receivable decreased $4,000 and interest receivable increased $1,000. Compute cash provided by operating activities using the direct method.

QS 12-23B Direct: Computing cash paid for operations P5

BTN Inc. reports operating expenses of $27,000. Its comparative balance sheet shows that accrued liabil- ities decreased $6,000 and prepaid expenses increased $2,000. Compute cash used in operating activities using the direct method.

QS 12-25B Direct: Computing cash received from customers

P5

Refer to the data in QS 12-7. 1. How much cash is received from sales to customers for year 2019? 2. What is the net increase or decrease in the Cash account for year 2019?

QS 12-26B Direct: Computing operating cash outflows

P5

Refer to the data in QS 12-7. 1. How much cash is paid to acquire inventory during year 2019? 2. How much cash is paid for “other expenses” during year 2019? Hint: Examine prepaid expenses and

wages payable.

QS 12-22B Direct: Computing cash payments to suppliers P5

Bioware Co. reports cost of goods sold of $42,000. Its comparative balance sheet shows that inventory decreased $7,000 and accounts payable increased $5,000. Compute cash payments to suppliers using the direct method.

For each separate case, compute the required cash flow information for BioClean.QS 12-24B Direct: Computing cash flows

P5 Case A: Compute cash interest received Interest revenue . . . . . . . . . . . . . . . . . . . . . . . . . . . $5,000

Interest receivable, beginning of year. . . . . . . . . . 600

Interest receivable, end of year . . . . . . . . . . . . . . 1,700

Case B: Compute cash paid for wages Wages expense . . . . . . . . . . . . . . . . . . . . . . . $9,000

Wages payable, beginning of year . . . . . . . . 2,200

Wages payable, end of year . . . . . . . . . . . . . 1,000

QS 12-27B Direct: Computing cash from operations P5

Refer to the data in QS 12-7. Use the direct method to prepare the operating activities section of Cruz’s statement of cash flows.

Chapter 12 Reporting Cash Flows 483

EXERCISES

Exercise 12-1 Indirect: Classifying cash flows

C1

Indicate where each item would appear on a statement of cash flows using the indirect method by placing an x in the appropriate column.

Statement of Cash Flows Noncash Not Investing Reported on Operating Investing Financing and Financing Statement Activities Activities Activities Activities or in Notes

a. Declared and paid a cash dividend . . . . . . . . . . b. Recorded depreciation expense . . . . . . . . . . . . c. Paid cash to settle long-term note payable . . . d. Prepaid expenses increased in the year . . . . . . e. Accounts receivable decreased in the year . . . f. Purchased land by issuing common stock . . . . g. Inventory increased in the year . . . . . . . . . . . . . h. Sold equipment for cash, yielding a loss . . . . . . i. Accounts payable decreased in the year . . . . . j. Income taxes payable increased in the year . . .

Hampton Company reports the following information for its recent calendar year. Prepare the operating activities section of the statement of cash flows using the indirect method.

Exercise 12-2 Indirect: Reporting cash flows from operations

P2Income Statement Data Sales. . . . . . . . . . . . . . . . . . . . . . . . . . . . . . . . $160,000 Expenses: Cost of goods sold. . . . . . . . . . . . 100,000 Salaries expense . . . . . . . . . . . . . 24,000 Depreciation expense . . . . . . . . . 12,000 Net income. . . . . . . . . . . . . . . . . . . . . . . . . . . $ 24,000

Selected Year-End Balance Sheet Data

Accounts receivable increase. . . . . . . . . . $10,000 Inventory decrease . . . . . . . . . . . . . . . . . . 16,000 Salaries payable increase . . . . . . . . . . . . . 1,000

Arundel Company disclosed the following information for its recent calendar year. Prepare the operating activities section of the statement of cash flows using the indirect method.

Income Statement Data

Revenues . . . . . . . . . . . . . . . . . . . . . . . . . . . . . . $100,000 Expenses: Salaries expense . . . . . . . . . . . . . . . 84,000 Utilities expense . . . . . . . . . . . . . . . 14,000 Depreciation expense . . . . . . . . . . . 14,600 Other expenses . . . . . . . . . . . . . . . . 3,400 Net loss . . . . . . . . . . . . . . . . . . . . . . . . . . . . . . . $ (16,000)

Selected Year-End Balance Sheet Data

Accounts receivable decrease . . . . . . . . . $24,000 Purchased a machine for cash . . . . . . . . . 10,000 Salaries payable increase . . . . . . . . . . . . . 18,000 Other accrued liabilities decrease . . . . . . 8,000

Exercise 12-3 Indirect: Reporting cash flows from operations

P2

Using the following income statement and additional year-end information, prepare the operating activi- ties section of the statement of cash flows using the indirect method.

Exercise 12-4 Indirect: Cash flows from operating activities

P2SONAD COMPANY Income Statement

For Year Ended December 31

Sales . . . . . . . . . . . . . . . . . . . . . . . . . . . . $1,828,000 Cost of goods sold . . . . . . . . . . . . . . . . . 991,000 Gross profit . . . . . . . . . . . . . . . . . . . . . . . 837,000 Operating expenses Salaries expense . . . . . . . . . . . . . . . . $245,535 Depreciation expense . . . . . . . . . . . . 44,200 Rent expense . . . . . . . . . . . . . . . . . . . 49,600 Amortization expense—Patents . . . . 4,200 Utilities expense. . . . . . . . . . . . . . . . . 18,125 361,660 475,340 Gain on sale of equipment . . . . . . . . . . . 6,200 Net income . . . . . . . . . . . . . . . . . . . . . . . $ 481,540

Selected Year-End Balance Sheet Data

Accounts receivable . . $30,500 increase

Inventory . . . . . . . . . . . 25,000 increase

Accounts payable . . . . 12,500 decrease

Salaries payable . . . . . 3,500 decrease

484 Chapter 12 Reporting Cash Flows

Fitz Company reports the following information. Use the indirect method to prepare the operating activi- ties section of its statement of cash flows for the year ended December 31.

Exercise 12-5 Indirect: Cash flows from operating activities

P2 Selected Annual Income Statement Data Selected Year-End Balance Sheet Data Net income . . . . . . . . . . . . . . . . . . . . . . . . . $374,000 Accounts receivable decrease . . . . . . . . . . . . . . $17,100 Depreciation expense . . . . . . . . . . . . . . . . 44,000 Inventory decrease . . . . . . . . . . . . . . . . . . . . . . . 42,000 Amortization expense. . . . . . . . . . . . . . . . . 7,200 Prepaid expenses increase. . . . . . . . . . . . . . . . . 4,700 Gain on sale of plant assets . . . . . . . . . . . . 6,000 Accounts payable decrease . . . . . . . . . . . . . . . . 8,200 Salaries payable increase. . . . . . . . . . . . . . . . . . 1,200

Salud Company reports the following information. Use the indirect method to prepare the operating ac- tivities section of its statement of cash flows for the year ended December 31.

Exercise 12-6 Indirect: Cash flows from operating activities

P2 Selected Annual Income Statement Data Selected Year-End Balance Sheet Data Net income . . . . . . . . . . . . . . . . . . . . . . . . . $400,000 Accounts receivable increase. . . . . . . . . . . . . . . $40,000 Depreciation expense . . . . . . . . . . . . . . . . 80,000 Prepaid expenses decrease . . . . . . . . . . . . . . . . 12,000 Gain on sale of machinery . . . . . . . . . . . . . 20,000 Accounts payable increase. . . . . . . . . . . . . . . . . 6,000 Wages payable decrease . . . . . . . . . . . . . . . . . . 2,000

Prepare the operating activities section of the statement of cash flows for GreenGarden using the indirect method.

Exercise 12-7 Indirect: Reporting cash flows from operations

P2 Annual Income Statement Data Sales. . . . . . . . . . . . . . . . . . . . . . . . . . . . . . . . . . $50,000

Expenses: Cost of goods sold. . . . . . . . . . . . . . 30,000

Wages expense . . . . . . . . . . . . . . . . 10,000

Amortization expense . . . . . . . . . . . 1,500

Net income. . . . . . . . . . . . . . . . . . . . . . . . . . . . . $ 8,500

Selected Year-End Balance Sheet Data

Prepaid expenses increase. . . . . . . . . . . . . . . . $3,000 Inventory increase . . . . . . . . . . . . . . . . . . . . . . 500 Accounts payable decrease . . . . . . . . . . . . . . . 1,000

Use the following information to determine cash flows from investing activities. a. Equipment with a book value of $65,300 and an original cost of $133,000 was sold at a loss of $14,000. b. Paid $89,000 cash for a new truck. c. Sold land costing $154,000 for $198,000 cash, yielding a gain of $44,000. d. Stock investments were sold for $60,800 cash, yielding a gain of $4,150.

Exercise 12-8 Cash flows from investing activities

P3

Use the following information to determine cash flows from financing activities. a. Net income was $35,000. b. Issued common stock for $64,000 cash. c. Paid cash dividend of $14,600. d. Paid $50,000 cash to settle a note payable at its $50,000 maturity value. e. Paid $12,000 cash to acquire its treasury stock. f. Purchased equipment for $39,000 cash.

Exercise 12-9 Cash flows from financing activities

P3

For each of the following separate transactions, (a) prepare the reconstructed journal entry and (b) identify the effect it has, if any, on the investing section or financing section of the statement of cash flows. 1. Sold a building costing $30,000, with $20,000 of accumulated depreciation, for $8,000 cash, resulting

in a $2,000 loss. 2. Acquired machinery worth $10,000 by issuing $10,000 in notes payable. 3. Issued 1,000 shares of common stock at par for $2 per share. 4. Notes payable with a carrying value of $40,000 were retired for $47,000 cash, resulting in a $7,000

loss.

Exercise 12-10 Reconstructed entries

P3

Chapter 12 Reporting Cash Flows 485

The following financial statements and additional information are reported. (1) Prepare a statement of cash flows using the indirect method for the year ended June 30, 2019. (2) Compute the company’s cash flow on total assets ratio for fiscal year 2019.

Exercise 12-11 Indirect: Preparing statement of cash flows

A1 P2 P3IKIBAN INC. Income Statement

For Year Ended June 30, 2019

Sales . . . . . . . . . . . . . . . . . . . . . . . . . . . . . . . . $678,000

Cost of goods sold . . . . . . . . . . . . . . . . . . . . . 411,000

Gross profit . . . . . . . . . . . . . . . . . . . . . . . . . . . 267,000

Operating expenses

Depreciation expense . . . . . . . . . . . . . . . . $58,600

Other expenses . . . . . . . . . . . . . . . . . . . . . 67,000

Total operating expenses. . . . . . . . . . . . . . . . 125,600

141,400

Other gains (losses)

Gain on sale of equipment. . . . . . . . . . . . . 2,000

Income before taxes. . . . . . . . . . . . . . . . . . . . 143,400

Income taxes expense . . . . . . . . . . . . . . . . . . 43,890

Net income . . . . . . . . . . . . . . . . . . . . . . . . . . . $ 99,510

IKIBAN INC. Comparative Balance Sheets

At June 30 2019 2018

Assets Cash. . . . . . . . . . . . . . . . . . . . . . . . . . . $ 87,500 $ 44,000

Accounts receivable, net . . . . . . . . . . 65,000 51,000

Inventory . . . . . . . . . . . . . . . . . . . . . . . 63,800 86,500

Prepaid expenses . . . . . . . . . . . . . . . . 4,400 5,400

Total current assets . . . . . . . . . . . . . . 220,700 186,900

Equipment. . . . . . . . . . . . . . . . . . . . . . 124,000 115,000

Accum. depreciation—Equipment . . . (27,000) (9,000)

Total assets . . . . . . . . . . . . . . . . . . . . . $317,700 $292,900

Liabilities and Equity Accounts payable . . . . . . . . . . . . . . . . $ 25,000 $ 30,000

Wages payable . . . . . . . . . . . . . . . . . . 6,000 15,000

Income taxes payable. . . . . . . . . . . . . 3,400 3,800

Total current liabilities . . . . . . . . . . . . 34,400 48,800

Notes payable (long term) . . . . . . . . . 30,000 60,000

Total liabilities . . . . . . . . . . . . . . . . . . . 64,400 108,800

Equity Common stock, $5 par value . . . . . . . 220,000 160,000

Retained earnings. . . . . . . . . . . . . . . . 33,300 24,100

Total liabilities and equity. . . . . . . . . . $317,700 $292,900

Additional Information a. A $30,000 note payable is retired at its $30,000 carrying (book)

value in exchange for cash. b. The only changes affecting retained earnings are net income and

cash dividends paid. c. New equipment is acquired for $57,600 cash. d. Received cash for the sale of equipment that had cost $48,600, yield-

ing a $2,000 gain. e. Prepaid Expenses and Wages Payable relate to Other Expenses on

the income statement. f. All purchases and sales of inventory are on credit. Check (1b) Cash paid for dividends, $90,310

(1d) Cash received from equip. sale, $10,000

Use the following information to prepare a statement of cash flows for the current year using the indirect method. Exercise 12-12 Indirect: Preparing statement of cash flows

P2 P3 MONTGOMERY INC.

Comparative Balance Sheets

At December 31 Current Year Prior Year

Assets Cash. . . . . . . . . . . . . . . . . . . . . . . . . . . $ 30,400 $ 30,550

Accounts receivable, net . . . . . . . . . . 10,050 12,150

Inventory . . . . . . . . . . . . . . . . . . . . . . . 90,100 70,150

Total current assets . . . . . . . . . . . . . . 130,550 112,850

Equipment. . . . . . . . . . . . . . . . . . . . . . 49,900 41,500

Accum. depreciation—Equipment . . . (22,500) (15,300)

Total assets . . . . . . . . . . . . . . . . . . . . . $157,950 $139,050

Liabilities and Equity Accounts payable . . . . . . . . . . . . . . . . $ 23,900 $ 25,400

Salaries payable . . . . . . . . . . . . . . . . . 500 600

Total current liabilities . . . . . . . . . . . . 24,400 26,000

Equity Common stock, no par value . . . . . . . 110,000 100,000

Retained earnings. . . . . . . . . . . . . . . . 23,550 13,050

Total liabilities and equity. . . . . . . . . . $157,950 $139,050

MONTGOMERY INC. Income Statement

For Current Year Ended December 31

Sales . . . . . . . . . . . . . . . . . . . . $45,575 Cost of goods sold . . . . . . . . . (18,950) Gross profit . . . . . . . . . . . . . . . 26,625 Operating expenses Depreciation expense . . . . $7,200 Other expenses . . . . . . . . . 5,550 Total operating expenses . . . 12,750 Income before taxes . . . . . . . 13,875 Income tax expense . . . . . . . . 3,375 Net income . . . . . . . . . . . . . . . $10,500

Additional Information on Current-Year Transactions a. No dividends are declared or paid. b. Issued additional stock for $10,000 cash. c. Purchased equipment for cash; no equipment was sold.

486 Chapter 12 Reporting Cash Flows

A company reported average total assets of $1,240,000 in Year 1 and $1,510,000 in Year 2. Its net operat- ing cash flow was $102,920 in Year 1 and $138,920 in Year 2. (1) Calculate its cash flow on total assets ratio for both years. (2) Did its cash flow on total assets improve in Year 2 versus Year 1?

Exercise 12-13 Analyzing cash flow on total assets A1

Indicate where each item would appear on a statement of cash flows using the direct method by placing an x in the appropriate column.

Exercise 12-15B Direct: Classifying cash flows

C1 P5 Statement of Cash Flows Noncash Not Investing Reported on Operating Investing Financing and Financing Statement Activities Activities Activities Activities or in Notes

a. Retired long-term notes payable by issuing common stock . . . . . . . . . . . b. Paid cash toward accounts payable . . . c. Sold inventory for cash . . . . . . . . . . . . d. Paid cash dividends . . . . . . . . . . . . . . . e. Accepted note receivable in exchange for plant assets . . . . . . . . . . . . . . . . f. Recorded depreciation expense . . . . . g. Paid cash to acquire treasury stock . . . h. Collected cash from sales . . . . . . . . . . i. Borrowed cash from bank by signing a nine-month note payable . . . . . . . j. Paid cash to purchase a patent . . . . . .

Complete the following spreadsheet in preparation of the statement of cash flows. (The statement of cash flows is not required.) Prepare the spreadsheet as in Exhibit 12A.1 under the indirect method. Identify the debits and credits in the Analysis of Changes columns with letters that correspond to the following trans- actions and events a through h. a. Net income for the year was $100,000. b. Dividends of $80,000 cash were declared and paid. c. The only noncash expense was $70,000

of depreciation. d. Purchased plant assets for $70,000 cash.

Exercise 12-14A Indirect: Cash flows spreadsheet

P4

SCORETECK CORPORATION Spreadsheet for Statement of Cash Flows—Indirect Method

For Year Ended December 31, 2019

Dec. 31, 2019Debit Credit Analysis of Changes

Dec. 31, 2018 Balance Sheet—Debit Bal. Accounts Cash Accounts receivable Inventory Plant assets

Balance Sheet—Credit Bal. Accounts Accumulated depreciation Accounts payable Notes payable Common stock Retained earnings

Statement of Cash Flows Operating activities Net income Increase in accounts receivable Decrease in inventory Decrease in accounts payable Depreciation expense Investing activities Cash paid to purchase plant assets Financing activities Cash paid for dividends Cash from issuance of notes

$ 80,000 120,000 250,000 600,000

$1,050,000

$ 100,000 150,000 370,000 200,000 230,000

$1,050,000

$ 60,000 190,000 230,000 670,000

$1,150,000

$ 170,000 140,000 390,000 200,000 250,000

$1,150,000

e. Notes payable of $20,000 were issued for $20,000 cash.

f. $70,000 increase in accounts receivable. g. $20,000 decrease in inventory. h. $10,000 decrease in accounts payable.

Chapter 12 Reporting Cash Flows 487

Refer to the information in Exercise 12-11. Using the direct method, prepare the statement of cash flows for the year ended June 30, 2019.

Exercise 12-17B Direct: Preparing statement of cash flows P5

Refer to information in Exercise 12-4. Use the direct method to prepare the operating activities section of Sonad’s statement of cash flows.

Exercise 12-18B Direct: Cash flows from operating activities P5

For each of the following separate cases, compute the required cash flow information. Exercise 12-16B Direct: Computing cash flows

P5 Case X: Compute cash received from customers Sales. . . . . . . . . . . . . . . . . . . . . . . . . . . . . . . . $515,000

Accounts receivable, Beginning balance. . . . 27,200

Accounts receivable, Ending balance . . . . . 33,600

Case Y: Compute cash paid for rent Rent expense. . . . . . . . . . . . . . . . . . . . . . . . . $139,800

Rent payable, Beginning balance. . . . . . . . . 7,800

Rent payable, Ending balance . . . . . . . . . . . 6,200

Case Z: Compute cash paid for inventory Cost of goods sold . . . . . . . . . . . . . . . . . . $525,000

Inventory, Beginning balance . . . . . . . . . 158,600

Accounts payable, Beginning balance . . . 66,700

Inventory, Ending balance . . . . . . . . . . . . 130,400

Accounts payable, Ending balance . . . . . 82,000

Use the following information about Ferron Company to prepare a complete statement of cash flows (di- rect method) for the current year ended December 31. Use a note disclosure for any noncash investing and financing activities.

Exercise 12-19B Direct: Preparing statement of cash flows and supporting note

P5Cash and cash equivalents, Dec. 31 prior year-end. . . . . . . . . . . . . . . . . . . $ 40,000

Cash and cash equivalents, Dec. 31 current year-end. . . . . . . . . . . . . . . . . 148,000

Cash received as interest. . . . . . . . . . . . . . . . . . . 3,500

Cash paid for salaries. . . . . . . . . . . . . . . . . . . . . . 76,500

Bonds payable retired by issuing common stock (no gain or loss on retirement) . . . . . . . 185,500

Cash paid to retire long-term notes payable . . . 100,000

Cash received from sale of equipment . . . . . . . . 60,250

Cash received in exchange for six-month note payable . . . . . . . . . . . . . . . . . . . . . . . . . . . $ 35,000

Land purchased by issuing long-term note payable . . . . . . . . . . . . . . . . . . . . . . . . . . . 105,250

Cash paid for store equipment . . . . . . . . . . . . . . . 24,750

Cash dividends paid . . . . . . . . . . . . . . . . . . . . . . . 10,000

Cash paid for other expenses. . . . . . . . . . . . . . . . 20,000

Cash received from customers . . . . . . . . . . . . . . . 495,000

Cash paid for inventory . . . . . . . . . . . . . . . . . . . . . 254,500

The following Cash T-account shows the total debits and total credits to the Cash account of Thomas Corporation for the current year. 1. Prepare a complete statement of cash flows for the current year using the direct method. 2. Refer to the statement of cash flows prepared for part 1 to answer the following questions. (a) Which

section—operating, investing, or financing—shows the largest cash (i) inflow and (ii) outflow? (b) What is the largest individual item among the investing cash outflows? (c) Are the cash proceeds larger from issuing notes or issuing stock? (d) Does the company have a net cash inflow or outflow from borrowing activities?

Exercise 12-20B Direct: Preparing statement of cash flows from Cash T-account

P1 P3 P5

Balance, Dec. 31, prior year ........... Receipts from customers ................ Receipts from dividends ................. Receipts from land sale ................... Receipts from machinery sale ....... Receipts from issuing stock ........... Receipts from borrowing ................

Balance, Dec. 31, current year .......... ?

333,000 5,000,000

208,400 220,000 710,000

1,540,000 3,600,000

Payments for inventory ..................... Payments for wages .......................... Payments for rent ............................... Payments for interest ........................ Payments for taxes ............................. Payments for machinery ................... Payments for stock investments .... Payments for note payable .............. Payments for dividends .................... Payments for treasury stock ............

2,590,000 550,000 320,000 218,000 450,000

2,236,000 1,260,000

386,000 500,000 218,000

Cash

488 Chapter 12 Reporting Cash Flows

Required

Prepare the operating activities section of the statement of cash flows using the indirect method for the current year.

Check Cash from operating activities, $17,780

Refer to the information in Problem 12-1A.

Required

Prepare the operating activities section of the statement of cash flows using the direct method for the current year.

Problem 12-2AB Direct: Computing cash flows from operations

P5

PROBLEM SET A

Problem 12-1A Indirect: Computing cash flows from operations

P2

Lansing Company’s current-year income statement and selected balance sheet data at December 31 of the current and prior years follow.

LANSING COMPANY Income Statement

For Current Year Ended December 31

Sales revenue . . . . . . . . . . . . . . . . . . . . $97,200 Expenses Cost of goods sold . . . . . . . . . . . . . . 42,000 Depreciation expense. . . . . . . . . . . . 12,000 Salaries expense. . . . . . . . . . . . . . . . 18,000 Rent expense . . . . . . . . . . . . . . . . . . 9,000 Insurance expense . . . . . . . . . . . . . . 3,800 Interest expense . . . . . . . . . . . . . . . . 3,600 Utilities expense . . . . . . . . . . . . . . . . 2,800 Net income. . . . . . . . . . . . . . . . . . . . . . . $ 6,000

LANSING COMPANY Selected Balance Sheet Accounts

At December 31 Current Year Prior Year Accounts receivable . . . . . . . . $5,600 $5,800 Inventory . . . . . . . . . . . . . . . . . 1,980 1,540 Accounts payable . . . . . . . . . . 4,400 4,600 Salaries payable . . . . . . . . . . . 880 700 Utilities payable . . . . . . . . . . . 220 160 Prepaid insurance. . . . . . . . . . 260 280 Prepaid rent . . . . . . . . . . . . . . 220 180

Forten Company’s current-year income statement, comparative balance sheets, and additional information fol- low. For the year, (1) all sales are credit sales, (2) all credits to Accounts Receivable reflect cash receipts from customers, (3) all purchases of inventory are on credit, (4) all debits to Accounts Payable reflect cash payments for inventory, and (5) Other Expenses are paid in advance and are initially debited to Prepaid Expenses.

Problem 12-3A Indirect: Statement of cash flows

A1 P2 P3

FORTEN COMPANY Comparative Balance Sheets

December 31

Current Year Prior Year Assets Cash . . . . . . . . . . . . . . . . . . . . . . . . . . . . . . . . . . . . $ 49,800 $ 73,500 Accounts receivable . . . . . . . . . . . . . . . . . . . . . . . 65,810 50,625 Inventory . . . . . . . . . . . . . . . . . . . . . . . . . . . . . . . . 275,656 251,800 Prepaid expenses . . . . . . . . . . . . . . . . . . . . . . . . . 1,250 1,875 Total current assets . . . . . . . . . . . . . . . . . . . . . . . . 392,516 377,800 Equipment . . . . . . . . . . . . . . . . . . . . . . . . . . . . . . . 157,500 108,000 Accum. depreciation—Equipment . . . . . . . . . . . . (36,625) (46,000) Total assets . . . . . . . . . . . . . . . . . . . . . . . . . . . . . . $513,391 $439,800

Liabilities and Equity Accounts payable . . . . . . . . . . . . . . . . . . . . . . . . . $ 53,141 $114,675 Short-term notes payable . . . . . . . . . . . . . . . . . . . 10,000 6,000 Total current liabilities . . . . . . . . . . . . . . . . . . . . . . 63,141 120,675 Long-term notes payable . . . . . . . . . . . . . . . . . . . 65,000 48,750 Total liabilities . . . . . . . . . . . . . . . . . . . . . . . . . . . . 128,141 169,425

Equity Common stock, $5 par value . . . . . . . . . . . . . . . . 162,750 150,250 Paid-in capital in excess of par, common stock . . . 37,500 0 Retained earnings . . . . . . . . . . . . . . . . . . . . . . . . . 185,000 120,125 Total liabilities and equity . . . . . . . . . . . . . . . . . . . $513,391 $439,800

FORTEN COMPANY Income Statement

For Current Year Ended December 31

Sales . . . . . . . . . . . . . . . . . . . . . . . . . . . . . . . . $582,500 Cost of goods sold . . . . . . . . . . . . . . . . . . . . . 285,000 Gross profit . . . . . . . . . . . . . . . . . . . . . . . . . . . 297,500 Operating expenses Depreciation expense . . . . . . . . . . . . . . . . $ 20,750 Other expenses . . . . . . . . . . . . . . . . . . . . . 132,400 153,150 Other gains (losses) Loss on sale of equipment. . . . . . . . . . . . . (5,125) Income before taxes. . . . . . . . . . . . . . . . . . . . 139,225 Income taxes expense . . . . . . . . . . . . . . . . . . 24,250 Net income . . . . . . . . . . . . . . . . . . . . . . . . . . . $114,975

Additional Information on Current-Year Transactions a. The loss on the cash sale of equipment was $5,125 (details in b). b. Sold equipment costing $46,875, with accumulated depreciation of

$30,125, for $11,625 cash. c. Purchased equipment costing $96,375 by paying $30,000 cash and

signing a long-term note payable for the balance. d. Borrowed $4,000 cash by signing a short-term note payable. e. Paid $50,125 cash to reduce the long-term notes payable. f. Issued 2,500 shares of common stock for $20 cash per share. g. Declared and paid cash dividends of $50,100.

Chapter 12 Reporting Cash Flows 489

Required

1. Prepare a complete statement of cash flows using the indirect method for the current year. Disclose any noncash investing and financing activities in a note.

Analysis Component

2. Analyze and discuss the statement of cash flows prepared in part 1, giving special attention to the wisdom of the cash dividend payment.

Check Cash from operating activities, $40,900

Refer to Forten Company’s financial statements and related information in Problem 12-3A.

Required

Prepare a complete statement of cash flows using the direct method. Disclose any noncash investing and financing activities in a note.

Problem 12-5AB Direct: Statement of cash flows P5 Check Cash used in financing activities, $(46,225)

Golden Corp.’s current-year income statement, comparative balance sheets, and additional information follow. For the year, (1) all sales are credit sales, (2) all credits to Accounts Receivable reflect cash receipts from customers, (3) all purchases of inventory are on credit, (4) all debits to Accounts Payable reflect cash payments for inventory, (5) Other Expenses are all cash expenses, and (6) any change in Income Taxes Payable reflects the accrual and cash payment of taxes.

Problem 12-6A Indirect: Statement of cash flows

P2 P3

GOLDEN CORPORATION Comparative Balance Sheets

At December 31 Current Year Prior Year

Assets Cash . . . . . . . . . . . . . . . . . . . . . . . . . . . $ 164,000 $107,000 Accounts receivable . . . . . . . . . . . . . . 83,000 71,000 Inventory . . . . . . . . . . . . . . . . . . . . . . . 601,000 526,000 Total current assets . . . . . . . . . . . . . . . 848,000 704,000 Equipment . . . . . . . . . . . . . . . . . . . . . . 335,000 299,000 Accum. depreciation—Equipment . . . (158,000) (104,000) Total assets . . . . . . . . . . . . . . . . . . . . . $1,025,000 $899,000

Liabilities and Equity Accounts payable . . . . . . . . . . . . . . . . $ 87,000 $ 71,000 Income taxes payable . . . . . . . . . . . . . 28,000 25,000 Total current liabilities . . . . . . . . . . . . . 115,000 96,000

Equity Common stock, $2 par value . . . . . . . 592,000 568,000 Paid-in capital in excess of par value, common stock . . . . . . 196,000 160,000 Retained earnings . . . . . . . . . . . . . . . . 122,000 75,000 Total liabilities and equity . . . . . . . . . . $1,025,000 $899,000

GOLDEN CORPORATION Income Statement

For Current Year Ended December 31

Sales . . . . . . . . . . . . . . . . . . . . . $1,792,000 Cost of goods sold . . . . . . . . . . 1,086,000 Gross profit . . . . . . . . . . . . . . . . 706,000 Operating expenses Depreciation expense . . . . . $ 54,000 Other expenses . . . . . . . . . . 494,000 548,000 Income before taxes. . . . . . . . . 158,000 Income taxes expense . . . . . . . 22,000

Net income . . . . . . . . . . . . . . . . $ 136,000

Additional Information on Current-Year Transactions a. Purchased equipment for $36,000 cash. b. Issued 12,000 shares of common stock for $5 cash per

share.

c. Declared and paid $89,000 in cash dividends.

Refer to the information reported about Forten Company in Problem 12-3A.

Required

Prepare a complete statement of cash flows using a spreadsheet as in Exhibit 12A.1 using the indirect method. Identify the debits and credits in the Analysis of Changes columns with letters that correspond to the following list of transactions and events. a. Net income was $114,975. b. Accounts receivable

increased. c. Inventory increased. d. Prepaid expenses decreased. e. Accounts payable decreased. f. Depreciation expense was

$20,750.

Problem 12-4AA Indirect: Cash flows spreadsheet

P4

Check Analysis of Changes column totals, $600,775

g. Sold equipment costing $46,875, with accumulated depreciation of $30,125, for $11,625 cash. This yielded a loss of $5,125.

h. Purchased equipment costing $96,375 by paying $30,000 cash and (i.) by signing a long-term note payable for the balance.

j. Borrowed $4,000 cash by signing a short-term note payable. k. Paid $50,125 cash to reduce the long-term notes payable. l. Issued 2,500 shares of common stock for $20 cash per share. m. Declared and paid cash dividends of $50,100.

490 Chapter 12 Reporting Cash Flows

Refer to the information reported about Golden Corporation in Problem 12-6A.

Required

Prepare a complete statement of cash flows using a spreadsheet as in Exhibit 12A.1 under the indirect method. Identify the debits and credits in the Analysis of Changes columns with letters that correspond to the following list of transactions and events. a. Net income was $136,000. b. Accounts receivable increased. c. Inventory increased. d. Accounts payable increased. e. Income taxes payable increased.

Problem 12-7AA Indirect: Cash flows spreadsheet

P4

f. Depreciation expense was $54,000. g. Purchased equipment for $36,000 cash. h. Issued 12,000 shares at $5 cash per share. i. Declared and paid $89,000 of cash dividends.Check Analysis of Changes

column totals, $481,000

Refer to Golden Corporation’s financial statements and related information in Problem 12-6A.

Required

Prepare a complete statement of cash flows using the direct method for the current year.

Problem 12-8AB Direct: Statement of cash flows P5 Check Cash used in financing activities, $(29,000)

Refer to the information in Problem 12-1B.

Required

Prepare the operating activities section of the statement of cash flows using the direct method for the cur- rent year.

Problem 12-2BB Direct: Computing cash flows from operations

P5

Gazelle Corporation’s current-year income statement, comparative balance sheets, and additional infor- mation follow. For the year, (1) all sales are credit sales, (2) all credits to Accounts Receivable reflect cash receipts from customers, (3) all purchases of inventory are on credit, (4) all debits to Accounts Payable reflect cash payments for inventory, and (5) Other Expenses are paid in advance and are initially debited to Prepaid Expenses.

Problem 12-3B Indirect: Statement of cash flows

A1 P2 P3

PROBLEM SET B

Problem 12-1B Indirect: Computing cash flows from operations

P2

Salt Lake Company’s current-year income statement and selected balance sheet data at December 31 of the current and prior years follow.

SALT LAKE COMPANY Income Statement

For Current Year Ended December 31

Sales revenue . . . . . . . . . . . . . . . . . . . . $156,000

Expenses

Cost of goods sold . . . . . . . . . . . . . . 72,000

Depreciation expense. . . . . . . . . . . . 32,000

Salaries expense. . . . . . . . . . . . . . . . 20,000

Rent expense . . . . . . . . . . . . . . . . . . 5,000

Insurance expense . . . . . . . . . . . . . . 2,600

Interest expense . . . . . . . . . . . . . . . . 2,400

Utilities expense . . . . . . . . . . . . . . . . 2,000

Net income. . . . . . . . . . . . . . . . . . . . . . . $ 20,000

SALT LAKE COMPANY Selected Balance Sheet Accounts

At December 31 Current Year Prior Year

Accounts receivable . . . . . . . . $3,600 $3,000 Inventory . . . . . . . . . . . . . . . . . 860 980 Accounts payable . . . . . . . . . . 2,400 2,600 Salaries payable . . . . . . . . . . . 900 600 Utilities payable . . . . . . . . . . . 200 0 Prepaid insurance. . . . . . . . . . 140 180 Prepaid rent . . . . . . . . . . . . . . 100 200

Required

Prepare the operating activities section of the statement of cash flows using the indirect method for the current year.

Check Cash from operating activities, $51,960

Required

Prepare a complete statement of cash flows using the indirect method for the current year. Check Cash from operating activities, $122,000

Chapter 12 Reporting Cash Flows 491

GAZELLE CORPORATION Comparative Balance Sheets

December 31

Current Year Prior Year Assets

Cash . . . . . . . . . . . . . . . . . . . . . . . . . . . . . $123,450 $ 61,550

Accounts receivable . . . . . . . . . . . . . . . . 77,100 80,750

Inventory . . . . . . . . . . . . . . . . . . . . . . . . . 240,600 250,700

Prepaid expenses . . . . . . . . . . . . . . . . . . 15,100 17,000

Total current assets . . . . . . . . . . . . . . . . . 456,250 410,000

Equipment . . . . . . . . . . . . . . . . . . . . . . . . 262,250 200,000

Accum. depreciation—Equipment . . . . . (110,750) (95,000)

Total assets . . . . . . . . . . . . . . . . . . . . . . . $607,750 $515,000

Liabilities and Equity Accounts payable . . . . . . . . . . . . . . . . . . $ 17,750 $102,000

Short-term notes payable . . . . . . . . . . . . 15,000 10,000

Total current liabilities . . . . . . . . . . . . . . . 32,750 112,000

Long-term notes payable . . . . . . . . . . . . 100,000 77,500

Total liabilities . . . . . . . . . . . . . . . . . . . . . 132,750 189,500

Equity Common stock, $5 par . . . . . . . . . . . . . . 215,000 200,000

Paid-in capital in excess

of par, common stock . . . . . . . . . . . . . 30,000 0

Retained earnings . . . . . . . . . . . . . . . . . . 230,000 125,500

Total liabilities and equity . . . . . . . . . . . . $607,750 $515,000

GAZELLE CORPORATION Income Statement

For Current Year Ended December 31

Sales . . . . . . . . . . . . . . . . . . . . . . . . . $1,185,000

Cost of goods sold . . . . . . . . . . . . . . 595,000

Gross profit . . . . . . . . . . . . . . . . . . . . 590,000

Operating expenses

Depreciation expense . . . . . . . . . $ 38,600

Other expenses . . . . . . . . . . . . . . 362,850

Total operating expenses. . . . . . . . . 401,450

188,550

Other gains (losses)

Loss on sale of equipment. . . . . . (2,100)

Income before taxes. . . . . . . . . . . . . 186,450

Income taxes expense . . . . . . . . . . . 28,350

Net income . . . . . . . . . . . . . . . . . . . . $ 158,100

Additional Information on Current-Year Transactions a. The loss on the cash sale of equipment was $2,100 (details in b). b. Sold equipment costing $51,000, with accumulated

depreciation of $22,850, for $26,050 cash. c. Purchased equipment costing $113,250 by paying $43,250

cash and signing a long-term note payable for the balance. d. Borrowed $5,000 cash by signing a short-term note payable. e. Paid $47,500 cash to reduce the long-term notes payable. f. Issued 3,000 shares of common stock for $15 cash per share. g. Declared and paid cash dividends of $53,600.

Required

1. Prepare a complete statement of cash flows using the indirect method for the current year. Disclose any noncash investing and financing activities in a note.

Analysis Component

2. Analyze and discuss the statement of cash flows prepared in part 1, giving special attention to the wisdom of the cash dividend payment.

Check Cash from operating activities, $130,200

Refer to the information reported about Gazelle Corporation in Problem 12-3B.

Required

Prepare a complete statement of cash flows using a spreadsheet as in Exhibit 12A.1 using the indirect method. Identify the debits and credits in the Analysis of Changes columns with letters that correspond to the following list of transactions and events. a. Net income was $158,100. b. Accounts receivable decreased. c. Inventory decreased. d. Prepaid expenses decreased. e. Accounts payable decreased. f. Depreciation expense was $38,600. g. Sold equipment costing $51,000, with accumulated depreciation of $22,850, for $26,050 cash. This

yielded a loss of $2,100. h. Purchased equipment costing $113,250 by paying $43,250 cash and (i.) by signing a long-term note

payable for the balance. j. Borrowed $5,000 cash by signing a short-term note payable. k. Paid $47,500 cash to reduce the long-term notes payable. l. Issued 3,000 shares of common stock for $15 cash per share. m. Declared and paid cash dividends of $53,600.

Problem 12-4BA Indirect: Cash flows spreadsheet

P4

Check Analysis of Changes column totals, $681,950

492 Chapter 12 Reporting Cash Flows

Refer to Gazelle Corporation’s financial statements and related information in Problem 12-3B.

Required

Prepare a complete statement of cash flows using the direct method. Disclose any noncash investing and financing activities in a note.

Problem 12-5BB Direct: Statement of cash flows P5 Check Cash used in financing activities, $(51,100)

Satu Company’s current-year income statement, comparative balance sheets, and additional information follow. For the year, (1) all sales are credit sales, (2) all credits to Accounts Receivable reflect cash receipts from customers, (3) all purchases of inventory are on credit, (4) all debits to Accounts Payable reflect cash payments for inventory, (5) Other Expenses are cash expenses, and (6) any change in Income Taxes Payable reflects the accrual and cash payment of taxes.

Problem 12-6B Indirect: Statement of cash flows

P2 P3

SATU COMPANY Comparative Balance Sheets

At December 31 Current Year Prior Year

Assets Cash . . . . . . . . . . . . . . . . . . . . . . . . . . . . . $ 58,750 $ 28,400 Accounts receivable . . . . . . . . . . . . . . . . 20,222 25,860 Total current assets . . . . . . . . . . . . . . . . . 78,972 54,260 Inventory . . . . . . . . . . . . . . . . . . . . . . . . . 165,667 140,320 Equipment . . . . . . . . . . . . . . . . . . . . . . . . 107,750 77,500 Accum. depreciation—Equipment . . . . . (46,700) (31,000) Total assets . . . . . . . . . . . . . . . . . . . . . . . $305,689 $241,080

Liabilities and Equity Accounts payable . . . . . . . . . . . . . . . . . . $ 20,372 $157,530 Income taxes payable . . . . . . . . . . . . . . . 2,100 6,100 Total current liabilities . . . . . . . . . . . . . . . 22,472 163,630

Equity Common stock, $5 par value . . . . . . . . . 40,000 25,000 Paid-in capital in excess of par, common stock . . . . . . . . . . . . . 68,000 20,000 Retained earnings . . . . . . . . . . . . . . . . . . 175,217 32,450 Total liabilities and equity . . . . . . . . . . . . $305,689 $241,080

SATU COMPANY Income Statement

For Current Year Ended December 31

Sales . . . . . . . . . . . . . . . . . . . . . $750,800

Cost of goods sold . . . . . . . . . . 269,200

Gross profit . . . . . . . . . . . . . . . . 481,600

Operating expenses

Depreciation expense . . . . . $ 15,700

Other expenses . . . . . . . . . . 173,933 189,633

Income before taxes. . . . . . . . . 291,967

Income taxes expense . . . . . . . 89,200

Net income . . . . . . . . . . . . . . . . $202,767

Additional Information on Current-Year Transactions a. Purchased equipment for $30,250 cash. b. Issued 3,000 shares of common stock for $21 cash per

share.

c. Declared and paid $60,000 of cash dividends.

Required

Prepare a complete statement of cash flows using the indirect method for the current year. Check Cash from operating activities, $57,600

Refer to the information reported about Satu Company in Problem 12-6B.

Required

Prepare a complete statement of cash flows using a spreadsheet as in Exhibit 12A.1 under the indirect method. Identify the debits and credits in the Analysis of Changes columns with letters that correspond to the following list of transactions and events. a. Net income was $202,767. b. Accounts receivable decreased. c. Inventory increased. d. Accounts payable decreased. e. Income taxes payable decreased.

Problem 12-7BA Indirect: Cash flows spreadsheet

P4

f. Depreciation expense was $15,700. g. Purchased equipment for $30,250 cash. h. Issued 3,000 shares at $21 cash per share. i. Declared and paid $60,000 of cash dividends.

Check Analysis of Changes column totals, $543,860

Refer to Satu Company’s financial statements and related information in Problem 12-6B.

Required

Prepare a complete statement of cash flows using the direct method for the current year.

Problem 12-8BB Direct: Statement of cash flows P5 Check Cash provided by financing activities, $3,000

Chapter 12 Reporting Cash Flows 493

SERIAL PROBLEM Business Solutions (Indirect)

P2 P3

This serial problem began in Chapter 1 and continues through most of the book. If previous chapter seg- ments were not completed, the serial problem can begin at this point.

SP 12 Santana Rey, owner of Business Solutions, decides to prepare a statement of cash flows for her business. (Although the serial problem allowed for various ownership changes in earlier chapters, we will prepare the statement of cash flows using the following financial data.)

©Alexander Image/Shutterstock

BUSINESS SOLUTIONS Comparative Balance Sheets

December 31, 2019, and March 31, 2020

Mar. 31, 2020 Dec. 31, 2019

Assets Cash . . . . . . . . . . . . . . . . . . . . . . . . . . . . . . . . . . $ 68,057 $48,372 Accounts receivable . . . . . . . . . . . . . . . . . . . . . . 22,867 5,668 Inventory . . . . . . . . . . . . . . . . . . . . . . . . . . . . . . . 704 0 Computer supplies . . . . . . . . . . . . . . . . . . . . . . . 2,005 580 Prepaid insurance. . . . . . . . . . . . . . . . . . . . . . . . 1,110 1,665 Prepaid rent . . . . . . . . . . . . . . . . . . . . . . . . . . . . 825 825 Total current assets . . . . . . . . . . . . . . . . . . . . . . 95,568 57,110 Office equipment . . . . . . . . . . . . . . . . . . . . . . . . 8,000 8,000 Accumulated depreciation—Office equipment . . . . . . . . . . . . . . . . . . . . . . . . . . . (800) (400) Computer equipment . . . . . . . . . . . . . . . . . . . . . 20,000 20,000 Accumulated depreciation— Computer equipment. . . . . . . . . . . . . . . . . . . (2,500) (1,250) Total assets . . . . . . . . . . . . . . . . . . . . . . . . . . . . . $120,268 $83,460

Liabilities and Equity Accounts payable . . . . . . . . . . . . . . . . . . . . . . . . $ 0 $ 1,100 Wages payable . . . . . . . . . . . . . . . . . . . . . . . . . . 875 500 Unearned computer service revenue . . . . . . . . 0 1,500 Total current liabilities . . . . . . . . . . . . . . . . . . . . 875 3,100

Equity Common stock . . . . . . . . . . . . . . . . . . . . . . . . . . 98,000 73,000 Retained earnings. . . . . . . . . . . . . . . . . . . . . . . . 21,393 7,360 Total liabilities and equity. . . . . . . . . . . . . . . . . . $120,268 $83,460

BUSINESS SOLUTIONS Income Statement

For Three Months Ended March 31, 2020

Computer services revenue . . . . . . . . . . . . . . $25,307 Net sales . . . . . . . . . . . . . . . . . . . . . . . . . . . . . 18,693 Total revenue . . . . . . . . . . . . . . . . . . . . . . . . . 44,000 Cost of goods sold . . . . . . . . . . . . . . . . . . . . . $14,052 Depreciation expense—Office equipment . . . 400 Depreciation expense— Computer equipment. . . . . . . . . . . . . . . . . 1,250 Wages expense. . . . . . . . . . . . . . . . . . . . . . . . 3,250 Insurance expense . . . . . . . . . . . . . . . . . . . . . 555 Rent expense . . . . . . . . . . . . . . . . . . . . . . . . . 2,475 Computer supplies expense . . . . . . . . . . . . . 1,305 Advertising expense. . . . . . . . . . . . . . . . . . . . 600 Mileage expense . . . . . . . . . . . . . . . . . . . . . . 320 Repairs expense—Computer . . . . . . . . . . . . . 960 Total expenses . . . . . . . . . . . . . . . . . . . . . . . . 25,167 Net income . . . . . . . . . . . . . . . . . . . . . . . . . . . $18,833

Required

Prepare a statement of cash flows for Business Solutions using the indirect method for the three months ended March 31, 2020. Recall that owner Santana Rey contributed $25,000 to the business in exchange for additional stock in the first quarter of 2020 and has received $4,800 in cash dividends.

Check Cash flows used by operations: $(515)

COMPANY ANALYSIS A1

Accounting Analysis

AA 12-1 Use Apple’s financial statements in Appendix A to answer the following. 1. Is Apple’s statement of cash flows prepared under the direct method or the indirect method? 2. For each fiscal year 2017, 2016, and 2015, identify the amount of cash provided by operating activities

and cash paid for dividends. 3. In 2017, did Apple have sufficient cash flows from operations to pay dividends? 4. Did Apple spend more or less cash to repurchase common stock in 2017 versus 2016?

GENERAL LEDGER PROBLEM

The following General Ledger assignments highlight the impact, or lack thereof, on the statement of cash flows from summary journal entries derived from consecutive trial balances. Prepare summary journal entries reflecting changes in consecutive trial balances. Then prepare the statement of cash flows (direct method) from those entries. Finally, prepare the reconciliation to the indirect method for net cash provided (used) by operating activities.

GL 12-1 General Ledger assignment based on Exercise 12-11

GL 12-2 General Ledger assignment based on Problem 12-1

GL

GL 12-3 General Ledger assign- ment based on Problem 12-6

APPLE

494 Chapter 12 Reporting Cash Flows

ETHICS CHALLENGE C1 A1

BTN 12-1 Katie Murphy is preparing for a meeting with her banker. Her business is finishing its fourth year of operations. In the first year, it had negative cash flows from operations. In the second and third years, cash flows from operations were positive. However, inventory costs rose significantly in Year 4, and cash flows from operations will probably be down 25%. Murphy wants to secure a line of credit from her banker as a financing buffer. From experience, she knows the banker will scrutinize operating cash flows for Years 1 through 4 and will want a projected number for Year 5. Murphy knows that a steady progression upward in operating cash flows for Years 1 through 4 will help her case. She decides to use her discretion as owner and considers several business actions that will turn her operating cash flow in Year 4 from a decrease to an increase.

Required

1. Identify two business actions Murphy might take to improve cash flows from operations. 2. Comment on the ethics and possible consequences of Murphy’s decision to pursue these actions.

Beyond the Numbers

Required

1. Compute the recent two years’ cash flow on total assets ratios for Apple and Google. 2. For the current year, which company has the better cash flow on total assets ratio? 3. For the current year, does cash flow on total assets outperform or underperform the industry (as-

sumed) average of 15% for (a) Apple and (b) Google?

AA 12-2 Key figures for Apple and Google follow.

Apple Google

$ millions Current Year 1 Year Prior 2 Years Prior Current Year 1 Year Prior 2 Years Prior

Operating cash flows . . . . . . . . . $ 63,598 $ 65,824 $ 81,266 $ 37,091 $ 36,036 $ 26,572

Total assets . . . . . . . . . . . . . . . . . 375,319 321,686 290,345 197,295 167,497 147,461

COMPARATIVE ANALYSIS A1

APPLE GOOGLE

BTN 12-2 Your friend, Diana Wood, recently completed the second year of her business and just received annual financial statements from her accountant. Wood finds the income statement and balance sheet informative but does not understand the statement of cash flows. She says the first section is especially confusing because it contains a lot of additions and subtractions that do not make sense to her. Wood adds, “The income statement tells me the business is more profitable than last year and that’s most important. If I want to know how cash changes, I can look at comparative balance sheets.”

Required

Write a half-page memorandum to your friend explaining the purpose of the statement of cash flows. Speculate as to why the first section is so confusing and how it might be rectified.

COMMUNICATING IN PRACTICE C1

BTN 12-3 Access the April 14, 2016, filing of the 10-K report (for year ending December 31, 2015) of Mendocino Brewing Company, Inc. (ticker: MENB) at SEC.gov.

Required

1. Does Mendocino Brewing use the direct or indirect method to construct its consolidated statement of cash flows?

TAKING IT TO THE NET A1

GLOBAL ANALYSIS C1

AA 12-3 Key comparative information for Samsung, Apple, and Google follows.

Required

1. Compute the recent two years’ cash flow on total assets ratio for Samsung. 2. Is the change in Samsung’s cash flow on total assets ratio favorable or unfavorable? 3. For the current year, is Samsung’s cash flow on total assets ratio better or worse than (a) Apple’s and

(b) Google’s?

Samsung APPLE GOOGLE

Samsung Apple Google

Current 1 Year 2 Years Current 1 Year Current 1 Year In millions Year Prior Prior Year Prior Year Prior

Operating cash flows . . . . . W 62,162,041 W 47,385,644 W 40,061,761 $ 63,598 $ 65,824 $ 37,091 $ 36,036

Total assets . . . . . . . . . . . . . 301,752,090 262,174,324 242,179,521 375,319 321,686 197,295 167,497

Chapter 12 Reporting Cash Flows 495

BTN 12-4 Team members are to coordinate and independently answer one question within each of the follow- ing three sections. Team members should then report to the team and confirm or correct teammates’ answers. 1. Answer one of the following questions about the statement of cash flows: (a) What are this statement’s

reporting objectives? (b) What two methods are used to prepare it? Identify similarities and differ- ences between them. (c) What steps are followed to prepare the statement? (d) What types of analyses are often made from this statement’s information?

2. Identify and explain the adjustment from net income to obtain cash flows from operating activities using the indirect method for one of the following items: (a) Noncash operating revenues and expenses. (b) Nonoperating gains and losses. (c) Increases and decreases in noncash current assets. (d) Increases and decreases in current liabilities.

3.B Identify and explain the formula for computing cash flows from operating activities using the direct method for one of the following items: (a) Cash receipts from sales to customers. (b) Cash paid for inventory. (c) Cash paid for wages and operating expenses. (d) Cash paid for interest and taxes.

TEAMWORK IN ACTION C1 A1 P2 P5

Note: For teams of more than four, some pairing within teams is nec- essary. Use as an in-class activity or as an assignment. If used in class, specify a time limit on each part. Conclude with reports to the entire class, using team rotation. Each team can prepare responses on a transparency.

BTN 12-5 Review the chapter’s opener involving Vera Bradley and its founder, Barbara Bradley.

Required

1. In a business such as Vera Bradley, monitoring cash flow is always a priority. Explain how cash flow can lag behind net income.

2. What are potential sources of financing for Vera Bradley’s future expansion?

ENTREPRENEURIAL DECISION C1 A1

BTN 12-7 Visit The Motley Fool’s web page on cash flow based valuation (Fool.com/how-to-invest/ how-to-value-stocks-cash-flow-based-valuations.aspx).

Required

1. How does the Motley Fool define cash flow? What is the reasoning for this definition? 2. Per the Fool’s instruction, why do analysts focus on earnings before interest and taxes (EBIT)? 3. Visit other links at this website that interest you such as “How to Read a Balance Sheet,” or find out

what the “Fool’s Ratio” is. Write a half-page report on what you find.

HITTING THE ROAD C1

BTN 12-6 Jenna and Matt Wilder are completing their second year operating Mountain High, a downhill ski area and resort. Mountain High reports a net loss of $(10,000) for its second year, which includes an $85,000 unusual loss from fire. This past year also involved major purchases of plant assets for renovation and expansion, yielding a year-end total asset amount of $800,000. Mountain High’s net cash outflow for its second year is $(5,000); a summarized version of its statement of cash flows follows.

ENTREPRENEURIAL DECISION C1 A1

Required

Write a one-page memorandum to the Wilders evaluating Mountain High’s current performance and as- sessing its future. Give special emphasis to cash flow data and their interpretation.

Net cash flow provided by operating activities . . . . . . . . . . . . . . $ 295,000 Net cash flow used by investing activities . . . . . . . . . . . . . . . . . . (310,000) Net cash flow provided by financing activities . . . . . . . . . . . . . . 10,000

Design elements: Lightbulb: ©Chuhail/Getty Images; Blue globe: ©nidwlw/Getty Images and ©Dizzle52/Getty Images; Chess piece: ©Andrei Simonenko/ Getty Images and ©Dizzle52/Getty Images; Mouse: ©Siede Preis/Getty Images; Global View globe: ©McGraw-Hill Education and ©Dizzle52/Getty Images; Sustainability: ©McGraw-Hill Education and ©Dizzle52/Getty Images

2. For the year ended December 31, 2015, what is the largest item in reconciling the net income (or loss) to net cash provided by operating activities?

3. In the recent two years, has the company been more successful in generating operating cash flows or in generating net income? Identify the figures to support the answer.

4. In the year ended December 31, 2015, what was the largest cash outflow for investing activities and for financing activities?

5. What item(s) does the company report as supplemental cash flow information? 6. Does the company report any noncash financing activities for 2015? Identify them, if any.

Learning Objectives

CONCEPTUAL C1 Explain the purpose and identify the

building blocks of analysis.

C2 Describe standards for comparisons in analysis.

PROCEDURAL P1 Explain and apply methods of horizontal

analysis.

P2 Describe and apply methods of vertical analysis.

P3 Define and apply ratio analysis.

ANALYTICAL A1 Summarize and report results of analysis.

A2 Appendix 13A—Explain the form and assess the content of a complete income statement.

Chapter Preview

13 Analysis of Financial Statements

HORIZONTAL ANALYSIS

P1 Application of: Comparative balance sheets

Comparative income statements

Trend analysis

VERTICAL ANALYSIS

P2 Application of: Common-size balance sheet

Common-size income statement

Common-size graphics

BASICS OF ANALYSIS

C1 Analysis purpose Building blocks

C2 Standards for comparisons

Analysis tools

RATIO ANALYSIS AND REPORTING

P3 Liquidity and efficiency

Solvency

Profitability

Market prospects

A1 Analysis reports

NTK 13-3NTK 13-1 NTK 13-2

497

“Expect to win!”—Carla Harris

Numbers Rule

NEW YORK—“I grew up as an only child in a no-nonsense, no- excuses household,” recalls Carla Harris. “My parents gave me the sense that I was supposed to do well.” Fast-forward and Carla is now vice chair of Morgan Stanley’s (MorganStanley.com) prized Global Wealth Management division and past-chair of the Morgan Stanley Foundation.

Carla Harris and her colleagues at Morgan Stanley analyze financial statements for profit. One of Morgan Stanley’s key tools for analysis is ModelWare. ModelWare is a framework to analyze the nuts and bolts of companies’ financial statements and then to compare those companies head-to-head. One of its key aims is to provide comparable information that focuses on sustainable performance.

Morgan Stanley uses the accounting numbers in financial statements to produce comparable metrics using techniques such as horizontal and vertical analysis. It also computes finan- cial ratios for analysis and interpretation. Those ratios include return on equity, return on assets, asset turnover, profit margin, price-to-earnings, and many other accounting measures. The focus is to uncover the drivers of profitability and to predict future levels of those drivers.

Carla has experienced much success through analyzing financial statements. As Carla likes to say, “I’m tough and

analytical!” She says that people do not take full advantage of information available in financial statements.

Carla plays by the rules and asserts that those with account- ing know-how continue to earn profits from financial statement analysis and interpretation. Carla is proud of her success and adds: “Always start from a place of doing the right thing.”

Sources: Morgan Stanley website, January 2019; MorganStanleyIQ, November 2007; Alumni.HBS.edu/Stories, September 2006; Fortune, August 2013 and March 2016

©Jonathan Leibson/AOL/Getty Images

Financial statement analysis applies analytical tools to financial statements and related data for making business decisions.

Purpose of Analysis Internal users of accounting information manage and operate the company. They include man- agers, officers, and internal auditors. The purpose of financial statement analysis for internal users is to provide information to improve efficiency and effectiveness.

External users of accounting information are not directly involved in running the company. External users use financial statement analysis to pursue their own goals. Shareholders and creditors assess company performance to make investing and lending decisions. A board of directors analyzes financial statements to monitor management’s performance. External audi- tors use financial statements to assess “fair presentation” of financial results.

The common goal of these users is to evaluate company performance and financial condition. This includes evaluating past and current performance, current financial position, and future performance and risk.

Building Blocks of Analysis Financial statement analysis focuses on one or more of the four building blocks of financial statement analysis. The four building blocks cover different, but interrelated, aspects of financial condition or performance. Liquidity and efficiency—ability to meet short-term obligations and to efficiently

generate revenues. Solvency—ability to meet long-term obligations and generate future revenues. Profitability—ability to provide financial rewards to attract and retain financing. Market prospects—ability to generate positive market expectations.

Point: Financial statement analysis is a topic on the CPA, CMA, CIA, and CFA exams.

BASICS OF ANALYSIS C1 Explain the purpose and identify the building blocks of analysis.

Solvency Market prospects

Liquidity &

efficiency

Profitability

498 Chapter 13 Analysis of Financial Statements

Information for Analysis Financial analysis uses general-purpose financial statements that include the (1) income statement, (2) balance sheet, (3) statement of stockholders’ equity (or statement of retained earnings), (4) statement of cash flows, and (5) notes to these statements.

Financial reporting is the communication of financial information useful for making invest- ment, credit, and other business decisions. Financial reporting includes general-purpose finan- cial statements, information from SEC 10-K and other filings, press releases, shareholders’ meetings, forecasts, management letters, and auditors’ reports.

Management’s Discussion and Analysis (MD&A) is one example of useful information out- side usual financial statements. Apple’s MD&A (available at Investor.Apple.com and “Item 7” in the annual report) begins with an overview, followed by critical accounting policies and esti- mates. It then discusses operating results followed by financial condition (liquidity, capital resources, and cash flows). The final few parts discuss risks. The MD&A is an excellent starting point in understanding a company’s business.

Standards for Comparisons When analyzing financial statements, we use the following standards (benchmarks) for com- parisons. Benchmarks from a competitor or group of competitors are often best. Intracompany and industry measures are also good. Guidelines can be applied, but only if they seem reason- able given recent experience. Intracompany—The company’s current performance is compared to its prior performance

and its relations between financial items. Apple’s current net income, for example, can be compared with its prior years’ net income and in relation to its revenues or total assets.

Competitor—Competitors provide standards for comparisons. Coca-Cola’s profit margin can be compared with PepsiCo’s profit margin.

Industry—Industry statistics provide standards of comparisons. Intel’s profit margin can be compared with the industry’s profit margin.

Guidelines (rules of thumb)—Standards of comparison can develop from experience. Examples are the 2:1 level for the current ratio or 1:1 level for the acid-test ratio.

Tools of Analysis There are three common tools of financial statement analysis. This chapter describes these anal- ysis tools and how to apply them.

1. Horizontal analysis—comparison of financial condition and performance across time. 2. Vertical analysis—comparison of financial condition and performance to a base amount. 3. Ratio analysis—measurement of key relations between financial statement items.

C2 Describe standards for comparisons in analysis.

Point: Each chapter’s Accounting Analysis problems cover intra- company analysis. Comparative Analysis problems cover competi- tor analysis (Apple vs. Google vs. Samsung).

Income Statement

Balance Sheet

Statement of Stockholders' Equity

Statement ofCash Flows

Notes

Income Statement

Balance Sheet

Statement of Stockholders’ Equity

Statement ofCash Flows

Notes

Stock in Trade Blue chips are stocks of big, established companies. The phrase comes from poker, where the most valuable chips are blue. Brokers execute orders to buy or sell stock. The term comes from wine retailers—individuals who broach (break) wine casks. ■

Decision Insight

Horizontal analysis is the review of financial statement data across time. Horizontal comes from the left-to-right (or right-to-left) movement of our eyes as we review comparative financial statements across time.

Comparative Statements Comparative financial statements show financial amounts in side-by-side col- umns on a single statement, called a comparative format. Using Apple’s financial statements, this section explains how to compute dollar changes and percent changes for comparative statements.

HORIZONTAL ANALYSIS P1 Explain and apply methods of horizontal analysis.

2018 Report 2019 Report

Chapter 13 Analysis of Financial Statements 499

Dollar Changes and Percent Changes Comparing financial statements is often done by analyzing dollar amount changes and percent changes in line items. Both analyses are relevant because small dollar changes can yield large percent changes inconsistent with their im- portance. A 50% change from a base figure of $100 is less important than a 50% change from a base amount of $100,000. We compute the dollar change for a financial statement item as follows.

Dollar change = Analysis period amount − Base period amount

Analysis period refers to the financial statements under analysis, and base period refers to the financial statements used for comparison. The prior year is commonly used as a base period. We compute the percent change as follows.

Percent change (%) = Analysis period amount − Base period amount

Base period amount × 100

We must know a few rules in working with percent changes. Let’s look at four separate cases. Cases A and B: When a negative amount is in one period and a positive amount is in the

other, we cannot compute a meaningful percent change. Case C: When no amount is in the base period, no percent change is computable. Case D: When a positive amount is in the base period and zero is in the analysis period, the

decrease is 100%.

Example: When there is a value in the base period and zero in the analysis period, the decrease is 100%. Why isn’t the reverse situation an increase of 100%? Answer: A 100% increase of zero is still zero.

Change Analysis Analysis Base

Case Period Period Dollar Percent

A $ 1,500 $(4,500) $ 6,000 — B (1,000) 2,000 (3,000) — C 8,000 — 8,000 — D 0 10,000 (10,000) (100%)

Comparative Balance Sheets Analysis of comparative financial statements begins by focusing on large dollar and percent changes. We then identify the reasons and implications for these changes. We also review small changes when we expected large changes.

Exhibit 13.1 shows comparative balance sheets for Apple Inc. (ticker: AAPL). A few items stand out on the asset side. Apple’s short-term marketable securities increased by 15.5%, and its long-term marketable securities increased by 14.2%. This combined for a large $31,505 million increase in securities. In response, Apple raised its dividend and announced plans to spend at least $210 billion buying back stock by the end of the next year. Dividends and share repurchase plans are likely to slow Apple’s growth of short-term securities. Other notable increases occur with (1) property, plant and equipment, partially related to its new headquarters, and (2) inventories, which had a high percentage increase but relatively small dollar increase.

On Apple’s financing side, we see its overall 16.7% increase is driven by a 24.7% increase in liabilities; equity increased only 4.5%. The largest increase is from long-term debt, which increased by $21,780 million, or 28.9%. Much of this increase results from bond offerings by Apple to take advantage of low interest rates. We also see a modest increase of 2.0% ($1,966 million) in retained earnings, which was increased by a strong income of $48,351 million and reduced by cash dividends and stock repurchases.

Comparative Income Statements Exhibit 13.2 shows Apple’s comparative in- come statements. Apple reports an increase in sales of 6.3%. Cost of sales increased to a greater extent than sales (7.4%), which is not a positive sign. The 10.7% increase in operating expenses is primarily driven by the 15.3% increase in research and development costs, from which management and investors hope to reap future income. While Apple’s net income increased just 5.8%, its basic earnings per share increased 11.0%. This is largely due to Apple’s share buyback program.

500 Chapter 13 Analysis of Financial Statements

APPLE INC. Comparative Year-End Balance Sheets

$ millions Current Yr Prior Yr Dollar Change Percent Change

Assets Cash and cash equivalents . . . . . . . . . . . . . . . . . . . . . . . . $ 20,289 $ 20,484 $ (195) (1.0)% Short-term marketable securities . . . . . . . . . . . . . . . . . . . 53,892 46,671 7,221 15.5 Accounts receivable, net . . . . . . . . . . . . . . . . . . . . . . . . . . 17,874 15,754 2,120 13.5 Inventories . . . . . . . . . . . . . . . . . . . . . . . . . . . . . . . . . . . . . 4,855 2,132 2,723 127.7 Vendor non-trade receivables . . . . . . . . . . . . . . . . . . . . . . 17,799 13,545 4,254 31.4 Other current assets . . . . . . . . . . . . . . . . . . . . . . . . . . . . . . 13,936 8,283 5,653 68.2 Total current assets . . . . . . . . . . . . . . . . . . . . . . . . . . . . 128,645 106,869 21,776 20.4 Long-term marketable securities . . . . . . . . . . . . . . . . . . . . 194,714 170,430 24,284 14.2 Property, plant and equipment, net . . . . . . . . . . . . . . . . . . 33,783 27,010 6,773 25.1 Goodwill . . . . . . . . . . . . . . . . . . . . . . . . . . . . . . . . . . . . . . . 5,717 5,414 303 5.6 Acquired intangible assets, net . . . . . . . . . . . . . . . . . . . . . 2,298 3,206 (908) (28.3) Other non-current assets . . . . . . . . . . . . . . . . . . . . . . . . . . 10,162 8,757 1,405 16.0 Total assets . . . . . . . . . . . . . . . . . . . . . . . . . . . . . . . . . . . . . $375,319 $321,686 $53,633 16.7

Liabilities Accounts payable . . . . . . . . . . . . . . . . . . . . . . . . . . . . . . . . $ 49,049 $ 37,294 $11,755 31.5% Accrued expenses . . . . . . . . . . . . . . . . . . . . . . . . . . . . . . . 25,744 22,027 3,717 16.9 Deferred revenue . . . . . . . . . . . . . . . . . . . . . . . . . . . . . . . . 7,548 8,080 (532) (6.6) Commercial paper . . . . . . . . . . . . . . . . . . . . . . . . . . . . . . . 11,977 8,105 3,872 47.8 Current portion of long-term debt . . . . . . . . . . . . . . . . . . . 6,496 3,500 2,996 85.6 Total current liabilities . . . . . . . . . . . . . . . . . . . . . . . . . . 100,814 79,006 21,808 27.6 Deferred revenue—non-current . . . . . . . . . . . . . . . . . . . . . 2,836 2,930 (94) (3.2) Long-term debt . . . . . . . . . . . . . . . . . . . . . . . . . . . . . . . . . . 97,207 75,427 21,780 28.9 Other non-current liabilities . . . . . . . . . . . . . . . . . . . . . . . . 40,415 36,074 4,341 12.0 Total liabilities . . . . . . . . . . . . . . . . . . . . . . . . . . . . . . . . . . . 241,272 193,437 47,835 24.7

Stockholders’ Equity Common stock . . . . . . . . . . . . . . . . . . . . . . . . . . . . . . . . . . 35,867 31,251 4,616 14.8 Retained earnings . . . . . . . . . . . . . . . . . . . . . . . . . . . . . . . . 98,330 96,364 1,966 2.0 Accumulated other comprehensive income . . . . . . . . . . . (150) 634 (784) — Total stockholders’ equity . . . . . . . . . . . . . . . . . . . . . . . . . 134,047 128,249 5,798 4.5 Total liabilities and stockholders’ equity . . . . . . . . . . . . . . $375,319 $321,686 $53,633 16.7

EXHIBIT 13.1 Comparative Balance Sheets

APPLE

APPLE INC. Comparative Income Statements

$ millions, except per share Current Yr Prior Yr Dollar Change Percent Change

Net sales . . . . . . . . . . . . . . . . . . . . . . . . . . . . . . . . . . . . . . . $229,234 $215,639 $13,595 6.3% Cost of sales . . . . . . . . . . . . . . . . . . . . . . . . . . . . . . . . . . . . 141,048 131,376 9,672 7.4 Gross margin . . . . . . . . . . . . . . . . . . . . . . . . . . . . . . . . . . . . 88,186 84,263 3,923 4.7 Research and development . . . . . . . . . . . . . . . . . . . . . . . . 11,581 10,045 1,536 15.3 Selling, general and administrative . . . . . . . . . . . . . . . . . . 15,261 14,194 1,067 7.5 Total operating expenses . . . . . . . . . . . . . . . . . . . . . . . . . . 26,842 24,239 2,603 10.7 Operating income . . . . . . . . . . . . . . . . . . . . . . . . . . . . . . . . 61,344 60,024 1,320 2.2 Other income, net . . . . . . . . . . . . . . . . . . . . . . . . . . . . . . . . 2,745 1,348 1,397 103.6 Income before provision for income taxes . . . . . . . . . . . . 64,089 61,372 2,717 4.4 Provision for income taxes . . . . . . . . . . . . . . . . . . . . . . . . . 15,738 15,685 53 0.3 Net income . . . . . . . . . . . . . . . . . . . . . . . . . . . . . . . . . . . . . $ 48,351 $ 45,687 2,664 5.8 Basic earnings per share . . . . . . . . . . . . . . . . . . . . . . . . . . $ 9 .27 $ 8 .35 $ 0.92 11.0 Diluted earnings per share . . . . . . . . . . . . . . . . . . . . . . . . . $ 9 .21 $ 8 .31 $ 0.90 10.8

EXHIBIT 13.2 Comparative Income Statements

APPLE

Point: Percent change is also computed by dividing the current period by the prior period and then subtracting 1.0.

Chapter 13 Analysis of Financial Statements 501

Trend Analysis Trend analysis is computing trend percents that show patterns in data across periods. Trend percent is computed as follows.

$0 2017 2007 2014 2005

$100

Millions Rati

$200 $300 $400 $500 $600 $700

$900

15%

0.0%

30%

45%

$800

Financial Results

400 600 1,200

Point: Index refers to the compari- son of the analysis period to the base period. Percents determined for each period are called index numbers.

Trend percent (%) = Analysis period amount

Base period amount × 100

Trend analysis is shown in Exhibit 13.3 using data from Apple’s current and prior financial statements.

$ millions Current Yr 1 Yr Ago 2 Yrs Ago 3 Yrs Ago 4 Yrs Ago

Net sales . . . . . . . . . . . . . . . . . . . . . $229,234 $215,639 $233,715 $182,795 $170,910

Cost of sales . . . . . . . . . . . . . . . . . . 141,048 131,376 140,089 112,258 106,606

Operating expenses . . . . . . . . . . . . 26,842 24,239 22,396 18,034 15,305

EXHIBIT 13.3 Sales and Expenses

The trend percents—using data from Exhibit 13.3—are shown in Exhibit 13.4. The base period is the number reported four years ago, and the trend percent is computed for each year by divid- ing that year’s amount by the base period amount. For example, the net sales trend percent for the current year is 134.1%, computed as $229,234/$170,910.

In trend percent Current Yr 1 Yr Ago 2 Yrs Ago 3 Yrs Ago 4 Yrs Ago

Net sales . . . . . . . . . . . . . . . . . . . . . 134 .1% 126 .2% 136 .7% 107 .0% 100 .0%

Cost of sales . . . . . . . . . . . . . . . . . . 132 .3 123 .2 131 .4 105 .3 100 .0

Operating expenses . . . . . . . . . . . . 175 .4 158 .4 146 .3 117 .8 100 .0

EXHIBIT 13.4 Trend Percents for Sales and Expenses

Exhibit 13.5 shows the trend percents from Exhibit 13.4 in a line graph, which helps us see trends and detect changes in direction or magnitude. It shows that the trend line for operating expenses exceeds net sales in each of the years shown. This is not positive for Apple. Apple’s net income will suffer if expenses rise faster than sales.

Exhibit 13.6 compares Apple’s revenue trend line to those of Google and Samsung. Google was able to grow revenue in each year relative to the base year. Apple was able to grow revenue overall in the last five years, but at a slower pace than Google. Samsung’s revenue was mainly flat.

Trend analysis can show rela- tions between items on different

4 Yrs Ago 3 Yrs Ago 2 Yrs Ago 1 Yr Ago Current Yr

Tr en

d Pe

rc en

t

50%

0%

200%

150%

100%

250% Net sales Cost of sales Operating expenses

EXHIBIT 13.5 Trend Percent Lines for Apple’s Sales and Expenses

Apple Google Samsung

4 Yrs Ago 3 Yrs Ago 2 Yrs Ago 1 Yr Ago Current Yr

Tr en

d in

R ev

en ue

100%

50%

0%

200%

150%

250% EXHIBIT 13.6 Revenue Trend Percent Lines—Apple, Google, and Samsung

Point: Trend analysis expresses a percent of base, not a percent of change.

APPLE

Samsung GOOGLE

502 Chapter 13 Analysis of Financial Statements

financial statements. Exhibit 13.7 compares Apple’s net sales and total assets. The increase in total assets (81.3%) has exceeded the increase in net sales (34.1%). Is

this result favorable or not? One interpretation is that Apple was less efficient in using its assets in the current year versus four years ago.

EXHIBIT 13.7 Sales and Asset Data for Apple

$ millions Current Yr 4 Yrs Ago Change

Net sales . . . . . . . . . . . . . . . $229,234 $170,910 34.1% Total assets . . . . . . . . . . . . . 375,319 207,000 81.3

Auditor Your tests reveal a 3% increase in sales from $200,000 to $206,000 and a 4% decrease in expenses from $190,000 to $182,400. Both changes are within your “reasonableness” criterion of ±5%, and thus you don’t pursue additional tests. The audit partner in charge questions your lack of follow-up and mentions the joint relation between sales and expenses. What is the partner referring to? ■ Answer: Both individual accounts (sales and expenses) yield percent changes within the ±5% acceptable range. However, a joint analysis shows an increase in sales and a decrease in expenses producing a more than 5% increase in income. This client’s profit margin is 11.46% ([$206,000 − $182,400]/$206,000) for the current year compared with 5.0% ([$200,000 − $190,000]/$200,000) for the prior year—a 129% increase!

Decision Maker

Compute trend percents for the following accounts using 3 Years Ago as the base year. Indicate whether the trend appears to be favorable or unfavorable for each account.

Horizontal Analysis

NEED-TO-KNOW 13-1

P1 $ millions Current Yr 1 Yr Ago 2 Yrs Ago 3 Yrs Ago

Sales . . . . . . . . . . . . . . . . . . . . . . $500 $350 $250 $200

Cost of goods sold . . . . . . . . . . . 400 175 100 50

Solution

$ millions Current Yr 1 Yr Ago 2 Yrs Ago 3 Yrs Ago

Sales . . . . . . . . . . . . . . . . . . . . . . 250% 175% 125% 100% ($500∕$200) ($350∕$200) ($250∕$200) ($200∕$200)

Cost of goods sold . . . . . . . . . . . 800% 350% 200% 100% ($400∕$50) ($175∕$50) ($100∕$50) ($50∕$50)

Analysis: The trend in sales is favorable; however, we need more information about economic condi- tions and competitors’ performances to better assess it. Cost of goods sold also is rising (as expected with increasing sales). However, cost of goods sold is rising faster than the increase in sales, which is bad news.

Do More: QS 13-3, QS 13-4, E 13-3

Vertical analysis, or common-size analysis, is used to evaluate individual financial statement items or a group of items. Vertical comes from the up-down [or down-up] movement of our eyes as we review common-size financial statements.

Common-Size Statements The comparative statements in Exhibits 13.1 and 13.2 show the change in each item over time. Common-size financial statements show changes in the relative importance of each financial statement item. All individual amounts in common-size statements are shown in common-size percents. A common-size percent is calculated as

VERTICAL ANALYSIS P2 Describe and apply methods of vertical analysis.

Sales Expenses Income

10,000 6,000 4,000

Income Statement

Common-size percent (%) = Analysis amount

Base amount × 100

Point: Numerator and denominator in common-size percent are taken from the same financial statement and from the same period.

Chapter 13 Analysis of Financial Statements 503

Common-Size Balance Sheets Common-size statements show each item as a per- cent of a base amount, which for a common-size balance sheet is total assets. The base amount is assigned a value of 100%. (Total liabilities plus equity also equals 100% because this amount equals total assets.) We then compute a common-size percent for each asset, liability, and equity item using total assets as the base amount.

Exhibit 13.8 shows common-size comparative balance sheets for Apple. Two results that stand out on both a magnitude and percentage basis include (1) issuance of long-term debt—a 2.5% increase from 23.4% to 25.9%, the largest of any liability, and (2) a 3.8% decrease in retained earnings and 1% decrease in cash and cash equivalents, largely the result of cash divi- dends and stock buybacks. The absence of other substantial changes in Apple’s balance sheet suggests a mature company, but with some lack of focus as evidenced by the large amounts for securities. This buildup in securities is a concern as the return on securities is historically smaller than the return on operating assets.

Common-Size Income Statements Analysis also involves the use of a common- size income statement. Revenue is the base amount, which is assigned a value of 100%. Each income statement item is shown as a percent of revenue. If we think of the 100% revenue amount

Point: Common-size statements often are used to compare com- panies in the same industry.

APPLE INC. Common-Size Comparative Year-End Balance Sheets

Common-Size Percents* $ millions Current Yr Prior Yr Current Yr Prior Yr

Assets Cash and cash equivalents . . . . . . . . . . . . . . . . . . . . . . . . $ 20,289 $ 20,484 5.4% 6.4% Short-term marketable securities . . . . . . . . . . . . . . . . . . . 53,892 46,671 14.4 14.5 Accounts receivable, net . . . . . . . . . . . . . . . . . . . . . . . . . . 17,874 15,754 4.8 4.9 Inventories . . . . . . . . . . . . . . . . . . . . . . . . . . . . . . . . . . . . . 4,855 2,132 1.3 0.7 Vendor non-trade receivables . . . . . . . . . . . . . . . . . . . . . . 17,799 13,545 4.7 4.2 Other current assets . . . . . . . . . . . . . . . . . . . . . . . . . . . . . . 13,936 8,283 3.7 2.6 Total current assets . . . . . . . . . . . . . . . . . . . . . . . . . . . . 128,645 106,869 34.3 33.2 Long-term marketable securities . . . . . . . . . . . . . . . . . . . 194,714 170,430 51.9 53.0 Property, plant and equipment, net . . . . . . . . . . . . . . . . . 33,783 27,010 9.0 8.4 Goodwill . . . . . . . . . . . . . . . . . . . . . . . . . . . . . . . . . . . . . . . 5,717 5,414 1.5 1.7 Acquired intangible assets, net . . . . . . . . . . . . . . . . . . . . . 2,298 3,206 0.6 1.0 Other non-current assets . . . . . . . . . . . . . . . . . . . . . . . . . . 10,162 8,757 2.7 2.7 Total assets . . . . . . . . . . . . . . . . . . . . . . . . . . . . . . . . . . . . . $375,319 $321,686 100.0% 100.0%

Liabilities Accounts payable . . . . . . . . . . . . . . . . . . . . . . . . . . . . . . . . $ 49,049 $ 37,294 13.1% 11.6% Accrued expenses . . . . . . . . . . . . . . . . . . . . . . . . . . . . . . . 25,744 22,027 6.9 6.8 Deferred revenue . . . . . . . . . . . . . . . . . . . . . . . . . . . . . . . . 7,548 8,080 2.0 2.5 Commercial paper . . . . . . . . . . . . . . . . . . . . . . . . . . . . . . . 11,977 8,105 3.2 2.5 Current portion of long-term debt . . . . . . . . . . . . . . . . . . . 6,496 3,500 1.7 1.1 Total current liabilities . . . . . . . . . . . . . . . . . . . . . . . . . . 100,814 79,006 26.9 24.6 Deferred revenue—noncurrent . . . . . . . . . . . . . . . . . . . . . 2,836 2,930 0.8 0.9 Long-term debt . . . . . . . . . . . . . . . . . . . . . . . . . . . . . . . . . . 97,207 75,427 25.9 23.4 Other non-current liabilties . . . . . . . . . . . . . . . . . . . . . . . . 40,415 36,074 10.8 11.2 Total liabilities . . . . . . . . . . . . . . . . . . . . . . . . . . . . . . . . . . . 241,272 193,437 64.3 60.1

Stockholders’ Equity Common stock . . . . . . . . . . . . . . . . . . . . . . . . . . . . . . . . . . 35,867 31,251 9.6 9.7 Retained earnings . . . . . . . . . . . . . . . . . . . . . . . . . . . . . . . 98,330 96,364 26.2 30.0 Accumulated other comprehensive income . . . . . . . . . . . (150) 634 0.0 0.2 Total stockholders’ equity . . . . . . . . . . . . . . . . . . . . . . . . . 134,047 128,249 35.7 39.9 Total liabilities and stockholders’ equity . . . . . . . . . . . . . . $375,319 $321,686 100.0% 100.0%

EXHIBIT 13.8 Common-Size Comparative Balance Sheets

APPLE

*Percents are rounded to tenths and thus may not exactly sum to totals and subtotals.

504 Chapter 13 Analysis of Financial Statements

EXHIBIT 13.9 Common-Size Comparative Income Statements

APPLE

as representing one sales dollar, the remaining items show how each revenue dollar is distrib- uted among costs, expenses, and income.

Exhibit 13.9 shows common-size comparative income statements for each dollar of Apple’s net sales. The past two years’ common-size numbers are similar with two exceptions. One is the increase of 0.4 cents in research and development costs, which can be a positive development if these costs lead to future revenues. Another is the increase in cost of sales of 0.6 cent and increase in selling, general and administrative costs of 0.1 cent. We must monitor the growth in these expenses.

Common-Size Graphics Exhibit 13.10 is a graphic of Apple’s current-year common-size income statement. This pie chart shows the contribution of each cost component of net sales for net income.

Exhibit 13.11 takes data from Apple’s Segments footnote. The exhibit shows the level of net sales for each of Apple’s five operating seg- ments. Its Americas segment gener-

ates $96.6 billion net sales, which is roughly 42% of its total sales. Within each bar is that segment’s operating income margin (Operating income/Segment net sales). The Americas seg- ment has a 32% operating income margin. This type of graphic can raise questions about the profitability of each segment and lead to discussion of further expansions into more profitable segments. For example, the Japan segment has an operating margin of 46%. A natural question for management is what potential is there to expand sales into the Japan segment and maintain

Cost of sales 61.5%

Selling, general, administrative,

and other income 6.7%

Research and development

5.1%

Income taxes 6.9%

Net income, excluding non-

operating income and expenses

19.8%

EXHIBIT 13.10 Common-Size Graphic of Income Statement

N et

S al

es (i

n bi

l.)

$0

$20

$40

$100

$80

$60

35%46%30%32%

$15.2$17.7

$54.9

$96.6

38%

$44.8

Americas Europe China Japan Asia Pacific

Segment percentages based on: Operating income/Net sales

EXHIBIT 13.11 Sales and Operating Income Margin Breakdown by Segment

APPLE INC. Common-Size Comparative Income Statements

Common-Size Percents* $ millions Current Yr Prior Yr Current Yr Prior Yr

Net sales . . . . . . . . . . . . . . . . . . . . . . . . . . . . . . . . . . . . . . . . . . . $229,234 $215,639 100.0% 100.0% Cost of sales . . . . . . . . . . . . . . . . . . . . . . . . . . . . . . . . . . . . . . . . 141,048 131,376 61.5 60.9 Gross margin . . . . . . . . . . . . . . . . . . . . . . . . . . . . . . . . . . . . . . . . 88,186 84,263 38.5 39.1 Research and development . . . . . . . . . . . . . . . . . . . . . . . . . . . . 11,581 10,045 5.1 4.7 Selling, general and administrative . . . . . . . . . . . . . . . . . . . . . . 15,261 14,194 6.7 6.6 Total operating expenses . . . . . . . . . . . . . . . . . . . . . . . . . . . . . . 26,842 24,239 11.7 11.2 Operating income . . . . . . . . . . . . . . . . . . . . . . . . . . . . . . . . . . . . 61,344 60,024 26.8 27.8 Other income, net . . . . . . . . . . . . . . . . . . . . . . . . . . . . . . . . . . . . 2,745 1,348 1.2 0.6 Income before provision for income taxes . . . . . . . . . . . . . . . . 64,089 61,372 28.0 28.5 Provision for income taxes . . . . . . . . . . . . . . . . . . . . . . . . . . . . . 15,738 15,685 6.9 7.3 Net income . . . . . . . . . . . . . . . . . . . . . . . . . . . . . . . . . . . . . . . . . $ 48,351 $ 45,687 21.1% 21.2%

*Percents are rounded to tenths and thus may not exactly sum to totals and subtotals.

Chapter 13 Analysis of Financial Statements 505

this operating margin? This type of analysis can help determine strategic plans.

Graphics also are used to identify (1) sources of financing, including the distribution among current liabilities, noncurrent liabilities, and equity capital, and (2) focuses of investing activities, including the distribution among current and noncurrent assets. Exhibit 13.12 shows a common-size graphic of Apple’s assets, a high percentage of which are in securities, followed by property, plant and equipment.

Common-size financial statements are useful in comparing companies. Exhibit 13.13 shows com- mon-size graphics of Apple, Google, and Samsung on financing sources. This graphic shows the larger percent of equity financing for Google versus Apple and Samsung. It also shows the larger non- current debt financing of Apple versus Google and Samsung. Comparison of a company’s common- size statements with competitors’ or industry com- mon-size statistics alerts us to differences in the structure of its financial statements.

EXHIBIT 13.12 Common-Size Graphic of Asset Components

Cash and cash equivalents 5%

Short-term marketable securities 14%

Accounts receivable, net 5%

Long-term marketable securities 52%

Acquired intangible assets, net 1%

Other long-term assets 3%

Inventories 1%

Goodwill 2%

Property, plant & equipment, net 9%

Vendor non-trade receivables 5%

Other current assets 3%

Truth Be Told In a survey of nearly 200 CFOs of large companies, roughly 20% say that firms use accounting tools to report earnings that do not fully reflect the firms’ underlying operations. One goal of financial analysis is to see through such ploys. The top reasons CFOs gave for this were to impact stock price, hit an earnings target, and influence executive pay (The Wall Street Journal). ■

Ethical Risk

Apple Google Samsung

36%

Current liabilities

Equity

Noncurrent liabilities 27%

37%

71%

22% 7%

77%

12% 11%

EXHIBIT 13.13 Common-Size Graphic of Financing Sources— Competitor Analysis

APPLE

Samsung GOOGLE

Express the following comparative income statements in common-size percents and assess whether this company’s situation has improved in the current year.

P2 Vertical Analysis

NEED-TO-KNOW 13-2

Comparative Income Statements

For Years Ended December 31 Current Yr Prior Yr

Sales . . . . . . . . . . . . . . . . . . . . . . . . . . . . . $800 $500

Total expenses . . . . . . . . . . . . . . . . . . . . . 560 400

Net income . . . . . . . . . . . . . . . . . . . . . . . . $240 $100

Current Yr Prior Yr

Sales . . . . . . . . . . . . 100% 100% ($800∕$800) ($500∕$500)

Total expenses . . . . 70% 80% ($560∕$800) ($400∕$500)

Net income . . . . . . . 30% 20%

Analysis: This company’s situation has improved. This is evident from its substantial increase in net income as a percent of sales for the current year (30%) relative to the prior year (20%). Further, the company’s sales increased from $500 to $800 (while expenses declined as a percent of sales from 80% to 70%).

Do More: QS 13-5, E 13-4, E 13-5, E 13-6

Solution

506 Chapter 13 Analysis of Financial Statements

Ratios are used to uncover conditions and trends difficult to detect by looking at individual amounts. A ratio shows a relation between two amounts. It can be shown as a percent, rate, or proportion. A change from $100 to $250 can be shown as (1) 150% increase, (2) 2.5 times, or (3) 2.5 to 1 (or 2.5:1). To be useful, a ratio must show an economically important relation. For example, a ratio of cost of goods sold to sales is useful, but a ratio of freight costs to patents is not.

This section covers important financial ratios organized into the four building blocks of financial statement analysis: (1) liquidity and efficiency, (2) solvency, (3) profitability, and (4) market pros- pects. We use four standards for comparison: intracompany, competitor, industry, and guidelines.

Liquidity and Efficiency Liquidity is the availability of resources to pay short-term cash requirements. It is affected by the timing of cash inflows and outflows along with prospects for future performance. A lack of

liquidity often is linked to lower profitability. To creditors, lack of liquidity can cause delays in collecting payments. Efficiency is how productive a company is in using its assets. Inefficient use of assets can cause liquidity problems. This section covers key ratios used to assess liquidity and efficiency.

Working Capital and Current Ratio The amount of current assets minus cur- rent liabilities is called working capital, or net working capital. A company that runs low

on working capital is less likely to pay debts or to continue operating. When evaluating a compa- ny’s working capital, we look at the dollar amount of current assets minus current liabilities and at their ratio. The current ratio is defined as follows (see Chapter 3 for additional explanation).

RATIO ANALYSIS P3 Define and apply ratio analysis.

Ratios

Current ratio = Current assets

Current liabilities

Apple’s working capital and current ratio are shown in Exhibit 13.14. Also, Google’s (5.14), Samsung’s (2.19), and the industry’s (2.5) current ratios are shown in the margin. Although its ratio (1.28) is lower than competitors’ ratios, Apple is not in danger of defaulting on loan payments. A high current ratio suggests a strong abil- ity to meet current obligations. An exces-

sively high current ratio means that the company has invested too much in current assets compared to current obligations. An excessive investment in current assets is not an efficient use of funds because current assets normally earn a low return on investment (compared with long-term assets).

Many analysts use a guideline of 2:1 (or 1.5:1) for the current ratio. A 2:1 or higher ratio is con- sidered low risk in the short run. Analysis of the current ratio, and many other ratios, must consider type of business, composition of current assets, and turnover rate of current asset components. Business Type A service company that grants little or no credit and carries few inventories

can probably operate on a current ratio of less than 1:1 if its revenues generate enough cash to pay its current liabilities. On the other hand, a company selling high-priced clothing or furniture requires a higher ratio because of difficulties in judging customer demand and cash receipts.

Asset Composition The composition of assets is important to assess short-term liquidity. For instance, cash, cash equivalents, and short-term investments are more liquid than ac- counts and notes receivable. An excessive amount of receivables and inventory weakens a company’s ability to pay current liabilities.

Turnover Rate Asset turnover measures efficiency in using assets. A measure of asset efficiency is revenue generated.

Global: Ratio analysis is unaffected by currency but is affected by dif- ferences in accounting principles.

EXHIBIT 13.14 Apple’s Working Capital and Current Ratio

$ millions Current Yr Prior Yr

Current assets . . . . . . . . . . . . . . $128,645 $106,869

Current liabilities . . . . . . . . . . . . 100,814 79,006

Working capital . . . . . . . . . . . . $ 27,831 $ 27,863 Current ratio $128,645/$100,814 = 1.28 to 1 $106,869/$79,006 = 1.35 to 1

Current ratio Google = 5.14 Samsung = 2.19 Industry = 2.5

Chapter 13 Analysis of Financial Statements 507

Banker A company requests a one-year, $200,000 loan for expansion. This company’s current ratio is 4:1, with current assets of $160,000. Key competitors have a current ratio of 1.9:1. Using this information, do you approve the loan? ■ Answer: The loan application is likely approved for at least two reasons. First, the current ratio suggests an ability to meet short-term obligations. Second, current assets of $160,000 and a current ratio of 4:1 imply current liabilities of $40,000 (one-fourth of current assets) and a working capital excess of $120,000. The working capital is 60% of the loan.

Decision Maker

Acid-Test Ratio Quick assets are cash, short-term investments, and current receivables. These are the most liquid types of current assets. The acid-test ratio, also called quick ratio and introduced in Chapter 4, evaluates a company’s short-term liquidity.

Acid-test ratio = Cash + Short-term investments + Current receivables

Current liabilities

Apple’s acid-test ratio is computed in Exhibit 13.15. Apple’s acid-test ratio (0.91) is lower than those for Google (4.97), Samsung (1.71), and the 1:1 common guideline for an acceptable acid-test ratio. As with analysis of the current ratio, we must consider other factors. How frequently a company converts its current assets into cash also affects its ability to pay current obligations. This means analysis of short- term liquidity should consider receivables and inventories, which we cover next.

Accounts Receivable Turnover Accounts receivable turnover measures how fre- quently a company converts its receivables into cash. This ratio is defined as follows (see Chapter 7 for additional explanation). Apple’s accounts receivable turnover is computed next to the formula ($ millions). Apple’s turnover of 13.6 exceeds Google’s 6.8 and Samsung’s 9.2 turnover. Accounts receivable turnover is high when accounts receivable are quickly collected. A high turnover is favorable because it means the company does not tie up assets in accounts receivable. However, accounts receivable turnover can be too high; this can occur when credit terms are so restrictive that they decrease sales.

EXHIBIT 13.15 Acid-Test Ratio

$ millions Current Yr Prior Yr

Cash and equivalents . . . . . . . . . . . . $ 20,289 $20,484

Short-term securities . . . . . . . . . . . . 53,892 46,671

Current receivables . . . . . . . . . . . . . 17,874 15,754

Total quick assets . . . . . . . . . . . . . . . $ 92,055 $82,909

Current liabilities . . . . . . . . . . . . . . . . $100,814 $79,006

Acid-test ratio $92,055/$100,814 = 0.91 to 1 $82,909/$79,006 = 1.05 to 1

Acid-test ratio Google = 4.97 Samsung = 1.71 Industry = 0.9

Accounts receivable turnover Google = 6.8 Samsung = 9.2 Industry = 5.0

Accounts receivableturnover = Net sales

Average accounts receivable, net = $229,234

($15,754 + $17,874)/2 = 13.6 times

Inventory Turnover Inventory turnover measures how long a company holds inventory before selling it. It is defined as follows (see Chapter 5 for additional explanation). Next to the formula we compute Apple’s inventory turnover at 40.4. Apple’s inventory turnover is higher than Samsung’s 6.0 but lower than Google’s 89.6. A company with a high turnover requires a smaller investment in inventory than one producing the same sales with a lower turnover. However, high inventory turnover can be bad if inventory is so low that stock-outs occur.

Inventory turnover = Cost of goods sold Average inventory

= $141,048

($2,132 + $4,855)/2 = 40.4 times

Days’ Sales Uncollected Days’ sales uncollected measures how frequently a company collects accounts receivable and is defined as follows (Chapter 6 provides additional explana- tion). Apple’s days’ sales uncollected of 28.5 days is shown next to the formula. Both Google’s days’ sales uncollected of 60.4 days and Samsung’s 48.5 days are more than the 28.5 days for Apple. Days’ sales uncollected is more meaningful if we know company credit terms. A rough

Inventory turnover Google = 89.6 Samsung = 6.0 Industry = 7.0

©VCG/Getty Images

508 Chapter 13 Analysis of Financial Statements

Total Asset Turnover Total asset turnover measures a company’s ability to use its as- sets to generate sales and reflects on operating efficiency. The definition of this ratio follows (Chapter 8 offers additional explanation). Apple’s total asset turnover of 0.66 is shown next to the formula. Apple’s turnover is greater than that for Google (0.61), but not Samsung (0.85).

Days’ sales in inventory = Ending inventory Cost of goods sold

× 365 = $4,855

$141,048 × 365 = 12.6 days

Days’ sales in inventory Google = 6.0 Samsung = 70.5 Industry = 35

Days’ Sales in Inventory Days’ sales in inventory is used to evaluate inventory liquid- ity. We compute days’ sales in inventory as follows (Chapter 5 provides additional explanation). Apple’s days’ sales in inventory of 12.6 days is shown next to the formula. If the products in Apple’s inventory are in demand by customers, this formula estimates that its inventory will be converted into receivables (or cash) in 12.6 days. If all of Apple’s sales were credit sales, the conversion of inventory to receivables in 12.6 days plus the conversion of receivables to cash in 28.5 days implies that inventory will be converted to cash in about 41.1 days (12.6 + 28.5).

Point: Average collection period is estimated by dividing 365 by the accounts receivable turnover ra- tio. For example, 365 divided by an accounts receivable turnover of 12.6 indicates a 29-day aver- age collection period.

Total asset turnover = Net sales

Average total assets =

$229,234 ($375,319 + $321,686)/2

= 0.66 times Total asset turnover Google = 0.61 Samsung = 0.85 Industry = 1.1

Solvency Solvency is a company’s ability to meet long-term obligations and generate future revenues. Analysis of solvency is long term and uses broader measures than liquidity. An important part of solvency analysis is a company’s capital structure. Capital structure is a company’s makeup of equity and debt financing. Our analysis here focuses on a company’s ability to both meet its obligations and provide security to its creditors over the long run.

Debt Ratio and Equity Ratio One part of solvency analysis is to assess a company’s mix of debt and equity financing. The debt ratio (described in Chapter 2) shows total liabilities as a percent of total assets. The equity ratio shows total equity as a percent of total assets. Apple’s debt and equity ratios follow. Apple’s ratios reveal more debt than equity. A company is considered less risky if its capital structure (equity plus debt) has more equity. Debt is consid- ered more risky because of its required payments for interest and principal. Stockholders cannot require payment from the company. However, debt can increase income for stockholders if the company earns a higher return than interest paid on the debt.

Point: For analysis purposes, noncontrolling interest is usually included in equity.

Point: Total of debt and equity ratios always equals 100%.

$ millions Current Yr Ratios

Total liabilities . . . . . . . . . . . . . . . . . . . . . . $241,272 64.3% [Debt ratio] Total equity . . . . . . . . . . . . . . . . . . . . . . . . 134,047 35.7% [Equity ratio] Total liabilities and equity . . . . . . . . . . . . . $375,319 100 .0%

Debt ratio :: Equity ratio Google = 22.7% :: 77.3% Samsung = 28.9% :: 71.1% Industry = 35% :: 65%

Days’ sales uncollected = Accounts receivable, net

Net sales × 365 =

$17,874 $229,234

× 365 = 28.5 daysDays’ sales uncollectedGoogle = 60.4 Samsung = 48.5

guideline states that days’ sales uncollected should not exceed 11⁄3 times the days in its (1) credit period, if discounts are not offered, or (2) discount period, if favorable discounts are offered.

Debt-to-Equity Ratio The debt-to-equity ratio is another measure of solvency. We com- pute the ratio as follows (Chapter 10 offers additional explanation). Apple’s debt-to-equity ratio of 1.80 is shown next to the formula. Apple’s ratio is higher than those of Google (0.29) and Samsung (0.41), and greater than the industry ratio of 0.6. Apple’s capital structure has more

Chapter 13 Analysis of Financial Statements 509

debt than equity. Debt must be repaid with interest, while equity does not. Debt payments can be burdensome when the industry and/or the economy experience a downturn.

Times Interest Earned The amount of income before subtracting interest expense and in- come tax expense is the amount available to pay interest expense. The following times interest earned ratio measures a company’s ability to pay interest (see Chapter 9 for additional explanation).

Debt-to-equity ratio = Total liabilities

Total equity =

$241,272 $134,047

= 1.80 Debt-to-equity Google = 0.29 Samsung = 0.41 Industry = 0.6

$48,351 + $2,323 + $15,738 $2,323

= 28.6 times Times interest earned Google = 250.5 Samsung = 86.7

Times interest earned = Income before interest expense and income tax expense

Interest expense

The larger this ratio is, the less risky the company is for creditors. One guideline says that creditors are reasonably safe if the company has a ratio of two or more. Apple’s times interest earned ratio of 28.6 follows. It suggests that creditors have little risk of nonrepayment.

Profitability Profitability is a company’s ability to earn an adequate return. This section covers key profit- ability measures.

Profit Margin Profit margin measures a company’s ability to earn net income from sales (Chapter 3 offers additional explanation). Apple’s profit margin of 21.1% is shown next to the formula. To evaluate profit margin, we must consider the industry. For instance, an appliance company might require a profit margin of 15%, whereas a retail supermarket might require a profit margin of 2%. Apple’s 21.1% profit margin is better than Google’s 11.4%, Samsung’s 17.6%, and the industry’s 11% margin.

Profit margin = Net income

Net sales =

$48,351 $229,234

= 21.1% Profit margin Google = 11.4% Samsung = 17.6% Industry = 11%

Return on Total Assets Return on total assets is defined as follows. Apple’s return on total assets of 13.9% is shown next to the formula. Apple’s 13.9% return on total assets is higher than Google’s 6.9% and the industry’s 8%, but lower than Samsung’s 15.0%. We also should evaluate any trend in the return.

Return on total assets = Net income

Average total assets =

$48,351 ($375,319 + $321,686)/2

= 13.9% Return on total assets Google = 6.9% Samsung = 15.0% Industry = 8%

The relation between profit margin, total asset turnover, and return on total assets follows.

Profit margin × Total asset turnover = Return on total assets

Net income Net sales

× Net sales

Average total assets =

Net income Average total assets

Both profit margin and total asset turnover affect operating efficiency, as measured by return on total assets. This formula is applied to Apple as follows. This analysis shows that Apple’s supe- rior return on assets versus that of Google is driven by its high profit margin and good asset turnover.

21.1% × 0.66 = 13.9% (with rounding) Google = 11.4% × 0.61 ≃ 6.9% Samsung = 17.6% × 0.85 ≃ 15.0%

(with rounding)

510 Chapter 13 Analysis of Financial Statements

Return on Common Stockholders’ Equity The most important goal in operating a company is to earn income for its owner(s). Return on common stockholders’ equity measures a company’s ability to earn income for common stockholders and is defined as follows.

Return on common stockholders’ equity = Net income − Preferred dividends

Average common stockholders’ equity

Apple’s return on common stockholders’ equity is computed as follows. The denominator in this computation is the book value of common equity. Dividends on cumulative preferred stock are subtracted from income whether they are declared or are in arrears. If preferred stock is non- cumulative, its dividends are subtracted only if declared. Apple’s 36.9% return on common stockholders’ equity is superior to Google’s 8.7% and Samsung’s 20.5%.

$48,351 − $0 ($128,249 + $134,047)/2

= 36.9% Return on common equity Google = 8.7% Samsung = 20.5% Industry = 15%

Take It to the Street Wall Street is synonymous with financial markets, but its name comes from the street location of the original New York Stock Exchange. The street’s name comes from stockades built by early settlers to protect New York from pirate attacks. ■

Decision Insight

Market Prospects Market measures are useful for analyzing corporations with publicly traded stock. These market measures use stock price, which reflects the market’s (public’s) expectations for the company. This includes market expectations of both company return and risk.

Price-Earnings Ratio Computation of the price-earnings ratio follows (Chapter 11 provides additional explanation). This ratio is used to measure market expectations for future growth. The market price of Apple’s common stock at the start of the current fiscal year was $154.12. Using Apple’s $9.27 basic earnings per share, we compute its price-earnings ratio as follows. Apple’s price-earnings ratio is less than that for Samsung and Google, but it is higher than the industry norm for this period.

23.90 15.00 15.34 17.89 19.45 13.67 13.60 25.65 15.45 18.85 23.56 18.85 17.23

+3.58% +12.3% +5.34% +5.94% +2.13% +6.43% -11.6% +23.1% +5.56% -3.67% +11.3% +2.54% +12.3%

400.20 253.95 285.32 248.20 989.26 320.34 208.98 432.62 765.23 564.23 256.25 524.65 754.62

530.000 320.000 430.000 900.000 600.000 380.000 220.000 750.000 250.000 120.000 158.000 245.000 658.000

Price-earnings ratio = Market price per common share

Earnings per share =

$154.12 $9.27

= 16.6 PE (year-end) Google = 57.3 Samsung = 22.9 Industry = 11

Dividend Yield Dividend yield is used to compare the dividend-paying performance of different companies. We compute dividend yield as follows (Chapter 11 offers additional expla- nation). Apple’s dividend yield of 1.6%, based on its fiscal year-end market price per share of $154.12 and its $2.40 cash dividends per share, is shown next to the formula. Some companies, such as Google, do not pay dividends because they reinvest the cash to grow their businesses in the hope of generating greater future earnings and dividends.

Dividend yield = Annual cash dividends per share

Market price per share =

$2.40 $154.12

= 1.6%

Point: Low expectations = low PE. High expectations = high PE.

Bull Session A bear market is a declining market. The phrase comes from bear-skin hunters who sold the skins before the bears were caught. The term bear was then used to describe investors who sold shares they did not own in anticipation of a price decline. A bull market is a rising market. This phrase comes from the once-popular sport of bear and bull baiting. The term bull means the opposite of bear. ■

Decision Insight

Dividend yield Google = 0.0% Samsung = 1.6%

©Partner Media GmbH/Alamy Stock Photo

Chapter 13 Analysis of Financial Statements 511

Ratio Formula Measure of

Liquidity and Efficiency

Current ratio = Current assets

Current liabilities Short-term debt-paying ability

Acid-test ratio = Cash + Short-term investments + Current receivables

Current liabilities Immediate short-term debt-paying ability

Accounts receivable turnover = Net sales

Average accounts receivable, net Efficiency of collection

Inventory turnover = Cost of goods sold Average inventory

Efficiency of inventory management

Days’ sales uncollected = Accounts receivable, net

Net sales × 365 Liquidity of receivables

Days’ sales in inventory = Ending inventory

Cost of goods sold × 365 Liquidity of inventory

Total asset turnover = Net sales

Average total assets Efficiency of assets in producing sales

Solvency

Debt ratio = Total liabilities

Total assets Creditor financing and leverage

Equity ratio = Total equity Total assets

Owner financing

Debt-to-equity ratio = Total liabilities

Total equity Debt versus equity financing

Times interest earned = Income before interest expense and income tax expense

Interest expense Protection in meeting interest payments

Profitability

Profit margin ratio = Net income Net sales

Net income in each sales dollar

Gross margin ratio = Net sales − Cost of goods sold

Net sales Gross margin in each sales dollar

Return on total assets = Net income

Average total assets Overall profitability of assets

Return on common stockholders’ equity = Net income − Preferred dividends

Average common stockholdersʼ equity Profitability of owner investment

Book value per common share = Shareholdersʼ equity applicable to common shares

Number of common shares outstanding Liquidation at reported amounts

Basic earnings per share = Net income − Preferred dividends

Weighted-average common shares outstanding Net income per common share

Market Prospects

Price-earnings ratio = Market price per common share

Earnings per share Market value relative to earnings

Dividend yield = Annual cash dividends per share

Market price per share Cash return per common share

EXHIBIT 13.16 Financial Statement Analysis Ratios

Summary of Ratios Exhibit 13.16 summarizes the ratios illustrated in this chapter and throughout the book.

512 Chapter 13 Analysis of Financial Statements

Use the following financial statements of Precision Co. to complete these requirements. 1. Prepare comparative income statements showing the percent increase or decrease for the current year

in comparison to the prior year. 2. Prepare common-size comparative balance sheets for both years. 3. Compute the following ratios for the current year and identify each one’s building block category for

financial statement analysis. a. Current ratio b. Acid-test ratio c. Accounts receivable turnover d. Days’ sales uncollected e. Inventory turnover f. Debt ratio

COMPREHENSIVE

Applying Horizontal, Vertical, and Ratio Analyses

NEED-TO-KNOW 13-4

g. Debt-to-equity ratio h. Times interest earned i. Profit margin ratio j. Total asset turnover k. Return on total assets l. Return on common stockholders’ equity

For each ratio listed, identify whether the change in ratio value from the prior year to the current year is favorable or unfavorable.

Solution

P3 Ratio Analysis

NEED-TO-KNOW 13-3

Ratio Current Yr Prior Yr Change

1 . Profit margin ratio . . . . . . . . . . . . . . . . . . . . . . 6% 8% Unfavorable

2 . Debt ratio . . . . . . . . . . . . . . . . . . . . . . . . . . . . . 50% 70% Favorable

3 . Gross margin ratio . . . . . . . . . . . . . . . . . . . . . . 40% 36% Favorable

4 . Accounts receivable turnover . . . . . . . . . . . . . 8 .8 9 .4 Unfavorable

5 . Basic earnings per share . . . . . . . . . . . . . . . . $2 .10 $2 .00 Favorable

6 . Inventory turnover . . . . . . . . . . . . . . . . . . . . . . 3 .6 4 .0 Unfavorable

Do More: QS 13-6 through QS 13-13, E 13-7, E 13-8, E 13-9, E 13-10, E 13-11,

P 13-4

Ratio Current Yr Prior Yr

1 . Profit margin . . . . . . . . . . . 6% 8%

2 . Debt ratio . . . . . . . . . . . . . . 50% 70%

3 . Gross margin . . . . . . . . . . . 40% 36%

Ratio Current Yr Prior Yr

4 . Accounts receivable turnover . . . . . . 8 .8 9 .4

5 . Basic earnings per share . . . . . . . . . $2 .10 $2 .00

6 . Inventory turnover . . . . . . . . . . . . . . . 3 .6 4 .0

Analysis ReportingDecision Analysis

A financial statement analysis report usually consists of six sections.

1. Executive summary—brief analysis of results and conclusions. 2. Analysis overview—background on the company, its industry, and the economy. 3. Evidential matter—financial statements and information used in the analysis, including ratios,

trends, comparisons, and all analytical measures used. 4. Assumptions—list of assumptions about a company’s industry and economic environment, and

other assumptions underlying estimates. 5. Key factors—list of favorable and unfavorable factors, both quantitative and qualitative, for com-

pany performance; usually organized by areas of analysis. 6. Inferences—forecasts, estimates, interpretations, and conclusions of the analysis report.

We must remember that the user dictates relevance, meaning that the analysis report should include a brief table of contents to help readers focus on those areas most relevant to their decisions. Finally, writing is important. Mistakes in grammar and errors of fact compromise the report’s credibility.

A1 Summarize and report results of analysis.

Short and Sweet Short selling refers to selling stock before you buy it. Here’s an example: You borrow 100 shares of Nike stock, sell them at $55 each, and receive money from their sale. You then wait. You hope that Nike’s stock price falls to, say, $50 each and you can replace the borrowed stock for less than you sold it, reaping a profit of $5 each less any transaction costs. ■

Decision Insight

Chapter 13 Analysis of Financial Statements 513

PRECISION COMPANY Comparative Income Statements

For Years Ended December 31 Current Yr Prior Yr

Sales . . . . . . . . . . . . . . . . . . . . . . . . . . . . . . $2,486,000 $2,075,000

Cost of goods sold . . . . . . . . . . . . . . . . . . . 1,523,000 1,222,000

Gross profit . . . . . . . . . . . . . . . . . . . . . . . . . 963,000 853,000

Operating expenses

Advertising expense . . . . . . . . . . . . . . . . 145,000 100,000

Sales salaries expense . . . . . . . . . . . . . . 240,000 280,000

Office salaries expense . . . . . . . . . . . . . 165,000 200,000

Insurance expense . . . . . . . . . . . . . . . . . 100,000 45,000

Supplies expense . . . . . . . . . . . . . . . . . . 26,000 35,000

Depreciation expense . . . . . . . . . . . . . . 85,000 75,000

Miscellaneous expenses . . . . . . . . . . . . 17,000 15,000

Total operating expenses . . . . . . . . . . . 778,000 750,000

Operating income . . . . . . . . . . . . . . . . . . . . 185,000 103,000

Interest expense . . . . . . . . . . . . . . . . . . . . . 44,000 46,000

Income before taxes . . . . . . . . . . . . . . . . . . 141,000 57,000

Income tax expense . . . . . . . . . . . . . . . . . . 47,000 19,000

Net income . . . . . . . . . . . . . . . . . . . . . . . . . $ 94,000 $ 38,000

Earnings per share . . . . . . . . . . . . . . . . . . . $ 0 .99 $ 0 .40

PRECISION COMPANY Comparative Year-End Balance Sheets

At December 31 Current Yr Prior Yr

Assets Current assets

Cash . . . . . . . . . . . . . . . . . . . . . . . . . . . . . . . $ 79,000 $ 42,000

Short-term investments . . . . . . . . . . . . . . . . 65,000 96,000

Accounts receivable, net . . . . . . . . . . . . . . . 120,000 100,000

Merchandise inventory . . . . . . . . . . . . . . . . 250,000 265,000

Total current assets . . . . . . . . . . . . . . . . . . . 514,000 503,000

Plant assets

Store equipment, net . . . . . . . . . . . . . . . . . . 400,000 350,000

Office equipment, net . . . . . . . . . . . . . . . . . 45,000 50,000

Buildings, net . . . . . . . . . . . . . . . . . . . . . . . . 625,000 675,000

Land . . . . . . . . . . . . . . . . . . . . . . . . . . . . . . . 100,000 100,000

Total plant assets . . . . . . . . . . . . . . . . . . . . . 1,170,000 1,175,000

Total assets . . . . . . . . . . . . . . . . . . . . . . . . . . . . $1,684,000 $1,678,000

Liabilities Current liabilities

Accounts payable . . . . . . . . . . . . . . . . . . . . . $ 164,000 $ 190,000

Short-term notes payable . . . . . . . . . . . . . . 75,000 90,000

Taxes payable . . . . . . . . . . . . . . . . . . . . . . . . 26,000 12,000

Total current liabilities . . . . . . . . . . . . . . . . . 265,000 292,000

Long-term liabilities

Notes payable (secured by mortgage on buildings) . . . . . . . . . . . . . . 400,000 420,000

Total liabilities . . . . . . . . . . . . . . . . . . . . . . . . . . 665,000 712,000

Stockholders’ Equity Common stock, $5 par value . . . . . . . . . . . . . . 475,000 475,000

Retained earnings . . . . . . . . . . . . . . . . . . . . . . . 544,000 491,000

Total stockholders’ equity . . . . . . . . . . . . . . . . 1,019,000 966,000

Total liabilities and equity . . . . . . . . . . . . . . . . . $1,684,000 $1,678,000

PLANNING THE SOLUTION Set up a four-column income statement; enter the current-year

and prior-year amounts in the first two columns and then enter the dollar change in the third column and the percent change from the prior year in the fourth column.

Set up a four-column balance sheet; enter the current-year and prior-year year-end amounts in the first two columns and then com- pute and enter the amount of each item as a percent of total assets.

Compute the required ratios using the data provided. Use the average of beginning and ending amounts when appropriate (see Exhibit 13.16 for definitions).

SOLUTION 1.

PRECISION COMPANY Comparative Income Statements

For Years Ended December 31 Current Yr Prior Yr Dollar Change Percent Change

Sales . . . . . . . . . . . . . . . . . . . . . . . . . . . . . . . . . $2,486,000 $2,075,000 $411,000 19.8% Cost of goods sold . . . . . . . . . . . . . . . . . . . . . . 1,523,000 1,222,000 301,000 24.6 Gross profit . . . . . . . . . . . . . . . . . . . . . . . . . . . . 963,000 853,000 110,000 12.9 Operating expenses Advertising expense . . . . . . . . . . . . . . . . . . . 145,000 100,000 45,000 45.0 Sales salaries expense . . . . . . . . . . . . . . . . . 240,000 280,000 (40,000) (14.3) Office salaries expense . . . . . . . . . . . . . . . . 165,000 200,000 (35,000) (17.5) Insurance expense . . . . . . . . . . . . . . . . . . . . 100,000 45,000 55,000 122.2 Supplies expense . . . . . . . . . . . . . . . . . . . . . 26,000 35,000 (9,000) (25.7) Depreciation expense . . . . . . . . . . . . . . . . . 85,000 75,000 10,000 13.3 Miscellaneous expenses . . . . . . . . . . . . . . . 17,000 15,000 2,000 13.3 Total operating expenses . . . . . . . . . . . . . . 778,000 750,000 28,000 3.7 Operating income . . . . . . . . . . . . . . . . . . . . . . . 185,000 103,000 82,000 79.6 Interest expense . . . . . . . . . . . . . . . . . . . . . . . . 44,000 46,000 (2,000) (4.3) Income before taxes . . . . . . . . . . . . . . . . . . . . . 141,000 57,000 84,000 147.4 Income tax expense . . . . . . . . . . . . . . . . . . . . 47,000 19,000 28,000 147.4 Net income . . . . . . . . . . . . . . . . . . . . . . . . . . . . $ 94,000 $ 38,000 $ 56,000 147.4 Earnings per share . . . . . . . . . . . . . . . . . . . . . . $ 0 .99 $ 0 .40 $ 0.59 147.5

514 Chapter 13 Analysis of Financial Statements

2. PRECISION COMPANY

Common-Size Comparative Year-End Balance Sheets Common-Size Percents

At December 31 Current Yr Prior Yr Current Yr* Prior Yr*

Assets Current assets

Cash . . . . . . . . . . . . . . . . . . . . . . . . . . . . . $ 79,000 $ 42,000 4.7% 2.5% Short-term investments . . . . . . . . . . . . . . 65,000 96,000 3.9 5.7 Accounts receivable, net . . . . . . . . . . . . . 120,000 100,000 7.1 6.0 Merchandise inventory . . . . . . . . . . . . . . . 250,000 265,000 14.8 15.8 Total current assets . . . . . . . . . . . . . . . . . 514,000 503,000 30.5 30.0 Plant assets

Store equipment, net . . . . . . . . . . . . . . . . 400,000 350,000 23.8 20.9 Office equipment, net . . . . . . . . . . . . . . . . 45,000 50,000 2.7 3.0 Buildings, net . . . . . . . . . . . . . . . . . . . . . . 625,000 675,000 37.1 40.2 Land . . . . . . . . . . . . . . . . . . . . . . . . . . . . . . 100,000 100,000 5.9 6.0 Total plant assets . . . . . . . . . . . . . . . . . . . 1,170,000 1,175,000 69.5 70.0 Total assets . . . . . . . . . . . . . . . . . . . . . . . . . . $1,684,000 $1,678,000 100.0% 100.0%

Liabilities Current liabilities

Accounts payable . . . . . . . . . . . . . . . . . . . $ 164,000 $ 190,000 9.7% 11.3% Short-term notes payable . . . . . . . . . . . . 75,000 90,000 4.5 5.4 Taxes payable . . . . . . . . . . . . . . . . . . . . . . 26,000 12,000 1.5 0.7 Total current liabilities . . . . . . . . . . . . . . . 265,000 292,000 15.7 17.4 Long-term liabilities

Notes payable (secured by

mortgage on buildings) . . . . . . . . . . . . 400,000 420,000 23.8 25.0 Total liabilities . . . . . . . . . . . . . . . . . . . . . . . . 665,000 712,000 39.5 42.4

Stockholders’ Equity Common stock, $5 par value . . . . . . . . . . . . 475,000 475,000 28.2 28.3 Retained earnings . . . . . . . . . . . . . . . . . . . . . 544,000 491,000 32.3 29.3 Total stockholders’ equity . . . . . . . . . . . . . . . 1,019,000 966,000 60.5 57.6 Total liabilities and equity . . . . . . . . . . . . . . . $1,684,000 $1,678,000 100.0% 100.0%

*Columns do not always exactly add to 100 due to rounding.

3. Ratios: a. Current ratio: $514,000/$265,000 = 1.9:1 (liquidity and efficiency) b. Acid-test ratio: ($79,000 + $65,000 + $120,000)/$265,000 = 1.0:1 (liquidity and efficiency) c. Average receivables: ($120,000 + $100,000)/2 = $110,000 Accounts receivable turnover: $2,486,000/$110,000 = 22.6 times (liquidity and efficiency) d. Days’ sales uncollected: ($120,000/$2,486,000) × 365 = 17.6 days (liquidity and efficiency) e. Average inventory: ($250,000 + $265,000)/2 = $257,500 Inventory turnover: $1,523,000/$257,500 = 5.9 times (liquidity and efficiency) f. Debt ratio: $665,000/$1,684,000 = 39.5% (solvency) g. Debt-to-equity ratio: $665,000/$1,019,000 = 0.65 (solvency) h. Times interest earned: $185,000/$44,000 = 4.2 times (solvency) i. Profit margin ratio: $94,000/$2,486,000 = 3.8% (profitability) j. Average total assets: ($1,684,000 + $1,678,000)/2 = $1,681,000 Total asset turnover: $2,486,000/$1,681,000 = 1.48 times (liquidity and efficiency) k. Return on total assets: $94,000/$1,681,000 = 5.6% or 3.8% × 1.48 = 5.6% (profitability) l. Average total common equity: ($1,019,000 + $966,000)/2 = $992,500 Return on common stockholders’ equity: $94,000/$992,500 = 9.5% (profitability)

Chapter 13 Analysis of Financial Statements 515

APPENDIX

Sustainable Income 13A When a company’s activities include income-related events not part of its normal, continuing operations, it must disclose these events. To alert users to these activities, companies separate the income statement into continuing operations, discontinued segments, comprehensive income, and earnings per share. Exhibit 13A.1 shows such an income statement for ComUS. These separations help us measure sustain- able income, which is the income level most likely to continue into the future. Sustainable income is com- monly used in performance measures.

A2 Explain the form and assess the content of a complete income statement.

ComUS Income Statement

For Year Ended December 31

Net sales . . . . . . . . . . . . . . . . . . . . . . . . . . . . . . . . . . . . . . . . . . . . . . . . . . . . . . . . . . . . . . . $8,478,000

Operating expenses

Cost of goods sold . . . . . . . . . . . . . . . . . . . . . . . . . . . . . . . . . . . . . . . . . . . . . . . . . . . . . $5,950,000

Depreciation expense . . . . . . . . . . . . . . . . . . . . . . . . . . . . . . . . . . . . . . . . . . . . . . . . . . 35,000

Other selling, general, and administrative expenses . . . . . . . . . . . . . . . . . . . . . . . . . . 515,000

Interest expense . . . . . . . . . . . . . . . . . . . . . . . . . . . . . . . . . . . . . . . . . . . . . . . . . . . . . . . 20,000

Total operating expenses . . . . . . . . . . . . . . . . . . . . . . . . . . . . . . . . . . . . . . . . . . . . . . . . (6,520,000)

Other unusual and/or infrequent gains (losses)

Loss on plant relocation . . . . . . . . . . . . . . . . . . . . . . . . . . . . . . . . . . . . . . . . . . . . . . . . . (45,000)

Gain on sale of surplus land . . . . . . . . . . . . . . . . . . . . . . . . . . . . . . . . . . . . . . . . . . . . . . 72,000

Income from continuing operations before taxes . . . . . . . . . . . . . . . . . . . . . . . . . . . . . . . 1,985,000

Income tax expense . . . . . . . . . . . . . . . . . . . . . . . . . . . . . . . . . . . . . . . . . . . . . . . . . . . . . . (595,500)

Income from continuing operations . . . . . . . . . . . . . . . . . . . . . . . . . . . . . . . . . . . . . . . . . . 1,389,500

Discontinued segment Income from operating Division A (net of $180,000 taxes) . . . . . . . . . . . . . . . . . . . . . . . 420,000

Loss on disposal of Division A (net of $66,000 tax benefit) . . . . . . . . . . . . . . . . . . . . . . . (154,000) 266,000

Net income . . . . . . . . . . . . . . . . . . . . . . . . . . . . . . . . . . . . . . . . . . . . . . . . . . . . . . . . . . . . . $ 1,655,500

Earnings per common share (200,000 outstanding shares) Income from continuing operations . . . . . . . . . . . . . . . . . . . . . . . . . . . . . . . . . . . . . . . $ 6 .95

Discontinued operations . . . . . . . . . . . . . . . . . . . . . . . . . . . . . . . . . . . . . . . . . . . . . . . . 1 .33

Net income (basic earnings per share) . . . . . . . . . . . . . . . . . . . . . . . . . . . . . . . . . . . . . $ 8 .28

1

2

3

⎫ ⎪ ⎬ ⎪ ⎭

⎫ ⎪ ⎬ ⎪ ⎭

⎫ ⎪ ⎪ ⎪ ⎪ ⎪ ⎪ ⎪ ⎪ ⎪ ⎬ ⎪ ⎪ ⎪ ⎪ ⎪ ⎪ ⎪ ⎪ ⎪ ⎭

EXHIBIT 13A.1 Income Statement (all-inclusive) for a Corporation

1 Continuing Operations Section 1 shows revenues, expenses, and income from continu- ing operations. This information is used to predict future operations, and most view this section as the most important. Gains and losses that are normal and frequent are reported as part of continuing operations. Gains and losses that are either unusual and/or infrequent are reported as part of continuing operations but after the nor- mal revenues and expenses. Items considered unusual and/or infrequent include (1) property taken away by a foreign government, (2) condemning of property, (3) prohibiting use of an asset from a new law, (4) losses and gains from an unusual and infrequent calamity (“act of God”), and (5) financial effects of labor strikes.

2 Discontinued Segments A business segment is a part of a company that is separated by its products/services or by geographic location. A segment has assets, liabilities, and financial results of operations that can be separated from those of other parts of the company. A gain or loss from selling or closing down a segment is separately reported. Section 2 of Exhibit 13A.1 reports both (a) income from operating the discontinued segment before its disposal and (b) the loss from disposing of the segment’s net assets. The income tax effects of each are reported separately from the income tax expense in section 1 .

3 Earnings per Share Section 3 of Exhibit 13A.1 reports earnings per share for both continuing operations and discontinued segments (when they both exist). Earnings per share is covered in Chapter 11. Changes in Accounting Principles Changes in accounting principles require retrospec- tive application to prior periods’ financial statements. Retrospective application means applying a different

Point: FASB no longer allows extraordinary items.

516 Chapter 13 Analysis of Financial Statements

accounting principle to prior periods as if that principle had always been used. Retrospective application enhances the consistency of financial information between periods, which improves the usefulness of information, especially with comparative analyses.

Small Business Owner You own an orange grove near Jacksonville, Florida. A bad frost destroys about one-half of your oranges. You are currently preparing an income statement for a bank loan. Where on the income statement do you report the loss of oranges? ■ Answer: The frost loss is likely unusual, meaning it is reported in the nonrecurring section of continuing operations. Managers would highlight this loss apart from ongoing, normal results so that the bank views it separately from normal operations.

Decision Maker

BASICS OF ANALYSIS Liquidity and efficiency: Ability to meet short-term obligations and efficiently generate revenues. Solvency: Ability to meet long-term obligations and generate future revenues. Profitability: Ability to provide financial rewards to attract and retain financing. Market prospects: Ability to generate positive market expectations. General-purpose financial statements: Include the (1) income statement, (2) balance sheet, (3) statement of stockholders’ equity (or statement of re- tained earnings), (4) statement of cash flows, and (5) notes to these statements.

HORIZONTAL ANALYSIS Comparative financial statements: Show financial amounts in side-by- side columns on a single statement. Analysis period: The financial statements under analysis. Base period: The financial statements used for comparison. The prior year is commonly used as a base period.

Apple comparative balance sheet: The prior year is the base period and current year is the analysis period.

Summary: Cheat Sheet

Dollar change formula: Dollar change = Analysis period amount − Base period amount

Percent change formula:

Percent change (%) = Analysis period amount − Base period amount

Base period amount × 100

$ millions Current Yr Prior Yr Dollar Change Percent Change

Assets Cash and cash equivalents . . . . . . . . . . . . $20,289 $20,484 $ (195) (1.0)% Short-term marketable securities . . . . . . . 53,892 46,671 7,221 15.5 Accounts receivable, net . . . . . . . . . . . . . . 17,874 15,754 2,120 13.5

Trend analysis: Computing trend percents that show patterns in data across periods.

Trend percent (%) = Analysis period amount

Base period amount × 100

Apple trend analysis: 4 years ago is the base period, and each subsequent year is the analysis period.

Base amount: Comparative balance sheets use total assets, and compara- tive income statements use net sales.

VERTICAL ANALYSIS Common-size financial statements: Show changes in the relative impor- tance of each financial statement item. All individual amounts in common- size statements are shown in common-size percents. Common-size percent formula:

Common-size percent (%) = Analysis amount

Base amount × 100

In trend percent Current Yr 1 Yr Ago 2 Yrs Ago 3 Yrs Ago 4 Yrs Ago

Net sales . . . . . . . . . . . . . . . . . 134 .1% 126 .2% 136 .7% 107 .0% 100 .0% Cost of sales . . . . . . . . . . . . . . 132 .3 123 .2 131 .4 105 .3 100 .0 Operating expenses . . . . . . . . 175 .4 158 .4 146 .3 117 .8 100 .0

Apple common-size balance sheet:

RATIO ANALYSIS AND REPORTING Ratio Formula

Liquidity and Efficiency

Current ratio = Current assets

Current liabilities

Acid-test ratio = Cash + Short-term investments + Current receivables

Current liabilities

Accounts receivable turnover = Net sales

Average accounts receivable, net

Inventory turnover = Cost of goods sold Average inventory

Days’ sales uncollected = Accounts receivable, net

Net sales × 365

Days’ sales in inventory = Ending inventory

Cost of goods sold × 365

Total asset turnover = Net sales

Average total assets Solvency

Debt ratio = Total liabilities

Total assets

Equity ratio = Total equity Total assets

Debt-to-equity ratio = Total liabilities

Total equity

Times interest earned = Income before interest expense and income tax expense

Interest expense Profitability

Profit margin ratio = Net income Net sales

Gross margin ratio = Net sales − Cost of goods sold

Net sales

Return on total assets = Net income

Average total assets

Return on common stockholders’ equity = Net income − Preferred dividends

Average common stockholdersʼ equity

Basic earnings per share = Net income − Preferred dividends

Weighted-average common shares outstanding Market Prospects

Price-earnings ratio = Market price per common share

Earnings per share

Dividend yield = Annual cash dividends per share

Market price per share

Apple common-size income statement: Common-Size Percents

$ millions Current Yr Prior Yr Current Yr Prior Yr

Net sales $229,234 $215,639 100.0% 100.0% Cost of sales 141,048 131,376 61.5 60.9 Gross margin 88,186 84,263 38.5 39.1

Common-Size Percents $ millions Current Yr Prior Yr Current Yr Prior Yr

Goodwill 5,717 5,414 1.5% 1.7% Acquired intangible assets, net 2,298 3,206 0.6 1.0 Other assets 10,162 8,757 2.7 2.7 Total assets $375,319 $321,686 100.0 100.0

Chapter 13 Analysis of Financial Statements 517

A Superscript letter A denotes assignments based on Appendix 13A.

Icon denotes assignments that involve decision making.

1. Explain the difference between financial reporting and financial statements.

2. What is the difference between comparative financial state- ments and common-size comparative statements?

3. Which items are usually assigned a 100% value on (a) a common-size balance sheet and (b) a common-size income statement?

4. What three factors would influence your evaluation as to whether a company’s current ratio is good or bad?

5. Suggest several reasons why a 2:1 current ratio might not be adequate for a particular company.

6. Why is working capital given special attention in the process of analyzing balance sheets?

7. What does the number of days’ sales uncollected indicate?

8. What does a relatively high accounts receivable turn- over indicate about a company’s short-term liquidity?

9. Why is a company’s capital structure, as measured by debt and equity ratios, important to financial statement analysts?

10. How does inventory turnover provide information about a company’s short-term liquidity?

Discussion Questions

Business segment (515) Common-size financial statement (502) Comparative financial statement (498) Efficiency (497) Equity ratio (508) Financial reporting (498)

Financial statement analysis (497) General-purpose financial

statements (498) Horizontal analysis (498) Liquidity (497) Market prospects (497)

Profitability (497) Ratio analysis (498) Solvency (497) Vertical analysis (498) Working capital (506)

Key Terms

Multiple Choice Quiz

1. A company’s sales in the prior year were $300,000 and in the current year were $351,000. Using the prior year as the base year, the sales trend percent for the current year is a. 17%. c. 100%. e. 48%. b. 85%. d. 117%.

Use the following information for questions 2 through 5.

2. What is Ella Company’s current ratio? a. 0.69 d. 6.69 b. 1.31 e. 2.39 c. 3.88

3. What is Ella Company’s acid-test ratio? a. 2.39 d. 6.69 b. 0.69 e. 3.88 c. 1.31

4. What is Ella Company’s debt ratio? a. 25.78% d. 137.78% b. 100.00% e. 34.74% c. 74.22%

5. What is Ella Company’s equity ratio? a. 25.78% d. 74.22% b. 100.00% e. 137.78% c. 34.74%

ELLA COMPANY Balance Sheet December 31

Assets Cash . . . . . . . . . . . . . . . . . $ 86,000

Accounts receivable . . . . . 76,000

Merchandise inventory . . 122,000

Prepaid insurance . . . . . . 12,000

Long-term investments . . 98,000

Plant assets, net . . . . . . . . 436,000

Total assets . . . . . . . . . . . . $830,000

Liabilities Current liabilities . . . . . . . . $124,000

Long-term liabilities . . . . . . 90,000

Equity Common stock . . . . . . . . . . 300,000

Retained earnings . . . . . . . 316,000

Total liabilities and equity . . $830,000

ANSWERS TO MULTIPLE CHOICE QUIZ

1. d; ($351,000∕$300,000) × 100 = 117% 2. e; ($86,000 + $76,000 + $122,000 + $12,000)∕$124,000 = 2.39 3. c; ($86,000 + $76,000)∕$124,000 = 1.31

4. a; ($124,000 + $90,000)∕$830,000 = 25.78% 5. d; ($300,000 + $316,000)∕$830,000 = 74.22%

518 Chapter 13 Analysis of Financial Statements

11. What ratios would you compute to evaluate manage- ment performance?

12. Why would a company’s return on total assets be dif- ferent from its return on common stockholders’ equity?

13. Where on the income statement does a company report an unusual gain not expected to occur more often than once every two years or so?

14. Refer to Apple’s financial statements in Appendix A. Compute its profit margin for the years ended September 30, 2017, and September 24, 2016.

15. Refer to Google’s financial statements in Appendix A to compute its equity ratio as of December 31, 2017, and December 31, 2016.

16. Refer to Samsung’s financial statements in Appendix A. Compute its debt ratio as of December 31, 2017, and December 31, 2016.

17. Use Samsung’s financial statements in Appendix A to compute its return on total assets for fiscal year ended December 31, 2017.APPLE

GOOGLE

Samsung

Samsung

QUICK STUDY

QS 13-1 Financial reporting

C1

Identify which of the following items are not included as part of general-purpose financial statements but are part of financial reporting.

a. Income statement f. Statement of cash flows b. Balance sheet g. Stock price information and analysis c. Shareholders’ meetings h. Statement of shareholders’ equity d. Financial statement notes i. Management discussion and analysis of financial e. Company news releases performance

QS 13-2 Standard of comparison

C2

Identify which standard of comparison, (a) intracompany, (b) competitor, (c) industry, or (d) guidelines, best describes each of the following examples.

1. Compare Ford’s return on assets to GM’s return on assets. 2. Compare a company’s acid-test ratio to the 1:1 rule of thumb. 3. Compare Netflix’s current-year sales to its prior-year sales. 4. Compare McDonald’s profit margin to the fast-food industry profit margin.

QS 13-3 Horizontal analysis

P1

Compute the annual dollar changes and percent changes for each of the following accounts.

Current Yr Prior Yr

Short-term investments . . . . . . . . . . . . $374,634 $234,000

Accounts receivable . . . . . . . . . . . . . . . 97,364 101,000

Notes payable . . . . . . . . . . . . . . . . . . . . 0 88,000

QS 13-5 Common-size analysis P2

Refer to the information in QS 13-4. Determine the prior-year and current-year common-size percents for cost of goods sold using net sales as the base.

QS 13-6 Computing current ratio and acid-test ratio P3

Pritchett Co. reported the following year-end data: cash of $15,000; short-term investments of $5,000; ac- counts receivable (current) of $8,000; inventory of $20,000; prepaid (current) assets of $6,000; and total current liabilities of $20,000. Compute the (a) current ratio and (b) acid-test ratio. Round to one decimal.

QS 13-7 Computing accounts receivable turnover and days’ sales uncollected P3

Mifflin Co. reported the following for the current year: net sales of $60,000; cost of goods sold of $38,000; beginning balance in accounts receivable of $14,000; and ending balance in accounts receivable of $6,000. Compute (a) accounts receivable turnover and (b) days’ sales uncollected. Round to one decimal. Hint: Recall that accounts receivable turnover uses average accounts receivable and days’ sales uncollected uses the ending balance in accounts receivable.

QS 13-4 Trend percents

P1

Use the following information to determine the prior-year and current-year trend percents for net sales using the prior year as the base year.

$ thousands Current Yr Prior Yr

Net sales . . . . . . . . . . . . . . . . . . . . . . . . $801,810 $453,000

Cost of goods sold . . . . . . . . . . . . . . . . 392,887 134,088

Chapter 13 Analysis of Financial Statements 519

SCC Co. reported the following for the current year: net sales of $48,000; cost of goods sold of $40,000; beginning balance in inventory of $2,000; and ending balance in inventory of $8,000. Compute (a) inven- tory turnover and (b) days’ sales in inventory. Hint: Recall that inventory turnover uses average inventory and days’ sales in inventory uses the ending balance in inventory.

QS 13-8 Computing inventory turnover and days’ sales in inventory P3

Dundee Co. reported the following for the current year: net sales of $80,000; cost of goods sold of $60,000; beginning balance of total assets of $115,000; and ending balance of total assets of $85,000. Compute total asset turnover. Round to one decimal.

QS 13-9 Computing total asset turnover P3

Paddy’s Pub reported the following year-end data: income before interest expense and income tax expense of $30,000; cost of goods sold of $17,000; interest expense of $1,500; total assets of $70,000; total liabil- ities of $20,000; and total equity of $50,000. Compute the (a) debt-to-equity ratio and (b) times interest earned. Round to one decimal.

QS 13-10 Computing debt-to-equity ratio and times interest earned P3

Edison Co. reported the following for the current year: net sales of $80,000; cost of goods sold of $56,000; net income of $16,000; beginning balance of total assets of $60,000; and ending balance of total assets of $68,000. Compute (a) profit margin and (b) return on total assets.

QS 13-11 Computing profit margin and return on total assets P3

Franklin Co. reported the following year-end data: net income of $220,000; annual cash dividends per share of $3; market price per (common) share of $150; and earnings per share of $10. Compute the (a) price-earnings ratio and (b) dividend yield.

QS 13-12 Computing price-earnings ratio and dividend yield P3

For each ratio listed, identify whether the change in ratio value from the prior year to the current year is usually regarded as favorable or unfavorable.

QS 13-13 Ratio interpretation

P3 Ratio Current Yr Prior Yr Ratio Current Yr Prior Yr

1 . Profit margin . . . . . . 9% 8% 5 . Accounts receivable turnover . . . . . 5 .5 6 .7

2 . Debt ratio . . . . . . . . 47% 42% 6 . Basic earnings per share . . . . . . . . . $1 .25 $1 .10

3 . Gross margin . . . . . 34% 46% 7 . Inventory turnover . . . . . . . . . . . . . . 3 .6 3 .4

4 . Acid-test ratio . . . . . 1 .00 1 .15 8 . Dividend yield . . . . . . . . . . . . . . . . . 2 .0% 1 .2%

QS 13-14 Analyzing short-term financial condition

A1

Morgan Company and Parker Company are similar firms operating in the same industry. Write a half- page report comparing Morgan and Parker using the available information. Your discussion should in- clude their ability to meet current obligations and to use current assets efficiently.

Team Project: Assume that the two companies apply for a one-year loan from the team. Identify additional information the compa- nies must provide before the team can make a loan decision.

Current ratio Acid-test ratio

Accounts receivable turnover Merchandise inventory turnover

Working capital

Morgan

1.7 1.0

30.5 24.2

$70,000

Current Yr 1 Yr Ago 1.6 1.1

25.2 21.9

$58,000

2 Yrs Ago 2.1 1.2

29.2 17.1

$52,000

Parker

3.2 2.8

16.4 14.5

$131,000

Current Yr 1 Yr Ago 2 Yrs Ago 2.7 2.5

15.2 13.0

$103,000

1.9 1.6

16.0 12.6

$78,000

Which of the following gains or losses would Organic Foods account for as unusual and/or infrequent? a. A hurricane destroys rainwater tanks that result in a loss for Organic Foods. b. The used vehicle market is weak and Organic Foods is forced to sell its used delivery truck at a loss. c. Organic Foods owns an organic farm in Venezuela that is seized by the government. The company

records a loss.

QS 13-15A Identifying unusual and/or infrequent gains or losses

A2

520 Chapter 13 Analysis of Financial Statements

EXERCISES

Exercise 13-1 Building blocks of analysis

C1

Match the ratio to the building block of financial statement analysis to which it best relates. A. Liquidity and efficiency B. Solvency C. Profitability D. Market prospects

1. Equity ratio 6. Accounts receivable turnover 2. Return on total assets 7. Debt-to-equity ratio 3. Dividend yield 8. Times interest earned 4. Book value per common share 9. Gross margin ratio 5. Days’ sales in inventory 10. Acid-test ratio

Exercise 13-3 Computing and analyzing trend percents

P1

Compute trend percents for the following accounts using 2015 as the base year. For each of the three accounts, state whether the situation as revealed by the trend percents appears to be favorable or unfavorable.

2019 2018 2017 2016 2015

Sales . . . . . . . . . . . . . . . . . . . . . . . . $282,880 $270,800 $252,600 $234,560 $150,000

Cost of goods sold . . . . . . . . . . . . . 128,200 122,080 115,280 106,440 67,000

Accounts receivable . . . . . . . . . . . . 18,100 17,300 16,400 15,200 9,000

Exercise 13-5 Determining income effects from common-size and trend percents

P1 P2

Common-size and trend percents for Roxi Company’s sales, cost of goods sold, and expenses follow. Determine whether net income increased, decreased, or remained unchanged in this three-year period.

Common-Size Percents Trend Percents

Current Yr 1 Yr Ago 2 Yrs Ago Current Yr 1 Yr Ago 2 Yrs Ago

Sales . . . . . . . . . . . . . . . . . . . 100 .0% 100 .0% 100 .0% 105 .4% 104 .2% 100 .0%

Cost of goods sold . . . . . . . . 63 .4 61 .9 59 .1 113 .1 109 .1 100 .0

Total expenses . . . . . . . . . . . 15 .3 14 .8 15 .1 106 .8 102 .1 100 .0

Exercise 13-2 Identifying financial ratios

C2

Identify which of the following six metrics a through f best completes questions 1 through 3 below. a. Days’ sales uncollected d. Return on total assets b. Accounts receivable turnover e. Total asset turnover c. Working capital f. Profit margin 1. Which two ratios are key components in measuring a company’s operating efficiency?

Which ratio summarizes these two components? 2. What measure reflects the difference between current assets and current liabilities? 3. Which two short-term liquidity ratios measure how frequently a company collects its accounts?

Exercise 13-4 Computing and interpreting common-size percents

P2

Compute common-size percents for the following comparative income statements (round percents to one decimal). Using the common-size percents, which item is most responsible for the decline in net income?

GOMEZ CORPORATION Comparative Income Statements

For Years Ended December 31 Current Yr Prior Yr

Sales . . . . . . . . . . . . . . . . . . . . . . . $740,000 $625,000

Cost of goods sold . . . . . . . . . . . . 560,300 290,800

Gross profit . . . . . . . . . . . . . . . . . . 179,700 334,200

Operating expenses . . . . . . . . . . . 128,200 218,500

Net income . . . . . . . . . . . . . . . . . . $ 51,500 $115,700

Simon Company’s year-end balance sheets follow. (1) Express the balance sheets in common-size per- cents. Round percents to one decimal. (2) Assuming annual sales have not changed in the last three years, is the change in accounts receivable as a percentage of total assets favorable or unfavorable? (3) Is the change in merchandise inventory as a percentage of total assets favorable or unfavorable?

Exercise 13-6 Common-size percents

P2

Chapter 13 Analysis of Financial Statements 521

At December 31 Current Yr 1 Yr Ago 2 Yrs Ago

Assets Cash . . . . . . . . . . . . . . . . . . . . . . . . . . . . . . . . . . . . . . . . $ 31,800 $ 35,625 $ 37,800

Accounts receivable, net . . . . . . . . . . . . . . . . . . . . . . . 89,500 62,500 50,200

Merchandise inventory . . . . . . . . . . . . . . . . . . . . . . . . . 112,500 82,500 54,000

Prepaid expenses . . . . . . . . . . . . . . . . . . . . . . . . . . . . . 10,700 9,375 5,000

Plant assets, net . . . . . . . . . . . . . . . . . . . . . . . . . . . . . . 278,500 255,000 230,500

Total assets . . . . . . . . . . . . . . . . . . . . . . . . . . . . . . . . . . $523,000 $445,000 $377,500

Liabilities and Equity Accounts payable . . . . . . . . . . . . . . . . . . . . . . . . . . . . . $129,900 $ 75,250 $ 51,250

Long-term notes payable secured by mortgages on plant assets . . . . . . . . . . . . . . . . . . . 98,500 101,500 83,500

Common stock, $10 par value . . . . . . . . . . . . . . . . . . . 163,500 163,500 163,500

Retained earnings . . . . . . . . . . . . . . . . . . . . . . . . . . . . . 131,100 104,750 79,250

Total liabilities and equity . . . . . . . . . . . . . . . . . . . . . . . $523,000 $445,000 $377,500

Refer to Simon Company’s balance sheets in Exercise 13-6. (1) Compute the current ratio for each of the three years. Did the current ratio improve or worsen over the three-year period? (2) Compute the acid-test ratio for each of the three years. Did the acid-test ratio improve or worsen over the three-year period? Round ratios to two decimals.

Exercise 13-7 Analyzing liquidity

P3

Refer to the Simon Company information in Exercises 13-6 and 13-8. For both the current year and one year ago, compute the following ratios: (1) debt ratio and equity ratio—percent rounded to one decimal, (2) debt-to-equity ratio—rounded to two decimals; based on debt-to-equity ratio, does the company have more or less debt in the current year versus one year ago? and (3) times interest earned—rounded to one decimal. Based on times interest earned, is the company more or less risky for creditors in the current year versus one year ago?

Exercise 13-9 Analyzing risk and capital structure

P3

Refer to Simon Company’s financial information in Exercises 13-6 and 13-8. For both the current year and one year ago, compute the following ratios: (1) profit margin ratio—percent rounded to one decimal; did profit margin improve or worsen in the current year versus one year ago? (2) total asset turnover—rounded to one decimal, and (3) return on total assets—percent rounded to one decimal. Based on return on total assets, did Simon’s operating efficiency improve or worsen in the current year versus one year ago?

Exercise 13-10 Analyzing efficiency and profitability

P3

Refer to Simon Company’s financial information in Exercises 13-6 and 13-8. Additional information about the company follows. For both the current year and one year ago, compute the following ratios: (1) return on common stockholders’ equity—percent rounded to one decimal, (2) dividend yield—percent rounded to one decimal, and (3) price-earnings ratio on December 31—rounded to one decimal. Assuming Simon’s com- petitor has a price-earnings ratio of 10, which company has higher market expectations for future growth?

Exercise 13-11 Analyzing profitability

P3

Common stock market price, December 31, current year . . . . . $30 .00 Annual cash dividends per share in current year . . . . $0 .29

Common stock market price, December 31, 1 year ago . . . . . . 28 .00 Annual cash dividends per share 1 year ago . . . . . . . . 0 .24

Refer to the Simon Company information in Exercise 13-6. The company’s income statements for the cur- rent year and one year ago follow. Assume that all sales are on credit and then compute (1) days’ sales uncollected, (2) accounts receivable turnover, (3) inventory turnover, and (4) days’ sales in inventory. For each ratio, determine if it improved or worsened in the current year. Round to one decimal.

Exercise 13-8 Analyzing and interpreting liquidity

P3

For Year Ended December 31 Current Yr 1 Yr Ago

Sales . . . . . . . . . . . . . . . . . . . . . . . . . . . . . $673,500 $532,000

Cost of goods sold . . . . . . . . . . . . . . . . . . $411,225 $345,500

Other operating expenses . . . . . . . . . . . . 209,550 134,980

Interest expense . . . . . . . . . . . . . . . . . . . . 12,100 13,300

Income tax expense . . . . . . . . . . . . . . . . . 9,525 8,845

Total costs and expenses . . . . . . . . . . . . . 642,400 502,625

Net income . . . . . . . . . . . . . . . . . . . . . . . . $ 31,100 $ 29,375

Earnings per share . . . . . . . . . . . . . . . . . . $ 1 .90 $ 1 .80

522 Chapter 13 Analysis of Financial Statements

Following are data for BioBeans and GreenKale, which sell organic produce and are of similar size. 1. Compute the profit margin and the return on total assets for both companies. 2. Based on analysis of these two measures, which company is the preferred investment?

Exercise 13-13 Analyzing efficiency and profitability

P3 BioBeans GreenKale

Average total assets . . . . . . . . . . . $187,500 $150,000

Net sales . . . . . . . . . . . . . . . . . . . . 75,000 60,000

Net income . . . . . . . . . . . . . . . . . . 15,000 9,000

Following is an incomplete current-year income statement.Exercise 13-14 Reconstructing an income statement with ratios

P3 Income Statement

Net sales . . . . . . . . . . . . . . . . . . . . . . . . . . . . . . . . . . . . . . . . $ (a) Cost of goods sold . . . . . . . . . . . . . . . . . . . . . . . . . . . . . . . . (b) Selling, general, and administrative expenses . . . . . . . . . . 7,000 Income tax expense . . . . . . . . . . . . . . . . . . . . . . . . . . . . . . . 2,000

Net income . . . . . . . . . . . . . . . . . . . . . . . . . . . . . . . . . . . . . . (c)

Determine amounts a, b, and c. Additional information follows: ∙ Return on total assets is 16% (average total assets is $68,750). ∙ Inventory turnover is 5 (average inventory is $6,000). ∙ Accounts receivable turnover is 8 (average accounts receivable is $6,250).

Refer to the information in Exercise 13-15. 1. Which company has the better (a) profit margin, (b) asset turnover, and (c) return on assets? 2. Which company has the better rate of growth in sales? 3. Did Roak successfully use financial leverage in the current year? Did Clay?

Exercise 13-16 Interpreting financial ratios

A1 P3

In the current year, Randa Merchandising, Inc., sold its interest in a chain of wholesale outlets, taking the company completely out of the wholesaling business. The company still operates its retail outlets. A list- ing of the major sections of an income statement follows. A. Net sales less operating expense section B. Other unusual and/or infrequent gains (losses)

Exercise 13-17A Income statement categories

A2

Exercise 13-12 Computing current ratio and profit margin

P3

Nintendo Company, Ltd., recently reported the following financial information (amounts in millions). Compute Nintendo’s current ratio and profit margin. Round to two decimals.

Current assets . . . . . . . . . . . . . . . . . . . . . . . . . . . . . $ 9,036 Net sales . . . . . . . . . . . . . . . . . . . . . . . . . . . . . . . $4,464

Total assets . . . . . . . . . . . . . . . . . . . . . . . . . . . . . . . 11,477 Net income . . . . . . . . . . . . . . . . . . . . . . . . . . . . . 146

Current liabilities . . . . . . . . . . . . . . . . . . . . . . . . . . 871

Exercise 13-15 Analyzing efficiency and financial leverage

A1

Roak Company and Clay Company are similar firms that operate in the same industry. Clay began opera- tions two years ago and Roak started five years ago. In the current year, both companies pay 6% interest on their debt to creditors. The following additional information is available.

Roak Company Clay Company

Current Yr 1 Yr Ago 2 Yrs Ago Current Yr 1 Yr Ago 2 Yrs Ago

Total asset turnover . . . . . . . 3 .1 2 .8 3 .0 1 .7 1 .5 1 .1

Return on total assets . . . . . 7 .4% 7 .0% 6 .9% 4 .8% 4 .5% 3 .2%

Profit margin ratio . . . . . . . . 2 .4% 2 .5% 2 .3% 2 .8% 3 .0% 2 .9%

Sales . . . . . . . . . . . . . . . . . . . $410,000 $380,000 $396,000 $210,000 $170,000 $110,000

Write a half-page report comparing Roak and Clay using the available information. Your analysis should include their ability to use assets efficiently to produce profits. Comment on their success in employing financial leverage in the current year.

Chapter 13 Analysis of Financial Statements 523

Use the financial data for Randa Merchandising, Inc., in Exercise 13-17A to prepare its December 31 year- end income statement. Ignore the earnings per share section.

Exercise 13-18A Income statement presentation A2

HAROUN COMPANY Comparative Income Statements

For Years Ended December 31

$ thousands 2019 2018 2017 2016 2015 2014 2013

Sales . . . . . . . . . . . . . . . $1,694 $1,496 $1,370 $1,264 $1,186 $1,110 $928

Cost of goods sold . . . . 1,246 1,032 902 802 752 710 586

Gross profit . . . . . . . . . . 448 464 468 462 434 400 342

Operating expenses . . . 330 256 234 170 146 144 118

Net income . . . . . . . . . . $ 118 $ 208 $ 234 $ 292 $ 288 $ 256 $224

HAROUN COMPANY Comparative Year-End Balance Sheets

At December 31, $ thousands 2019 2018 2017 2016 2015 2014 2013

Assets Cash . . . . . . . . . . . . . . . . . . . . $ 58 $ 78 $ 82 $ 84 $ 88 $ 86 $ 89

Accounts receivable, net . . . 490 514 466 360 318 302 216

Merchandise inventory . . . . . 1,838 1,364 1,204 1,032 936 810 615

Other current assets . . . . . . . 36 32 14 34 28 28 9

Long-term investments . . . . . 0 0 0 146 146 146 146

Plant assets, net . . . . . . . . . . 2,020 2,014 1,752 944 978 860 725

Total assets . . . . . . . . . . . . . . $4,442 $4,002 $3,518 $2,600 $2,494 $2,232 $1,800

Liabilities and Equity Current liabilities . . . . . . . . . . $1,220 $1,042 $ 718 $ 614 $ 546 $ 522 $ 282

Long-term liabilities . . . . . . . 1,294 1,140 1,112 570 580 620 400

Common stock . . . . . . . . . . . 1,000 1,000 1,000 850 850 650 650

Other paid-in capital . . . . . . . 250 250 250 170 170 150 150

Retained earnings . . . . . . . . . 678 570 438 396 348 290 318

Total liabilities and equity . . . $4,442 $4,002 $3,518 $2,600 $2,494 $2,232 $1,800

PROBLEM SET A

Problem 13-1A Calculating and analyzing trend percents

P1

Selected comparative financial statements of Haroun Company follow.

C. Taxes reported on income (loss) from continuing operations D. Income (loss) from operating a discontinued segment, or gain (loss) from its disposal Indicate where each of the following income-related items for this company appears on its current-year income statement by writing the letter of the appropriate section in the blank beside each item.

Section Item Debit Credit

_______ 1 . Net sales . . . . . . . . . . . . . . . . . . . . . . . . . . . . . . . . . . . . . . . . . . . . . $2,900,000

_______ 2 . Gain on state’s condemnation of company property . . . . . . . . . . 230,000

_______ 3 . Cost of goods sold . . . . . . . . . . . . . . . . . . . . . . . . . . . . . . . . . . . . . $1,480,000

_______ 4 . Income tax expense . . . . . . . . . . . . . . . . . . . . . . . . . . . . . . . . . . . . 217,000

_______ 5 . Depreciation expense . . . . . . . . . . . . . . . . . . . . . . . . . . . . . . . . . . 232,000

_______ 6 . Gain on sale of wholesale business segment, net of tax . . . . . . . 775,000

_______ 7 . Loss from operating wholesale business segment, net of tax . . . 444,000

_______ 8 . Loss of assets from meteor strike . . . . . . . . . . . . . . . . . . . . . . . . . 640,000

524 Chapter 13 Analysis of Financial Statements

Required

1. Compute trend percents for all components of both statements using 2013 as the base year. Round percents to one decimal.

Analysis Component

2. Refer to the results from part 1. (a) Did sales grow steadily over this period? (b) Did net income as a percent of sales grow over the past four years? (c) Did inventory increase over this period?

Check (1) 2019, Total assets trend, 246.8%

Problem 13-3A Transactions, working capital, and liquidity ratios

P3

Plum Corporation began the month of May with $700,000 of current assets, a current ratio of 2.50:1, and an acid-test ratio of 1.10:1. During the month, it completed the following transactions (the company uses a perpetual inventory system).

May 2 Purchased $50,000 of merchandise inventory on credit. 8 Sold merchandise inventory that cost $55,000 for $110,000 cash. 10 Collected $20,000 cash on an account receivable. 15 Paid $22,000 cash to settle an account payable. 17 Wrote off a $5,000 bad debt against the Allowance for Doubtful Accounts account. 22 Declared a $1 per share cash dividend on its 50,000 shares of outstanding common stock. 26 Paid the dividend declared on May 22. 27 Borrowed $100,000 cash by giving the bank a 30-day, 10% note. 28 Borrowed $80,000 cash by signing a long-term secured note. 29 Used the $180,000 cash proceeds from the notes to buy new machinery.

Check May 22: Current ratio, 2.19; Acid-test ratio, 1.11

May 29: Current ratio, 1.80; Working capital, $325,000

Required

1. Compute each year’s current ratio. Round ratios to one decimal. 2. Express the income statement data in common-size percents. Round percents to two decimals. 3. Express the balance sheet data in trend percents with 2017 as base year. Round percents to two decimals.

Analysis Component

4. Refer to the results from parts 1, 2, and 3. (a) Did cost of goods sold make up a greater portion of sales for the most recent year? (b) Did income as a percent of sales improve in the most recent year? (c) Did plant assets grow over this period?

Check (3) 2019, Total assets trend, 131.71%

KORBIN COMPANY Comparative Balance Sheets

At December 31 2019 2018 2017

Assets Current assets . . . . . . . . . . . . . $ 52,390 $ 37,924 $ 51,748

Long-term investments . . . . . . 0 500 3,950

Plant assets, net . . . . . . . . . . . 100,000 96,000 60,000

Total assets . . . . . . . . . . . . . . . $152,390 $134,424 $115,698

Liabilities and Equity Current liabilities . . . . . . . . . . . $ 22,800 $ 19,960 $ 20,300

Common stock . . . . . . . . . . . . 72,000 72,000 60,000

Other paid-in capital . . . . . . . . 9,000 9,000 6,000

Retained earnings . . . . . . . . . . 48,590 33,464 29,398

Total liabilities and equity . . . . $152,390 $134,424 $115,698

KORBIN COMPANY Comparative Income Statements

For Years Ended December 31 2019 2018 2017

Sales . . . . . . . . . . . . . . . . . . . . . $555,000 $340,000 $278,000

Cost of goods sold . . . . . . . . . . 283,500 212,500 153,900

Gross profit . . . . . . . . . . . . . . . . 271,500 127,500 124,100

Selling expenses . . . . . . . . . . . . 102,900 46,920 50,800

Administrative expenses . . . . . 50,668 29,920 22,800

Total expenses . . . . . . . . . . . . . 153,568 76,840 73,600

Income before taxes . . . . . . . . . 117,932 50,660 50,500

Income tax expense . . . . . . . . . 40,800 10,370 15,670

Net income . . . . . . . . . . . . . . . . $ 77,132 $ 40,290 $ 34,830

Selected comparative financial statements of Korbin Company follow.Problem 13-2A Ratios, common-size statements, and trend percents

P1 P2 P3

Chapter 13 Analysis of Financial Statements 525

Required

Prepare a table, similar to the following, showing Plum’s (1) current ratio, (2) acid-test ratio, and (3) working capital after each transaction. Round ratios to two decimals.

1

2

3

A B

Beginning

Current Assets

$700,000

Transaction

C

D

E

2.50

F G

1.10 —

Current Ratio

Acid-Test Ratio

Working Capital

Current Liabilities

Quick Assets

Selected current year-end financial statements of Cabot Corporation follow. All sales were on credit; selected balance sheet amounts at December 31 of the prior year were inventory, $48,900; total assets, $189,400; common stock, $90,000; and retained earnings, $33,748.

Problem 13-4A Calculating financial statement ratios

P3

Required

Compute the following: (1) current ratio, (2) acid-test ratio, (3) days’ sales uncollected, (4) inventory turnover, (5) days’ sales in inventory, (6) debt-to-equity ratio, (7) times interest earned, (8) profit margin ratio, (9) total asset turnover, (10) return on total assets, and (11) return on common stockholders’ equity. Round to one decimal place; for part 6, round to two decimals.

Check Acid-test ratio, 2.2 to 1; Inventory turnover, 7.3

CABOT CORPORATION Income Statement

For Current Year Ended December 31

Sales . . . . . . . . . . . . . . . . . $448,600

Cost of goods sold . . . . . . 297,250

Gross profit . . . . . . . . . . . . 151,350

Operating expenses . . . . . 98,600

Interest expense . . . . . . . . 4,100

Income before taxes . . . . . 48,650

Income tax expense . . . . . 19,598

Net income . . . . . . . . . . . . $ 29,052

CABOT CORPORATION Balance Sheet

December 31 of Current Year

Assets Liabilities and Equity Cash . . . . . . . . . . . . . . . . . . $ 10,000 Accounts payable . . . . . . . . . . . . . . . . . $ 17,500

Short-term investments . . 8,400 Accrued wages payable . . . . . . . . . . . . 3,200

Accounts receivable, net . 33,700 Income taxes payable . . . . . . . . . . . . . 3,300

Merchandise inventory . . . 32,150 Long-term note payable, secured

Prepaid expenses . . . . . . . 2,650 by mortgage on plant assets . . . . . . 63,400

Plant assets, net . . . . . . . . 153,300 Common stock . . . . . . . . . . . . . . . . . . . 90,000

Retained earnings . . . . . . . . . . . . . . . . 62,800

Total assets . . . . . . . . . . . . $240,200 Total liabilities and equity . . . . . . . . . . $240,200

Barco Kyan Barco Kyan Company Company Company Company

Data from the current year-end balance sheets Data from the current year’s income statement Assets Sales . . . . . . . . . . . . . . . . . . . . . . . . . . . . . . $770,000 $880,200 Cash . . . . . . . . . . . . . . . . . . . . . . . . . . . . . . . $ 19,500 $ 34,000 Cost of goods sold . . . . . . . . . . . . . . . . . . . 585,100 632,500

Accounts receivable, net . . . . . . . . . . . . . . 46,500 64,600 Interest expense . . . . . . . . . . . . . . . . . . . . 7,900 13,000

Merchandise inventory . . . . . . . . . . . . . . . . 84,440 132,500 Income tax expense . . . . . . . . . . . . . . . . . 14,800 24,300

Prepaid expenses . . . . . . . . . . . . . . . . . . . . 5,000 6,950 Net income . . . . . . . . . . . . . . . . . . . . . . . . 162,200 210,400

Plant assets, net . . . . . . . . . . . . . . . . . . . . . 290,000 304,400 Basic earnings per share . . . . . . . . . . . . . . 4 .51 5 .11

Total assets . . . . . . . . . . . . . . . . . . . . . . . . . $445,440 $542,450 Cash dividends per share . . . . . . . . . . . . . 3 .81 3 .93

Liabilities and Equity Beginning-of-year balance sheet data Current liabilities . . . . . . . . . . . . . . . . . . . . . $ 61,340 $ 93,300 Accounts receivable, net . . . . . . . . . . . . . . $ 29,800 $ 54,200

Long-term notes payable . . . . . . . . . . . . . . 80,800 101,000 Merchandise inventory . . . . . . . . . . . . . . . 55,600 107,400

Common stock, $5 par value . . . . . . . . . . . 180,000 206,000 Total assets . . . . . . . . . . . . . . . . . . . . . . . . 398,000 382,500

Retained earnings . . . . . . . . . . . . . . . . . . . . 123,300 142,150 Common stock, $5 par value . . . . . . . . . . 180,000 206,000

Total liabilities and equity . . . . . . . . . . . . . . $445,440 $542,450 Retained earnings . . . . . . . . . . . . . . . . . . . 98,300 93,600

Summary information from the financial statements of two companies competing in the same industry follows.

Problem 13-5A Comparative ratio analysis

P3

526 Chapter 13 Analysis of Financial Statements